Sie sind auf Seite 1von 636

ABM Abdullah

Short Cases

In
Clinical Medicine
Fifth Edition

ABM Abdullah MRCP (UK), FRCP [Edin]


Dean, Faculty of Medicine
Professor of Medicine
Bangabandhu Sheikh Mujib Medical University
Dhaka, Bangladesh

ELSEVIER
ELSEVIER
A division of
Reed Elsevier India Private Limited
Contents

Foreword v
Preface to the Fifth Edition vii
Preface to the First Edition ix

~i§4h.)", Short Cases 1

Chapter 1 General Examination _ 3


Introduction-i-S] General Examination-41 Clubbing=-S] Koilonychia-Ill Nail Changes in
Different Diseases-e l Z] Cervical Lymphadenopathy= to] Generalised Lymphadenopathy-I8
I Pigmentation-20 I Erythema Ab Igne-22I Gynaecomastia-23I Erythema Nodosum-26I
Leprosy-28 I Superior Vena Caval Obstruction-30 I Hirsutism-32 I Leg Ulcer-3S I
Pyoderma Gangrenosum-37I Diabetic Foot-39I Diabetic Amyotrophy-4lI Lipodystrophy
of Thigh-42 I Necrobiosis Lipoidica Diabeticorum-43 I Unilateral or Bilateral Leg
Swelling-43 I Deep Venous Thrombosis (DVf)-46 I Cellulitis and Erysipelas-49 I Periph-
eral Vascular Disease-50 I Laurence-Moon-Bardet-Biedl Syndrome-52 I Generalized
Oedema-53 I Tongue-54 I Hairy Leukoplakia-58 I Dupuytren Contracture-59 I Paget
Disease-60 I Examination of Hands=-Sl ] Lupus Pernio-6S

Chapter 2 Cardiovascular System _ 68


Introduction-68I Examination Routine-68I Important Discussions in Relation to CVS-70
I Mitral Stenosis-7S I Mitral Regurgitation-79 I Mitral Valve Prolapse (Barlow Syndrome
or Floppy Mitral Valve)-80 I Mitral Stenosis with Mitral Regurgitation (Mixed Mitral Valve
Disease)-BI I Aortic Stenosis-B2 I Aortic Regurgitation-B4 I Aortic Stenosis with Aortic
Regurgitation (Mixed Aortic Valve Disease)-B71 Tricuspid Regurgitation (TR)-BBI Pulmo-
nary Stenosis (PS)-B91 Prosthetic HeartValves=Dn ICongenital Heart Disease-92 ITetralogy
of Fallot (TOF)-92I Ventricular Septal Defect (VSD)-95 I Atrial Septal Defect (ASD)-9BI
Patent Ductus Arteriosus (PDA)-99 I Examination of Pulse-lOll Atrial Fibrillation-1021
Slow Pulse Rate lComplete Heart Block (CHB) ]-1041 Takayasu Disease-IDS I Coarctation of
Aorta-1061 Marfan Syndrome-lOBI Dextrocardia= l ll ] Pericardial Effusion-112I Chronic
Constrictive Pericarditis-e l l S] Acute Pericarditis-lIS IRheumatic Fever-1161 Hypertrophic
Cardiomyopathy-119

Chapter 3 Respiratory System =~~ __ --- _ 122


Introduction-122I Examination Routine-122I Examination of the Chest-123I Discussions
on Routine Examination-124 I Crepitation or Pleural Rub-130 I Pleural Effusion-130 I
Pneumothorax-l 36 I Bronchiectasis-140 I Bronchiectasis with Cystic Fibrosis-l 43 I DPLD
(Previously Called FibrosingAlveolitis or ILD)-1451 Consolidation-14 71 LungAbscess-153
I Collapse of Lung-ISS I Fibrosis of Lung-l56 I Chronic Obstructive Pulmonary Disease
(COPD)-IS71 Chronic Bronchitis-l 61 I Emphysema-c-Le.S ] Brief Discussion About Lung
Function Test (Spirometry)-165I Mass Lesion in Lung (Bronchial Carcinoma)-166
iiiiiia CONTENTS
Chapter 4 Abdomen _ 172
Introduction-l72 I Examination Routine (Abdomen)-I72 I Some Important Findings
on Inspection-175 I Hepatosplenomegaly-178 I Hepatosplenomegaly (MaJaria)-179 I
Hepatosplenomegaly (Kala-Azar)-1821 Post-Kala-Azar Dermal Leishmaniasis (PKDL)-18S
I Splenomegaly (Not Hepatomegalyj-c-Ix? I Splenomegaly (Tropical Splenomegaly
Syndrome)-189 I Hepatomegaly-190 I Hepatomegaly (Hepatoma)-l92 I Ascites
with Splenomegaly (Tuberculous Peritonitis)-194 I Hepatomegaly (Tender Liver, Viral
Hepatitis)-195I Brief Discussion on Weil Disease (Leptospirosis)-197 IHepatomegaly (Liver
Abscess)-198I Hepatomegaly (Hydatid Cyst)-200 I Chronic Liver Disease or Cirrhosis of
Liver-2021 Portosystemic Encephalopathy (PSE or Hepatic Precoma)-2091 Hepatomegaly
(Primary Biliary Cirrhosis)-211 I Hepatomegaly (Haemochromatosis)-2131 Ascites=-zl S]
Abdominal Mass-218I Causes of Mass in Different Sites of Abdomen-218I Carcinoma of
Stomach-222I Carcinoma of Head of Pancreas (Mass in the Epigastrium)-224I Pancreatic
Pseudocyst-226 1Mass in the Epigastrium (Aneurysm of A011a)-227 1 Mass in Right Iliac
Fossa (IleocaecaJ Tuberculosis )-2281 Crohn Disease-229I Carcinoma of Colon-232

Chapter 5 Haematology _ 234


Generalized Lymphadenopathy (Hodgkin Lymphoma)-234I Generalized Lymphadenopathy
(Non-Hodgkin Lymphoma)-237 1 Generalized Lymphadenopathy (Chronic Lymphatic
Leukaemia)-239 I Purpura-241 1 Idiopathic Thrombocytopaenic Purpura-243 I Hen-
och-Schonlein Purpura-245 I Splenomegaly (Hereditary Haemolytic Anaemia)-247 1
Splenomegaly (Chronic Myeloid Leukaemia)-249 1 Splenomegaly (Myelofibrosis)-2S1 I
Iron-deficiency Anaemia-2S2

Chapter 6 Endocrinology _ 257


Introduction-257 1 Examination of Thyroid Gland-257 1 Face in Thyroid Disease (by
Looking at the Face)-259 1Thyrotoxicosis-260 IGraves Disease-264I Simple Multinodular
Goitre-267 I Toxic Nodular or Multinodular Goitre-268 1 Simple Diffuse Goitre-269 I
Solitary Nodule or Simple Nodular Goitre-270 IHypothyroidism-czvs ] ACIomegaly-279I
Cushing Syndrome-284 I Addison Disease-288 I Tall Stature-292 I Klinefelter
Syndrome-293I Short Stature-29SI Diabetes Mellitus-296

( haprer 7 Ncpl 1.1l ,"'. _ t J

Mass in Flank (ReoaJ)-302 IBilateral Renal Mass (Polycystic Kidney Disease)-303 1


Unilateral Mass (Renal Cell Carcinoma)-304 IMass in Left or Right Iliac Fossa (Transplanted
Kidney)-306 INephrotic Syndrome-308

Chapter 8 Neurology _ 313


Introduction-313 I Neurological Examination of Lower Limb-313 1 Spastic Paraplegia
(Spinal Cord Compression)-316 1 Monoplegia (Brown-Sequard Syndrome)-320 I
Monoplegia-321 1 Multiple Sclerosis (Presenting as Spastic Paraplegia)-322 I Flaccid
Paraplegia (GBS)-32S I Polyneuropathy-328 I Polyneuropathy [Subacute Combined
Degeneration-(SCD)J-330 IMotor Neuron Disease (MND)-332I FriedreichAlaxia-336I
Cerebellar Lesion-338 1 Parkinsonism-340 I Chorea-34S 1Tremor-348 I Speech-349 I
Oysphasia-350 I Oysarthtia-352 I Examination of Hands (Wasting of Small Muscles of
Hands)-3S3 1 Syringomyelia-354 I Myotonic Dystrophy-3S6 I Myopathy (Muscular
Dystrophy)-358 1 Carpal Tunnel Syndrome-360 I Ulnar Nerve Palsy-362 1 Radial Nerve
Palsy-363 1Cranial Nerves-363I Facial (VIIth) Nerve Palsy-366I Vlth Nerve Palsy-3691
Multiple Cranial Nerve Palsy-370 I CVD with Hemiplegia-372 I Gail AbnormaJity-376I
Orofacial Dyskinesia-3771 Foot Drop-377I Charcot-Marie-Tooth Disease-378
CONTENTS

Chapter 9 Rheumatology _ 381


Introduction-381 1 Systemic Sderosis-382 1 Raynaud Phenomenon or Disease-386 1
Rheumatoid Hand-388 1 Knee Joint Arthritis (Examination of Knee Joint)-396 1 Juvenile
Idiopathic Arthritis (JIA/JCA)-3991 Septic Arthritis-4021 HaemophilicArthritis-4031 Reiter
Syndrome-4061 Psoriatic Arthropathy-4081 Ankylosing Spoudylitis-e-tlt ] Dermatomyositis
or Polymyositis-4151 Proximal Myopathy-420 1Gout-4221 Systemic Lupus Erythematosus
(SLE)-4251 Vasculitis-4331 Osteoarthrosis-4341 Charcot Joint-43s

Chapter 10 Examination of the Eye _ 437


Introduction-4371 Optic Atrophy-438 I Papilloedema (Pure)-440 1 Central Retinal Vein
Occlusion (CRVO)-442 I Hypertensive Retinopathy-443 1 Retinal Haemorrhage-444 I
DiabeticRetinopathy-44sJ Retinitis Pigmentosa-4491 Ptosis-449I Horner Syndrome-452
1 Argyll Robertson Pupil (AR Pupil)-453 I Different Types of Pupil-454 I Holmes-Adie
PupU-4s5 1 Exophthalmos-455 1 Nystagmus-457 I Subhyaloid Haemorrhage-459
1 Choroidoretinitis-460 I Retinal Detachment-461 1 Bitemporal Hemianopia-461 1
Miscellaneous-462

Chapter 11 Dermatology _ 465


Introduction-465 1 Psoriasis-466 1 Erythema Multiforme and Stevens-Johnson
Syndrome-4701 Dermatitis Herpetiformis-4 721 Herpes Zoster (Shingles )-4 741 Acanthosis
Nigricans-476 1 Ichthyosis-477 1 Pemphigus Vulgaris-479 1 Bullous Pemphigoid-481 !
Arsenicosis (Chronic Arsenic Toxicity)-483 1 Scabies-485 1 Lupus Vulgaris-487 1 Lichen
Planus-488 1 Exfoliative Dermatitis-491 ! Alopecia-493 ! Vitiligo-495 1 Neurofi-
broma-496 I Mycosis Fungoides-498 ! Tuberous Sderosis-499 I Kaposi Sarcoma-500 I
Pityriasis Versicolor-SOl

Chapter 12 Miscellaneous _ 502


Face in Different Diseases-s02 1 Butterfly Rash-504 I Turner Syndrome-50S I Down
Syndrome-s-So? I Bilateral Parotid Enlargement-s08 I Xanthelasma-509 I Sturge-Weber
Syndrome-SIll Hereditary Haemorrhagic Telangiectasia-512I Yellow Nail Syndrome-513
I Normal Case-514

'ifiit.H'~jCommon Interpretations in Medicine 515


Chapter 13 Data Interpretation _ 517
Hepatology-517INephrology-5191 Neurology-5211 Haematology-523! Respiratory-529
1Endocrine-s29

Chapter 14 X-Ray,cr and MRl _ 536


Pleural Effusion-536 I Mass Lesion-537 I Solitary Pulmonary Nodule-538 I Multiple
Secondaries in Lung-538 I Consolidation-539 J Lung Abscess-S40 I Pulmonary
Tuberculosis-540 1Miliary Tuberculosis-541! Calcification of the Lung Parenchyma=-Sqz ]
Emphysema-542J Bullae-s43J Surgical Emphysema-5431 Pneumothorax-S44I Hydrop-
neumothorax-545 J Collapse of Lung-S45 1 Rib Resection-546 I Bilateral Hilar
Lymphadenopathy-546 I Sarcoidosis-547 1Azygos Lobe-547 I Cavity Superimposed on
Cardiac Shadow-548 I Bronchiectasis-548 I Homogeneous Opacity of One Hemitho-
rax-s49J Mediastinal Widening-549I Gas Under the Diaphragm-cssu JSitus Inversus-550
I Dextrocardia-551 1 Mitral Stenosis-551 I Ventricular Aneurysm-s-Sfiz 1 Atrial Septal
Defect-5s2 I Tetralogy of Fallot-553 1 Cardiornegaly=.S'B I Pericardia I Effusion-554 I
CONTENTS

Pericardial Calcification-554 I Pulmonary Oedema-555 I Pacemaker-SSG I Metallic


Prosthetic Valve-556 I Ankylosing Spondylitis-557 I Pott Disease-558 I Multiple
Myeloma-558 I Hereditary Haemolytic Anaemia-559 I Acromegaly-560 I Tophi-560 I
Rheumatoid Hand-56lIResorption of Terminal Phalanges-56lIRickets-562IScurvy-563
I Avascular Necrosis of Femoral Head-563 I Endoscopic Retrograde Cholangiopancreatogra-
phy (ERCP)-564 I Achalasia Cardia-564 I Carcinoma of Oesophagus-565 I Carcinoma
Stomach-565I Gastric Outlet Obstruction-566 I Pancreatic Calcification-566 I Polycystic
Kidney Disease-567I crSCAN-567I MRI SCAN-568I Few Common cr
Scans-568

Chapter IS ECG _ 572


Basic Concept of ECG-572 I Interpretation of ECG-573 I Normal ECG-574 I Details of
Waves and Intervals-S74I Rhythm of Heart-578I Calculation of Heart Rate-578I Cardiac
Axis-579I ECG 01: Atrial Fibrillation-579I ECG 02: Atrial Flutter-S80 IECG 03: Ventricular
Tachycardia-580 I ECG 04: Acute Myocardial Infarction-58l I ECG 05: Old Myocardial
Infarction-582 I ECG 06: First-Degree AV Block-583 I ECG 07: Second-Degree AV Block
(Mobitz Type 1)-583 I ECG 08: Second-Degree AV Block (Mobitz Type 11)-5841 ECG 09:
Complete Heart Block-584I ECG 10: Left Bundle Branch Block-585I ECG 11: Right Bundle
Branch Block-585I ECG 12: Left Ventricular Hypertrophy-586I ECG 13: Right Ventricular
Hypertrophy-587I ECG 14: P Mitrale-S88I ECG 15: P Pulmonale-S89I ECG 16: Pulsus
Bigeminy-S89I ECG 17: Acute Pericarditis-590 I ECG 18: Pacemaker-59ll ECG 19: Sinus
Tachycardia-591 I ECG 20: Sinus Bradycardia-592 I ECG 21: Supraventricular Tachycar-
dia-592I ECG 22: Ventricular Premature Beat-593I ECG 23: Ventricular Fibrillation-594I
ECG 24: Hyperkalaemia-594

Chapter 16 Pictures _ 596


Picture 01: Infected Scabies-S96 I Picture 02: Purpura-597 I Picture 03: Rheumatoid
Arthritis-597I Picture 04: Arthritis Ml,ltilans-598I Picture 05: Graves Disease-S98I Picture
OG: Psoriasis-599 I Picture 07:SYstem.t:c Lupus Erythematosus-coon I Picture 08: Erythema
Multiforme-GOO I Picture 09:)-epr.o$Y~;~O:iJ P~c~~re 10: Stevens-Johnson Syndrome-GOl I
Picture 11: Ring Worm-6021"PTct.are:f2: Subconjuctival Haemorrhage-G021 Picture 13: Bell
Palsy-603lPicture 14: Ramsay HuntSyndrome-603 IPicture 15: Goitrous Myxoedema-604
I Picture 16: Conorrhoea=-Su-l ] Picture 17: Hirsutism-60S IPicture 18: Osteoarthrosis-605
I Picture 19: Peripheral Vascular Disease-60G I Picture 20: Hypopituitarism-60G I Picture 21:
Herpes Zoster-6071 Picture 22: Obstructive Jaundice-G071 Picture 23: Wegener Granuloma-
tosis-6081 Picture 24: Xanthelasma-6081 Picture 25: Baker Cyst-609 I Picture 26: Pectus
Carinatum-6091 Picture 27: Corneal Arcus-609 , Picture 28: Acromegaly-GlO , Picture 29:
Primary Optic Atrophy-61O , Picture 30: Papilloedema=ril l ] Picture 31: Central Retinal Vein
Occlusion-611

Chapter 17 Instruments _ 612


Instrument 01: Bone Marrow Aspiration Needle-6I2 , Instrument 02: Lumbar Puncture
Needle-613 I Instrument 03: Liver Biopsy Needle-6IS , Instrument 04: Pleural Biopsy
Needle-6IG I Instrument 05: Aspiration Needle with Rubber Tube-6I7 , Instrument 06:
Intravenous Cannula=-Sl S] Instrument 07: Foley Catheter=-tilS] Instrument 08: Nasogastric
Tube (Ryle Tube)-G191 Instrument 09: Air Way Tube-61 91 Instrument 10: ESRTube-620'
Instrument 11: Metered-Dose Inhaler-620 I Instrument 12: Accuhaler-621I Instrument 13:
Evohaler-621 , Instrument 14: Peak Flow Meter-622 , Instrument 15: AMBU Bag-622 I
Instrument 16: Tongue Depressor=-Szz ] Oral Rehydration Salt-G23
Bibliography 625
Index 627
I

Section A

Short Cases
CHAPTER 1

GENERAL EXAMINATION
'Doctors must be good observants like detectives"
-Author's View

Introd uction
During examination of short case, examiner usually • Look at the face (thyrotoxic, myxoedematous
asks to examine a particular part of the body, or some- or Cushingoid).
times the whole system or whole body. Candidates • Perform the general examination (obesity,
should prepare themselves to examine accordingly, but features of myxoedema, pigmentation in
systematically. Common instructions by the examiner Addison disease).
or short cases in any examinations are:
7. Rheumatology:
1. General examination: Perform the general exami- • Examine the hands (mostly rheumatoid hand,
nation of this patient. systemic sclerosis).
2. In cardiovascular system (CVS): • Examine the knee joints (or any particular
• Examine the precordium. joint-monoarthritis, or polyarthritis, effusion
• Palpate and auscultate the precordium. in knee joints).
• Examine the pulse. • Examine the face and relevant [features of
•. Examine the CVS. systemic sclerosis, systemic lupus erythematosus
(SLE), dermatomyositis].
3. In respiratory system: • Look at the skin (dermatomyositis).
• Examine the chest (front or back or both).
• Auscultate for respiratory system. 8. Dermatology:
• Percuss the back and auscultate. • Perform the general examination (psoriasis,
exfoliative dermatitis).
4. Abdomen:
• Look at the skin of this part. What are the
• Examine the abdomen.
possibilities (skin rash, erythema multi-
• Palpate the abdomen and relevant. forme, purpura, Stevens-Johnson syndrome,
• Examine the gastrointestinal system. blisters )?
S. Neurology:
9. Eyes:
• Examine the lower limb or upper limb.
• Examine the eyes [ptosis, squint, pigmen-
• Examine the cranial nerves, or a particular tation of sclera, Kayser-Fleischer (KF) ring,
cranial nerve (e.g. facial nerve). corneal arcus, subconjunctival haemorrhage,
• Show me the cerebellar signs. exophthalmos ].
• Look at the gait of the patient. • Perform fundoscopy (optic atrophy, retin-
6. Endocrinology: opathy).
• Examine the neck (mostly thyroid).
• Examine the thyroid gland and relevant 10. Miscellaneous:
(thyroid disorder and features of hyper- or • Depends on particular type of case, e.g. Turner
hypothyroidism ). syndrome, Down syndrome, xanthelasma, etc.
_ SHORT CASES IN CLINICAL MEDICINE

• Look at the tongue. What is the diagnosis? • Examine the hands. What are the diseases that
What are the diseases that can be diagnosed by can be diagnosed by examining the hands?
looking at the tongue? • What diseases are diagnosed by looking at
• Look at the face. Name some diseases that can the nail?
be diagnosed by looking at the face.

General Examination

Be sure that you are on the right side of the patient.


Introduce yourself ask for permission, and be gen-
tle and polite. A friendly handshake may offer
patient's reassurance and cooperation, also may
help to get informations such as warm and sweaty
hands (thyrotoxicosis), cold and sweaty hands
(anxiety), dry and coarse hands (hypothyroid),
doughy feeling (acromegaly) and slow relaxation
of grip (myotonia).
Before starting the examination, take a few moments Cachexia
to look quickly from head to foot (by keeping
open your wide-angled lens), which may give a clue Main Points in General Examination
regarding the diagnosis. Many cases can be diag-
nosed by just looking at a glance (such as emaciation • Appearance (ill-looking, depressed, anxious,
or cachexia, obesity, acromegaly, myxoederna and Cushingoid or expressionless face).
Cushing syndrome). • Built (obese, emaciated or cachexic, tall, short
Facial expression will give a good clue for the diag- or normal).
nosis (see page 502), for example: • Nutrition (well nourished, poor or normal).
• Poverty in expression or mask-like face (in • Decubitus (on choice, propped up or Moham-
Parkinsonism). medan prayer position).
• Anaemia (palpebral part of conjunctiva, tongue,
• Agitated or terror face (in thyrotoxicosis).
palm, nails and whole body).
• Exophthalmos (in Graves disease). • Jaundice (sclera, undersurface of tongue, palm
• Puffy face (in myxoederna or nephrotic syndrome). or whole body).
• Apathy (in depression). • Cyanosis (tip of nose, lips, ear lobule, tongue, tip
of fingers and toes).
Look at the height:
• Tall stature (gigantism, Marfan syndrome,
Klinefelter syndrome).
• Short stature (constitutional, genetic or familial,
achondroplasia, cretinism or juvenile hypothy-
roidism, hypopituitarism).
Besides routine general examination, some extra find-
ings in individual cases should be seen. For example:
• In Cushing syndrome, some extra findings
are very essential, e.g. striae, moon face and
buffalo hump.
• If butterfly rash is seen on the face, check for rash
in other parts of the' body. Also, heliotrope rash
and alopecia. Tall stature (Marfan syndrome)
~~ __ 1.:_. GENERAL EXAMINATION _

• Osler node and splinter haemorrhage [in


subacute bacterial endocarditis (SBE)].
• Heberden node.
• Bouchard node.
• Gangrene or nail fold infarct or nail fold
telangiectasia.
• Ulceration.
• Wasting.
Nail (Ieuconychia) • Skin rash or Gottron patch (dermatomyositis).
3. Other findings:
• Spider angioma.
• Parotid gland enlargement (unilateral or bilateral).
• Skin rash.
• Striae.
• Campbell de Morgan spots.
• Purpura.
• Vitiligo.
Jaundice • Deformity (kyphosis, scoliosis and lordosis).
• Clubbing (see fluctuation of nail base, angle If these findings are present, you must mention them.
between the nail and its base, curvature of nails,
look for hypertrophic osteoarthropathy by pressing N.B. Mention, if the patient has any cannula, catheter,
the lower end of tibia-fibula or radius-ulna. Be nasogastric (NG) tube, central venous CCV) line,
aware of differential clubbing-it means clubbing arteriovenous CAY) fistula, etc.
in toes but not in fingers).
Never forget to examine the lower limbs (there may
• Koilonychia (dryness, brittleness, flattening,
be unilateral leg swelling, deep vein thrombosis [DVT],
thinning and spooning of nails).
differential cyanosis, differential clubbing, trophic
• Leuconychia (white spots or white nail).
ulcer, gangrene or infarction at the tip of toe).
• Oedema (in leg above medial malleolus, sacrum if
Finally, present the case systematically. Remember, you
the patient is recumbent).
must follow the examiner's instructions, For example,
• Dehydration (skin turgor or dry tongue).
• Pigmentation (exposed parts, face, neck, palmar examiner may ask:
creases, knuckles, inner side of mouth or recent scar). • What are your findings? (You should mention the
• Lymph nodes (examine systematically in different positive and important negative findings.)
areas). • Have you finished your examination? Now tell
• Thyroid gland (if enlarged, examine in detail and about your findings. (You should tell systematically
see the features of toxicosis or hypothyroidism). the positive and important negative findings.)
• Breasts.
• Body hair distribution (including head to see Examples of Presentation of a Case
alopecia).
• Pulse. I

• Blood pressure (BP). Case No.1 r

• Temperature.
• The patient is ill-looking, emaciated or cachexic
• Respiratory rate.
with poor nutrition.
Relevant findings should be examined according to • Mildly anaemic, nonicteric and noncyanosed.
individual cases, for example: • There is clubbing involving all the fingers and
1. In the face: If xanthelasma is present, see. corneal toes, no leuconychia and no oedema.
arcus, xanthomatous nodules in others parts (elbow, • Thyroid gland is normally palpable and there is
knees, extensor surfaces, Achilles tendon and palmar no lymphadenopathy.
creases). • He is normally pigmented.
2. In the hands: • Pulse: 80/min, BP: 120/8 mmHg, respiration:
• Dupuytren contracture. 18/min and temperature: normal.
• Palmar erythema.
_ SHORT CASES IN CLINICAL MEDICINE

Examiner may ask:


Case No.5
Q: What are the causes of cachexia in this elderly patient?
A: Tuberculosis, diabetes mellitus, thyrotox-icosis, The patient is tall and obese.
malignancy, malnutrition or malabsorption.
Q: What are the causes of cachexia in this young lady? Examiner may ask:
A: Tuberculosis (TB), diabetes mellitus (DM), thyro-
toxicosis, malnutrition or malabsorption, anorexia Q: Look at this patient. What else do you want to see?
nervosa, Addison disease. A: I want to see gynaecomastia, testis (atrophy), sec-
ondary sex characters and voice. This may be a case
Q: What are the causes of clubbing in this case?
of Klinefelter syndrome.
A: See later in the topic 'clubbing'.

Case No.2 Case No.6

• This patient is obese, mildly anaemic, nonicteric, Young patient appearing pigmented.
noncyanosed.
• No clubbing, koilonychia or leuconychia.
Q: What else do you want to see?
A: Blood pressure (Jow in Addison disease. I want to see
Examiner may ask: BP at both standing and lying position to see postural
hypotension) and also I want to see pigmentation in
Q: What do you think are the causes of obesity? What
other parts of the body (such as inner side of mouth,
else do you like to see in obesity?
palmar crease, recent scar).
A: I want to look for striae, central obesity, moon face
and buffalo hump (in Cushing syndrome). For
details, see in the topic 'obesity'. Case No.7

Case No.3 I
I
The patient has generalized oedema.

• The patient has generalised lymphadenopathy


(tell the site and size that are firm, discrete, Q: What are the possibilities? What else do you want to
nontender, not fixed with overlying skin or examine?
underlying structure). A: Generalized oedema may be due to nephrotic
syndrome, hypoproteinaemia, cirrhosis of the liver,
etc. (For details, see the relevant topics later in the
Q: What are the causes of generalised lymphadenopathy? respective chapters.)
A: See later in lymphadenopathy.

Some Important Topics in General


Case No.4 Examination
The patient is tall, lean and thin.
Causes of Pallor (Face or Whole Body)
Examiner may ask: • Anaemia (the commonest cause).
Q: Look at the patient. What are your findings? • Hypopituitarism (or hypogonadism).
A: The patient is tall, lean and thin; extremities are • Others:
long, face is narrow and elongated; arachnodac- o Shock.
tyly (long, elongated fingers)-likely to be a case of o Syncope.
Marfan syndrome. o Left ventricular failure (LVF).
Q: What else do you like to examine?
A: I want to examine the eyes (dislocated lens), heart
Causes of Yellow Body
(aortic regurgitation), high-arched palate, arm span • Jaundice (the commonest cause).
and height. • Carotenaemia.
_~ 1_. GENERAL EXAMINATION _

Causes of Carotenaemia Causes:


o Exposure to cold.
• Excess intake of carotene-containing food (carrot, o Raynaud phenomenon.
mango and tomato). o Heart failure.
• Hypothyroidism (due to impaired metabolism of
carotene by liver). • Central: Either due to imperfect oxygenation of
blood in lung or admixture of venous and arterial
Q: How to differentiate between jaundice and blood. It is evident when 02 saturation falls below
carotenaemia? 80-85%. Best site to see is tongue.
A: In jaundice, sclera is involved. In carotenaemia,
Causes:
sclera is not involved. o Respiratory: There is defect in oxygenation of
blood in the lungs:
Cyanosis
- Chronic obstructive pulmonary disease (COPO).
It is the bluish discolourationof skin and mucous - Severe pneumonia.
membrane due to increased amount of deoxygen- - Acute severe bronchial asthma.
ated haemoglobin in the blood. Cyanosis is not seen - Massive pulmonary embolism.
until the amount of deoxygenated haemoglobin (Hb) - Pulmonary infarction.
is >5 gm%. - Diffuse parenchymal lung disease (DPLD).
o Cardiac:
- Cyanotic congenital heart disease: Fallot
tetralogy, transposition of great vessels.
- Shunt anomaly [right-to-left shunt called
Eisenmenger syndrome): Atrial septal defect
(ASD), ventricular septal defect (VSD), patent
ductus arteriosus (PDA).
- Heart failure.
- Cardiogenic shock.
o Others
- High altitude (physiological).
CyanosiS at fingers - Polycythaernia.

Q: What are the differences between central and


peripheral cyanosis?
A: The differences between central and peripheral
cyanosis are as follows:

Points Central Peripheral


Mechanism Imperfect oxygenation of Local
blood in lung or admixture vasoconstriction
of venous and arterial or reduction of
blood in heart disease. arterial flow.

Cyanosis Generalised Localised

Cyanosis at tip of nose and lip Affected Warm Cold


part

Two types of cyanosis: Application Does not disappear Disappears


of warm
• Peripheral: Due to localised reduction of blood
Oxygen Cyanosis may disappear Disappears
flow on exposure to cold causing capillary
in pulmonary case (except
vasoconstriction (lip is blue in cold weather). Also,
in right-to-Ieft shunt)
occurs in reduced cardiac output (heart failure or
Tongue Always involved Never involved
shock). Tongue is spared in peripheral cyanosis.
_ SHORT CASES IN CLINICAL MEDICINE

Q: Why tongue is not involved in peripheral cyanosis? Q: What is enterogenous cyanosis? How to diagnose?
A: Because tongue is always warm, and circulation is A: Discolouration of skin due to the presence of
good in tongue. abnormal pigments in blood, as in sulphaemo-
globinaemia or methaemoglobinaemia. History
Q: Why there is no cyanosis in severe anaemia? of intake of some drugs (sulphonamide, phenace-
A: Because in severe anaemia, Hb is low and fully tin and dapsone). No dyspnoea in enterogenous
saturated, no excess deoxygenated Hb (in poly- cyanosis or no other respiratory symptoms.
cythaemia, cyanosis may occur even in mild hypoxia).

Oedema (non-pitting) Oedema (pitting)

Clubbing
Proceed as follows:
• First look whether gross clubbing is present or not.
• If not, look carefully at the angle between
nail bed and skin margin at the level of the
observer's eye (normal angle is 150 between nail
0

and cuticle).
• Next, see the fluctuation at the nail bed.
• Place the corresponding opposite nails of both
hands (normally, small gap is present; if clubbing,
Clubbing of all fingers
there is reduction or absence of the gap called
window sign or Schamroth sign).
• Feel the terminal parts of fingers (bulbousness due
to increased thickening of nail bed); also look at
the nails to see convexity from side-to-side.
• In advance stage, drumstick and later parrot-beak
appearance.
• Always see hypertrophic osteoarthropathy; press
the ends of long bones (radius and Ulna, tibia
and fibula). Look at the face (patient winces due
to pain).

N.B. You must look at both the fingers and toes. Clubbing in toes
______ , • GENERAL EXAMINATION _

N.8. The commonest cause in the elderly is bronchial


Presentation of a Case 11-- -,
carcinoma and in young is bronchiectasis.
• This patient has generalised clubbing involving Q: What is differential clubbing? What are the causes?
all the fingers and toes with drumstick A: It means clubbing in the toes, but not in the fingers.
appearance (or parrot beak) and hypertrophic
osteoarthropathy (mention, if any).
Causes of differential clubbing

Q: What else do you want to examine? • POA with reverse shunt (also there is cyanosis
A: I want to take the family history of clubbing and to in toes, not in finger called differential
examine the respiratory system, heart and abdomen cyanosis).
to find out the causes. • Infected abdominal aortic aneurysm.
• Coarctation of abdominal aorta.
Q: What is Schamroth sign?
A: When finger nails of the corresponding fingers of Causes of unilateral clubbing
each hand are placed against each other, normally • Axillary artery aneurysm.
there is a diamond-shaped gap between them called • Bronchial arteriovenous aneurysm.
Schamroth sign. It is lost in finger clubbing. • Others: Aneurysm of ascending aorta,
subclavian or innominate artery.
Q: What are the causes of clubbing?
A: Tell the causes according to the age. Causes of clubbing in a single finger

• Trauma (the commonest cause).


If the patient is middle-aged or elderly, the • Chronic tophaceous gout.
• Sarcoidosis.

:J
causes are:

1. Respiratory Causes of clubbing with cyanosis


• Bronchial carcinoma (squamous cell type,
• Fibrosing alveolitis.
unusual in small cell type).
• Cyanotic heart disease (Fallot tetralogy).
• Suppurative lung disease (bronchiectasis,
• Cystic fibrosis.
lung abscess and empyema thoracis).
• Bilateral extensive bronchiectasis.
• Fibrosing alveolitis (or ILO).
• Pulmonary TB (in advanced stage with
fibrosis).
• Pleural mesothelioma.
Stages of Clubbing
2. Cardiac (SBE). • Early sign: Increased sponginess of proximal nail
3. Others: Cirrhosis of liver, inflammatory bed (fluctuation is positive). This is due to increased
bowel disease, familial (rare) and idiopathic. proliferation of cells at nail base.
If the patient is young or of early age, the • Swelling of subcutaneous tissue in nail base
causes are: leading to shiny, red colouration and obliteration
of skin creases.
1. Respiratory
• Suppurative lung disease (bronchiectasis • Increased curvature of nails. Hence nails become
and lung abscess). convex, and the angle between the nail and its base
is obliterated.
• Cystic fibrosis.
• Pulmonary TB (in advanced stage with • Later, drumstick appearance.
fibrosis). • Lastly, parrot-beak appearance.
2. Cardiac • Hypertrophic osteoarthropathy may occur.
• SBE.
• Fallot tetralogy (clubbing with cyanosis). Q: What are the pathogeneses or mechanisms of
3. Others: Chronic liver disease (CLO) (Wilson clubbing?
disease and alpha-l-antitrypsin deficiency). A: Actual mechanism is unknown. But there are
possible hypotheses:
_ SHORT CASES IN CLINICAL MEDICINE

• Arterial hypoxaemia. Q: What history should be taken in clubbing?


• Humoral substances that cause vasodilatation A: As follows:
(bradykinin, prostaglandins, 5-hydroxytryptamine). • Family history.
• Neurogenic factors through vagal stimulation • Respiratory problem such as cough, haemoptysis,
(vagotomy revert dubbing in some cases). chest pain, breathlessness.

• Platelet-derived growth factor (PDGF) released • Cardiac: History of cyanotic congenital heart disease.
from megakaryocyte and platelet emboli in • Gastrointestinal tract (GIT): Diarrhoea, blood in
nail bed is thought to be a probable factor, the stool.
which causes increased capillary permeability, • History of liver disease.
fibroblastic activity and arterial smooth muscle
hyperplasia in the nails resulting in dubbing. Q: What investigations should be done in clubbing?
• Tumour necrosis factor has been implicated. A: As follows:
• Full blood count.
• Chest X-ray.
• Ultrasonography (USC) of whole abdomen.
• Echocardiography.
• Other, according to suspicion of cause (barium
enema, follow through, colonoscopy. etc. for
inflammatory bowel disease, liverfunction test, etc.).

Hypertrophic Osteoarthropathy
Thyroid acropachy in both hands
• Hypertrophic osteoarthropathy (HOA) is the triad
of clubbing, arthritis and subperiosteal new bone
formation (periosteal inflammation at the distal
ends of long bones in radius, ulna, tibia, fibula;
although any bone may be involved).
• There is swelling and tenderness at the lower ends
of forearm and leg. Clubbing is usual, but not
invariable in HOA (rarely occur without HOA).
• HOA may be primary or secondary to any cause
of clubbing.
• The commonest causes of HOA are bronchial
Clubbing with cyanosis carcinoma (squamous cell type) and pleural
mesothelioma.
Q: What is thyroid acropachy? • Mechanism and pathogenesis of HOA are same
A: If hyperthyroidism is associated with clubbing, dif- as clubbing.
fuse thickening of distal extremity and subperiosteal
new bone formation of hands and feet, it is called
thyroid acropachy.
• Usually involves metacarpals and phalanges.
• Long bones are rarely involved.
• Usually associated with exophthalmos.
• Thyroid acropachy is present on ly in Gravesdisease.

Q: If there is clubbing, can it be reversible by any


therapy?
A: Yes, clubbing may be reversible, rarely, if vagot-
omy is done (in idiopathic or sometimes familial
clubbing). Thyroid acropachy
_________ 1 • GENERAL EXAMINATION _

Primary hypertrophic osteoarthropathy:


• It is also called pachyderrnoperiostosis.
• Usually begins in childhood or puberty, inherited
as autosomal dominant, more in male.
• There is thickening of the skin of face and scalp,
which is greasy, pigmented, furrowed forehead
giving rise to 'Leonine facies'. Also, increased
sweating especially of palm and sole.
• Usually marked clubbing is present and distal
Pachydermoperiostosis
extremities are large and thick, grving rise to
elephant feet (very large feet, confuses with
mucopolysaccharidoses) .

Koilonychia
Usual instructions are:
• Perform the general examination.
• Look at the nail. What is your diagnosis?

Presentation of a Case If-- -----,

• The patient is severely anaemic.


• There is koilonychia involving all the nails of
fingers and toes.
• Tongue is smooth and shiny with atrophy of Koilonychia
papillae.

Diagnosis is severe anaemia with koilonychia (due to


iron-deficiency anaemia).

Q: What are the causes of koilonychia?


A: As follows: (
1. Iron-deficiency anaemia (the commonest
cause).
2. Others (rare, better do not mention until
asked)
• Trauma (rarely in garage mechanics, who
regularly fit tyres). Pallor of palm
• Thyrotoxicosis.
• Fungal infection. Q: What are the stages of koilonychia?
• Raynaud disease. A: As follows:
• Dryness, brittleness and ridging (first stage).
Q: What is koilonychia?
• Flattening and thinning (second stage).
A: A disorder in which nail is concave or spoon
• Spooning or concavity (third stage).
shaped.
Q: What else do you want to ask or see? Why?
Q: What is the mechanism of koilonychia?
A: Unknown, probably results from slow growth of A: I want to take history of dysphagia. Also, I want
nail plate. to look at the tongue to see evidence of glossitis.
- Presence of these two findings associated with
koilonychias is suggestive of Plummer-Vinson
syndrome.

Q: What is Plummer-Vinson syndrome? What are the


features? How to treat?
A: It is the combination of:
• Iron-deficiency anaemia.
• Dysphagia (due to postcricoid web secondary to
Glossitis with angular stomatitis in
epithelial degeneration).
iron-deficiency anaemia
• Glossitis.
It is also called Paterson-Brown-Kelly syndrome,
common in women; its cause is unknown. There is
constriction in upper oesophageal sphincter in the
postcricoid region and appears radiologically as a
web. This web may be asymptomatic or may pro-
duce dysphagia. It may be difficult to see endoscopi-
cally. Rarely, there is increased risk of squamous cell
carcinoma.
Treatment: Iron therapy. Rarely, dilatation may
be required. If severe anaemia, blood transfusion Pencil cells, target cells, microcytosis,
should be given. hypochromia in iron-deficiency anaemia

Nail Changes in Different Diseases


Instruction to the examinee: • Renal cause [nephrotic syndrome, chronic renal
failure (CRF)).
• Examine the hands of this patient.
• Liver diseases [chronic liver disease (CLD),
• Look at the nail. What is your diagnosis?
cirrhosis of liver).
• Malnutrition (malabsorption, less intake offood).
Presentation of a Case • May be normal finding in some cases.
(Supposing White Nail) • Others: Rare (lymphoma, fungal infection,
congenital).
• All the nails of the toes of toes and fingers are • Striate leuconychia (a normal finding due to
white. minor trauma).
Mechanism of leuconychia is unknown; it may be due
to compression of capillary flow by extracellular fluid.
My diagnosis is leukonychia.
Q: What does leukonychia indicate?
A: It indicates hypoalbuminaemia. However, it may be
physiologically normal.

Q: Which organ is involved?


A: Liver, kidney, GIT; also, less intake of food.

Q: What is leukonychia? What are the causes?


A: It means white nail. It may be diffuse, punctate,
linear or striate (white transverse flecks-a normal
finding). Causes are: Leuconychia
1 •

• Systemic sclerosis.
• Rheumatoid arthritis (RA).
• Polyarteritis nodosa.

Splinter haemorrhage: Linear dark brown,


longitudinal flecks, parallel to long axis of nail.
Causes are:
• Trauma (the commonest).
• SBE.
• Septicaemia.
• Collagen disease (vasculitis): SLE, RA and
polyarteritis nodosa.
Nail fold infarction • Others: Haematological malignancy, severe
anaemia, psoriasis. Rarely in trichinosis (usually
transverse haemorrhage).

Half-and-half nail: Proximal part of nail is white-


to-pink and distal part is red or brown. Causes are:
• CRF (the commonest cause).
• Cirrhosis of liver.
• Occasionally, in normal person.
• Red half moon occurs in congestive cardiac
failure (CCF).

Splinter haemorrhage

Q: How will you investigate leukonychia?


A: 1 will take the history of the patient. Investigation
should be done according to the history and physi-
cal finding such as:
• If history is suggestive of CLD: I will do liver
function test.
• If history is suggestive of renal disease: I will
investigate accordingly.
• If history is suggestive of CIT: I will investigate
accordingly.
Half-and-half nail
Q: What diseases are diagnosed by examining the nail?
A: Nail abnormality may occur in many local, systemic
and dermatological diseases. Hence, these should
be examined carefully. A good visual impression
is very essential and a spot diagnosis can be done
easily. These are described below.
Clubbing, koilonychias, leukonychia: Already
described.
Pale nail: Found in anaemia.
Nail fold infarction: Causes are (usually vasculitis
due to any cause):

• SLE.
• Dermatomyositis. Nail fold telangiectasia
_ SHORT CASES IN CLINICAL MEDICINE

Beau line Onycholysis/fungal nail

Nail fold telangiectasia: Causes are:

• SLE.
• Systemic sclerosis.
• Dermatomyositis.
• Mixed connective tissue disease (MCTD).
• Raynaud phenomenon.

Beau line: Nonpigmented transverse line or grooves


Meeline
in nail due to transient arrest of nail growth. This
appears at the same time, on all the nails, a few
weeks after an acute illness. Causes are:

• Chronic illness (chronic infection, mal ignancy


and collagen disease).
• Prolonged fever.
• Pneumonia.
• Coronary artery disease.
• Others: Cachexia, malnutrition, psychiatric
illness, use of cytotoxic drugs.
Yellow nail
Onycholysis: Separation of distal nail plate from
the nail bed (free edge looks white). Causes are: Yellow nail: Found in yellow nail syndrome, an
• Psoriasis (the commonest). inherited disease in which the nails are thick, yel-
low or pigmented with separation of distal part of
• Fungal infection.
nail bed due to hypoplasia of lymphatic system. It is
• Thyrotoxicosis (Plummer sign). associated with lymphoedema of legs, bronchiectasis
• Idiopathic. and pleural effusion.
• Occasionally drugs (tetracycline and psoralen).
• Porphyria. Loss of nail (or dystrophy): Causes are:
• Trauma or faulty manicure. • Severe lichen planus.
• Epidermolysis bullosa.
Mee line: Single transverse white band in nail. • Trauma (tooth biting).
Causes are:

• Chronic arsenic poisoning. Nail pitting (depression in nail): Causes are:


• CRF. • Psoriasis.
• Also, after chemotherapy and severe illness. • Alopecia areata.
_________ ,_. GENERAL EXAMINATION _

• Atopic eczema (when involves proximal nail bed). Absent or small, dysplastic nail: Its causes are:
• Pityriasis rosacea. • Nail patella syndrome [autosomal dominant
(AD), associated with no or hypoplastic patella
Brittle nail (easily broken): Causes are:
and glomerulonephritis, abnormalities in eye I.
• Iron-deficiency anaemia. • Others: Congenital, traumatic and vasculitis.
• Peripheral vascular disease.
Nail hyperpigmentation: May occur due to some
• Fungal infection.
drugs (such as zidovudine, doxorubicin, bleomycin,
• Hypocalcaemia.
cyclophosphamide, fluorouraciJ, melphalan and
• Psoriasis.
ni trosoureas).
• Injury (nail biting).
• Idiopathic. Terry nail: Proximal part is white or pink, but nail
tip is red or brown. It is due to decrease in vascular-
Blue nail: Normal white lunulae become blue, ity and an increase in connective tissue within the
found in Wilson disease due to deposition of copper nail bed. Causes are:
(normally, half-moon lunulae at the proximal end
of nail is white-blue half moon). Also found in • Old age (normally present in elderly).
cyanosis and ochronosis. • Cirrhosis of liver.
• CCF.
• Hyperthyroidism.
• Malnutrition.
• Renal failure.
Dark nail: May be a normal finding, mostly in
black people. Sometimes may be due to subungual
melanoma.

N.B. Remember the following points:


• Normal nail growth is 0.1 rum/day, finger nails
grow quickly than toe nails.
• Nail plate grows continuously and slowly at the
Fungal nail
rate of 1 em every 3 months. Hence, renewal of
finger nails take about 3-6 months, and toe
nails that grow more slowly take 1 year.
• Rapid growth of nail plate occurs in psoriasis.
• Nail growth is arrested by acute illness and
ischaemia (Beau line).

Hypoplastic nail

Red nail: May be normal finding. Also in poly-


cythaemia, carbon monoxide poisoning (cherry red).
Brown nail: Usually present in chronic kidney
disease (CKD).
Periungual or subungual fibroma: Suggests tuber-
ous sclerosis (epiloia).
Fungal nail: Nail has thick, white, green, black,
discolouration and crust formation. Periungual fibroma
SHORT CASES IN CLINICAL MEDICINE

Cervical Lymphadenopathy
The usual instructions are:
• Perform the general examination or examine the
neck.

Once lymph node (LN) is palpable, examine the


following points:

• Site (anterior or posterior chain, supraclavicular,


submental or submandibular in right or left or
both sides).
• Number (single or multiple).
• Size (large LN is abnormal, > 1 em is pathological.
Size 2 x 2 cm may be suggestive of neoplastic
lesion). Cervical lymphadenopathy (scar mark seen)
• Consistency (soft or firm, or rubbery or hard).
• Tenderness (suggests acute inflammation).
Presentation of a Case ,1-----------,
• Discrete or matted (matted in TB).
• Fixation (to underlying structure or overlying • There is cervical lymphadenopathy involving the
skin suggests malignancy). anterior chain in right or left side (mention the
• Skin (sinus, u]cerandsignsofacuteinflammation). position).
Tethering of skin suggests malignancy. Note for • LNs are of variable size and shape and firm
scar mark (biopsy), if any. in consistency; some are matted, nontender
and free from the underlying structure and the
overlying skin.

Q: What are the causes of cervical lymphadenopathy?


A: According to the characteristics of the lymph nodes
of that patient, mention the common causes as
follows:

If matted cervical lymphadenopathy, the


causes are:
• Tuberculous lymphadenitis (the commonest).
• Infection by atypical mycobacteria.
Cervical lymphadenopathy • Actinomycosis.
• Sometimes, lymphoma (in advanced stage),
metastasis.

If lymphadenopathy with sinus, the causes


are:
• Tuberculous lymphadenitis.
• Actinomycosis.
If lymphadenopathy with biopsy marking,
the causes are:
• Tuberculosis.
• Lymphoma.
• Secondaries.
Tuberculous sinus
1 • GENERAL EXAMINATION _

If hard lymphadenopathy, the causes are: FNACor biopsy of LNin TB shows the
• Metastatic malignancy (e.g. from bronchial following:
carcinoma).
If tender cervical lymphadenopathy,
causes are:
the
• In 50% cases, AFB is positive.
• ~n 70~~0% cases mycobacterial CIS
IS positive.
_j
• Acute inflammation (may be secondary to • Granuloma is present in most cases.
dental sepsis, tonsillitis and mastoiditis).
• Infection of LNs itself.
Q: How to treat tuberculous lymphadenitis?
If lymphadenopathy is discrete, the
A: With standard anti-Koch for 9 months to 1 year.
causes are:
• Lymphoma.
N.B. Following anti-TB drug therapy, the LNs may
• Infectious mononucleosis.
be enlarged. It is due to hypersensitivity reac-
• Reactive hyperplasia.
tion to tuberculoprotein, released from dead
If lymphadenopathy with goitre, the mycobacteria.
cause is:
• Papillary carcinoma of thyroid with metastasis. Q: What are the atypical mycobacteria?

If lymphadenopathy is soft, fleshy, rubbery A: Atypical mycobacteria, also called nontubercu-


lous mycobacteria (NTM) or mycobacteria other
and discrete, the cause is:
than TB (MOTT). The following are atypical
• Lymphoma. mycobacteria:
If lymphadenopathy is immobile, fixed to i
• Mycobacterium scrofulaceum
skin, the cause is:
• Mycobacterium avium intracellulare complex (MAC)
• Metastatic malignancy.
• Mycobacterium kansasii
Q: In which disease, biopsy should not be done? • Mycobacterium bovis
A: Leukaemia. (With biopsy marking, leukaemia is • Mycobacterium xenopi
unlikely. Leukaemia is diagnosed by blood and • Mycobacterium chelonei
bone marrow examination. Biopsy is not done in • Mycobacterium malmoense
leukaemia, rather it is contraindicated.) • Mycobacterium marinum
• Mycobacterium [artuitum
Consistency of LNssuggests the
Atypical mycobacteria commonly affect the
following, if: children. It may also cause disseminated infection
• Rubbery: Lymphoma. in HIV, pulmonary infection (by MAC).
• Firm and matted: TB.



Hard and craggy: Malignancy.
Stony hard: Calcified LN.
Soft, cystic: Cold abscess. _j Q: How to treat atypical mycobacteria?
A: If there is localised involvement in cervical LN, per-
form surgical excision. Most organisms are resistant
to standard anti-TB drug.
Q: What investigations are done in tuberculous
Drugs used are the combination of:
lymphadenitis?
• Clarithromycin 500 mg twice daily or
A: As follows:
azithromycin 500 mg daily plus
• Complete blood count (CBC) and erythrocyte • Ethambutol 15 mg/kg plus
sedimentation rate (ESR) (high). • Rifabutin 300 mg daily.
• Chest X-ray, posterior-anterior (PA) view (to see However, standard anti-TB therapy with rifampicin,
TB in chest). ethambutol, isoniazid (INH) and pyrazinamide
• Tuberculin test. are commonly used. Injection streptomycin is also
• For confirmation, perform fine needle aspiration effective.
cytology (FNAC) or biopsy [for histology, acid-
fast bacillus (AFB), mycobacterial culture and N.B. Remember, if no response, there is possibility
sensitivity (CIS)]. ofMDR-TB.
_ SHORT CASES IN CLINICAL MEDICINE

• Cervical lymphadenopathy (examine the


Causes of localised lymphadenopathy iO dif- mouth, tonsil, teeth, face, ears and sealp).
ferent sites
• Axillary lymphadenopathy (examine the
1. Epitrochlear: This is always pathological. Its breasts, chest and upper limbs). Supraclavicu-
causes are: lar lymphadenopathy (examine the chest for
• Localised infection in hand or arm bronchial carcinoma).
(the commonest cause).
• Left supraclavicular lymphadenopathy or
• Lymphoma (usually non-Hodgkin). Virchow gland (palpate for epigastric mass,
• Sarcoidosis. carcinoma of stomach).
• Secondary syphilis. • Inguinal lymphadenopathy (examine the
• Tularaemia. lower limbs for any septic focus, genitalia and
perineum).
2. Unilateral axillary lymphadenopathy: Its causes
are:
• Local infection in upper extremity.
• Carcinoma of breast with metastasis.
• Lymphoma (non-Hodgkin commonly).
• Brucellosis.
• Cat-scratch disease.

3. Scalene LNs involvement: Its causes are:


• Lymphoma.
Cervical lymphadenopathy (lymphoma; side view)
• Metastasis from carcinoma of bron:JchS.
• Sarcoidosis.
• TB.

N.B. Remember the following points:


1. If only left supraclavicular LN is palpable, it is
called Troisier sign. Then palpate abdomen to
see any epigastric mass carcinoma of stomach.
2. For a particular area of lymphadenopathy,
examine the drainage area: Cervical lymphadenopathy (lymphoma; front view)

Generalised Lymphadenopathy

Examine systematically starting from the neck, axilla, Q: What relevant do you like to see?
inguinal and para-aortic. A: As follows:
• LNs in other parts (axillary, inguinal, para-aortic,
Presentation of a Case ,1----------, when asked to examine the neck only).
• Liver and spleen (lymphoma and leukaemia).
• There is generalised lymphadenopathy involving • Anaemia and bony tenderness (leukaemia).
cervical, supraclavicular, axillary and inguinal.
• Purpura or bruise or petechiae (haematological
• LNs are of variable size and shape, firm
malignancy).
in consistency; some are rubbery, discrete,
• Palatal petechial haemorrhage (infectious mono-
nontender, and free from underlying structure
nucleosis and leukaemia).
and overlying skin.
• Cachexia (disseminated TB or secondaries).
1 • GENERAL EXAMINATION _

Q: What are the causes of generalised lymphadenopathy? • Infectious mononucleosis.


A: (Mention the common causes of that patient, con- • SLE.
sidering the age) • Sarcoidosis, brucellosis and toxoplasmosis.
• HIV.
If the patient is young or child, the
causes are: Q: What are the causes of generalized lymphaden-
opathy with fever?
• Lymphoma (usually Hodgkin).
A: As follows:
• Acute lymphoblastic leukaemia (ALL).
• Viral infection (infectious mononucleosis and • Lymphoma.
cytomegalovirus infection). • ALLand CLL.
• Others: Disseminated TB and SLE. • Viral infections (e.g. infectious mononucleosis,
CMV infection).
If the patient is middle-aged or elderly, the
• Disseminated tuberculosis.
causes are:
• Brucellosis.
• Lymphoma. • Sarcoidosis.
• Chronic lymphatic leukaemia (CLL). • Toxoplasmosis.
• Viral infection (infectious mononucleosis and
cytomegalovirus infection). ~ Q: What investigations do you suggest in generalised
• Disseminated TB. lymphadenopathy?
• Others: Sarcoidosis, brucellosis, toxoplasmosis A: As follows:
andHN.
1. CBC, erythrocytic sedimentation rate (ESR) and
peripheral blood film (PBF) (to exclude leu-
Q: What are the causes of generalized lymphadenopa-
kaemia, increase of eosinophil in lymphoma,
thy? (theoretically).
atypical lymphocyte in infectious mononucleosis
A: As follows:
and high ESRin TB).
1. Haernatological malignancy: Lymphoma,
2. Chest X-ray (to see evidence ofTE, bilateral hilar
acute lymphoblastic leukaemia (ALL), chronic
lymphadenopathy (BHL) in lymphoma and
lymphocytic leukaemia (CLL).
lymphatic leukaemia).
2. Viral fever: Infectious mononucleosis, cytome-
3. Ultrasonography or CT scan of abdomen (to see
galovirus (CMV) infection, human immuno-
hepatomegaly, splenomegaly, para-aortic and
deficiency virus (HIV).
other lymphadenopathy).
3. Other infections: Disseminated tuberculosis,
4. Others according to the suspicion of cause:
brucellosis, toxoplasmosis.
• If lymphoma is suspected: FNAC or biopsy
4. Sarcoidosis.
(biopsy is preferable) of lymph nodes.
5. SLE.
• If leukaemia is suspected: Bone marrow study.
6. Drugs: Phenytoin or diphenylhydantoin (called
• If disseminated tuberculosis: Mantoux test
pseudo lymphoma ).
(MT), lymph node FNAC or biopsy.
Q: If the patient has generalized lymphadenopathy • If HN is suspected: HIV screening test.
with arthritis, what are the likely causes? • If SLE: Antinuclear antibody (ANA), anti-
A: As follows: double stranded DNA (anti-ds-DNA), etc.
• Juvenile idiopathic arthritis (JIA) (early age). Q: If a patient presents with generalized lymphaden-
• SLE. opathy, how will you proceed to·diagnose?
• Sarcoidosis. A: As follows:
• Viral-fever-like infectious mononucleosis, CMV
infection. 1. Detailed history of the patient: Fever, loss of
weight, sweating, itching,
• Brucellosis.
2. Physical examination: Detailed findings of
Q: What are the causes of lymphadenopathy with lymph node (e.g. site, size, consistency, tender-
splenomegaly (and/or hepatomegaly)? ness, overlying skin and underlying structure,
A: As follows: any sinus, etc). Also, hepatosplenomegaly, skin
• Lymphoma. rash, joints, etc.
• ALLor CLL. 3. Investigation (as above).
__ SHORT CASES IN CLINICAL MEDICINE

N.B. Remember the following points: • Preauricular.


• Normal LNsmay be palpable in axilla,groin, usu- • Postauricular.
ally up to 0.5 an, which are soft, rubbery, shotty. • Suboccipital.
Submandibular LNs <1 an is normal in children Five groups of axillary LNs:
and inguinal LNs <2 an is normal in adult • Anterior.
• ReactiveLNsexpand rapidly and may be painful. • Posterior.
• Localised lymphadenopathy means single • Lateral.
anatomical area ofLN involvement. • Central.
• Generalised lymphadenopathy means three • Apical.
or more anatomical noncontiguous areas of
LN involvement.
• Enlargement of supraclavicular and scalene
LNs are always pathological.

Lymphoreticular System
Lymphoreticular system includes LNs, spleen, tonsil,
adenoid, Peyer patch of ileum and Kupffer cells in liver.
Groups of cervical LNs:
• Submental.
• Submandibular.
• Anterior cervical (in front of the anterior border of
the sternomastoid).
• Posterior cervical (behind the posterior border of
the sternomastoid).
Generalised lymphadenopathy
• Supraclavicular.

Pigmentation
Usual instructions: Q: What do you think are the causes in this case?
• Perform the general examination. A; (Mention about the causes of that patient):

• Look at the patient. What do you think? • Kala-azar (history of fever).


(Pigmentation may be found). • Addison disease.
Once there is pigmentation, look carefully at the • Haernochromatosis or primary biliary cirrhosis
following points: (PBC).
• Drugs (busulphan. amiodarone and phenothiazine).
• Generalised or localised.
• Chronic debilitating illness (malignancy, CRF
• Lips, gum, inner side of mouth or buccal mucosa,
and TB).
and tongue.
• Chronic arsenic toxicity.
• Exposed parts (face, neck and extensors of arm). • Malabsorption syndrome (Whipple disease).
• Palm (especially the creases).
• Pressure areas (knee or elbow).
• Scars (especially recent).
• Nipples.
• Genitalia.

Presentation of a Case :1---------,


• There is generalised (or localised) pigmentation
involving gums, buccal mucosa, tongue and
other parts of skin (mention the location).
• Vitiligo is present (describe, if any).
Generalised pigmentation (haemochromatosis)
___________ 1 • GENERAL EXAMINATION _

4. Physical examinations:
• BP (low in Addison disease).
• Hepatosplenomegaly (kala-azar).
• Signs of CLD or haemochromatosis or PBC.
• Abdomen (scar of bilateral adrenalectomy in
Nelson syndrome).
• Evidence of other chronic illness.
5. Laboratory investigations: According to the his-
tory and suspicion of causes (kala-azar, Addison
disease and haemochromatosis).

Q: Why drug causes pigmentation?


Generalised pigmentation (systemic sclerosis) A: This is due to melanocyte stimulation or deposition
of drug in the skin.

Q: What are the causes of localised pigmentation?


A: Any cause of generalised pigmentation may be
manifested as localised pigmentation initially. The
other causes are:
• Local radiation therapy, friction or scratching.
• Melasma.
• Erythema ab igne (see page 23).

Q: What is Peutz-Ieghers syndrome?


A: It is a disorder inherited as autosomal dominant,
characterised by mucocutaneous pigmentation
Generalised pigmentation
in the lips, around the mouth, eyes, nose, buccal
mucosa (never tongue), hands and feet, associated
with polyp in CIT.
Polyp is usually hamartomatous, benign, involves
any part of CIT, commonly small bowel. It may
cause bleeding, recurrent abdominal pain, intesti-
nal obstruction and intussusception. Almost never
or very rarely it turns to malignancy.
Treatment involves polypectomy. Small bowel
resection is usually avoided (polyp may recur).

Addison disease

Q: How to investigate a patient with pigmentation?


A: As follows:
1. History offever (such as kala-azar).
2. History of drugs.
3. History suggestive of chronic disease (CLD and
CRF) and Addison disease. Perioral pigmentation in Peutz-Jeghers syndrome
_ SHORT CASES IN CLINICAL MEDICINE

2. Infections (kala-azar and TB).


3. CLD:
• Haemochromatosis Lgreenish or bronze,
less involvement of mucous membrane).
• Cirrhosis of liver (common in PBe).
4. CIT malabsorption syndrome (Whipple
disease, Peutz-Ieghers syndrome).
5. Chronic debilitating illness:
• Internal malignancy [commonly ectopic
adrenocorticotropic hormone (ACfH)
Pigmentation in palmar crease and face (Addison disease) syndrome].
• CRF.
• Any chronic illness.
6. Drugs:
• Busulphan.
• Bleomycin (diffuse brown).
• Amiodarone (violaceous or brown or
blue or slaty-grey, in exposed parts).
• Phenothiazine (slaty-grey).
• Phenytoin (melasma-like pigmentation).
• Adrenocorticotropic hormone.
• Oral contraceptive pill.
Ochronosis (ear pigmentation)
• Chloroquine (blue-grey).
• Clofazimine (red or pinkish).
• Psoralen (brown).
Causes of pigmentation • Minocydine.
A. Physiological (familial, racial, pregnancy and • Other cytotoxic drugs.
sun bath). 7. Others:
B'. Pathological: • Chronic arsenic poisoning.
1. Endocrine causes • Pellagra (necklace area and exposed part).
• Addison disease (brown or dark brown). • Systemic sclerosis.
• Cushing syndrome. • Ochronosis (mainly in the joint, nose,
• Acromegaly. ear and face).
• Nelson syndrome (bilateral adrenalec- • Argyria (slaty g«y hue due to silver I
tomy, look for abdominal scar, visual deposition).
field defect). • Porphyria cutanea tarda.
• Thyrotoxicosis. • Acanthosis nigricans.

Erythema Ab Igne
The usual instructions are: Diagnosis is erythema ab igne.
• Look here. What is the diagnosis? Q: What is the probable underlying disease? What
history do you like to take?
A: Hypothyroidism or severe pain. History of applica-
Presentation of a Case
tion of hot water bag OJ bottle to relieve pain or to
• There is reticular pattern of pigmentanon relieve from cold.
involving (mention the location).
_____ ,_. GENERAL EXAMINATION _

Complications of erythema ab igne: In long-standing


cases, premalignant keratosis and squamous cell
carcinoma.

Differential diagnosis of reticular pattern of


pigmentation:
• Erythema ab igne.
• Livedo reticularis.
• Cutis marrnorata (a physiological reaction to
Erythema ab igne (legs)
cold in children and adult).
Q: What is erythema ab igne? What are the causes?
A: It is characterised by reticular pattern of pigmentation
that occurs due to prolonged exposure to excessive
heat without production of burn. It is due to repeated
infrared heat injury. Occurs on any part of the body,
where heat is applied. Histology shows increased
amount of elastic tissue in dermis. Its causes are:
• Those who sit near the fire or those who use hot
water bottle on belly or thigh or on back due to
severe pain.
• Hypothyroidism (patient applies hot because of
coldness).
Treatment of erythema ab igne:
• 0.1% retinoic acid with 0.1 % dexamethasone
cream applied locally.
• 5% hydroquinone applied locally. Erythema ab igne (abdomen)

Gynaecomastia
The usual instructions are: Q: What else or relevants do you want to examine to
• Perform the general examination. find out causes?
• Look at the chest. What is your finding? What else A: As follows:
do you want to examine? . • Age (in young, puberty gynaecomastia).
Proceed as follows: • History of taking drugs (spironolactone, digoxin,
• Look at the chest (unilateral or bilateral breast cimetidine and oestrogen).
enlargement) . • Signs of CLD.
• Bronchial carcinoma (in the elderly) with
• Palpate the glandular tissue, first with palm, then
cachexia, dubbing with nicotine staining,
with fingers (to differentiate from lipomastia in
radiation mark in chest.
obese).
• Body habitus (tall and obese, small testes, absent
• Tender or non tender.
or less secondary sex characters in Klinefelter
• Any secretion. syndrome).
• Palpate the testes (to see atrophy, tumour or
ambiguous genitalia).
Presentation of a Case II-----~---. • Endocrine abnormality (hypopituitarism,
thyrotoxicosis and Addison disease).
• There is gynaecomastia, right or left or bilateral,
tender (or nontender). Right or left is larger. Q: What do you think are the causes?
A: Describe the causes according to age of the patient.
__ SHORT CASES IN CLINICAL MEDICINE

If the patient is young, the causes are:

• Puberty (normal physiological).


• Drugs (spironolactone, digoxin, cimetidine
and INH).
• Testicular tumour (teratoma and Leydig cell
tumour).
• Secondary testicular failure (trauma,
orchidectomy and leprosy).
• CLD (due to Wilson disease and alpha-I-
antitrypsin deficiency).
If the patient is elderly or middle-aged, the
causes are: Lipomastia with striae in Cushing syndrome
• Drugs (as above, also oestrogen therapy for
carcinoma of prostate).
• Bronchial carcinoma.
• Chronic liver disease.
• Testicular tumour.
• Secondary testicular failure (trauma, orchid-
ectomy and leprosy).

Q: How to differentiate gynaecomastia from lipomastia?


A: As follows:
• Lipomastia is due to deposition of fat in the
breast. Therefore, it is soft.
• Gynaecomastia is the enlargement of male breast
Bilateral gynaecomastia
due to glandular tissue proliferation. Hence, it is
firm, hard or rubbery.

N.B. Remember the following points:


• Unilateral gynaecomastia in the elderly is
highly suspicious of malignancy (hard, fixed
to underlying tissue, associated with skin
tethering and nipple discharge).
• Carcinoma of breast is 16 times common in
Klinefelter syndrome.

Lipomastia

Q: What are the mechanisms of gynaecomastia?


A: The four possible mechanisms are:
1. Increased oestrogen, and its causes are:
• CLD.
• Bronchial carcinoma [due to increased human
chorionic gonadotropin (HCG) production].
• Leydig cell tumour of the testis (due to
Unilateral gynaecomastia (left) increased oestrogen).
___________ , • GENERAL EXAMINATION _

• Adrenal carcinoma and congenital adrenal 4. Endocrine disease (acromegaly, thyrotoxicosis,


hyperplasia. hypothyroidism, adrenal carcinoma, Addison
• Thyrotoxicosis (due to increased conversion disease).
of oestrogen from androgen). S. Drugs (spironolactone, digoxin, cimetidine,
• Starvation (due to disturbance of liver INH, oestrogen therapy for prostate carcinoma,
function). alcohol, alkylating agent, methyldopa, mari-
2. Reduction of circulating androgens: juana, amiodarone).
• Klinefelter syndrome. 6. Chromosomal abnormalities: Klinefelter
• Primary and secondary hypogonadism. syndrome, KaJiman syndrome [anosmia, cryp-
• Testicular failure (orchitis, trauma, surgery torchidism, small testis, cleft lip or palate, low
and radiation). gonadotrophin-releasing hormone (GnRH),
3. Antagonism of androgen action (using spironol- hence low luteinizing hormone (LH) and follicle
actone and cimetidine). stimulating hormone (FSH), low testosterone].
4. Androgen-resistant state or insufficiency (in 7. Others: Testicular feminisation syndrome, star-
testicular feminisation syndrome). vation and idiopathic.

Q: What is gynaecomastia? What are the causes? Q: What are the causes of painful gynaecomastia?
A: Enlargement of male breast due to proliferation of A: As follows:
glandular components. It is due to disturbance of nor- • Puberty.
mal ratio of active androgen to oestrogen in plasma • Drugs (spironolactone and cimetidine).
or breast (normal ratio of testosterone:oestrogen is • Chronic liver disease.
100:1 and normal ratio of these in blood is 300:1). Q: Why gynaecomastia in CLD?
Imbalance occurs either due to less testosterone A: Probable mechanisms are:
production or action or increased oestrogen • Excess oestrogen due to increased conversion
synthesis or both. from androgens and altered oestrogen metabo-
lism by liver.
Causes of gynaecomastia:
• Drug (spironolactone therapy for ascites).
A. Physiological
1. Pubertal (SO% cases) may be unilateral due to Q: Why alcohol causes gynaecomastia?
transient ina-ease in oestradiol level. Resolves A: Alcohol may cause gynaecomastia by causing CLD
spontaneously in 6-18 months. or by damaging Leydig cells of testis without CLD.
2. Senile (40% or more) due to increased oestro- Q: Bow will you investigate a case of gynaecomastia?
gen from conversion of androgen to oestrogen A: Depending on the age. If the patient is young, it
(also decline of Leydig cell in testis). may be due to puberty. No need of further investi-
3. Newborn (due to transplacental transfer of gation. Otherwise as follows:
maternal oestrogen). 1. History of drug intake.
2. Chest X-ray (to exclude bronchial carcinoma).
B. Pathological 3. Liver function tests (in CLD).
1. CLD (common in alcoholic liver disease), 4. Endocrine evaluation:
hepatocellular carcinoma (HCC) (HCG- • In hypogonadism, serum testosterone, LH,
secreting). FSH, oestradiol, prolactin and HCG should
2. Bronchial carcinoma (S% case, HCG-sea-eting). be estimated.
3. Hypogonadism:
• If Ll-land FSHare high, but testosterone is low,
• Primary testicular diseases (testicular then the cause is primary testicular failure.
tumour, teratoma and Leydig cell tumour). • If both LH and testosterone are low, the
• Testicular failure (trauma, orchidectomy, cause is increased oestrogen production from
radiation and leprosy, TB, mumps orchi- tumour of testis.
tis, haemochromatosis and Kli'nefelter • If both LH and testosterone are high, there is
syndrome). androgen-resistant state or gonadotrophin-
• Secondary testicular failure (hypopitu- secreting tumour.
itarism, hyperprolactinaemia, Kallman • 24 h urinary 17-ketosteroid or serum and
syndrome). androstenedione should be done.
_ SHORT CASES IN CLINICAL MEDICINE

• If plasma ~-HCG (human chorionic gonado- Q: How will you treat gynaecomastia?
trophin) is increased, it indicates testicular A: As follows:
tumour. It is also increased in bronchial • Explanation and reassurance to the patient,
carcinoma. especially in younger age. Usually improve or
S. Other investigations: According to suspicion disappears spontaneously.
of causes (chromosomal analysis in Klinefelter • Treatment of primary cause. If any offending drug
syndrome). is responsible, it should be stopped.
• If severe and progressive or suspicion of
malignancy, mastectomy should be done.

Erythema Nodosum
The usual instructions are: • Purpura.
• Cellulitis.
• Look at the leg. What is your finding? What else do
• Erythema induratum.
you want to examine?
• Others: Nodular panniculitis, meningococcal
Look carefully the following points (look and feel): septicaemia, vasculitis ISLE and polyarteritis
• Swelling (multiple, nodular, variable in size and nodosa (PAN)].
shape).
Q: What is erythema nodosum? What are the histo-
• Colour (red-to-bluish or pigmented). logical findings?
• Palpate (Tender nodule: Look at the face, patient A: It is characterised by nonsuppurative, painful,
winces, if pain). palpable, erythematous nodular lesion in the
skin due to vasculitis in dermis and subcutane-
ous fat. Usually, it is associated with fever and
Presentation of a Case
arthralgia, and is common in shin below the knee.
• There are multiple nodular, tender lesions of Nodules may be 2-6 em in diameter, occur in crops,
variable size and shape; some are red and some over 2 weeks, then resolve slowly over months
.are pigmented in the right or left or both shins. leaving a bruise staining in the skin. It never
ulcerates, may be recurrent and common in adult
female.
Differential diagnosis includes:
Microscopy: Panniculitis (inflammatory reaction in
• Drug rash. fat), infiltration of lymphocytes, histiocytes. multi-
• Erythema nodosum. nucleated giant cells and eosinophils, immune-
• Erythema multiforme. complex deposition in dermal vessels.

Erythema nodosum
1 • GENERAL EXAMINATION _

Q: In erythema nodosum. what history should be • LN biopsy (sarcoidosis, lymphoma and TB).
taken? What else do you like to examine? • Inflammatory bowel disease [barium enema
A: As follows: (double-contrast), sigmoidoscopy or colonos-
1. History of: copy, barium follow through].
• Drugs (see below). • Other investigations: According to suspicion of
• Fever cause.
• Sore throat or tonsillitis (streptococcal Q: How to treat erythema nodosum?
tonsillitis). A: As follows:
• Other infection (TB, leprosy and 1. Treatment of primary cause (e.g. penicillin, if
histoplasmosis) . streptococcal infection). It is self-limiting, may
• Arthritis or arthralgia or other features of resolve in 3-6 weeks.
sarcoidosis. 2. Other treatment:
• CIT symptoms like diarrhoea, dysentery, pain
• Rest.
abdomen, etc. (inflammatory bowel disease). • Nonsteroidal anti-inflammatory drugs
2. On examination: (NSAIDs) (indomethacin).
• Throat (tonsillitis). • In severe cases, corticosteroid should be given.
• LN (sarcoidosis, TB, lymphoma and fungal • Empirically, a short course of potassium iodide
infection). ill a dose of 400-900 mg daily may be helpful.
• Liver, spleen (sarcoidosis and lymphoma).
• Skin changes (lupus pernio in sarcoidosis, Q: What is erythema induratum (Bazin disease)?
anaesthetic patch or erythematous or nodular A: Erythema induratum of Bazin type is a nodular
lesion in leprosy). vasculitis (which is a multifactorial syndrome of
• Ulceration in mouth, genitalia (Behcer lobular panniculitis), related to tuberculous origin.
syndrome). It is one of the sequelae of immunologic reactions
• Evidences of rheumatic fever. against antigenic components of Mycobacterium
tuberculosis, which spreads through blood. It rep-
Q: What are the causes of erythema nodosum? resents a delayed-type hypersensitivity reaction to
A: As follows: tubercle bacillus. Recently, hepatitis C virus has
• Sarcoidosis (usually in acute). been suspected as a cause.
• Streptococcal beta-haemolyticus infection (throat). It is more common in women, aged 20-30 years,
• Primary pulmonaryTB. mostly occurring in lower extremities, usually in
• Drugs (sulphonamide, penicillin, oestrogen- calves, may also be in shins. Differential diagnoses
containing oral contraceptive pill, salicylates, bar- are chilblain, erythema nodosum, erythema no do-
biturates, sulphonylureas, bromides and iodides). sum leprosum. pancreatic panniculitis, lupus pan-
• Inflammatory bowel disease (Crohn disease, niculitis, etc. Diagnosis is based on routine blood
ulcerative colitis). count, ESR, PCR and biopsy of the involved tissue.
• Fungal infections (histoplasmosis and coccidi- Treated with antituberculous therapy. Steroid may be
oidomycosis, and is common in the USA). indicated. Potassium iodide is sometimes needed for
• Protozoal (toxoplasmosis). local application. Complication of untreated or inade-
• Leprosy [erythema nodosum leprosum (ENL)]. quately treated erythema induratum involves persistent
• Idiopathic (up to 50% cases). ulceration. If treated properly, the prognosis is good.
• Others: Brucellosis, rickettsial and mycoplasma
infection, viral infection, pregnancy, lymphoma, Q: What are the differences between erythema no do-
cat-scratch disease, Behcet syndrome and SLE. sum and erythema induratum?
A: As follows:
Q: What investigations are done in erythema nodosum?
A: As follows: Erythema Erythema
• CBC, ESR, PBF (leucocytosis in streptococcal Points nodosum induratum
infection, high ESR in TB).
1. Duration Short Long
• Antistreptolysin 0 (ASO) titre, throat swab. for
2. Site Shin Calf
CIS, blood for CIS (in streptococcal infection).
3. Occurrence Lesions occur Lesions occur serially
• Chest X-ray (TB and sarcoidosis).
simultaneously in crops
• MT and sputum for APB (in case ofTB).
_ SHORT CASES IN CLINICAL MEDICINE

4. Ulceration Absent Present


S. Pain More Less
6. Scarring Absent Present
7. Cause Multiple causes UsuallyTB
8. Histology Septal panniculitis Lobular panniculitis

~.~ Erythema nodosum is not vasculitic in ongin:


rather it is a septal panniculitis, where the inflam-
mation is mostly in the septa of fat, with little or
Erythema induratum
no vascul itis.

Leprosy

Q: What investigations are done to diagnose


Presentationof a Case:11----------,
leprosy?
• Multiple nodules of variable size and shape A: Slit skin smear for APB staining, also skin biopsy.
involving right or left or both ear lobule, also In tuberculoid type, epithelioid granuloma may be
face and nose. Skin is thick. found. In lepromatous leprosy, Mycobacterium leprae
may be found in skin macrophage (also Schwann
cells and perineurium).
Differential diagnoses are:
• Post-kala-azar dermal leishmaniasis (PKDL). Q: What are the types of classification of leprosy?
• Lepromatous leprosy. A: There are five types:
• Sarcoidosis. • Tuberculoid leprosy CIT).
• Drug rash. • Borderline tuberculoid (BT).
• Acne rosacea. • Borderline leprosy (BB).
• Dermatomyositis. • Borderline lepromatous leprosy (BL).
• Lepromatous leprosy (LL).
Presentationof a Case:211-------~ Also, classified into two types:
• Paucibacillary: Skin smear for Mycobacterium
• Multiple hypopigmented patches of variable size leprae bacilli is negative or few 'IT and BT.
and shape in the trunk, upper abdomen and • Multibacillary: Skin smear for Mycobacterium
back. No loss of sensation. leprae bacilli is positive BT, all BB, BLand LL.

Differential diagnoses are:


• fungal infection.
• Tinea versicolor.
• PKDL (early stage).
• Drug reaction.
• SLE.
• Lepromatous leprosy.
• Psoriasis.

Q: What else do you like to see?


A: Sensation in patch (loss in leprosy). Also, see nerve
thickening (radial in humerus, ulnar in elbow,
median in wrist, common peroneal in back of knee,
posterior tibial and sural at ankle, and great auricu-
lar in posterior triangle in neck). Leprosy
1 • GENERAL EXAMINATION _

Leprosy (hypopigmented patch and nodule in front)

Erythema nodosum leprosum

Q: How to treat leprosy?


A: As follows:
• Paucibacillary (three to five skin lesions, skin smear
negative or few, tuberculoid and BT): Rifampicin
600 mg monthly (supervised) plus dapsone
100 mg daily (self-administered) for 6 months.
• Multibacillary (more than five skin lesions, skin
smear positive, BT, all BB, BLand LL): Rifampicin
600 mg and clofazimine 300 mg monthly (super-
vised) plus dapsone 100 mg and clofazimine
PKDL 50 mg daily (self-administered) for 12 months.
• Paucibacillary single lesion: Ofloxacin 400 mg
plus rifampicin 600 mgplus minocycline 100 mg,
in single dose.

Q: What is erythema nodosum leprosum (ENL)?


How it is treated?
A: It is a type 2 lepra reaction, which is due to immune
complex deposition (type 3 hypersensitivity reaction),
occurs in BL and LLpatient who produce antibodies
and have a high antigen load. Characterised by fever,
arthralgia and crops of small pink painful nodules on
the face and limbs. Iritis and episderitis are common.
Other signs are neuritis, orchitis, myositis, bone pain,
Leprosy (hypopigmented patch and nodule on the back)
lymphadenitis and arthritis.
ENL may be the first manifestation of leprosy,
50% in lepromatous and 25% in BL, either during
the course of the disease or more commonly in the
second year of treatment. ENL may continue inter-
mittently for several years.
Treatment:
• Antileprosy therapy must be continued.
• In mild cases, analgesics (aspirin 600 mg, 6 hourly).
• In severe cases, thalidomide (100 mg, 6 hourly).
When symptoms improve, reduce the dose
slowly over weeks or months, maintenance dose
Leprosy (nodule in dorsum and toe) is 50-100 mg daily. It is a potent teratogenic
_ SHORT CASES IN CLINICAL MEDICINE

drug (avoid during pregnancy). If thalidomide is Type 1 occurs in 30% borderline patients (BT,
not available, prednisolone (20-40 mg) should BB and BL) due to delayed hypersensitivity reac-
be given, and reduce the dose over 1-6 months. tions. It occurs spontaneously or precipitated by
Chloroquine can also be used. Increase the dose treatment. Nerve function is lost rapidly, foot
of clofazimine (300 mg daily) for few weeks. drop may occur overnight. Skin lesions become
It will reduce the reaction and help to reduce the erythematous and peripheral nerves are painful.
dose of prednisolone. Reversal of reactions may occur spontaneously
after starting treatment and also after completion
Q: What is lepra reaction? of multidrug therapy.
A: It is defined as episodes of inflammation in the
Treatment:
pre-existing lesion ofIeprosy. It may be the first mani-
festation of the disease. Leprareaction may be insidious • In mild case, aspirin 600 mg, 6 hourly.
or rapid, destroying the affected tissue within hours. It • In severe case, prednisolone 40-60 mg daily;
is of two types: Type 1 and type 2 (see above). reduce the dose to 5 mgJday each month.

Superior Vena Caval Obstruction

The usual instructions are:


• Look at the patient. What are your findings? What
else do you want to examine?
(Patient may be dyspnoeic or may have stridor.)

See the following points:


1. Face (oedernatous or puffy, red, plethoric, suffused
and cyanosed).
2. Eyes:
• Periorbital oedema.
• Red eyes, congested conjunctiva (bloodshot
eyes).
• Chemosis (conjunctival oedema).
3. Neck: Swollen and neck veins are engorged and SVC obstruction (engorged veins in chest)
nonpulsatile (in CCF, engorged and pulsatile).
4. Visible tortuous and dilated veins in chest wall and
abdomen (see the flow is downwards).
5. Upper limb may be oedernatous with prominent
engorged veins.
6. Then examine the following points to find out the
causes of superior vena cava (SVC) obstruction:
• LNs for supraclavicular or cervical lymphoma
or metastasis.
• Thyroid may be enlarged (Jook for retrosternal
thyroid, trachea shifted).
• Clubbing (bronchial carcinoma).
• Radiation mark in chest (bronchial carcinoma).
• Chest (to see bronchial carcinoma).
• Signs of Horner syndrome.
• Ophthalmoscopy (dilated vessels, haemorrhage,
exudate, rarely papilloedema). SVC obstruction (engorged veins in arm)
__ ~ 1.:__. GENERAL EXAMINATION _

• Symptoms are aggravated on lying down or bend-


ing forward (indicates mediastinal involvement).
• The patient may have stridor [tracheal compres-
sion), hoarseness of voice (recurrent laryngeal
nerve involvement), Horner syndrome (cervical
sympathetic chain involvement).
Q: What are the causes of death in SVC obstruction?
A: Death is due to:
• Respiratory obstruction.
• Intracranial haemorrhage.
Q: Tell one investigation that will help the diagnosis
of SVC obstruction.
A: Chest X-ray (bronchial carcinoma and lymphoma).
SVC obstruction (engorged veins in abdomen)
Q: What investigations would youlike to do?
A: As follows:
Presentation of a Case: If-------_ • Chest X-ray.
• Sputum for malignant cells.
• The face is puffy, plethoric and cyanosed. • cr or MRI of chest contrast venography or
• There is periorbital oedema; eyes are congested magnetic resonance venography (MRV).
with chemosis. • IfLN, FNAC or biopsy.
• The neck veins are engorged and nonpulsatile. • Others (according to suspicion of causes): Bron-
• There are also engorged veins in chest wall and choscopy and mediastinoscopy, and occasionally
abdomen, and the flow is downwards. thoracotomy may be needed.
• Both the arms are oedematous, clubbing with
Q: How will you treat the case?
nicotine staining.
• Undersurface of tongue shows multiple venous A: Treatment is according to cause:
angiomata. • The commonest cause is bronchial carcinoma:
Radiotherapy in non-small cell carcinoma and
Diagnosis is SVC obstruction. chemotherapy for small cell carcinoma.
• If lymphoma, treat accordingly (usually
Q: What relevant do you like to examine in this patient?
chemotherapy) .
A: As follows: • To relieve oedema, intravenous frusemide, head
• Chest (bronchial carcinoma). should be raised. Dexamethasone may be used.
• LNs (lymphoma and metastasis). • To relieve severe obstruction, balloon angioplasry
• Liver and spleen (lymphoma). and expandable metallic stent may be used
• Clubbing, nicotine stain (bronchial carcinoma).
(placed in SVC) as palliative measure.
• Thyroid (to see retrostemal thyroid).
Q: What are the causes of SVC obstructioni Causes of SVCobstruction
A: Describe the causes considering the age:
1. Bronchial carcinoma (the commonest, in
• In the elderly or middle-aged, the causes are
75%).
bronchial carcinoma and lymphoma.
2. Lymphoma (early age, also in the elderly).
• Tnyoung or early age, the cause is lymphoma.
3. Other causes:
Q: What are the presentations of SVC obstruction?
• Any tumour in mediastinum, such as
A: The patient may complain of: thymoma, germ cell tumour, metastasis
• Breathlessness, cough, hoarseness of voice and to mediastinum.
dysphagia.
• Retrosternal thyroid.
• Flushing, red, puffy and oedematous face'.
• Chronic fibrotic mediastinitis (which
• Headache (early morning), which becomes
may be idiopathic or secondary to
severe with coughing. May be syncope, dizziness
tuberculosis, histoplasmosis, pyogenic
or blackout, stupor, seizure (due to increased
infection, radiation, drugs like
intracranial pressure).
methysergide used in migraine).
_ SHORT CASES IN CLINICAL MEDICINE

N.B. Remember the following points:


• Giant aneurysm of the aortic arch.
• SVCobstruction of recent onset is likely to be due
• Carcinoma of oesophagus. to malignancy.
• Rarelythrombosis, invasion bymalignancy • SVC obstruction of long-standing is due to
and chronic constrictive pericarditis. nonmalignant origin.

Hirsutism
The usual instructions are: Q: What other history would you like to take?
A: As follows:
• Look at the patient. What is your diagnosis? What
• Family history.
else do you want to see?
• If Cushing syndrome is suspected: History of
prolonged intake of steroid.
Presentation of a Case ~I--------,
• Age of onset of hirsutism: In younger and early
(a Female Patient) age, common cause is PCOS. So history of amen-
orrhoea, weight gain, infertility, etc. should be
• There is hirsutism. taken. In elderly or postmenopausal.
• Patient is obese with Cushingoid face (if any).
• Weight gain.
• Drug history: Steroids, androgen, phenytoin,
cyclosporine, minoxidil.
• Rate of progression of hirsutism.
• Thinning of scalp hair.
Q: What else do you want to see in this case of
hirsutism?
A: As follows:
1. Hair in. other parts of the body, chest and back
(increased); hair in midline below the umbili-
cus to groin (ina-eased); excess hair in the upper
and lower limbs.
2. Evidence of virilisation:
• Male baldness (frontal baldness).
• Male body habitus.
Hirsutism in Cushing syndrome • Deepening of voice.
• Others (c1itoromegaly, atrophy of breast,
Q: What are the causes in this case?
male pattern of pubic hair, acne, greasy skin).
A: Describe according to your finding considering
3. Abdominal mass (PCOS, ovarian tumour,
the age:
adrenal carcinoma).
1. If Cushingoid face, diagnosis is Cushing 4. History of drugs causing hirsutism (see below).
syndrome. 5. Menstrual history (amenorrhoea in PCOS).
2. If patient is obese and young, diagnosis is poly-
cystic ovarian syndrome (PCOS). Q: What is the difference between hirsutism and
hypertrichosis?
3. Other causes:
A: As follows:
• Idiopathic (in most cases).
• Hirsutism is male pattern of hair growth in
• Familial, racial and postmenopausal.
women due to excess of androgen.
• Drugs (see below).
• Hypertrichosis is generalised excess hair growth
• Late-onset congenital adrenal hyperplasia.
in any sex, which is nonandrogenic in origin.
• Adrenal causes (carcinoma or androgen-
secreting adrenal tumour). Q: What is hirsutism? What are the causes?
• Ovarian cause (androgen-secreting ovarian A: Hirsutism is excessgrowth of terminal hair in women
tumour). as male pattern due to excessive secretion of androgen.
_________ '_·_G_E __
NERAL EXAMINATION _

Hair growth in beard or chin or moustache. Also


in breast, chest, axilla, abdominal midline, pubic
and thigh area; all are related to androgen. Increase
in hair growth indicates excessive androgen, either
adrenal or ovarian in origin.

Causes of hirsutism:
• Hirsutism without virilisation.
• Hirsutism with virilisation.
Hirsutism without virilisation:
• Idiopathic (in most cases). Hypertrichosis
• Familial.
• In congenital adrenal hyperplasia, serum
• Drugs (steroid, phenytoin, cyclosporine, andro-
17-hydroxyprogesterone is high, and also there is
gen, minoxidil and progesterone).
high pregnanetriol and ACfH.
• Others: PCOS (in mild cases, hirsutism is more
• Other tests such as Cf scan, MRI and Japaroscopy
and virilisation is less), acromegaly and porphyria
may be done.
cutanea tarda.
Another method of investigation
Hirsutism with virilisation:
• Ovarian causes: PCOS (severe case), androgen- Rapid onset of hirsutism

secreting ovarian tumour, arrhenoblastoma.


If yes Ifno
• Adrenal causes: Late onset of congenital adrenal
hyperplasia, Cushing syndrome, adrenal !
Cause more likely
~
History of drugs
carcinoma or androgen-secreting adrenal tumour. ovarian or adrenal Family history

Q: How will you investigate a case of hirsutism? !


Perform USG and
If no ~

Non-neoplastic
A: As follows: other investigations androgen secretion
• Good history (see above). according to
t
site of lesion
• History of drugs (see above).
• Family history.
(CT, MRI, laparoscopy) J
PC OS CAH
l Idiopathic

After exclusion of these, causes may be ovarian or


adrenal: N.B. A good history regarding hirsutism is essential:
• Blood for testosterone, LH, FSH and prolactin. If • If the onset is shortly after menarche, tumour
testosterone is high (twice the normal) associated is unlikely.
with low LH and FSH, it is unlikely to be PCOS • If it occurs in childhood, more chance of
and idiopathic hirsutism. underlying disease.
Next, USC of abdomen to see ovarian or adrenal • If menstruation is regular, more likely to be
mass. constitutional rather than tumour or other
A. In case of ovarian origin, perform the following test: pathology.
• If LH is high and FSH is normal or high (ratio • Greater the menstrual abnormality (irregular
ofLH:FSH is 2 or 3), suggest PCOS. or cessation), more likely there is a serious
• Sex hormone binding globulin (SHBG) (high). disease (ovarian or adrenal).
• Androgens (high, but testosterone is normal • Rapid onset, prepubertal or late onset is
or low). suggestive of underlying disease (ovarian or
• Other tests such as Cf scan, MRI and adrenal).
laparoscopy may be done. • Increased libido and signs of virilisation
indicates increased androgen.
B. In case of adrenal origin, do the following tests:
• If adrenal carcinoma or adenoma, urinary Q: How to treat hirsutism?
17-ketosteroid is high. A: As follows:
• Dexamethasone suppression test may be done 1. Treatment of primary cause. If due to drug, it
(for failure of suppression). should be stopped.
_ SHORT CASES IN CLINICAL MEDICINE

2. Local therapy:
Polycystic Ovarian Syndrome
• Plucking, bleaching, depilatory cream, shaving,
electrolysis, epilation. Usual instructions:
• Topical eflornithine cream applied locally for • Perform the general examination, or,
6 months is also effective. • Look at the patient. What are your findings?
3. Systemic therapy (use in severe cases): (Usually hirsutism, obesity in a young girl)
• Cyproteroneacetate (antiandrogen) 50-1 00 mg
daily for 1-14 days of each cycle. In women
Presentation of a Case
of childbearing age, contraception is essential.
(a Female Patient)
• Oestrogen (in oral contraceptive) is helpful in
idiopathic or PCOS. It reduces free androgens • This young patient is obese and there is hirsutism.
by increasing SHBG, where this is low.
• Other antiandrogens: Spironolactone, fluta-
mide or finasteride are also helpful. My diagnosis is: In this young lady with obesity and
hirsutism, more likely it is a case of PCOS.

Q: What are the differential diagnoses?


A: As follows:
1. PCOS.
2. Congenital adrenal hyperplasia (late onset. In
such case, there is raised serum 17u-hydroxy-
progesterone) .
3. Cushing syndrome (other features are present).
4. Virilizing tumour of the adrenal or ovary (rapid
onset of signs of virilization, very high serum
testosterone) .
5. Obesity.
Hirsutism (front view)
6. Hypothyroidism.
7. Drugs (e.g. danazol, androgenic progestins).

Q: What is polycystic ovarian syndrome (PCOS)?


A: It is syndrome in which there are multiple cysts in
the ovary and hyperandrogenemia, characterized by:
• Amenorrhoea or oligornenorrhoea.
• Obesity.
• Hirsutism.
• Infertility (due to anovulation).
• Virilisation (in severe cases).

PCOS (originally called severe form of Stein-Leven-


thal syndrome) is associated with increased LH due
Hirsutism (side view) to abnormal ity of pulsatile GnRH secretion (there
is chronic anovulation without specific underlying
disease of adrenal, pituitary or ovary).

Q: What are the diagnostic criteria of PCOS?


A: Two of the following three features are required for
PCOS to be diagnosed:
• Menstrual irregularities (oligomenorrhoea or
amenorrhoea) .
• Clinical or biochemical evidence of androgen
excess.
• Multiple cysts in the ovaries (most readily
Hirsutism in peos detected by transvaginal ultrasonography).
________ ~1~. GENERAL EXAMINATION _

Q: What investigations should be done to diagnose • Clomiphene 50-100 mg/day, from days 2-6
PCOS? of cycle. Dexamethasone 0.5 mg at bedtime
A: As follows: with clomiphene may increase the likelihood
1. USG as first investigation (it shows thickened of ovulation by suppressingACTH
capsule, multiple 3-5 mm cyst and hyperecho- • If no response to clomiphene, metforrnin
genic stroma). may be added, 500 mg three times daily. It
2. Biochemical tests: may enhance ovulation.
• Serum testosterone (usually high). • Prednisolone in reverse circadian rhythm
• LH (increased). (2.5 mg in the morning and 5 mg at bedtime)
• FSH (normal or low, in a ratio of LH:FSH may suppress pituitary ACTH, upon which
> 2 or 3). adrenal androgen partly depend. With this
• Androgens (androstenedione and dehydro- therapy, regular ovulatory cycle often ensue.
epiandrosterone are increased). 4. For menstrual disturbance:
• SHBG: Decreased.
• Metformin 500 mg three times daily improves
• Prolactin (slightly increased, rarely greater
menstrual cycle and ovulation. Also may
than 1500 mUlL).
improve hirsutism and obesity.
• Oestrogen (oestradiol is usually normal).
• Cyclical low-dose oestrogen or progesterone
3. To exclude other cause, investigations should be administration.
done according to suspicion:
• Patien t who does not desire pregnancy should
• Serum 17 a OH progesterone [high in late-
get medroxyprogesterone 10 mg daily orally
onset congenital adrenal hyperplasia (CAH)].
for first 10 days of each month. It ensures
• Cf or MRI of adrenal (in suspected tumour).
regular shedding of endometrium.
• Dexamethasone suppression test.
5. Wedge resection or laser surgery of ovary or laparos-
• A short ACfH stimulation test with measure-
copy ovarian drilling may be done in some cases.
ment of 17a-hydroxyprogesterone is done to
6. For obesity: Metformin may be used.
diagnose CAH.

Q: How to treat PCOS? N.B. Adrenal androgen is under control of ACTH


A: As follows: and ovarian androgen is under control of LH.
1. General measures:
• Reduction of weight. Q: What are the complications in PCOS?
• Regular exercise and diet control. A: As follows:

2. For hirsutism, as described above. • Hyperinsulinaernia and insulin resistance.


3. For fertility: • Glucose intolerance.
• Metformin: May improve ovulation and • Type 2 DM.
achieve conception. • Hypertension, dyslipidemia and cardiovascular risk

Leg Ulcer
Usual instructions are: • Any slough or necrotic area or gangrene.
• Oozing (pus or serous fluid) or dry area.
• Look at the leg; or, examine the leg; or, perform
• Any blister (rupture).
general examination.
• Surrounding area (healthy or pigmented).
Look at the following points carefully:
• Temperature (cold or warm).
• Site of ulcer (one or both legs or feet or tip of toes).
• Pulse (absent or bounding).
• Single or multiple.
• Sensation (light touch, pain, vibration and
• Margin (irregular, raised, undermined, everted and
position sense).
punched out).
_ SHORT CASES IN CLINICAL MEDICINE

• Infection (TB, pyogenic infection, leprosy,


leishmaniasis) .
• Pyoderma gangrenosum.
If the patient is of younger or of early age,
the causes are:

• As above (except atherosclerosis).


• Collagen disease (SLE, RA, Felty syndrome). I
• Haematological diseases include sickle
cell anaemia, thalassaemia, hereditary
spherocytosis.

leg ulcer in SlE


Q: What history do you like to take in leg ulcer?
A: History of trauma, smoking, DM, hypertension,
evidence of infection, collagen diseases.

Q: What investigations should be done in leg ulcer?


A: As follows:
• Hb%, total count (TC), differential count (DC)
and £SR.
• Blood sugar.
• Lipid profile (in atherosclerosis).
• Pus (if any), for CIS, AFB, Leishman-Donovani
(LD) bodies.
• Doppler USG of lower limb vessels.
• Biopsy.
Leg ulcer (enlarged)
• X-ray of leg (calcification in artery may be seen).
• Arteriography.
• Others: According to suspicion of causes, e.g.
Presentation of a Case ~f--------- anti-nuclear antibody (ANA), cold agglutinin
and rheumatoid arthritis (RA) test.
• There is ulcer (ulcers) involving one or both shin
of legs or feet, with a clear margin; some parts are Q: How to treat leg ulcer?
necrosed or gangrenous, with oozing. A: As follows:
• Leg is cold, pulse is absent, sensation is normal. • Smoking should be stopped.
• Joint abnormality, trophic ulcers and gangrene • Control of DM, hypertension, obesity (if any).
(mention, if any). • Treatment of primary cause.
• Local care (cleaning, removal of necrotic tissue
and dressings).
Q: What do you think about the causes ofleg ulcer?
• Antibiotic, if any infection.
A: Mention the causes according to the age of the patient:
• Consult with chiropodist.
• Low-dose aspirin.
If the patient is middle-aged or elderly, the • In some cases, surgery (balloon dilatation) and if
causes are: necessary, amputation.
• Education to the patient: Avoid walking bare
• Trauma. foot, avoid tight shoes, use appropriate footwear,
• Atherosclerosis (in old age).
crutch may be used to avoid weight bearing,
• DM. cutting toe nails carefully, look for blisters in
• Peripheral neuropathy.
foot, which should be cleared.
___ ~ 1.;_. GENERAL EXAMINATION _

Causes of leg ulcer


• Traumatic.
• Infection (pyogenic, TB, leprosy and leishma-
niasis).
• Arterial (ischaemic, occurs more on lateral
side called Buerger disease) causes are
atherosclerosis, peripheral vascular disease
and arterial occlusion, and arterial ulcer is
usually tender, punched-out ulcer on leg, cold
skin, loss of hair, and absent or reduced pulse.
• Venous ulcer occurs usually due to varicose vein.
Varicose ulcer
(Venous ulcer is usually around malleoli or in
medial side; the leg is oedematous. There may
be wet gangrene, pigmentation and eczema.)
• Neuropathic ulcer (DM, tabes dorsalis,syringomy-
elia, leprosy, polyneuropathy due to any cause).
• Collagen disease (SLE, RA, Felty syndrome,
cryoglobulinaemia, polyarteritis nodosa).
• Haematological (sicklecellanaemia, thalassaemia,
hereditary spherocytosis, paroxysmal nocturnal
haemogiobinuria, thrombotic thrombocytopae-
nic purpura and maaogiobulinaemia).
• Neoplastic (squamous cell carcinoma, basal cell
carcinoma, lymphoma and Kaposi sarcoma).
• Pyoderma gangrenosum. Ulcer in foot

Pyoderma Gangrenosum

Instruction to the examinee:


• Look at the foot or leg; or, perform general
examination.

·~. . -.. ,.

X .
. ,
~"
.~ , '.!

Pyoderma gangrenosum (skin peeled off,


~ -.
• '.,> ,. . ~ _"-
..'
,..

severe advanced stage)

Diagnosis is pyoderma gangrenosum.

Pyoderma gangrenosum (early stage) Q: What are the differential diagnoses?


A: As follows:
Presentation of a Case ~I--------'
• Trauma.
• Venous ulcer.
• There is a large necrotic ulcer with ragged bluish- • Infection (TB, pyogenic infection, leprosy,
red, gangrenous overhanging margin, purulent leishmaniasis ).
surface with pustules and plaque. • Collagen disease (SLE, RA,Felty syndrome,
cryoglobulinaemia) .
_ SHORT CASES IN CLINICAL MEDICINE

• Vasculitis (polyarteritis nodosa, Behcetsyndrorne, Q: What is the treatment of pyoderma gangrenosum?


Wegener granulomatosis). A: As follows:
• Haematological (sickle cell anaemia). 1. Treatment of underlying diseases.
• Neoplastic (squamous cell carcinoma, basal cell 2. General measures:
carcinoma, cutaneous lymphoma). • Control of infection.
• Local dressing
Q: Ask one question or what history do you like to
• Analgesic for relief of pain.
take?
3. Topical:
A: History of diarrhoea or bloody diarrhoea (sugges-
• Highly potent corticosteroid may be sufficient
tive of inflammatory bowel disease).
in some patients. Triamcinolone may be
Q: What is pyoderma gangrenosum? What are the injected into the ulcer edge (alone or with
causes? systemic treatment).
A: It is a noninfective, necrotising ulceration with • Tacrolimus may be given.
clear-cut bluish-red overhanging edge, usually in 4. Systemic:
the shin. The lesion starts as a blister or pustule, • Oral prednisolone in high dose isneeded in most
breaks down centrally, expands rapidly to an ulcer patients. In some patients, methylprednisolone
with indurated or undermined purplish or pustu- 1 gm intravenous (IV) for 3 days may be given
lar edge. initially for quick remission.
It occurs commonly in legs, but may be anywhere • Minocycline 100 mg/day may help to reduce
in the body surface, may be single or multiple. It the dose of steroid.
is common in adults (25-54 years). Diagnosis is • Immunosuppressive agents like ciclosporin,
usually clinical, and biopsy shows nonspecific tacrolimus or azathioprine may be used to
findings. reduce steroid dependence or in resistant cases.
• Anti-TNf (J., (infliximab, etanercept) may be
Q: What is the pathogenesis of pyoderma gangrenosum?
used if others fail.
A: Actual pathogenesis is unclear; there is depres-
• Dapsone may be used in milder cases.
sion of immune system. Failure of macrophage to
• Other drug therapy: Colchicine, clofazimine,
respond to tissue 'injury or to clear the obnoxious
cyclophosphamide, mycophenolate mofetil,
agent is another factor.
etc. These may help in few patients.
Q: What arethe types of pyoderma gangrenosum?
A: There are four types: Causes of pyoderma gangrenosum
• Ulcerative.
• Inflammatory bowel disease: Ulcerative colitis
• Pustular.
(common), less in Crohn disease.
• Bullous.
• Vegetative. • RA.
• Polycythaemia rubra vera.
Q: What investigations will you do? • Chronic granulocytic leukaemia (also in acute
A: As follows: granulocytic leukaemia).
1. CBC, ESR.
• Multiplemyelomaandotherparaproteinaemias
2. Blood sugar. (especially, 19Atype).
3. Biopsy from the lesion.
• Myelofibrosis.
4. Other investigations according to suspicion of
• Wegener granulomatosis.
cause:

_j
• For IBD: Barium enema, colonoscopy. • Chronic active hepatitis.
• For collagen disease: ANA, anti-dsDNA, • HTVinfection.
anti phospholipid antibody. • Idiopathic in >20% cases.
• for vasculitis: Perinuclear antineutrophil
cytoplasmic antibodies (pANCA), N.B. In ulcerative colitis, pyoderma gangrenosum
cytoplasmic anti neutrophil cytoplasmic indicates severe disease. It may precede the onset
antibodies (cANCA). of inflammatory bowel disease. Healing paral-
• For myeloma: Protein electrophoresis, bone lels with cure of ulcerative colitis and colectomy
marrow, etc. allows rapid healing.
__________ 1_. GENERAL EXAMINATION _

Diabetic Foot
Usual instruction:
• Look at the foot of this diabetic patient. Or, do the
general examination of this diabetic patient.
Look carefully the following points:
Inspection:
• Ulcer (site, single, multiple, oozing, gangrene and
the surrounding area).
• Look at the tip of all toes, sole and spaces between
the toes.
• If gangrene is present, see whether it is dry or
wet, and the demarcation between healthy and
unhealthy skin. Diabetic ulcer in dorsum (with infarction and cellulitis)

• Colour change of skin and hair loss.


• Amputation of toe (may be).
• Pigmented scar, small round plaques with raised
border lying in a linear fashion over shin (diabetic
dermopathy).
• Necrobiosis lipoidica diabeticorum (central yellow
with raised margin, telangiectasia at the margin).
• Joints (Charcot joint or joint swelling).
• Thigh wasting, atrophy or hypertrophy (insulin
therapy).
Palpation: Ask the patient whether the foot is sore. Diabetic ulcer in sole (without gangrene)
• Temperature (warm or cold, and compare with
other foot).
• Pulse (arteria dorsalis pedis and posterior tibial), if
reduced, absent or bounding.
• Sensation, if reduced or absent (see light, touch,
pain, joint sense).
• Reflex,ifreduced or absent, see also plantarresponse.
• Vibration and position sense (may be lost due to the
posterior column lesion called diabetic pseudo tabes).
• In some cases, test for proximal myopathy (there
may be diabetic amyotrophy, which shows
asymmetrical wasting of thigh). Diabetic ulcer in dorsum (with gangrene and oozing)

Presentation of a Case

• There is an ulcer (ulcers) involving dorsum of right


or left foot, toes (if any) (Mention if there is any
gangrene and point out whether, it is dry or wet).
• There is also pigmented area surrounding the
ulcer extending up to. " (mention the area) with
loss of hair and shine.
• Foot is cold.
• Pulse is absent.
• Loss of sensation in foot, leg up to ...
(mention the area).
Diabetic ulcer in sole (with gangrene)
~ SHORT CASES IN CLINICAL MEDICINE

With this finding, my differential diagnoses are: • Loss of pulsation.


• Traumatic. • Foot is cold, shiny; and there is loss of hair.
• DM (diabetic foot).
• Atherosclerosis. Q: How to prevent diabetic foot ulcer?
• Infective (TB, leprosy and leishmaniasis). A: Along with good control of diabetes, ensure the
• Buerger disease. following:
• Vasculitis [collagen disease: SLE,polyarteritis nodosa 1. Advice to a]) diabetic patients:
(PAN) and RAJ. • Inspect and wash feet every day.
Q: What are the causes of ulcer in DM? • Cut toe nails regularly, very carefully.
A: As follows: • If skin is dry, use moisturizer.
• Ischaemia. • Wear suitable good-fitting shoes.
• Neuropathy. • Check footwear for foreign bodies.
• Combined ischaemia and neuropathy. • Change socks or stockings every day.
• Secondary infection. • Avoid walking barefoot.
Q: What is the pathology of ischaemic ulcer? • Cover minor cuts with sterile dressings.
A: As follows: • Do not burst blisters.
• Usually microangiopathy. • Avoid over-the-counter corn or callus remedies.
• Associated atherosclerosis of large and medium 2. Additional advice to high-risk patients:
vessels. • Do not attempt corn removal.
N.B. If the ulcer is painful, the cause is vasculitis • Avoid high and low temperatures.
and if it is painless, the cause is neuropathic. 3. Involve a podiatrist in the care of the patient.
4. Special footwear should be used in Charcot neu-
Q: What are the findings, if the ulcer is due to roarthropathy.
neuropathy?
A: As follows: Q: How to treat a case of diabetic ulcer?
• Ulcer is painless and mostly plantar. A: As follows:
• Area is warm, dry and pink. • Good control of DM.
• Pulse: Present and may be bounding. • Smoking should be stopped.
• Sensory function: Reduced or absent. Vibration • Local dressing and removal of dead tissue.
sense: Reduced or absent. • Antibiotic to control secondary infection.
• Reflexes: Reduced or absent, plantar: Absent or
• Consult with chiropodist.
equivocal.
• Surgery may be required (removal of callus,
amputation or angioplasty).
Q: What are the findings, if the ulcer is due to
• Patient education: Avoid barefoot, tight shoes,
ischaemia?
careful cutting of nails, avoid weight bearing.
A: As follows:
• Other measures: As in foot ulcer.
• Ulcer is painful, over heels and toes. • Liaison should be maintained among physician,
• Area is cold, shiny, atrophied and loss of hair. chiropodist and surgeon.
• Pulse: Absent or reduced.
• Sensory function: Normal.
Q: What are the features of diabetic foot?
• Reflexes: Normal.
A: As follows:
Q: What are the findings, if the ulcer is due to com- 1. Neuropathic: Ulcer, sepsis, abscess, osteo-
bined neuropathy and ischaemia? myelitis, digital gangrene, Charcot joint.
A: In such case, the findings are: 2. Ischaemic: Ulcer, sepsis, gangrene.
• Ulcer is on foot and callosities and pressure
points. Q: What are the differences between ischaemic ulcer
• Loss of arch of foot. and neuropathic ulcer?
• Sensory loss of all modalities in stocking pattern. A: As follows:
1 • GENERAL EXAMINATION _

N.B. Doppler ultrasound study of the lower limb


Ischaemic ulcer
Features Neuropathic ulcer (arterial)
vesselsand femoral angiography may be needed
to see the localized area of occlusion. Either
Symptoms No pain. Paraesthesia Pain, history of bypass surgery or angioplasty may be needed.
or tingling may be claudication.
present.

Foot High arched, clawing Dependant rubor,


of toes, no trophic trophic changes are
change present (atrophy Causes of Neuropathic Ulcer
and loss of hair)
• Leprosy.
Area Warm, dry and pink Cold, shiny • OM.
Pulse Bounding Absent or reduced • Polyneuropathy (due to any cause).

Sensation Reduced or absent Normal


• Tabes dorsalis.
• Syringomyelia.
Ulcer Painless and mostly Painful, over heels

_j
plantar and toes • Amyloidosis.
• Porphyria.
Reflex Reduced or absent, Normal
equivocal plantar response • Progressive sensory neuropathy.

Diabetic Amyotrophy
The usual instructions are:
• Look at the thigh. What are your findings?

Presentation of a Case :1---------.

• There is wasting in right or left thigh (patient is


cachexic or emaciated).

Q: What is the likely underlying disease?


A: Diabetes mellitus.

Q: What is diabetic amyotrophy? Diabetic amyotrophy


A: A type of motor neuropathy in diabetic patient,
Q: Where is the site of lesion or cause of diabetic
characterised by asymmetrical wasting of muscles,
usually involving the quadriceps (also in upper amyotrophy?
limb). Affected area may be tender, commonly A: It is thought to involve acute infarction of lower
accompanied by severe pain in anterior aspect of motor neuron of lumbosacral plexus.
thigh. Q: In which type of diabetes is it common?
It may be the first presentation of OM. The patient A: It is more common in type 2 OM. It is common in
is occasionally cachexic (neuropathic cachexia) elderly.
and extremely ill, unable to get out of bed. Q: How to treat diabetic amyotrophy?
Hyperaesthesia or paraesthesia is common. There A: As follows:
is reduction of muscle power, tone, loss of knee • Good control of OM.
jerk with occasional extensor plantar response on • Intensive insulin therapy.
the affected side. Cerebrospinal fluid (CSF) protein • For pain amitriptyline, imipramine or carbamaz-
is increased. epine. Aldose reductase inhibitor may help.
_. SHORT CASES IN CLINICAL MEDICINE

Q: What is the prognosis? • Diabetic amyotrophy.


A: Prognosis is good, usually recovers, but may take a • Old poliomyelitis.
long time (over months to 2 years). • Arthritis or trauma.
Q: What are the causes of unilateral wasting of leg? • Cerebral palsy.
A: As follows: • Disc prolapse.

lipodystrophy of Thigh
The usual instructions are:
• Look at the thigh. What are your findings?
There may be two findings in the thigh, together called
lipodystrophy:
• Lipoatrophy.
• Lipohypertrophy.

Presentation of a Case
(Lipoatrophyor Lipohypertrophy of Thigh)

• There is atrophy or wasting or hypertrophy of


muscle of thigh (right or left) with multiple needle
puncture marks.

Diagnosis is lipoatrophy or lipohypertrophy (mention Diabetic lipohypertrophy


which one).

Diabetic lipoatrophy

Diabetic bullae

Q: What is the underlying diagnosis?


A: The patient is diabetic, receiving insulin injection.

Q: What is lipoatrophy?
A: It is the localised atrophy of subcutaneous fat due
to repeated injection of unpurified animal insulin
caused by immunogenic component of insulin.
Treated by injection of pure human insulin at the
margin and centre of the affected area, which results
in restoration of normal contour. It is rare now due to
Diabetic dermopathy less use of animal insulin.
Q: What is lipohypertrophy?
A: It is the localised hypertrophy of subcutaneous fat
due to repeated injection of purified insulin at the



Acanthosis nigricans.
Xanthelasma.
Granuloma annulare.
-
same site. It is caused by continued lipid synthesis at • Lipoatrophyand lipohypertrophy.
the affected site, and is treated by changing the site • Diabetic bullae.
of injection. • Others: Vitiligo, xanthoma and peripheral
anhydrosis.
Q: What are the skin changes in DM?
A: As follows:
• Necrobiosis lipoidica diabeticorum. Causes of Lipoatrophy of Skin
• Diabetic dermopathy (atrophic pigmented patch
• DM (receiving insulin injection).
in skin, precipitated by trauma associated with
neuropathy). • Localised scleroderma or morphea. ~
• Chronic relapsing panniculitis.
• Ulcer or gangrene.
• Mesangiocapillary glomerulonephritis.
• Secondary infections (boil, carbuncle and
candidiasis ). • HIV.

Necrobiosis Lipoidica Diabeticorum


The usual instructions are: It is two to three times common in females, usually
• Look at the leg. What are your findings? in young adult or early middle life. The cause is
unknown, may be due to small vessel damage.
Presentation of a Case Histology shows necrosis of collagen, infiltration
with epithelioid cells, giant cells with glycogen and
• There is sharply demarcated, atrophied skin or lipid deposition.
plaque in the skin of shin with shiny surface and
waxy yellow centre, brownish-red margin with
surrounding telangiectasia.

Diagnosis is necrobiosis lipoidica diabeticorum.


Q: What are the differential diagnoses?
A: As follows:
• Localised scleroderma.
• Sarcoidosis.
• Polyarteritis nodosa.
• Granuloma annulare.
Necrobiosis lipoidica diabeticorum
• Pyoderma gangrenosum.
Treatment:
Q: What is necrobiosis lipoidica diabeticorum?
A: Plaque-like lesion with central yellowish area • Good control of DM.
surrounded by brownish border on the anterior • Dressing, local application or injection of steroid
surface of leg. May be shiny, atrophic skin with may be helpful.
telangiectasia. Fibrosis with scarring from previous • Psoralen + UVA(PUVA) may be helpful.
ulceration may be seen. • Surgery with skin grafting.
This is a rare fmding in DM «1 %), may be in pre- • Low-dose aspirin, occasionally pentoxifylline may
diabetic and in 50%, without diabetes. But 85% cases be helpful.
with necrobiosis lipoidica diabeticorum develop DM. It may take long time to heal.

Unilateral or Bilateral Leg Swelling


Usual instructions: Look carefully at the following points:
.;. Look at the lower limbs. Or, examine the lower Inspection:
limbs.
• Swelling, extent of swelling (right or left).
_. SHORT CASES IN CLINICAL MEDICINE

• Veins (engorged .or varicose veins).


• Scratch mark.
• Pigmentation.
• Purpura.
• Hair change.
• Trophic ulcer.
• Look at the back of the leg.
• See associated joint swelling, dubbing and
cyanosis.

Palpation:
• Temperature (warm or cold).
• Pulse (arteria dorsalis pedis, posterior tibial and
Unilateral lymphoedema (left)
popliteal artery).
• Oedema (pitting or nonpitting).
• Calf tenderness and localised swelling (DVT,
ruptured Baker cyst).
• Hornans sign (may be dangerous, with risk of
pulmonary embolism).
• Associated LNs (popliteal. inguinal).

Presentation of a Case :1-----------,

• Right or left leg is swell en extending up to ...


(mention the site).
• Skin is rough, scaly and thick; less of lusture and
hair. There is excoriation or scratch mark.
• Nonpitting oedema (weedy texture).
• Local temperature is normal or cold or warm.
Bilateral leg swelling
There is no lymphadenopathy.
• Arterial pulse is normal.

Diagnosis is lymphoedema, which may be due te:


• Filariasis.
• Trauma.
• Recurrent lymphangitis or cellulitis.
• Neoplasm.
• Surgery.
• Radiation.
• Primary lymphoedema.

Bilateral lymphoedema

Q: What are the differential diagnoses of unilateral


leg swelling?
A: As follows:
• Lymphoedema.
• DVT.
Lymphoedema of left lower limb • Angioneurotic oedema.
1 • GENERAL EXAMINATION _

• Ruptured Baker cyst. Q: Why there is nonpitting oedema?


• Chronic venous insufficiency. A: Normally, small amount of albumin filtered
• Traumatic. through the capillaries is absorbed through lym-
• Compartment syndrome following trauma or phatics. In lymphatic obstruction, water and solutes
surgery (requires immediate fasciotomy). are reabsorbed into the capillaries, but the protein
remains. Fibrosis occurs in the interstitial space and
N.B. Unilateral swelling in arm may occur follow- the area becomes hard or thick.
ing mastectomy or radiotherapy.
Q: What are the causes of acute unilateral leg swelling?
Q: What are the causes of bilateral leg swelling? A: As follows:
A: As follows: • Deep venous thrombosis.
• Oedema due to any cause (CCF, nephrotic syn- • Ruptured Baker cyst.
drome, cirrhosis of the liver, hypoproteinaemia). • Cellulitis.
• Myxoedema. • Trauma.
• Some cases of lymphoedema. • Angie-oedema.
• Acromegaly.
• Inferior vena caval (IVC) obstruction (dilated tortu- Causes of Lymphoedema
ous veins in the lower limbs with upwards flow).
1. Primary:
• Iliac vein thrombosis.
• Pregnancy. • Secondary to agenesis or hypoplasia.
• Drugs (calcium channel blockers, such as • Hereditary (Milroy disease).
nifedipine or amlodipine). • Associated with Turner syndrome, Noonan
syndrome and yellow nail syndrome.
Q: How will you investigate a patient with bilateral
pedal oedema? 2. Secondary:
A: As follows: • Recurrent lymphangitis or cellulitis.
1. Routine: • Filariasis.
• Urine R/E. • Trauma.
• Complete blood count. • Tuberculosis.
• Serum creatinine. • Neoplasm.
• Serum total protein; A:C ratio. • Surgery (in the arm, it may be due~to
2. Other according to suspicion: mastectomy).
• If CCF: ECG, chest X-ray, echocardiogram. • Radiation.
• If nephrotic syndrome: 24-h urinary protein,
• Bum.
renal biopsy if needed.
• If myxoederna: TSH, FTs' FT4•
• Ultrasonogram of whole abdomen. Q: What investigations do you suggest in
• Duplex study of both lower limbs. lymphoedema?
A: As follows:
Q: How will you investigate a patient with unilateral
pedal oedema? • Hb%, TC, DC, ESR (eosinophil may be high in
A: As follows: filariasis) .
1. Routine: • Blood film to see filaria (usually at night, for
Wuchereria bancrofti and Brugia malayi). Also,
• Urine RIB.
found in chylous fluid and hydrocoele fluid.
• CBC.
2. Other according to suspicion: • Provocation test (by giving diethylcarbamazine,
• Colour Doppler ultrasonogram of the affected 50 mg orally; see the blood film for filaria after
limb (may be helpful to see DVT or venous 30 minutes).
insufficiency) . • Compliment fixation test (CFT) or indirect
fluorescent antibody test (IFAT)for filaria.
• For lymphoedema (see below).
• Filarial antigen detection.
Q: What are the causes of nonpitting oedema? • Lymphoscintigraphy.
A: As follows: • Lymphangiogram.
• Lymphoedema due to any cause. • Other investigations to exclude other disease:
• Myxoedema. USC of whole abdomen, Doppler ultrasound
_ SHORT CASES IN CLINICAL MEDICINE

scans of lower limbs (to exclude deep venous the skin of face, limbs and mucosa of the larynx can
thrombosis), cr scan or MRI of abdomen, chest occur. Laryngeal oedema may be life threatening.
X-ray and MT. Recurrent abdominal pain may occur. Nonheredi-
tary form may occur in lymphoproliferative disorder.
Q: How to treat lymphoedema?
A: No curative treatment. Mainly supportive to reduce
Diagnosis:
the swelling and control discomfort.
• Serum C2 and C4 (both low).
1. Intermittent elevation of the extremity by
• Measurement of Cl esterase inhibitor (low).
placing pillows, mainly during sleeping.
2. Compression treatments:
Treatment:
• Elastic sleeves or stockings.
• Bandages: The extremity is wrapped tightly to 1. In acute attack:
remove lymph out of the limb. • Epinephrine should be used in life-threatening
• Massaging the affected part either by hand or reactions.
by pneumatic compression can be useful. • Cl inhibitor concentrate should be given, if
• Exercises. available. A newer medicine called ecallantide
3. Surgical treatment: May be used to remove may be used instead.
excess fluid and tissue in severe cases. • Fresh frozen plasma containing Cl inhibitor
4. Secondary infections of skin and tissues associ- may be used.
ated with lymphoedema should be treated with 2. To prevent recurrent attack, danazol or stano-
antibiotics. zolol may be given. It stimulates the liver, which
5. If filariasis is suspected, diethylcarbamazine is synthesises the enzyme. (This drug may cause
used. fluid retention, menstrual irregularity, obesity
and androgenic effect.)
Q: What is angio-oedema?
A: It is the swelling of dermis and subcutaneous tissue.
There are two types: hereditary and acquired. N.B. Remember the following:
• Antihistamines and other treatments used
Q: What is hereditary angio-oedema? for angioedema are of limited benefit in
A: it is a disorder inherited as autosomal dominant due hereditary angioedema.
to Ct-esterase inhibitor deficiency. There is involve- • Helicobaaer pvlori can trigger abdominal
ment of the blood vessels. Family history is present. attacks. Antibiotics to treat H. pylori will
Attack is usually recurrent; episodes of oedema in decrease abdominal attacks.

Deep Venous Thrombosis (DVT)


Usual instructions are:
Presentation of a Case :1--------,
• Examine the lower limbs.
• Perform general examination. • The leg is swollen up to the knee, red or cyanotic,
warm with pitting oedema.
• Tenderness in the calf muscles.
• Superficial veins are prominent and tender.
• Homan sign is positive.

Q: What are the differential diagnoses?


A: As follows:
• Cellulitis.
• DVT.
• Ruptured Baker cyst.
Deep venous thrombosis (left lower limb) • Post-traumatic and calf haematoma.
__ ~ 1_. GENERAL EXAMINATION _

Q: What is Homans sign? Q: What history should be taken in a patient with DVf?
A: Dorsiflexion of foot causes pain in calf muscle A: A.sfollows:
(unreliable sign). This test is dangerous because of • Onset: Acute, chronic or recurrent.
risk of pulmonary embolism. It is also positive in • History of immobilization: Prolonged bed rest,
ruptured Baker cyst, trauma and inflammation in trauma, surgery, CVD, etc.
calf muscle. • History of air travel.
• Drug history (OCP).
Q: What are the common sites of DVE
• History of any primary disease like polycythaemia
A: Calf muscle veins, popliteal, femoral and iliac veins
rubra vera, malignancy, SLE,nephrotic syndrome,
(swelling up to thigh, femoral or iliac vein throm-
etc.
bosis).
• Family history.
Q: What are the causes of DVE
Q: How to diagnose DVT?
A: Causes are (Virchow's law):
A: As follows:
• Stasis. • Doppler USC of lower limb vessels (to note
• Vascular damage. velocity of blood flow in the vein).
• Hypercoagulability. • Byimaging deep vein with B-mode USC (duplex),
Stasis (which may be due to): thrombosis can be seen.
• Prolonged bed rest or immobilisation [after • D-dimer (high).
• Venography (confirmatory).
acute myocardial infarction (MI), cerebrovascular
accident (CVA) and fracture). N.R Elevated D-dimer level has a limited diagnostic
• Postoperative (prostatectomy, hip or pelvic valuesinceitmaybehighinotherconditionslike
surgery, and lower limbs surgery). pulmonary embolism, MI, pneumonia, sepsis,
• Pregnancy and puerperium. etc. However, a low D-dimer «500 ngfmL)
Vascular damage (which may be due to): may be useful in excluding DVT in low-risk
patients, and further tests will be unnecessary.
• Surgery (commonly after prostatectomy, and
In a high-risk patient, D-dimer is less impor-
abdominal and pelvic surgery).
tant; and other investigations should be done
• Trauma. regardless of the D-dimer value.
• V-aricosevein.
• Buerger disease. Q: What other investigations should be done to find
• Raynaud disease. out cause?
A: As follows:
Hypercoagulability (which may be due to): • CBC, ESR (may be evidence of polycythaemia
• Oral contraceptive pill. rubra vera, SLE).
• SLE (antiphospholipid syndrome). • Antithrombin IlJ, protein C or S.
• Nephrotic syndrome. • Factor V Leiden level.
• Haernatological disease (polycythaemia rubra • ANA, anti-ds-DNA, anti phospholipid antibody.
vera, paroxysmal nocturnal haemoglobinuria, • Homocysteine level.
myelofibrosis, essential thrombocythaemia, DJC). • X-ray chest
• Internal malignancy (pancreas, lung, ovary and • USC of whole abdomen
stomach). Q: What are the complications of DVf?
• Anticoagulant deficiency (antithrombin III, A: As follows:
proteins C and S, factor II or V Leiden). • Pulmonary embolism (commonly from
• Septicaemia. thrombosis in ileo-fernoral vein, less commonly'
Q: What are the causes of recurrent DVT? from below-knee thrombosis).
A: As follows: • Venous gangrene.
• Antiphospholipid antibody syndrome, SLE. • Postphlebitic syndrome (chronic DVr results in
• Protein S, C, antithrombin III deficiency. permanently swollen limb with ulceration).
• Factor V Leiden. Q: How to treat DVr?
• Polycythaemia rubra vera. A: As follows:
• Oral contraceptive pill use. 1. General treatment:
• Malignancy. • Bed rest. .
_ SHORT CASES IN CLINICAL MEDICINE

• Use of elastic stockings from midfoot to • Less inhibition of platelet (standard heparin can
below knee (in calf thrombosis). cause thrombocytopaenia).
• Relief of pain by analgesic.
Q: What is the mode of action of heparin?
• Intermittent elevation of foot during day and
A: Heparin acts by potentiating the activity of anti-
night (above the heart level).
thrombin, which inhibits procoagulant enzyme
• Mobilisation slowly, when the patient is fully
activity of factors Ila, VIla, lXa, Xa and XIa.
anticoagulated.
2. Anticoagulation: All patients with above-knee Q: What are the complications of long-term heparin
thrombosis must be anticoagulated (as there is use?
more chance of pulmonary embolism). Patients A: As follows:
with below-knee DVT should be anticoagulated • Osteoporosis.
for 6 weeks. • Thrombocytopaenia (if heparin is used for more
3. If anticoagulation is contraindicated or with than 7-10 days).
recurrent pulmonary embolism, rve filter • Hyperkalaemia (if heparin is used for more than
should be given. 7 days).
Anticoagulant is used as follows: Q: What is the antidote of heparin?
• Standard unfractionated heparin, initially a loa- A: Protamine sulphate.
ding dose of 5000 IU intravenously, followed
Q: What is the mode of action of warfarin?
by continuous infusion of 1000-2000 IU/hour
A: Inhibit vitamin K-dependent factors (11,VO,IXand X).
(20 lU/kg/hour), with infusion pump. APTT
monitoring is essential CAPrI' should be 1.5-2.5 Q: What is the antidote of warfarin?
times of control). Heparin should be given at A: Vitamin K.
least for 5 days.
• Low-molecular-weight heparin such as enoxaparin N.B. After warfarin therapy, if INR is high, above
is preferred (1.5 mg/kg daily subcutaneously). the desired, withhold or reduce the dose. If
• Oral anticoagulant (warfarin) should be started INR is more than 8, then start with vitamin K,
with heparin (oral anticoagulant may take 48 h 2 mg N slowly or 5 mg orally. In case of
for its full effect). Then heparin may be stopped. severe bleeding, concentrate factor contain-
ing II, Ill, IX and X, or fresh frozen plasma
N.B. Warfarin is given 10 mg daily for 2 days. It is may be given.
followed by a maintenance dose of 3-9 mg,
Q: What are thrombophlebitis and
according to international normalized ratio
phlebothrombosis?
(INR) (target INR should be 2.5). If there is
only single episode of thromboembolism, it A: Thrombophlebitis (superficial vein thrombosis):
Inflammation involving superficial veins (after intra-
should be continued for at least 3 months.
venous fluid or in varicose vein). It is characterised by:
However, if a definite cause is found and
treated, then 4-6 weeks of warfarin may be • Pain-the main feature.
sufficient. If no cause is found or permanent • Increased local temperature.
risk factors are present, it should be continued • Skin is inflamed.
for 6 months. In recurrent cases, it should be • Vein, prominent superficial vein, hard and tender.
continued for longer time, even life long. • Thrombosis is attached to the vein. Hence, there
is less chance of pulmonary embolism than
Q: Why low-molecular-weight heparin is preferred? phlebothrombosis.
A: It has the following advantages: Phlebothrombosis (DVI'):Thrombosis in deep veins
• Does not affect APlT (no need of frequent APTT is noninflammatory in nature. It is characterised by:
testing). • Pain and swelling of the leg.
• Long half-life (may be given in single dose daily). • It is in the deeper vein, mostly asymptomatic.
• High bioavailability. • No inflammation.
• Greater activity against factor Xa. • There is more chance of pulmonary embolism
• Can be given in a fixed dose. than thrombophlebitis.
Q: How to prevent DVI'?
A: As follows:
• Early mobilisation.
• Leg exercise.
• Elastic stockings.
• Low-dose aspirin.
• Low-molecular-weight heparin should be given
following surgery or acute MI (enoxaparin 40 mg
SIC daily).
• Avoid oral contraceptive pill.
• Treat the primary cause. Deep venous thrombosis (left)

Cellulitis and Erysipelas


Cellulitis
Usual instructions:
• Examine the lower limbs.
• Look at here. What is your diagnosis?

Presentation of a Case
• There is erythematous and darkly pigmented
area involving the dorsum of left foot extending
up to the lower part of leg.
• There are multiple vesicles or blisters or crusts.
• Local temperature is raised and the part is also Cellulitis-side of leg (with blister)
tender.
Q: What is cellulitis? What are the causative organisms?
A: It is the acute spreading inflammation of skin
Diagnosis is cellulitis. and subcutaneous tissue with local pain, swelling
and erythema, It may be secondary to infection in
surgery, burn or fungal infection in feet or toe.

Causes: Commonly due to the infection of group A


Streptococcus pyogenes; also because of Staphylococcus au-
reus. In immunosuppressed or diabetic patients, Gram-
negative organisms or anaerobes should be suspected.
Skin on the face or lower legs (shin and ankle) are
commonly affected by this infection; though cellulitis
can occur inany part of the body. It appears as a swollen,
red area of the skin, which is hot and tender, and may
spread rapidly. It left untreated, the spreading infection
may rapidly turn into life-threatening condition.
Treatment: Antibiotic (phenoxymethyl penicillin,
erythromycin, fludoxacillin or cephalexin). In severe
cases, intravenous therapy followed by oral therapy
is given. If there is recurrent cellulitis, low-dose
antibiotic prophylaxis with phenoxymethylpenicillin
Cellulitis (dorsum) 500 mg twice daily should be given.
~ SHORT CASES IN CLINICAL MEDICINE

Q: What are the differential diagnoses of cellulitis? Clinical features:


A: As follows:
• Erysipelas is characterised by rapidly enlarging
• DVT. erythematous skin lesion that commonly involves
• Trauma. the face, but may also occur on limbs and trunk.
• Acute arthritis.
The lesion is red, swollen, warm, indurated and
• Ruptured Baker cyst.
painful. The margin is raised and has a sharply
demarcated border (differentiating it from other
Erysipelas skin infection).
• In severe cases, there may be vesicles, blisters and
Q: What is erysipelas? What are the causes and risk even skin necrosis. There may be marked oedema,
factors? with eyes closed.
A: It is an acute, superficial form of cellulitis, which • LN draining the area is enlarged and tender.
occurs classically in the cheek, but may occur in Occasionally, a red streak extending to the LN
other parts of the skin as well, caused by Streptococcus can be seen.
~-haemolyticus. The infection involves dermis and • There are usually systemic features like high fever,
lymphatics, and is more superficial subcutaneous shaking, chills, fatigue, arthralgia, etc.
infection than cellulitis.
N.B. In erysipelas, pustule and gangrene are usually
Causes: Most common cause is group Astreptococcus;
absent. It heals without scar.
less common are group G, C and B streptococci.
Staphylococcal infection is rare.
Differential diagnoses:
Risk factors: Elderly, infants, children, diabetes
mellitus, alcoholism, immunodeficiency, lymphat- • Cellulitis
ic obstruction, etc. • Contact dermatitis.
• Angioedema.
• Herpes zoster.
• Diffuse inflammatory carcinoma of the breast.
Treatment: Antibiotics (such as penicillin, clinda-
rnycin. erythromycin)-oral or intravenous.

Complications:
• Septicaemia leading to septic arthritis.
• Glomerulonephritis.
• Necrotizing fasciitis.
• Recurrent infection.
Erysipelas • Lymphatic damage.

Peripheral Vascular Disease

Usual instructions are: • Skin is shiny with loss of hairs.


• Examine the leg or perform the general examination • Local temperature is reduced.
of this patient. • There is an ulcer at the tip of great toe and sole
of right foot.
• Pulse is absent in arteria dorsalis pedis and
Presentation of a Case reduced in posterior tibial.
• No femoral bruit .
• The right leg and foot is pale, and the toes are
bluish red. • No sensory abnormality.
• Reduced capillary return (press the nail and note
• There is wasting of right leg with prominent
the return of normal red colour, which is slow).
veins.
_______ ,_. GENERAL EXAMINATION _

My diagnosis is peripheral vascular disease, more


likely Buerger disease (in young patient). If the patient
is elderly, likely diagnosis is peripheral vascular disease
due to atherosclerosis.

. ~
.,
8

.,
c.
o
::l

Buerger disease (gangrene of the tip of toes)

Q: Ask one question to the patient.


A: Ask about smoking habit, with number of sticks
and duration.

Q: Ask another question.


A: History of intermittent claudication; also claudica-
tion distance.

Q: What bedside test should be done in Buergerdisease?


A: Buerger test: Elevate the limb at 45 -there is pallor
0

and patient may complain of pain. Then, the patient


sits with the feet hanging lower to the ground for
2-3 minutes. There is cyanotic hue in the affected
foot (Buerger sign).

Q: What are the differential diagnoses?


A: As follows:
• Raynaud disease: More in female, and commonly
involves the upper limb. History such as exposure
Raynaud disease (gangrene of the tip of fingers)
to cold will produce typical colour change
(see page 386). Buerger disease itself can cause
Raynaud phenomenon.
• Atherosclerosis: Common in the elderly. There
may be bruit over femoral artery. It may be present
in young, if associated with hypercholesterolaemia.
Look for corneal arcus, tendon xanthoma, pal mar
xanthoma and xanthelasma.
• Vasculitis due to any cause.
• Takayasu disease.
Q: What are the risk factors for peripheral vascular
disease?
A: Mostly, these are the risk factor for atherosclerosis:
• Family history of heart attacks or strokes.
• Age >50 years.
Buerger disease (amputation of great toe) • Smoking.
• Diabetes mellitus. • Antiplatelet agent (aspirin 75 mg daily or
• Hypertension. clopidogrel 75 mg daily)
• Obesity. • Pentoxifylline: It improves blood flow by
• Sedentary lifestyle. decreasing the viscosity of blood and making
• Dyslipidaemia. red blood cells more flexible.
• Cilostazol: It prevents clumping of platelets.
Q: What investigation should be done for peripheral
It also helps dilate the blood vessels,
vascular disease?
encouraging the flow of blood.
A: As follows:
• CBC, ESR. 3. Surgical:
• CRP. • Percutaneous transluminal angioplasty,
• Urine R/M/E (haematuria) stenting, endarterectomy or bypass graft.
• Chest X-ray (pulmonary infiltrate is Wegener
• Sometimes sympathectomy may be needed.
granulomatosis, PAN).
• X-ray of the involved area: May show calcification • Amputation may be necessary in case of
of the arteries. gangrene.
• Serum lipid profile.
• pANCA and cANCA, ANA, anti-ds-DNA.
• Doppler study of the lower limb vessels. Buerger Disease
• Arteriography may be needed in some cases. It is an inflammatory, obliterative, arterial disease with
Q: How to treat the peripheral vascular disease? proliferative lesion in the medium and small arteries
A: As follows: and veins of limbs, giving rise to claudication or rest
1. General measures: Smoking should be stopped, pain in fingers or toes. Superficial migratory thrombo-
reduction of weight, if obese, regular exercise, phlebitis is common. Wrist and ankle pulse are absent,
limb should be kept warm but avoid local heat, but brachial and popliteal pulse are characteristically
local care of foot, use of appropriate footwear, etc. palpable. Arteriography shows narrowing or occlusion
2. Medical treatment: of artery below the knee, relatively healthy above the
knee. It is common in young male, 20-40 years of age
• Treatment of any risk factors like diabetes and in moderate to heavy smokers.
mellitus, hypertension, dyslipidaemia.

Laurence-Moon-Bardet-Biedl Syndrome
Usual instruction:

• Perform the general examination.

Polydactyly (six toes)

Obesity and short stature Gynaecomastia with polydactyly (six fingers)


1 • GENERAL EXAMINATION _

Diagnosis is Laurence-Moon-Bardet-Biedl syndrome.

Q: What else do you like to see?


A: Signs of hypogonadism and fundoscopy to see
retinitis pigmentosa.

Q: What is Laurence-Moon-Bardet-Biedl syndrome?


A: It is an autosomal recessive disorder, characterised by:

• Obesity.
• Short stature (not always).
• Mental retardation.
Polydactyly (six fingers)
• Polydactyly.
• Hypogonadism (gynaecomastia and small testis).
Presentation of a Case
• Retinitis pigmentosa.
• The patient is obese, looks short. Hypogonadism, mental retardation and polydactyly
• There is polydactyly (one extra finger in each are less frequently found in females. Renal structural
hand and one extra toe in each foot). and functional abnormalities are very common.
Interstitial nephritis may lead to renal failure.

Generalized Oedema
Q: How can you differentiate oedema of cardiac, renal
Presentation of a Case :1--------. and liver disease clinically?
A: As follows:
• There is generalized oedema (also called ana-
• CCF: Oedema is usually dependant, mostly in
sarca) involving the whole body, which is pitting.
the leg. Other features are engorged and pulsatile
neck vein, enlarged tender liver. Evidence of
cardiac disease is usually present.
• Nephrotic syndrome: Oedema usually starts from
the face or periorbital, then descents, later becomes
generalized. Urine shows massive proteinuria.
• AGN: Oedema is periorbital, associated with
scanty, froathy, smokey micturition.
• Chronic kidney disease: Previous history of renal
disease or hypertension, DM are usually present.
Other features of CKD are present.
• Cirrhosis of liver: Usually there is ascites, later in
advanced stage there may be generalized oedema.

Q: What is oedema? What are the types?


A: It may be defined as excessive accumulation of fluid in
Generalized oedema in nephrotic syndrome
the interstitial space. It may be pitting and nonpitting,
My differential diagnoses are (causes of generalized Q: What are the causes of pitting oedema?
oedema):
A: As follows:
• Nephrotic syndrome.
• CCF.
• Hypoproteinaemia due to any cause
• Nephrotic syndrome
• Hypothyroidism (nonpitting). • Hypoproteinaernia due to any other cause (protein
• CKD (in advanced stage). loosing enteropathy or less protein intake).
• Decompensated cirrhosis of liver • Deep venous thrombosis.
(advanced stage). • Compression of large veins by tumour or lymph
• Congestive heart failure (advanced stage). nodes.
_ SHORT CASES IN CLINICAL MEDICINE

• Chronic venous insufficiency (varicose vein). • Surgical emphysema


• Drugs (calcium channel blockers [e.g. nifedipine, • Orbital cellulitis
amlodipine]. some NSAlDs). • Malignant exophthalmos (in Graves disease)
• Idiopathic (also called 'fluid retention syndrome', • Dermatomyositis
common in women).
Q: How will you investigate a patient with generalized
Q: What are the causes of nonpitting limb oedema?
c: oedema?
o A: As follows:
'0 A: Investigation should be according to history, physi-
'" 1. Myxoedema.
.S cal finding and suspicion of cause .
S 2. Chronic lymphatic obstruction or lymphoedema
'" • Nephrotic syndrome: Urine RIE, blood for total
~ due to any cause (see below). protein, 24 hour urinary protein
Q: What are the causesof periorbital oedema or puffiness? • CCF: Chest X-ray, ECG, echocardiogram
A: As follows: • Cirrhosis of liver: LFf (total protein, A:G ratio,
• Nephrotic syndrome prothrombin time, ultrasonography etc)
• AGN • Hypoproteinaemia: Serum total protein, other inves-
• Myxoedema tigation accordingto the history to find out cause.
• Angioneurotic oedema • Hypothyroidism: FI3, Ff4, TSH.

Tongue

Presentation of
Case No.1
• The tongue is pale, smooth and shiny with
atrophy of papillae.

Oral candidiasis

Smooth pale tongue

My differential diagnoses are:


• Iron-deficiency anaemia.
• Vitamin Bl2-deficiency anaemia. Oral candidiasis
• Riboflavin deficiency.
My diagnosis is oral candidiasis.
Presentation of I
I
Q: What are the underlying diseases associated with
Case No.2 oral candidiasis?
A: As follows:
• There are multiple white patches over the surface of
the tongue with some denuded area on the margin. • Poor oral hygiene.
• Diabetes mellitus.
1 • GEN6RAL EXAMINATION _

• Immunosuppressive disease, e.g. lymphoma, 1. Dry or moist:


leukaemia, malignancy, HIV. • DIY: Dehydration, mouth breathing, xerosto-
• Prolonged use of antibiotics, steroids and mia (in Sjogren syndrome), anticholinergic
immunosuppressive drugs. drug therapy.
• Steroid inhaler use. • Moist: Sialorrhoea in postencephalitic Parkin-
sonism, local mouth infection, gastroesophageal
Q: What is the typical finding in mouth in HlV? reflux disease (GERD), heavy metal poisoning.
A: Hairy leukoplakia (commonly on the lateral margin 2. Colour:
of tongue).
• Pale: Anaemia.
Q: How would you treat oral candidiasis?
• Yellow: Jaundice (mainly in undersurface of
tongue).
A: As follows:
• Bluish: Central cyanosis, methaemoglo-
• Topical antifungal: Nystatin, econazole,
binaernia, sulphaernoglobinaernia (mainly
miconazole.
involves the sides of the tongue], blue col-
• Systemic antifungal: Fluconazole, clotrimazole, etc.
oured food material.
• Maintenance of oral hygiene; antiseptic
• Bluish red: Polycythaemia.
mouthwash may be used.
• Treatment of the underlying disease. • Black tongue (lingua nigra): Ingestion of
bismuth, liquorice, charcoal, etc., Addison
Q: What is disseminated candidiasis? disease (pigmented).
A: It is characterized by fever, pulmonary involvement, • Brownish: CKD.
retinal abscess, endocarditis, skin abscess, brain • Magenta coloured: Riboflavin (vitamin BJ
abscess, osteomyelitis, etc. Intravenous ampho- deficiency.
tericin B may be used in such cases. • Raw beefy tongue (red, swollen and painful):
Vitamin BI2 deficiency, niacin deficiency
Presentation of (pellagra) .
Case No.3 • White patches over tongue: Candidiasis,
leukoplakia, chronic superficial glossiti.s.
• Thetongue is black and rough with pale margin. • Black hairy tongue: Smoking, fungal infection,
tetracycline, penicillins.
• White or greyish coating or "furred tongue":
Smoking, chronic debilitating disease.
• White and red strawberry tongue: Scarlet fever.
• Geographical tongue (there are irregular red
and white patches on the tongue. These lesions
look like a geographic map. Slowly changing
red rings and lines that occur on the surface
of the tongue). It has no clinical significance,
but, can be a sign of riboflavin deficiency.
• Scrotal tongue (deep horizontal fissure): No
clinical significance.
Black hairy tongue • Mushroom-like tongue (sore tongue with
white slough): Corrosive poisoning.
My diagnosis is black hairy tongue.
• Blotting paper-like pallor with black
Q: What are the causes? pigmentation in the margin: Hook worm
A: Causes are poor oral hygiene, prolonged use of anti- infestation.
biotics, steroids, oral contraceptive pill, bismuth. • Angry looking tongue (central coating with
smoking, mouth breathing, radiotherapy etc. It is red tip and margins): Enteric fever.
usually a benign condition. • Glossitis or bald tongue (total loss or atrophy
Q: What diagnoses are possible by looking at the of papillae, smooth tongue): Vitamin BI2
tongue? deficiency, iron-deficiency anaemia, coeliac
A: As follows: disease, pellagra, tropical sprue.
_ SHORT CASES IN CLINICAL MEDICINE

3. Mass or ulcers: Q: What are the causes of mouth ulcer?


• Ulcers: Aphthous, malignant, tuberculous, A: As follows:
snail track ulcer in secondary syphilis, • Aphthous ulcer (usually idiopathic, sometimes
denture, Crohn disease. premenstrual).
• Bite mark: Convulsion. • Trauma.
• Growth in tongue: Squamous cell carcinoma. • GIT disease: Crohn disease, ulcerative colitis,
• Hairy leukoplakia (painless white corrugated coeliac disease.
lesion on sides): Found in AIDS due to EBV
• Rheumatological disease: SLE,Behcet syndrome,
infection.
Reiter syndrome.
• Papilloma (viral wart).
• Local infection: Viral (herpes simplex), syphilis
• Median rhomboid glossitis (lozenge shaped
(chancre in primary and snail tract ulcer
area with loss of papillae and fissuring in
in secondary syphilis), tuberculosis, Vincent
the midline of the tongue, anterior to the
angina.
foramen caecum). It is a congenital anomaly.
• Stevens-Johnson syndrome.
• Cysts in the floor of the mouth: Ranula,
sublingual dermoid cyst. • Pemphigus vulgaris, pemphigoid.
• Erosive lichen planus.
4. Size and shape:
• Cytotoxic drugs.
• Macroglossia: Found in Down syndrome,
acromegaly, cretinism, myxoedema. primary
Q: What are the sign and symptoms of oral cancer?
amyloidosis, mucopolysaccharidosis (e.g.,
A: As follows:
Hurler syndrome), lymphangioma, tumour
infiltration. • Single ulcer (without local trauma).
• Microglossia (atrophy or hemiatrophy): Found • Single white patch ('leukoplakia').
in bulbar and pseudobulbar palsy, lower motor
• Single red patch.
neuron (LMN) lesion ofXIIth cranial nerve.
• Fixed mass.
• Tongue-tie (ankyloglossia).
• Cervical lymphadenopathy.
• Acute swelling:Infection, angioneurotic oedema.
5. Neurological disease: Q: What are the causes of halitosis (bad breath)?
• Flaccid wasted tongue with fasciculation: A: As follows:
BuLbarpalsy.
• Poor oral hygiene.
• Spastic tongue without fasciculation:
• Fetor hepaticus (like dead mouse. It is
Pseudobulbar palsy.
due to methyl mercaptan, found in hepatic
• Jack in the box sign: Rheumatic chorea. precoma).
• Tremor: Anxiety neurosis, thyrotoxicosis,
• Acetone breath (present in diabetic ketoac-
chronic alcoholism, Parkinsonism.
idosis).
• Deviation of the tongue: Deviated to the
• Fishy ammoniacal (present in renal failure).
opposite side is due to upper motor lesion of
• Others: Smoking, alcoholism, lung abscess
the 12~'cranial nerve. Deviated to the same side
(foetid), bronchiectasis, Zenker diverticulum,
is due to lower motor lesion of the 12th nerve.
offensive faecal smell in gastrocolic fistula.
• Loss of taste sensation: Anterior 2/3rd by facial
nerve, posterior 1/3rd by glossopharyngeal Q: What are the causes of gum hypertrophy?
nerve. (Site of taste sensation - sweet at the A: As follows:
tip, sour at the margin, bitter at the back and
salty at any part of the tongue). • Pregnancy.
• Fasciculation: .Bulbar palsy, LMt-J palsy of • Drug: Phenytoin, nifedipine, ciclosporin, oral
Xllth cranial nerve. contraceptive pill with high oestrogen.
• Trombone tongue: Rapid forward and • Acute leukaemia (mainly myelomonocytic
backward movement of the tongue, found in leukaemia).
genera] paresis of insane (GPI). • Gingivitis.
• Chewing tongue: Found in athetosis • Scurvy.
1 • GENERAL EXAMINATION _

Black tongue

Macroglossia

Geographical tongue

Oral lichen planus

Gum hypertrophy with oral candidiasis

Haemangiaoma Raw beefy tongue


Spirit ingestion Stevens-Johnson syndrome

Hairy Leukoplakia

Usual instruction: Q: What is the cause of hairy leukoplakia?


A: Epstein-Barr virus.
• Examine the mouth or look at the tongue.
Q: What is the underlying disease?
Presentation of a Case :f--------, A: It is found in HIV seropositive patient, rare in non-
HIV immunosuppressed patient. Presence of hairy
• There are white, slightly raised, corrugated, leukoplakia indicates rapid progression to AIDS.
irregular, hairy lesions on the margin of the It may be an early finding of HIV infection.
tongue with few furring.
Q: What is the common site of hairy leukoplakia in
tongue?
A: It usually involves the lateral border of the tongue.

Q: What is the treatment?


A: Zidovudine or acyclovir may be helpful. Podophyllin
and retinoid may be used.
Q: Can it become malignant?
A: No, it is not premalignant.

Q: What is leukoplakia?
A: It is the white thickening of the tongue or oral
Hariy leukoplakia mucosa of unknown cause. It is premalignant.
My diagnosis is hairy leukoplakia. It may be associated with alcohol and smoking.
Biopsy should always be taken. Treatment is unsat-
Q: What is your differential diagnosis? isfactory: isotretinoin may be used.
A: Oral candidiasis and lichen planus.
Q: What are the causes of white intraoral lesion ?
Q: What is the difference between hairy leukoplakia A: As follows:
and oral candidiasis? • Candidiasis.
A: Oral candidiasis can be rubbed off and respond to • Leukoplakia.
local and systemic antifungal therapy. Hairy leuko- • Hairy leukoplakia.
plakia cannot be rubbed off and do not respond to • Lichen planus.
antifungals. • SLE.
1 • GENERAL EXAMINATION _

• Smoking. Q: What are the premalignant oral lesions?


• Poor dental hygiene. A: As follows:
• Aphthous ulcer. • Leukoplakia.
• Squamous papilloma. • Lichen planus.
• Secondary syphilis. • Erythroplakia (red patches).
• Carcinoma. • Submucous fibrosis. ..
• Idiopathic keratosis.

Dupuytren Contracture

Presentation of a Case • Diabetes mellitus (diabetic cheiroarthropathy,


confuses with systemic sclerosis).
• There is thickening of the palmar fascia, more
• May be associated with tUberCUIOSiS._j
marked along the ulnar side with flexion
• Retroperitoneal fibrosis.
contracture of 4th and 5th fingers of both hands
• Peyronie disease.
(may be all the fingers). • Idiopathic (in many cases).

Q: How to treat Dupuytren contracture?


My diagnosis is Dupuytren contracture.
A: As follows:
Q: What is your differential diagnosis? • Exercises, warm water application or splints may
A: Diabetic cheiroarthropathy. be helpful.
Q: What is Dupuytren contracture? What are the causes?
• If the palmar thickening is growing rapidly, triam-
cinolone may be injected into the growingnodule.
A: It is characterized by thickening, fibrosis and short-
• Radiation therapy may be used in early stages.
ening of superficial palmar fascia, causing flexion
• Surgery:When there is severe flexion contracture. Tn
contracture of fingers. The ring and little fingers are
50% cases, it may recur with in 10 years after surgery.
commonly affected.
• New treatment: Injection of collagenase into the
In Dupuytren contracture, there is inability to ex-
scarred or fibrous tissue.
tend the fingers, puckering of the skin and presence
of palpable nodules; it may affect the sole of foot. It Q: What is the prognosis or natural course of
is usually painless and often bilateral, more common Dupuytren contracture?
after 40 years, but increases in incidence with advanc- A: Natural course is unpredictable. It may be slowly
ing age. Five times common in male than female; it progressive with little disability over many years.
may be familial with dominant inheritance. It is more In some patients, it may progress rapidly with severe
common in Whites and Europeans. deformity and functional disability.
Mechanism is unknown. Palmar fascia contains
Q: What is diabetic cheiroarthropathy?
large amount of xanthine, which may be related to
A: It is a complication of long-standing diabetes mel-
the pathogenesis.
litus. In this condition, skin of the dorsum of fingers
is tight, waxy, shiny, and depigmented with joint
Causes of Dupuytren contracture: stiffness and flexion deformities of many fingers.
Usually painless, limitation of joint movement
• Cirrhosis of liver (commonly alcoholic).
may be present. There is inability to extend the
• Alcoholism (itself, not necessarily by cirrhosis).
metacarpophalangeal (MCP) or interphalangeal
• Prolonged use of antiepileptic drug
(IP) joint of at least one finger bilaterally. This can be
(phenytoin) .
better detected by prayer sign. Occasionally, affects
• Manual. worker (gardener) and chronic
the wrist and shoulders. Cause of cheiroarthropathy
vibration injury.
is unknown, probably there is cross-linking and
• Traumatic.
thickening of collagen. It occurs in any type of
• Familial (as autosomal dominant, associated
diabetes mellitus, and is confused with systemic
with Garrod patch on dorsum of hand).
sclerosis. There is no specific treatment.
_ SHORT CASES IN CLINICAL MEDICINE

Dupuytren contracture Dupuytren contracture

Paget Disease

I
• Local warmth.
Presentation of • Hearing (deafness).
Case No.1 • Heart (there may be evidence of high-output
cardiac failure).
• This elderly man has asymmetrical enlargement
• Fundoscopy (to see optic atrophy and angioid
of skull, more on the right.
streaks in retina).
• Right side of the face is also enlarged.
I
Presentation of I

Case No.2

• This elderly man has bowing of both tibias, more


on the right side.

Asymmetrical enlargement of skull.


Bowing of tibias.
My diagnosis is Paget disease.
My diagnosis is Paget disease.
Q: What else do you like to see in this patient? Q: What are the causes of bowing of tibia?
A: As follows: A: As follows:
• Bone in other parts of the body: Leg (bowing • Paget disease.
of tibia), spine (kyphosis), any other bone • Rickets.
enlargement. • Congenital syphilis (sabre shin).
__ ~ 1_. GENERAL EXAMINATION _

Q: What is Paget disease (osteitis deformans)? What • Serum alkaline phosphates (Usually high; may be
are the features? normal in 10%casesdue to monostotic involvement).
A: Paget disease is characterized by excessive and • Isotope bone scan (to see extent of bone
disorganized resorption and formation of bone, involvement. It cannot differentiate between Paget
resulting in deformity and fracture. Commonly disease and osteoblastic metastatic carcinoma).
involved bones are pelvis, femur, tibia, lumbar • Urinary hydroxyproline (high; it is a marker of
spines, skull and scapula. Common over 55 years bone breakdown).
of age, is rare under 40 years. More in temperate • cr scan or MRI of bone may be done.
climate; male and female ratio is 2:3. • Rarely bone biopsy.
In Paget disease, there is increased osteoclastic • Calcium and phosphate: Normal (calcium may
bone resorption and increased osteoblastic activ- be high in prolonged immobilization or fracture).
ity followed by abnormal bone formation. Bone
formation exceeds resorption; the new bone is Q: How to treat Paget disease?
bigger, but weaker and filled with new blood A: As follows:
vessels. The disease may involve one bone (monos- 1. For pain: NSAIDs
totic, 10-15%) or many bones (polyostotic). 2. To prevent further bone break down:
• Bisphosphonates: Pamidronate, zoledronate,
Features are: risedronate are more effective. Also etidro-
nate and tiludronate. (Hypocalcaemia may
• Many cases are asymptomatic (60-80%), detected occur, so adequate calcium and vitamin D
radiologically. should be taken.)
• There may be bone pain, joint pain or stiffness, • Calcitonin: May be used subcutaneously
bowing of the legs, deformities (in weight-bearing 100-200IU, three times weekly,for2-3 months.
bones such as femur, tibia), pathological fracture, It is less convenient and more expensive.
enlarged head and other visible features. 3. Orthopaedic surgery: Joint replacement or
• Neurological features such as deafness, cranial osteotomy may be needed. Neurosurgery may
nerve defect, nerve root pain, spinal cord be needed in spinal cord compression.
compression and spinal stenosis may occur due to
enlargement of affected bone. Q: What are the complications of Paget disease?
A: As follows:
• Warm skin over the affected bone, high-output
cardiac failure due to hyperdynamic circulation • Bone fractures and deformities.
(due to increased vascularity of the bone), etc. • Deafness due to otosclerosis of the ossicles, less
due to compression of VI11thcranial nerve.
Cause is unknown. Genetic factors are important.
• High-output heart failure.
Some slow viruses like measles may be involved.
• Secondary osteoarthrosis.
• Optic atrophy.
Q: What investigations should be done? • Spinal cord compression causing paraplegia.
A: As follows: • Spinal stenosis,
• Bone X-ray: Enlargement of bone with typical lytic • Basilar invagination (platybasia) causing brain-
and sclerotic lesion. X-ray skull shows lytic lesion stem sign.
osteoporosis circumscripta, also enlargement • Osteosarcoma (Rare but serious. Occurs in 1% in
and sclerosis, thickening of trabeculae. those for >10 years).

Examination of Hands
The usual instructions are: • Systemic sclerosis (see in the Chapter 9).
• Look at the hands. What is the diagnosis? • Tophaceous gout.
• Examine the hands. • Bouchard nodes, Heberden node (in
osteoarthritis ).
By looking, the obvious findings may be:
• Skin rash and Gottron patch (dermatomyositis).
• Rheumatoid arthritis (to examine rheumatoid • Large size (acromegaly).
hand, see in the Chapter 9). • Claw hand.
_ SHORT CASES IN CLINICAL MEDICINE

• Wrist drop. • 'Point your thumb towards the ceiling and stop
• Myotonic dystrophy (diagnosed by handshake or me from bending it' (test for abductor pollicis
asking the patient to close and open the hands). brevis).
• Raynaud disease or phenomenon. • 'Fixthe tip of little finger and thumb, and stop me
• Arachnodactyly. from separating it' (test for opponens pollicis).
• Syndactyly. • 'Spread your fingers wide apart and stop me'
pushing them together' [test for dorsal interossei
• Polydactyly.
(DAB) means dorsal abduction].
• Short 4th metacarpal.
• 'Hold a piece of paper between two fingers and
• Palmar erythema. stop me from taking it out' [test for palmar
• Nail and nail bed change: Clubbing, koilonychia, interossei (PAD) means palmar adduction].
leukonychia, half-and-half nail, splinter haemor- • 'Hold a paper between thumb and index
rhage (infective endocarditis), Beau line, Mee line, finger and stop me from taking it out' [test for
nail fold telangiectasia and erythema (SLE), fungal adductor pollicis]. If the muscle is paralysed,
infection, periungual fibroma, nail pitting (pso- the patient can hold the paper by flexing
riatic arthritis), yellow nail syndrome, absent nail thumb. It is called Froment sign. (It Indicates
(nail patella syndrome), onycholysis, nicotine stain. ulnar nerve lesion.)
• Dupuytren contracture. • 'Squeeze my fingers' (test for C8 and Tt lesion).
• Trophic change (gangrene, ulceration). long and short flexors of the fingers.
• Wasting: Thenar, hypothenar or generalized. Q: What are the findings in hand in infective
• Tremor. endocarditis?
• Warm and sweaty palm (thyrotoxicosis), cold and A: As follows:
sweaty palms (anxiety). • Osler nodes (small painful violaceous raised
• Single palmar crease (in Down syndrome). nodule, 0.5-1.5 ern. present on the tip of the
fingers and toes, also palmar aspect, probably
Ifit is RA or systemic sclerosis, then follow the examina-
due to development of vasculitis or septic
tion accordingly.
emboli).
If wasting is obvious, see the site-generalised or local- • Splinter haemorrhage.
ized (thenar or hypothenar): • Clubbing.
• Janeway lesion (large painless erythematous
• If there is thenar wasting only: It indicates median
macule containing bacteria on palm, pulp of the
nerve lesion.
fingers. It may be found in the sole).
• If there is appearance of hypothenar and other
• Petechial haemorrhage.
muscles wasting (except thenar): Indicates ulnar
nerve lesion. • Infarction on the tip of the fingers.
• On the dorsum: Wasting with dorsal guttering
Q: What are the findings in hand in CLD?
(interossei) indicates ulnar nerve lesion.
A: As follows:
• Generalised wasting indicates C8 and Tl lesion.
• Palmar erythema.
Then, perform the neurological examination as
follows: • Dupuytren contracture.
• Clubbing.
1. Sensory tests in the sensory supply along ulnar and
• Leukonychia.
median nerve [(if wasting is present, but sensory
is intact, more likely diagnosis is motor neurone • Flapping tremor.
disease (MND) I. • Spider angioma.
• Pigmentation.
2. Next, perform the motor function of hand muscles
(ask as follows): • Jaundice.
• 'Open and close the hands as quickly as • Scratch mark.
possible' (observe the weakness and evidence • Xanthoma.
of myotonia dystrophica). • Cyanosis.
1 • GENERAL EXAMINATION _

Arachnodactyly Dupuytren contracture with palmar erythema

Clubbing with Beau line Heberden node

Clubbing with leukonychia Koilonychias

Digital infarction in peripheral arterial disease Myotonic dystrophy


_ SHORT CASES IN CLINICAL MEDICINE

Myotonic dystrophy

Prayer sign

Osler lesion in SSE

Short 4th metacarpal of the left hand

Polydactyly

Rheumatoid hand Single palmar crease in Down syndrome


_~ __ ~~~_1_. GENERAL EXAMINATION _

Splinter haemorrhage Systemic sclerosis

Syndactyly in left hand with one extra finger Ulnar claw hand

Syndactyly in right index and middle fingers Wasting of thenar and hypothenar muscles

Lupus Pernio
Usual instruction:
• Look at the patient's face. Or perform the general
examination.

Presentation of a Case:

• There is bluish (or violaceous) discoloration at


the tip and ala of the nose.

My diagnosis is lupus pernio.

Lupus pernio.
_ SHORT CASES IN CLINICAL MEDICINE

Q: What are the sites of lupus pernio? Q: What is Lofgren syndrome?


A: Nose, cheeks, earlobes, hands and feet. A: In sarcoidosis when there is erythema nodosurn, pol-
yarthralgia and BHL, it is called Lofgren syndrome.
Q: What is the underlying disease?
Q: What is Heerfordt syndrome (uveoparotid fever or
A: It is due to sarcoidosis. It indicates chronic pulmo-
nary sarcoidosis (may progress to fibrosi.s of lung; Heerfordt-Waldenstrom syndrome)?
also chronic uveitis and bone cyst in the phalanges A: It is characterized by fever, bilateral parotid enlarge-
ment, anterior uveitis and lower motor neuron
may be present).
facial palsy.
Q: What is the differential diagnosis of lupus pernio? Q: What investigation should be done?

-
~
....
CI)
A: SLE, rosacea, rhinophyma, lupus vulgaris, leprosy.

Q: What is sarcoidosis? What are the causes?


A: As follows:
1. CBC, ESR (lymphopenia, high ESR).
5
o A: It is a multisystem granulomatous disease of 2. MT (usually negative).
unknown aetiology, characterized by noncaseating 3. Serum calcium and y-globulin (usually high).
granuloma in different organs. Cause is unknown. 4. X-ray chest (BHL, lung infiltrate, pulmonary
There is an imbalance between subset ofT lympho- fibrosis, honeycomb shadow, miliary mot-
cyte and disturbance of cell-mediated immunity. tling, eggshell calcification).
5. X-ray of hands or feet (cyst may be found in
Q: What else do you want to see in sarcoidosis? the phalanges).
A: Skin lesions in other parts of the body like plaque, 6. X-ray kidney (may show nephrocalcinosis).
erythema nodosum, maculopapular lesion, hyper- 7. High-resolution computed tomograph.y
or hypopigrnentation, subcutaneous nodule, etc. (HRCT) of chest.
Also, I want to see other features of sarcoidosis like 8. Lung function tests (restrictive lung disease;
generalized lymphadenopathy, hepatosplenomegaly, also reduction of gas transfer).
lung (to see evidence of fibrosis), eyes (uveitis), bilat- 9. Liver function tests (usually abnonnaJ).
eral parotid involvement. neurological examination 10. Bronchoscopy (shows cobble-stone appear-
(to see evidence of neurosarcoid), etc. ance of mucosa).
11. Bronchoalveolar lavage (shows increased
Q: wha: history do you like to take? CD4:CD8 T-cell ratio. Increase neutrophil in
A: Fever,arthritis, arthralgia, cough Or breathlessness, etc. pulmonary fibrosis).
12. Lung biopsy: Transbronchial or percutaneous
Q: How the patient of sarcoidosis usually presents? (pneumothorax may develop), open biopsy
A: Common in young adult. in 3rd or 4th decade, may be required (by thoracotomy).
slightly more in female. Presentation may be: 13. FNAC or biopsy from other involved site may
• Asymptomatic [commonly affecting young be needed: Lymph node, skin nodule, liver,
adults, usually presents BHL in X-ray chest or lacrimal gland, etc.
abnormal liver function test (LFT)I. 14. Others:
• Symptomatic: Fever, polyarthritis or arthralgia, • Angiotensin-converting enzyme in blood
erythema nodosum, other skin lesions such as (increased level indicates active sarcoidosis;
lupus pernio, plaque, skin rash and other organ this test is not helpful for diagnosis).
involvement. • 67Gallium scanning of lung (abnormal
• Other features: Bilateral parotid enlargement. eye diffuse uptake).
signs (episcleritis, scleritis), cardiac involvement • Kveim test: Intradermal injection of prepared
(arrhythmia, cardiomyopathy, cardiac failure), sarcoid tissue in the forearm. In the positive ease,
pulmonary involvement. neurological features formation of a nodule after 6 weeks. Biopsy is
such as cranial nerve palsy, meningism, seizure, done from the nodule, which shows typical
psychosis, diabetes insipidus, etc. sarcoid lesion (this test is not done now).
Q: What are th.e histopathological findings in
N.B. Presence of fever, arthritis or arthralgia, ery- sarcoidosis?
thema nodosum plus BHL on x-ray is highly
A: Noncaseating granuloma consisting of epithe-
J suggestive of sarcoidosis. lioid cells, rnacrophages and lymphocytes (mostly
___________ , • GENERAL EXAMINATION _

T cells), and multinucleated giant cells (in TB, there to alternate day treatment with prednisolone
is caseating epithelioid granuloma). 15 mg for 6 to 12 months.
3. Other treatment:
Q: What are the radiological stages of sarcoidosis? • Avoid strong sunlight (may preciprtate
A: 4 stages: hypercalcaemia and renal impairment).
• Stage 1: BHL (usually symmetrical, may be par- • Topical steroid for uveitis.
atracheal lymphadenopathy. May be associated • Inhaled corticosteroid.
with erythema nodosum, fever and polyarthral- • Chloroquine, hydroxychloroquine, low-dose
gia. Spontaneous resolution occurs in 1 year). thalidomide may be useful in cutaneous
• Stage 2: BHL with parenchymal pulmonary infil- sarcoid.
tration (often diffuse, spontaneous resolution • In patient with severe disease: Methotrexate
may occur). 10-20 mgweekly, or azathioprine 50-100 mg
• Stage 3: Diffuse pulmonary infiltrate without BHL dailyorTNF-a blocker [infliximab, etanercept).
(less likely to resolve spontaneously). • Single lung transplantation may be done in
• Stage 4: Pulmonary fibrosis. The patient may selected case.
complain of shortness of breath, cough. There
is progressive ventilator failure, pulmonary Indications of steroid:
hypertension and cor pulmonale. Spontaneous
• Severe symptoms such as persistent erythema
resolution is less likely to occur. (X-ray chest may
nodosum, fever, arthritis or arthralgia.
show eggshell calcification.)
• Parenchymal lung disease in any form.
• Vital organ involvement (eye, central nervous
Q: Can there be sarcoidosis and tuberculosis
system, heart, kidney).
together?
• Hypercalcaemia.
A: Yes. In sarcoidosis, there may also be tuberculosis.
Prognosis: Mortality is 1-5%. Death is due to cardiac
Q: How to treat sarcoidosis? involvement pulmonary fibrosis, and cor-pulmonale
A: As follows: or renal damage.
1. Acute with erythema nodosum: Bed rest with
NSAID may be sufficient. If symptoms are Features suggestive of less favourable outlook:
severe, short-course steroid may be given. Spon- • Age <40 years.
taneous resolution occurs usually. • Persistent symptoms >6 months.
2. If the disease is not improved 6 months after • Involvement of more than three organs.
the diagnosis, prednisolone should be given. • Lupus pernio and stage 3 and 4 radiologically.
Dose: Prednisolone 30 mg for 6 weeks, reduced • Afro-Caribbean population.
I

CHAPTER 2

CARDIOVASCULAR SYSTEM
"The heart moves of itself and does not stop unless forever"
- Leonardo da Vinci

I
Introduction

Tn any clinical examination, a case of cardiovascular • What are the diseases that can be diagnosed by
system (CVS) is frequently selected. Also, it is an palpation? (See later.)
extremely common system tested in any oral exami- • Auscultate the precordium. What are your
nation. Very often, examiner asks, 'Examine the CVS'. findings? (Present systematically, starting from the
However, many a times, examiner may ask to perform heart sounds, murmur and any extra finding.)
a particular part of CVS rather than the whole system
and asks questions on that particular part. For example, To attain a good skill, see and examine more cases. Also,
make a good practice to present systematically.
• Examine the CVS. Remember, the examiner may interrupt at any part
• Examine the precordium. of your examination and ask, 'What is your finding?
• Auscultate here ... (examiner may point a particular What is the more likely diagnosis for this finding
part.) only'?
• Palpate the pulse. What are your findings? (There Once you are asked to examine the CVS, the more
may be no pulse or irregular, small- or high-volume likely underlying diseases are:
pulse, bradycardia, tachycardia and pulse delay.)
• Palpate the pulse and auscultate the precordium. • Mitral stenosis [(MS) pure).
Describe your findings. [There may be low-volume, • Mitral regurgitation (MR).
slow-raising pulse. On auscultation, ejection • MS with pulmonary hypertension (PH).
systolic murmur (ESM) may be present due to • MS with MR (mixed mitral valve disease).
aortic stenosis (AS).] • AS or aortic regurgitation (AR)
• Palpate the precordium. What are your findings? • AS with AR (mixed aortic valve disease).
(There may be tapping or shifting of the apex beat, • Congenital heart disease [atrial septal defect
heaving or thrusting in nature, thrill, palpable P2, (ASD), ventricular septal defect (VSD), patent
left parasternal lift.) ductus arteriosus (PDA) and dextrocardia].

Examination Routine

If the examiner asks to examine CVS, start examining 1. Look at the patient carefully:
systematically, including some general examination in • Dyspnoeic or orthopnoeic (left ventricular
relation to CVS. failure (LVF)), cachexia (in severe heart failure).
2. Face:
Proceed as follows:
• Malar flush (in MS).
Introduce yourself 'I am Dr. ... May I examine your • Marfanoid face.
heart please'? • Corneal arcus and xanthelasma [related to
Position the patient at 45° with a backrest or pillows. atherosclerosis in ischaemic heart disease
With permission, remove the clothing to expose the (IHD)], .ArgyllRobertson pupil (related to AR),
chest and neck (be careful with a female patient). mouth (high arch palate in Marfan syndrome).
2 • CARDIOVASCULAR SYSTEM _

3. Anaemia. • Volume (make sure you lift the arm to see collaps-
4. Cyanosis [tetralogy of Fallot (TOF) and Eisen- ing pulse).
menger syndrome). • Character.
5. Oedema [leg and sacrum in congestive cardiac fail- • Condition of the vessel wall.
ure (CCF)). • Radiofemoral delay and radioradial delay or
6. In hands: inequality.
• Clubbing. Compare other pulses simultaneously (carotid pulse
• Koilonychia. should not be seen simultaneously). Volume and char-
• Cyanosis. acter of pulse are better seen in brachial and carotid
• Splinter haemorrhage. artery. Collapsing pulse in .t\R and pulsus alternans in
• Osler node (red, raised, palpable, tender nodule
LVFare better seen in radial.
on the pulp of finger or toes; also in thenar or
hypothenar area).
• Janeway lesion (nontender, red, maculopapular Neck Veins (JVP)
lesion on palm or pulp of finger). • The patient should be at 45 0

• Xanthoma: Palmar or tendon (atherosclerosis • Normal or engorged (internal jugular vein, lies
in IHD). medial to sternomastoid). If visible, see any
• Tobacco stain (smoker, IHD). prominent wave. See hepatojugular reflux by
pressing firmly with palm over the middle of
abdomen, which raises the upper limit of JVP.
Next, measure the height from sternal angle. (It
indicates mean right atrial pressure. Normally, it is
at the level of sternal angle and invisible.)

Other signs in the neck:


• Tall, sinuous pulsation, oscillating up to the ear
lobule (prominent V-wave) is found in tricuspid
regurgitation (TR).
• Dancing carotid pulse (Corrigan sign in AR).
• Vigorous arterial pulsation in neck (coarctation of
Osler node (in finger) aorta).
• Other pulsation in neck (carotid aneurysm or
subclavian artery aneurysm).

Blood Pressure
• Measure BP (normal or high). If needed, see in
both arms. Also, in standing and lying (to see
postural hypotension).
• Low systolic, normal diastolic and narrow pulse
pressure (in AS).
• High systolic, low diastolic and wide pulse pressure
(in AR).

Osler node (in toe)


Precordium

Now examine the pulse, jugular venous pressure (JVP), Inspection


blood pressure (BP) and finally precordium.
• Deformity of chest (kyphosis, scoliosis, lordosis,
Pulse pectus excavatum or carinatum).
See the following points in radial pulse: • Visible cardiac impulse (visible apex beat).
• Rate. • Other impulses (epigastric, suprasternal or any
• Rhythm. other impulse).
~ HORT-CASES IN CLINICAL MEDICIN . .;..;;..E__ ~~ _

• Scar mark in the midline [valve replacement IAuscultation


or coronary artery bypass grafting (CABG)],
thoracotomy scar (valvotomy in MS). 1. See first and second heart sounds in all areas. At
• Pacemaker or cardioverter defibrillator box may the same time, palpate right carotid pulse simul-
be seen. taneously with thumb. (Yes! Examiner notices.)
First heart sound coincides with carotid pulse, the
Palpation second sound does not (comes after).
2. Murmur:
1. Apex beat: • Site (apical, parasternal, aortic or pulmonary area).
• Site (Localise the intercostal space. Do not • Nature: Systolic (pansystolic or ejection systo-
forget dextrocardia). lic), diastolic (mid-diastolic or early-diastolic)
• Distance from midline. by feeling carotid pulse at the same time
• Nature (normal, tapping, heaving, thrusting (systolic coincides with carotid pulse, and
and diffuse). diastolic does not coincide).
2. Thrill: • Radiation (PSM to left axilla and ESM to neck).
• Site (apical or basal or other intercostal space). • Relation with respiration (right-sided murmur
• Nature (systolic or diastolic): Feel carotid pulse increases on inspiration and left-sided murmur
at the same time. If coincides with carotid pulse, increases on expiration).
it is systolic and if it does not coincide (comes • Grading of murmur (2/G or 4/G).
after or before), it is diastolic. 3. Added sounds (pericardial rub and opening snap).
4. Auscultate the back of chest for crepitations (pul-
N.B. See apical and basal thrill. Apical thrill is best monary oedema).
~ seen by turning the patient to left lateral posi-
5. Ask for permission to palpate the liver (enlarged
tion with breathing held after expiration; apex
tender liver in CCF, pulsatile liver in TR) and
comes closer to the chest wall. Basal thrill is best
spleen (splenomegaly in SBE).
seen by palm with the patient sitting and bend-
ing forward, breathing held after expiration N.B. Remember the following points:
-,.-
(base of heart comes closer to the chest wall).
• If mid-diastolic murmur (MDM) is present
at apex, make sure that you have listened by
3. Left parasternal heave or lift: Place the flat of right
turning the patient in left lateral position with
palm in left parasternal area and feel by giving gentle
the bell of stethoscope with breathing held
sustained pressure [presence ofteft parasternal heave
after expiration. (Yes! Examiner notices.)
indicates right ventricular hypertrophy (RVH)).
• If early-diastolic murmur (EDM) is present,
4. Palpable P2 (if present, indicates PH).
make sure that you have listened with the
5. Epigastric pulsation.
patient sitting and bending forward with
breathing held after expiration.
Percussion
• If presystolic murmur (PSM) is present, put
Usually not done; may be helpful to diagnose pericar- your stethoscope in axilla to see radiation.
dia] effusion (area of cardiac dullness is increased) and • IfESM is present, put your stethoscope on right
emphysema (cardiac dullness is obliterated). carotid to see radiation.

Important Discussions in Relation to CVS

Pulse • Atrial fibrillation (AF) and atrial flutter.


• Ventricular tachycardia.
A. Rate
Causes of sinus tachycardia:
• Tachycardia: When the pulse rate is >lOO/min. 1. Physiological (anxiety, emotion, exercise and
• Bradycardia: When the pulse rate is <GO/min. pregnancy).
Causes of tachycardia: 2. Pathological: Hyperdynamic circulation [fever,
• Sinus tachycardia due to any cause (see below). anaemia, thyrotoxicosis, arteriovenous (AV) fistula
• Supraventricular tachycardia. and beriberi],
• CCF. A: By physical exercise. With exercise, ectopics will
• Myocarditis. disappear, but AF will be more prominent [needs
• Chronic constrictive pericarditis. electrocardiography (ECG) for confirmation].
• Shock (except vasovagal attack).
• Acute anterior myocardial infarction (MI; except IC. Volume of Pulse
inferior MI). Causes of high-volume pulse
• Sick sinus syndrome.
• Pulmonary embolism. • AR.
• Drugs (salbutamoJ, atropine and other • Hyperdynamic circulation due
sympathomimetics). to any cause.
• PDA.
Causes of bradycardia:
• Hypertension.
1. Sinus bradycardia due to any cause (see below).
2. Second-degree heart block. Causes of low-volume pulse
3. Complete heart block.
• Shock.
4. Nodal rhythm.
• AS.
Causes of sinus bradycardia • MS.
1. Physiological (due to increased vagaL tone): Athlete, • Chronic constrictive pericarditis.
during sleep. • Pericardia! effusion.
2. Pathological: • PH
• Acute inferior Ml.
• Myxoedema (due to reduction of sympathetic I D. Character of Pulse
activity). 1. Anacrotic pulse is a slow-raising, small-volume
• Hypothermia. Raised intracranial tension (due pulse (notch on upstroke). It is caused due
to inhibitory effect on sympathetic outflow). to AS.
Obstructive jaundice (due to deposition of 2. Plateau pulse is of small volume with slow
bilirubin in conducting system). upstroke. It is caused due to AS.
• Drugs (digoxin, ~-blockers, amiodarone and 3. Bisferiens pulse is the double peak of pulse, which
verapamil) . is felt better in carotid (due to combination of slow
raising and collapsing). Caused due to combined
lB. Rhythm AS and AR.
4. Water hammer pulse is typically found in AR
It is the interval between successive pulses. It may be
(see page 85).
regular or irregular. Irregular rhythm may be irregularly
5. Pulsus alternans is an alternate strong and weak
irregular or regularly irregular.
beat (suggestive of LYF).
Causes of irregular pulse: 6. Jerky pulse is seen in carotid artery. It is caused due
to hypertrophic cardiomyopathy (HCM).
1. Regularly irregular (follows a definite pattern of
7. Pulsus paradoxus: When voLume of pulse reduces
regular rhythm followed by irregular rhythm).
on inspiration and increases on expiration, it is
It occurs in:
called pulsus paradoxus (also systolic SP falls
• Sinus arrhythmia (pulse rate increases on each
during inspiration, normally <10 mmHg). It
inspiration, decreases on each expiration). It is
is the exaggeration of normal phenomenon
abolished by exercise.
(normally present in children). It is better
• Occasional ectopics. Second-degree heart block
detected by sphygmomanometer. Abnormal, if
(Mobitz type I, Wenckebach type).
>10 mmHg.
2. Irregularly irregular means irregular in rhythm
and volume. Its causes are: Causes of pulsus paradoxus:
• Atrial fibrillation.
• Pericardial effusion (especially, cardiac
• Multiple ectopics.
tamponade).
• Atrial flutter with variable block.
• Chronic constrictive pericarditis.
• Paroxysmal atrial tachycardia with variable block.
• Acute severe asthma and chronic obstructive
Q: How to differentiate between AF and multiple pulmonary disease (COPD).
ectopics in bedside? • Massive pulmonary embolism.
Mechanism of pulsus paradoxus: During inspira-
Venous Arterial
tion, intrathoracic pressure falls, blood pools
in pulmonary vessels and hence left heart fill- 1. It is wavy (two peaks with Not wavy
ing is reduced with reduction of cardiac output. cardiac cycle)
Therefore, the pulse volume is low, which is 2. It has an upper limit No definite upper limit
reverse on expiration.
3. Upper limit falls with Not so
8. Collapsing pulse: There is a rapid upstroke and inspiration
descent of pulse, seen by raising the arm above the
4. Varies with posture Independent of posture
head. Causes are:
• AR (the commonest cause).
s. It is better seen than palpation It is better felt than seen

• Hyperdynamic circulation due to any cause 6. Upper limit is increased by Not so


(see above). pressing the abdomen
(hepatojugular reflux)
• PDA.
• Rupture of sinus of Val salva. Large arteriovenous 7. Just felt or lightly felt Thrusting
communication.
8. It is obliterated by light pres- Cannot be obliterated
Q: What is the difference between high-volume pulse sure at the root of the neck
and collapsing pulse? and then filled from above

A: Collapsing pulse is always a high-volume pulse,


but all the high-volume pulses are not necessarily Causes of cannon wave (giant 'a' wave): Occurs when
collapsing. atria contracts against closed tricuspid valve

• Complete heart block Ca' wave is intermittent).


JVP • Junctional rhythm (regular cannon wave).
• Ventricular tachycardia Ca' wave is intermittent).
(See internal jugular vein, which lies along the medial
border of sternomastoid. External jugular vein is not Giant 'v' wave is tall, sinuous, oscillating up to ear
examined as it is tortuous and subject to compression.) lobule. It is caused due to TR.
Normally, it is 2 cm. If >3 ern, right heart filling pres- Causes of giant X descent are chronic constrictive
sure is raised. It is a sign of right heart failure or volume pericarditis and cardiac tamponade.
overload. Cause of slow Y descent is TS.
Kussmaul sign: It means raised JVP during inspi-
Causes of raised JVP:
ration due to reduction of right ventricular output.
• CCF (right heart failure). It is best seen with the patient at 900 with normal
• Pericardial effusion. breathing (normally, JVP falls during inspiration). Its
• Chronic constrictive pericarditis. causes are:
• Pulmonary embolism.
• Pericardial effusion (usually, cardiac tamponade).
• Tricuspid regurgitation (TR) and tricuspid stenosis
• Chronic constrictive pericarditis.
(TS).
• Right ventricular infarction.
• Pulmonary regurgitation and pulmonary stenosis
(PS). Corrigan sign: Dancing carotid pulse is called Corrigan
• Superior vena caval (SVC) obstruction sign. It is caused due to AR.
(nonpulsatile). Carotid pulse is prominent and forceful; it may be
• Others: Occasionally may occur in pregnancy, seen in:
exercise, anxiety and anaemia.
• Coarctation of aorta.
Causes of prominent 'a' wave in JVP (comes just before
• Aneurysm of aorta.
carotid pulse):
• Carotid artery aneurysm.
• PH. • Subclavian artery aneurysm.
• Pulmonary embolism. • Others: Hyperdynamic circulation, AR and anxiety.
• TR and TS.
• PS. Precordium
Q: What are the differences between venous pulse or Deformity of chest (kyphosis, scoliosis, pectus exca-
arterial pulse in neck? vatum and pectus carinatum) may be associated with
A: As follows: Marfan syndrome, cor pulmonale.
Apex Beat 2. Second heart sound: Splitting is better heard in
pulmonary area, in inspiration due to prolonged
It is the lowermost and outermost, definitely palpable right ventricular systole.
cardiac impulse. It is normally 9 em from midline or
1 cm internal to the midclavicular line in left fifth inter- Causes of wide splitting of second sound:
costal space. Apex beat may be heaving, thrusting or • PS.
tapping in nature. • Right bundle branch block (RBBB).
• Atrial septal defect (ASD).
• Heaving: Forceful, sustained, lifting the examining
• Cause of wide and fixed splitting of second
finger (pressure overload). It indicates left ventricular
sound: ASD.
hypertrophy (LVH) (due to hypertension and AS).
• Thrusting: Forceful, less sustained, lifting the Causes of reverse splitting of second sound (A2 is
examining finger (volume overload). It indicates delayed):
left ventricular dilatation (as in MR and AR). Also • Left bundle branch block (LBBB).
called hyperkinetic or dyskinetic apex. Felt over • AS.
larger area of precordium. • Hypertension.
• Tapping apex: Neither sustained nor forceful, not
Causes of loud second sound:
lifting the finger. It is the palpable first heart sound.
• Systemic hypertension.
Found in MS (rarely TS).
• PH.
Causes of double apex beat
Causes of soft second sound:
• Ventricular aneurysm.
• Calcified or severe AS.
• HCM.
• Severe PS.
Causes of impalpable apex beat
• AR.
• Thick chest wall (obesity).
• If the apex is behind the rib. 3. Third heart sound: Low-pitched, distant sound
• Emphysema. due to ventricular filling, and comes after second
• Pericardial effusion. sound. It is better heard at the apex with bell of
• Dextrocardia (apex is on the right side). stethoscope. Its causes are:
• Physiological: In young, in athlete, during
Causes, of diffuse apex beat
pregnancy and in fever.
• Anterior MJ (better seen).
• Pathological: LVF, MR, anaemia, cardiomy-
• Occasionally, in left ventricular aneurysm.
opathy, AR, chronic constrictive pericarditis.
Left parasternal heave or lift indicates RVH (in PH, cor (This third sound is called pericardial knock.)
pulmonale, PS and pulmonary regurgitation, and TR).
Causes of epigastric pulsation: 4. Fourth heart sound: Low-pitched sound due to
atrial contraction, precedes first heart sound, better
• Normally palpable (lean and thin person). heard at the apex. Causes are (ventricular failure
• RVH. and ventricular hypertrophy):
• Aneurysm of abdominal aorta (expansile
• Due to left heart, hypertension, AS, IHD and
pulsation) .
HCM.
• Mass overlying aorta (carcinoma of stomach).
• Due to right heart, PS and PH.
• Pulsatile liver (in TR).
Suprasternal pulse: Usually arterial (due to aneurysm
ITriple Rhythm
of aorta and atherosclerosis).
Pulsation around scapula: Due to coarctation of aorta When in addition to first and second heart sounds,
(due to anastomotic vessels). there is another sound that is either third or fourth, it is
called triple rhythm.
Heart Sounds
1. First heart sound:
IGallop Rhythm
• Loud in MS, 1'$ (occasionally in anxiety, When triple rhythm is associated with tachycardia
exercise). (>100), it is called gallop rhythm.
• Soft or absent in MR, myocarditis and It is called 'gallop' as the cadence of sound resembles
cardiomyopathy. a galloping horse.
Gallop rhythm is of three types: • Austin Flint murmur in AR.
• Carey Coombs murmur (due to mitral valvulitis in
• Protodiastolic: Extra sound (third heart sound), in
acute rheumatic fever, RF).
early- or mid-diastole.
• ASD (due to increased flow through tricuspid
• Presystolic: Extra sound (fourth heart sound) in
valve).
late diastole.
• Summation: When third and fourth heart sounds Continuous murmur
coalesce, it is called summation gallop, usually
Present in both systole and diastole. Its causes are:
with heart rate >120.
• PDA.
Presence of gallop rhythm indicates cardiac failure • AVfistula (coronary, pulmonary or systemic).
(LVF). • Aortopulmonary fistula (may be congenital or
Blalock-Taussig shunt).
IMurmur • Venous hum.
• Rupture of sinus of Valsalva to right ventricle or
It is a blowing sound produced by turbulent blood flow atrium.
in the heart. The murmur is produced by either normal
volume of blood passing through abnormal valve or Innocent murmur
increased volume of blood passing through a normal Benign, usually soft systolic, present in upper sternal
valve or congenital defect. Murmur may be: edge (commonly pulmonary area), found in some nor-
• Systolic. mal people without heart problem. Not associated with
• Diastolic. thrill and grade of murmur is up to 2/6. (Systolic mur-
• Continuous. mur may be present also in anaemia, thyrotoxicosis and
pregnancy.)
Systolic murmur
Pansystolic murmur (PSM). Its causes are:
Grading of murmur
• MR.
• TR. • Grade 1/6: Very soft murmur, just audible.
• VSD. • Grade 2/6: Soft.
• . Aortopulmonary shunt. • Grade 3/6: Moderately loud.
Ejection systolic murmur (ESM; harsh and high • Grade 4/6: Loud (thrill present).
pitched). Its causes are: • Grade 5/6: Very loud (thrill present).
• Grade 6/6: Very much loud, heard without placing
• AS.
the stethoscope over the chest (thrill present).
• PS.
• HCM. Thrill is present in grades 4-6.
• ASD (due to increased flow through pulmonary
valve).
• Other causes are due to increased flow (in Pericardial Rub
pregnancy, fit athlete). • Superficial, harsh, scratching, creaking, grating and
Late systolic; it is found in mitral valve prolapse and leathery sound.
also in papillary muscle dysfunction. • Present in both systole and diastole.
• Louder with the patient sitting, breathing out and
Diastolic murmur bending forward.
EDM (soft, high pitched, blowing). Its causes are: • Augmented by pressing the stethoscope. It is
present with breath holding.
• AR.
• Pulmonary regurgitation (Graham Steell murmur • Heard in any area of precordium, better heard in
in left second, third, and fourth space).- bare area of heart (part of heart not covered by the
lung, this part is close to chest wall, in left lower
MDM. Its causes are: sternum).
• MS. • Presence of pericardial rub indicates pericarditis
• TS. due to any cause [such as acute MI, viral infection,
• Left atrial myxoma. uraemia and tuberculosis (TB)J.
2 • CARDIOVASCULAR SYSTEM _

Opening Snap found in the neck above the clavicle and upper part of
chest, more on the right side of sternum.
Short, sharp, high-pitched sound heard immediately The hum can be obliterated by pressure on the neck,
after second heart sound (during diastole), produced by or lying down or altering the position of neck (as there
sudden opening of mitral valve due to raised left atrial is reduction of venous obstruction).
pressure (in MS). It is accentuated by sitting with head extended and
Presence of opening snap indicates that the valve turned to the side opposite to that auscultated.
cusps are still mobile. It is absent, if the valve is calci- Venous hum has no clinical significance, commonly
fied. Opening snap is better heard in left lower sternal present in children and should not be confused with
edge. any pathology.

Venous Hum Other Sounds


It is a continuous murmur due to kinking and partial • Tumour plop is found in myxoma.
obstruction of one of the large veins in the neck. It is • Metallicsounds are found in prosthetic metallic valve.

Mitral Stenosis

My diagnosis is MS.
Presentation of
Case No.1 N.B. If the murmur is not audible, ask permission
that the patient should perform physical exer-
• Pulse: 84/min, low volume, and normal in cise. This will increase the heart rate, increase the
rhythm and character.
flow across the mitral valve and murmur will be
• JVP: Normal Ca' wave is prominent, if PH is prominent. However, exercise should be avoided
present, and 'a' wave is absent in AF).
in very ill patient.
• BP:' 100/60 rnml-lg.

On inspection:
Presentation of
• Visible cardiac impulse in mitral area.
Case No.2
On palpation:
Present as above plus.
• Apex beat: In left fifth intercostal space, ... em
• Palpable P2.
from midline and tapping in nature.
• Left parasternal heave.
• Thrill: Present in apical area, diastolic in nature
• Loud P2 on auscultation.
(also see in left lateral position with breath held
after expiration).
On auscultation: My diagnosis is MS with PH.
• First heart sound: Loud in all areas (occasionally
Q: What are the signs of pulmonary hypertension?
only in mitral area and normal in other areas).
A: Signs of pulmonary hypertension are:
• Second heart sound: Normal in all areas [if
pulmonary hypertension (PH), P2 is loud]. • Low-volume pulse.
• There is an MDM in mitral area, which is low • Prominent 'a' wave in JVP.
pitched-localised-rough-rumbling (LLRR),.best • Palpable P2.
heard with bell of stethoscope in left lateral • Left parasternal heave (indicates RVH).
position, breathing held after expiration, with • Epigastric pulsation (indicates RVH).
presystolic accentuation. • Loud P2 on auscultation.
• Opening snap (tell, if present). • EDM (Graham Steell murmur due to pulmonary
regurgitation) .
_ SHORT CASES IN CLINICAL MEDICINE

Q: Why not this is left atrial myxoma?


Presentation of A: In left atrial myxoma, physical signs and murmur
Case No.3 change with posture. Also, there may be history of
fever, weight loss, myalgia, arthralgia, skin rash,
• Pulse: lID/min, irregularly irregular.
Raynaud phenomena (which are absent in this
• Pulsus deficit is present.
case). To be confirmed, 2-D echocardiography
• No 'a' wave in JVP.
should be done.
• No presystolic accentuation.
• Other findings are same as in MS. Q: What are the causes of MS?
A: As follows:
1. Chronic rheumatic heart disease (the commonest
My diagnosis is MS with atrial fibrillation.
cause). In 50% cases, there may be history ofRF
or rheumatic chorea.
2. Other causes are (very rare, do not mention
unless asked):
• Congenital.
• Calcification of valve (usually in elderly).
• Carcinoid syndrome.
(MS is more common in females. F:M = 2:1).

Q: What is Lutembacher syndrome?


A: Combination of ASD with acquired rheumatic MS
(occurs in 4% cases of ASD).

Q: Why apex beat is tapping?


Mitral facies A: It is due to accentuated first heart sound.
Q: What do you think is the cause in this case?
Q: When and why the presystolic accentuation is
A: Chronic rheumatic heart disease.
present?
Q: If this patient is unconscious, what is the likely cause? A: It is due to atrial systole, present in sinus rhythm.
A: Cerebral embolism (usually with right-sided hemi- It occurs due to increased flow across the stenosed
plegia because of lesion in the internal capsule due valve from left atrium to left ventricle causing loud-
to involvement of the lenticulostriate branch of left- ness of murmur. Presystolic accentuation is absent
middle cerebral artery). inAF.

Q: What are the findings if the patient develops CCF? Q: What is mitral facies Or malar flush?
A: Three cardinal signs of CCF are: A: It is the rosy colouration of cheeks, may be bluish
• Engorged and pulsatile neck veins. tinge, due to arteriovenous anastomosis and vascu-
• Enlarged tender liver. lar stasis on the cheeks. It is not pathognomonic
• Dependant oedema. and may be present in normal person, and persons
with hypothyroidism, polycythaemia and PH.
Q: Which disease confuses with MS? (Or, what are the
differential diagnoses of MS?). Q: What are the signs of severe MS?
A: As follows: A: Normal area of mitral valve is 4-6 ern" Severe, if
• Left atrial myxoma (murmur will change with <1 em",
posture, see below).
• Ball-valve thrombus in left atrium. Signs of severe MS
• Pulse: Low volume.
• TS.
• First heart sound: Soft.
Q: Why not this is TS? • MDM: Prolonged.
A: In TS, MDM is prominent in left lower parasternal • Opening snap: Closer to second heart sound.
edge, which increases during inspiration. Also, there • Signs of PH.
may be other features such as raised JVP and signs of • Later, opening snap disappears and MDM quiet
right heart failure. (due to low cardiac output).
2 • CARDIOVASCULAR SYST~M _

Q: What are the mechanisms of pulmonary hyperten- • 2-D echocardiogram and colour Doppler.
sion in MS? • Cardiac catheterisation in some cases.
A: As follows:
Q: What are the findings in chest X-ray in MS?
• Passive backward transmission of raised left atrial
A: Chest X-ray shows:
pressure.
• Upper lobe veins are dilated (early feature):
• Reflex pulmonary artery vasoconstriction.
Upper lobe diversion (normally, ratio between
• Organic obliterative change in pulmonary
upper and lower lobe veins is 1:3, which is
vascular bed.
altered to 1:1).
Q: What are the complications of MS? • Straightening of left border of heart, fullness of
A: As follows: pulmonary conus and filling of pulmonary bay
• Atrial fibrillation. (due to enlarged left atrium).
• Pulmonary oedema. • Kerley B-lines (horizontal septal lines in
• PH leading to CCF. costophrenic angle indicates PH).
• Pulmonary infarction. • Double shadow in right border of heart (due to
• Systemic embolism: Commonly cerebral enlarged left atrium).
(cerebral infarction with hemiplegia), also in • Widening of carina.
mesenteric, renal and peripheral. • Left bronchus is horizontal (due to enlarged left
• Haernoptysis. atrium).
• Ortner syndrome (enlarged left atrium exerts • Pulmonary oederna.
pressure on left recurrent laryngeal nerve and • Mottling or reticulonodular shadow due to
causes hoarseness of voice). pulmonary haernosiderosis.
• Dysphagia due to enlarged left atrium. • Calcified shadow of mitral valve.
• Interstitial lung disease due to prolonged Q: What are the echocardiogram findings in MS?
pulmonary oedema. A: As follows:
• Chest pain in 10% cases (due to PH). • Thick mitral valve leaflet (with restricted opening),
• Infective endocarditis (very rare). diastolic doming of anterior mitral leaflet and
Q: If the patient suddenly becomes unconscious, what restricted movement of posterior mitral leaflet.
is the likely cause? • Reduction of valvular area (narrow): Button-like
A: Cerebrovascular disease [(CVD) cerebral infarc- or funnel shaped.
tion [. usually with right sided hemiplegia. Usually • Calcification of valves (increased echogenicity).
CVD occurs when there is associated atrial fibrilla- • Shortening of chordae tendineae.
tion (AF). • Enlarged left atrium.
• Characteristic 'M' -shape of movement of anterior
Q: What may be the cause of CVD in this case? leaflet normally seen in diastole is lost and the
A: Cerebral embolism (involving lenticulostriate diastolic slope (EF) is reduced.
branch of the left middle cerebral artery causing • Thrombus may be seen.
infarction of the internal capsule).
Q: How to treat MS medically?
Q: Why syncope may occur in MS? A: As follows:
A: Due to reduction of cardiac output. Also may be due • Restrictive activity.
to atrial fibrillation with fast ventricular rate, PH, • Anticoagulant to reduce the risk of embolism.
pulmonary embolism, ball valve thrombus, cerebral • If atrial fibrillation: Digoxin, p-blocker, rate-limiting
embolism. calcium antagonist (e.g. verapamil, diltiazem).
• If there is CCF: Diuretics, digoxin.
Q: What is the cause ofhaemoptysis in MS? • Infective endocarditis is very unusual in MS.
A: Rupture of pulmonary or bronchial veins associated So, routine prophylaxis with antibiotic is not
with PH (pulmonary apoplexy). Also, haemoptysis recommended.
may be due to pulmonary infarction.
Q: What are the indications of anticoagulant (warfarin)
Q: What investigations do you suggest? inMS?
A: As follows: A: As follows:
• Chest X-ray. • Systemic and pulmonary embolism.
• ECG: P is bifid (Pvrnitrale). RVH and RAH. • Atrial fibrillation.
_ SHORT CASES IN CLINICAL MEDICINE

• Left atrial thrombus. • Correction of nutrition.


• Left ventricular systolic dysfunction. • If severe, symptomatic and tight MS: Mitral
valvotomy may be done (usually in middle
Q: What are the indications of surgery in MS? trimester),
A: As follows:
• All patients should go into full term and
• Symptomatic moderate or severe MS when Caesarean section should be done.
balloon valvuloplasty is unavailable. • Advice the patient to restrict number of pregnancy
• Moderate or severe MS with moderate or (1-2).
severe MR.
• Recurrent thromboembolism. N.B. Symptoms of MS are usually more marked
• Episodes of pulmonary oedema without preci- in second trimester, which is due to increase
pitating cause. in blood volume that increases pulmonary
• Associated atrial fibrillation, which does not pressure. The symptoms improve in third
respond to medical therapy. trimester due to decrease in blood volume.
• PH or recurrent haemoptysis.
• Occasionally in pregnancy, with pulmonary
Q: What is myxoma of the heart? What are the features?
oedema (surgery may be done in second trimester
as blood volume 'increases significantly with How to investigate and treat?
A: It is the common primary tumour of heart, usually
increased pulmonary pressure).
benign, may be pedunculated, polypoid, gelatinous,
Q: What surgery is usually done? attached by a pedicle to the atrial septum. It may be
A: As follows: sporadic and familial. It occurs at any age (third-
• Valvotomy [closed mitral commissurotomy sixth decade oflife) and any sex (more in female).
(CMC), open mitral commissurotomy (OMC)].
Sites of origin:
• Valvuloplasty (percutaneous balloon mitral
valvuloplasty): Treatment of choice. • Left atrium (75%), near the fossa ovalis or its
• Valve replacement. margin.
• Right atrium, rarely from ventricles.
Q: What are the criteria for valvuloplasty?
A: As follows: Clinical features: There are three groups of
• Significant symptoms. manifestations.
• Pure MS. 1. Obstructive features: Such as MS, signs vary with
• No or trivial MR. posture. Occasionally, there is a low-pitched
• Valve: Mobile, no calcification. sound called tumour plop, There may be syncope
• Left atrium: No thrombus. or vertigo.
2. Embolic features: Either systemic or pulmonary
Q: What are the indications of valve replacement?
embolism.
A: As follows:
• Associated MR. 3. Constitutional features: Such as fever, malaise,
• If the valve is calcified and rigid. weakness, loss of weight, myalgia, arthralgia,
• Thrombus in left atrium despite anticoagulation. clubbing, skin rash, Raynaud phenomenon.

Q: What are the complications of surgery? Investigations:


A: As follows: • CBC: Anaemia, leucocytosis, polycythaernia,
• MR. high ESR, thrornbocytopaenia or thrombocytosis
• Thromboembolism.
• Hypergammaglobulinaemia.
• Restenosis.
• Chest X-ray (may be similar to MS).
Q: What is the contraindication of surgery in MS? • Echocardiograrn: 2-0 or transoesophageal.
A: Active rheumatic carditis.
• CT scan or MRI may be done.
Q: How to treat MS in pregnancy? Treatment: Surgical excision. Recurrence may occur.
A: As follows:
• Bed rest. N.B. Other tumours of the heart are rhabdomyoma
• Correction of anaemia. and sarcoma. All are rare.
2 • CARDIOVASCULAR SYSTEM _

Mitral Regurgitation
Usual instructions are: • Raised JVP with prominent V-wave.
• Examine the precordium. Or, palpate and auscul- • Liver is enlarged, tender and pulsatile.
tate the precordium. Q: Why not VSD?
A: Because in VSD:
• Systolic thrill in left parasternal area (fourth or
Presentation of a Case :1---------, fifth space).
• PSM in left parasternal area (fourth or fifth
• Pulse: 80/min, normal volume, rhythm and
space). No radiation of murmur to axilla.
character.
• JYP: Normal. Q: What are the causes of MR?
• BP: 120/70 mmHg. A: As follows:
On inspection: • Chronic rheumatic heart disease (rheumatic MR
is more common in male).
• Visible cardiac impulse in mitral area.
• Mitral valve prolapse.
On palpation: • Papillary muscle dysfunction (due to acute
• Apex beat in left ... intercostal space, ... cm from inferior myocardial infarction).
midline, diffuse, thrusting in character. • Rupture of chordae tendineae (due to infarction,
• Systolic thrill in left ... intercostal space. subacute bacterial endocarditis, trauma or
spontaneous).
On auscultation:
• Infective endocarditis.
• First heart sound: Soft in mitral area, normal in • Valvotomy or valvuloplasty.
other areas. • Connective tissue diseases (RA and SLE).
• Second heart sound: Normal in all the areas • Ankylosing spondylitis.
(third heart sound: may be present). • Cardiomyopathy.
• There is a PSM in mitral area, which radiates to • Secondary to LV dilatation (hypertension, aortic
the left axilla (PSM is reduced on inspiration and valve disease).
more in expiration). • Associated with Marfan syndrome, pseudoxan-
thoma elasticum and Ehlers-Danlos syndrome.

My diagnosis is MR. Q: What are the causes of acute MR?


A: As follows:
Q: Can there be any MDM in MR?
• Trauma or surgery (valvotomy).
A: Yes, it may be present due to increased flow of blood
• SBE (due to perforation of mitral valve leaflet or
through mitral valve (or if associated with MS).
chordae).
Q: What are the causes of pansystolic murmur? • Acute MJ (due to rupture of papillary muscle).
A: As follows: • Acute rheumatic fever (RF) (due to mitral
• MR. valvulitis) .
• TR. • Spontaneous rupture of chorda tendineae or
• VSD. myxomatous degeneration of valve.

Q: What are your differential diagnoses? Q: Where does the murmur radiate following rupture
A: As follows: of chorda tendineae?
• TR.
A: As follows:
• VSD. • Rupture of anterior leaflet of chorda tendineae,
murmur radiates to axilla and back.
Q: Why not this is tricuspid regurgitation (TR).?
• Rupture of posterior leaflet of chorda tendineae,
A: Because in TR:
murmur radiates to cardiac base and carotid
• PSM is in left lower parasternal area. arteries.
• No radiation to axilla.
• Murmur is prominent on inspiration and less on Q: What are the complications of MR?
expiration. A: As follows:
_ SHORT CASES IN CLINICAL MEDICINE

• Acute LVE • Presence of third heart sound.


• Infective endocarditis. • Appearance of short MOM (due to rapid filling
• Systemic embolism (less than MS). of left ventricle).
• Atrial fibrillation.
Q: How to treat MR?
• CCF.
A: As follows:
Q: What investigations do you suggest? 1. Mild to moderate case, give symptomatic treat-
A: As follows: ment:
• Chest X-ray (heart size is enlarged in transverse • Diuretics.
diameter TO). • Vasodilators (ACE inhibitor may be used).
• ECG (LYH). • Follow-up every 6 months byechocardiogram.
• 2-D echocardiogram and colour Doppler.
2. Severe or progressive MR or if the ejection frac-
• Cardiac catheterisation, in some cases.
tion falls to 55% and left ventricular dilatation
Q: What are the signs of severe MR? is > 60 mrn, then valve replacement is needed.
A: As follows: If papillary muscle rupture or chorda tend-
• Enlarged left ventricle (apex is shifted and ineae rupture in SBE, early valve replacement is
thrusting). suggested.

Mitral Valve Prolapse


(Barlow Syndrome or Floppy Mitral Valve)

Usual instructions are: • Asymptomatic. It may be present in healthy


women in up to 10% cases. Symptoms increases
• Examine the precordium. Or, palpate and
with aging.
auscultate the precordium. • The commonest symptom is atypical chest
pain, usually in left submammary region and
stabbing in quality (due to abnormal ventricular
Presentation of a Case if---------, contraction or atrial or ventricular dysrhythmia).
• Rarely, it may be confused with anginal pain.
Present the case as described for MR plus the
• There may be palpitation, dyspnoea, fatigue,
following:
benign arrhythmia or rarely fatal ventricular
• Mid-systolic click. arrhythmia. Embolic stroke and TIA are rare
• Late-systolic murmur (loudest at the left sternal complications.
edge).
Q: What are the complications and associations of
mitral valve prolapse?
Cardinal signs are (hallmark) mid-systolic click and A: Complications are:
late systolic murmur (loudest at the left sternal edge). • May cause MR.
• Infective endocarditis (rare).
Q: What is mitral valve prolapse? • Embolic stroke and TIA are rare complications.
A: It is also called Barlow syndrome or floppy mitral
valve. In this condition, a mitral valve leaflet (most Associations:
commonly the posterior leaflet) prolapses into the • SLE.
left atrium during ventricular systole. It is one of • Marfan syndrome.
the commonest causes of MR. It may be congeni- • Ehlers-Danlos syndrome.
tal anomaly or due to degenerative myxomatous • Osteogenesis imperfecta.
• Pseudoxanthorna elasticum.
changes.
• HCM or congestive cardiomyopathy.
Q: How the patient presents? • Muscular dystrophy.
A: Mitral valve prolapse is more common in thin, • ASD (20%).
young women, may be familial. • Anorexia nervosa.
2 • CARDIOVASCULAR SYSTEM _

Investigations: • If there is significant MR or AF, anticoagulation is


• ECG: Nonspecific ST- or T-wave change. indicated to prevent thromboembolism (aspirin
• Echocardiography. may be given).
Treatment: • If MR is severe, mitral valve repair or replacement
should be done.
• Asymptomatic patient only needs reassurance,
periodic echocardiography may be done.
• Atypical chest pain and palpitation are treated N.B. Prophylaxis for infective endocarditis in most
with ~-blockers. Other antiarrhythmic drugs may cases is not recommended. Overall prognosis
be needed. is good.

Mitral Stenosis with Mitral Regurgitation


(Mixed Mitral Valve Disease)
Usual instructions are: Q: What are the findings, if MR is predominant?
A: If MR is predominant:
• Examine the precordium. Or, palpate and
auscultate the precordium. • Pulse: Normal volume.
(In mixed valvular lesion, candidate may be asked • Apex beat: Shifted and thrusting in nature.
about dominant lesion.) • First heart. sound: Soft or absent.
• Third heart sound: May be present.

Presentation of l Q: What is the cause of mixed MS and MR?


Case No.1: Predominant MS A: Chronic rheumatic heart disease.

• Pulse: 88/min, low in volume, normal rhythm Presentation of


and character. Case No.2: Predominant MR
• JVP:Normal.
• BP: 115/60 mmHg. Present as in Case no. I, except the following:
• Pulse: Normal in volume.
On inspection:
• Apex beat: Shifted and thrusting in nature.
• Visible cardiac impulse in mitral area. • First heart sound: Soft.
On palpation: • There is also third heart sound (mention if any).
• Apex beat: In left ... intercostal space, ... cm from
midline, and tapping in nature. My diagnosis is MR with MS.
• Thrill: Present in apical area, both systolic and Q: What is the predominant lesion and why?
diastolic.
A: Predominant lesion is MR, for reasons see above.
On auscultation: Q: What happens, if MS is predominant?
• First heart sound, louder in all areas (may be in A: See above.
mitral area, normal in other areas).
Q: What investigations do you suggest?
• Second heart sound, normal in all areas.
A: As follows:
• There is an MDM in mitral area and also a PSM
in mitral area, which radiates to left axilla. • Chest X-ray, PA view.
• There is also opening snap (mention, if any). • ECG.
• 2-D echocardiogram and colour Doppler.
• Cardiac catheterisation, in some cases.
My diagnosis is MS with MR.
N.B. Occasionally, difficulty arises, when, for exam-
Q: What is the predominant lesion and why? ple, the patient has loud first heart sound and
A: Predominant lesion is MS, because: apex beat is also displaced. In such cases,
• Pulse: Low volume. you should say that, 'it is difficult to be sure
• Apex beat: Tapping in nature and not shifted. clinically about the dominant lesion. Echocar-
• First heart sound: Loud. diogram is necessary'.
_ SHORT CASES IN CLINICAL MEDICINE

Q: In which lesion-MR or MS,endocarditis is common? • Mild and asymptomatic: Follow-up


A: Endocarditis is common in MR. is needed.
Q: How to treat mixed MS and MR? • In severe and symptomatic case:Valve replacement
A: As follows: is required.

Aortic Stenosis
Usual instructions are: Q: Why not HeM?
• Examine the precordium. Or, palpate and auscul- A: InHeM, the findings are:
tate the precordium. • Pulse is jerky.
• Prominent 'a' wave in JVP.
Presentation of a Ca.se • Double impulse at the apex (palpable fourth
heart sound due to left atrial hypertrophy).
• Pulse: 76/min, low volume and slow raising,
• Systolic thrill in left lower parasternal area.
normal in rhythm.
• Associated PSM due to MR.
• JVP: Normal.
• BP: 90/80 mmHg (low systolic, normal diastolic • Family history of HeM may be present or there
and narrow pulse pressure). may be history of sudden death in family.

On inspection: N.B. In HeM, echocardiography is very helpful for


• Visible cardiac impulse in mitral area (or nothing
diagnosis. EeG shows LVHand bizarre abnor-
significant) . malities like pseudoinfarction pattern, deep
On palpation: T-wave inversion.
• Apex beat: In left ... intercostal space, ... cm from
midline, heaving in nature. Q: Does the loudness of murmur indicates severity?
• Systolic thrill: Present in aortic area, radiates to A: No, prolongation of murmur indicates severity.
the right side of neck. Loudness of murmur may be associated with mild
On auscultation: stenosis.
• First heart sound: Normal in all the areas.
• Second heart sound: A2 is soft in all areas and Q: What is the type of pulse in AS?
P2 is normal. A: Plateau or anacrotic pulse is slow rising, may be
• There is a harsh ejection systolic murmur in small-volume pulse (pulsus parvus) or late peaking
aortic area, which radiates to right side of neck. (pulsus tardus).
• May be reversed splitting of second heart sound,
and fourth heart sound may be present. Q: Why anginal pain occurs in AS?
A: There is left ventricular hypertrophy, so there
My diagnosis is AS. is more oxygen demand. Also, there is reduced
coronary flow due to less cardiac output (limitation
Q: What are your differential diagnoses?
of duration of diastole).
A: As follows:
• PS.
Q: What are the signs of severe AS?
• HeM.
A: Signs of severe AS are:
Q: Why not PS?
• Pulse is feeble or absent. Slow rising plateau
A: In PS, findings are:
pulse may be present.
• Systolic thrill in pulmonary area.
• Left parasternal lift and epigastric pulsation may • Systolic aortic thrill.
be present (due to RVH). • Absent or soft A2 (or single S2).
• P2 is soft, A2 is normal (wide splitting of the • Harsh, loud, prolonged murmur with late
second heart sound may be present). peaking (soft and short ESM with early peaking
• Ejection systolic murmur in pulmonary area, suggest mild stenosis).
which radiates to the left side of neck (murmur is • Reversed splitting of second heart sound.
more on inspiration). • Presence of fourth heart sound.
• Apex is normal (not heaving as in AS). • Presence of heart fa.ilure or LVF(late sign).
2 • CARDIOVASCULAR SYSTE'M _

N.B. Also remember the following points: Q: What are the types of AS?
• Normal area of aortic valve is 1.5-2 em". It A: There are three types of AS,viz.:
is severe, if the area is <1 em- or valve mean • Valvular (involving valve cusps).
pressure gradient is >50 mmHg. • Subvalvular (membranous diaphragm or fibrous
• Crit.ical AS: If the valve area is <0.7 ern" or ridge just below the aortic valve).
valve pressure gradient is >70 1.111.11 Hg. • Rarely, supravalvular (narrowing in ascending
aorta or fibrous diaphragm just above aortic
Q: What are the complications of aortic stenosis? valve). It may be associated with characteristic
A: As follows: face such as broad forehead, widely set eyes
• LVF. and pointed chin, mental retardation and
• Infective endocarditis (10% cases). hypercalcaemia called William syndrome.
• Sudden death due to ventricular fibrillation.
• Complete heart block (in case of calcification of Q: What is aortic sclerosis? How to differentiate it
aortic valve). from AS?
• Systemic embolism. A: It is a degenerative condition characterised by thick-
ening of aortic valve cusps. It is common in the
Q: What are the causes of aortic stenosis? elderly. This does not produce any obstruction to
A: As follows: the outflow of blood.
• Chronic rheumatic heart disease.
• Bicuspid aortic valve (com mon in male). Features Aortic stenosis Aortic sclerosis
• Calcification in old age. 1. Pulse volume Low, slow raising Normal
• Congenital (in early age). 2. Apex beat Heaving Normal
3. Thrill Systolic No thrill

Cause of aortic stenosis according to age: 4. A2 Absent or soft Normal


5. ESM Present, and radiates Present, usually
• Infants, children and adolescents:
to the neck no Radiation
o Congenital aortic stenosis.
o Congenital subvalvular aortic stenosis.
o Congenital supravalvular aortic stenosis.
N.B. Risk of aortic sclerosis: There may be calcifica-
tion leading to AS.There are some risk factors
• Young adults to middle-aged: like hyperlipidaemia, diabetes mellitus, smok-
o Calcification and fibrosis of congenitally
ing, hypertension, which make a person prone
bicuspid aortic stenosis. to develop aortic calcification.
• Middle-aged to elderly:
o Senile degenerative aortic stenosis. Q: What are the presentations of aortic stenosis?
o Calcification of bicuspid valve. A: As follows:
o Rheumatic aortic stenosis. • Asymptomatic in mild case.
• Breathlessness, mostly on exertion.
Rememb~r the following points in elderly
• Palpitation.
patient: • Syncope during effort (due to inadequate cardiac
• AS is the most common form of valve disease output or reflex vasodilatation after exercise or
in old age. dysrhythmia, causing cerebral hypoperfusion).
• It is a common cause of syncope, angina and • Angina (in 50% cases with or without coronary
heart failure in this age group. artery disease).
• Low pulse pressure and slow rising pulse may • Sudden death (probably due to ventricular
not be present due to stiffening of the arteries. fibrillation).
• Surgery can be successful in the absence of • In elderly may be associated with complete heart
comorbidity, but operative mortality is higher. block or LBBB.
Prognosis is poor in symptomatic patients
Q: What investigations do you suggest in AS?
without surgery.
A: As follows:
• Biological valve is more preferable than
• X-ray chest (may be normal in early case.
mechanical valves. Anticoagulant is not
Enlarged left ventricle and dilated ascending
needed.
aorta, calcification of valve on lateral view).
• ECG (LVH,may be LBBB,complete AVblock due with calcific AS (valve replacement is necessary in
to calcification of ring may be found). such cases).
• Echocardiogram, preferably colour Doppler • In children, elderly or pregnancy: Valvotomy
echo. may be done.
• Cardiac catheterization (mainly to identify • Anticoagulant is only necessary if there is
associated coronary artery disease. Also to associated atrial fibrillation or mechanical valve
measure the gradient between left ventricle and prosthesis is used.
aorta).
Q: Which prosthetic valve is preferred in elderly
Q: What is the role of ETI' in AS? patient?
A: EIT should be avoided in symptomatic patient with A: Usually biological or tissue valve is preferred than
AS, as it may be fatal. However, it can be done in mechanical one, as biological valve does not require
asymptomatic cases with high-grade AS. £IT may be anticoagulation. In younger age, mechanical valve is
helpful in deciding the role of surgery. preferred, but anticoagulant should be given.
Q: How to treat aortic stenosis? Q: What are the indications of surgery?
A: As follows: A: As follows:
• In mild or asymptomatic cases and if valvular • All symptomatic patients (such as syncope).
pressure gradient <50 mmHg: Follow-up • If mean systolic pressure gradient is >50 mmHg
(periodic echocardiogram should be done). (left ventricular systolic pressure> aorta).
Conservative symptomatic management is given. • If the valve area is <0.7 em? (normal: 2.5-3 ern").
• If symptomatic or even single syncopal attack: • Asymptomatic patient undergoing surgery for
Immediate valve replacement. coronary disease, other valve, LV dysfunction,
• In asymptomatic patient with severe AS and a progressive decline in LVEF,marked LVH.
deteriorating ECG: Valve replacement is also • Abnormal BP in response to exercise.
recommended.
• VT.
• If the patient is unfit for surgery, percutaneous
valvuloplasty may be attempted. Q: If the patient with AS has bleeding per rectum, what
• Aortic balloon valvuloplasty is useful in is the likely underlying cause?
congenital AS. But this is of no value in old age A: Angiodysplasia of the colon (Heyde syndrome).

Aortic Regurgitation

Usual instructions are:


• Thrill: Present in left parasternal area, diastolic
• Examine the precordium. Or, palpate and auscul-
in nature (patient sitting and bending forward).
tate the precordium.
On auscultation:
Presentation of a Case • First heart sound: Normal in all the areas.
• Second heart sound: A2 is absent and P2 is
• Pulse: 92/min, high volume, collapsing type, normal.
normal in rhythm. • There is an EOM which is high pitched, blowing,
• IVP: Normal. best heard in left lower parasternal area in third
• There is dancing carotid pulse in the neck or fourth intercostal space, with patient bending
(Corrigan sign). forward and breathing held after expiration.
• BP: 180/55 mmHg (high systolic, low diastolic
and wide pulse pressure).
On inspection: N.B. Mention, if the following findings are present:
T • Ejection systolic murmur in aortic area, which
• Visible cardiac impulse (mayor may not be).
radiates to right side of neck.
On palpation: • An MOM called Austin Flint murmur.
• Apex beat: In left ... intercostal space, ... cm from • Duroziez murmur (over the femoral artery).
midline, thrusting in nature. • Capillary pulsation (seen in nail bed, inner side
, of lip, fundus during ophthalmoscopy).
My diagnosis is aortic regurgitation. • Duroziez sign: Usually diastolic (may be systolic)
murmur over femoral artery on gradual compres-
Q: What is your differential diagnosis?
sion of the artery and auscultated proximally,
A: Pulmonary regurgitation (PR).
• Pistol shot murmur: May be heard over femoral
Q: How to differentiate AR from PR? artery (Traube sign),
A: In PR findings are: • Hill sign: Higher BP in legs than arms (systolic)
• Early diastolic murmur in pulmonary area and indicates severe AR.
(called Graham Steel murmur), which is more • Mueller sign: Pulsation in uvula with heart beat.
prominent on inspiration. (Most of the signs are rare and unhelpful.)
• Evidence of PH may be present (e.g. palpable P2,
Q: What will you look for in the eyes and mouth in AR?
left parasternal heave, epigastric pulsation, etc.).
A: As follows:
N.B. PR is rare as an isolated phenomenon, usually • Eye: Argyll Robertson pupil (syphilis), dislocated
associated with pulmonary artery dilatation due lens, irregular pupils and iridodonesis (Marfan
to PH, secondary to other diseases (e.g. MS). syndrome),
• Mouth: High-arched palate (Marfan syndrome).
Q: Mention one investigation to confirm your diagnosis. Evidence of ankylosing spondylitis and rheumatoid
A: Echocardiography, preferably colour Doppler. arthritis.
Q: What investigations do you suggest in this case? Q: What are the causes of wide pulse pressure?
A: As follows: A: As follows:
• X-Jay chest (cardiomegaly, dilated ascending
• AR.
aorta, pulmonary oedema).
• PDA.
• ECG (LVH). • AV fistula.
• Echocardiogram, preferably colour Doppler. • Hyperdynamic circulation (thyrotoxicosis,
• Cardiac catheterization. anaemia, beriberi and pregnancy).
• Other investigations to find out the cause
according to the clinical suspicion. Q: What are the causes of EDM?
A: As follows:
Q: Why is it called Water hammer pulse?
• AR.
A: The name was originated from a Victorian toy, 'Con- • Pulmonary regurgitation (signs of PH are present
sisted of a sealed tube, half filled with water and and other peripheral signs of AR are absent).
half being vacuum, Inversion of the tube causes the
fluid to fall rapidly without air resistance and strike Q: Why ESM in AR?
the other end with a noise like hammer blow'. A: Due to increased flow through aortic valve without
AS (or may be associated with AS).
N!_B. To diagnose AR, remember the formula of three:
Q: Why MDM in AR (Austin Flint murmur)?
• Three pulse: Collapsing or water hammer,
A: Due to regurgitant flow from aortic valve causing
dancing carotid and capillary pulsation.
vibration of anterior leaflet of mitral valve,
• Three BP: High systolic, low diastolic and
wide pulse pressure. Q: Why is this MDM not due to MS?
• Three murmur: EDM, Austin Flint murmur A: In MS, there should be:
and Duroziez murmur, • Tapping apex beat.
• Loud first heart sound.
Q: In AR, what other signs are to be seen (if asked to
• MDM associated with presystolic accentuation,
examine precordium only)?
• Opening snap,
A: As follows:
• BP (see above), Q: What are the signs of severe AR?
• Quincke sign: Capillary pulsation at nail bed A: Signs of severe AR:
(alternate flushing and paleness of skin at the • Prolonged EDM.
root of nail while pressure is applied at the tip of • A2 absent or soft.
nail), It may be normally present and better seen • Presence of left ventricular third heart sound.
with glass slide, • Presence of Austin Flint murmur.
• de Musset sign: Head nodding with each heart • Signs ofLVF.
beat (with each pulse), • Signs of enlarging heart.
_ S110RT CASES IN CLINICAL MEDICINE

Q: What are the causes of angina in AR? Q: How to differentiate between AR of rheumatic ori-
A: As follows: gin and due to other causes?
• Low diastolic BP compromise the coronary A: In rheumatic origin:
perfusion pressure causing angina. • History of rheumatic fever.
• Marked compensatory LYR. • Other valvular lesion, commonly mitral.
• Echocardiogram, if there is thickening and
Q: What are the causes of AR?
shortening of cusps, .fusion of commissure.
A: As follows:
(If AR is due to other causes, there is dilatation of
• Chronic rheumatic heart disease.
aorta or valve ring.)
• Infective endocarditis.
• Syphilitic aortitis. Q: How to differentiate syphilitic AR and rheumatic
• Bicuspid aortic valve. AR?
• Dissecting aneurysm affecting ascending aorta. A: As follows:
• Hypertension (by aortic dilatation).
• Marfan syndrome. Features Syphilitic AR Rheumatic AR
• Seronegative arthritis (ankylosing spondylitis,
1. Age >40 years Early age
Reiter syndrome).
2. History of Syphilis Rheumatic fever
• Rheumatoid arthritis.
• Cystic medial necrosis. 3. EDM In aortic area In left lower
• Congenital. parasternal

Cause of AR according to the site or abnormality: 4. Peripheral signs Usually absent Present

5. Lesion Only AR and Both maybe


1. Due to involvement of valve:
never AS present
• Rheumatic fever.
• Infective endocarditis. 6. Echocardiogram No cusp Cusp involvement
involvement
• Bicuspid aortic valve.
• Trauma. 7. Aorta Dilated, and No calcification
2. Aortic root dilatation: calcification may
• Marfan syndrome. occur
• Dissecting aneurysm affecting ascending aorta.
• Syphilitic aortitis. N.~. Syphilis never causes AS, only aortic
• Hypertension. regurgitation.
• Trauma.
• Seronegative arthritides (ankylosing spond- Q: How to treat AR?
ylitis, Reiter syndrome, psoriatic arthropathy). A: As follows:
1. In asymptomatic moderate to severe AR with
• SLE.
• Rheumatoid arthritis. normal LVfunction: Long-acting nifedipine.
• Pseudoxanthoma elasticum. 2. In symptomatic patient with:
• Osteogenesis imperfecta. • Normal LVfunction: Long-acting nifedipine.
• Rare causes: Appetite suppressants (like fen- • LV dysfunction: Digitalis, ACE inhibitor,
fluramine, phentermine). diuretic.
• Heart failure: Digitalis, ACE inhibitor, diuretic.
Q: What are the causes of acute AR? 3. In severe case: Valve replacement.
A: As follows:
• Acute bacterial endocarditis. Indications of surgery:
• Acute RF (due to valvulitis). 1. Symptomatic patient.
• Dissecting aneurysm affecting ascending aorta. 2. Asymptomatic patient with:
• Trauma. • LVsystolic dysfunction (EF <50%)
• LVdilatation (LVend systolic dimension >55
N.B. In acute AR,there is soft, short, early diastolic mm or LVend diastolic dimension >75 mm).
murmur with diastolic thrill. Most patients • Aortic root dilatation >50 mm.
have heart failure. Peripheral signs and car-
diomegaly are absent. It should be treated by N.B. The valve should be replaced before signifi-
emergency surgery. I cant left ventricular dysfunction occurs.
____~ 2. CARDIOVASCULAR SYSTEM 1IIIIIIII
Aortic Stenosis with Aortic Regurgitation
(Mixed Aortic Valve Disease)

UsuaJ instructions are: Q: What is your differential diagnosis?


• Examine the precordium. Or, palpate and auscul- A: There is combined systolic and diastolic murmur.
tate the precordium. So this may be confused with conditions that pre-
sent with continuous murmur like:
• PDA (murmur is called machinery murmur or
Presentation of a Case
train in a tunnel)
(Predominant AS): Case No. 1
• Pulmonary arteriovenous fistula.
• Pulse: IOO/min, low volume and slow rising, • PS with pulmonary regurgitation.
normal in rhythm, pulsus Bisferiens is present Q: What is the likely cause?
(in carotid). A: As follows:
• NP: Normal. • Chronic rheumatic heart disease.
• BP: Low systolic and normal diastolic, narrow • Congenital bicuspid aortic valve.
pulse pressure.
Q: Could it be due to syphilis?
On inspection: A: No, syphilis never causes AS.
• Visible cardiac impulse (mayor may not be). Q: How to treat the case?
On palpation: A: As follows:
• Apex beat in left ... intercostal space, ... cm from • In mild-to-moderate case, follow-up and
midline, heaving in nature. prophylactic penicillin to prevent endocarditis
• Systolic thrill: Present in aortic area, radiates to are suggested.
the right side of neck. • In severe case, valve replacement is required.

On auscultation: Presentation of a Case


• First heart sound: Normal in all the areas. (Predominant AR): Case No.2
• Second heart sound: A2 is soft or absent and P2
is normal. Present as in Case no. I except:
• There is an ejection systolic murmur in aortic • Pulse: High volume and collapsing in nature.
area, which radiates to the right side of neck and • SP: High systolic, low diastolic and wide pulse
also there is an EDM in the left lower parasternal pressure.
area. • Apex beat: Shifted and thrusting in nature.

My diagnosis is aortic regurgitation with AS.


My diagnosis is AS with aortic regurgitation.
Q: Which one is predominant?
A: AR is predominant (see above).
Q: Which one is the predominant lesion and why?
A: Predominant lesion is AS because: Q: Could it be purely ARwithout stenosis?
• Pulse: Low volume and slow rising. A: Yes, ESM may be found in ARdue to increased flow
• BP: Low systolic, normal diastolic and narrow through aortic valve without AS.
pulse pressure.
Q: What investigations do you suggest?
• Apex beat is heaving.
A: As follows:
• Systolic thrill in aortic area.
• X-ray chest.
• A1 is absent.
• ECG (LVIi).
• Echocardiogram, preferably colour Doppler.
Q: What are the findings, if AR is predominant!
• Cardiac catheterization.
A: If AR is predominant, then:
• Pulse: High volume and collapsing. Q: How to treat the case?
• .Apex beat: Shifted and thrusting. A: As follows:
• BP: High systolic, low diastolic and wide pulse • In mild-to-moderate case: Follow-up.
pressure. • In severe case: Valve replacement.
_ SHORT CASES IN CLINICAL MEDICINE

Tricuspid Regurgitation (TR)

It is unusual to get pure TR. Often there is an associa- Q: What are the differences between TR and MR?
tion with other valvular lesion (e.g. MS with TR or MS A: As follows:
and MR with TR). Whenever you get tv).Sor MR look
carefully to find any evidence ofTR. MR ffi
1. PSM in mitral area PSM in left lower
Usual instructions are:
parasternal area
• Examine the precordium. Or, palpate and auscul-
tate the precordium. 2. PSM radiate to left axilla No radiation

3. Murmur increases with Murmur increases with


Presentation of a Case: :I-----~ expiration inspiration
Pure TR 4. No pulsatile liver Pulsatile liver

5. No V-wave in JVP V-wave in JVP


• Pulse: 74/min, normal in volume, rhythm and
character.
• JVP: Raised, there is giant V-wave oscillating up Q: What are the causes ofTR?
to ear lobule. A: As follows:
On inspection: • Functional (PH, cor pulmonale, right heart
failure).
• There may be nothing significant.
• Chronic rheumatic heart disease (usually
On palpation: associated with mitral or aortic valve disease).
• Apex beat: In left ... intercostal space, ... cm • Infective endocarditis (right heart commonly
from midline, normal character. involved in drug addicts).
• Thrill: Absent. • Congenital heart disease (Ebste.in anomaly).
• Left parasternal lift and epigastric pulsation are • Carcinoid syndrome.
present. Right ventricular papillary muscle infarction, trauma
On auscultation: or steering wheel injury in chest.
• first heart sound: Soft in tricuspid area, and Q: What are the causes of pulsatile liver?
normal in other areas. A: As follows:
• Second sound: Normal in all the areas. • TR (the commonest cause).
• There is a PSM in left lower parasternal area, no • Arteriovenous (AV) fistula.
radiation and louder with inspiration. • Angioma of liver (rare).
• MOM may be present, louder with inspiration, • Presystolic pulsation in PS.
due to increased flow through tricuspid valve.
Q: What is the commonest cause ofTR? How to treat
TR?
My diagnosis is TR. A: Functional (secondary to dilatation of right ventri-
cle in PH, cor pulmonale or right heart failure).
Q: What else do you want to see?
A: Liver: Enlarged, tender and pulsatile (Occasionally Treatment:
ascites, oedema and pleural effusion may occur in • Treatment of primary cause.
TR; right nipple may dance with heart beat. In leg, • In severe organic TR operative repair
dilated pulsatile veins may be present rarely). (annuloplasry or pIication) is suggested.
Occasionally, valve replacement is needed.
N.B. Cardinal findings in TR:
Q: What is Ebstein anomaly?
• Prominent V-wave in JVP.
A: It is a congenital heart disease associated with
• PSM in left lower parasternal area, louder
downward displacement of tricuspid valve into
with inspiration (Carvallo sign) and reduce
the right ventricle. Hence, right atrium is large and
in expiration or during Valsalva manoeuvre.
right ventricle is small. Characteristically, multiple
• Liver: Enlarged, tender and pulsatile.
clicks occur due to asynchronous closure of tricus-
Q: What is your differential diagnosis? pid valve. ASO is commonly associated with this
A: MR. anomaly.
2 • CARDIOVASCULAR SYSTEM _

Pulmonary Stenosis (PS)


Usual instructions are: Q: What are the signs of severe PS?
• Examine the precordium. Or, palpate and auscul- A: Signs of severe PS:
tate the precordium. • P2 is absent; and there is wide splitting of second
heart sound.

Presentation of a Case • Murmur is prolonged, loud and harsh.


• Left parasternal lift (RVH).
(Usually, the patient is an adult or elderly)
• ECG shows RVH and RAH.
• Pulse: 70/min, normal in volume, rhythm and
• Chest X-ray shows poststenotic dilatation of
character.
pulmonary artery.
• JVP: Normal (may be raised with prominent 'a'
wave due to RAH).
Q: What are the clinical presentations in PS?
A: As follows:
On inspection:
• May be asymptomatic.
• Nothing significant.
• Symptoms such as fatigue, weakness and effort
On palpation: syncope may occur.
• Apex beat: Palpable in left ... intercostal space, • Other features include cyanosis, right ventricular
. . . em from midline. failure and dysrhythmia (AF).
• Left parasternal lift and epigastric pulsation (due
to RVH). Q: What are the types of PS?
A: There are three types, viz.:
• Systolic thrill: Present in pulmonary area.
• Valvular.
On auscultation: • Subvalvular.
• First heart sound: Normal in all the areas. • Supravalvular.
• Second heart sound: P2 is soft in pulmonary area
and A2 is normal (wide splitting of the second Q: What are the complications of PS?
sound may be present). A: As follows:
• There is a harsh ejection systolic murmur in • Right heart failure.
pulmonary area, which radiates to the neck • Pulmonary embol ism (no systemic).
(more on inspiration). Murmur may be preceded
by ejection click. N.B. Infective endocarditis is unusual in PS.
• Fourth heart sound may be present (due to right Prophylactic antibiotic is unnecessary.
atrial contraction).
Q: What are the radiological findings in PS?
A: ltshows:
My diagnosis is PS. • Enlarged pulmonary conus, with poststenotic
dilatation (in valvular stenosis).
Q: What are the causes of PS? • Oligaemic lung fields.
A: As follows:
• Congenital (the commonest cause). Q: How to treat PS?
• Associated with Carcinoid syndrome. A: As follows:
• Associated with Noonan syndrome. • In mild case, compatible with normal life (no
• Associated with Fallot tetralogy. specific treatment).
• Rarely, in chronic rheumatic heart disease. • In severe symptomatic case, when the pressure
gradient is >50 mmHg, balloon valvuloplasty is
N.B. PS may occur, if there is rubella in pregnancy. the treatment of choice.
_ SHORT CASES IN CLINICAL MEDICINE

Prosthetic Heart Valves


(Remember: There is a vertical midsternal scar mark. N.B. Tissue valve-two types:
Think of either valve replacement or CABG. In metallic • Xenograft: Made from porcine valve or bovine
valve replacement. there is metallic sound on ausculta- pericardium. These are less durable and may
tion; and in tissue valve replacement, and there is dick.) require replacement after 8-10 years. Antico-
agulation is not required unless there is AP.
Usual instructions are:
• Homograft: These are cadaveric valves (aortic
• Examine the precordium. Or, palpate and auscul- or pulmonary human valve). They are
tate the precordium. particularly useful in young patient and in
Presentation of a Case: the replacement of infected valves.
• Present the case starting from pulse, JVP as Q: How to detect that the patient has prosthetic valves?
described before.
A: It is detected in the following ways:
Add the following points: • Usually there is a vertical midsternal scar mark of
thoracotomy. (It may also be due to CABG.)
Case No.1 I
I
• In metallic valve replacement, there is metallic
sound on auscultation. However, modem
• There is a midstemal scar. mechanical valve (St. Jude valve) makes softer
• On auscultation, a metallic sound in mitral area opening and closing sound than older valves.
that coincides with first heart sound. • In case of tissue valve prosthesis, there is no
• Second sound is normal. metallic sound or plopping sound. Rather, there
• There is a metallic opening snap. is a dick.

Q: How to detect clinically whether the replaced valve


My diagnosis is metallic mitral valve prosthesis. is mitral or aortic?
A: As follows:
Case No.2 I
1. Mitral valve prosthesis is detected by:
• Site: Over the mitral valve.
• There is a midsternal scar. • Metallic sound is present, coincides with first
• On auscultation, there is a metallic sound in heart sound (sharp closing).
aortic area that coincides with the second heart • Normal second heart sound.
sound. • Diastolic flow murmur (MDM) may be
• First heart sound is normal. present normally.
• There is a metallic opening snap. • Sharp opening or closing sound or click that
coincides with the carotid pulse.
2. Aortic valve prosthesis is detected by:
My diagnosis is metallic aortic valve prosthesis. • Site: Over the aortic valve.
• Metallic sound is present, coincides with
Q: What happens, if there is tissue valve?
second heart sound (sharp closing).
A: In tissue valve prosthesis, no metallic sound or • Normal first heart sound. Ejection systolic
plopping sound is present. The click is present. murmur (ESM) may be present normally.
• Sharp opening or closing sound or click that
Q: What are the types of prosthetic valve?
occurs shortly after the carotid pulse.
A: Prosthetic valves are of two types:
1. Metallic valves: N.B. Mostly Starr-Edwards valve is used.
• Starr-Edwards valve (ball-cage valve).
• Bjork-Shiley valve (tilting disc). Q: How to detect, if prosthetic valve is leaking?
• St. Jude valve (bi-leaflet, double tilting disc). A: As follows:
2. Tissue valves: • In mitral valve prosthesis: Appearance of PSM
• Carpentier-Edwards valve. indicates leaking (MDM may be normally
• Hancock porcine valve. present. Ball-cage valves project into the left
• Ionescu-Shiley valve (less used). ventricle and cause a low-intensity ESM. Tissue
2 • CARDIOVASCULAR SYSrE. _

valve and bi-leaflet valve can have a low-intensity Q: What are the complications of prosthetic valve?
diastolic murmur. Consider any systolic murmur A: As follows:
of loud intensity to be a sign of regurgitation and • Thromboembolism: More on metallic valve
failure of the valve.). (common in mitral than aortic). Anticoagulant
• In aortic valve prosthesis: Appearance of EOM is necessary (INR should be between 3 and 4.5).
indicates leaking. (ESM may be normally present • Primary valve failure: Rare in metallic valve
as all types of valves produce a degree of outflow (common in tissue valve).
obstruction, and thus have an £SM.) • Valve leaking.
Q: What happens, if there is dysfunction of prosthetic • Dehiscence or detachment of valve from the site
or valve ring resulting in paraprosthetic leak
valve?
A: Absence of opening and clicking or closing sounds • Valve obstruction by thrombosis or calcification.
indicates dysfunction. Unexplained heart failure • Mechanical or microangiopathic haemolytic
anaemia (mainly in aortic valve in 10-20% in 10
may be due to dysfunction. Biological valve dysfunc-
tion is usually associated with regurgitant murmur. years. Occurs due to metallic valve).
• In tissue valve, there may be perforation, rupture
Q: What are the advantages and disadvantages of and degenerative changes due to calcium
different valves? deposition.
A: As follows: • Infective endocarditis, especially in dental
1. In case of metallic valve: procedure or catheterisation. Common organism is
• Advantage: Incidence of valve failure is less Staphylococcus epidetmidis. Occasionally, treatment
and more durable. may be difficult, may cause high mortality and
• Disadvantage: Incidence of thrombosis is may require replacing the valve again. If infection
usually high, requiring long-time anticoagu- occurs within 60 days of valve replacement (early),
lant therapy, even life-long. There may be it is mostly by contamination of intravenous (N)
microangiopathic haemolytic anaemia. cannula; and if the infection occurs after 60 days
2. In case of tissue valve: (late), it is like other valve endocarditis.
• Advantage: Incidence of thrombosis is less;
hence, long-time anticoagulant therapy is not N.B. Remember the following points:
required (short-term anticoagulant is used • Ball-cage valve causes haemolysis more than
in postoperative period. Also, anticoagulant other valve.
is required, if associated with AF). No • Tilting disc is more thrombogenic.
haemolysis.
• Disadvantage: Incidence of valve failure is Q: How to choose a particular valve?
high due to stiffening and later tearing of A: In the following way:
valve leaflets over 10 years, and requires • In young patients, if no contraindication for
repeat valve replacement and is less durable. anticoagulant therapy, metallic valve prosthesis
There is degeneration and calcification in is preferred.
advanced stage. • In elderly patient or if there is contraindication
to anticoagulant therapy, tissue valve prosthesis
Q: What are the complications of metallic valve?
is preferred.
A: As follows:
• Thromboembolism. Q: Which prosthesis is used in a woman at childbear-
• Anticoagulation is needed, which may lead to
ing age?
bleeding. A: Mechanical valves are now preferred for woman at
• Microangiopathic haemolytic anaemia childbearing age. During pregnancy, warfarin is sub-
• Infective endocarditis. stituted with N unfractionated heparin in the first
Q: What are the complications of tissue valve? 6-] 2 and last 2 weeks. This is associated with a low
A: As follows: rate of warfarin embryopathy and that of bleeding.
• Primary valve failure. Subcutaneous heparin and low-molecular-weight
• Perforation or rupture. heparin are not recommended at present.
• Degenerative changes. Recent studies have showed that women who
• Calcification. need low doses of warfarin (5 mg or less) are at
• Infective endocarditis. low risk for fetal warfarin embryopathy, bleeding,
stillbirth or abortion. In these women, warfarin pregnancy. But with tissue valve, there is increased
may be given throughout pregnancy but should be risk of early structural valve deterioration (SVD)
closely monitored. during or shortly after the end of pregnancy. In
Previously tissue valve was used for young addition, tissue valve is less durable and repeat valve
woman considering the risk of anticoagulation at replacement may be needed after about 10 years.

Congenital Heart Disease

Common congenital heart diseases are described in this • Maternal viral infection, such as rubella in first
chapter. Usual cases selected in the examination are VSD trimester of pregnancy is associated with PDA
(see page 95), ASD (see page 98), PDA (see page 99), pulmonary valvular and arterial stenosis, and ASD.
Fallot tetralogy (see page 93), coarctation of aorta (see • Maternal illness [e.g. SLE,which is associated with
page 106). Others PS (see page 89), TR (see page 88) are congenital complete heart block diabetes and
sometimes selected; for details see the aforementioned phenylketonuria ].
topics on the given pages.
Types of Congenital Heart Diseases
A Brief Note on Congenital Heart Diseases A. Acyanotic congenital heart diseases (communica-
Any defect or malformation in one or more structures of tion between systemic and pulmonary circulation):
the heart or blood vessels that occurs during pregnancy These are of two types, viz.:
is called congenital heart disease. It affects about 1 in With left-to-right shunt
100 babies. This may remain asymptomatic, or symp-
• ASD.
toms may appear after birth, at childhood or in adult.
• VSD.
• PDA.
Causes of Congenital Heart Diseases
With no shunt
Actual cause is unknown; but some factors may increase • Coarctation of aorta.
the risk of congenital heart disease. These are: • Bicuspid aortic valve.
• Genetic abnormality (e.g. Marfan and DiGeorge • Congenital AS.
syndromes). • PS or pulmonary regurgitation.
• Chromosomal abnormalities (e.g. Down syndrome • Ebstein anomaly.
is associated with septal defects, and mitral • Tricuspid valvular disease.
and tricuspid valve defects; Turner syndrome is B. Cyanotic congenital heart diseases:
associated with coarctation of aorta). • TOF.
• Maternal alcohol abuse is associated with septal • Eisenmenger syndrome (PH with right-to-left shunt).
defects. • Transposition of great arteries.
• Maternal drug abuse or drug treatment and • Others: Truncus arteriosus, tricuspid atresia and
radiation exposure (use of thalidomide during total anomalous pulmonary venous drainage.
pregnancy may be associated with amelia or
paramelia). !'l.:B. The commonest congenital heart disease is VSD.

Tetralogy of Fallot (TOF)

Usual instructions are:


Presentation of a Case
• Examine the precordium. Or, palpate and auscul-
tate the precordium. (Usually a child)
• Pulse: 80/min, normal in volume, rhythm and
(Look at the patient carefully. Is there clubbing and character.
central cyanosis?)
2 • CARDIOVASCULAR SYSTEM _

• JVP: Normal (may be prominent 'a' wave due to


RVH).
On inspection:
• The patient may be short, dyspnoeic, cyanosed,
with plethoric face.
On palpation:
• Apex beat: Palpable in left ... intercostal space, ...
cm from midline.
• Left parasternal lift and epigastric pulsation (due
to RVH).
• Systolic thrill: Present in pulmonary area. Tetralogy of Fallot

On auscultation: Q: Mention some cyanotic congenital heart disease.


• First heart sound: Normal in all the areas. A: As follows:
• Second heart sound: P2 is soft or absent in • Tricuspid atresia.
pulmonary area, and A2 is normal. • Transposition of great vessels.
• There is a harsh ejection systolic murmur in • Pulmonary atresia.
pulmonary area, which radiates to the neck, • Ebstein anomaly.
more on inspiration.
Q: What are the cardinal features ofTOF?
A: As follows:
• Child with growth retardation.
My diagnosis is TOF.
• Clubbing.
Q: What is TOF? • Cyanosis.
A: It is a cyanotic congenital heart disease consisting of: • Pulmonary ejection systolic murmur.
• PS. • History of cyanotic spells during exercise (rei ieved
• Overriding and dextroposition of aorta (aortic by squatting).
origin-two-third from left ventricle and one-
Q: How to assess the severity ofTOF?
third from right ventricle).
A: As follows:
• Right ventricular hypertrophy.
• Mild case: Loud and prolonged murmur.
• VSD (peri membranous, usually large, subaortic).
• Severe case: Reduced or no murmur.
N.B. Right ventricular outflow obstruction may be Q: How the patient usually presents?
subvalvular (infundibular), valvular or sup- A: As follows:
ravalvular. The most common obstruction is • Young children usually present with cyanoticspell
subvalvular, either alone (50%) or in combi- (Fallot spell) during exertion, feeding or crying.
nation with PS (25%). They may become apnoeic and unconscious.
• In older children, Fallot spells are uncommon
but cyanosis becomes increasingly apparent
with clubbing and polycythaemia. There may be
Fallot sign.
• Shortness of breath on exertion, easy fatiguability.
• Growth retardation.
• Syncope, seizure, cerebrovascular events or even
sudden death.

Q: Why syncope occurs in TOF during exercise?


A: During exercise, there is increased pulmonary resist-
ance and reduced systemic vascular resistance. Hence,
there is increased right-to-left shunt, admixture of
blood of right and left ventricle. As a result, there is
Normal heart reduced cerebral oxygenation causing syncope.
_ SHORT CASES IN CLINICAL MEDICINE

Q: Why cyanosis in TOP? • If pulmonary artery is hypoplastic or anatomy is


A: Because of overriding of aorta, there is admixture of unfavourable, then temporarily Blalock-Taussig
blood of right and left ventricles. Cyanosis is absent shunt is performed. Corrective surgery is done
in newborn or acyanotic Fallot. later 00.
Other surgical procedure:
Q: When is cyanosis aggravated and why?
A: Cyanosis is aggravated during exercise called • Modified Blalock-Taussig shunt: Interposition of
cyanotic spell or Fallot spell. It is also aggravated tubular graft between subclavian and pulmonary
during feeding or crying. Cyanosis is reduced by artery.
squatting. Child may be apnoeic and unconscious. • Pulmonary balloon valvuloplasty.
Syncope, seizure, cerebrovascular accident (CVA) or • Waterston shunt: Anastomosis of the back of
sudden death may occur. Fallot spell is uncommon the ascending aorta to the pulmonary artery. It
in older children. is performed when surgery is required under the
Cyanotic spell is due to increased obstruction as age of 3 months because the subclavian artery is
a result of increased sympathetic stimulation that very small.
occurs during exercise, feeding and crying. Q: What is Blalock-Taussig shunt?
A: It is the anastomosis between left subclavian artery
Q: How squatting relieves cyanosis? and left pulmonary artery. This improves pulmonary
A: During squatting position, abdominal aorta and blood flow and pulmonary artery development, and
femoral artery are compressed. There is increased may facilitate definitive surgery later on.
arterial resistance, so increase pressure at the left
ventricular level, diminished right-to-left shunt, Q: How to detect Blalock-Taussig shunt at bedside?
increased flow through pulmonary artery, and A: As follows:
thereby reduction of admixture of right and left • Thoracotomy scar.
ventricular blood. • Left radial pulse: Absent or feeble.
• BP on the left arm: Absent or undetectable.
Q: Why no murmur ofVSD in TOF? • Arm on the left side looks smaller than right.
A: Because VSD is large, and there is equal pressure in
Q: How to treat during cyanotic spell?
right and left ventricle.
A: As follows:
• Knee-chest position of child, high concentration
Q: What are the complications ofTOF?
of O;
A: As follows:
• Morphine or diamorphine injection (which
• Infective endocarditis (common, 10% cases).
relaxes right ventricular outflow obstruction).
• Paradoxical emboli.
• ~-blocker.
• Cerebral abscess (10% cases).
• If medical therapy fails, emergency surgical shunt
• Polycythaemia (due to hypoxaemia). It may
may be considered.
cause CVA and myocardial infarction.
'prognosis is good after surgery, especially if operation
• Coagulation abnormality.
is done in childhood. After surgery, restenosis,
recurrence of septal defect and rhythm disorder may
Q: What investigations are done in TOF?
occur. Regular follow-up is required in every case.
A: As follows:
• Chest X-ray: Boot-shaped heart, pulmonary conus
is concave (small pulmonary artery), right ventricle Pentalogy of Fallot
is enlarged (prominent elevated apex), oligaernic
• When TOF is associated with ASD.
lung (right-sided aortic arch in 25% cases).
• 2-D echocardiography and colour Doppler. trilogy of Fallot
• Other investigations: ECG (RVH), cardiac
• ASD with PS with right ventricular hypertrophy.
catheterisation in some cases.
Acyanotlc Fallot
Q: How to treat TOF?
• When TOF is associated with infundibular PS.
A: As follows:
Outflow obstruction is mild and. there is no
• Usually using surgical correction, and ideally
cyanosis.
using total correction (prior to 5 years of age).
2 • CARDIOVASCULAR SYSTEM _

Ventricular Septal Defect (VSD)

On auscultation:
• First and second heart sounds are normal in all
the areas.
• There is a PSM in left parasternal area in fourth
or fifth intercostal space [may be MDM due to
increased flow through mitral valve (Third heart
sound may be present.)].

My diagnosis is VSD.
Q: What are your differential diagnoses?

Normal heart
A: As follows:
• MR.
• TR.

Q: Why not this is a case of MR?


A: In MR findings are:
• First heart sound is soft.
• PSM in mitral area, which radiates towards the
left axilla.

Q: Why not this is a case ofTR?


A: In TR findings are:
• JVP may be raised. There may be giant V-wave,
oscillating up to the ear lobule.
• First heart sound is soft in tricuspid area.
• There is a PSM in left lower parasternal area with
Ventricular septal defect
no radiation and the murmur is louder with
Usual instructions are: inspiration.
• Examine the precordium. Or, palpate and • Also, there may be enlarged, tender, pulsatile liver.
auscultate the precordium.
Q: Mention one investigation to confirm your
diaghosis.
A: Colour Doppler echocardiogram.
Presentation of a Case
Q: What are the causes ofVSD?
• Pulse: 90/min, normal in volume, rhythm and
A: As follows:
character.
• Commonly congenital (VSD is the commonest
• NP: Normal.
congenital heart disease).
• Acquired rupture of interventricular septum after
On inspection:
acute myocardial infarction, rarely trauma.
• Nothing significant.
Q: What is the site ofVSD?
On palpation: A: It is commonly seen in the perimembranous part of
• Apex beat: In left ... intercostal space, ... em from intraventricular septum (in 90% cases).
midline, diffuse, thrusting in character.
• Systolic thrill: Present in left parasternal area Q: Is the loudness of murmur related to size ofVSD?
(fourth or fifth intercostal space). A: Small VSD is associated with loud murmur, and
large defect is associated with soft murmur.
Q: Does the presentation varies with the size ofVSD? 2. Moderate-to-large: Surgical correction is needed
A: VSD may be of three types according to the size. if pulmonary-to-systemic flow ratio> 1.5: 1. Per-
These are: cutaneous transcatheter closure may be done.
3. When Eisenmenger syndrome develops:
• Small VSD (Maladie de Roger): It is asymptomatic
and usually closes spontaneously. But, there is a Surgery is contraindicated, as it aggravates right-
future risk of development of aortic regurgitation sided heart failure. Then following treatments
or endocarditis even after spontaneous closure. are given:
The systolic murmur is loud and prolonged. • Diuretic.
• Moderate VSD: The patient presents with fatigue • Digoxin in some cases.
and dyspnoea. Heart is usually enlarged with a • Venesection, especially if there is
polycythaemia.
prominent apex beat. There is often a palpable
systolic thrill and a loud 'tearing' PSM at the left • Heart-lung transplantation may be done.
lower sternal edge. Mortality rate is very high than heart
transplantation alone.
• Large VSD: The murmur is soft. It may lead to
PH, and Eisenmenger complex may result. N.B. VSD may be associated with Turner syndrome,
Q: What are the complications ofVSD? Down syndrome or maternal rubella during
A: As follows: pregnancy.
• Infective endocarditis (more common in small Q: Can VSD be dosed spontaneously?
VSD). A: If small VSD, spontaneous closer may occur.
• Pulmonary hypertension with reversal of shunt
(Eisenmenger syndrome). Q: What is the impact of pregnancy on VSD?
• Heart failure. A: Small defect does not make any problem with
N.B. When Eisenmenger syndrome develops, there
pregnancy. Moderate-to-large defect with PH may
is cyanosis, clubbing and evidence of PH. get worse during pregnancy and right ventricular
failure may develop. So, pregnancy should be
PSM may disappear because of equalisation
avoided if there is PH.
of pressure in right and left ventricle.

Q: What investigations do you suggest in your case? Eisenmenger Syndrome


A: As follows:
Usual instructions are:
• ECG (LVH, biventricular hypertrophy).
• Examine the precordium. Or, palpate and
• X-ray chest (cardiomegaly, large pulmonary
conus, large hilar arteries, plethoric lung fields). auscultate the precordium.
• Echocardiography, preferably colour Doppler.
• Cardiac catheterization may be necessary in Presentation of a Case
some cases.
• CMR (cardiac magnetic resonance) angiography • Pulse: 104/min, low volume.
may be helpful. • JVP: Raised, with prominent 'a' wave.
Q: What are the causes of plethoric lung field? • BP: 110/70 mmHg.
A: As follows:
Precordium:
• ASD. Inspection:
• VSD.
• PDA. • Visible cardiac impulse in pulmonary area.
• Also in CCF. Palpation:
Q: How to treat VSD? • Apex beat: In the left ... intercostal space, ... cm
A: As follows: from midsternalline.
1. If small: Surgery is not needed; only follow-up • Left parasternal lift: Present.
should be done. Spontaneous closure may occur • Palpable P2: Present.
in infants, if it is in the muscular pa11. Prophy- • Epigastric pulsation: Present.
lactic penicillin for SBEmay be given.
• Syncope.
Auscultation:
• Angina.
• First heart sound: Normal in all the areas. • Haemoptysis.
• Second heart sound: Louder in all the areas, P2 is • Features of CCF.
accentuated in pulmonary area.
On examination:
• There is a PSM in the Jeft third and fourth
intercostal space, in left parasternal area without • Central cyanosis (not corrected by giving 100%
any radiation. oxygen). Differential cyanosis (cyanosis in toes,
not in the hand) occurs in PDA.
• Clubbing (differential clubbing-clubbing in
My diagnosis is Eisenmenger syndrome due to VSD. toes, not in the hand; occurs in PDA).
Q: What are your differential diagnoses? • Pulse: Low volume.
A: As follows: • Prominent 'a' wave in JVP.
• Other signs of PH: Palpable P2, left parasternal
• CCF.
lift, epigastric pulsation due to RVH. Ejection click
• Chronic cor pulmonale.
and ejection systolic murmur may be present.
Q: Why not CCF? • TR may occur (in such case, prominent V-wave
A: In CCF, the triad of engorged and pulsatile neck in TVP; also there may be a PSM in left lower
vein, enlarged tender liver and dependant oedema parasternal area).
should be present. It is usually secondary to other • Polycythaemia.
causes like MS or left-sided heart failure, which are • Original murmur of VSD, ASD or PDAL:
absent in this case. Decreases in intensity, even may disappear.
Q: Why not this is a case of chronic cor pulmonale?
N.B. Remember, if any patient is having cyanosis
A: Cor pulmonale is defined as enlargement of right
with evidences of PH, the more likely diagnosis
ventricle with or without failure, which may be
is Eisenmenger syndrome.
due to causes in the lungs parenchyma, pulmonary
vessels or chest wall (like kyphosis, scoliosis, etc.).
Q: What are the causes of death in Eisenmenger
All of these are absent in th is case.
syndrome?
Q: What investigations do you suggest in this case? A: As follows:
A: As follows:
• Right heart failure.
1. X-ray chest (enlargement of central pulmonary • Pulmonary infarction.
arteries with peripheral pruning of pulmonary • Infective endocarditis.
vessels). • Cerebral thrombosis or abscess.
2. ECG (RVH, RAH, right axis deviation). • Dysrhythrnias.
3. Echocardiography.
Q: How to treat?
Q: What is Eisenmenger syndrome? What are the A: As.follows:
causes?
A: PH with reversal of shunt is called Eisenmenger • Diuretic.
syndrome. Causes are: • Digoxin may be given in some cases.
• Venesection may be required, especially if there
• VSD. is polycythaemia.
• ASD. • Long-term intravenous epoprostenol may be tried.
• PDA. • Heart-lungtransplantation maybe done (mortality
Q: What is Eisenmenger complex? rate is very high than heart transplantation alone).
A: PH with reversal of shunt, when due to VSD is called Surgery is contraindicated in Eisenmenger syn-
Eisenmenger complex. drome, as it aggravates right sided heart failure.
Q: What are the clinical features of Eisenmenger Q: What is the effect of pregnancy in Eisenmenger
syndrome? syndrome?
A: As follows: A: Pregnancy should be avoided as it aggravates right-
• Dyspnoea. sided heart failure, early spontaneous abortion.
• Fatigue. Also, mortality is high in mother.
__ SHORT CASES IN CLINICAL MEDICINE

Atrial Septal Defect (ASD)

Usual instructions are:


• Examine the precordium. Or, palpate and
auscultate the precordium.

Presentation of a Case :1---------, Atrial-....,...;,...__,


septum
• Pulse: 86/min, normal in volume, rhythm and
character. Atrial --+---"
septum
• JVP: Normal.
• SP: 120/75 mmHg.
On inspection:
• Nothing significant.
On palpation: En larged right ventricle

• Apex beat: In left ... intercostal space, ... cm from Atrial septal defect
midline.
• Thrill: Absent.
On auscultation: Q: What are your differential diagnoses?
A: As follows:
• First heart sound is normal in all the areas.
• Wide and fixed splitting of second heart sound • PS.
(very important finding). • VSD.
• There is an ejection systolic murmur in left
Q: Why not this is a case of PS?
second and third intercostal spaces.
A: In PS, the findings are:
• There is (or may be) MDM in tricuspid area.
• Soft or absent P2.
• Thrill present in pulmonary area.
• Wide and fixed splitting of second heart sound
is absent.
My diagnosis is ASD. • ESM may radiate to the neck.

Q: Why not this is a case ofVSD?


A: In VSD, the findings are:
• Systolic thrill in the left lower parasternal area.
• PSM in the left lower parasternal area.

Q: What is wide and fixed splitting of second heart


sound and why is it wide and fixed splitting?
A: Wide and fixed splittings mean that it is remaining
same in inspiration and expiration.
• Wide because of delay in right ventricular
ejection.
• Fixed because of equalisation of pressure between
left and right atrium; hence no change in second
heart sound with respiration (normally, there is
wide splitting during inspiration due to delay of
Normal heart closure of pulmonary valve).
2 • CARDIOVASCULAR SYSTEM _

Q: What are the murmurs in ASD? • Systolic ejection sound is accentuated.


A: The two murmurs are: • Other features include PH.
• ESM, due to increased flow through pulmonary Q: What is Lutembacher syndrome?
valve. A: ASD with an acquired rheumatic MS.
• MDM, due to increased flow through tricuspid
valve. Q: What investigations do you suggest?
(No murmur due to ASD because of equal pressure A: As follows:
between left and right atrium.)
• Chest X-ray: Cardiomegaly (right ventricle and
Q: What are the types of ASD? right atrium are enlarged, also small left atrium
A: They are of two types: and normal aorta), large pulmonary conus, large
hilar arteries, lung fields are plethoric (if PH,
• Ostium primum (10% cases) results from
atrioventricular defect in septum and there is there is oligaemic lung).
involvement of AVvalve; hence there may be MR • ECG.
orTR. • 2-D echocardiography and colour Doppler, but
transoesophageal echo is better.
• Ostium secundum (90% cases) defect mainly at
the fossa ovalis. • Cardiac catheterisation in some cases
(ASDis common in females; M:F ratio is 1:2. Ostium (fluoroscopy shows hilar dance).
primum may occur in Down syndrorne.) • MRI or CMR may be helpful.
Q: How to treat ASD?
Q: What is the ECG finding in ASD?
A: As follows:
A: ECG shows:
• Small ASD: Surgery is not needed, only follow-up
• In primum type, RBBBwith left-axis deviation.
should be done (the patient usually lives a
• In secundum type, RBBBwith right-axis deviation.
normal life).
Q: What are the complications of ASD? • Moderate to large: Surgical closure should be
A: As follows: done, if the ratio of pulmonary flow to systemic
flow is 2:1 or more.
• PH with reversal of, shunt (Eisenmenger
• Angiographic closure is possible with trans-
syndrome).
catheter clamshell device.
• Dysrhythmia (AP, the commonest).
• If Eisenmenger syndrome develops: Surgical
• Embolism (pulmonary and systemic) and brain
closer is contraindicated (for other treatment, see
abscess.
in Eisenmenger syndrome).
Q: What are the findings when there is a reversal of
Q: What is the effect of ASD on pregnancy?
shunt?
A: In uncomplicated ASD, pregnancy is well tolerated.
A: As follows:
But if Eisenmenger syndrome develops, pregnancy
• Both murmurs reduce in intensity. should be avoided because of high maternal and
• P2 is loud. foetal mortality.

Patent Ductus Arteriosus (PDA)

Usual instructions are:


On inspection:
• Examine the precordium. Or, palpate and
• Visible cardiac impulse in apical area and another
auscultate the precordium.
impulse in pulmonary area.
On palpation:
Presentation of a Case
• Apex beat: In left .. intercostal space, ... cm from
• Pulse: 80/min, high volume or collapsing, normal midline, thrusting or heaving in nature.
in rhythm. • Systolic thrill: Present in pulmonary area (may


• )YP. Normal.
BP: 130/70 mmHg.
be diastolic also).
• Pulmonary arterial pulsation may be felt.
_ SHORT CASES IN CLINICAL MEDICINE

N.B. During foetal life, ductus arteriosus connects pul-


On auscultation:
monary artery at its bifurcation to the descending
• First and second heart sounds are normal in all aorta just below the origin ofleft subclavian artery
the areas (may be reverse splitting of second and permits blood flow from pulmonary artery to
heart sound, if large shunt). aorta. After birth, within hours or days, it closes
• There is a continuous murmur in left second spontaneously and remains as ligamentum arte-
and third intercostal space, more prominent in riosum. In PDA, it allows blood to flow from
systole (murmur is prominent on expiration, aorta to pulmonary artery. Up to 50% of left ven-
may be heard posteriorly), and radiates to the tricular output may enter into pulmonary artery
neck. because pressure in aorta is higher.
• There may be MDM (due to increased flow).
Q: What are your differential diagnoses?
A: The typical continuous murmur is highly suggestive
ofPDA. However, any cause of continuous murmur
should be excluded such as:
My diagnosis is PDA.
• Arteriovenous fistula (coronary, pulmonary or
systemic).
• Venous hum.
• Rupture of sinus of Valsalva to the right ventricle
or atrium.
Q: What is venous hum?
A: It is a continuous murmur due to kinking and par-
tial obstruction of one of the large veins in the neck.
It is found in the neck above the clavicle and upper
part of chest, more on the right side of sternum. The
hum can be obliterated by pressure on the neck or
lying down or altering the position of neck (as there
is reduction of venous obstruction). It is accentu-
ated by sitting with head extended and turned to the
side opposite to that auscultated. Venous hum has
no clinical significance, commonly present in chil-
dren, should not be confused with any pathology.
Q: What is the murmur in PDA?
Normal heart A: Continuous murmur (machinery murmur like train
in a tunnel), with late systolic accentuation.
Q: What are the causes of continuous murmur?
A: As follows:
• PDA.
• Arteriovenous fistula (coronary, pulmonary or
systemic).
• Aortopulmonary fistula (may be congenital or
Blalock-Taussig shunt).
• Venous hum.
• Rupture of sinus of Valsalva to the right ventricle
or atrium.
Q: What are the causes ofPDA?
A: Common in females; M:F ratio is 1:3. Probable aeti-
ological factors are:
• Matemal rubella in the first trimester.
• Birth at high altitude with continuous prenatal
hypoxia.
Patent ductus arteriosus • Prematurity.
2 • CARDIOVASCULAR SYSTEM __

Q: What are the findings in reversal of shunt? • 2-D echocardiography and colour Doppler
A: As follows: echocardiography.
• Cyanosis and clubbing in lower limb, absent • Cardiac catheterisation may be necessary in some
in upper limb (called differential cyanosis or cases.
differential clubbing). • Angiography may be done.
• Murmur is quiet or absent or systolic only • MRI or CMR angiography may be helpful.
(diastolic is absent). Q: How to treat PDA?
• Evidence of PH. A: As follows:
Q: What are the complications of PDA? • Majority of PDA is small and can be closed at
A: A..<; follows: cardiac catheterisation using percutaneously
• Pulmonary hypertension with reversal of shunt delivered device.
(Eisenmenger syndrome). • Surgical closure for large PDA.
• CCF. • Prophylaxis for infective endocarditis.
• Infective endocarditis. • In neonate (1-3 weeks old) indomethacin (0.2
• Dysrhythmia (AF). mg/kg IV) may be given, which may constrict
• Duct may rupture or calcify. and close PDA (by inhibiting prostaglandin E
synthesis, and prior to birth, duct is kept patent
Q: What investigations do you suggest?
by the effect of circulating prostaglandin). It is
A: As follows:
not helpful in older children.
• ECC (normal, LVH, and RYH in Eisenmenger • If Eisenmenger syndrome develops, surgery is
syndrome). contraindicated (see in Eisenmenger syndrome).
• Chest X-ray: Cardiomegaly (left ventricle and
left atrium are enlarged, also large aorta), large Prognosis: If untreated, one-third individuals die
pulmonary conus, large hilar arteries, lung fields from heart failure, PH or endocarditis by the age of
are plethoric. 40 and two-third by the age of 60.

Examination of Pulse

The usual instructions are: Q: If collapsing pulse is present, what else do you
want to see?
• Examine the pulse.
A: Then it is essential to see the signs of AR:
• Examine the pulse and relevants.
• Examine the pulse and auscultate the heart. • BP: High systolic, low diastolic and wide pulse
pressure.
Usually any of the following findings will be present:
• Neck: Dancing carotid pulse.
• Irregular pulse (AF and ectopics). • Heart: EDM.
• High-volume pulse or water hammer pulse.
Q: What are the causes of bradycardia?
• Bradycardia (complete heart block).
A: See page 71.
• Unequal radial pulse.
• Absent pulse.
• Radiofemoral delay and radioradial delay or Causes of unequal radial pulse
inequality. • Atherosclerosis (usually elderly).
(See radial pulse; look for rate, rhythm, volume, charac- • Congenital anomaly or aberrant radial artery.
ter, pulse delay and condition of the vessel wall. Finally, Coarctation of aorta (before the origin of left
examine all other pulses and compare on both sides). subclavian artery).
• Dissecting aneurysm.
Q: What are the causes of irregular pulse? • Takayasu disease.
A: See page 71. • Occlusion of subclavian artery (by ribs and
neoplasm).
Q: Is collapsing and high-volume pulse synonymous?
• Aneurysm of aortic arch.
A: No, collapsing pulse is a high-volume pulse; but all • Iatrogenic (Blalock-Taussig shunt in TOF).
high-volume pulses may not be collapsing.
_ SHORT CASES IN CLINICAL MEDICINE

Causes of absent radial pulse • Dissecting aneurysm.


• Coarctation of aorta (before the origin of left
• Anatomical aberration. subclavian artery). Brachial artery catheter
• Blockage by embolism or narrowing. with poor technique or tied during surgery.
• Takayasu syndrome.
• Iatrogenic (Blalock-Taussig shunt in TOF and
AV fistula for haemodialysis). N.J1 Remember, scar mark with thrill in wrist indicates
AVfistula for haemodialysis.

Atrial Fibrillation

The usual instructions are: • Thyrotoxicosis.


• Hypertensive heart disease.
• Examine the pulse.
• Idiopathic or lone AF (10% cases).
• Examine the pulse and relevants.
• Others include chronic constrictive pericarditis,
• Examine the pulse and auscultate the heart.
cardiomyopathy, acute pericarditis, congenital
Presentation of irregular pulse: heart disease (ASD), sick sinus syndrome, acute
infection (pneumonia), thoracicsurgery, electrolyte
• The pulse is 11O/min, irregularly irregular (irregular
imbalance (hypokalaemia and hyponatraemia),
in rhythm and volume).
alcohol abuse and pulmonary embolism.
Q: What are the causes of irregularly irregular pulse?
A: As follows: N.B. Remember the following points:
• Atrial fibrillation. • First five causes should always be mentioned
• Multiple ectopics. sequentially at the top of the list.
• Others: Atrial flutter with variable block, • Mention the causes of atrial fibrillation
paroxysmal atrial tachycardia with variable block. according to the age of the patient in that
particular long case (see below).
Q: What else do you want to see in atrial fibrillation?
A: As follows:
Q: If the patient is young, what are the causes of atrial
• Heart (heart rate to see pulse deficit, mitral fibrillation?
valvular or other cardiac disease). A: As follows:
• Thyroid status (warm sweaty hands, tremor of
outstretched hands, tachycardia, exophthalmos • Chronic rheumatic heart disease with valvular
lesions, commonly MS.
and thyroid gland).
• History ofIHD and BP. • Thyrotoxicosis.
• Others: Atrial septal defect (ASD), acute
• History of other diseases causing AF (see below).
pericarditis, myocarditis, pneumonia.
Q: How to differentiate between AF and multiple
ectopics in bedside? Q: If the patient is elderly, what are the causes of atrial
A: By physical exercise ectopics will disappear or fibrillation?
diminish, but fibrillation will be more prominent A: As follows:
or worse (ECG for confirmation, see below). • Coronary artery disease (commonly acute
myocardial infarction).
Q: What is the ECG in AF?
• Thyrotoxicosis.
A: P is absent (P may be replaced by small fibrillary
• Hypertension.
F-wave) and RR interval is irregular.
• Lone atrial fibrillation (idiopathic in 10% cases).
Q: What are the causes of AF? • Others: See above (unusual or less in chronic
A: As follows: rheumatic heart disease).

• Chronic rheumatic heart disease, usually with MS. Q: What are the noncardiac causes of AF?
• Il-lDs (commonly, acute MY). A: See above.
2 • CARDIOVASCULAR SYSTEM __

Q: What is lone atrial fibrillation? 2. Persistent atrial fibrillation:


A: Lone atrial fibrillation means atrial fibrillation • Control of heart rate: p-blocker, digoxin or
without any cause. Genetic predisposition may be calcium channel blocker (veraparnil, diltiazem).
responsible. Combination of digoxin and atenolol may be
used.
• 50% patients with paroxysmal atrial fibrillation
• To control rhythm: Direct current (DC)
and 20% with persistent or permanent atrial
cardioversion may be done safely. It may be
fibrillation have no cause and heart is normal.
repeated, if relapse occurs. Concomitant use
• Lone atrial fibrillation usually occurs below 60
of P-blocker or amiodarone may be used to
years of age.
prevent recurrence.
• It may be intermittent later may become perma-
3. Permanent atrial fibrillation:
nent.
• Control of heart rate: Digoxin, P-blocker,
• Prognosis: Low risk of CVD (O.S% per year).
calcium channel blocker (verapamil or
Usually life span is normal.
diltiazem).
Q: What are the complications of AF? • In some cases: Transvenous radiofrequency
A: As follows: ablation may be done (It induces complete
heart block. So, permanent pacemaker should
• Systemic and pulmonary embolism (systemic
be given. This is known as 'patch and ablate
from left atrium and pulmonary from right
strategy'. )
atrium). Annual risk is 5% (1-12).
• Heart failure.
Q: What is the role of anticoagulant in atrial
Q: If a patient with AFis unconscious, what is the likely fibrillation?
cause? A: Usually, warfarin is given to those who are at risk
A: Cerebral embolism (usually with right-sided of stroke. Target INR is 2-3. It reduces stroke in
hemiplegia) . two-third cases. Aspirin reduces stroke in one-fifth
cases. Anticoagulation is indicated in patient with
Q: How to treat AF?
atrial fibrillation having risk factors for throm-
A: Assessment with details ofhistory, physical examina-
boembolism.
tion and investigation to find out the primary cause.
If AF is due to acute illnesses, such as pneumonia,
pulmonary embolism and others then the treatment N.B. Remember the following points:
of primary disease will restore sinus rhythm. • In lone atrial fibrillation: Aspirin may be
given to prevent thromboembolism.
• Age <65 years and young with no structural
heart disease: Aspirin may be beneficial. No
warfarin.
• Target INR following anticoagulation is 2-3.
ECGofAF

Treatment (according to the type):


1. Paroxysmal atrial fibrillation:
• If asymptomatic: Does not require any TypesofAF
treatment, follow-up of the case? Two types according to heart rate:
• If troublesome symptoms are present:
p-bJocker. Other drugs: Flecainide or • Fast AF (pulse >lOO/min).
propafenone may be given. • Slow AF (pulse <lOO/min).
• Amiodarone is effective in prevention. Three types according to its nature:
• Low-dose aspirin to prevent
• Paroxysmal: Discrete self-limiting episodes;
thromboembolism.
may be persistent, if underlying disease
• If bradycardia is present (in sinoatrial
progresses.
disease): Permanent over drive atrial pacing
• Persistent: Prolonged episode that can
(60% effective).
be terminated by electrical or chemical
• In some intractable cases: Radiofrequency
cardioversion.
ablation may be required, who do not have
• Permanent: Sinus rhythm cannot be restored.
structural heart disease (70% effective).
__ SHORT CASES IN CLINICAL MEDICINE

Slow Pulse Rate [Complete Heart Block (CHB)]


The usual instructions are: Other causes of CHB:
• Examine the pulse. • Cardiomyopathy (ischaemic or idiopathic
• Examine the pulse and precordium. dilated), myocarditis.
• Drugs (digoxin, ~-blockers and amiodarone).
• Cardiac surgery (aortic valve replacement, VSD
Presentation of a Case :f--------, repair and CABG).
• Radiofrequency ablation of AVnode.
• Pulse: 40/min, high volume, normal rhythm; • Infiltrative disease (sarcoidosis, amyloidosis and
there is no radiofemoral delay and the condition haemochromatosis ).
of the vessel wall is normal. • Infection (infective endocarditis, Chagas disease
and Lyme disease).
• Collagen disease (SLEand rheumatoid arthritis).
My diagnosis is bradycardia. • Congenital complete heart block: Common in
children of mother with SLE(due to transplacental
Q: What is the most likely cause in this case? transfer of anti-Ro antibody or SSA).
A: As follows: • Neuromuscular (Duchenne muscular dystrophy).
• Complete heart block Q: What is the cause of congenital heart block?
• Drugs, such as ~-blocker, digoxin, verapamil. A: Congenital heart block usually occurs, if the mother
Q: What are the causes of.bradycardia? is suffering from SLE due to the presence of anti-
A: As follows: Ro (SSA) antibody, which crosses the placenta and
causes congenital heart block.
Causes of bradycardia:
Q: What are the signs of complete heart block?
• Sinus bradycardia due to any cause.
A: As follows:
• Second-degree heart block.
• Complete heart block. • Pulse: Bradycardia, 20-40 beats/min (40 beats/
• Nodal rhythm. min), high volume, does not increase by exercise
or injection atropine.
Causes of sinus bradycardia • BP: High systolic, normal diastolic and high
1. Physiological (due to increased vagal tone): pulse pressure.
Athlete, during sleep. • Neck vein: Cannon waves (large 'a' wave) may
2. Pathological: be present.
• Acute inferior myocardial infarction. • Heartsounds: Variable intensity of first heart sound.
• Myxoedema (due to reduction of sympathetic • Murmur: Systolic flow murmur (due to increased
activity). stroke volume).
• Hypothermia. Raised intracranial tension
Q: Why is there variable intensity of first heart sound
(due to inhibitory effect on sympathetic
and systolic murmur?
outflow). Obstructive jaundice (due to
A: As follows:
deposition of bilirubin in conducting system).
• Drugs (digoxin, ~-blockers, amiodarone and • Variable intensity of first heart sound is due to
verapamil). loss of AVsynchrony.
• Systolic flow murmur is due to increased stroke
Q: What are the causes of complete heart block?
volume.
A: The commonest causes are:
Q: What is the mechanism of cannon wave?
1. Acute CRB:
A: Due to loss of AV synchrony, when atria contracts
• Acute MI (commonly inferior).
against closed tricuspid valve, backward pressure
2. Chronic CHB: Idiopathic fibrosis due to:
produces cannon wave.
• Progressive fibrosis of distal His-Purkinje
system (Lev disease), in elderly. Q: What is the ECG change in CHB?
• Progressive fibrosis of proximal His- Purkinje A: Complete dissociation between P and QRS. (See
system (Lenegre disease), in younger. ECG on page 105.)
2 • CARDIOVASCULAR SYSTEM _

Q: If pulse rate is high, what are the likely causes of Q: What is Stokes-Adam attack? What are the clinical
CHB? features and treatment?
A: As follows: A: It is the brief attack of syncope or blackout in a patient
with complete heart block due to ventricular asystole.
• Pulse rate is high in congenital complete heart
Stokes-Adam attack may also occur in Mobitz type
block and does not require treatment.
II heart block, ventricular tachycardia or fibrillation,
• If the block occurs more proximally in AV node
sinoatrial disease.
(narrow complex escape rhythm).
Clinical features:
• Syncope or blackout with or without preceding
dizziness.
• During attack patient is unconscious, looks pale
and may have convulsion.
ECGof complete heart block • If asystole persists, there may be cyanosis, pulse
is absent, incontinence of urine, pupil is fixed
Q: What are the presentations of complete heart block? and dilated, plantar is extensor.
A: Dizziness, blackout or sudden loss of consciousness • Usually consciousness recovers rapidly followed
or syncope (Stokes-Adam attack). by flushing.

Treatment:
Q: How to treat complete heart block? • Permanent pacemaker (even after a single
A: If symptomatic, permanent pacemaker should be syncopal attack).
given. Treatment is based on cause. • During attack CPR should be done.

Takayasu Disease

The usual instructions are: arteries commonly aorta and its major branches,
carotid, ulnar, brachial, radial and axillary. Occa-
• Examine the pulse.
sionally, may involve pulmonary artery, rarely
• Examine the pulse and relevant.
abdominal aorta, renal artery resulting in obstruc-
tion. F:M ratio is 8:1.
Presentation of a Case :1----------,
Q: What are the pathological changes in Takayasu
(The patient is usually young female) diseases?
• All the pulses of upper limbs are absent, but A: Panarteritis, intimal hyperplasia, thickening of
present in lower limbs. media, thickening of adventitia, and later on fibrosis.
• BP is undetectable in upper limb and normal or
high in lower limb (to see BP in lower limb, an Q: What are the clinical features? What are the types?
18-cm cuff is required). A: Common in young female, 25-30 years, more in
• Bruit is present (mention the location). Asians.
• No other abnormality in CVS (occasionally
• In acute stage, may present with fever, malaise,
evidence of AR are found).
weight loss, arthralgia, myalgia and high ESR.
• There may be less development of upper part of
• In chronic case, dizziness, giddiness, headache,
the body.
syncope, daudication in the upper limb. This
may be AR, renal artery stenosis or anginal pain.
Hypertension in 32-93%.
My diagnosis is Takayasu disease (pulseless disease or
aortic arch syndrome). Types ofTakayasu disease (there are offour types):

Q: What is Takayasu disease? • Type 1: Involves aortic arch and its major branches.
A: It is a chronic, inflammatory, granulomatous pan- • Type 2: Involves descending aorta and abdominal
arteritis of unknown cause involving the elastic aorta.
__ SHORT CASES IN CLINICAL MEDICINE

• Type 3: Involves both type 1 and type 2. This may • Chest X-ray shows cardiomegaly and widening of
be complicated by aortic regurgitation. aorta.
• Type 4: Involves the pulmonary arteries. • Aortography of aortic arch and its branches; renal
angiogram shows narrowing, coarctation and
Q: How to diagnose Takayasu arteritis? aneurysmal dilatation.
A: Takayasu arteritis is characterized by at least three of • Serum immunoglobulin is high.
the following criteria.. Treatment:
• Age at onset of disease s; 40 years. • High-dose corticosteroid: Prednisolone
• Claudication of extremities. 40-60 mg daily or 1-2 mg/kg. If refractory to
• Decreased brachial artery pulse. steroid or difficult to taper steroid, methotrexate
• BP difference >10 mmHg between left and up to 25 mg weekly. Cyclophosphamide may be
right arm. used in resistant case also.
• Bruit over subclavian arteries or aorta. • Reconstructive vascular surgery in selected case.
• Arteriographic abnormality. Angioplasry, stenting or bypass surgery may be
done, if there is vascular complication.
Investigations: • Treatment of hypertension,
• CBC (high ESR and normocytic normochromic Prognosis: 95% survive at 15 years.
anaemia). Complications: Heart failure, stroke.

Coarctation of Aorta

The usual instructions are:


On auscultation:
• Examine the pulse. • Heart sounds are normal.
• Examine the pulse and relevants. • Systolic murmur audible in ... intercostal space
• Examine the pulse and auscultate the heart. close to the sternum and better heard in fourth
intercostal space posteriorly (site of coarctation).
Presentation of a Case May be ejection click, ESM in aortic area and
EDM (bicuspid aortic valve or dilatation of aortic
• Pulse: 70/min, normal in rhythm, high volume valve due to aneurysm causing AR).
in upper limb, femoral pulse is very feeble. There
is radiofemoral delay.
• BP: 220/110 mmHg in upper limb (and low in
lower limb). My diagnosis is coarctation of aorta.
• JVP: Normal. Q: Why is it a case of coarctation of aorta?
• Carotid pulse: High volume and vigorous. A: Because radial pulse is high volume, femoral pulse
• There is visible suprasternal, right carotid pulse is feeble and there is radiofemoral delay. Also, the
and supraclavicular pulsation. BP is very high in upper limbs and very low in lower
On inspection: limbs.

• Visible cardiac impulse. Q: Why murmur in coarctation of aorta?


• Visible dilated tortuous artery around the scapula, A: Usually due to increased flow through collateral
anterior axilla and over the left sternal border vessels; also, may be due to associated congenital
(collateral vessels are best seen by sitting and bicuspid aortic valve.
bending forward, with arm hanging by the side).
On palpation: Q: What are the types of coarctation of aorta?
A: There are two types, viz.:
• Apex beat: In left ... intercostal space, heaving in
nature. • Postductal (adult type): Commonly below the
• There may be thrill over the collateral vessels. origin of left subclavian artery, where ductus
arteriosus joins the aorta.
2 • CARDIOVASCULAR SYSTEM _

• Preductal (infantile type, 2%): Above the origin of Q: What is reverse coarctation?
left subclavian artery. In such cases, left Left radial A: When pulse is absent in upper limb, but present in
pulse is weak and rib is notched on right side. lower limb, it is called reverse coarctation. It occurs
Without other communication, the patient does in Takayasu disease.
not survive, and may be associated with PDA.
Q: What are the complications of coarctation of aortal
Q: What are the sites of collaterals? A: As follows:
A: Severe narrowing of the aorta encourages the for-
mation of collateral arterial circulation involving • Hypertension and its complication (LVF and
the periscapular, internal mammary and intercostal CVA).
• Infective endocarditis.
arteries; may result in localized bruit.
• Rupture at the coarctation site.
Q: What type of hypertension develops and why? • Dissecting aneurysm.
A: Usually systolic hypertension; diastolic may be • Aneurysm of aorta.
normal. Causes of hypertension are: • Subarachnoid haemorrhage (rupture of berry
aneurysm of circle of Willis ).
• Mechanical.
• Renin-angiotensin mechanism (due to coarcta-
Q: What are the causes of death in coarctation of
tion, less blood flow to kidney).
aorta?
• Resetting of baroreceptors.
A: As follows:
• Acute LVF.
N.B. Collateral vessels are formed involving perisca-
• Cerebral haemorrhage.
pular, internal mammary and intercostal arteries.
• Dissecting aneurysm.
• Subarachnoid haemorrhage (due to rupture of
aneurysm of circle of Willis ).

Q: What are the associations in coarctation of aorta!


A: As follows:
• Bicuspid aortic valve (in 50% cases).
• VSD.
• PDA.
• Aneurysm of circle of Willis (5-10% cases).
• In female, Turner syndrome.
• Occasionally, Marfan syndrome.

N.B. Remember the following points:


• Coarct.ation of aorta is twice more in male
Normal heart
than female. M:F ratio is 2:l.
Coarctation • It is 7% of all congenital heart diseases.
• Even if there is hypertension, renal
involvement is unusual and fundal changes
are also unusual.

Q: What are the causes of coarctation?


A: As follows:
• Congenital (the commonest).
• Rarely, may be acquired in trauma, Takayasu
disease.

Q: What are the presentations of coarctation of aorta?


What investigations do you suggest?

Presentations:
Coarctation of aorta • May be asymptomatic.
~ SHORT CASES IN CLINICAL MEDICINE

• Symptoms such as headache, nose bleeding, Q: What is the treatment of coarctation of aorta?
claudication of lower limbs and cold legs (due to A: As follows:
poor blood flow in lower limb).
• Coarctation of aorta should be treated surgi-
Investigations: cally as early as possible, preferably before 5
]. X-ray chest PAview: years of age. Surgical resection and end-to-end
anastomosis is usually done. lf coarctation is
• Poorly developed aortic knuckle (or elongated extensive, then prosthetic vascular graft may
aortic knuckle), cardiomegaly, poststenotic be done. (If surgery is done during adolescence
dilatation of aona. or adulthood, hypertension may persist in up
• Rib notching, mostly at the middle pan to 70% cases because of irreversible changes
posteriorly, due to enlargement of intercostal in arterioles or renal damage. If done in early
aneries from third rib downwards (first childhood, hypertension usually resolves
and second ribs are not affected because completely. )
intercostal aneries here arise from subclavian
• Balloon angioplasty may be helpful. It is
artery above the constriction).
particularly effective after restenosis.
• Figure of three (constriction at coarctation,
prestenotic and poststenotic dilatation).
Prognosis after surgery:
2. ECG (LVH).
3. Others: cr scan and CMR angiography are ideal • Surgical correction in childhood gives a good 25
for confirming the diagnosis. Echocardiogram year survival in 83%. If surgery is delayed until
and aortography may also be done. adulthood, 25 year survival rate drops to 75%.
Without surgery, only 25% live up to 50 years of
age, while cardiac failure occurs in two-third of
Q: What are the causes of unilateral rib notching?
surviving patients over 40.
A: As follows:
• In few cases, there is restenosis as the child
• Coarctation of aorta (before the origin of left grows. This can be treated by balloon
subclavian artery). angioplasty.
• Blalock-Taussig shunt (iatrogenic, done in Fallot • If operation is delayed, patient may have
tetralogy). persistent hypertension because of irreversible
• Subclavian artery obstruction. changes in the arterioles.
• Neurofibromatosis.
• May develop paradoxical hypertension due to
• Congenital.
baroreceptor-induced increased sympathetic
activity (detected by increased serum and urinary
Q: What are the causes of bilateral rib notching? catecholamines ).
A: Usual causes are:
• Coexistent bicuspid aortic valve, which occurs
• Coarctation of aorta (after the origin of left in over 50% of cases, may lead to progressive
subclavian artery). AS or regurgitation and also requires long-term
• Neurofibromatosis. follow-up.
• Congenital.

Marfan Syndrome

The usual instructions are: • Measure height from pubis to vertex and pubis
• Look at the patient and do the relevants. to sole of foot (pubis:sole > pubis:vertex), lower
• Examine the heart and relevants. segment > upper segment, height and arm span
(arm span> height).
Proceed as follows: • Look at the palate, eyes, any bony deformity and
• Look at the patient carefully (patient looks tall, hyperextensibility of joints.
lean and thin). • Finally examine the heart.
2 • CARDIOVASCULAR SYSTEM ~

Presentation of a Case • Eyes with blue sclera, subluxation oflens


(ectopia lentis usually upward and outward,
• The patient is tall, lean and thin. Face is long and iridodonesis, atrophy of iris).
narrow. • Palate is high arched.
• Long upper and lower limbs. There is arach- • Chest is pectus excavatum, carinatum and
nodactyly (long, thin, spider fingers and toes). kyphoscoliosis.
Hyperextensibility of joints. Arm span > height • Heart evidences of AR or MR.
(mention the exact measurement in centimetre or
inch). Lower segment> upper segment (mention
the exact measurement in centimetre or inch).

Arachnodactyly (hand) Arachnodactyly (fingers and toes)

Dislocation of lens High arch palate

My diagnosis is Marfan syndrome. of chromosome 15 (ISq21.1). Marfan syndrome


is characterised by triad of eye, skeletal and cardiac
Q: What are the associations in Marfan syndrome?
abnormalities.
A: A.<; follows:
1. Eye:
• Cystic disease in lung may cause spontaneous • Blue sclera.
pneumothorax (may be recurrent), bullae, apical • Subluxation or dislocation of lens (ectopia
fibrosis, aspergilloma and bronchiectasis. lentis ).
• Inguinal or femoral hernia and decreased • Iridodonesis (tremor of iris).
subcutaneous fat. • Heterochromia iris (various colour of iris).
• Small nodule or papule in skin of neck (Miescher • Myopia.
elastoma). • Retinal detachment.
Q: What is Marfan syndrome? • Glaucoma.
A: It is a connective tissue disorder inherited as auto- 2. Skeletal:
somal dominant (AD) trait due to mutation in • Tall, lean and thin, arachnodactyly.
the fibrillin-l gene-a component of extracellular Hyperextensibility of joints.
matrix. The fibrillin gene is located in the long arm • High arch palate.
_ SHORT CASES IN CLINICAL MEDICINE

• Kyphosis or scoliosis. High pedal arch or pes Causes of death in Marfan syndrome:
planus. • Dissecting aneurysm.
• Pectus excavatum or carinatum or asymmetry • Heart failure.
of chest.
Q: What investigation should be done in Marfan
3. CVS:
syndrome?
• AR (due to aortic root dilatation, secondary
A: As follows:
to cystic medial necrosis involving aorta).
• X-ray chest (may be normae may show features
• MR (with mitral valve prolapse).
of aortic aneurysm, and unfolding or widened
mediastinum. Pneumothorax or scoliosis may be
present).
• ECG (to see any arrhythmia).
• Echocardiogram (mitral valve prolapse, aortic
regurgitation, MR aortic root dilatation).
• CT or CMR (to see aortic dilatation).

Q: How to treat Marfan syndrome?


A: As follows:
• ~-blocker (propranolol) reduces aortic dilatation
and prevents the risk of aortic rupture or
dissecting aneurysm.
• ACE receptor blocker: In Marfan syndrome there
is upregulation of TNF-~, which is specifically
inhibited by ACE blocker. It prevents aortic root
Pectus excavatum dilatation.
• Avoid strenuous exercise to prevent aortic
dissection.
• Surgery: Elective replacement of ascending aorta
and aortic valve in patient with progressive
dilatation of aorta (>5 cm).
• Prophylaxis for infective endocarditis.
• Regular checkup (echocardiography) annually.
• Genetic counselling and orthopaedic measures.

Q: What is the effect of pregnancy in Marfan


syndrome?
A: Pregnancy is wen tolerated, if there is no serious car-
diac problem. It is avoided, if aortic root dilatation
is >4 em with AR. Maternal death may occur due to
aortic dissection during pregnancy. Early premature
abortion may also occur. Echocardiogram should
be done every 6-8 weeks throughout pregnancy
and 6 month postpartum. BP should be regularly
monitored. Vaginal delivery is possible; Caesarean
section is not done routinely. If aortic root is >4.5
N.B. Marfan syndrome may cause dissecting aneu-
em delivery should be done at 39 weeks by induc-
rysm, infective endocarditis may be associated
with coarctation of aOl1a. tion or Caesarean section. ~-blocker should be
continued throughout pregnancy.
Diagnosis of Marfan syndrome is made if:
Q: What is your differential diagnosis and how to dif-
• Positive family history and features of two ferentiate?
different systems. A: Homocystinuria (due to deficiency of enzyme
• There are features in three different systems. cystathionine synthetase).
2 • CARDIOVASCULAR SYSTEM __

Features Homocystinuria Marfan syndrome

1. Inheritance AR AD

2. Lens Dislocated downward Dislocated upward

3. Skeletal abnormality Osteoporosis-common. Osteoporosis-less common, but flat foot,


scoliosis, pectus excavatum are common

4. Aortic regurgitation Rare Common


o

...,
0-
S. Mental retardation Common No o·
<

(J)

6. Vascular complication Prone to develop thrombosis No


8
Reduced from cardiovascular risk May be normal
i.>
...,
7. Life expectancy
(/)

8. Test (urine) Cyanide nitroprusside (positive) Cyanide nitroprusside (negative) ~,.,


(1)

9. Spectroscopic examination (urine) Homocystine detected No :3


10. Pyridoxine May respond No

Dextrocardia

Usual instructions: Q: What is mesocardia?


A: When cardiac apex is in the midline, it is called
• Examine the precordium.
mesocardia.

Q: If the patient has dextrocardia, what else do you


want to see?
Presentation of the Case f------~
A: Lung base for crepitations and dubbing (bron-
• Apex beat is absent on left side, but present on chiectasis), and associated with Kartagener
right side, in... intercostal space,... cm from syndrome. Also need to see liver for situs inversus.
midline.
Q: Suppose a patient has dextrocardia. Suggest one
• Heart sounds are less heard in left side, but better
investigation.
heard in right side of chest.
A: PNS X-ray (to see frontal sinusitis or agenesis in
Kartagener syndrome).

Q: What is Kartagener syndrome?


My diagnosis is dextrocardia. A: It is characterised by:
Q: What is dextrocardia? • Dextrocardia.
A: When heart is on the right side of chest, but other • Bronchiectasis.
visceras remain on their usual sites, it is called dex- • Frontal sinusitis or frontal sinus agenesis.
trocardia. • Other features include situs inversus, infertility,
otitis media, and ciliary immotiliry.
Q: What is situs inversus?
A: Dextrocardia with reversal of sites of other visceras Q: What is the prognosis in situs inversus?
(stomach on right side, liver on left side, right lung A: Usually normal lifespan.
is on the left side, left lung is on the right side and
the appendix on the left side). Q: What is the clinical importance in situs inversus?
A: As follows:
Q: What is levocardia? • Diagnosis of appendicitis may be missed (it is on
A: When heart is on left side, but there is reversal the left side).
of the site of other visceras, it is called levocar- • Liver disease may be missed and liver biopsy may
dia (stomach on right side, liver on the left side, be mistakenly done on right side.
right lung is on the left side and left lung is on the
right side). N.B. Dextrocardia may occur in Turner syndrome.
__ SHORT CASES IN CLINICAL MEDICINE

Pericardial Effusion

Usual instructions: • Myxoedema.


• Lymphoma.
• Examine the CVS.
• Neoplasm (secondary from carcinoma of breast
and bronchial carcinoma).
Presentation of a Case • Uraemia and dialysis.
• Trauma.
• Pulse: l l Ojrnin (tachycardia), low volume, there
• Aortic dissection.
may be pulsus paradoxus (indicates cardiac
• Drugs (practolol, phenylbutazone, procainamide
tamponade).
and hydralazine).
• IVp: Raised, Kussmaul sign positive (raised JVP
• After radiotherapy.
during inspiration).
• BP: Low systolic, normal diastolic and narrow Q: What investigations are done in pericardial
pulse pressure. effusion?
On inspection: A: As follows:
1. Chest X-ray (PA view shows heart is enlarged in
• Nothing significant.
TD, globular, pear shaped with clear margin and
On palpation: lung fields are oligaemic).
• Apex beat: Difficult to palpate (or could not be 2. Full blood count (ESR high in TB and SLE).
palpated). 3. ECG (low-voltage tracing and tachycardia).
4. Echocardiogram, the colour Doppler may show
On percussion:
increased flow through tricuspid and pulmo-
• Area of cardiac dullness is increased. nary valve, decreased flow through mitral valve
On auscultation: during inspiration.
5. Pericardiocentesis, see the physical character,
• Heart sounds are muffled (or distant).
analysis of pericardial fluid to find out the cause
• Bronchial sound at the left inferior angle
(Gram staining, cytology, biochemistry, culture
of scapula (Ewart sign, which is due to the
and sensitivity, APBand malignant cell). Pericar-
'compression of the base of the left lung due to
dial biopsy may be needed sometimes.
enlarged heart).
6. MRI (very helpful to see haemopericardium or
loculated pericardial effusion).
My diagnosis is pericardia) effusion. 7. Other investigations according to suspicion of
causes:
Q: How to confirm your diagnosis?
A: Byechocardiogram (it shows echo-free zone). Peri-
• TB (MT, sputum for APB).
• Collagen disease (RA test, ANA, anti-double
cardiocentesis is definitive.
strand DNA).
Q: If echocardiogram is not available, how to diagnose • Hypothyroidism (Fry Fr4 and TSH).
pericardial effusion?
A: Chest X-ray in a Trendelenburg position (base of Q: How to confirm a case of tuberculous pericardial
heart will be wide). effusion?
A: Isolation of organism from pericardial fluid by:
Q: What are the BeG findings in pericardial effusion?
• AFBstaining (positive in 25% cases);
A: Low-voltage tracing and sinus tachycardia.
• Culture (Lowenstein-Iensen media);
Q: What are the causes of pericardial effusion? • PCR.
A: As follows:
• Following acute infective pericarditis (bacterial Q: How to treat pericardial effusion?
and viral). A: The commonest cause is TB. Hence, antitubercu-
• TB. lous drug plus prednisolone should be given. Other
• Collagen diseases (SLEand rheumatoid arthritis). treatment is needed according to primary cause (e.g.
• Cardiac causes (post-MI, postcardiotomy and SLE). Surgical drainage may be necessary, especially
Dressler syndrome). if viscus loculated or recurrent effusion.
2 • CARDIOVASCULAR SYSTEM __

Q: What is cardiac tamponade?


A: It is a state of compression of heart in rapidly devel-
oping pericardial effusion, which interferes with
diastolic filling of heart and the patient develops
features of shock. If there is rapid accumulation,
even 200 mL of fluid can cause cardiac tampon-
ade. However, if slow accumulation of fluid occurs,
2000 mL may be required for cardiac tamponade.

Q: What are the causes of cardiac tamponade? What


are the features of cardiac tamponade? How to
Pericardial effusion
treat?
Q: What are the causes of recurrent pericardial A: As follows:
effusion? • Trauma or cardiac surgery (causing
A: Mostly due to malignancy, may be in uraemia. In haemopericardium) .
such case, pericardial fenestration (creation of win- • Malignancy (repeated effusion may occur).
dow in the pericardium) is done to allow slow release • Myocardial rupture.
of fluid in the surrounding tissue. Surgical drainage • Dissecting aortic aneurysm.
or partial pericardiectomy may also be necessary. • Sometimes, any cause of pericardial effusion can
cause.
Q: How pericardiocentesis is done? What are the com-
plications of paracentesis? Clinical features:
A: It is better done under ultrasonographical or echo car-
• Heaviness and compression in chest.
diographical guidance. Aspiration needle is introduced
• Dyspnoea.
through left costoxiphoid junction, directed upward,
• Features of shock.
backward and towards the left shoulder. Needle may
Signs of cardiac tamponade (see signs of pericar-
also be introduced lateral to the apex beat.
dial effusion).
Q: What are the complications of paracentesis?
Treatment:
A: As follows:
• Injury to coronary artery and ventricles. • It is a medical emergency. Pericardiocentesis
• Dysrhythmia. should be done.
• Bleeding (which may aggravate cardiac • Treatment of primary cause.
tamponade).

Chronic Constrictive Pericarditis


Usual instructions:

• Examine the CVS.


On inspection:

Presentation of a Case • Nothing significant.

• Pulse: 120/min (tachycardia), low volume. On palpation:


Pulsus paradoxus may be present. • Apex beat: Palpable (in the left ... intercostal
• JVP: Raised. space, ... ern from the midline).
• BP: 100/70 mmHg.
• Kussmaul sign positive (raised JVP on On auscultation:
inspiration ). • Pericardia I knock (a third heart sound, due to
• Fall ofY descent (Friedrich sign). rapid ventricular filling).
- GRt CASES IN CLINICAL MEDICINE

• Infection (TB and coxsackie B infection).


• Haemopericardium (which may be due to trauma,
myocardial rupture after infarction and dissecting
aneurysm).
• Collagen disease (rheumatoid arthritis).
• Cardiac operation.
• Mediastinal irradiation.
• Fungal infection (histoplasmosis).
• Rarely, after acute purulent pericarditis.
• Idiopathic.
N.B. Calcification commonly involves right side of
the heart and can be seen by fluoroscopy. Calci-
fication does not always means constriction. RF
does not causes chronic constrictive pericarditis.
Q: What are the complications of chronic constrictive
pericarditis?
A: As follows:
• Atrial fibrillation (in 30% cases).
Pericardial calcification • Ascites.
• Myocardial fibrosis.
My diagnosis is chronic constrictive pericarditis.
Q: What investigations are done?
Q: What other signs or relevant do you like to look for? A: As follows:
A: Ascites (early feature), oedema (late feature) and 1. Chest X-ray (PAand lateral view): Relatively small
hepatomegaly. heart, pericardial calcification in 50% cases.
Q: What is chronic constrictive pericarditis? 2. BCG (low-voltage tracing, tachycardia, and
A: It is a disorder characterised by progressive thick- T inversion).
ening, fibrosis and calcification of pericardium. 3. &hocardiogram (thick calcified pericardium,
Commonly, involves right side of the heart. small ventricular cavities with normal wall thick-
ness.Iarge atrium, dilatation of inferior vena cava,
Q: What are the presentations and signs of chronic
abnormal septal motion and immobile hean).
constrictive pericarditis?
4. Colour Doppler (reduction of flow along mitral
A: Most features are due to systemic venous conges-
valve and pulmonary vein during inspiration.
tion. Symptoms are:
Opposite occurs during expiration).
• Cough, breathlessness on exertion, may be ortho-
5. CT scan or CMR.
pnoea, paroxysmal nocturnal dyspnoea.
6. Cardiac catheterisation shows that diastolic
• Weakness, dizziness, giddiness, anorexia, nausea
pressure is equal in all chambers (left and right
and vomiting.
ventrides), end-diastolic pressure (EDP) is
• Abdominal swelling, later ankle swelling.
equal in left and right atrium.
Signs are: 7. Other investigations (according to suspicion of
• Tachycardia, low-volume pulse. cause).
• Pulsus paradoxus may be present. 8. Endomyocardial biopsy: May be necessary to
• JVP: Raised, fall of Y descent (Friedrich sign). differentiate from restrictive cardiomyopathy in
Kussmaul sign positive (raisedJVPoninspiration). difficult cases.
• Pericardial knock (a third heart sound due to Q: How to treat chronic constrictive pericarditis?
rapid ventricular filling).
A: As follows:
• Enlarged tender liver. • Surgery: Complete resection of pericardium
• Ascites. (helpful in 50% cases).
• Peripheral oedema later on. • IfTB is present, early pericardiectomy with anti-TB
Q: Whatarethecausesofchronicconstrictivepericarditis? drug should be given. If no calcification, anti-TB
A: As follows: drug should be given first. If the haemodynamic
• Any cause of calcification of pericardium can status of the patient remains static or deteriorates
cause chronic constrictive pericarditis. after 4-6 weeks, pericardiectomy should be done.
2 • CARDIOVASCULAR SYSTEM _

• Treatment of primary cause should be done. 5. Oedema Early feature Ascites early, oedema
• After surgery, persistent constriction and myocar- than ascites late feature
dial fibrosis may be present. AF may occur after 6. On auscultation Murmur may Pericardial knock is
full recovery. be present present

Q: What are the differential diagnoses of chronic


Q: How to differentiate chronic constrictive peri-
constrictive pericarditis?
carditis from restrictive cardiomyopathy?
A: As follows:
A: As follows:
• Restrictive cardiomyopathy.
• CCF. Chronic constric- Restrictive
Features tive pericarditis cardiomyopathy
Q: How to differentiate chronic constrictive pericardi-
tis from CCE 1. Apex beat Not felt Well felt

A: As follows: 2. Heart Normal and May be enlarged,


pericardial knock is LVHand LVFmay
Chronic constrictive present be present
Features CCF pericarditis 3. ECG Low-voltage Bundle branch
1. Breathlessness Common, Not common in rest, tracing, tachycardia block, Q-wave
more on marked only on may be present
exertion exertion 4. Echocardiogram Changes in Myocardium is
2. Pulsus paradoxus No May be present pericardium thick

3. NP Raised, but no Raised, Kussmaul sign 5. Colour Respiratory varia- No change


Kussmaul sign positive, may be Y Doppler tion in A:V flow
descent 6. CTorCMR Pericardium is thick Thick ventricle
4. Cardiomegaly Present Absent with calcification

Acute Pericarditis

Usual instructions: ST is elevated with upward concavity (chair shaped


or saddle shaped) that is better seen in L1, LII'aVL,
• Examine the CVS.
aVF, V4 to V6.
T may be upright in acute phase.
Presentation of a Case
• Pulse: 80/min, normal in volume and character. Q: How to differentiate acute pericarditis from acute
• NP: Not raised. MI by ECG?
• BP: 120/75 mmHg. A: As follows:
In MI, there is ST elevation with upward convexity.
On inspection:
In pericarditis, there is Sf elevation with upward
• Nothing significant.
concavity. (Pericardium envelops the heart; hence,
On palpation: ST changes are more generalised and seen in most
• Apex beat: Palpable (in the left ... intercostal leads.)
space, ... cm from the midline).
On auscultation:
• Heart sounds are normal.
• There is pericardial rub in left third and fourth
intercostal spaces near the left lower parasternal
border.

My diagnosis is acute pericarditis.

Q: What are the EeG findings in acute pericarditis?


A: As follows (see the ECG on the right): 5T elevation in II,aVF,Vz to V6
Q: What is the clinical finding in acute pericarditis? • Collagen disease (SLE,scleroderma).
A: Pericardial rub. It is a high-pitched, harsh, scratch- • Others: Postradiation, postsurgical, idiopathic.
ing, grating, leathery sound, to-and-fro in quality.
Q: What are the commonest causes of acute
Better heard over the left lower parasternal area with
pericarditis?
the patient leaning forward (bare area of heart; it is
A: Viral infection and myocardial infarction.
the part of heart that is not covered by lung). It is
usually heard in systole, but may be present in dias- Q: What are the features of pericarditis after myocardial
tole, augmented by pressing the stethoscope and infarction?
is present after holding the breath (to differentiate A: Pericarditis occurs in 20% of patient in the first
from pleural rub). few days following myocardial infarction, more
Q: What are the presentations of acute pericarditis? commonly anterior and ST elevation myocardial
A: Chest pain, which is retrosternal. It is usually sharp infarction with high serum cardiac enzymes. Inci-
or stabbing in nature, may radiate to the shoulder dence is less (5-6%) with thrombolysis. Pericarditis
and neck, aggravated by movement, lying down, may occur as Dressler syndrome, 2-10 weeks after
deep breathing, exercise and swallowing. This may infarction.
be relieved by sitting or bending forward.
Q: How would you treat acute pericarditis?
Q: What are the causes of acute pericarditis? A: As follows:
A: As follows: • To relieve pain: NSAlDs (indomethacin or
• Following acute myocardial infarction (usually in ibuprofen or aspirin) is given.
second or third day) or Dressler syndrome (later). • In severe or recurrent case, corticosteroid should
• Viral (CoxsackieB, echovirus, also HlV): Common be given.
cause. • If no response to steroid, azathioprine or
• Bacterial (Staphylococcus aureus, Streptococcus, colchicine may be given.
Pneumococcus, Haemophilus inf/uenzae). • If recurrence with no response to medical
• Tuberculous pericarditis. treatment, pericardiotomy may be done.
• Fungal (histoplasmosis, coccidioidomycosis). • Treatment of primary cause should be done.
• Rheumatic fever. Antibiotic, if bacterial infection. Anti-Koch, ifTB
• Uraemia (an indication of urgent dialysis). is suspected. Other treatment according to cause.
• Malignancy (from carcinoma of bronchus, breast,
lymphoma, leukaemia), rarely primary tumour N.B. 20% cases of acute pericarditis may develop

T
of heart (mesothelioma). idiopathic re.lapsing pericarditis. Treatment is
• Myxoedematous pericarditis. as above. If pericarditis persists 6-12 months
• Drugs (doxorubicin, cyclophosphamide}. following acute attack, it is considered chronic.

Rheumatic Fever

The usual instructions are: My differential diagnoses are:


• Examine the knee joints (or other joints]. • RF (if deformity is present, it is against RF).
• Examine the upper limbs or lower limbs. What are • Reactive arthritis or Reiter syndrome.
your findings? What would you like to see in the • Juvenile idiopathic arthritis OIA or lCA).
heart? • Seronegative arthritis (ankylosing spondylitis and
enteropathic arthritis).
Presentation of a Case • Rheumatoid arthritis.
• Psoriatic arthritis (if any skin lesion, nail change).
(Supposing both knee and elbow joints) • Acute gouty arthritis.
• Both the knee and elbow joints are swollen. • Septic arthritis.
• Skin is red and shiny. • Traumatic arthropathy .
• Local temperature is raised and the joints are
Q: What is the likely diagnosis in this case?
very tender.
A: Rheumatic fever.
• There is restricted movement of the joints
(because of pain). Q: What relevants do you like to examine in RF?
A: As follows:
• I would like to examine the heart to see evidence
of carditis (pericarditis, myocarditis, and
endocarditis ).
Q: What is Aschoff nodule?
A: It is a granulomatous nodule composed of central
fibrinoid necrosis and multinucleated giant cells,
--
• Erythema marginatum (pinkish rash with slightly surrounded by macrophage and T-lymph ocytes. It
raised margin, mostly over the trunk and limbs). occurs throughout the heart. It is pathognomonic
• Subcutaneous nodules (found over bony promi- ofRF.
nences, joints and tendons. These are small and Q: Which joints are commonly involved in ARF?
painless and are found over the extensor surface). A: Commonly large joints, ankle, wrist, knee and
• Rheumatic chorea. elbow (usually does not involve small joints of the
Q: What are the presenting complaints of a patient hands and feet, rarely involves hip joint).
with RF? Q: What are the diagnostic criteria of RF?
A: RF usually occurs in children and young adults. A: It is diagnosed by revised 'Jones criteria'. Following
an attack of Streptococcus pharyngitis, there is usually
Features are:
a latent period of 1-3 weeks.
• Migrating (fleeting), nondeforming polyarthritis
involving the large joints (knee, ankle and elbow) 1. Major criteria:
and wrists with fever, may be continuous, high • Carditis.
grade is the presenting feature in 75% cases. • Shifting polyarthritis.
• Palpitation and chest pain (due to carditis in • Rheumatic chorea.
50% cases). • Erythema marginatum.
• Skin rash (erythema marginatum), subcutaneous • Subcutaneous nodule.
nodules. 2. Minor criteria:
• Involuntary movement (chorea in 10-30% cases). • Fever.
• Malaise, weakness and fatigue. • Arthralgia.
• Previous history of RF.
Q: What investigations will you do in RF? • High ESRor CRP.
A: As follows: • Leucocytosis.
1. Hb, TC, DC, ESR (high ESRand leucocytosis). • First- or second-degree AVblock in ECG.
2. Cvreactive protein [(CRP) high]. All of the above along with supportive evidence of
3. Antistreptolysin 0 (ASO titre)-may be high, in previous streptococcal infection, such as recent scar-
adult >200, in children >300). let fever, raised ASO or other streptococcal antibody
4. Throat swab culture (to find Streptococcus haemo- titre (anti-DNAse or antihyaluronidase) and posi-
lyticus). tive throat culture are seen.
5. Chest X-ray (cardiomegaly, pulmonary oedema Diagnosis is done by two or more major criteria, or
may be present). one major and two or more minor criteria plus sup-
6. ECG. portive evidence of streptococcal infection.
7. Echocardiography (to see valve abnormality and
cardiomegaly). Q: What are the signs of carditis?
8. RAfactor (to exclude RA) and ANA (to exclude A: RF can cause carditis involving all the layers of the
SLE), if needed. heart (endocardium, myocardium and pericar-
dium) called pancarditis.
Q: What is RF? What is the mechanism or pathogen- Signs of endocarditis:
esis ofRF? • Heart sounds are.soft.
A: It is a multisystem disorder, occurs as a sequela to
• PSM (due to MR).
pharyngitis by Streptococcus B-haemolyticus group A. • Mid-diastolic murmur lMDM (Carey Coombs
It is due to autoimmune reaction between the anti- murmur)].
gen (M protein) of Streptococcus haemolyticus, and • EDM (duetoAR).
cardiac myosin and sarcolemmal membrane protein
(laminin). As a result, antibody is produced against Signs of myocarditis:
streptococcal enzyme, causing inflammation in the • Tachycardia: Soft heart sound, S3 gallop.
endocardium, myocardium and pericardium as well • Cardiomegaly: Features of heart failure.
as joints and skin. There is formation of 'Aschoff Signs of pericarditis:
nodule' in heart only. • Pericardial rub (pericardial effusion may occur).
__ SHORT CASES IN CLINICAL MEDICINE

Q: What is erythema marginatum? Q: What is the prophylactic treatment of RF? How


A: It is characterised by transient raised pi nk or red rash, long it should be continued?
blanches on pressure, with clear centre and round A: Recurrence is common in patients who had carditis
margin. It occurs in 10% of cases, found mostly on during initial episode. In children, 20% recurrence
the trunks and proximal limbs (but not on face). It occurs within 5 years. Recurrence is uncommon
may coalesce into crescent- or ring-shaped patch. after 5 years and in patients over 25 years of age.

Q: What is subcutaneous nodule? To prevent recurrence:


A: These are small, firm and painless pea-shaped nod- • Oral phenoxymethyl penicillin 250 mg 12 hourly
ules, felt over bony prominence and tendons or or injection benzathine penicillin l.2 million
joints in extensor surface, present in 10-15% cases. units deep 1Mmonthly should be given.
• In penicillin-sensitive patients, erythromycin
Q: What is Sydenham chorea (St. Vitus' dance)?
(250 mg 12 hourly) or sulphadiazine (1 g daily)
A: It is a neurological manifestation of acute RF,
may be used.
usually occurs after 3 months of an acute attack,
• Prophylactic drug should be continued up to
when almost all other signs disappear.
21 years of age or 5 years after the last attack
• It occurs in one-third of cases,common in child and
(recurrence after 5 years is rare), whichever is longer.
adolescent, more in female of 5-15 years of age.
• It should be extended if an attack has occurred in
• Associated with emotional instability, irritability, the last 5 years, if the patient lives in area of high
inattentiveness and confusion. It may occur
prevalence or has a high exposure to streptococcal
without any features of acute RF. Carditis is infection.
common, may be the first manifestation.
• If there is residual heart disease: Prophylactic
• Speech may be explosive and halting. should be continued for 10 or 40 years of age,
• ESR,ASO titre and CRP are usually normal. whichever is longer.
• Rheumatic chorea is usually self-limiting and
recovers within few months. Q: What are the signs of activity in RF?
• Relapse may occur only in few cases, occasionally A: Persistent fever, tachycardia, high ESR, leucocytosis
during pregnancy (chorea gravidarum) or in and evidence of carditis.
those who use ora] contraceptive pill.
• Treatment needs sedation along with other
N_.B.Remember the following points:
treatment and prophylaxis of RF.
• Skin infection with streptococci is not
• 25% of cases develop chronic rheumatic heart
associated with RF.
disease in course of time.
• Streptococcal sore throat may not be present
Q: Howto treat acute RF? in some cases.
A: As follows: • Rheumatic fever licks the joints and kills the
1. Complete bed rest (until disease activity heart.
resolves). • More than 50% patients of RF with carditis
2. Oral phenoxymethyl penicillin 250 mg 6 hourly will develop chronic valvular disease after
for 10 days or single injection of benzathine pen- usually 10-20 years.
icillin 1.2 million units, deep 1Min the buttock • All the cardiac valves may be involved as a
(to eliminate the streptococcal infection). Eryth- sequela of complication of RF called chronic
romycin may be given if allergic to penicillin. rheumatic heart disease. It commonly
3. Analgesic (to relieve pain): Aspirin 60 mgJkg affects the mitral valve (70%), aortic valve
per day in divided doses. Higher dose may be (40%). Rarely, involves tricuspid (10%) and
needed. pulmonary (2%) valves.
4. Other treatment for symptomatic relief of • In chronic rheumatic heart disease, there is
cardiac failure, pericarditis and others: no history of RF in 50-60% cases.
• For carditis or severe arthritis: Corticosteroid
may be given (prednisolone 1-2 mgJkg daily). Q: What are the causes of migrating polyarthritis?
• For chorea: Diazepam for mild case and A: Rheumatic fever, septicaemia, gonococcal arthritis,
haloperidol in severe case may be given. syphilitic arthritis, Iyme arthritis, hyperlipidaernia
• For erythema marginatum and subcutaneous (type II), SLE, sarcoidosis, bacterial endocarditis,
nodules: No treatment is necessary. Whipple disease.
2 • CARDIOVASCULAR SYSTEM __

Swelling of knee joint in rheumatic fever


Effusion in right knee in rheumatic fever

Hypertrophic Cardiomyopathy

Instruction by the examiner: Q: What are the factors that alter the intensity of the
murmur at left lower sterna border?
• Examine the CVS. A: It increases during standing and valsalva manoeu-
vre, and decreases during squatting or sustained
Presentation of a Case hand grip.
1. Pulse: 88/min, normal volume, normal in Q: Why did you find murmur of MR?
rhythm, no radiofemoral or radio-radial delay. A: MR may be associated with HCM.
2. Carotid pulse: Jerky.
Q: Why not AS?
3. JVP: Normal ('a' wave may be prominent). A: In AS, the findings are:
4. BP: 95/80 mmHg (low systolic, normal
• Pulse is low volume and slow rising.
diastolic and narrow pulse pressure).
• BP shows low systolic, normal diastolic and
5. Precordium shows:
narrow pulse pressure.
• Apex beat is in the left ... intercostal space
• Systolic thrill in aortic area.
. .. cm from midline, heaving in nature
• Second heart sound shows soft A2 in all the
(may be double apical impulse).
areas, P2 is normal, may be reversed splitting of
• A systolic thrill is palpable at apex (or the
second heart sound.
lower left sternal border).
• There is a harsh ejection systolic murmur in
• First and second heart sound: Normal in all
aortic area and radiates towards the neck.
the areas.
• Fourth heart sound may be present (due to Q: What investigations should be done in HCM?
atrial contraction). A: As follows:
• There is a harsh ejection systolic murmur at • X-ray chest (may be normal).
the left lower sternal border. • ECG: May show LVI-I, left axis deviation,
• There is a PSM at the apex. infarction pattern (Q-wave in inferior and lateral
chest lead), deep T inversion (confused with
subendocardial myocardial infarction), atrial
My differential diagnoses are: fibrillation, also bizarre pattern.
• MR. • Echocardiogram (diagnostic): Typically shows
• AS. asymmetric septal hypertrophy, systolic anterior
• HCM. motion (SAM) of mitral valve. May also show
Q: What is the more likely diagnosis? Why? MR and small LVcavity.
A: HCM because of double apical impulse and jerky • Cardiac MR.
pulse. • Genetic analysis.
H1114CliNICAL MEDICINE

Q: What is cardiomyopathy? What are the types? 1. Specific measures:


A: Cardiomyopathies are a group of disease that pri- • ~-blocker, rate-limiting calcium channel
marily affect the heart muscle and are not due to blocker (veraparnil, diltiazem), and disopyra-
the result of congenital, acquired valvular, hyperten- mide for symptomatic relief and prevention
sion, coronary arterial or pericardia! abnormalities. of syncope.
• Amiodarone may be helpful to prevent
It is of three types:
arrhythmia.
• Dilated cardiomyopathy (ischaemic).
• In some patients with significant left ven-
• HCM. tricular outflow obstruction and symptoms,
• Restrictive cardiomyopathy.
dual chamber pacing may be needed.
Q: What is hypertrophic cardiomyopathy (HCM)? • Outflow tract obstruction can be improved
A: Hypertrophic cardiomyopathy (HCM) is a disease by partial surgical resection (myectomy) or
by iatrogenic infarction of the basal septum
of the heart muscle characterized by hypertrophy
of cardiac muscle with misalignment of the cardiac (septal ablation) by a catheter-delivered
fibres. Hypertrophy may be generalized or localized alcohol solution.
to the interventricular septum (asymmetrical septal • Implantable cardioverter-defibrillator (ICD)
hypertrophy) or other regions (apical HCM). should be considered in patient with clinical
risk factors for sudden death.
• Cardiac transplantation may be needed in
N.B..:Remember the following points: CHF not responding to treatment.
• It is a common type of cardiomyopathy. Prev- • Infective endocarditis prophylaxis may be
alence is 1:500 to 1:1000, inherited as AD, needed.
but may be sporadic in 50% case.
• About half of the patients have a positive family 2. Advice:
history. First-degreerelativesshould be screened. • Vigorous exercise and dehydration should be
• Previously, it was called hypertrophic avoided.
obstructive cardiomyopathy (HOCM), but • Genetic counselling of patients and relatives
left ventricular outflow obstruction is found is essential.
only in one-third patients.
Q: What are the risk factors for sudden death in
Q: What are the types of HCM? HCM?
A: As follows: A: As follows:
• Asymmetrical septal hypertrophy (70%). • A history of previous cardiac arrest or
• Basal septal hypertrophy (15-20%). sustained VT.
• Concentric (8-10%). • Recurrent syncope.
• Apical/lateral waJl «2%). • An adverse genotype and/or family history of
sudden cardiac death (SCD) «SO years old).
Q: What are the presentations ofHCM? • Failure of BP to rise during exercise (no change
A: Patient may be asymptomatic. Other features are: or hypotension).
• Angina on effort. • Nonsustained vr on 24-h Holter monitoring.
• Dyspnoea on effort. • Marked increase in left ventricular wall thickness
• Presyncope or syncope on effort. (>30 mm on echocardiography).
• Sudden death. • Delayed gadolinium enhancement on cardiac
MRl.
Q: What are the complications of HCM?
A: As follows: Q: What drugs should be avoided?
• Sudden death. A: As follows:
• Atrial fibrillation. • Digoxin.
• Infective endocarditis. • Vasodilators.
• Systemic embolism. • Diuretics.
• Nitrates.
Q: How to treat HCM? • Dihydropyridine calcium channel blockers.
A: As follows: • Alcohol (may cause vasodilatation).
2 • CARDIOVAS

Q: What is the effect of HeM in pregnancy? What • The patient should have regular follow-up in
precautions should be taken? well-equipped centre with expertise in high-risk
A: HeM is not a contraindication to pregnancy. The pregnancies and cardiac disease.
patient usually tolerates pregnancy well if not • ~-blockers or calcium channel blockers should
severely symptomatic prior to conception. There be continued.
is no evidence that pregnancy increases the risk of
SeD. Following precautions should be taken: Q: What disease is associated with HeM?
o
.,
....
0-
• Prenatal counselling regarding risk of disease in A: Friedreich ataxia. ~.
offspring. Vl

8
l;>
....
CHAPTER 3

RESPIRATORY SYSTEM
"Work out the best method for examination, and practice it until it is a second nature to you"
-Anonymous

Introduction

Usual instructions by tbe examiners: What is the percussion note? Supposing it is dull,
what are the causes of dullness on percussion?
• Examine the respiratory system.
What are your findings on auscultation? Supposing
• Examine the chest from front and/or from back.
bronchial sound, what are the causes of bronchial sound?
• Just percuss and auscultate. What are your findings?
Hence during examination, you must be ready to
• Put your stethoscope here. What are your findings?
answer the expected questions. After completing, pre-
Common short cases sent the case according to the examiner's instruction,
for example:
• Obvious findings during inspection (e.g.tachypnoea,
Present your case or what are your findings? (Mention
dyspnoea, lip pursing and thoracic deformity).
systematically in the form of inspection, palpation,
Puffy, plethoric face with congested eyes [superior
percussion and auscultation],
vena caval (SVC) obstruction].
Have you finished? What is your diagnosis? (Yes,
• Pleural effusion (unilateral or bilateral).
T have finished my examination. Diagnosis is right-
• Bronchiectasis (unilateral or bilateral).
sided pleural effusion.)
• . Consolidation.
Why pleural effusion? (Because .... Mention the signs
• Collapse of the lung.
of pleural effusion systematically.)
• DPLD [previously called interstitial lung disease
(ILD) or fibrosing alveolitis].
• Emphysema. N.B. If asked to examine the back of chest only, ask
• Pneumothorax. the patient to keep both hands on the shoulder,
i.e. right hand on left shoulder and the left on right.
Examiner may interrupt at any step during examination Look for clubbing, any change of joints (rheuma-
and ask any question, for example: toid arthritis), skin change (systemic sclerosis,
What is the vocal fremitus? What are the causes of dermatomyositis, a good clue for diagnosis, may
increased or decreased vocal fremitus? be associated with ILD).

Examination Routine

Proceed as follows: If you are asked to examine the respiratory system, the
following important general examinations are needed:
• Introduce yourself, remaining on the right side of
the patient. 1. While talking, hoarseness of voice (laryngitis and
• Undress the patient up to the waist (with recurrent laryngeal nerve palsy).
permission). The patient should lie flat, but if 2. Dyspnoeic or orthopnoeic or tachypnoeic (count
acutely ill or dyspnoeic, can be examined at 45 or0
the respiratory rate).
the position in which the patient feels comfortable. 3. Cough with wheeze (bronchial asthma and
• Look at the patient to see the following points chronic obstructive pulmonary disease) or stridor
before examination. (large airway obstruction).
3 • RESPIRATORY SYSTEM _

4. Prominent accessory muscles of respiration [indi- • Wasting (due to involvement of lower trunk of
cates chronic obstructive pulmonary disease brachial plexus by bronchial carcinoma, C8 and
(COPD)]. Tl Iesions}.
5. Lip pursing (in emphysema). • Pulse: Tachycardia, pulsus paradoxus (due to
6. Cyanosis. severe obstructive airway disease), high-volume
7. Cachexic or emaciated [due to tuberculosis (TB), and bounding pulse (due to CO2 excess).
bronchial carcinoma, lean and thin in emphy- • Warm and sweaty hands (due to anoxic cor
sema), obesity (associated with sleep apnoea pulmonale).
syndrome). • Flapping tremor (due to severe respiratory
8. Raised jugular venous pressure (JVP) (cor pulmo- failure and CO2 retention).
nale) or SVC obstruction. • Other diseases involving the hands, such as
9. In the face: rheumatoid arthritis and systemic sclerosi.s (all
• Pink puffer (due to emphysema). can cause fibrosing alveolitis).
• Blue bloater (due to chronic bronchitis). 11. In thelegs:
• Puffy, oedernatous, plethoric and red eye (SVC • Oedema (cor pulmonale).
obstruction) . • Deep vein thrombosis (DVf) can cause
• Skin change (systemic sclerosis). pulmonary embolism.
• Butterfly rash [dermatomyositis and systemic • Clubbing and cyanosis.
lupus erythematosus (SLE)]. 12. Heart (to see evidence of pulmonary hypertension
• Eye with Homer syndrome (partial ptosis and and cor pulmonale).
constricted pupil) in Pancoast tumour. ] 3. In the abdomen:
10. In the hands: • Liver (cor pulmonale).
• Nicotine staining in nails (may be associated • See, if paradoxically abdomen shows inward
with bronchial carcinoma). motion during inspiration (which indicates
• Clubbing. diaphragmatic paralysis). .
• Cyanosis. 14. Lymph nodes (lymphoma, metastasis, TB and
sarcoidosis).
- -

Examination of the Chest

(Examination should be performed systematically: • Visible, engorged vein in chest (SVC obstruction).
Inspection, palpation, percussion and auscultation; • Others: Suprasternal and supraclavicular
and, also present the case in the same way. Always excavation (hyperinflation), prominent accessory
examine both f.ront and back of chest and compare both muscles, gynaecomastia. needle puncture mark
right and left side during each part of examination.) and tattooing.
Inspection: Palpation:

• Shape of chest (asymmetry or deformity or • Position of trachea (central. deviated to right or


kyphoscoliosis, pectus excavatum or carinatum). left).
flattening and drooping of the shoulder (fibrosis • Apex beat.
or collapse). • Vocal fremitus.
• Movement of chest (unilateral or bilateral • Chest expansion.
restriction), movement upward (in emphysema). • Tracheal tug (descent of trachea during inspiration,
See paradoxical inward motion of abdomen during examine by placing fingers over the trachea; it
inspiration with the patient in supine position indicates hyperinflation).
(indicates diaphragmatic paralysis). • Cricosternal distance is the distance between
• Intercostal space (ICS) (fullness, indrawing of suprasternal notch and cricoid cartilage (normally,
lower ribs). three fingers or more. If it is less, indicates
• Scar marks (thoracotomy scar and thoracoplasty) hyperinflation) .
and radiation marks. • Rib tenderness (due to trauma or fracture,
• Visible impulse (cardiac impulse and epigastric secondary deposit), tenderness in costochondral
pulsation). junction (due to Tietze syndrome).
IIIIIDII SHORTCASESIN __C_Ll_N_IC_A_L
__M_E_D_IC_I_N_E ~

Percussion: • Added sounds:


o Rhonchi (high or low pitched, localized or
• Percussion note (norma] resonance, hyperresonance
generalized) .
or stony dull or woody dull, impaired).
o Pleural rub.
• Area of liver dullness (normally in right sixth rib o Crepitations (fine or coarse or end inspiratory.
or fifth intercostal space in midclavicular line, If crepitations are present, ask the patient to
obliterated or lower down in emphysema or severe cough and see again). Posttussive crepitation
asthma). (appears after cough) sometimes found after
• Area of cardiac dullness (obliterated due to cough due to TE.
emphysema or severe asthma).
Now ask the patient to sit forward, and examine back of
chest systematically.
Auscultation:
Finally, see the following points:
(Turn the head of the patient to the left side and tell
• If your diagnosis is COPD, see forced expiratory
him, 'Keep your mouth open, take deep breath in and
time (FEr) by asking the patient to exhale forcefully
out for me'. Place the stethoscope, and check both right
after full inspiration, while you listen by placing
and left side alternately.)
your stethoscope over the trachea.
• Breath sounds (normal vesicular, vesicular with • Normally, it is <6 s; >6 s indicates airway
prolonged expiration and bronchial). obstruction.
• Vocal resonance (normal, increased, decreased or • Finally, if any sputum cup is available nearby, look
absent). at the sputum and comment on it.

Discussions on Routine Examination


N.B. Remember the following points:
Respiratory Rate
--- • Normally, respiration in male is abdornino-
• Normally, it is 14-18jmin (in adult). thoracic (as man uses diaphragm more than
• Increased rate of respiration is called tachypnoea I intercostal muscles).
• Respiration in female is thoracoabdominal
(>20). Usually, it is faster in children, slower in the
elderly.
I (as woman uses intercostal muscles more
than diaphragm). In babies, it is abdominal
• Increased depth of respiration is called hyperpnoea.
(diaphragmatic).
Causes of tachypnoea (increased rate of respiration): • If respiratory movement is exclusively abdominal,
1. Physiological: Anxiety, fear, exercise or exertion.
think of the following causes: Ankylosing
spondylitis, intercostal muscle paralysis, pleuritic
2. Pathological:
pain or any painful condition of chest.
• Respiratory causes (bronchial asthma, • If respiratory movement is exclusively thoracic,
pneumonia or other respiratory infections, it indicates reduced diaphragmatic movement
pulmonary embolism, COPD). due to peritonitis, ascites and pregnancy.
• Cardiac causes [acute anterior myocardial
infarction (MI), acute left ventricular failure, left
ventricular failure (LVF)). Abnormal Breathing
• Others: Fever, metabolic acidosis (diabetic Kussmaul breathing (air hunger): It is characterized
ketoacidosis, lactic acidosis and renal failure), by deep, sighing, rapid respiration at regular rate due to
hysterical conversion reaction (HCR) and sti mulation of respiratory centre. Its causes are (due to
cerebrovascular accident (CVA). metabolic acidosis):
Causes of reduced respiratory rate: • Diabetic ketoacidosis.
• Renal failure.
• Sleep.
• Depression of respiratory centre (by narcotic drugs, (Occasionally in severe respiratory failure and hepatic
in some CVD). failure.)
3 • RESPIRATORY SYSTEM _

Ataxic breathing (Biot breathing): Characterized by


irregular respiration in timing and depth. It indicates
brain stem damage (CVAand head injury).
Cheyne-Stokes breathing: Cyclical variation in the
depth of respiration characterized by gradual deepen-
ing of respiration till a maximum is attained, followed
by gradual diminished respiration until a period of
apnoea occurs (apnoea alternates with hyperpnoea). It
is due to diminished sensitivity of respiratory centre to
CO2, This cycle may last for 2 min. Its causes are:
• Acute LVF(or severe heart failure).
• Coma due to any cause.
• Brain damage due to head injury and cerebral Pectus carinatum
haemorrhage.
• Narcotic drug poisoning.
• Sometimes, at high altitude.
Apneustic breathing: Characterized by a postinspira-
tory pause in breathing. It indicates pontine damage.
Paradoxical breathing: If abdomen sucks inward dur-
ing inspiration. It indicates diaphragmatic paralysis.

Shape of the Chest (Deformity or


Asymmetry)

1. Asymmetry: It may occur due to kyphosis, scolio-


sis or lordosis, and flattening.
2. Pigeon chest (pectus carinatum): It is the local-
Pectus excavatum
ized prominence, outward bowing of sternum and
costal cartilage. Its causes are:
• Congenital.
• Rickets.
• Malian syndrome.
• Homocystinuria.
• Repeated respiratory infection in childhood
(causing strong diaphragmatic contraction as
the thorax still remains pliable).
• Bronchial asthma since childhood.
• Osteogenesis imperfecta.
3. Pectus excavaturn (funnel chest, saucer or cup):
It is the localized depression at the lower end of
sternum or rarely depression of whole length of the
body of sternum and of costal cartilage attached to
it. It does not usually cause any problem; rarely, it Harrison sulcus
may cause lung and cardiac problems (pulmonary 4. Harrison sulcus: It is a groove directed outwards
hypertension and cor pulmonale). Its causes are: and slightly downwards from sternum in the lower
• Congenital (common cause). part of the chest anteriorly, along the line of attach-
• Rickets. ment of diaphragm (due to indrawing of ribs). Its
• Marfan syndrome. causes are:
• Homocystinuria. • Chronic bronchial asthma since childhood.
• Osteogenesis imperfecta. • Rickets.
5. Barrel-shaped chest: In this type, anteroposterior 8. Thoracoplasty: In this procedure, some ribs are
diameter of the chest is increased than the transverse resected on one side of the chest to achieve perma-
diameter (TD), ribs look more horizontal, intercos- nent collapse of the lung. Usually one side of the
tal spaces appear full, chest remains like the stage upper part of the chest is depressed. Previously, it
of full inspiration, subcostal angle >90 (normally
0
was done to treat pulmonary tuberculosis (before
90 or less).
0
the development of chemotherapy).
It is detected by placing two hard boards-one
on the front and another on the back of the chest- Reduction of Movement of Chest
and measure the distance between these two for Any respiratory disease is associated with reduction of
anterior-posterior (A-P) diameter (normal ratio movement on the affected side. Causes of restricted or
of A:P diameter is 5:7). Its causes are: reduction of movement:
• Emphysema.
• Severe chronic asthma. 1. Unilateral:
• Pleural effusion.
• Pneumothorax.
• Collapse.
• Fibrosis.
• Consolidation.
2. Bilateral:
• COPD (especially in emphysema).
• Diffuse pulmonary fibrosis.

Expansion of the Chest


It is measured by placing the hands in lower part of the
chest (also in middle part), fingers are placed on the
side of chest keeping the thumbs in midline and close
Barrel-shaped chest to each other (can also be measured using tape at the
level of nipple). Ask the patient to take a deep breath in
6. Rickety rosary: It is the prominent swelling at the
and out. Normal expansion is >5 cm.
costochondral junction due to rickets.
Causes of reduced expansibility:
• Emphysema (in severe emphysema, it is <1 cm).
• Pleurisy.
• Ankylosing spondylitis.
• Respiratory muscle paralysis.

Trachea
Normally, it is slightly deviated to the right. Causes of
shifting of trachea on:
1. Same side (on the side of lesion):
• Collapse.
• Fibrosis.
• Pneumonectomy.
2. Opposite side (opposite to the side of lesion):
• Pleural effusion.
• Pneumothorax.

Rickety rosary Apex Beat


7. Flail chest: It occurs in multiple fractures of ribs May be shifted, causes are as in trachea (see above).
and sternum. There is paradoxical movement of (Apex beat is also shifted in kyphoscoliosis, pectus
chest inwards during inspiration. excavarurn and cardiac cause.)
3 • RESPIRATORY SYSTEM __

N.B. Remember the following points: Breath sound may be normal (vesicular), bronchial,
vesicular with prolonged expiration, diminished or
• Trachea may be shifted in retrosternal goitre.
absent.
• Level of tracheal bifurcation: At the lower end
of manubrium sterni (angle of Louis) in front 1. Normal vesicular (similar to wind rustling in
and between fourth and fifth thoracic vertebrae leaves): It is louder and longer in inspiration, and
behind. Shifting of apex beat and trachea expiration is short, without any gap. Vesicular sound
indicates mediastinal shifting. However, these is produced in large airways. When heard through
may not be shifted, if mediasti num is fixed due to normal lung, filtering effect through the alveoli pro-
idiopathic fibrosing mediastinitis, radiotherapy duces attenuated and low-pitched sound.
or methysergide therapy.

Vocal Fremitus
Maybe normal, or increased or decreased (less prac- Vesicular Vesicular with Bronchial
tised, as vocal resonance is more sensitive). prolonged expiration

Causes of increased vocal fremitus (three Cs)


2. Bronchial: Harsh, louder, blowing quality, inspi-
• Consolidation. ration and expiration are of similar length and
• Collapse with patent bronchus. intensity with a gap in between inspiration and
• Cavity (large). expiration. It is produced in large airways, but
• Also in fibrosis. when the lung between airway and chest wall is
Causes of decreased vocal fremitus airless (e.g. consolidation), loss of filtration effect
results in high-pitched bronchial sound. Bronchial
• Pleural effusion.
sounds are of two types:
• Thickened pleura.
• Pneumothorax. • High-pitched (tubular): Found in consolidation
• Collapse with complete bronchial obstruction. and collapse with patent bronchus.
• Mass lesion (bronchial carcinoma and hydatid cyst). • Low-pitched (also called cavernous): Found in
cavitation.
Percussion Causes of bronchial sound (three Cs)
Percussion note may be normal resonance, hyper- • Consolidation.
resonance, dull or stony dull. • Collapse with patent bronchus.
Causes of hyperresonance • Cavity (nearer chest wall).
• Also fibrosis (low pitched).
• Pneumothorax.
• Big cavity. 3. Causes of vesicular with prolonged expiration:
• Big bullae. • COPD (chronic bronchitis and emphysema).
• Emphysema (usually increase resonance). • Bronchial asthma.
Causes of dullness
• Pleural effusion (stony dull). Vocal Resonance
• Thickened pleura (impaired dull).
• Consolidation (woody dull]. This may be normal, increased or decreased. Vocal reso-
• Collapse. nances are of three types:
• Mass lesion.
• Bronchophony: It appears to be near the ear piece,
• Raised diaphragm due to hepatomegaly (dullness
and found in consolidation.
on the right side)
• Aegophony: Nasal quality or goat-like (Greek: aix
means goat and phony means sound), found in
Auscultation consolidation and above the fluid level of pleural
Listen to the breath sound, compare on both sides effusion.
using the diaphragm of stethoscope. Use the bell above • Whispering pectoriloquy: Ask the patient to tell
the clavicle to hear lung apices. 'ninety-nine' or 'one-one one', as if whisper.
__ SHORT CASES IN CLINICAL MEDICINE

When auscultated with stethoscope, it appears as if Pleural rub Pericardial rub


the patient whispers in the examiner's ear. It may Anywhere on chest Over precordium, but better
be found in consolidation, collapse or cavitation in left lower parasternal area
(three Cs). (bare area of the heart)
Absent, if respiration is No relation with respiration
N.B. If bronchial sound is present, whispering pecto- stopped
riloquy is also present. Hence, always auscultate It is due to pleurisy It is due to pericarditis
for whispering pectoriloquy.
Crepitations
Causes of increased vocal resonance (three Cs)
These are bubbling or crackling sounds, occur due to pas-
• Consolidation.
sage of air through fluid in alveoli. Crepitations may be
• Collapse with patent bronchus.
fine or coarse; present in inspiration, expiration or both.
• Cavity.
• Also in fibrosis. • Early inspiratory crepitations, commonly found in
chronic bronchitis.
Causes of decreased vocal resonance
• End or pan-inspiratory crepitations, found in
• Pleural effusion. fibrosing alveolitis.
• Thickened pleura. Causes of coarse crepitation (see also page 141):
• Pneumothorax. • Bronchiectasis.
• Collapse with complete bronchial obstruction. • Resolution stage of pneumonia.
• Mass lesion.
Crepitations reduce or disappear after coughing in the
following diseases:
Added Sounds • Resolving pneumonia.
• Bronchiectasis.
• Pleural rub. • Lung abscess.
• Crepitation. • Pulmonary oedema.
• Rhonchi.
Crepitation not changed after coughing is found in:
• DPLD.
Pleural Rub
Q: Why end-inspiratory coarse crepitation in DPLD?
• It is a localized grating, creaking, rubbing and A: It is due to reopening of collapsed alveoli at the end
leathery sound produced by movement of visceral of inspiration (not due to fluid in alveoli).
pleura over parietal pleura.
Q: What are the differences between pleural rub and
• Heard both in inspiration and expiration,
crepitation?
disappears when breathing is hold.
A: As follows:
• The rub is augmented by pressing the stethoscope.
• May be palpable and associated with local pain. Pleural rub Crepitation
Presence of pleural rub indicates pleurisy (which may It is grating or creaking or Bubbling or crackling sound
be due to viral or other infections, pneumonia, pulmo- rubbing sound
nary infarction and bronchial carcinoma). No change with cough Changes with cough
It may be palpable Not palpable
PleuropericardiaJ rub: Augmented by pressing Notso
stethoscope
• When pleurisy involves the pleura adjacent to
Local pain due to pleurisy No local pain
pericardium, pleuropericardial rub is heard (there
is no pericarditis).
• It is due to rough pleural surface adjacent to Rhonchi
pericardium, which moves across one upon It is the musical sound produced by passage of air
another by cardiac pulsation. through narrow airways (due to mucosal oedema or
Q: What are the differences between pleural rub and spasm of bronchial musculature). It is of two types:
pericardia] rub? • High pitched: Indicates small airway obstruction.
A: As follows: • Low pitched: Indicates large bronchi obstruction.
3 • RESPIRATORY SYSTEM __

Causes of rhonchi: • Amiodarone.


• Bronchial asthma (medium or high pitched, more • Nitrofurantoin.
in expiration). • Azathioprine.
4. Pulmonary eosinophilia:
• Chronic bronchitis (low or medium pitched, both
in inspiration and expiration). • Antibiotics (sulphonamide, penicillin,
tetracycline and nitrofurantoin).
Localized rhonchi (indicates partial obstruction of • Antirheumatic agent (gold, aspirin,
large bronchus); usually a fixed, low-pitched rhonchi. penicillamine and naproxen).
Its causes are: • Anticonvulsant (phenytoin and
• Neoplasm (bronchial carcinoma and adenoma). carbamazepine) .
• Foreign body. • Antidepressant or antipsychotic (imipramine,
chlorpromazine and dothiepin).
• Mucous plugs (disappear after coughing).
• Cytotoxic (methotrexate, bleomycin and
• Congenital bronchial stenosis. procarbazine) .
5. Acute respiratory distress syndrome CARDS):
Bedside Lung Function Tests
• Aspirin and opium (in overdose).
• PET (forced expiratory time): Ask the patient to • Streptokinase.
exhale forcefully after fuJI inspiration while you • Hydrochlorothiazide.
listen by placing your stethoscope over the trachea. 6. Pleural disease or effusion:
Normally, patient can empty in <6 s. If >6 s, • Bromocriptine,
indicates airway obstruction (remember the age, • Nitrofurantoin.
add 1 s for every decade, e.g. a 40-year-old person • Methotrexate.
can exhale in 4 s). • Methysergide.
• PEFR (peak expiratory flow rate): Done using • By SLE (hydralazine, INH, procainarnide,
Wright peak flow meter. Normally in young males, phenytoin,carbamazepineandchlorpromazine).
it is 600 L/min; and in females, 400 L/min (value 7. Mediastinal widening or hilar lymphadenopathy
varies with age, sex and height; hence, the table for (pseudolymphoma):
the normal value should be seen). It is reduced in • Phenytoin or diphenylhydantoin.
COPD and bronchial asthma. 8. Respiratory failure:
• Opium.
Drug-induced Lung Disease • Sedative or hypnotic.
• Alcohol.
1. Drugs that cause or aggravate bronchial asthma: • Tricyclic antidepressant.
• ~-Blockers. Q: What is dyspnoea?
• Aspirin and other NSAIDs. A: It is the unpleasant, subjective awareness of breathing.
• Tarnoxifen.
Causes of acute or sudden dyspnoea:
• Dipyridamole.
• Acute severe asthma.
• Nebulized pentamidine [used for treatment
• Pulmonary oedema (acute LVF).
of Pneumocystis carinii (now called Pneumocystis
• Pulmonary embolism.
jiroveci) infection].
• Spontaneous pneumothorax.
2. Drugs causing cough: • Adult respiratory distress syndrome.
• ACE inhibitors (the commonest cause, more • Hysterical conversion reaction (HCR) or panic
in females; actual mechanism is unknown). attack.
This drug causes conversion of angiotensin-L
to angiotensin-2. Also, causes breakdown of Cause of dyspnoea on supine:
bradykinin and substance P. All these may cause • Phrenic nerve palsy (bilateral diaphragmatic
cough. paralysis) .
3. DPLD (noneosinophilic alveolitis): Platypnoea: It means dyspnoea that worsens on
• Busulphan. upright position. It occurs in arteriovenous (AV)
• Bleomycin. malformation at lung bases resulting in increased
• Methotrexate. shunting and hypoxia in upright position.
_ SHORT CASES IN CLINICAL MEDICINE

Crepitation or Pleural Rub

Usual instructions:
Presentation of a Case: 211--------,
• Put your stethoscope here (on chest). What is your
finding? • There are multiple crepitations (mention fine or
coarse) present in both inspiration and expiration
Presentation of a Case: 1 :1------__, (mention whether unilateral or bilateral,and the site).

• There is pleural rub (tell the site). Q: What do you think the cause of crepitation?
A: As follows (mention according to unilateral or bilateral):
Q: Describe pleural rub. If unilateral, causes are:
A: It is a localized scratchy, grating, cracking, rubbing or • Bronchiectasis.
leathery sound produced by movement of visceral • Resolution stage of pneumonia.
pleura over parietal pleura, augmented by pressing • Lung abscess.
the stethoscope. It is present both in inspiration and
If bilateral, causes are:
expiration, and disappears when breathing is stopped.
It may be palpable and associated with local pain.
• Bilateral bronchiectasis.
• Pulmonary oedema.
Q: What does it indicate? • DPLD.
A: Pleurisy.
Q: What is crepitation?
Q: What are the causes of pleurisy? A: These are bubbling or crackling sounds that occur
A: It may be due to viral (commonly coxsackie B) or due to passage of air through fluid in alveoli. Crepi-
other infections, pneumonia, pulmonary infarction tations may be fine or coarse, present in inspiration,
and bronchial carcinoma. expiration or both.
Q: What is patient's complaint? Q: How to differentiate between pleural rub and
A: Pleuritic chest pain that is characterized by sharp, crepitation?
localized pain which is aggravated by coughing, A: See on the page 128.
deep breathing, change of posture and movement. Q: What are the differences between pleural rub and
Q: What is the differential diagnosis of pleural rub? pericardial rub?
A: Crepitation. A: See on the page 128.

Pleural Effusion

Most signs are easily found by examining the chest from On percussion:
back, especially in small effusion. Look for any needle
• Stony dullness (up to ... , mention the location).
puncture mark, local dressing with gauze and tape and
any scar mark, and mention it. On auscultation:
• Breath sound: Diminished or absent.
Presentation of a Case • Vocal resonance: Diminished or absent.
(Supposing right-sided, tell the findings in right • No added sound.
side)
On inspection:
My diagnosis is right-sided pleural effusion.
• Restricted movement.
(There is one puncture mark, gauze and tape, Q: What are the differential diagnoses?
mention if any.) A: As follows:
On palpation: • Thickened pleura.
• Trachea and apex beat: Shifted to the left side. • Mass lesion.
• Vocal fremitus: Reduced or absent in right side
(up to ... , mention the location). Q: Why this is not thickened pleura?
• Chest expansion is reduced. A: In thickened pleura, no mediastinal shifting and
dullness is impaired.
3 • RESPIRATORY SYSTEM __

Q: Why not this is consolidation? • Others: Lymphoma and SLE in female (also
A: In consolidation, dullness is woody, there is no pulmonary infarction).
shifting of mediastinum, bronchial sound is present If the patient is middle-aged or elderly, the causes are:
and vocal resonance is increased.
• Pulmonary Tb.
Q: Why not this is collapse? • Parapneumonic.
A: In collapse, the apex beat and trachea will be shifted • Bronchial carcinoma.
to the same side (also, in collapse with patent bron-
Four common causes of pleural effusion:
chus, there is bronchial breath sound and increase
• Pulmonary TB.
vocal resonance).
• Parapneumonic.
Q: Why not pneumothorax? • Bronchial carcinoma.
A: In pneumothorax, there is hyperresonance on per- • Pulmonary infarction.
cussion.
N.B. Remember the following points in pleural
N.B. Just above the upper level of effusion, the follow-
effusion:
ing findings may be present (due to compression
collapse of lung): • Pleural fluid normally present: 5-15 mL.
• At least, 500 mL of fluid is necessary to detect
• Bronchial sound.
clinically.
• Increased vocal resonance.
• At least, 300 mL of fluid is necessary to detect
• Whispering pectoriloquy.
radiologically in PA view.
• Pleural rub. • At least, 100 mL of fluid is necessary to detect
Q: What is pleural effusion? radiologically in lateral decubitus position.
A: Accumulation of excessive amount of fluid in pleu- • Less than 100 mL or small amount of fluid is
ral cavity is called pleural effusion. detected by ultrasonography (even 20-25 mL
fluid can be detected).
Q: Can there be chest pain in pleural effusion?
A: Chest pain may occur due to pleurisy (may be pre-
Q: How to confirm if there is small effusion? (If not
sent in mild effusion).
detected by chest X-ray, PA view.)
Q: What are the characteristics of pleuritic chest pain? A: By doing:
A: Pleuritic chest pain is localized, sharp or lancinating • X-ray in lateral decubitus position.
in nature-worse on coughing, deep inspiration or • Ultrasonogram (USC) of lower part of the chest.
movement. • Occasionally, cr scan of chest may be needed.
Q: What are the causes of dullness on percussion over
Q: If clinically pleural effusion, but there is no fluid
lower chest?
during aspiration, then mention the causes.
A: As follows:
A: As follows:
• Pleural effusion (stony dullness).
• Fluid may be thick (empyema).
• Thickened pleura.
• Thickened pleura.
• Consolidation (woody dullness).
• Mass lesion.
• Collapse of the lung.
• Raised right hemidiaphragm (due to Q: What are the causes of predominantly right- or left-
hepatomegaly or liver pushed up). sided pleural effusion? .
• Mass lesion. A: As follows:
Q: What are the definitive signs of pleural effusion? Causes of right-sided pleural effusion:
A: Stony dullness and reduced or absent breath sound • Liver abscess.
(confirm by aspiration). • Meigs syndrome.
Q: What do you think of the causes of pleural effusion • Dengue haemorrhagic fever.
in this case? Causes of left-sided pleural effusion:
A: Mention the causes considering the age and sex, in • Acute pancreatitis.
the following way: • Rheumatoid arthritis.
If the patient is young, the causes are: • Dressler syndrome.
• Common causes: Pulmonary TB and parapneu- • Oesophageal rupture [Boerhaave syndrome).
monic. • Dissecting aneurysm.
SHORT CASES IN CLINICAL MEDICINE

Q: What are the types of pleural effusion according to • Physical appearance (straw coloured, serous,
the colour? haemorrhagic and chylous).
A: According to colour, pleural effusion may be: • Gram staining, cytology (routine) and
exfoliative cytology (malignant cells).
• Serous (hydrothorax).
• Biochemistry (protein and sugar); also a
• Straw.
simultaneous blood sugar, protein and lactate
• Purulent (empyema or pyothorax).
dehydrogenase (LDH) may be done.
• Haemorrhagic (haemothorax).
• Adenosine deaminase (ADA) (high in
• Milky or chylous (chylothorax).
tuberculosis) .
• Culture and sensitivity (CjS).
• Acid-fast bacilli (AfB) and mycobacterial CjS.
5. Others (of pleural fluid), according to suspicion
of causes:
• Cholesterol, LDH and rheumatoid factor
(in RA).
• Amylase (high in acute pancreauus,
oesophageal rupture and malignancy).
• Triglyceride (in chylothorax).
6. Pleural biopsy by Abram or Cope needle (posi-
tive in 80% cases with TB and 40-60% cases
with bronchial carcinoma).

Haemorrhagic
7. If cough, sputum for Gram staining, CjS, AFB
Straw colour
and mycobacterial CjS, and malignant cells
(exfoliative cytology).
8. If palpable lymph node, fine-needle aspiration
cytology (FNAC) or biopsy (for lymphoma,
metastasis) .
9. Other investigations according to suspicion of
causes include:
• ANF, anti-double stranded DNA (SLE).
• Liver function test (LFf) [chronic liver disease
(CLD)].
• Urine for protein and serum total protein
(nephrotic syndrome).
10. cr scan in some cases (it helps to clarify pleural
Serous Chylous abnormalities more readily than chest X-ray and
ultrasonogram; and also helps to distinguish
N.B. Clinically, only pleural effusion should be between benign and malignant diseases).
mentioned. After drawing the fluid and
according to its colour, other diagnosis may
be done, e.g. if pus, it is empyema.

Q: What investigations will you do in pleural effusion?


A: Mention according to the age of patient and sus-
pected cause:

1. Chest X-ray PA view (if small effusion, lateral


decubitus view).
2. Hb%, total count (TC), differential count (DC),
erythrocyte sedimentation rate (ESR) (high ESR
in TB, leucocytosis in pneumonia).
3. Mantoux: test (MT).
4. Aspiration of fluid for analysis: Right-sided pleural effusion
3 • RESPIRATORY SYSTEM __

Q: What is the role of pleural fluid amylase? Read the Following Topics in Relation
A: Pleural fluid amylase may be higher than serum to Pleural Effusion
amylase in acute pancreatitis, bacterial pneumonia,
oesophageal rupture and malignancy. It is high Causes of pleural effusion: There are two types:
in adenocarcinoma of lung and may be useful in Exudative and transudative.
differentiating it from mesothelioma.
1. Exudative (protein >3 gm%):
• Pulmonary TB.
Q: What are the causes of high eosinophil in the
pleural fluid (also high in the blood)? • Pneumonia.
A: Pulmonary eosinophilia, polyarteritis nodosa and • Bronchial carcinoma.
rarely lymphoedema. • Pulmonary infarction.
• Collagen diseases (SLEand RA).
N.B. Presence of high eosinophil in pleural fluid, • Lymphoma.
but not in blood is likely due to pulmonary • Dressler syndrome [postmyocardial infarction
embolism. High eosinophil in pleural fluid is (post-Ml) syndrome characterized by pain,
unlikely to be malignant. pyrexia, pericarditis, pleurisy and pneumonitis].
• Others (acute pancreatitis, subphrenic abscess,
liver abscess, pleural mesothelioma, secondaries
Q: How to differentiate between exudative and transu-
and yellow nail syndrome).
dative pleural effusion?
A: As follows: 2. Transudative (protein ~3 gm%):
• Congestive cardiac failure (CCF).
• Nephrotic syndrome.
Criteria Exudative Transudative
• Cirrhosis of liver.
Appearance Straw, purulent, Clear or serous
hazy, chylus or • Malnutrition.
blood stained • Hypothyroidism.
Protein >3g% <3g% • Meigs syndrome (ovarian fibroma, ascites and
right-sided pleural effusion).
Glucose Low Normal
• Chronic constrictive pericarditis.
Cholesterol >60 mg/dl <60 mg/dl
• Acute rheumatic fever.
lDH Greater than two- Lessthan two-
thirds of the upper thirds of the upper
limit of normal limit of normal Q: What is pseudotumour (phantom tumour)?
serum LDH serum LDH. A: It is the accumulation of fluid in interlobular
fissure, usually found along the lateral chest wall.
Pleural fluid lDH: >0.6 <0.6
SerumlDH
Chest X-ray shows rounded homogeneous opacity,
misdiagnosed as a tumour. It is confirmed by
Pleural fluid >0.5 <0.5
Ultrasonography (USC) (localized or encysted
protein: Serum
effusion) or cr scan. It disappears with resolution
protein
of effusion. It is commonly found in CCF.
Serum-effusion <1.2 g/dl >1.2 g/d
albumin gradient
(serum albumin - Q: What is yellow nail syndrome?
pleural fluid A: It is a congenital disorder characterized by:
albumin)
• Nails: Yellow, thick onycholysis.
• Lymphoedema of legs.
N.B. Remember the following: • Pleural effusion or bronchiectasis.
• Pleural fluid cholesterol level <60 mgldl
indicates transudate. In aU malignant effusion, Q: What are the features of parapneumonic effusion?
pleural fluid cholesterol> 60 mgldl. So, this test How to treat?
is useful to separate these two types of effusion. A: Usually small and localized, and may be noninfected
• High pleural fluid ADA indicates tubercular or infected fluid. Any loculated effusion is highly
pleural effusion. suggestive of empyema.
_ SHORT CASES IN CLINICAL MEDICINE

Treatment: Q: How to diagnose empyema thoracis clinically?


1. Uncomplicated parapneumonic effusion is A: As follows:
treated with antibiotic.
2. Complicated parapneumonic effusion: Thora- • High fever, sometimes hectic, may be associated
cotomy tube should be inserted in the following with chill, rigor and sweating. Fever is persistent
situations: or recurrent despite treatment with a suitable
• Gross pus. antibiotic.
• Gram staining of fluid shows positive • Malaise, weight loss.
organism. • Pleuritic chest pain, breathlessness.
• Glucose is <50 mgfdL. • Copious purulent sputum if empyema ruptures
• pH < 7.00. into a bronchus (bronchopleural fistula).
3. If pleural fluid is loculated and not adequately
• Toxic, emaciated.
drained, then give streptokinase injection
• Tachypnoea.
250,000. Or, urokinase 100,000 units should
be injected intrapleurally to dissolve fibrin • Tachycardia.
membrane. • Features of pleural effusion.
4. If still inadequate drainage, thoracoscopy with • Clubbing.
breakdown of adhesion or thoracotomy with • To be confirmed: Aspiration that shows pus or
decortication should be done. purulent fluid.
Q: How can you suspect malignant effusion?
A: Elderly emaciated patient with clubbing and Q: What is empyema necessitans?
nicotine stain, palpable lymph node and radiation A: In empyema thoracis, fluid may come out subcu-
mark in chest. Pleural fluid is haemorrhagic and taneously in the chest wall. This is called empyema
there is rapid accumulation after aspiration. necessitans,

d
Characteristics of empyema fluid:
Causes of haemothorax (blood-stained fluid):
• Fluid is purulent.
• Chest injury or trauma.
• Thick.
• Bronchial carcinoma.
• Biochemical: Glucose low, <3.3 rnrnol/L, protein
• Pleural mesothelioma.
exudative, LDH > 1000 UIL.
• Pulmonary infarction.
• Others: SLE,lymphoma and acute pancreatitis. • CIS: Organism may be found.
• Pleural biopsy may be done to exclude
Differences between traumatic haemothorax and tuberculosis.
haemorrhagic pleural effusion
Q: What is the treatment of empyema thoracis?
Haemorrhagic pleural
A: According to cause:
Traumatic haemothorax effusion
Less uniform Uniformly mixed 1. Nontuberculous:
Usually clots on standing Does not clot • Drainage of pus with wide bore intercostal
Gross haemorrhage, RBC > RBC > 10,OOO/mm3 tube using water seal drainage.
100,OOO/mm3 • Antibiotic for 2-6 weeks. IIV co-amoxidav or
RBC not crenated (fresh RBC) RBC may be crenated cefuroxime plus metronidazole. May be given
according to CIS.
• Surgical intervention if pus is thick or
Causes of empyema: loculated. Surgical decortication of the lung
• Bacterial pneumonia. may be needed, if visceral pleura are grossly

::J
• Lung abscess (bursting in pleural cavity). thickened.
• Bronchiectasis. 2. Tuberculous empyema:
• TB. • Antitubercular drug.
• Secondary infection after aspiration.
• Rupture of subphrenic abscess or liver abscess. • Wide-bore needle aspiration or intercostal
tube drainage.
• Infected haemothorax.
• Sometimes surgical ablation of pleura.
3 • RESPIRATORY SYSTEM __

Causes of bilateral effusion: Causes of chylothorax (milky or whitish fluid


due to lymph) are injury or obstruction of
• All causes of transudative effusion (CCF, nephrotic
syndrome, cirrhosis of liver, malabsorption or thoracic duct due to any of the following
malnutrition or hypoproteinaemia). causes:
• Collagen diseases (rheumatoid arthritis and SLE). • Traumatic (surgery and trauma to the thoracic
• Lymphoma. duct).
• Bilateral extensive pulmonary TB. • Neoplastic (bronchial carcinoma and metastasis).
• Pulmonary infarction. • Infective (TB and filariasis).
• Malignancy (usually multiple metastases involving • Lymphoma involving thoracic duct.
both lungs).
Causes of recurrent pleural effusion: Q: How to differentiate between chylothorax and
• Bronchial carcinoma (the commonest cause). empyema?
• Pleural mesothelioma. A: In both cases, fluid may be doudy. It is centrifuged
• Lymphoma. and following is observed:
• Collagen disease (SLE). ~ • If clear, empyema.
• All causes of transudate. • If persistently cloudy or milky, chylothorax.
Treatment of recurrent effusion: By pleurodesis. __
Q: What are the mechanisms of pleural effusion?
A: Excess pleural fluid accumulation occurs when pleu-
ral fluid formation exceeds absorption or normal
pleural fluid formation with reduced absorption.
Probable mechanisms are:
• Increased hydrostatic pressure (as in CCF).
• Reduced plasma colloidal osmotic pressure
(as in hypoproteinaemia).
• Involvement of pleura causing increased
permeability (as in TB and tumour).
• Impaired lymphatic drainage of pleural space (as
in obstruction oflymphatic system by tumour, TB
and radiation).
• Transdiaphragmatic passage of fluid (in liver
disease, ascites and acute pancreatitis).

Q: What is the mechanism of tuberculous pleural


Bilateral pleural effusion effusion?
A: Hypersensitivityto tuberculous protein in pleural space.
Q: How is pleurodesis done?
Q: What is subpulmonary pleural effusion?
A: In the following way:
A: Effusion between the lower surface of lung and
• A plain rubber tube is introduced in the intercostal upper surface of diaphragm. Confused with sub-
space and fluid is removed as far as possible. phrenic abscess. Detected by chest X-ray in lateral
Introduce the drug, tetracycline (500 mg) or decubitus position, or USG or CT scan.
kaolin or talc through the tube, clamp it and keep
for 4-8 h (may be overnight). In malignant pleural
effusion, bleomycin 30-60 mg is introduced. Features of rheumatoid pleural effusion
• Patient's posture should be changed 2 hourly to 1. Common in male.
allow the drug to spread in pleural space. 2. Occurs in 3% cases of RA.
• After 4-8 h, remove any remaining fluid and take 3. Usually small effusion, more in left side (cause
out the drainage tube at the height of inspiration. unknown), may be bilateral.
• The patient usually complains of severe chest 4. Pleural fluid is never blood stained, serous in early
pain after pleurodesis. In such case, analgesic case, later turbid.
should be given.
__ SHORT CASES IN CLINICAL MEDICINE

Q: How to treat pleural effusion?


5. Rheumatoid factor is usually positive.
A: Treatment should be according to cause. For
6. Nodules usually present in the lung.
example:
7. Systemic features are more.
8. Pleural fluid study: • Tftuberculosis: Full-course antitubercular therapy.
• Exudative (high protein) and glucose is low Prednisolone 20-30 mg daily may be given for
(glucose is unable to enter into pleural space, 4-6 weeks, especially in large effusion.
thus block entrance). • If parapneumonic: Aspiration of fluid; may
• High cholesterol (fluid looks turbid), with high be repeated if necessary. Antibiotic should be
LDH, low C3 and C4. given. If complicated case, especially empyema,
• Rheumatoid factor in pleural fluid may be thoracostomy may be done. Sometimes, if all fails,
positive. thoracotomy with decortication may be necessary.
• Cytology of elongated macrophage, giant multi- • If malignancy: Treatment given accordingly.
nucleated macrophage and few mesothelial cells Because of recurrent effusion, pJeurodesis is
may be present. necessary.
Characteristics of tuberculous pleural Q: How much fluid may be drawn at a time?
effusion A: Usually up to 1500 m!. If more is drawn, there may





Straw or amber colour.
Exudative.
High lymphocyte in pleural fluid.
AFBis found in 20% cases.
:J
Culture for AFBis found in one-third cases.
be risk of re-expansion pulmonary oedema.
The mechanism is: because of the fact that due to
effusion, lung is compressed and there is ischaemia
to lung parenchyma and necrosis of pulmonary ves-
sels. If more fluid is drawn, there is rapid expansion
• Pleural biopsy is positive in 80% cases. of the lung, as there is no regeneration of necrotic
vessels, so more leakage of fluid causing pulmonary
Q: What are the Causes of low pH and low glucose in oedema.
pleural fluid? Q: What is the role of steroid in pleural effusion?
A: As follows: A: Steroid is mostly given in tubercular pleural effusion.
• Infection (empyema). Although its role is controversial, some evidence
• Tuberculosis. suggests that it promotes rapid absorption of pleu-
• Advanced malignancy. ral fluid and gives the patient quick symptomatic
• SLE. relief. It also prevents pleural fibrosis and adhesion.
• Rheumatoid arthritis. Steroid should be used along with antitubercular
• Oesophageal rupture. therapy.

Pneumothorax

Presentation of a Case On percussion:


• Hyperresonance, but normal in left side.
(Supposing right sided, describe the findings in
On auscultation:
right side)
• Breath sound diminished or absent, but vesicular
On inspection: in left side.
• Restricted movement. • Vocal resonance diminished or absent, but
• Intercostal spaces: May appear full. normal in left side.

On palpation: My diagnosis is right-sided pneumothorax.


• Trachea and apex beat shifted to the left. Q: What are the differential diagnoses?
• Vocal fremitus reduced in right side, but normal A: Giant bullae and big pulmonary cavity.
in left side.
Q: What are the definitive signs of pneumothorax?
• Chest expansion: Reduced on the right side.
A: Hyperresonance on percussion and diminished or
absent breath sound.
3 • RESPIRATORY SYSTEM __

Q: What is pneumothorax? N.B. Remember the following points:


A: Pneumothorax means presence of air in the pleural • In young patient, common cause is rupture of
cavity. subpleural bleb.
• In patient >40 years of age, common cause is
Q: What is the usual presentation of pneumothorax? chronic bronchitis with emphysema.
A: The patient usually presents with sudden onset. of Q: What are the types of spontaneous pneumothorax?
unilateral pleuritic chest pain, breathlessness. A: Three types (anatomical):
1. Closed: Communication between the lung and
Q: Mention one investigation for your diagnosis.
pleural space is sealed off. Intrapleural pressure
A: X-ray chest PIA view. < atmospheric pressure. Trapped air is slowly
reabsorbed; lung re-expands in 2-4 weeks.
Q: What investigations do you suggest? Closed pneumothorax may be mild, moderate
A: As follows: and large.
• Complete blood count and ESR. 2. Open: Communication between the lung
• Chest X-ray PIA view. and pleural space persists (bronchopleural
• Sometimes, cr scan of chest. fistula). Intrapleural pressure and atmospheric
pressure are equal throughout the respiratory
Q: What are the types of pneumothorax? cycle, which prevents re-expansion of the
collapsed lung. Hydropneumothorax develops.
A: As follows:
Infection is common and empyema develops.
1. Spontaneous: It may be primary and secondary.
Physical examination shows features of
2. Traumatic. hydropneumothorax. Causes are-
• Rupture of emphysematous bullae
Q: What are the causes of pneumothorax? • Small pleural bleb
A: As follows: • Tuberculous cavity
1. Spontaneous: • Lung abscess into pleural cavity.
a. Primary: Without underlying lung disease. 3. Valvular: There is a communication between. the
It. is due to- pleura and the lung, which acts as a one-way
• Rupture of apical subpleural bleb due to valve allowing air to enter into the pleural space
congenital defect in connective tissue of during inspiration, but does not let air escape on
alveolar walls. Common in young, 15-30 expiration. Intrapleural pressure becomes greater
years of age. M:F ratio is 4: 1. Patient is than the atmospheric pressure. It results in com-
tall, lean and thin; there is 25% chance of pression of the lung, shifting of mediastinum to
recurrence. the opposite side, compression of heart and the
• Rupture of emphysematous bullae or opposite lung also. It reduces the venous return
pulmonary end of pleural adhesion. by compressing the SVC. There is rapidly pro-
b. Secondary: Occurs in pre-existing lung gressing breathlessness, cyanosis, shock, etc. It
disease. Causes are- is a medical emergency; death may occur within
• Commonly COPD and tuberculosis. minutes.
• Others: Lung abscess, acute severe
asthma, bronchial carcinoma, pulmonary
infarction, all forms of fibrotic and cystic
lung disease, Malian syndrome, Ehlers-
Danlos syndrome and eosinophilic
granuloma.
2. Traumatic:
a. Iatrogenic: During aspiration of pleural fluid,
thoracicsurgery. lung biopsy or pleural biopsy,
positive pressure ventilation, thoracocentesis
and subclavian vein catheterization.
b. Chest wall injury. Closed type
__ SHORT CASES IN CLINICAL MEDICINE

Treatment:
1. Chemical pleurodesis. Done by injecting tetra-
cycline (500 mg), kaolin or talc into the pleural
cavity through intercostal tube.
2. Surgical pleurodesis. Done by parietal pleurec-
tomy or pleural abrasion during thoracotomy or
thoracoscopy. Indications are:
• All patients after a second pneumothorax.
• Considered after first episode of secondary
pneumothorax, if there is low respiratory
reserve.
Open type
• Patient who plan to continue acuvity,
where pneumothorax would be particularly
dangerous (e.g. flying or diving) should
undergo definitive treatment after first episode
of primary spontaneous pneumothorax.

N.B. Pleurodesis is avoided in patient with cystic


fibrosis, as lung transplantation may be
required and pleurodesis may make this con-
dition technically not feasible.

Q: What are the indications of surgery (open


Valvular type
thoracotomy) ?
A: As follows:
• Failure of the lung to re-expand after 5 days of
N.B. Normal intrapleural pressure is negative (sub-
tube thoracotomy.
atmospheric), usually -2.5 to -6.0 mmHg.
• Recurrence (usually on third recurrence).
• Bilateral pneumothorax.
Q: How would you grade pneumothorax?
A: According to British Thoracic Society: Q: What is catamenial pneumothorax?
A: If pneumothorax develops at the time of men-
• Mild: Small rim of air around the lung, <20% of struation, it is called catamenial pneumothorax.
the radiographic volume. It is usually on the right side, occurs within 48 h of
• Moderate: Lung collapse, >20-50% of the onset of menstruation, common in 25-30-year-old
radiographic volume. female and is due to intrapleural endometriosis.
• Large: Lung collapse, >50% of the radiographic It is treated by hormone therapy to suppress
volume. ovulation (by progesterone or androgen therapy)
• Tension: Pneumothorax with cardiorespiratory or simply by oral contraceptive pills. Sometimes,
distress; features of shock. surgical exploration and pleurodesis may be
needed.
Q: What are the causes of recurrent pneumothorax
(more than twice)? How to treat? Q: What is tension pneumothorax? What are the
A: Causes are: causes? How to treat?
A: It is a valvular type pneumothorax in which there
• Apical subpleural bleb or cyst (congenital). is a communication between lung and pleural cav-
• Emphysematous bullae. ity with one-way valve, which allows air to enter
• Cystic fibrosis. during inspiration and prevents to leave during
• Others: Marfan syndrome, catamenial expiration. It causes shifting of mediastinum to the
pneumothorax, Ehlers-Danlos syndrome, opposite side, compresses opposite lung and heart
cx,l-antitrypsin deficiency and histiocytosis X, (pressure in pleural space is positive and rises above
honey comb lung. atmospheric level).
3 • RESPIRATORY SYSTEM __

Features of tension pneumothorax: is also advised to inflate air pillows or balloons,


• Severe chest pain (pain is worse with cough and which will help in the expansion of collapsed lung.
relieves on sitting position).
• Severe and progressively increasing dyspnoea. Q: How to follow-up a patient after chest tube
• Cough. insertion?
• Tachypnoea, tachycardia, pulsus paradoxus. A: As follows:
• Features of shock (hypotension, central cyanosis • Bubbling: Whether it disappears or persists
and tachycardia). (indicates leaking).
• Raised Ivf', engorged neckvein due to compression
• Blockage of the tube by clot or kinking.
of the heart.
• Malposition.
• Shifting of mediastinum.
• Retrograde flow back into the chest.
N.B. Cardinal features are progressively increasing
Q: How to treat pneumothorax?
dyspnoea and features of shock.
A: Depends on whether it is primary or secondary,
Causes of tension pneumothorax: open, dosed or tension or presence of symptoms.
• Traumatic.
1. In primary small pneumothorax:
• Mechanical ventilation at high pressure.
• Spontaneous resolution occurs. Follow-up
• Rarely, spontaneous pneumothorax.
at 2-week interval (repeat chest X-ray).
Treatment: • Normal activity.
• Immediate insertion of wide-bore needle in • Avoid strenuous exercise.
second intercostal space in midclavicular line, 2. In primary moderate-to-large with breathless-
with the patient in sitting position. ness: Percutaneous needle aspiration of air (2-5
• Intrathoracic tube is inserted in fourth, fifth or L. Stop, if resistance to suction is felt or patient
sixth intercostal space in midaxillary line; and coughs).
the tip of the tube should be advanced in apical 3. In secondary pneumothorax: Patient with
direction. It is connected to an underwater seal or COPD, even small pneumothorax can cause
one-way Heimlich valve. respiratory failure. Hence, water sea] drainage
• Patient should be kept propped up with oxygen should be given.
inhalation. 4. Open pneumothorax: Surgery (as is due to
• Morphine 5-10 mg subcutaneously. bronchopleural fistula).
• If bubbling ceases, repeat chest X-ray. If the lung 5. Tension pneumothorax (described as above).
re-expands, tube may be removed after 24 h. Tube
should be removed during expiration or Valsalva Advise to the patient:
manoeuvre (the tube need not be clamped before • Must stop smoking.
removing). • Avoid air travel for 6 weeks after normal chest
• If no response or continued bubbling for 5-7 X-ray.
days, surgical treatment may be necessary. • Diving should be permanently avoided.

Q: How do you know that the water seal drainage is Q: If you are working at a remote place and a patient
working properly or not? presents with tension pneumothorax, what meas-
ures should you take?
A: As follows:
A: Immediately I shall insert a wide-bore needle (may
• Bubbling of air in water. be canula/venflon) in the second intercostal space
• During expiration or coughing, more bubbling in rnidclavicular line. This will allow the trapped air
occurs . to escape (producing an audible hiss). Then I shall
• During inspiration, water column ascends within send the patient to the nearest hospital. (Do not
the tube, which remains under water. remove the canula; tape it securely.)

Q: What advice is given to the patient with water seal Q: How long the lung takes to re-expandi
drainage? A: Air is absorbed at the rate of 1.25% of the total radi-
A: Never raise the bottle above the chest wall. The bottle ographical volume/day. lienee, if there is 50% lung
must be kept below the level of thorax. The patient collapse, it will take 40 days to expand.
~ SHORT CASES IN CLINICAL MEDICINE

Q: What are the possible causes of failure of Q: What is hydropneumothorax? What are the causes?
re-expansion of lung? A: When there is accumulation of fluid and air in
A: As follows: pleural cavity, it is called hydropneumothorax. Its
• Water seal drainage is not working properly or causes are:
may be blocked. • Iatrogenic (during aspiration of pleural fluid).
• Presence of bronchopleural fistula. • Pulmonary TB.
• A major bronchus may be obstructed. • Bronchopleura) fistula.
• Lung is completely fibrosed with permanent • Trauma (penetrating chest injury and thoracic
damage. surgery).
• Rupture of lung abscess.
• Oesophageal rupture.
• Erosion of bronchial carcinoma.

Q: What is the bedside test in hydropneumothorax?


A: Succussion splash.

Q: What are the signs of hydropneumothorax?


A: In lower part, signs of pleural effusion; and in upper
part, signs of pneumothorax.

Q: What is the treatment of hydropneumothorax?

Right-sided pneumothorax
A: Water seal drainage and treatment of primary cause.

Q: What are the indications of chest tube or IT tube


drainage?
A: As follows:
• Tension pneumothorax.
• Large second spontaneous pneumothorax if >50
years.
• Malignant pleural effusion.
• Empyema thoracis or complicated
parapneumonic effusion.
• Hydropneumothorax.
• Traumatic haernopneurnothorax.
• Postoperatively, e.g. thoracotomy,
Left-sided hydropneumothorax oesophagectomy, cardiothoracic surgery.

Bronchiectasis

(Most physical signs are found on the back of chest.) My diagnosis is bronchiectasis (right or left or bilateral
or extensive).
Presentation of a Case :1--------, Q: What are your differential diagnoses?
A: As follows:
(Describe systematically, only auscultatory findings
• Diffuse parenchymal lung disease (DPLD)
given. Findings on inspection, palpation and
(idiopathic pulmonary fibrosis (IPF)].
percussion are usually norrnal.]
• Pulmonary TB.
• Breath sound is normal.
• Vocal resonance is normal in all the areas. Q: Why it is bronchiectasis?
• Multiple coarse crepitations in right or left or A: There are generalized clubbing and bilateral basal
both lung bases, altered by cough. coarse crepitations altered by coughing. All are
highly suggestive of bronchiectasis.
3 • RESPIRATORY SYSTEM _

Q: What else do you want to examine? breathlessness in advance cases. Features of secondary
A: I want to examine for clubbing. infection may be present.

Q: What history do you like to take if bronchiectasis? Q: What is dry bronchiectasis (bronchiectasis sicca)?
A: History of cough with profuse expectoration of A: It is a type of bronchiectasis in which dry cough is
sputum, which is more marked in the morning after associated with intermittent episodes of haemop-
waking up from sleep. tysis. It may be massive, even life threatening as
bleeding is from bronchial vessels with systemic
Q: Why not IPF? pressure. Common in patients with granulomatous
A: In IPF, crepitations are usually bibasilar, inspiratory infection, especially TB, and usually involves upper
(dry or Velcro' type in quality), unaltered by lobe.
coughing. (In such case, usually there is history
of persistent cough that mayor may not be with Q: Why hae.moptysis?
profuse expectoration, progressively increasing A: It is due to bronchial wall hypertrophy; hence
breathlessness or exertional dyspnoea.) mucosa becomes friable, sloughs out, capillary
opens and bleeding occurs. Erosion of hypertrophic
Q: Why not pulmonary oedema?
bronchial artery may result in massive haemoptysis.
A: In pulmonary oedema, there is no generalized
clubbing. Also, in pulmonary oedema, crepitations
Q: What is bronchiectasis and what are the causes
are usually fine and present during both inspiration
of it?
and expiration.
A: It is the abnormal, permanent dilatation of one or
(In pulmonary oedema, there is history of orthopnoea more bronchi with destruction of bronchial wall
or paroxysmal nocturnal dyspnoea (PND) or history proximal to the terminal bronchiole. Its causes are:
suggestive of any cardiac disease.)
1. Congenital or hereditary:
Q: What are the causes of basal crepitation? • Cystic fibrosis.
A: As follows: • Kartagener syndrome (triad of bronchiectasis,
1. Unilateral basal crepitation: dextrocardia, and sinusitis or frontal sinus
• Unilateral bronchiectasis. agenesis).
• Primary ciliary dyskinesia (including
• Resolution stage of pneumonia.
imrnotile ciliary syndrome).
• Lung abscess.
• Localized fibrosis of lung. • Hypogammaglobulinaemia (of 19A and
IgG; it causes recurrent infection and
2. Bilateral basal crepitation:
bronchiectasis) .
• Bilateral bronchiectasis.
• Yellow nail syndrome.
• Fibrosing alveolitis or lLD.
• Young syndrome (obstructive azoospermia
• Pulmonary oedema.
and chronic sino pulmonary infection,
3. Causes of bilateral crepitation with clubbing:
thought to be due. to mercury intoxication).
• Bilateral bronchiectasis.
• Sequestrate segment of lung.
• Fibrosing alveolitis or ILD.
2. Acquired:
Q: What is the commonest site of bronchiectasis? • In children, pneumonia complicating
measles, whooping cough, primary TB and
A: Left lower lobe and lingula.
foreign body.
Q: What is posttussive crepitation? What is its signifi- • In adults, bronchial neoplasm, pulmonary
cance? TB, recurrent aspiration or suppurative
A: Crepitation that appears after cough is called pneumonia.
posttussive crepitation. It is usually at the apex and • Allergic bronchopulmonary aspergillosis
indicates TB. (causes proximal bronchiectasis).

Q: What are the presentations of bronchiectasis? N.B. In children, postmeasles or whooping cough
A: Cough with profuse expectoration of sputum, are commonly associated with bronchiectasis.
which is usually more marked in the morning after In adults, post-tubercular bronchiectasis is the
waking from sleep. Occasionally, haemoptysis and common one.
_ SHORT CASES IN CLINICAL MEDICINE

Q: What are the types of bronchiectasis? • Urine for proteinuria (if amyloidosis is
A: There are three types: suspected)
• USG of whole abdomen (if situs inversus is
• Saccular or cystic (more severe form). suspected).
• Cylindrical.
• Fusiform.

Q: What are the characteristics of sputum in


bronchiectasis?
A: If the sputum is kept in a bottle, there are three
layers:
• Lower sediment (epithelial debris and bacteria)
layer.
• Middle thick or liquid layer.
• Upper frothy layer.

Q: What investigations do you suggest in


bronchiectasis?
A: As follows:
1. Chest X-ray PAview (may be normal), and there
may be:
Left-sided bronchiectasis
• Ring with clear centre (like honeycomb).
• Ring with or without fluid level, may be
mul ti pie (cystic bronchiectasis).
• Linear streaks (tram line).
• Thick bronchi, signet ring or patchy opacity.
2. High-resolution CT scan: Definitive (preferred
investigation) .
3. Others:
• Complete blood count (CBC) with ESR (may
be neutrophilic leucocytosis).
• X-ray of paranasal sinuses (PNS) (in Karta-
gener syndrome).
• Lung function tests (obstructive type usually;
may be restrictive in advanced case).
• Sputum for Gram staining, CIS, AFB, malig-
nant cell (mention if necessary according to
the history). CTscan showing bronchiectasis
• Sometimes bronchoscopy to locate the site of
obstruction. Q: If a patient with bronchiectasis develops nephrotic
• Aspergillus precipitin antibody or skin syndrome (or urine shows proteinuria), what is the
prick test [if allergic bronchopulmonary likely diagnosis?
aspergillosis (ABPA) is suspected]. A: Amyloidosis (in amyloidosis, there may be
• Serum immunoglobulin: If hypogamma- splenomegaly). In longstanding bronchiectasis,
globulinaemia is suspected (10% of adu Its amyloidosis may occur as a complication.
with bronchiectasis have antibody deficiency,
mainly 19A). Q: What is the role of CT scan in diagnosis of
• Sweat test for chloride content: If cystic bronchiectasis?
fibrosis is suspected. A: Conventional CT has sensitivity of 60-80%,
• Mucociliary clearance (nasal clearance of whereas high resolution CT scan has a sensitivity
saccharin): Normally less than 30 min. If of more than 90% (previously bronchography was
more, it indicates ciliary dysfunction. done).
3 • RESPIRATORY SYSTEM __

Q: What is the difference between standard CT Q: How will you treat bronchiectasis?
scan and high-resolution computed tomography A: As follows:
(HRCT)? • Postural drainage, keeping the affected part
A: In standard CT, the resolution is lO-mm thick. But remaining up and percussion over it. It is done
in HRCT, resolution is 1-2-mm thick. for 5-10 min, once or twice daily.
• Antibiotic, if infection.
• Chest physiotherapy.
Q: What are the complications of bronchiectasis?
• Surgery (lobectomy).
A: As follows:
• Bronchodilator drugs. Also, nebulized salbutarnol
• Secondary infection (pnewnonia and pleurisy), may be used in asthma, COPD, cystic fibrosis
common organisms are Staphylococcus aureus, and ABPA.
Haemophilus influenzae and Pseudomonas aernginosa. • Inhaled or oral steroid can decrease the rate of
• Lung abscess. progression. Also, helpful in ABPA.
• Pleural effusion, empyema or pneumothorax. • In bilateral extensive bronchiectasis, lung
transplantation is required.

=.
• Pulmonary hypertension and cor pulmonale.
• Respiratory failure.
Indications of surgery:
• Amyloidosis (commonly involving spleen or
kidney) in longstanding case. Usually in young patient. Indications
• Brain abscess (metastatic cerebral abscess). • Unilateral and localized to a single lobe or segment.
• Aspergiloma in the bronchiectatic cavity. • Severe and recurrent haemoptysis.

Bronchiectasis with Cystic Fibrosis

Q: What are the clinical features of cystic fibrosis?


Presentation of a Case :1----------. A: As follows:
1. Neonate: Failure to thrive, meconium ileus, rectal
• Patient is young with features of bronchiectasis
[seein bronchiectasis). prolapse.
2. Children and young adult:
• Respiratory: Cough, wheeze, recurrent
My diagnosis is bronchiectasis. infection, bronchiectasis, lung abscess,
pneumothorax, lobar collapse, haemoptysis,
Q: What do you think the cause is in this young patient? respiratory failure, cor pulmonale, asthma,
A: I want to take the history. It may be pneumonia otitis media, sinusitis, nasal polyps, etc.
complicating measles or- whooping cough in • Abdominal: Diabetes mellitus, steatorrhoea,
childhood. Also, it may be due to cystic fibrosis. malabsorption,cholestero]gallstone,secondary
biliary cirrhosis and portal hypertension,
Q: Why cystic fibrosis? distal intestinal obstruction syndrome (meco-
A: Because there is bilateral extensive bronchiectasis nium ileus equivalent syndrome). There is
involving both lungs. It is also associated with increased incidence of peptic ulceration and
generalized dubbing. gastrointestinal malignancy.
• Others: Male infertility (due to failure of
Q: What is cystic fibrosis? development of vas deferens and epididymis),
A: Cysticfibrosis is a life-threatening autosomal recessive delayed puberty and skeletal maturity, arthritis,
disease characterized by abnormal transport of chlo- osteoporosis, vasculitis,hypertrophic pulmonary
ride and sodium ions across an epithelium causing osteoarthropathy, dubbing, stress incontinence
thick, viscous secretions and leading to bronchopul- due to repeated forced cough. About 20% of
monary infection and pancreatic insufficiency. female with cysticfibrosis are infertile.
There is an abnormality in the gene encoding a Q: What is distal intestinal obstruction syndrome
chloride ion channel in the nasal epithelium, lungs, (meconium ileus equivalent syndrome)?
salivary glands, pancreas, intestine and bile ducts. A: It is a form of small intestinal obstruction occurring
Prevalence is 1/2500. during infancy and onwards in a patient with
SHORT CASES IN CLINICAL MEDICINE

cystic fibrosis, resulting from a combination of


steatorrhea and viscid intestinal secretions, causing
faecal impaction in ascending colon or in ileocaecal
junction.

How to diagnose a case of cystic fibrosis?


A: It is based on clinical history and:
• Family history of cystic fibrosis.
• Sweat test: High sweat sodium and chloride
concentration over 60 mmol/L,
• Blood DNA analysis of gene defect.
• Radiological features of bronchiectasis.
• Absent vas deferens and epididymis. Bilateral bronchiectasis in cystic fibrosis
• Blood immunoreactive trypsin levels (for screening
purpose).
Q: How to treat cystic fibrosis?
Q: What investigations should be done in a patient A: As follows:
with cystic fibrosis? 1. General care:
A: As follows: • Nutritional support.
• Blood: eBC, liver function tests, creatinine, • Fat-soluble vitamin
electrolyte, coagulation study. supplement.
• Vitamin A, D, E levels. • Strict glucose control.
• Glucose tolerance test (annually). • Smoking cessation.
• Bacteriology: Cough swab, sputum culture. • Vaccination with influenza
• X-ray chest (hyperinflation, bronchiectasis). and pneumococcal
• USG of whole abdomen (fatty liver, cirrhosis, vaccines
chronic pancreatitis) 2. For respiratory problems:
• Spirometry (obstructive defect) • Regular physiotherapy (postural drainage,
• Aspergillus serology or skin test active cycle techniques, forced expiratory
techniques, etc.).
• Faecal fat analysis.
• Antibiotic for acute infective exacerbations
(oral or IV) and prophylactically (oral
flucloxacillin or nebulized colomycin or
tobramycin ).
• Symptomatic relief by mucolytic,
bronchodilators, inhaled corticosteroid. In
some patients, inhalation of recombinant
DNAse or hypertonic saline may give some
relief.
• Oxygen therapy, as needed.
• Pulmonary rehabilitation.
3. For advanced lung disease: Oxygen, diuretics
(for cor pulmonale), noninvasive ventilation,
lung or heart-lung transplantation.
4. For abdominal problems:
• Pancreatic enzyme replacement.
• If acute abdomen due to intestinal obstruction:
Bilateral bronchiectasis Nothing by mouth, N fluid and nasogastric
3 • RESPIRATORY SYSTEM __

suction should be given. Acetylcysteine given N.B. Remember the following points:
intravenously or through the nasogastric tube • Diagnosis of cystic fibrosis should be suspected
has been shown to be useful in resolving in any young patient who presents with chronic
bowel obstruction. respiratory and chronic gastrointestinal problem.
• Ursodeoxycholic acid for impaired liver • Gastrointestinal problems, malabsorption and
function diabetes mellitus in patient with cystic fibrosis is
due to pancreatic insufficiency.
• Liver transplantation may be ultimately
• Faecal elastase is used as a screening test for
needed in cirrhosis.
exocrine pancreatic dysfunction.
5. Others: Treatment for osteoporosis, arthritis, • Pseudomonas aeruginosa is the commonest
sinusitis, vasculitis and infertility. organism, causing recurrent respiratory infection.
6. Genetic counselling. • Prognosis: Median survival is now over 30 years.

DPLD (Previously Called Fibrosing Alveolitis or ILD)

Usual instructions are: N.B. Triad of clubbing, cyanosis and bilateral


• Examine the back of the chest and relevant.
lend-inspiratory fine crepitation is highly
suggestive of DPLD.
Presentation of a Case
Q: What are your differential diagnoses?
1. Inspection: A: As follows:
• The patient is dyspnoeic, hyperventilating • Bilateral bronchiectasis.
with cyanosis (mention if present). • Fibrosis of the lung due to other causes.
• Respiratory rate is 30/min. • Chronic heart failure (pulmonary oedema).
2. Palpation: Q: Why not pulmonary oedema?
• Trachea is central in position. A: In pulmonary oedema, crepitations are usually fine,
• Apical beat is in left fifth intercostal space, just present during both inspiration and expiration, and
medial to the midclavicular line. are altered by coughing.
• chest expansion on both sides is reduced If there is clubbing, it is against pulmonary oedema.
symmetrically. Also, in the history, there may be orthopnoea or PND
• Vocal fremitus is normal. or history suggestive of any cardiac disease.
3. Percussion:
• Percussion note is resonant over both lung Q: Why is there end-inspiratory crepitation in DPLD?
fields. A: In DPLD, alveoli remain collapsed. During forceful
4. Auscultation: inspiration, sudden opening of coIlapsed alveoli
• Breath sound is vesicular with prolonged produces crepitations.
expiration.
Q: What are the causes of bilateral crepitations?
• Bilateral basal end-inspiratoryfinecrepitations,
A: As follows:
unaltered by coughing, are present.
• Vocal resonance is normal. • LVF(fine crepitation, may be altered by coughing,
has other evidence of LVF).
• Bilateral brochiectasis (crepitations both in inspi-
My diagnosis is DPLD (fibrosing alveolitis or ILD). ration and expiration, are altered by coughing).
• DPLD.
Q: What relevants do you like to see?
A: As follows: Q: What are the causes of clubbing with bilateral
basal crepitations?
• Clubbing. A: As follows:
• Cyanosis (usually central). • Bilateral brochiectasis.
• Others to find out cause: Rheumatoid arthritis, • DPLD.
scleroderma, dermatomyositis, SLE, history of • Bronchial carcinoma.
drugs, to exclude secondary causes. See below. • Bilateral extensive TB.
_ SHORT CASES IN CLI~ICAL MEDICINE

Q: What is DPLD? 6. Other forms of DPLD, e.g. histiocytosis X


A: DPLD are a heterogenous group of diseases char- (Langerhans cell histiocytosis), Goodpasture
acterized by diffuse lung injury and inflammation syndrome, idiopathic pulmonary haerno-
that can progress to lung fibrosis. Previously, it was siderosis, diffuse alveolar haemorrhage,
called interstitial lung disease (ILD). iymphangioleiomyomatosis, pulmonary
alveolar proteinosis.
Q: Why is it called DPLD?
A: The term DPLD is preferred than ILD because the Q: What investigations should be done in IPF
pathological lesion involves the alveoli along with A: As follows:
interstitium. 1. Full blood count (shows different changes in
different diseases):
Q: What history do you like to take in DPLD?
• ESR may be high (polycythaemia is rare).
A: As follows:
• Lymphopaenia (in sarcoidosis).
• Onset of the disease: Acute or chronic.
• Eosinophilia (in pulmonary eosinophilia
• History of connective tissue disease
and drug reactions).
like rheumatoid arthritis, scleroderma,
dermatomyositis, SLE. • Neutrophilia (in hypersensitivity
pneumonitis) .
• History of drugs.
2. X-ray chest: Initially ground glass appear-
• Occupational and environmental history.
ance, bilateral reticulonodular shadow mainly
in lower zone. Lung size may be reduced;
Q: What are the presentations with IPF?
diaphragm may be raised. In advanced stage,
A: Patient is usually elderly, uncommon <50 years.
there may be a honeycomb appearance.
• Cough, usually dry.
3. HRCI': Helpful in early diagnosis, even when
• Progressive breathlessness, usually exertional.
chest X-ray is normal. Changes are usually
• Arthralgia, cyanosis and finger clubbing (20-50%
bilateral, peripheral and mainly in the lower
cases).
lobes. There is patchy subpleural reticular
abnormality with minimal or no ground
Q: Classify DPLD. glass changes, honeycomb (thick-walled cysts
A: DPLD is classified into six groups: of 0.5-2 ern in terminal and respiratory
1. Granulomatous DPLD (e.g. sarcoidosis). bronchioles) and traction bronchiectasis.
2. Granulomatous DPLD with vasculitis (e.g. Wege- 4. Pulmonary function tests:
ner granulomatosis, Churg-Strauss syndrome,
• Restrictive pattern (FVC and FEV] are
microscopic vasculitis). proportionately low, and ratio is normal or high).
3. Idiopathic interstitial pneumonia (lIP):
• Lung volumes are reduced (may be
a. Idiopathic pulmonary fibrosis [IPF, also
paradoxically preserved in patient with
called usual interstitial pneumonia (UIP). It
concomitant emphysema).
was previously called cryptogenic fibrosing
• Reduced carbon monoxide (CO) transfer.
alveolitis 90%1.
• Peak flow rate may be normal.
b. Idiopathic interstitial pnewnonia other than
5. Arterial blood gas: Hypoxaemia with normal or
IPF (10%):
low PaC02 (due to hyperventilation).
• Desquamated interstitial pneumonia.
6. Bronchoscopy: Bronchoalveolar lavage shows
• Acute interstitial pneumonia. increased number of cells, particularly neu-
• Nonspecific interstitial pneumonia. trophils and macrophages, There may be
• Respiratory bronchiolitis. increased lymphocytes in sarcoidosis, extrinsic
• Cryptogenic organizing pneumonia allergic alveolitis and drug-induced lung dis-
[(COP, also called bronchiolitis obliterans ease. Transbronchial lung biopsy may be needed
organizing pneumonia (BOOP)]. for confirmation.
7. Others (according to suspicion of cause):
• Lymphocytic interstitial pneumonia.
4. Pulmonary autoimmune rheumatic diseases • C-reactive protein (CRP) may be raised.
(e.g. rheumatoid arthritis, SLE). • LDH (high level indicates disease activity inD PLD).
5. Drugs (busulfan, bleomycin, methotrexate, • For sarcoidosis: Calcium may be elevated.
nitrofurantoin, amiodarone). Urinary calcium excretion and liver biopsy may
3 • RESPIRAT0

be useful. Serum ACE is an indicator of disease N.B. Lung biopsy is not a routine investigation.

l
activity. Gallium scanning may be done. Typical clinical features and HRCT are suffi-
• For autoimmune diseases: Rheumatoid factor cient for diagnosis. However, lung biopsy is
and anti-cyclic citrullinated peptide (anti- strongly indicated in young patient.
CCP) antibodies. Rheumatoid arthritis (RA)
test and antinuclear antibodies (ANA) are Q: What is the treatment of IPF?
positive in 30% cases (in low titre). A: As follows:
• Hypergammaglobulinaemia.
1. Prednisolone 0.5 mg/kg with azathioprine
8. For confirmation, lung biopsy is the defini-
2-3 mg/kg is recommended in:
tive investigation. It is done in selected cases,
if diagnosis is uncertain. Biopsy is done in the • Highly symptomatic.
following methods: • Rapidly progressive disease.
• Transbronchial biopsy. • Ground glass opacity on CT scan.
• Video-assistedthoracoscopic (VA1S)lung biopsy. • Sustained >50% fall of forced vital capacity
• Open lung biopsy in some cases. (FVC) over 3-6 month period.
2. Antifibrotic therapy (on trial):
• Interferon-v-I b.
• Pirfenidone.
• Sildenafil.
• Boseman.
• Etanercept (biological agent).
• Thalidomide.
• N-acetyl cysteine.
3. Single lung transplantation in young patient at
advanced stage. Survival is 1 year in 60% cases.
Prednisolone is given for at least 2 months and
Flbrosing alveolitis (DPLD) then tapered to a maintenance dose of 10-12.5 mg
daily.

Q: What is the prognosis of IPF?


A: Usually 3-s-year survival in 50% cases (65%
in steroid responder and 25% in steroid
non-responder).

Q: What are the complications of IPF?


A: As follows:
• Pulmonary hypertension and cor pulmonale.
• Respiratory failure.
• Others include infection and bronchial
CT scan showing fibrosing alveolltis (DPLD) carcinoma.

Consolidation

Presentation of a Case On palpation:


• Trachea: Central and apex.beat in normal position.
(Supposing right sided, mention all findings in • Vocal fremitus is increased.
right side): On percussion:
On inspection: • There is dullness (woody), and mention the
• Restricted movement. location ....
__ SHORT CASES IN CLINICAL MEDICINE

On auscultation: 2. X-ray chest: Homogenous opacity with air


bronchogram (usually found after 12-18 h).
• Breath sound is bronchial.
If it is associated with hilar adenopathy, then it
• Vocal resonance is increased and there is
whispering pectoriloquy. is suggestive of Mycoplasma pneumoniae.
• There is (mention, if any) few crepitations or 3'. Sputum: Gram staining, CIS (aerobic and
pleural rub. anaerobic).
4. Blood CjS (positive in pneumococcal pneumonia).
5. Arterial blood gas analysis.
My diagnosis is right-sided consolidation.
6. Others (according to aetiology):
Q: What are the differential diagnoses? • Pneumococcal antigen in serum.
A: As follows: • Mycoplasmaantibodydetection [agglutination,
• Lung abscess complement fixation test (CFT)], Coombs test,
• Bronchiectasis. CIS in special media.
Q: Why not lung abscess? • Antibody against virus, Chlamydia, Legionella.
A: In case of lung abscess, auscultation will show • Urinary Legionella pneumophila antigen.
coarse crepitations. Also, there may be combined • CRP (high).
features of cavitation and consolidation. Clubbing
may be present.
(In the history, there will be profuse foul-smelling
sputum. X-ray will show cavity with air-fluid level).
Q: Can it be tuberculosis?
A: In tuberculosis, usually there is no sign of
consolidation.
(Moreover, in the history, there will be low-grade
continued fever with evening rise, weight loss,
anorexia, night sweat, etc.)
Q: Why not bronchiectasis?
A: In bronchiectasis, usually there is no sign of consol-
idation. Auscultation will show coarse crepitations,
which are reduced or disappear on coughing. Club-
bing may be present.
(In the history, there is profuse sputum production,
more marked in the morning after waking from
sleep.)
Q: What is the typical character of sputum in
consolidation?
A: The sputum is usually rusty.
Q: Is there any crepitation
in consolidation? Why?
A: Crepitation may be present during resolution of
pneumonia.
Q: What investigations should be done in
consolidation?
A: As follows:
1. Hb%, TC, DC, ESR:
• In bacterial pneumonia: Polymorphonuclear
leucocytosis.
• In atypical pneumonia: Normal or slightly
increased leucocytes.
• In viral pneumonia: leucopaenia. Right-sided consolidation
3 • RESPIRATORY SYSTEM __

N.B. Chest X-ray may reveal complications, such as • Stage of grey hepatization: Persists for 4-8 days.
pleural effusion. Also there may be cavitation • Stage of resolution: 8-9 days or more.
(found in infection by Staphylococcus aureus
Q: What are the causes of community acquired pneu-
and pneumococcal serotype 3).
monia (CAP)?
Q: What is consolidation? A: It is caused by:
A: It means pneumonia, which is defined as inflam-
• Common organism: Streptococcus pneumoniae
mation in the lung parenchyma characterized by
(50%), Mycoplasma pneumoniae, Chlamydia
accumulation of secretion and inflammatory cells
pneumoniae and Legionella pneumophiia.
in alveoli.
• Others: Staphylococcus aureus, Haemophilus
Q: What are the types of pneumonia? inj1uenute, Chlamydia psittaci, Coxiella burnetii
A: As follows: (Q fever), Klebsiella, Actinomyses istaelii and viral
1. Anatomically of two types: (influenza, parainfluenza, measles, respiratory
• Lobar: Commonly involves one or more lobe. syncytial virus in infancy and varicella) infections.
• Lobular (bronchopneumonia): It is Q: What are the complications of pneumonia?
characterized by nonpatchy alveolar opacity A: As follows:
with bronchial and bronchiolar inflammation.
Commonly involves both lower lobes. • Pulmonary: Lung abscess, pleurisy, pleural
2. Clinically of four types: effusion, empyema thoracis, pneumothorax by S.
• Community acquired pneumonia (CAP). aureus, fibrosis of lung, collapse, adult respiratory
• Nosocomial (hospital acquired). distress syndrome (ARDS) delayed or slow
t

• Pneumonia in immunocompromised. resolution.


• Suppurative and aspiration pneumonia. • Cardiovascular: Pericarditis, myocarditis,
endocarditis, arrhythmia, thromboembolic
Q: What are the presentations of lobar pneumonia?
disease and peripheral circulatory failure.
A: The patient may present with:
• Neurological: Meningism, meningoencephalitis.
• Fever, may be with chill and rigor. • Musculoskeletal: Septic arthritis.
• Cough, initially short, painful and dry. Later on, • CIT: Meteorism (gaseous distension of stomach,
expectoration (during resolution). Rusty sputum intestine or abdomen).
(due to Streptococcus pneumoniae). • Others: Septicaemia, renal failure, hepatitis,
• May be haernoprysis. ectopic abscess formation by S. aureus.
• Chest pain, pleuritic (may radiate to shoulder or
Q: What are the causes of slow or delayed resolution
abdomen).
of pneumonia?
• Other features: Dyspnoea, anorexia, nausea and
vomiting. A: Delayed resolution means when the physical signs
persist for more than 2 weeks and radiological fea-
Q: What are the precipitating factors of pneumonia? tures persist for more than 4 weeks after antibiotic
A: As follows: therapy. Causes are:
• Streptococcus pneumonia often follows viral
• Incorrect microbiological diagnosis.
infection with influenza or parainfluenza virus.
• Fungal, tubercular or atypical pneumonia.
• Hospitalized ill patient.
• Improper antibiotic or insufficient dose.
• Smoking.
• Bronchial obstruction (bronchial carcinoma,
• Alcohol excess.
adenoma, foreign body).
• Bronchiectasis (e.g. in cystic fibrosis).
• Empyema or atelectasis.
• Bronchial obstruction.
• Imrnunocomprornised patient [HIV, diabetes
• Lmmunosuppression.
mellitus (OM), lymphoma" leukaemia, multiple
• IV drug abuser.
myeloma).
• Inhalation from oesophageal obstruction.
Q: What are the causes of recurrent pneumonia (three
Q: What are the pathological stages of CAP?
or more separate attacks)?
A: Four stages:
A: As follows:
• Stage of congestion: Persists for 1-2 days.
• Stage of red hepatization (red and solid-like • Bronchial obstruction (bronchial carcinoma,
liver): Persists for 2-4 days. adenoma, foreign body).
_ SHOR-T CASES IN CLINICAL MEDICINE

• Lung disease (bronchiectasis, lung abscess, cystic • Shock


fibrosis, sequestrate segment of lung: Commonly • Depressed consciousness.
left lower lobe).
Q: What is nosocomial pneumonia? What are the
• Aspiration (achalasia cardia, scleroderma,
causes and predisposing factors?
pharyngeal pouch).
A: New episode of pneumonia occurring at least
• Immunocompromised patient (HIV, DM,
2 days after admission in the hospital is called noso-
lymphoma, leukaemia, multiple myeloma).
comial pneumonia.
N.B. Remember the following points: Causes are: If it occurs within 4-5 days of admission
(early onset), organisms are similar to CAP.However,
• CAP usually spreads by droplet infection and
if it occurs later (late onset), common organisms are-
most cases occur in previously healthy individual.
• Physical signs of consolidation usuaJly appear • Gram-negative Enterobacteriaceae (Escherichia
within 2 days, and disappear within 2 weeks coli, Klebsiella and Pseudomonas aeruginosa] are
with proper treatment. common.
• Radiological opacity appears within 12-18 h, • S. aUTeus including methicillin resistant S. aureus
and disappears within 4 weeks with proper (MRSA).
treatment. • Anaerobic organism.
• If radiological opacity persists after 8 weeks Predisposing factors for nosocomial pneumonia:
(with treatment), it is called nonresolution. • Elderly patient.
• Bed bound, unconscious [e.g. cerebrovascular
Q: What are the criteria of assessment of severity of
accident (CVA)).
CAp?
• Postoperative case (thoracic or abdominal
A: CURB-65 criteria may be used for the assessment of
surgery).
severity of CAP. One point is scored for each of the
• Malignancy.
following features:
• Diabetes mellitus.
• Confusion (mini mental score 8 or less or new • Use of steroid, cytotoxic drugs, antibiotics.
disorientation in person, place or time). • Prolonged anaesthesia, intubation, tracheostomy,
• Urea >7 mrnol/L or >20 mgfdl. N canula.
• Respiratory rate >30/min. • Achalasia of cardia, dysphagia due to any cause,
• Blood pressure (systolic BP <90 mmHg and vomiting.
diastolic BP <60 mmHg). • Bulbar or vocal cord palsy.
• Age >65 years. • Nasogastric intubation.
This score is used for management: • Abdominal sepsis, infected emboli.

• Score 0 or 1: Home treatment. Q: How to diagnose nosocomial pneumonia?


• Score 2: Hospitalization. A: After admission in the hospital, associated with pre-
• Score 3 or more: Manage in hospital, may require disposing factors, if the patient develops purulent
ICU. sputum, fever associated with radiological infiltrate,
leucocytosis or leucopaenia unexplained increase in
Other markers of severity of pneumonia: oxygen requirement.
• Chest X-ray: More than one lobe involved.
Q: How to treat nosocomial pneumonia?
• Pa02 <8 kPa.
A: Empirical antibiotic therapy should be started intra-
• Low albumin «35 gIL).
venously. It should cover Gram-negative organisms:
• White cell count (<4000/cmm or >20,000/ cmm).
• Blood culture positive. • A third-generation cephalosporin (e.g. cefo-
t.axime) with an aminoglycoside (gentamicin) or
Q: What are the indications for referral for ITU?
• Meropenem or
A: As follows: • A monocyclic ~-lactam (e.g. aztreonam) and
• CURB score 4-5. flucloxacillin.
• Persistent hypoxia despite high concentration of • If MRSA is suspected, it is treated with
oxygen (P02 <8 kPa or 60 mmHg). N vancomycin. When possible, oral therapy may
• Progressive hypercapnoea. be considered with doxycycline, rifampicin or
• Severe acidosis. linezolid.
3 • RESPIRATORY SYSTEM _

• If pseudomonas infection is suspected, IV cipro- • Rising antibody titre [compliment fixation tests
Iloxacin or ceftazidime or doripenem should be (CFT)J for Mycoplasma pneumoniae.
given. • Others (CFT and haemagglutination test).
• Physiotherapy and oxygen, fluid and nutritional
Extrapulmonary complications of M. pneumaniae:
support should be given.
• Maculopapular skin rash, erythema multiforrne
N.B. Aspiration pneumonia is also common in and Stevens-Johnson syndrome.
hospital and involves multiple organisms • Myocarditis and pericarditis.
with anaerobe. Treatment: IVCo-amoxiclav or • Haemolytic anaemia (Coombs test may be
cefuroxime plus metronidazole. positive) and thrombocytopaenia.
• Meningoencephalitis, Cuillain-Barre syndrome
Q: What is bronchopneumonia? (GBS) and other neurological abnormalities.
A: It is defined as wide spread diffuse patchy alveolar • Myalgia and arthralgia.
opacity associated with bronchial and bronchiolar • Gastrointestinal symptoms like vomiting,
inflammation, often affecting both lower lobes. In diarrhoea.
children, it occurs as a complication of measles or
whooping cough; and in elderly, a complication Treatment:
following bronchitis or influenza. • Clarithromycin 500 mg twice daily orally or IV
or erythromycin 500 mg 6 hourly orally or rv for
Q: What is typical pneumonia? 7-10 days.
A: Typical pneumonia is characterized by high tem- • Or, doxycycline 100 mg twice daily.
perature with cough, pleuritic chest pain, features of • Rifampicin 600 mg 12 hourly.
consolidation, caused by S. pneumoniae, S. aureus,
etc. Respiratory symptoms are more with constitu- Q: How to diagnose and treat Legionella pneumophila?
tional symptoms. A: Three patterns of Legionnaires' disease may occur:
• Outbreak of infection is usually associated with
Q: What is atypical pneumonia?
contaminated water supply or cooling system, or
A: When pneumonia is caused by Mycoplasma,
from stagnant water in cistern or shower head.
Legionella, Coxiella, Chlamydia. In these cases, con-
• Sporadic case, where source is unknown. It is
stitutional symptoms are more than respiratory
usually common in middle-aged and elderly,
symptoms. Features are:
more in smokers.
• Gradual onset. • Outbreaks may occur in immunocompromised
• Dry cough. patients, e.g. those on corticosteroid therapy.
• Low-grade fever. Diabetes and chronic kidney disease (CKD) also
• Constitutional symptoms are more than increase risk.
respiratory symptoms (headache. myalgia, Features are: Initially viral-like illness with high
fatigue, nausea, vomiting). fever, chill and rigor, malaise, myalgia and headache.
• Less physical finding in the chest. Initially dry cough, later productive and purulent.
There may be nausea, vomiting, diarrhoea and pain
N.B. However, the term atypical pneumonia is abdomen. Mental confusion and other neurological
abandoned. signs; even coma may be present. Occasionally, renal
failure and haematuria may be seen.
Q: How to diagnose and treat Mycoplasma pneumoniae!
Investigations:
A: It is common in children and young adults, and
is usually associated with headache, malaise and • WBC: Lymphopaenia without marked
severe cough. Physical signs are less marked. leucocytosis.
Epidemics occur in cycle every 3-4 years. • Chest X-ray: Usually shows lobar and then
multilobar shadowing. A small pleural effusion
Investigations: may be present. Cavitation is rare.
• WEC (normal). • Hyponatraemia.
• Chest X-ray (commonly lower lobe involvement), • Hypoalbuminaemia .
may show bilateral patchy consolidation. • High serum aminotransferases, creatine
• Cold agglutinin (positive in 50% cases). phosphokinase.
_ SHORT CASES IN CLINICAL MEDICINE

• Direct immunofluorescent for Legionella in pleural iv. Arnoxicillin 1 g 6 hourly IV.


fluid, sputum or bronchial washings. Culture on v. If S. aUTeus is suspected: Fludoxacillin 2 g
special media can be done, but takes 3 weeks. 6 hourly N or sodium fusidate is added.
• Legionelia serology: fourfold rise is highly
Q: What are the aiteria for discharge of a patient with
suggestive.
pneumonia from hospital?
• Urine for antigen (highly specific). A: To discharge, the patient should be clinically sta-
• Urine R/E shows haematuria. ble with no more than one of the following clinical
Treatment: signs:

• Clarithromycin 500 mg twice daily orally or IIV • Temperature >37.8°C.


or Erythromycin 500 mg 6 hourly orally or IfV • Heart rate> 100/min.
for 7-10 days. • Respiratory rate >24/min.
• Rifampicin 600 mg 12 hourly. • Systolic BP <90 mmHg.
• $a02 <90%.
Prognosis: 10% mortality (may be up to 30% in • Inability to maintain oral intake.
elderly). • Abnormal mental status.
Q: How to treat pneumonia? Q: What are the differences between bacterial and viral
A: Sputum should be sent for CIS and before starting pneumonia?
antibiotic. The treatment involves: A: As follows:
l. General treatment: Rest, 02 therapy, adequate Points Bacterial Viral
hydration and chest physiotherapy. pneumonia pneumonia
2. Antibiotic (empirically with suspicion of cause).
Onset Abrupt Lessabrupt
Community acquired pneumonia (CAP):
History of upper Absent Present
1. Mild CAP: respiratory tract
• Arnoxicillin 500 mg 8 hourly orally or eryth- infection
romycin 500 mg 6 hourly or clarithromycin Systemic features High-grade fever Low-grade fever,
500 mg twice daiJy or azithromycin 500 mg with chill and rigor headache and
daily. malaise
• If S. aureus is suspected: Clarithromycin 500 mg
Cough Initially dry, later Usually dry
twice daily orally or N, plus flucloxacillin 1-2
rusty sputum or (extrathoracic
g 6 hourlyN.
purulent complains are
• If Klebsiella is suspected: Ciprofloxacin 200 more)
mg IV 12 hourly, plus gentamycin 60-80 mg
N 8 hourly (see serum creatinine level) or Pleuritic chest Common Lesscommon
gentamycin plus ceftazidime 1 g N 8 hourly. pain (respiratory
complains are less
Duration of treatment: 7-10 days (up to 14 days). common) and
constitutional
• If Mycoplasma, Legionella or atypical organism
symptoms
is suspected: Clarithromycin 500 mg twice daily
predominate
orally or erythromycin 500 mg 6 hourly orally,
or tetracycline or doxycycline may be used. Physical signs of Well marked Minimal
consolidation
Duration of treatment: 2-3 weeks.
Blood count Leucocytosis Normal or leuco-
2. Severe CAP: paenia
• Clarithromycin 500 mg twice daily N or Chest X-ray Lobar or segmen- Mottling or
erythromycin 500 mg 6 hourly IV.. tal homogeneous reticular pattern or
• Plus add one of the following - opacity with air patchy opacity or
i. Co-arnoxiclav 1-2 g 8 hourly N. bronchogram streaky
ii. Cefuroxirne 1.5 g 8 hourly N
Sputum culture Organism found No organism
iii. Ceftriaxone 1-2 g TV daily.
3 • RESPIRATORY SYSTEM ~

Lung Abscess

Presentation of a case: (Present as in consolidation)


My diagnosis is lung abscess.

Q: What are the differential diagnoses?


A: As follows:
• Consolidation (during resolution stage).
• Bronchiectasis.

Q: Why not consolidation?


A: Because in lung abscess:
• The patient is toxic with high temperature.
• Clubbing is present, which is against
consolidation.

Q: With these findings, can it be consolidation only?


A: Yes, it may be during resolution stage. Multiple lung abscesses

Q: Could it be bronchiectasis?
A: Yes, because there is clubbing with coarse crepita- Q: What is lung abscess? What are the causes?
tions. However, in bronchiectasis, crepitations are A: It is a localized area of suppuration within the lung
mostly on the basal area. Also, there is no fever and parenchyma that leads to parenchymal destruction
patient is not toxic in bronchiectasis (until there is and is manifested radiologically as a cavity with air-
secondary infection). fluid level (not due to 1'8).

Causes of lung abscess:


Q: What investigations should be done in lung
abscess? • Aspiration of nasopharyngeal or oropharyngeal
A: As follows: contents: Such as in vomiting, anaesthesia, tooth
• CBC (leucocytosis). extraction, tonsillectomy, unconscious patient,
• X-ray chest (cavity with air-fluid level). alcoholism and achalasia of cardia. Organisms
• Sputum examination: Gram staining, CIS (both are aerobic and anaerobic.
aerobic and anaerobic), AfB, fungus, malignant • Specific infections (5. pneumoniae type 3, S. aureus,
cells. K. pneumoniae and fungal). In HIV, Pneumocystis
• Bronchoscopy (to exclude mass and foreign body). jiroveci, Cryptococcus neojormans and Rhodococcus equi.
• CT or MRI (in some cases). • Obstruction by bronchial carcinoma, adenoma
• Blood sugar. and foreign body.
• Infection in pulmonary infarction (by
S. pneumoniae, S. aureus, Haemophilus inj1uenzae
and anaerobic).
• Spread from liver abscess and subphrenic abscess
(due to transdiaphragmatic spread).
• Haernatogenous from other infection as septic
emboli (pelvic abscess and salpingitis, right-
sided endocarditis, IV drug abuse).

Q: How the patient of Iung abscess usually presents?


A: As follows:
• Severe cough with profuse foul-smelling sputum,
may be foetid (anaerobic).
• Haernoprysis.
Left-sided lung abscess • Chest pain (pleuritic).
__ S~ORT CASES IN CLINICAL MEDICINE

• Fever, usually high with chill and rigour, with • Broad-spectrum antibiotic: Amoxicillin or Co-
profuse sweating. amoxiclav or erythromycin plus metronidazole.
• Malaise, weakness and loss of weight. Or, cefuroxime 1 g IV 6 hourly plus metro-
nidazole 500 mg IV 8 hourly for 5 days, followed
Q: What are the physical findings in lung abscess? by cefaclor plus metronidazole (in 70% cases
A: It depends on site. If deep seated within the lung anaerobic organisms are present, but mixed
parenchyma, there may not be any physical find- organisms are also common).
ings. If it is near the surface, findings are: • If improves, continue as above. If no response,
• Features of consolidation, usually. antibiotic is given according to CIS. Treatment
• Rarely, features of cavitation. should be continued for 4-6 weeks.
• Sometimes, combined features of consolidation • Postural drainage and chest physiotherapy.
and cavitation, if large abscess.
• If no response to medical therapy (occurs in
Q: What are the characteristics of sputum in lung 1-10% cases), percutaneous aspiration (USC or
abscess? cr guided).
A: If the sputum is kept in a bottle, there are three • Sometimes, surgery (lobectomy) may be done.
layers (as in bronchiectasis): • Treatment of the cause, if present.
• Lower: Sediment (epithelial debris and bacteria). Indications of surgery:
• Middle: Thick liquid.
• Upper: Frothy. • No clinical response.
• Increasing size of the abscess.
Q: What is the common site oflung abscess? • Massive haemorrhage or haemoptysis.
A: As follows:
Other causes of cavitary lesion in lung
1. Lung abscess is common in right middle lobe.
2. If it is due to aspiration, then the commonest • Tuberculosis.
site depends on the posture of the patient during • Cavitating bronchial carcinoma.
aspiration. • Pulmonary infarction.
• If the patient is lying down, abscess forms • Fungal infection (histoplasmosis).
in the posterior segment of upper lobe or • Wegener granulomatosis.
superior segment of lower lobe.
• Rheumatoid nodules.
• If the patient is in upright position, the
• Consolidation (St. pneumoniae serorypes 3).
common site is basal segment.

Q: Why lung abscess is more common on the right side? Q: What are the causes of multiple cavitary lesions in
A: Lung abscess is more common in right side due to the lung?
less obliquity of the right major bronchus. A: As follows:
• Tuberculosis.
Q: What are the complications of lung abscess?
• Staphylococcal lung abscess (in children).
A: As follows:
• Fungal infection.
• Pleurisy.
• Empyema. • Klebsiella.
• Bronchiectasis. • Amoebiasis.
• Fibrosis.
• Septicaemia. Q: What are the causes of multiple lung abscess?
• Cerebral abscess (common in parietal lobe or A: As follows:
posterior frontal region). • Aspiration of infected material.
• Amyloidosis (rare), in chronic cases. • S. aureus (in children).

Q: How to treat lung abscess?


• Klebsiella infection.
A: Sputum is sent for CIS and broad-spectrum • Fungal infection.
antibiotic should be started. • Amoebiasis.
Collapse of Lung
--
N.B. Signs of collapse depend on patency of the
Presentation of a Case bronchus:

(Supposing right sided, mention the findings in • If bronchus is completely obstructed (central
collapse), all the signs will be collapsed
right side)
(absent or reduced).
On inspection: • If bronchus is patent (peripheral collapse),
• Restricted movement. there is bronchial sound and increased vocal
• Slight flattening of upper part and crowding of ribs. resonance.
On palpation: Q: Why not this is a case of pleural effusion?
• Trachea: Shifted to the right. A: In pleural effusion, trachea and apex beat are shifted
• Apex beat: Shifted to the right (mention where). to the opposite side.
• Vocal fremitus: Reduced or increased. Q: Why not this is a case of thickened pleura?
On percussion: A: In thickened pleura, there is no shifting of trachea
• Dullness in upper part of chest (mention and apex beat.
up to ... , the position). Q: Why not consolidation?
• Upper border of liver dullness is in the right A: In consolidation trachea and apex beat will not be
fourth intercostal space in the middavicular line. shifted; percussion is woody dull.
On auscultation: Q: What are the types of collapse?
• Breath sound: Bronchial or reduced . A: As follows:
• Vocal resonance: Increased or decreased, there is a. Etiologically:
whispering pectoriloquy (present, if bronchial 1. Compression collapse (passive): Due
sound is present). to pleural effusion, pneumothorax and
hydropneumothorax. Trachea and apex beat
are shifted to the opposite side.
2. Absorption collapse (active): Caused by
My diagnosis is right-sided lung collapse. bronchial obstruction due to any cause. Air
is absorbed distal to the obstruction and
Q: Could it be fibrosis? Or what is your differential
affected part is collapsed. Causes are:
diagnosis?
A: It may be fibrosis. Signs of collapse and fibrosis are • Neoplasm (bronchial carcinoma,
almost similar. However, some findings are highly adenoma).
suggestive of fibrosis, as it is a chronic process: • Foreign body (inert object and artificial
• Flattening of chest, teeth).
• Mucous plug (due to bronchial asthma,
• Crowding of ribs, and
cystic fibrosis, postoperative).
• Drooping of shoulder on the affected side.
• AspergiIloma.
These findings may be present in collapse also, if it • Pressure on bronchus from outside (enlarged
is prolonged. All other signs of collapse are same as lymph node, aortic aneurysm, pericardial
in fibrosis, but with less degree. To be confirmed, effusion causing collapse of left lung and
chest X-ray will help, which shows: enlarged left atrium in mitral stenosis).
• Rarely, congenital collapse due to absence
• In collapse, the X-ray shows homogeneous
of surfactant.
opacity. Evidence of bronchial obstruction (mass
lesion) may be seen, and diaphragm may be b. Anatomically:
elevated. 1. Central: Mass lesion (shows more signs and
• In fibrosis, nonhomogeneous opacity, rib symptoms).
crowding and ring shadow due to dilatation of 2. Peripheral: Usually small collapse (shows
bronchi within fibrosis may be seen. less signs and symptoms).
__ SHORT CASES IN CLINICAL MEDICINE

Causes of collapse according to age:


• Neonate: Congenital due to low or absent
surfactant, aspiration of meconium.
• Children: Foreign body.
• Young: Bronchial adenoma and foreign body.
• Elderly: Bronchial carcinoma and foreign
body (artificial tooth).
Q: How can you differentiate central and peripheral
collapse clinically? Right upper lobe collapse Left lower lobe collapse
A: As follows:
• In central collapse: Bronchus is completely
obstructed. So, breath sound is diminished or
absent. Vocal resonance is diminished or absent.
• In peripheral collapse: Bronchus is patent. So,
breath sound is bronchial and vocal resonance is
increased.

N.B. Commonest cause of collapse:


• Central: Bronchial carcinoma or adenoma,
enlarged lymph node or foreign body.
• • Peripheral: Mucus plugging, bronchial cast. Left lung collapse

Q: What investigations should be done in collapse of Q: What is middle lobe syndrome?


lung? A: Middle lobe syndrome is the recurrent or per-
A: As follows: sistent atelectasis of right middle lobe. The right
middle lobe originates at acute angle and is com-
• CBC. pletely surrounded by lymph nodes. It is frequently
• Chest X-ray, PAview (homogenous opacity, heart obstructed by enlarged lymph nodes due to malig-
and lung shifted to the affected side), and lateral nancy or TB causing collapse of the lobe. There may
view, if needed. be bronchiectatic changes. Foreign body may also
• cr scan of chest. be responsible.
• Bronchoscopy (to see any obstruction, removal Diagnosis:
of foreign body, aspiration of any material, • cr scan or MRI.
biopsy). • Bronchoscopy is also helpful.

- - -- -

Fibrosis of lung

Q: What is the more likely diagnosis in this case? Why?


Presentation of a Case I
I
A: Fibrosis of the left lung because there is flattening
of upper part of chest, crowding of ribs, drooping
(Supposing left sided) of the shoulder, which indicates fibrosis that is a
• There is flattening of upper part of chest, chronic process.
crowding of ribs, drooping of the shoulder. Q: Can it be collapse also?
• Plus, other findings as in collapse (see collapse A: Yes, if it is longstanding or chronic.
of the lung).
Q: What are the causes of fibrosis?
A: Localized fibrosis may occur in:
My differential diagnoses are:
• Fibrosis of the left lung. • Upper lobe: TB (the commonest).
• Collapse of the left lung. • Lower lobe: bronchiectasis (the commonest).
3 • RESPIRATORY SYSTEM __

Causes of upper lobe (zone) fibrosis-mnemonic:


SCHART
• S: Silicosis, sarcoidosis.
• C: Coal worker's pneumoconiosis.
• H: Histiocytosis and histoplasmosis.
• A: Allergic bronchopulmonary aspergillosis and
ankylosing spondylitis.
• R: Radiation (typically in mid zone).
• T:TB.
Causes of lower lobe (zone) fibrosis-mnemonic: Bilateral apical fibrosis
Fibrosis of the left lung
RASIO
• R: Rheumatoid arthritis.
• A: Asbestosis.
• S: Scleroderma.
• I: Idiopathic pulmonary fibrosis.
• 0: Others (drugs: bleomycin, busulphan,
nitrofurantoin, methotrexate and amiodarone).
N.B. Generalized fibrosis involving both the lungs
occur in DPLD.

Q: What are the types of fibrosis of the lung?


A: As follows:
• Focal fibrosis due to inhalation of mineral dust
(pneumoconiosis) .
• Replacement fibrosis (damaged lung tissue
replaced by fibrosis), e.g. TE.
• Interstitial fibrosis due to any cause of DPLD
(RA, systemic sclerosis or SLE). Drooping of left shoulder with flattening of the chest

--

Chronic Obstructive Pulmonary Disease (COPO)

Presentation of a Case • Cricosternal distance (distance between suprasternal


notch and cricoid cartilage) is reduced (normally
• The patient is dyspnoeic with pursing of lips. three fingers or more).
On inspection: • Apex beat is not felt.
• The patient is dyspnoeic with pursing of lips, and • Chest expansion is reduced and chest movement
respiratory rate is 30/min. is vertical.
• The chest is barrel shaped. • Vocal fremitus is reduced on both sides.
• Indrawing of lower intercostal space on On percussion:
inspiration (due to low flat diaphragm).
• Increased resonance or hyperresonance in both
• Suprasternal and supraclavicular space excavation.
lung fields.
• Prominent accessory muscles of respiration.
• Obliteration of liver and cardiac dullness (liver
On palpation: dullness may be lower down).
• Trachea, central, tracheal tug is present (descent
of trachea during inspiration).
_ SHORT CASES IN CLINICAL MEDICINE

On auscultation: • Other tests: Lung volumes may be normal


or increased. Gas transfer coefficient of
• Breath sound, diminished; but vesicular with carbon monoxide is low, when significant
prolonged expiration. emphysema is present.
• Few rhonchi may be present, if associated with 6. PEFR (reduced).
chronic bronchitis. 7. Blood gas analysis:
• Vocal resonance normal. • Often normal at rest.
• P02 (reduced).
My diagnosis is COPD, more likely pulmonary • PC02 (normal or increased).
emphysema. • pH (acidosis).
8. High-resolution Cf: Assessment of COPD,
N.B. If plenty of rhonchi are present in both lung characters of emphysema, particularly bullae.
fields, then diagnosis is chronic bronchitis with 9. Sputum examination (if superadded infection).
emphysema. 10. (XI-antitrypsin deficiency: May be done in young,
nonsmoker patient with basal emphysema.
Q: What is your differential diagnosis?
A: Chronic, severe or persistent bronchial asthma.

Q: What is the basic difference between bronchial


asthma and COPD?
A: Bronchial asthma is reversible, but COPD is not
fully reversible and is progressive.

Q: How to confirm COPD?


A: By spirometry and reversibility test.

Q: What are the findings in spirometry?


A: As follows:
• FEV\ <80% predicted.
• FEVl: PVC <70% predicted.
• Bronchodilator reversibility test shows <15%
increase in FEV, after giving bronchodilator.

Q: What investigations should be done in COPD?


A: As follows: Emphysema
l. Complete blood count (there may be poly-
cythaemia and increased PCV due to persistent
hypoxaernia}.
2. Chest X-ray PA view (there may be features of
hyperinflation: Increased translucency, low
flat diaphragm, tubular heart, widening of
intercostal space, emphysematous bullae).
3. ECG (usually normal. In cor pulmonale, there
may be features of right ventricular hypertrophy
(RVH).
4. Echocardiogram (may show features of cor
pulmonale).
5. Lung function tests:
• FEVl and PVCare reduced. Ratio of FEVj:PVC
is also reduced (indicates obstructive airway
disease).
• Postbronchodilator FEVj <80% of the
predicted value and FEVJPVC is <70%. Lip pursing
3 • RESPIRATORY SYSTEM __

Q: What is COPD? • More in male and Caucasians.


A: It is a disease characterized by airflow limitation, • Biofuel mass.
which is chronic, slowly progressive, and not fully
Q: What organisms are associated with acute exacerba-
reversible (bronchial asthma is reversible). It is
tion of COPD?
diagnosed by history and spirometry. Postbron-
A: Common organisms: Haemophilus injluenzae and
chodilator shows FEVl < 80%, predicted, and
S. pneumoniae. Other less common organisms are
FEVl:FVC<70%, predicted. COPD includes chronic
Moraxella catarthalis, Chlamydia pneumoniae and
bronchitis and emphysema.
Pseudomonas aeruginosa.
Q: What is the mechanism of airflow limitation in
Q: What are the stages or classification of COPD?
COPD?
A: According to the GOLD criteria, COPD is classified
A: As follows:
as follows:
• Increased mucus production and reduced
mucociliary clearance. Stage Spirometry Symptoms
• Loss of elastic recoil. 0- At risk Normal Presence of
• Increased muscle tone. chronic symptoms
• Pulmonary hyperinflation. (cough, sputum
production)
Q: What are the presentations of CO PO?
A: Usually the patient is above 40 years, male and I-Mild FEV/FVC <70% None or mild

smoker. Features are: FEV, ~ 80%


predicted
• Chronic cough and sputum production, which is II-Moderate FEV/FVC <70% Mild-to-moderate
progressively increasing. FEV, ~ 50%, but symptoms
• Progressively increasing breathlessness. <80% predicted
• There may be haernoptysis, oedema and morning 111-Severe FEV/FVC <70% Breathlessness on
headache (due to hypercapnia). FEV, ~ 30%, but minimal exertion,
Q: What are the systemic features in COPD? <50% predicted e.g. dressing
A: Muscular weakness, peripheral oedema due to IV - Very severe FEV,IFVC <70% Breathlessness at
impaired salt and water excretion, weight loss due FEV, <30% rest
to altered fat metabolism, increased osteoporosis, predicted or FEV)
increased circulating inflammatory markers. <50% predicted
plus chronic
Q: What are the risk factors or causes of COPD? respiratory failure
A: Multiple factors may be responsible for COPD,
such as: Q: What are the complications of COPD?
A: As follows:
l. Exposure to:
• Smoking (the commonest): Active or passive. • Pulmonary hypertension.
• Indoor and outdoor air pollution. • Cor pulmonale.
• Occupation: Exposure to dust, fumes, smokes, • Respiratory failure.
chemicals, etc. (e.g. coal miners and those • Secondary infection.
who work with cadmium). • Polycythaemia.
• Urban dweller. Q: How to manage COPD?
• Low socioeconomic status. A: As follows:
• Low birth weight.
1. Smoking must be stopped.
• Poor lung growth that may be due to
2. Avoidance of dust, fume, smoke, etc.
childhood infections or maternal smoking.
3. Drug therapy according to the stage:
• Infections: Recurrent lung infection, persistent
adenovirus in lung tissue, HJV infection is
I-Mild • Avoid of risk factors, influenza
associated with emphysema. vaccination
• Cannabis smoking (controversial). • Short-acting inhaled bronchodilator
2. Host factors: P2
like agonist (salbutamol, terbutaline)
• Genetic factors: (Xl-antitrypsin deficiency. or anticholinergic (ipratropium) when

• Airway hyperreactivity. needed


_ SHORT CASES IN CLINICAL MEDICINE

II - Moderate Above treatment plus: Q: What is the role of inhaled steroid in COPD?
• Regular treatment with one or more A: Inhaled steroid is recommended for symptomatic
long-acting bronchodilator like ~2 patient with moderate-to-severe COPD and for
agonist (e.g. salmeterol, formoterol) patients with frequent exacerbations, but not in
or anticholinergic (tiotropium) when mild COPD. It reduces the frequency and sever-
needed ity of exacerbation. There is small improvement of
• Rehabilitation FEY];but it does not alter the natural history of FEVI
III - Severe Above treatment plus: decline.
• Inhaled steroid (fluticasone)
Q: What are the indications of long-term domiciliary
IV - Very severe Above treatment plus:
oxygen therapy in COPD?
• Long term oxygen, if chronic respiratory
A: As follows:
failure
• Surgical treatment, if needed 1. Pa02 <7.3 kPa (55 mmHg) irrespective ofPaC02
and FEV]<1.5 L.
4. Other therapy: 2. Pa02 7.3-8 kPa (55-60 mmHg) associated
• Oxygen, if needed. with:
• Mucolytic (acetylcysteine). • Pulmonary hypertension.
• Antibiotics (if infection). • Peripheral oedema.
• Diuretic (if oedema). • Nocturnal hypoxaemia.
• Pulmonary rehabilitation. • Secondary polycythaemia.
• Pneumococcal vaccination. 3. Carboxyhaemoglobin <3% (in patient who
• Reduction of obesity. have stopped smoking).
5. Surgical intervention: 4. Terminally ill patient of whatever cause with
• Bullectomy: If young patient, large bulla Pa02 <7.3 KPa.
compressing surrounding lung tissue, no N.B. Arterial blood gases should be measured in
generalized emphysema. clinically stable patients on optimal medical
• Lung volume reduction surgery (LVRS): therapy on at least two occasions, 3 weeks apart.
Predominant upper lobe emphysema,
preserved gas transfer and no evidence of Q: Why low concentration 02 is given in COPD? Or
pulmonary hypertension. what happens when high-flow 02 is given?
• Lung transplantation. A: In COPD; the patient is dependent on hypoxic
Q: How domiciliary oxygen is given? What is the aim drive. Low-flow oxygen is due to its effect in the
of the therapy? chernoresponsiveness of the respiratory centres in
A: 02 is given 2-4 L/min for 15 h/day by nasal prongs. the medulla (part of the brainstem). High-flow oxy-
The aim is to increase the Pa02 to at least 8 kPa gen blunts responsiveness of the respiratory centre
(60 mmHg) at sea level during rest or $a02 to at in the medulla and causes carbon dioxide retention,
least 90%. (Greater benefit may be seen in patients so aggravates respiratory failure (type 2 respiratory
who receive> 20 h/day.) failure).
Q: What is the role of oral steroid in COPD? What is
N.B. Regarding air travel: the indication of steroid in COPD?
• Preflight assessment should be done by A: Oral steroid is useful during exacerbation, but
spirometry and a hypoxic challenge test with maintenance therapy should be avoided.
15%oxygen.Tfsaturation is maintained >90%,
the patient can be allowed to travel. If not, air Indications are:
travel should be avoided or undertaken only • Stage III or IV disease.
with inspired oxygen therapy. • InstageII, iforalsteroid trial shows responsiveness.
• Sufficient supplementary oxygen should be • Severe exacerbations of COPD.
given during flight to keep the Pa02 above • Frequent episodes of exacerbations.
50 mmHg, which is achieved by increasing Q: What is the role of inhaled steroid in COPD?
the flow by 1-2 L/min. A: Inhaled steroid is recommended for symptomatic
• Patients who use to take continuous oxygen patients with moderate-to-severe COPD and for
at home will require this supplementation. patients with frequent exacerbations; but not in
3 • RESPIRATORY SYSTEM __

mild COPD. It reduces the frequency and severity • Chest physiotherapy. Secretions should be
of exacerbation. There is small improvement of removed by suction.
FEV" but it does not alter the natural history ofFEV, • Respiratory support: If above treatment fails or
decline. there is tachypnoea and acidosis (pH <7.35).
Noninvasive ventilatory technique like bilevel
Q: What is the prognosis of COPD?
positive airway pressure (BiPAP) is used first.
A: Prognosis of COPD is predicted by BODE index:
Continuous positive airway pressure (CPAP) is
occasionally needed.
Points on BODE index
• Respiratory stimulant: Less used. Ooxapram 1.5-
Variable 0 1 2 3
4.0 mg/rnin by slow IV infusion may be helpful.
Body mass index >21 ~21
N.B. Many patients with exacerbation of COPD can
Obstruction to airflow ~65 50-64 36-49 ~35
be managed at home with increased bronchodi-
(FEV,%predicted)
lator, short course of oral steroid and antibiotic.
Dyspnoea (MMRC scale) 0-1 2 3 4
Exercise capacity ~350 250-349 150-249 ~149 Indications of hospitalization are:
(metres walked in 6 min) • Severe symptoms or acute worsening that fails to
respond to outpatient management.
4-year mortality rate for BODE index 0-2 is 10%, • Presence of cyanosis.
whereas that of BODE index 7-10 is 80%. • Peripheral oedema.
N.B. Poor prognostic factors are advancing age • Alteration of consciousness.
(inversely related), fall of FEV1 over time, • Comorbidity and poor social circumstances.
weight loss and pulmonary hypertension. Q: What are the discharge criteria of COPD patient?
A: As follows:
Q: Howto manage acute exacerbation of COPO (type II
• The patient should be in a clinically stable condition
respiratory failure)?
and no parenteral therapy should be there for 24 h.
A: As follows:
• Inhaled bronchodilators are required less than
• Oxygen: Continuous low concentration oxy- 4 hourly.
gen via Venturi mask to raise PaOz >8 kPa • Oxygen delivery has ceased for 24 h.
(60 mmHg). Initially 24 or 28% oxygen is given • The patient is able to eat and sleep without
and increased gradually, provided PaCOz does significant episodes of dyspnoea.
not rise unacceptably. If PaC02 rises and pH falls • The patient or caretaker understands and is able
below 7.25, artificial ventilation or a respiratory to administer medications.
stimulant should be given. • Follow-up and home care arrangements [e.g.home
• Bronchodilator: Nebulized short-acting ~2-ago- oxygen, home care, Meals on Wheels, community
nist (e.g. salbutamol) with an anticholinergic nurse, allied health, general practitioners (GP),
agent (e.g. ipratropium). specialist] have been completed.
• Oral prednisolone 30 mg daily for 10 days. • If previously able, the patient is ambulating safely
• Antibiotic: Given if infection is suspected. and independently, and performing activities of
• Diuretic: If peripheral oedema. daily living.

Chronic Bronchitis

Presentation of a Case • Chest expansion: Reduced.


• Vocal fremitus: Normal.
Inspection: Percussion:
• Shape of the chest: Normal. • Percussion note: Normal resonance.
• Movement of the chest: Bilaterally restricted. • Area of liver dullness: In the right fifth intercoastal
• Intercostal space: Appears full. space in midclavicular line.
Palpation: • Area of cardiac dullness: Impaired.
• Trachea: Central. Auscultation:
• Apex beat: In the left fifth intercostal space in the • Breath sounds: Vesicularwith prolonged expiration.
midclavicular line.
• Added sounds: Plenty of rhonchi, in both lung 4. Lung function tests:
fields, present in both inspiration and expiration. • FEV. (reduced).
• Vocal resonance: Normal. • FVC (reduced).
[FET(forced expiratory time): 8 s (normally, <6 s)] • Ratio of FEV1:FVCis also reduced (indicates
obstructive airway disease).
• Other tests: Residual volume (RV) is increased,
My diagnosis is chronic bronchitis.
total lung capacity (TLC) is increased, gas
N.B. In many cases, signs of emphysema and chronic transfer (either normal or mildly reduced).
bronchitis may be present together. Then, the
5. PEFR (reduced).
diagnosis is chronic bronchitis with emphysema.
6. Blood gas analysis:
Q: What are your differential diagnoses?
• P02 (reduced).
A: As follows: • PC02 (normal or increased).
• Chronic persistent bronchial asthma. • pH (acidosis).
• COPD. 7. cr scan in some cases.
• Bilateral extensive bronchiectasis.
• Chronic LVF. Q: What are the complications of chronic bronchitis?
A: As follows:
Q: What is chronic bronchitis?
A: It is defined clinically as 'the presence of cough, pro- • Respiratory failure-both type I and type II.
ductive of sputum, not attributable to other causes, • Emphysema.
on most of the days, for three consecutive months, • Secondary polycythaemia.
at least for two successive years'. • Secondary infection.
• Pulmonary hypertension.
Q: What are the causes of cmonic bronchitis?
• Cor pulmonale.
A: Multiple factors are responsible:
Q: How to treat chronic bronchitis?
• Smoking. A: As follows:
• Exposure to dust, fume, foggy environment (may
be occupational). Dampness, sudden change in 1. Smoking must be stopped.
temperature-all exaggerate chronic bronchitis. 2. Avoid air pollution (dust. fume).
• Infection (H. injluenzae, S. pneumoniae, Moraxella 3. Control of infection with appropriate antibiotic.
catarthalis: All of these exaggerate chronic 4. Bronchodilator:
bronchitis. ) • Inhaled ~-agonist: Salbutamol (200 ug 4-6
Q: How does the patient presents with chronic hourly), terbutaline.
bronchitis? • Inhaled anti muscarinic: lpratropiurn (40 ug
A: It is common in smokers. Usual presentations are: 4 hourly), tiotropium (18 ug daily), oxitro-
pium (200 ug BD).
• Cough with sputum: Cough is more marked in • Long-acting ~-agonist: Salmeterol, formoterol.
the morning on exposure to cold and during • Oral theophylline (in some cases).
winter. The sputum is mucoid or mucopurulent,
5. Inhaled corticosteroid beclomethasone (400 pg
usually not associated with haernoptysis.
BD) or budesonide or fluticasone. In severe case,
• Tightness of the chest and breathlessness on
oral prednisolone 30 mg for 2 weeks, followed
exertion.
by maintenance dose.
• At advanced stage, features of pulmonary
6. Mucolytic agents like brornhexine or
hypertension and cor pulmonaJe are present.
N-acetylcysteine (200 mg 8 hourly orally for 8
The patient looks as blue bloater (cyanosed and
weeks) may be given.
oedematous) .
7. Other measures:
Q: What investigations should be done in chronic • Chest physiotherapy.
bronchitis? • Exercise and weight reduction, if obese.
A: As follows: • Long-term domiciliary oxygen.
1. Full blood count. • Pulmonary rehabilitation.
2. Chest X-rayPAviews (no significant abnormality). • Annual influenza vaccine,S yearly
3. ECG. (Usually normal. In cor pulmonale, there pneumococcal vaccine and Haemophilus
may be features ofRVH.) inJluenzae vaccine may be given.
3 • RESPIRATORY SYSTEM __

Q: How to treat acute exacerbations? • Oxygen inhalation (24%, 1-3 Ljmin).


A: As follows: • IV hydrocortisone arid oral steroid. (Steroid is
• Nebulized bronchodilators like terbutaline, only used in acute exacerbations; and, unlike
ipratropium bromide. in asthma, it does not influence the course of
• rv antibiotic to control infection. chronic bronchitis.)

Emphysema

)
• Bulla may be present (which is
Presentation of a Case )
pathognomonic).
• Prominent pulmonary arterial shadow in both
Inspection:
hilums (due to pulmonary hypertension).
• Chest is barrel shaped.
• Indrawing of the lower intercostal space on 2. Lung function tests:
inspiration (due to low, flat diaphragm). • FEV)and FVC are reduced. Ratio of FEV]:FVC
Palpation: is reduced (obstructive type).
• Trachea is central, tracheal tug (descent of trachea • Postbronchodilator FEV) is <80% predicted,
during inspiration) is present. and ratio ofFEV):FVC is <70% predicted.
• Cricosternal distance (distance between suprasternal • PEFR Reduced.
notch and cricoid cartilage): Reduced (normally • Lung volume with increased TLC and RV.
three fingers or more). 3. Arterial blood gas analysis:
• Apex beat is not felt.
• Low PC02 (due to hyperventilation).
• Chest expansion: Reduced (tell in centimetre).
• Low P02.
• Vocal fremitus: Reduced.
• Impaired gas transfer of CO.
Percussion:
4. Other investigations:
• Hyperresonance in both the lung fields.
• Obliteration of the liver and cardiac dullness • TC, DC, Hb%, ESR (polycythaemia may be
(liver dullness may be lowered down). present).
• cr scan of the chest, especially HRCT: Highly
Auscultation: suggestive.
• Breath sound: Diminished, vesicular with • To confirm: Biopsy of the lung tissue (not a
prolonged expiration. routine test).
• Vocal resonance: Reduced. • In young patient, serum level of al- antitrypsin
may be done.
• ECC may show tall P, right ventricular
My diagnosis is emphysema.
hypertrophy (RVH), right axis deviation
N.B. If plenty of rhonchi are present with above (HAD) in patient with cor pulmonale.
findings, diagnosis is chronic bronchitis with • Echocardiography.
emphysema.
Q: What is the definitive diagnosis of emphysema?
A: Histopathological (but not done routinely).
Q: What investigations should be done in emphysema?
A: As follows: Q: What is emphysema?
l. X-ray chest PIA view (shows features of A: It is the permanent distension of alveoli with
emphysema): destruction of their walls distal to the terminal
bronchioles.
• Increased translucency of both lung fields
with loss of peripheral vascular markings. Q: What are the presentations of emphysema?
• Low, flat diaphragm. A: Breathlessness on exertion and minimum cough
• Tubular heart. with lip pursing.
• Widening of intercostal space and ribs appear Q: What are the types of emphysema?
horizontal. A: Four types:
_ SHORT CASES LN CLINICAL MEDICINE

• Centriacinar: Involves the proximal part of acini, • Prominent sternomastoid and scalene muscles.
limited to respiratory bronchiole with relatively • Tracheal tug.
less change in acinus. • Reduced cricosternal distance (length of trachea
• Panacinar: All the alveoli and alveolar ducts in above suprasternal notch).
acinus are involved, both central and peripheral • Indrawing of lower intercostal space during
portion. It occurs mostly in ai-antitrypsin inspiration.
deficiency. • Horizontal ribs with wide intercostal spaces.
• Paraseptal: Along the septa, blood vessels and • Wide subcostal angle.
pleura. • ObI iteration of liver and cardiac dullness.
• Scar or irregular emphysema: Scarring and
Q: How to treat emphysema?
damage affecting the lung parenchyma without
involving acinus structure. A: As follows:
1. Smoking must be stopped.
Q: What are the causes of emphysema? 2. For breathlessness:
A: As follows:
• Inhaled salbutamol (200 mg 4-6 hourly) or
• Smoking. ipratropium (40 mg 4 hourly) or tiotropiurn
• Cold, dust (centrilobular]. (18 mg daily) or oxitropium (200 mg BD).
• (XI-antitrypsin deficiency. • If no response, inhaled corticosteroid
• Macleod syndrome (unilateral emphysema). beclomethasone (400 pgm BD).
• In severe case, oral prednisolone 30 mg for
Q: How smoking causes emphysema?
2 weeks, followed by maintenance dose.
A: Certain mechanisms are responsible:
3. Antibiotic, if secondary infection.
• Prolonged smoking causes inflammation In
4. In chronic cough: Mucolytic therapy (acetyl-
airways, release of oxidants and proteinase from
cysteine 200 mg 8 hourly orally for 8 weeks).
inflammatory cells, which are responsible for
5. Domiciliary 02 may be given.
irreparable damage to supporting connective
6. Vaccination, such as annual influenza vaccine,
tissue of alveolar septa.
five yearly pneumococcal vaccine and Haemo-
• There is increased proteinase synthesis and philus influenzae vaccine may be given.
inactivation of antiproteinase due to enzymes 7. Other treatment:
responsible for inactivating antiproteinase.
• (Xl-antitrypsin in case of deficiency.
• Imbalance between proteinase and
• Lung transplantation in young patient.
antiproteinase.
• Surgical intervention (if large bullae).
Q: Why lip pursing is present in emphysema? • Weight reduct jon, if obese.
A: By this in expiration through partly closed lips, • Exercise.
there is increased end-expiratory pressure that keeps Q: What is bullae? What are the causes? What is the
airway open, helping to minimize air trapping. treatment?
Q: What are the complications of emphysema? A: [t is the thin-walled air space produced by rupture
A: As follows: of alveolar walls. Bullae may be single or multiple,
large or small, and usually associated with emphy-
• Pulmonary hypertension.
sema. It is usually situated subpleurally. Rupture
• Cor pulmonale.
• Respiratory failure type 1. may lead to pneumothorax. Large bullae may com-
• Bullae, which mayrupturecausingpneumothorax. press the lung tissue and impair lung function. The
causes of bullae are:
• Secondary infection.
• Polycythaemia. • Emphysema.
• Congenital (rare).
Q: What are the signs of emphysema?
A: As follows: Treatment:
• The patient is dyspnoeic with lip pursing. • Small bullae: No treatment, treatment of primary
• Barrel-shaped chest. cause.
• Suprasternal and supraclavicular excavation • Large bullae with impaired lung function require
during inspiration. surgical ablation of bullae.
3 • RESPIRATORY SYSTEM __

• It is found in emphysema, commonly panacinar,


of age 50-75 years.
• Usually there is no cor pulmonale.
• Exertional dyspnoea is the main feature, and
cough less common.
• Arterial P02 and PC02 are relatively normal.
Blue bloater:
• The patient is cyanosed (blue) and oedernatous
(bloater). But not dyspnoeic (or mild
dyspnoea).
• It is found in chronic bronchitis, age 40-45 years.
Oedema is due to cor pulmonale.
Bullae • Cough with sputum is the main feature, dyspnoea
is less common.
Q: What is pink puffer and blue bloater? • Pulmonary hypertension, right ventricular
A: As follows: hypertrophy, cor pulmonale and secondary
Pink puffer: polycythaernia may develop (patient may appear
• The patient is not cyanosed (pink), but dyspnoeic plethoric).
with lip pursing (puffer). No oedema. • There is marked arterial hypoxaemia and
• Usually lean and thin. hypercapnia (low P02 and increased PCOJ

Brief Discussion About Lung Function Test (Spirometry)

Spirometry is a method of assessing the lung function. • FEF25-75 (forced expiratory flow in 25-75
FEYl and FYC are measured. The technique involves percentile): It is the graphical measurement of
a maximum inspiration followed by a forced expira- average expiratory flow in between 25% and 75%
tion (as long as possible) into the spirometer. FEV! of the expiration during FVC manoeuvre. This
is expressed as a percentage of FVC. It is done to diag- measurement denotes airflow condition in smaller
nose and differentiate obstructive airway disorders airways of <2 mm of diameter, which are devoid of
(e.g. COPD and asthma) from restrictive diseases cartilages. It is important in smokers (with COPD
(e.g. ILD). It can also be used to determine the severity and emphysema) and in children who cannot
of asthma and COPD. produce satisfactory fEYl.
By spirometry, five important measures can be PEFR: Measurement of PEFRon a regular basis at home
detected: with a portable peak flow meter is useful for patient
• FEV\ (forced expiratory volume in first second): over 5 years of age with moderate-to-severe persistent
The volume of air after forced expiration in the first asthma. The patient is asked to take a full inspiration
second, after full inspiration. as far as possible, and then to blowout forcefully into
• FVC (forced vital capacity): The total volume of air the peak flow meter. PEFR is best used to monitor the
expired forcibly in one breath after full inspiration. progress of the disease and its treatment.
• FEVJFVC: The ratio of FEY,:FVC is expressed as Regular measurement of PEFR on waking, in after-
percentage. noon and before bed demonstrates the wide diurnal
• PEF (peak expiratory flow): It is the highest flow variations in airflow limitations that characterize bron-
one can achieve during forceful expiration. It is chial asthma. Daily calculation of diurnal variability of
used as a short-term monitoring tool at a doctor's PEF provides a reasonable index of asthma stability and
chamber, or bedside and emergency room' during severity. Diurnal variability in peak flow is expressed by
exacerbation. Long-term monitoring of asthma the following formula:
can be done by seeing diurnal variability of PEF at
D·iurna I vana. bili
patient's home by maintaining peak flow chart. This (Highest PEF - Lowest PEF) x 100
1 ity = ~~--------~--
is essential for constructing self-management plan. HighestPEF
_ SHORT CASES IN CLINICAL MEDICINE

It should be noted that PEF physiologically falls at late


Other Tests
night or early morning. But this fall is normally <20% of
personal best result. Fall of PEF > 20% in early morning
1. Reversibility test: Bronchodilator reversibility test
is known as 'morning dipping of PEF'. It is characteris-
can be used to differentiate between asthma and
tic of uncontrolled asthma.
COPD. After bronchodilatation, both >12% and
Spirometry in obstructive airway disease shows that: >200 mL increase in FEV over prebronchodi-
• FEV, is reduced «80% of predicted value). lator level indicates "
positive reversibility test,
• FVC is reduced. suggesting diagnosis of bronchial asthma. Nega-
• FEY,:FYC ratio is reduced «75%). tive result indicates CO PO (or severe persistent
asthma).
Spirometry in restrictive airway disease shows that:
2. Bronchoprovocation test: In fall of FEV" > 20%
• FEV, is reduced «80% of predicted value, but in after inhalation of methacholine or hypertonic
proportion to FVC).
saline is used for diagnosis of hyperresponsive-
• FVC is reduced «80% of predicted value).
ness of airways in susceptible patients with normal
• FEV:FVC ratio is normal (>75%). spirometry. Susceptible patients are those with:
(i) cough-variant asthma, (ii) mild intermittent
Q: What are the restrictive airway disease and obstruc-
asthma, and (iii) chronic bronchitis with hyperre-
tive airway disease?
sponsive airways.
A: As follows:
3. Exercise challenge test: In fall of FEV" or PEFR
• Restrictive: ILDs, ankylosing spondylitis and > 15% from baseline value after vigorous exercise
kyphoscoliosis. (i.e. running or climbing stairs for 6 min) indicates
• Obstructive: Emphysema, chronic bronchitis and 'exercise-induced asthma'. Fall starts at 5-10 min
bronchial asthma. after stoppage of exercise and peak at 20-30 min
Differences between restrictive and obstructive air- and then resolves automatically. It can be reversed
way disease quickly using bronchodilator inhaler.

Points Restrictive Obstructive


FEV, and rvc Proportionately FEV1is markedly Bedside Assessment of Lung Function
reduced reduced and FVC
also reduced
• If your diagnosis is CO PO, forced expiratory
FEV,:FVe Normal Reduced time (FET) should be seen by asking the patient
RV Reduced or normal Increased to exhale forcefully through open mouth after
TLe Reduced Increased full inspiration, while you listen by placing your
RV:TLe Normal or slightly Markedly stethoscope over the trachea. Normally, it is <6 s. If
increased increased >6 s. then it indicates airway obstruction.
• Peak flow meter (as described above).
N.B. Both TLCO and transfer coefficient of carbon • Flow volume curve is an alternative to spirometry.
monoxide (Keo) are reduced in restrictive airway Flow volume curve may be measured using a
disease and obstructive airway disease. portable electronic device.

Mass Lesion in Lung (Bronchial Carcinoma)

Mass lesion in lung may be isolated bronchial carci-


Presentation of a Case
noma without definitive signs. There may be collapse
or pleural effusion, or SVC obstruction and radiation (Supposing right-sided lesion)
mark in chest.
On inspection:
The patient is middle-aged or elderly, emaciated or • Restricted movement of upper part of chest.
cachexic with generalized dubbing and nicotine stain • There is radiation mark on the chest (if any).
in fingers.
3 • RESPIRATORY SYSTEM __

Q: What will you see in the eyes of the patient?


On palpation:
A: I will look for partial ptosis, miosis and enoph-
• Trachea: Central and apex beat in normal area.
thalmos, which indicates Horner syndrome. It is
• Vocal fremitus: Reduced or absent (rib tenderness
may be present). commonly found in Pancoast tumour. Fundoscopy
may show papilloedema, if secondary metastasis in
On percussion: the brain.
• Dullness in right upper part of chest (mention
up to which space). Q: What are the differential diagnoses?
A: As follows:
On auscultation:
• Breath sound is reduced or absent. • Pleural effusion.
• Vocal resonance is reduced or absent. • Pulmonary tuberculosis.
• Other mass lesions (neurofibroma, dermoid cyst,
hydatid cyst, etc.).
My diagnosis is bronchial carcinoma.
Q: Why not this is pleura] effusion?
Q: What relevants do you like to see?
A: In pleural effusion, trachea and apex beat are shifted
A: As follows:
to the opposite side.
• Hand: Clubbing with nicotine stain, hypertrophic
Q: Why not this is collapse?
osteoarthropathy, wasting of small muscles of
hand (due to involvement of lower trunk of A: In collapse, trachea and apex beat are shifted to the
brachial plexus). same side.
• Supraclavicular, axillary lymph nodes Q: Why not consolidation?
(metastasis) . A: In consolidation, there is bronchial breath sound
• Eye: Horner syndrome (in Pancoast tumour). and increased vocal resonance.
• Evidence of SVC obstruction.
Q: What investigations do you suggest in bronchial
carcinoma?
A: As follows:
1. X-ray chest PA view (homogeneous irregular
opacity with sun-ray appearance may be seen.
There may be collapse, pleural effusion, hilar
lymphadenopathy, widening of mediastinum,
raised hernidiaphragm. and rib erosion or
destruction) .
2. cr scan of chest (MRI is not helpful for primary
lesion).
Clubbing 3. Sputum for malignant cells (exfoliative
cytology).
4. Cl-guided PNAC.
5. FNAC (or biopsy) of lymph nodes (if present).
6. Fibre optic bronchoscopy and biopsy (or bron-
chial washing and brushing).
7. PET-CT scan is the investigation of choice
(highly sensitive and specific for mediastinal
staging).
8. To see evidence of metastasis: USC of whole
abdomen, X-ray of skull, isotope bone scan,
etc. Sometimes, CT scan of chest and abdomen,
even MRl may be needed.
9. Others:
• Complete blood count, ESR.
X-ray showing hypertrophic osteoarthropathy • If pleural effusion, then fluid cytology.
at the lower end of ulna Pleura] biopsy may also be done.
_ SHORT CASES IN CLiNKAL MEDICINE

• Liver function test, renal function test (before • If carina is wide and there is loss of sharp angle
chemotherapy, if needed). of carina, it indicates presence of enlarged
• Pulmonary function test, specially FEY) mediastinal lymph nodes (may be malignant or
[diffusing capacity of the lung for carbon reactive). Biopsy can be taken by passing a needle
monoxide (DLeO) below 60% predicted is through bronchial wall.
associated with a mortality rate of 25% due • Vocal cord paralysis on the left indicates left
to pulmonary complications]. recurrent laryngeal nerve palsy, and indicates an
inoperable case.
Q: What are the types of bronchial carcinoma?
A: There are two types:
• Non-small-cell carcinoma in 80%.
• Small-cell carcinoma in 20%.
Q: What are the histological types of bronchial carci-
noma?
A: There are four types:
• Squamous-cell carcinoma in 35%.
• Small-cell carcinoma in 20%.
• Adenocarcinoma in 30%.
• Large-cell carcinoma in 15%.

Features of adenocarcinoma:
Right-sided bronchial carcinoma with phrenic nerve palsy • 10% of bronchial carcinoma.
• Arises from mucous cells in the bronchial
epithelium.
• Occurs in the periphery of lung.
• Usually develops in or around old scar.
• More in elderly.
• More in female.
• More in nonsmokers.
• More in far East.
• More in asbestosis.
• Invasion to pleura and mediastinal lymph nodes
is common.
• Metastasis to brain, bone and adrenal gland is
common.

CT scan showing right-sided bronchial carcinoma


Features of small-cell carcinoma:
• 20-30% of bronchial carcinoma.
Q: If sputum shows malignant cells, would you do • Arises from endocrine cells called Ku1chitsky
bronchoscopy and biopsy? cells. These cells are members of amine
A: Yes, to see histological type. This is helpful for precursor uptake decarboxylase (APUD)
therapy and prognosis. If squamous-cell carci- system, which secretes many polypeptide
noma, radiotherapy is the treatment. If small-cell hormones. Some of these hormones act in an
carcinoma, chemotherapy is necessary. autocrine fashion; they feedback on the cells
and cause cell growth.
Q: Why bronchoscopy should be done? • Highly malignant, rapidly growing, early
A: Bronchoscopy should be done: metastasis and inoperable at presentation.
• To see the mass and to take biopsy for tissue • Usually responds to chemotherapy and
diagnosis. This will guide further management. radiotherapy, less likely to be cured by surgery.
• Tfthe carcinoma involves first 2 em of either main • Overall prognosis is poor. Survival is short,
bronchus, it indicates that the tumour is inoperable. 3 months to 1. year.
3 • RESPIRATORY SYSTEM __

Features of bronchoalveolar cell carcinoma: involvement oflower trunk of brachial plexus


• Found in 1-2% of bronchial carcinoma. called Pancoasts syndrome.
• Occurs as peripheral solitary nodule or diffuse 2. Due to local spread in mediastinum:
nodular lesion of multicentric origin. • Hoarseness of voice and a bovine cough (due
• It may be associated with profuse mucoid sputum to involvement of left recurrent laryngeal
expectoration. nerve by left hilar lesion).
• Dysphagia (oesophageal obstruction, also
cardiac tamponade).
• SVC obstruction (with right-sided mass or
mediastinal lymphadenopathy ).
• If pericardium is invaded, there may be
pericardia] effusion or arrhythmia.
• Stridor, when lower trachea, carina and main
bronchi are narrowed by primary tumour or
compression by subcarinal and para tracheal
lymph nodes.
3. Distant metastasis (in liver, brain, bone,
adrenal, contralateral lung and lymph nodes.
Metastatic features usually common in adeno-
carcinoma) .
4. Nonmetastatic-extrapulmonary manifestations
Bronchial carcinoma (emaciated with prominent vein) (see below).
5. General features of malignancy: Anorexia,
Q: What are the presentations of bronchial carcinoma? weight loss, malaise, fatigue.
A: Usually in elderly patients with history of smoking: Nonmetastatic extra-pulmonary manifestations
1. Due to lung lesion: (paraneoplastic syndrome): Occur in 15-20% cases
• Cough: Dry or sputum production. Changing of bronchial carcinoma due to secretory products by
pattern of regular cough in a smoker is highly the tumour. These may precede, coincide or follow
suspicious. Bovine cough due to left recurrent after the cancer. Treatment of carcinoma improves
laryngeal nerve palsy. the features.
• Haernoptysis=-ccmmonly in central lesion.
1. Endocrine (10%, usually in small-cell carcinoma):
Sometimes, involvement of large blood
vessels causes massive haemoptysis. • SIADH (syndrome of inappropriate ADH,
usually in small-cell carcinoma).
• Breathlessness (due to bronchial obstruction,
• ACIH secretion causing Cushing syndrome
collapse, or massive pleural effusion or
(usually in small-cell carcinoma).
compression of phrenic nerve causing
paralysis of the diaphragm). • Carcinoid syndrome (usually in small-cell
carcinoma).
• Chest pain (due to involvement of pleura, rib
• Hypercalcaemia due to release of
or chest wall, intercostal nerve and brachial
parathorrnone-like substance (usually in
plexus).
squamous-cell carcinoma).
• Recurrent pneumonia at the same site or
• Gynaecomastia due to excess oestrogen
pneumonia with slow resolution is highly
(in large cell).
suspicious.
• Rarely, hypoglycaernia and thyrotoxicosis.
• Apical tumour (called Pancoast tumour)
involves cervical sympathetic chain. at or 2. Neurological (in any type):
above the stellate ganglia, causing Homer • Peripheral neuropathy (usuallysensori motor).
syndrome (characterized by ipsilateral • Cerebellar degeneration.
partial ptosis, enophthalmos, miosis and • Cortical degeneration (dementia).
anhydrosis of the face). Also, there is pain in • Myelopathy: Motor neuron disease-like feature.
shoulder and inner aspect of arm due to the • Retinal blindness (small-cell carcinoma).
_ SHORT CASES IN CLINICAL MEDICINE

3. Musculoskeletal: • Cigarette smoking is the major risk factor. Even


• Polymyositis or dermatomyositis (in all types). passive smoking causes 1.5 times increase in the
• Myasthenic myopathic syndrome (Eaton- risk of bronchial carcinoma.
Lambert syndrome). • Other factors are exposure to asbestos, silica,
• Clubbing and hypertrophic osteoarthropathy beryllium, cadmium, chromium, arsenic, iron
(non-small-cell type). oxide, radon, radiation, petroleum products and
4. Haematological (in all types): oils, coal tar, products of coal combustion.
• Migrating thrombophlebitis. • Adenocarcinoma may develop in nonsmokers
• Disseminated intravascular coagulation (DIC). and in old scar.
• Thrombotic thrombocytopaenic purpura. Q: How to treat bronchial carcinoma?
• Normocytic normochromic anaemia and A: As follows:
occasionally haemolytic.
1. Non-small-cell carcinoma:
• Eosinophilia. • Surgery should be done, if the tumour is
5. Heart (in adenocarcinoma): localized to lobe or segment. (It is curative if
• Marantic endocarditis (nonbacterial,
the stage is TlNOMO.).
thrombotic or verrucous endocarditis).
• If surgery is not possible, radiotherapy or
6. Skin (in all types):
chemotherapy or combined therapy should
• Acanthosis nigricans. be given.
• Dermatomyositis. • In squamous-cell type, radiotherapy is advised
• Herpes zoster. (it is especially indicated in SVC obstruction,
7. Renal:
repeated haemoptysis and chest pain caused
• Nephrotic syndrome due to membranous by chest wall invasion or skeletal metastasis).
glomerulonephritis (rare).
• Chemotherapy is less helpful in non-small-cell
B. Metabolic (universal at some stage):
type.
• Loss of weight. 2. Small-cell carcinoma:
• Lassitude. • Even small, metastasis occurs early. Surgery is
• Anorexia. less helpful. Chemotherapy is usually given.
Q: Why chest pain in bronchial carcinoma? Radiotherapy may be added [continuous
A: Due to metastasis in the rib or chest wall invasion. hyperfractionated accelerated radiotherapy
(CHART) in which total dose is given in
Q: What is Pancoast tumour and Pancoast syndrome?
smaller; but more frequent fractions offer the
A: As follows:
best survival].
• Pancoast tumour (superior sulcus tumour): It is • Usual chemotherapy: Intravenous CDV (cyclo-
the tumour that arises from apex of the lung. It may
phosphamide, doxorubicin and vincristine) or
involve the cervical sympathetic chain (at or above CE (cisplatin plus etoposide). Chemotherapy is
the stellate ganglion) causing Horner syndrome.
given every3 weeks for 3-6 cycles.
• Pancoast syndrome: This is characterized by 3. Other treatments: These are usually palliative.
pain in the shoulder or inner aspect of arm
• Laser therapy with fibreoptic bronchoscopy.
along the ulnar nerve distribution due to the
• Endobronchial therapy: Tracheobronch-
involvement of lower part of brachial plexus
ial stent, cryotherapy, laser, brachytherapy
(CB, Tl,T2). There maybewastingofsmallmusdes
(a radioactive source is placed closed to the
of hand due to CB and T'l nerve involvement.
tumour),
Local invasion by the tumour may cause pain
• Radiofrequency thermal ablation (RFT).
and tenderness of the first and second ribs with
• Pleural drainage or pleurodesis (in pleural
evidence of rib destruction radiologically. Horner
effusion).
syndrome may occur due to involvement of the
• Drug: Steroid to improve appetite, morphine
sympathetic pathway as it passes through T1 root. or diamorphine for pain (along with laxatives,
In Pancoast syndrome, treatment is by combined
if constipated). Oral candidiasis should be
surgery and radiotherapy.
treated.
Q: What are the causes or risk factors of bronchial • Short courses of palliative radiotherapy
carcinoma? are helpful for bone pain, severe cough or
A: As follows: haernoptysis.
3 • RESPIRATORY SYSTEM .._

RFf is helpful in some mal ignancy such as bronchial Q: What is the role of surgery in lung carcinoma?
carcinoma and hepatocellular carcinoma. It is done A: In non-small-cell carcinoma without metastasis,
by placing special needle into the tumour under surgery is the treatment of choice. But it has limited
guidance of cr scan. Electric current from radiof- role in small-cell carcinoma, as >90% patients have
requency current generator is passed through the metastasis at the time of presentation.
needle, which results in generation of heat causing
Q: What are the contra indications of surgery?
destruction of tumour. RFT is helpful, if the tumour
A: As follows:
size is <4 cm. Also, sometimes used in large tumour.
• Distant metastasis (Ml).
Complications of RFf: • Invasion ofcentral mediastinal structures induding
• Bleeding. heart, great vessels, trachea and oesophagus (T4).
• Pneumothorax. • Malignant pleural effusion (T4).
• Skin burn. • Contralateral mediastinal lymph node
• Secondary infection and lung abscess. involvement (N3).
• FEV) < 0.8 L.
TNM classification of lung cancer (it is for • Poor general condition, severe or unstable
non-small-cell carcinoma)- cardiac, or other medical problem.

• T: Extent of primary tumour. ~.B. Remember the following points:


• N: Involvement of lymph node. • In a fit individual, surgery is not absolutely
• M: Presence of distant metastasis. contraindicated in case of direct extension
Stages for T: of tumour into the chest wall, diaphragm,
mediastinal pleura or pericardium, or to
• Tx: Positive cytology only.
within 2 em of the main carina.
• Tl: <3 cm in diameter.
• T2: >3 em in diameter or extends to hilar region • Surgery is rarely appropriate in patients
or invades visceral pleura or partial atelectasis or over 65 years as the operative mortality rate
extends into main bronchus, but remains 2 cm or exceeds the 5-year survival rate.
more distal to carina. Q: What are the indications of radiotherapy?
• T3: Involvement of chest wan, diaphragm, A: As follows:
pericardium, mediastinum, pleura, total 1. As the main treatment in:
atelectasis, main bronchus <2 cm distal to carina. • Localized tumour where surgery is not possible.
• T4: Involvement of heart, great vessels, trachea, • Along with chemotherapy in small-cell
oesophagus, malignant effusion, vertebral body, carcinoma.
carina. Separate tumour nodules. • Before and after surgery in selected patients.
2. As a palliative therapy in:
Stages forN
• Pain (local or metastasis).
• NO: No nodal involvement. • SVC obstruction .
• Nl: Peribronchial, ipsilateral hilar or intra- • Pancoast tumour.
pulmonary lymph node involvement. • Obstruction of trachea and main bronchi.
• N2: Ipsilateral, mediastinal or subcarinal. • Recurrent haemoptysis.
• N3: Contralateral mediastinal, scalene or supra-
davicular. Q: What is the role of chemotherapy?
A: Small-cell carcinoma is usually treated with chem-
Stages forM
• MO: No distant metastasis.
• Ml: Distant metastasis.
_j otherapy.

boplatin.
Drugs used are cisplatin.
cyclophosphamide, vinblastine,
etoposide,
vindesine,
Newer drugs are docetaxel. paclitaxel,
irinotecan, vinorelbine, gemcitabine.
car-

Q: What is the Tole of staging? Q: What is the prognosis?


A: Staging is done to see the possibility of surgical A: If it is localized and surgical resection is possible,
resection of the carcinoma. TNM staging is done for the prognosis is good. Otherwise:
non-small-cell cancer only. In early stage, surgery • Non-small-cell carcinoma: 50% 2-year survival
may be curative. In advanced stages, surgery may without spread, 10%with spread.
not be possible. Small-cell carcinoma is treated • Small-cell carcinoma: Median survival is
according to whether it is limited or extensive. 3 months if untreated, 1-1V2 year with treatment.
ABDOMEN
"The art of medicine consists of amusing the patient while nature cures the disease"
-Voltaire

Introduction

Usual instructions by the examiner are: • Suppose the patient has splenomegaly or hepato-
megaly or hepatosplenomegaly. Examiner may ask,
• Examine the abdomen or examine the abdomen 'What are the causes? What investigations do you
and see the relevants, suggest? r
• Palpate the abdomen. What are your findings? • Suppose the patient has a mass in epigastrium or
• Look at the abdomen (in a case of distended iliac fossa. Examiner usually asks, 'What do you
abdomen, localized or generalized). What do you think are the causes? What else do you want to
think are the causes in this case? see? What investigations do you suggest?'
Once asked to examine the abdomen, very likely • If the abdomen is distended, examiner may ask,
findings are: 'What are the causes of abdominal distension?
Do you think the patient has ascites? How to
• Splenomegaly.
confirm ascites?'
• Hepatomegaly.
• There may be engorged veins. Examiner asks, 'What
• Hepatosplenomegaly.
are the causes of engorged vein? What else do you
• Abdominal mass (in epigastrium, right or left iliac
like to see?' (to see the flow of blood).
fossa, flanks, central or lower abdomen, renal
• If multiple striae are present, examiner may ask
mass bilateral or unilateral, polycystic kidney and
What are the likely causes?'
transplanted kidney).
• Ascites or ascites with splenomegaly or Sometimes the examiner may ask you to examine a
hepatomegaly. normal abdomen to see whether you can examine
• Abdominal aneurysm. properly. Be careful; you must not describe findings that
• Normal abdomen. are not present. Sometimes a finding may be missed
In majority of the cases, there will be splenomegaly, (e.g. small spleen or liver or mass). Tnsuch a case, a good
hepatomegaly, hepatosplenomegaly and, sometimes, examiner may teJl or give you a chance, 'see again'. If
abdominal mass or ascites. Keeping these in mind, exam- this happens, probably you have missed a finding like a
ine carefully and present the case systematically. At the just-palpable spleen, liver, mass, etc. In such situations,
same time, you must be ready to answer the relevant definitely you have missed some finding and should
questions, e.g.: examine more carefully.

Examination Routine (Abdomen)


Proceed as follows: abdomen exposed from inframammary region to
• r ntroduce yourself ask for permission, 'May 1exam- just above the pubic symphysis (do not expose the
ine your abdomen please?' genitalia to embarrass the patient).
• Remain on the right side; ensure that the patient • Quickly look from head to foot. May be you find
is lying flat in a supine position (remove any some clue for diagnosis. For example, chronic
extra pillow with permission), hands by the side, liver disease (CLD), chronic renal failure (CRF)
4. ABDOMEN __

or other renal diseases [nephrotic syndrome and Palpate the abdomen in the following way:
acute glomerulonephritis (AGN)J. In hereditary 1. First, perform superficial palpation, feel for rigid-
haemolytic anaemia, frontal and parietal bossing, ity or any mass. Hard periumbilical lymph node
mongoloid facies and so on. [called Sister Mary Joseph nodule is highly sug-
gestive of metastasis from pelvic or gastrointestinal
Inspection:
tract (CIT) primary tumour].
• Shape of abdomen may be normal, distended or 2. Tenderness (rebound .tenderness may be seen,
shrunken (scaphoid). If distended, generalized press the abdomen slowly and then release sud-
or localized (epigastrium, right or left hypochon- denly; presence of pain indicates peritonitis).
drium, iliac fossa or central pall). 3. Liver: Start from right iliac fossa, ask the patient,
• For movement of abdomen ask the patient to take 'turn your face to left side, keep your mouth open,
deep breath in and out, inspect from either leg- or take deep breath in and out'. Press and proceed
head-end to see whether the movement is equal at during inspiration and look at the patient's face.
all sides. If liver is palpable, see the following points:
• Visible peristalsis. • Measure in centimetre (with tape, measure
• Visible pulsation (in epigastrium). from costal margin in right midclavicular line,
• Umbilicus (inverted or everted). and not finger measurement). If left lobe is
• For visible veins, mention the location ... (central enlarged, measure from xiphoid process.
part, flank, below or above umbilicus, around • Margin (round or smooth or sharp).
umbilicus). If present, see the flow. • Surface (smooth or irregular).
• If striae, mention the location (colour, size, vertical • Tenderness.
or horizontal). • Consistency (soft or firm or hard).
• Any scar mark (due to surgery or trauma), fistula • Upper border of liver dullness (using heavy
(Crohn disease) or. stoma (colostomy, ileostomy percussion, but percussion of lower border
and ileal conduit). using light percussion).
• Pigmentation (linea nigra from below umbilicus, • Auscultation over liver for bruit or rub (often
erythema ab igne}. forgotten) .
• Swelling or mass (mention the location ... ; also N.B. Pulsatile liver may be present. Tnsuch a case,
mention whether it is intra-abdominal or extra-
put your left hand on the back, right hand
abdominal). over the liver, press gently and see movement
• Campbell de Morgan spot may be small, red, of right hand.
nodular lesion (in middle age Or elderly).
• Finally, look at the groin to see hernia (ask the 4. Spleen: Keepyour left hand in lowermost part of left
patient to cough), pubic hair and genitalia (with side of chest posterolaterally with slight pressure.
permission of the patient). Starting from the right iliac fossa, ask the patient,
'turn your face to left side, keep your mouth open,
Palpation: take deep breath in and out'. Ifit is notfelt, tum the
patient halfway towards right and palpate again.
• Enquire any pain in the abdomen (examine this
part in the end) and tell the patient, 'Please tell me, Once spleen is palpable, see the following points:
if I hurt you.' • Measure in centimetre along its long axis from
• Ensure that your hand is warm; put the palm costal margin in anterior axillary line towards
gently rather than tip of fingers, keeping the hand right iliac fossa (OI from costal margin in left
flat on abdominal wall with a gentle flexion of midclavicular line).
metacarpophalangeal (MCP) joints. • Feel splenic notch.
• Better if you are in horizontal position by kneeling • See, get above the swelling of spleen (insinuate
at the patients side, wrist and forearm in same right index finger between spleen and left costal
horizontal plane. margin).
• Percuss over spleen and continue up to left
• During palpation, look at face to see whether
lower part of chest (to see the continuation of
patient winces with pain.
splenic dullness).
N.B. If ascites, do not forget to palpate by dipping 5. Palpate both right and left kidneys (by ballottement).
technique. 6. Gall bladder.
__ SHORT CASES IN CLINICAL MEDICINE

7. Any mass (see below). • If the patient has splenomegaly or hepatosple-


8. Feel. for para-aortic lymph nodes (by tip of fingers). nomegaly, which may be due to lymphoma,
9. Palpate hernial orifice (ask the patient to cough, leukaemia or CLO, your answer will be, 'I want to
see and palpate). see lymph nodes in other parts of the body, bony
10. Palpate testis (with permission of patient). tenderness, anaemia or stigmata of CLD'.
11. Finally, PR examination (if the examiner asks). • In young age, if the patient has splenomegaly
5 Never done in the examination. or hepatosplenomegaly, which may be due to
Eo hereditary haemolytic anaemia, your answer will
v
be, 'I want to see anaemia, jaundice, frontal and
~ parietal bossing in head, prominent malar bones
or mongoloid facies, also family history'. If chronic
liver disease (CLD) is suspected, examiner may ask,
'What specific signs do you like to see in a child or
young age?' Your answer is 'KF ring in eye (Wilson
disease) and emphysema in chest (<x',-antitrypsin
deficiency), .
• If mass in epigastrium, which may be due to carci-
noma of stomach, your answer will be, 'I want to
Sister Mary Joseph nodule
see the left supraclavicular lymph node' (Virchow
gland).
Percussion:
• If you find a renal mass and the patient looks
• Usually a light percussion is done. Note normal anaemic, look for arteriovenous (AV) fistula in the
tympanic sound. wrist or below the clavicle (haemodialysis); also
• See area ofliver dullness and splenic dullness. look for blood pressure (BP), puffy face and oedema.
• If there is suspicion of ascites, see shifting dullness.
• Fluid thrill (if tense ascites). If you get any mass, see the following points:
First of all, see whether it is intra-abdominal or extra-
Auscultation:
abdominal. Ask the patient LO raise his head and press
• Auscultate; normal bowel sound. over the forehead. Now, see and palpate the mass
• Over liver (hepatic bruit or rub). again. If it disappears or is less prominent, it is intra-
• Renal bruit (2 em above the umbilicus and lateral abdominal. If it is more prominent, then it is likely to
to midline). be extra-abdominal. Next, see the following points:
• Bruit of aortic aneurysm (if any).
• Splenic rub (rarely found). • Site.
• Venous hum (heard between xiphisternum and • Size.
umbilicus). It is a continuous, low-pitch soft mur- • Shape (round, regular or irregular).
mur. Presence of venous hum indicates portal • Surface (smooth or irregular).
hypertension (rare finding). It is due to large volume • Consistency (soft or firm or hard).
of blood flowing in umbilical or paraumbilical vei n • Tenderness.
in falciparum ligament due to portosystemic shunt. • Mobility (mobile or fixed).

N.B. When venous hum is present with prominent


N. B. A fixed mass that does not move wi th respira-
veins in anterior abdominal wall, it is called
tion is highly suggestive of a retroperitoneal
Cruveilhier-Baumgarten syndrome.
(such as retroperitoneal sarcoma) or colonic
After finishing, ask permission, 'May 1 see the rele- mass.
vants please?" or the examiner may ask, "What relevants
do you like to see?' Remember, examiner may give clue during instruc-
Relevant findings to be seen according to 'your find- tion and will expect specific answers. For example:
ings in abdomen and suspicion of causes:
• This patient is referred to you from cardiology
• If the patient has ascites and splenomegaly that unit. Palpate the abdomen. (In such a case, there
may be due to CLO, your answer will be, '1 want to may be splenomegaly, which is due to infective
see the stigmata ofCLO' (see page 202). endocarditis. Or enlarged and tender liver, which
4 • ABDOMEN __

may be due to congestive cardiac failure (CCF) or • This young lady has thrornbocytopaenia. Now
chronic constrictive pericarditis. Or ascites due to palpate the abdomen [splenomegaly is suggestive
chronic constrictive pericarditis, even CCF). of systemic lupus erythematosus (SLE)I.
• Look at the patient and now palpate the abdomen. • This patient has extreme itching. Examine the
The patient may have plethoric face. Splenomegaly abdomen (splenomegaly is due to polycythaemia
indicates polycythaernia rubra vera or bilateral rubra vera).
renal mass due to polycystic kidney disease. • This lady of 21 years is referred to from neurology 6:
0-
• Look at the patient (may have rheumatoid hand) having CVD and right-sided hemiplegia. Examine o
and palpate the abdomen (splenomegaly indicates the abdomen (splenomegaly is suggestive of SLE S
g
Felty syndrome). with antiphospholipid syndrome).

Some Important Findings on Inspection

If there are obvious findings on inspection examiner 4. Hypogastric swelling (below umbilicus):
may ask 'What do you see? " What is this? ' and What • Bladder [urinary retention).
are the causes?' Examples: • In female (ovarian cyst and pregnancy).
Distended abdomen: The causes of generalized dis- • Intra-abdominal malignancy.
tension (five Fs): • Lymphoma.
• Tabes mesentericus or mesenteric cyst.
• Fat (obesity).
• Hydatid cyst.
• Fluid (ascites).
• Foetus (in female of child-bearing age).
• Flatus (gastrointestinal).
• Faeces (constipation).
(Others: Intra-abdominal growth, big ovarian cyst,
urinary retention).
Causes of localized distension:
1. Epigastrium:
• Left lobe of liver (hepatoma, secondaries, liver
abscess and hydatid cyst).
• Pancreas (pancreatic pseudocyst and carcinoma
of head of pancreas).
• Stomach (carcinoma of stomach and
lymphoma).
• Others include epigastric hernia, aneurysm of
aorta (pulsating mass) and growth in transverse
colon.
2. Right hypochondrium: Upper abdominal swelling
• Hepatic mass (hepatoma, secondaries, liver
abscess and hydatid cyst).
• Gall bladder (carcinoma, mucocoele, empyema
and carcinoma of head of pancreas).
• Mass in hepatic flexure or right side of transverse
colon (carcinoma colon).
3. Left hypochondrium:
• Splenomegaly (due to any cause).
• Renal mass and suprarenal mass.
• Mass in splenic flexure or left side of transverse
colon (carcinoma colon).
• Hydatid cyst.
• Carcinoma of pancreas (from tail). Lower abdominal distension (urinary retention)
-.a SHORT CASES IN CLINICAL MEDICINE

Causes of scaphoid abdomen (shrunken): Q: How 10 differentiate between SVC obstruction, lVC
• Starvation. obstruction and portal obstruction?
• Carcinoma (commonly, oesophagus and stomach). A: For differentiating features see the table given below.

SVC IVC Portal


obstruction obstruction obstruction
Veins Usually in Mostly in the In the upper and
the upper flanks and in the lower
abdomen and rarely in the abdomen, also
chest wall lower around the umbilicus
abdomen (caput medusae)
Flow Downwards Upwards Away from the
umbilicus (flow of
veins upwards above
Scaphoid abdomen the umbilicus and
downwards below it)
Causes of engorged veins:
1. Superior vena caval (SVC) obstruction (flow is N.B. There will be other signs ofSVC IVC or portal
downwards) . vein obstruction.
2. Inferior vena caval (rve) obstruction (flow is
upwards). Q: What are the signs of sve obstruction?
3. Portal obstruction (like normal flow) as follows: A: As follows:
• Veins above umbilicus (flow is upwards). • Face is plethoric or reddish, oedematous or puffy,
• Veins below umbilicus (flow is downwards). suffused.
4. In extreme cachexia, engorged veins are present • Eyes with conjunctival suffusion, chemosis and
due to loss of subcutaneous fat. periorbital oedema.
5. Normally present in lean and thin person. • Neck veins are engorged and nonpulsatile.

Q: What arc the signs of Ive obstruction {


A: As follows:
• Engorged vein in lower limbs.
• Lower limbs: Swollen and oedematous.
• Also buttock and groin: Swollen and oedematous.

N.B. In IVC obstruction, engorged veins are due to


dilated anastomotic channel between superficial
epigastric and circumflex iliac veins below and
lateral thoracic veins above, conveying blood
from long saphenous vein to axillary vein.

Q: What are the signs of portal obstruction?


A: As follows:
• Splenomegaly (the single most definitive sign).
Engorgifd veins • Ascites.
• Oesophageal varices by endoscopy and also
Q: If engorged vein, examiner may ask 'What else haemorrhoid by proctoscopy.
would you like to see and wl"\yr
Q: What is caput medusae?
A: I want to see the flow of blood to find out causes
A: Caput medusae refer to dilated veins radiating from
(see above).
the umbilicus, found in portal obstruction. It is
Q: What is the normal blood flow of veins? named after its resemblance to the Greek goddess
A: As follows: Medusae, whose head was adorned with snakes that
• Veins above umbilicus (flow is upwards). radiated from her head (Caput means head). It is a
• Veins below umbilicus (flow is downwards). rare finding.
4 • ABDOMEN __

Q: What are striae? What are the causes?


A: These are wrinkled, linear, white or pink-coloured
marks over the skin. Striae are due to stretching
of skin causing rupture of elastic fibres. Causes of
striae are:
• Striae gravidarum (white or pink, narrow lines, in
abdominal walls due to pregnancy, and usually
below the umbilicus).
• Obesity (whitish narrow lines, usually
longitudinal. < 2 mm).
• Cushing syndrome. (Wide lines, pink or purple Grey Turner sign
or red, mostly horizontal or oblique. Pink or red
colour is due to increased vascularity.) Q: What is Cullen sign?
• Ascites. A: It is the skin discolouration around the umbilicus,
usually a faint bluish hue. It is due to
haemoperitoneum.

Striae gravidarum Cullen sign

Causes of haemoperitoneum:
Causes of visible epigastric pulsation:
• Normally, in thin body build. • Rupture of spleen.
• Rupture of ectopic pregnancy.
• Right ventricular hypertrophy.
• Acute haemorrhagic pancreatitis.
• Aneurysm of the aorta.
• Any cause of bleeding (blood dyscrasia and excess
• Pulsatile liver.
aorta anticoagulant) .
• Mass overlying the abdominal
(e.g. carcinoma stomach). Multiple small, firm skin nodules in the ante-
rior abdominal wall may be due to:
Causes of eversion of umbilicus:
• Neurofibroma.
• Ascites (slit is transverse).
• Disseminated malignancy (secondaries).
• Ovarian cyst (slit is vertical).
• Sarcoidosis.
• Umbilical hernia.
• Lipoma.
Causes of multiple fistula in abdomen: A hard nodule around the umbilicus called Sister
• Crohn disease. Mary Joseph nodule indicates metastasis (from



Tuberculosis (TB).
Actinomycosis.
Trauma.
_j pelvic or CIT tumour).

Q: What is Riedel lobe?


A: It is a tongue-like projection from inferior surface of
Q: What is Crey Turner sign? right lobe of liver, felt in right hypochondrium. It is
A: It is the skin discolouration in the flanks of a normal finding, common in women, rarely may be
abdominal wall due to extraperitoneal bleeding very large up to the right iliac fossa. It may be confused
(haemoperitoneum due to any cause). with enlarged gall bladder or enlarged right kidney.
-.:J SHQRT CASES IN CLINICAL MEDICINE

- -

Hepatosplenomegaly
• Lymph nodes (in lymphoma, CLL and viral
Presentation of a Case :~--------, infection) .
• Signs of CLO (page 202).
• The abdomen may be (or is distended) general-
• Pigmentation (in kala-azar, also CLD and haerno-
ized or in right or left hypochondrium.
chromatosis) .
The liver is (mention your findings): • Bony tenderness (in leukaemia and lymphoma).
• Enlarged, ... cm from right costal margin in right • In young or child, anaemia, jaundice, frontal and
rnidclavicular line. parietal bossing, mongoloid facies (hereditary
• Margin is sharp or round. haemolytic anaemia).
• Surface is smooth, irregular and nodular (single Causes of CLD with hepatomegaly
or multiple). • Haemochromatosis.
• Consistency is firm (or soft or hard). • Primary biliary cirrhosis.
• Tender (or nontender). • Alcoholic liver disease.
• Upper border of the liver dullness is in right ... Q: Mention one simple investigation that may help to
intercostal space, in right midclavicular line.
diagnose or exclude some diseases.
• There is no (or yes) hepatic bruit (or rub).
A: Full blood count (FBC) with peripheral blood film
The spleen is (mention your findings): (PBF) examination. By this, I can diagnose:
• Enlarged; cm from left costal margin, in left • Malaria.
anterior axillary line (towards right iliac fossa). • Kala-azar [FBC shows leucopaenia, lymphocy-
• Soft or hard or firm in consistency. tosis and monocytosis. Repeated blood count
shows progressive leucopaenia. Rarely Leish-
man-Donovan body (LO body) may be found in
My diagnosis is hepatosplenomegaly. buffy coat).
• Leukaemia.
Q: What are the causes of hepatosplenomegaly?
• Myelofibrosis (pancytopaenia, leucoerythroblas-
A: Mention the causes of that patient according to the age:
tic blood picture, tear- or pear-drop poikilocyte).
If the patient is middle-aged or elderly, the causes are: • Hereditary haemolytic anaemia (microcytic
• Kala-azar. hypochromic blood picture).
• Malaria. • Multiple myeloma (high ESR and rouleaux
• Chronic myeloid leukaemia (CML). formation) .
• Myelofibrosis. Q: What investigations do you suggest in
• Lymphoma. hepatosplenomegaly'?
• CLO with portal hypertension. A: As follows:
If the patient is young or of early age, the causes are: 1.Hb%, TC, DC, ESR, platelet, PBF and malarial
• Kala-azar. - parasite.
• Malaria. 2. Chest X-ray (in lymphoma and leukaemia,
• Hereditary haemolytic anaemia (fs-thalassaemia shows bilateral hilar lymphadenopathy).
major, HbE disease and hereditary spherocytosis). 3. Ultrasonogram (USG) of whole abdomen
• Lymphoma. (lymphadenopathy or other mass).
• CLD with portal hypertension. 4. Bone marrow (LO bodies, leukaemia and
myelofibrosis) .
Q: What history do you like to take?
5. If lymph node is palpable, fine-needle aspira-
A: History of fever (malaria, kala-azar, lymphoma
tion cytology (FNAC) or biopsy.
and CML), previous history of liver disease and
6. Other investigations according to suspicion of
jaundice. In child, there will be a history of growth
causes:
retardation, anaemia, jaundice and family history
• For kala-azar (see page 183).
of hereditary haemolytic anaemia.
• Investigations for CLO (see page 203).
Q: What else or relevants do you want to see? • In hereditary haemolytic anaemia (Hb
A: As follows: electrophoresis and X-ray of skull).
Causes of hepatosplenomegaly: Q: What are the causes of lymphadenopathy with
hepatosplenomegaly?
1. Infection: Kala-azar, malaria, schistosomiasis
A: As follows:
(in Middle East and Africa), enteric fever, infec-
• Lymphoma.
tious mononucleosis and cytomegalovirus (CMV)
• Chronic lymphatic leukaemia (may be found in
infection.
CML with blastic crisis).
2. Myeloproliferative diseases: • Acute lymphatic leukaemia (ALL). ~
0-
• CML. • SLE. o
• Polycythaemia rubra vera. 8
• Disseminated TB. g
• Myelofibrosis. • Others include infectious mononucleosis, cyto-
• Essential thrombocythaemia. megaJovirus (CMV) infection, HlV, kala-azar (in
3. Lymphoproliferative diseases: Chinese, also African kala-azar), sarcoidosis, bru-
• Chronic lymphatic leukaemia (CLL). cellosis and toxoplasmosis.
• Multiple myeloma. Q: What are the causesof feverwith hepatosplenomegaJy?
• Waldenstrom macroglobulinaemia.
A: As follows:
• Lymphoma.
• Malaria.
4. Cirrhosis of liver with portal hypertension. • Kala-azar.
5. Others: • Enteric fever.
• Collagen diseases (SLE,Sjogren syndrome, Felty • Viral infection (infectious mononucleosis and
CMV infection).
syndrome).
• Sarcoidosis. • Lymphoma.
• Amyloidosis. • Leukaemia (CGL, CLL,ALL,AML).
• Thyrotoxicosis (in Graves disease, rare). • Collagen disease (SLE).
• Acromegaly. • Disseminated TB.
• Storage disease (Gaucher disease and glycogen- • Brucellosis.
storage disease). • Toxoplasmosis.
• Polycystic disease. • Sarcoidosis.

Hepatosplenomegaly (Malaria)

erythrocytic cycle such as ring, trophozoite,


Presentation of a Case 11-- --,
schizont and gametocyte are found.

Present as described previously. • Gametocytes are found after about 2 weeks.


• Malarial parasites may be found both during
febrile, also in afebrile period. But falciparum
Q: What investigations are done to diagnose malaria?
is difficult to find in afebrile period.
A: As follows:
• Usual staining by Leishman or Giemsa.
• Blood film (both thick and thin).
• Rapid antigen detection.
Q: Why thick and thin films are done?
A: As follows:
• In thick film, erythrocytes are lysed, releasing all
blood stages of parasite. It is helpful to detect the
parasite even with low level of parasitaemia.
• Thin film is necessary to detect the species and
also to quantify the parasite load in Plasmodium
falciparum.
N.B. Remember the following points:
• In falciparum malaria, only ring form is found.
• Other species (Plasmodium vivax, Plasmodium
malariae and Plasmodium ovale): All stages of Plasmodium vivax
_ SHORT CASES IN CLINICAL MEDICINE

Q: Why anaemia occurs in malaria?


A: Anaemia is due to:
• Haemoiysis of infected and uninfected red cells.
• Dyserythropoiesis.
• Splenomegaly (causing sequestration and
haemodilution) .
• Reduction of folate store.
Q: Which red cells are involved by different species of
malaria?
A: As follows:
Plasmodium falciparum
• Palciparum involves red cells of all ages. Hence,
haemolysis is severe.
• Vivax and ovale involve main ly reticulocyte.
Hence, haemolysis is less severe.
• Malariae involves normoblast; it can cause glomer-
ulonephritis and nephrotic syndrome in children.
Q: What is malignant malaria and benign malaria?
A: As follows:
• Malignant malaria: It is due to P. [alciparum
associated with widespread organ damage due to
capillary blockage. It is more serious.
• Benign malaria: It is mainly due to P. vivax, less
frequently by P. ovale and malariae. Less serious.
Plasmodium falciparum (ring form) Q: What are the features of severe P. falciparum
infection?
Q: What are the species having exoerythrocytic cycle A: It may involve any systems in the body. There may
(hepatic)? How to treat? be serious complications and death. A high para-
A: P. vivax and p. ova le persist in liver cell as dormant sitaemia (> 1% or RBC infected) is an indicator of
form called hypnozoite. This may develop into severe disease. The features are:
merozoite months or years later, causing relapse 1. CNS: Prostration, cerebral malaria (features are
of malaria. For this, after treatment of malaria, confusion, convulsion, diminished conscious-
primaquine 15 mg daily for 14 days is given to erad- ness, coma without localizing sign).
icate exoerythrocytic cycle. 2. Renal: Haemoglobinuria (blackwater fever),
P. falciparum and P. malariae have no exoerythro- oliguria, acute renal failure, acute tubular necrosis.
cytic cycle, but recrudescence of fever may occur from 3. Blood: Severe anaemia, DIC, bleeding.
multiplication of parasite in red cells, not eliminated 4. Respiratory: Tachypnoea, acute respiratory
by treatment and immune process. distress syndrome, acute pulmonary oedema.
5. Metabolic: Hypoglycaemia (particularly in
Q: What is radical cure?
children), metabolic acidosis.
A: It means eradication of hypnozoite of malarial
6. Gastrointestinal/liver: Diarrhoea, jaundice,
parasite (vivax and oval e) from the liver. It is done
splenic rupture.
by primaquine 15 mg daily for 14 days.
7. Other: Hyperpyrexia, shock.
N.B. Remember the following points: Q: Which disease or haernoglobinopathy protects
• Primaquine can cause haemolysis in G6PD against malaria?
deficiency. So, G6PD should be measured A: As follows:
before giving primaquin. However, if G6PD • Palciparum does not grow well in RBC containing
is present, primaquine 30 mg weekly for 8 haemoglobin F, Cor S. Sickle-cell disease protects
weeks may be given without harmful effect. against falciparum malaria.
• Primaquine can also cause cyanosis due to • Vivax cannot enter RBC that lacks Duffy blood
formation of methaemoglobin in red cells group (West African and American blacks are
(not dangerous). protected).
4. ABDOMEN __

Q: How to treat malaria? until patient can take orally. The loading
A: As follows: dose should not be given if the patient has
1. For P viuax, P ovale and P malariae received quinine, quinidine or mefloquine
(nonfalciparurn): during the previous 24 h.
• Chloroquine: First day 600 mg (4 tablets), • Other treatment:
then 300 mg (2 tablets) after 6 h. From next o Blood transfusion may be needed for
day, 150 mg (1 tablet) twice daily for 2 days. severe anaemia.
• For radical cure: Primaquine 15 mg daily for o Monitoring and management of water
14 days (after chloroquine). and electrolyte balance, renal failure,
2. Treatment of falciparum malaria: Many cases hepatic failure, if present. Quinine may
are resistant to chloroquine. cause hypoglycaemia. So monitoring
a. Mild or uncomplicated case: of blood glucose is essential.
• Coartemether (artemether plus lumefan- o Exchange transfusion may be helpful
trine): 4 tablets stat, 4 tablets after 8 h. and with persisting high parasitaemia
then 4 tablets 12 hourly for 2 days (total (>10% circulating erythrocyte).
24 tablets). o If coma persists, lumbar puncture and
• Or, quinine 600 mg (10 mg/kg) 8 hourly CSF study to exclude other diseases
for 7 days orally. Dose can be given 12 such as meningitis or encephalitis.
hourly, if quinine toxicity develops.
Q: How to prevent malaria in travellers
• Some quinine resistance is found. So after
(chemoprophylaxis) ?
quinine, single dose of sulfadoxine 1.5 g
A: As follows:
plus pyrimethamine 75 mg (Fansidar 3
1. High chloroquine resistance:
tablets), or doxycycline 200 mg daily for
• Mefloquine 250 mg weekly; started 2-3 weeks
7 days or clindamycin 450 mg 8 hourly
before travel and continued until 4 weeks
for 7 days, or Malarone (proguanil plus
after,
atovaquone) 4 tablets once daily for 3
• Or, doxycycline 100 mg daily; started 1 week
days may be given.
before and continued until 4 weeks after travel.
• Artesunate 200 mg daily for 3 days, and
• Or, malarone (atovaquone-proguanil) 1 tablet
mefloquine 1 g on day 2 and 500 mg on
daily, from 1-2 days before travel until 1 week
day 3 may be used.
after return.
• A new drug called Lapdap (chlorproguanil
2. No chloroquine resistance:
and dapsone) is in trial.
• Chloroquine 300 mg base weekly; started
b. Complicated or severe falciparum infection 1 week before and continued 4 weeks after
or cerebral malaria: travel.
• Injection arternether 2.4 mg/kg IV at 0, 12
3. Limited chloroquine resistance:
and 24 h and then once daily for 7 days is
• Chloroquine 300 mg base weekly and pro-
the treatment of choice. Intravenous (IV)
guanill00-200 mg daily; both started] week
treatment should be replaced by oral ther-
before and continued 4 weeks after travel.
apy (2 mg/kg once daily) when the patient
• Or, doxycycline 100 mg daily; started 1 week
is able to swallow. Total cumulative dose
before and continued until 4 weeks after travel.
is 17-18 mg/kg (It is a synthetic antima-
• Or, rnalarone (atovaquone-proguanil) 1 tablet
larial drug, derived from artemisinin, and
daily, from 1-2 days before travel until 1 week
should be avoided during pregnancy,
after return.
unless strongly indicated).
• Or, mefloquine 250 mg weekly; started 2-3
• Artemether may be given in the following
weeks before travel and continued until
dose: 80 mg intramuscular (IM) twice
4 weeks after.
daily for 1 day, followed by 80 mg daily for
4 days or 80 rng 1M twice daily for 3 days. Q: What are the complications of malaria in preg-
• Alternatively, quinine IV initially 20 mg/kg nancy and how to treat?
(maximum 1.4 g), with 500 cc 5% dextrose A: As follows:
in aqua over 4 h. then 10 mg/kg 8 hourly • Fetal complications: Still birth, low birth weight,
(maximum 700 mg per dose) for 7 days, foetal distress. High foetal death in falciparum
__ SHORT CASES IN CLINICAL MEDICINE

malaria, especially in the last trimester. • Algid type may be gastric, choleric and dysenteric.
Congenital malaria may occur in 5% cases. • Septicaemic type is characterized by high fever,
• Maternal complications: Maternal death, abor- pneumonia, cardiac failure, renal failure and
tion, premature labour, anaemia, hypoglycaemia jaundice.
and acute pulmonary oedema. Treatment: Similar to complicated falciparum (see
Treatment during pregnancy: above).
• Chloroquine should be given as a usual dose.
Q: What is blackwater fever? What is the mechanism,
• In P. vivax and p. ovale infection, no primaquine
clinical features and treatment of blackwater
until delivery. Chloroquine 600 mg weekly
fever?
should be given and continued until delivery.
A: It is a severe manifestation of falciparum malaria,
• If no response to chloroquine, quinine 600 mg
occurs in previously infected person, character-
8 hourly for 1 week.
ized by sudden intravascular haemolysis, fever and
• In falciparum malaria, quinine 600 mg 8 hourly
haernoglobinuria. It is invariably associated with
for 1 week
falciparum malaria, in those who had taken antima-
Q: What is pernicious malaria? Mention the causes, larial drugs irregularly; or in non immune person,
clinical features and treatment? who had taken irregular antimalarial prophylaxis.
A; It is a severe form of falciparum malaria in which It is common in C6PD deficiency.
there is widespread capillary blockage by the para-
sitized RBC causing multiple organ damage. If not Mechanism: Actual mechanism is unknown. Prob-
properly treated, the patient may die within 1-3 days. ably due to antibody-like haemolysin that is formed
It occurs when> 5% ofRBC are parasitized. due to autoimmune reaction against RBC that have
been altered by drug, parasite or both. With some
Pathogenesis: Parasitized RBC develop knob-like
precipitating factors such as cold, trauma and fatigue,
projections on the surface, becomes sticky and adhere
there is increased formation and release of haemoly-
to the endothelium of capillaries of different organs.
sin, causing haemolysis.
As a result, there is blockage with severe anoxia and
organ damage involving brain, kidneys, heart and Clinical features: High fever with chil1 and rigour,
CIT. Increased capil1ary permeability, secondary vomiting. diarrhoea, dark-to-black urine (haernoglo-
vasoconstriction and rupture of schizont cause toxin binuria], collapse and renal failure.
liberation, leading to further organ damage.
Diagnosis: Mostly dinical and by history, malarial
Infected RBC also adheres to uninfected RBC to
parasite is not found in blood during attack.
form rosette, causing further blockage. Blood film
shows heavy parasites (both schizont and ring form). Treatment:
Clinical types of pernicious malaria: • Similar to complicated malaria.
• Cerebral type characterized by high fever, coma • Blood transfusion.
without focal neurological signs, convulsion, • Prednisolone may be helpful.
extensor plantar response. CSF study is normal. • Treatment of renal failure.

Hepatosplenomegaly (Kala-Azar)
irregular, low-grade continuous, occasionally
Presentation of a Case undulant fever (pyrexia followed by apyrexial
period). There may be associated chill and rigor.
Present as described previously.
• Usually no anorexia, no malaise, no coated
tongue.
Differential diagnosis: As in hepatosplenomegaly • Bleeding (due to thrombocytopaenia).
(seepage 178).
Physical findings are:
Q: What are the presentations of kala-azari • Emaciation.
A: Incubation period is 1-2 months (may be months • Anaemia (a.lways present, may be severe).
to years, even 10 years). • Splenomegaly, which may be huge (firm-to-
1. Fever: Usually intermittent, either double or hard).
triple rise daily. Sometimes, fever may be • Hepatomegaly.
4 • ABDOMEN __

• Lymphadenopathy: Common in African and • Indirect haemagglutination assay (IHA)


Chinese kala-azar, rare in Indian kala-azar. • Indirect fluorescent antibody test (IFAT)
• Skin: Pigmented, dry thin, scaly. Sometimes • ELISA.
there may be diffuse, warty, nonulcerative • Complement fixation test (CFT): It is non-
skin lesion containing parasite. specific, positive after 3 weeks. False positive
Q: What is the appetite in Kala-azari occurs in TB, leprosy, cirrhosis of liver and
A: Usually good. multiple myeloma. This test is performed by g:
0-
using Kedrowski bacillus. o
S
('b

Organism: Leishmania donovan; complex • Aldehyde test: Still helpful where there is no :;:l

facility. This test is nonspecific and positive


• L. donovani (India and South East Asia) when there is associated hypergarnmaglob-
• Leishmania infantum (Middle East, Mediterranean ulinaemia. It may be false positive in TB,
area) leprosy, cirrhosis of liver, multiple myeloma,
• Leishmania chagasi (South, Central America) SLE, leukaemia and thalassaemia. This test is
Source/reservoir: Human (Indian kala-azar). positive in kala-azar in 2-3 months (negative
after 6 months).
Mode of transmission:
3. Detection of antigen: Done by latex aggluti-
• Bite by infected female sandfly (common~. nation test (Katex) for detecting leishrnanial
• Congenital: Transplacental. antigen in urine. This test is very simple, more
• Blood transfusion. specific than antibody-based test, highly sensi-
tive (96%) and also specific (100%). This test
N.B. Remember the following points: indicates active disease. The antigen is detected
• Human is the only reservoir in Indian in urine within a week and disappears from urine
subcontinent. within 3 weeks following successful treatment.
• Other reservoirs include dog, jackal, fox and wild Hence, this test is helpful for early diagnosis and
rodents in other country. also to see the response to therapy.
• Leishmania has two forms: Amastigote and 4. Definitive diagnosis by isolation of Leishman-
pro mastigote. Donovan (LD) body from bone marrow and
• Amastigote form (oval) is found in human; LO spleen puncture; also done from liver, lymph
body is found in monocyte-macrophage system. node and skin lesion. Smears are stained by
• Promastigote form (flagellate) is found in Leishman, Giemsa or Wright stain. Culture is
sandfly, also in culture media. done in NNN media (Nicolle-Nave-McNeal).
Q: What investigations are done to diagnose kala-azar? LD body is positive as follows:
A: As follows: • Spleen: 90-95% (splenic puncture is avoided
1. Complete blood count (CEC): Leucopaenia, in case of soft spleen, prolonged prothrombin
high lymphocyte and monocyte, neutropaenia, time (PT) and platelet count is <40,OOO/mm3).
absence of eosinophil, thrombocytopaenia and • Bone marrow and liver: 85%.
high ESR. If CBC is repeated after some days, • Lymph node: 65%.
there is progressive leucopaenia. • LD body is rarely found in periphera1 blood
2. Immunological tests based on antibody: in buffy coat preparation. It may also be
• Direct agglutination test (OAT): It is called found in thick film (present in monocyte).
OAT because pro mastigote is used as an 5. PCR (from lymph node or bone marrow aspira-
antigen. May be positive after 2 weeks (usually tion); positive up to 100% case.
within month). It remains positive years after
cure (so it does not indicate active infection). 6. Blood for total protein and A:G ratio (high total
Disadvantage of OAT: Cannot differentiate protein, low albumin and high globulin).
past, subclinical and active infection. False-
positive OAT may occur in leprosy, African N.B. Culture is done for identification of species;
trypanosomiasis, TE and hepatitis B. and if the number of organisms is less, it may
• lmmunochromatographical test (Ier): Also grow in culture media. Organism may grow
called rK39 dipstick rapid test. after 1 week, may take 4 weeks.
_ SHORT CASES IN CLINICAL MEDICINE

Q: What are the complications of kala-azar?


A: As follows:
• Secondary infection (pneumonia, tuberculosis).
• Anaemia.
• Bleeding.
• Gastroenteritis, bacillary dysentery.
• Liver disease (cirrhosis of liver).
• PKDL.
• Rarely, cancrum oris.
Causes of death in Kala-azar: If no treatment is
given, patient may die within 1-2 year due to:
• Secondary infection.
lDbody
• Bleeding.

Q: What are the mechanisms of anaemia in kala-azar? Q: How to treat kala-azar?


A: Due to: A: As follows:
• Hypersplenism (sequestration and splenic pool- • Sodium stibogluconate: 20 mg/kg for 28 days
ing, destruction ofRBC in spleen). IV or 1M. Can be given in infusion with normal
• Short lifespan ofRBCs. saline. 1Minjection is very painful, given if there
• Haemolysis. is ECG abnormality (arrhythmia or long QT
• Ineffective erythropoiesis, infiltration of marrow interval).
by parasite. • Meglumineantimoniate: 50 mg/kgis an alternative.
• Bleeding, haemodilution. • Liposomal amphotericin B: Drug of choice, 3-4
mg/kg daily, given on days 1~5, 14 and 21. May
Q: Why bleeding occurs in kala-azar? be repeated. Dose for irnmunoincompetent is
A: As follows: 4 mg/kg on day 1-5, 10, 17, 24, 31 and 38. It is
• Bleeding may occur before treatment due to expensive and not widely available. Preferable to
thrombocytopaenia. conventional amphotericin B,as it is more effective,
• Bleeding may occur during treatment with less toxic, less protein bound, concentrated and
sodium stibogluconate. retained in macrophage-rich organs (RE system)
Q: Why bleeding occurs during treatment with sodium other than kidney (not nephrotoxic). Safe in
stibogluconate! pregnancy.
A: Actual mechanism is unknown. It may be due to: • Conventional amphotericin B: 1 mg/kg for 20
• Abnormality in preparation of drug. days, given in slow infusion for 4-6 h. It is more
• Decomposed or expired date of drug. protein bound and nephrotoxic.
• Functional abnormality of platelet by the • New oral drug: Miltefosine, a cytotoxic drug, very
drug (pentavalent antimony is transformed to effective and safe for visceral leishmaniasis. Also
trivalent, which may cause thrombasthaenia). helpful in treating resistant kala-azar. Dose is
• Bleeding due to other causes (CLD and blood 50-100 mg or 2.5 mg/kg daily orally for 28 days
dyscrasias) . (50 mg, if < 25 kg body weight; and 100 mg, if
> 25 kg body weight). Side effects are less, may
Q: How to treat if there is bleeding?
cause vomiting, diarrhoea, transient rise of serum
A: As follows:
glutamic pyruvic transaminase (SGPT), blood
• Drug should be stopped.
urea nitrogen (BUN) and creatinine. It has more
• Platelet count, PT, APIT and liver function tests
foetal toxicity; thus not recommended during
should be done.
pregnancy.
• Other causes of bleeding should be looked for. • Paromomycin: 11 mg/kg/day intramuscularly for
• Symptomatic treatment should be given, may 21 days.
require blood transfusion.
• After improvement, drug should be started with N.B. Remember the following points while treating
low dose and then increase the dose (batch of with sodium stibogluconate:
the drug may be changed). If again bleeding, • Before starting the therapy, perform renal
alternative drug may be given. and hepatic function tests, ECG {to see
4. ABDOMEN_

dysrhythmia, prolonged QT and ischaemia) Q: How to treat if there is relapse?


and chest X-ray (latent TB, pneumonia). A: Same therapy with sodium stibogluconate for dou-
• During IV injection, if the patient complains of ble duration of initial treatment (56 days). Other
chest pain or cough, stop the drug immediately. drugs may be used.
• Monitor ECG, FBC including platelet, hepatic
and renal functions. Q: How to treat a resistant case of kaJa-azar?
• The drug should be stopped, if there A: In a resistant case of kala-azar, the following therapy ~
0...
is bleeding, ECG change (arrhythmia, should be given: o
S

prolonged corrected QT interval).
No limitation of dose (previously it was
• Pentamidine isethionate. 3-4 mg/kg. three times
per week for 5-25 weeks (at least 5 weeks and
s
thoughtthat dose should not exceed 1gjday). 15 injections should be given). Then, sodium
stibogluconate 20 mgfkgfor 30 days. Pentamidine
Q: How to see the response of therapy?
may cause hypoglycaemia, hyperglycaemia and
A: As follows:
shock. It is not used as a first drug because of
• Clinical improvement: Improvement of fever,
more side effects and development of quick
feeling of well-being.
resistance.
• Reduction in the size of spleen (may take
• Amphotericin B, preferably liposomal ampho-
months).
tericin B.
• Weight gain.
• Plus adjuvant therapy: "{-Interferon, allopurinol,
• Laboratory findings include increased Hb%, total
ketoconazole.
count ofWBC and increased albumin.
Q: What do you mean by relapse, re-infection and Q: How to treat kala-azar in pregnancy?
resistant kala-azar? A: Same treatment with sodium stibogluconate.
A: As follows: Although the safety of this drug during pregnancy
• Relapse means after cure, there is again kala-azar is not established, it should be treated since the dis-
usually within 6 months. ease is potentially dangerous.
• Re-infection means after cure; there is again kala-
azar after 6 months. Q: What is kaJa-azar treatment failure (KATF)?
• Resistance means no response to drug therapy. It A: When kala-azar does not respond to usual therapy
may be primary (no response from beginning of with sodium stibogluconate, it is called KATF. It is
therapy) or secondary (develops later after initial usually due to inappropriate or suboptimal dose
response). It is usually due to insufficient or or irregular therapy. Treatment-as in resistant
suboptimal dose or irregular therapy. kala-azar.

Post-Kala-Azar Dermal Leishmaniasis (PKDL)


• Neurofibromatosis.
Presentation of a Case 11--------, • Others: Lipomatosis, acne rosacea, leukaemia cutis,
secondary syphilis and rhinophyma (irregular
• There are multiple, pale, pink reddish, wart-like
thickening of skin of nose with enlarged follicular
nodules of variable size and shape involving
orifice).
nose, cheek and ear lobule.
• Skin is thick. Q: Ask one question to the patient.
A: Previous history of kala-azar.

My differential diagnoses are: Q: What is the more likely diagnosis in this case?
• PKDL. A: More likely diagnosis is PKDL.
• Lepromatous leprosy.
• SLE. Q: What is PKDU
• Dermatomyositis. A: It is a nonulcerative, cutaneous lesion that occurs
• Sarcoidosis. after successful treatment of visceral leishmaniasis.
• MCTD. Initially st.arts as macules, then erythematous patches,
_ SHORT CASES IN CLINICAL MEDICINE

followed by wart-like nodular lesions on the face,


ear lobules and limbs. Arnastigotes are scanty in the
lesion. After treatment, visceral infection disappears,
but organisms may remain in skin. After a variable
period, skin resistance is lost with resurgence of old
infection, which leads to PKDL.
In India, it occurs in a small minority of patients
6 months to 3 years or more after the initial infection,
creating a persistent human reservoir. They present
as macules, papules, nodules (most common) and
plaques on the face, mainly around the chin. The
face is often erythematous. Hypopigrnented macules
can OCCUf over all parts of the body, and are highly
variable in extent and location. There are no systemic Early PKDL (few nodules)
symptoms and no spontaneous healing.
In Sudan, about 50% develop skin manifestations
of PKDL concurrently with visceral leishmaniasis Of
within 6 months afterwards. The skin features are
as above. In addition, there may be a measles-like
micropapular rash all over the body. Children are
more frequently affected than in India. Sponta-
neous healing occurs in about three-fourth cases
within 1 year.

Q: What is the presentation of PKDL?


A: As follows:
• Multiple, pale, pink, reddish, wart-like nodules
of variable size and shape involving nose, cheek
and ear lobule.
• Thick skin. PKDL (nodules)

In early case, depigmented macules, erythematous,


well-circumscribed lesions may be seen or all stages
of lesions may be present, such as depigmented mac-
ules, erythematous lesions and nodules, or only mul-
tiple nodules of variable size and shape are seen.

PKDL (multiple nodules)

Q: What investigation is done to diagnose PKDL?


A: As follows:
• Demonstration of amastigote form of LD body
in lesions by slit-skin smear and culture. Smear
is prepared from nodular lesions (LD bodies are
Early PKDl {depigmented macules} not found in depigmented lesion).
4. ABDOMEN __

• Immunofluorescence and immunohistochemis- Total six cycles are needed with an interval of
try may demonstrate the parasite in skin tissue. 10 days. Total 120 injections are needed.
• In the majority of patients, serological tests (DAT • If a second course is required, it should be given
or k39 strip tests) are positive. It is not helpful for after an interval of 2 months.
diagnosis. • In Sudan, same injection for 2 months is
Q: How to treat PKDL?
adequate.
A: As follows: • Alternatively, in India, several courses of ampho-
• In India, injection of sodium stibogluconate tericin B infusions may be given.
(given in cycles) 20 mg/kg daily for 20 days is
given. After 10 days interval, the course is repeated. N.B. Pentamidine is ineffective in PKDL.

Splenomegaly (Not Hepatomegaly)


~.B. Remember the following points:
Presentation of a Case :~-------, • Spleen must be enlarged at least 2-3 times of
its usual size before it can be felt. Hence, once
• The spleen is enlarged, ... ern from costal margin
a spleen is palpable, it is definitely enlarged
from anterior axillary line towards right iliac
(always pathological).
fossa.
• Enlarged spleen may be: Mild <4 em,
moderate 4-8 cm or huge> 8 cm.
• Spleen lies against posterolateral wall of
Q: Why it is spleen, not kidney? abdominal cavity beneath 9th, 1O~l and 11th
A: This is spleen because: rib. Long axis of spleen lies along the 10th rib.
• The mass is in left hypochondrium. • Spleen enlarges towards the right iliac fossa.
• Moves with respiration downwards and towards However, in children, spleen may be enlarged
right iliac fossa. vertically towards the left iliac fossa.
• I could not get above the swelling, and hand • Ifkidney is hugely enlarged, colon ic resonance
could not be insinuated between the mass and may be absent and it may not be possible to
left lower rib. get above the swelling.
• There is a notch (definitive sign).
• Percussion is dull over the mass and continuous Q: What are the causes of mass in left hypochondrium
up to the lower part of left side of chest. [differential diagnoses (DD) of splenomegaly]?
A: As follows:
Differences between spleen and kidney • Splenomegaly.
Spleen Kidney • Enlarged left kidney.
It is in the left It is in the lumbar region or • Mass in the splenic flexure of colon (carcinoma).
hypochondrium loin • Carcinoma of the stomach.
• Mass in the tail of the pancreas (carcinoma).
Moves with respiration Moves downward and
forward
• Omental mass.
towards right iliac fossa

Well-defined medial border Round in shape Q: What are the causes of splenomegaly?
Notch is present Absent A: Mention the causes according to the age of the patient:
Get above the swelling: Present and finger can be If middle-aged or elderly, the causes are:
Absent and finger cannot be insinuated between the mass • Malaria.
insinuated between the mass and left costal margin • Kala-azar.
and left costal margin
• CML.
On percussion: Dullness over Colonic resonance over the • Myelofibrosis.
the mass,which is continu- mass • Lymphoma.
ous with the left lower chest • Cirrhosis of liver with portal hypertension
Palpable Ballotable • Chronic lymphatic leukaemia.
• Tropical splenomegaly syndrome.
__ SHORT CASES IN CLINICAL MEDICINE

If young or of early age, the causes are: Causes of fever with splenomegaly
• Malaria. • Enteric fever.
• Kala-azar. • Malaria.
• Hereditary haemolytic anaemia (~-thalassaemia • Kala-azar.
major, HbE disease and hereditary spherocytosis). • SBE.
• Lymphoma. • Viral infection (infectious mononucleosis and
• Cirrhosis of liver with portal hypertension (causes CMV infection).
are Wilson disease and <x)-antitrypsindeficiency). • Lymphoma.
Causes of huge splenomegaly (may cross the • Leukaemia.
midline) • Collagen disease (SLE).
• Chronic kala-azar. • Brucellosis.
• Chronic malaria. • Toxoplasmosis.
• Disseminated TE.
• CML.
• Myelofibrosis. Causes of splenomegaly with ascites
• Cirrhosis of liver with portal hypertension • Cirrhosis of liver with portal hypertension.
(occasionally, common in early age). • Collagen disease (SLE).
• Hairy cell leukaemia. • Lymphoma.
• Adult Gaucher disease. • Leukaemia.
• Rapidly progressive lymphoma. • Disseminated TB.
Just-palpable spleen (spleen tip) found in (all the Q: How consistency of spleen may help in diagnosis
causes of huge splenomegaly in the early stage plus of splenomegaly?
the following): A: It shows the following:
• Enteric fever. 1. If the spleen is soft, causes may be:
• Subacute bacterial endocarditis (SBE).
• Enteric fever.
• Viral infection (infectious mononucleosis and
• SBE.
CMV Infection).
• Viral infection.
• Collagen disease (SLE).
• Acute leukaemia.
• Sarcoidosis. 2. If the spleen is firm, causes may be:
• Polycythaemia rubra vera. • Cirrhosis of liver with portal hypertension.
Q: What relevants do you want to sec in a case of • CML.
splenomegaly? 3. If the spleen is hard, causes maybe:
A: As follows: • Malaria.
• Lymph nodes (lymphoma, leukaemia, SLE and • Kala-azar.
viral infection).
Q: What are the causes of splenic atrophy or
• In early age, anaemia, jaundice (hereditary
hyposplenism?
haemolytic anaemia, also see frontal and parietal
A: Causes of atrophy or hyposplenism are:
bossing, and mongoloid facies).
• Sickle-cell disease (autosplenectomy due to
• Signs of CLD (see page 202).
repeated infarction in the second decade after
• Heart to see murmur (SSE).
16 years of age).
• Bony tenderness (for leukaemia).
• Coeliac disease.
• Pigmentation (for kala-azar).
• Dermatitis herpetiformis.
Causes of lymphadenopathy with splenomegaly • Occasionally in inflammatory bowel disease
• Lymphoma. (ulcerative colitis).
• Leukaemia [acute lymphoblasticleukaemia (ALL) • Essential thrombocythaemia.
and chronic lymphocytic leukaemia (CLL)].
• Viral infection (infectious mononucleosis, CMV Q: What ate the risks of hyposplenism or
infection and HIV). splenectomy?
• Collagen disease (SLE). A: After splenectomy:
• Kala-azar (Africankala-azar, also Chinese kala-azar). • Increased risk of pneumococcal infection (patient
• Others (sarcoidosis, brucellosis, toxoplasmosis should receive pneumococcal vaccine, 2-3 weeks
and disseminated TB). before splenectomy).
4. ABDOMEN __

• Meningococcal and Haemophilus infection. • Huge splenomegaly with pressure symptoms.


• Malaria. • Hairy cell leukaemia.
• Rupture of spleen.
Q: What is the blood picture after splenectomy?
• Evidence of hypersplenism (suggested by
A: Blood pictures after splenectomy or hyposplenism:
repeated requirement of blood transfusion in a
• RBCshows Howell-Jolly body (nuclear remnants
short interval).
in RBG), target cell, acanthocytic cells (crenated
or irregular RBG) and increased reticulocyte. Q: What is hypersplenism? What are the causes?
• WBCs initially, leucocytosis and increased A: Itis a syndrome characterized by:
neutrophil; it is normal after a few weeks. Later, • Splenomegaly.
increased lymphocytes and monocytes. • Reduction of one or more of blood cells
• Platelet increases immediately, normal in 1-2 (pancytopaenia) .
months. In some cases, thrombocytosis may • Increase cellularity of bone marrow.
persist. • Improvement of blood picture after splenectomy.

Q: What are the indications of splenectomy? Any cause of splenomegaly may cause hypersplen-
A: As follows: ism, but is commonly found in:
• Hereditary spherocytosis. • Haematological disease.
• ITP (when there is failure of drugs). • Portal hypertension.
• Autoimmune haemolytic anaemia (when there is • Felty syndrome.
failure of drugs). • Lymphoma.

Splenomegaly (Tropical Splenomegaly Syndrome)


• There may be anaemia, pancytopaenia and
Presentation of a Case evidence of hypersplenism.

Present the cases as in splenomegaly (see page 187). • Acute haemolytic episode may occur: and portal
hypertension may occur.

Cause of death: Due to infection. Respiratory and


Mention tropical splenomegaly syndrome (TSS) as a
skin infections are common.
cause of splenomegaly lastly, if asked.
Diagnostic features of TSS:
Q: What is tropical splenomegaly syndrome?
A: In hyperendemic malarial area, there is gross • Hyperendemic area of malaria.
splenomegaly associated with exaggerated immune • Gross splenomegaly.
response to repeated malarial infection called • Absent or low parasitaemia (no malarial parasite
tropical splenomegaly syndrome (also called hyper- is found in peripheral blood or spleen).
reactive malarial splenomegaly; also previously • Increase in serum IgM.
it was called Banti syndrome). It is characterized • Malarial antibodies may be present in blood.
by anaemia, massive splenomegaly and high 19M • Liver biopsy shows aggregates of IgM in Kupffer
levels. Malarial parasites are scanty or absent. cells detected by immunofluorescence, sinusoidal
lymphocytosis and hyperplasia.
Causes:
Aberrant immunological response to repeated infec- Q: How to diagnose and treat TSS?
tion by any of the species of malarial parasite. There is A: Diagnosis is done by exclusion of other diseases.
increased production of cytotoxic IgM antibody to Common investigations are:
CD8+ T lymphocytes, CD5+ T lymphocytes and
• Full blood count (FBG) (anaemia, thrombocyte-
increased ratio of CD4+T cells:CD8+ T cells.In African
paenia and lymphocytosis).
patient, there is increase in lymphocytes (B-type) and
• In bone marrow (lymphocytes infiltration).
may be confused with chronic lymphatic leukaemia.
• Liver biopsy shows Kupffer cells hyperplasia,
Other features ofTSS: lymphocytes infiltration and round-cell infiltra-
• Common in older children and adults. Splenomeg- tion in portal tract. In some cases, fibrosis leading
aly is common; also hepatomegaly is common. to portal hypertension.
_ SHORT CASES IN CLINICAL MEDICINE

Treatment: 2. In nonendemic area: Treatment of malaria.


1. In endemic area, chemoprophylaxis should be • Splenomegaly and anaemia usually resolve
taken: over a period of months with treatment. Long-
• Proguanil 100 rug/day plus chloroquine term treatment may be necessary to prevent
600 mg weekly. relapse.
• Folic acid 5 mg/day.

Hepatomegaly
2. If the liver is hard and nodular, the causes are:
Presentation of a Case '1---------, • Hepatoma.
• Secondaries in the liver.
• The liver is enlarged, ... ern from costal margin
• Hydatid cyst.
in the right middavicular line.
• Polycystic liver.
• Margin is sharp (or rounded).
• Others (amyloidosis, occasionally, cirrhosis
• Surface is smooth (or irregular).
of liver with macronodular).
• Tender (or nontender).
3. If firm hepatomegaly with irregular surface,
• Firm (or hard or soft in consistency).
causes are:
• Upper border of liver dullness is in right, ...
• Multiple secondaries.
intercostal space (mention the location).
• Hydatid cyst.
• There is (no or yes) hepatic bruit (or rub).
• Macronodular cirrhosis.
• Granuloma (sarcoidosis).
My diagnosis is hepatomegaly. • Amyloidosis.
4. If enlarged, tender liver, the causes are:
Q: Why it is liver?
A: Because: • Viral hepatitis.
• The mass is in right hypochondrium and in the • Liver abscess (pyogenic or amoebic).
epigastrium (left lobe). • Congestive cardiac failure (CCF).
• Moves with respiration. • Chronic constrictive pericarditis.
• Get above the swelling-is not possible. • Budd-Chiari syndrome.
• Well-defined lower margin is parallel with the • Others (hepatoma and cholangiohepatitis).
costal margin. 5. Causes of hepatomegaly (soft or slightly firm
• Percussion is dull over it, which is continuous in with smooth surface, nontender):
lower part of chest. • Malaria.
• Kala-azar.
Q: Can liver be palpable without hepatomegaly?
• Enteric fever.
A: Yes, if it is pushed downwards by any pathology in
• Myeloproliferative disease (CML).
the right side of chest, such as emphysema, pleural • Lymphoproliferative disease (lymphoma,
effusion or pneumothorax. It is normally palpable CLLand multiple myeloma).
in neonate.
• Sarcoidosis.
Q: What are the causes of hepatomegaly in this case? 6. Cause of enlargement of the left lobe:
A: Answer according to your findings and age of the • Hepatoma.
patient. • Secondary in the liver.
1. If huge hepatomegaly (firm or hard), the • Hydatid cyst.
causes are: • Liver abscess (if tender).
• Hepatoma.
Q: Why is it tender?
• Secondaries in the liver.
A: Due to stretching of Glisson capsule.
• Cirrhosis of liver (alcoholic cirrhosis, primary
biliary cirrhosis and haemochromatosis). Q: What are the causes of hepatic bruit!
• CCF. A: As follows:
• Polycystic liver. • Hepatoma is the commonest cause (due to
• Hydatid cyst. increased vascularity).
• Amyloidosis. • Acute alcoholic hepatitis.
4. ABDOMEN __

• AVfistula in liver (due to trauma or iatrogenic in 3. If viral hepatitis is suspected, perform liver func-
liver biopsy). tion tests (UTs):
• Haemangioma of the liver. • Serum bilirubin.
• SGPT.
Q: What are the causes of hepatic rub?
• Alkaline phosphatase.
A: As follows:
• Viral markers for Hepatitis B virus (HBV)
• Secondary deposit in the liver.
(HBsAg,HBeAgand anti-HBc), anti-HEV, anti- g:
• Trauma. 0-
o
HCVand anti-HAV.
• Infarction. :3
• Perihepatitis in pelvic inflammatory disease
• Prothrombin time. s
(gonococcal or chlamydial in females, called 4. If CLD is suspected:
Fitz-Hugh-Curtis syndrome). • Total protein, AG ratio and prothrombin
time (PT); also viral markers (for HBV and
Q: What else do you want to see, if liver is enlarged anti-HCV).
and tender? • Endoscopy (to see oesophageal varices).
k As follows: • Proctoscopy (to see haemorrhoid in CLD).
• Jaundice (in viral hepatitis).
• CTor MRl.
• Engorged and pulsatile neck veins, dependent
• Liver biopsy.
oedema and to see heart [in CCF, chronic
constrictive pericarditis). 5. If hepatoma, then check for c-fetoprotein.
• Tenderness and oedema in right lower chest
(in liver abscess). N.B. Normal liver edge may be just palpable; common
in children. Other causes of normal palpable
Q: What is the cause of pulsatile liver? liver are emphysema, bronchial asthma and
A: Tricuspid regurgitation (also vascular anomaly, AV subphrenic abscess.
fistula ).

Q: What are the causes of hepatomegaly in cardiac Q: How liver biopsy is done?
diseases? A: There are four methods:
A: As follows: • Percutaneous (by Vim-Silverman needle, Meng-
hini needle or Tru-cut needle).
• CCF.
• Pericardial effusion. • Transjugular (ifPT is prolonged and platelet count
• Chronic constrictive pericarditis. is low).
• Cardiomyopathy secondary to alcoholism, • Laparoscopy (if there is bleeding, it can be
haemochromatosis, amyloidosis. stopped easily through laparoscope).
• During laparotomy (if done for other reason).
Q: What are the causes of hepatomegaly associated
with jaundice? Before liver biopsy, ensure the following points:
A: As follows: • Consent and cooperation of the patient.
• Acute viral hepatitis. • Exclude biliary obstruction, marked ascites,
• Wei! disease. severe anaemia and high bilirubin.
• Haemolytic anaemia.
• Prothrombin time should be normal (not more
• Obstructive jaundice. than 4 s that of control).
Q; Mention one investigation that is helpful for the • Activated partial thromboplastin time (APIT)
diagnosis. should be normal.
A: Ultrasonography of hepatobil iary system. • Platelet count should not be <lOO,OOO/mm3.

Q: What investigations do you suggest? Follow-up after biopsy:


A: Investigations should be done according to the sus- • The patient should be in complete bed rest for
picion of cause: 24 h.
1. USG of whole abdomen (first preferred test). • Regular monitoring of pulse and BP.
2. Complete blood count (CSC) (leucocytosis in • Blood should be kept ready for transfusion, if
pyogenic liver abscess). necessary.
_ SHORT CASES IN CLINICAL MEDICINE

N.B. If bilirubin is high, liver biopsy should not be Q: What is liver span?
done, as liver tissue does not take the stain. A: By percussion, upper border of liver dullness is
Complications of liver biopsy: in the sixth intercoastal space (ICS) or rib in right
• Bleeding. midclavicular line and the distance between this
• Pain (may radiate to shoulder). upper border and lower border is called liver span.
• Rarely, biliary peritonitis and pneumothorax. Normally, it is less than 13 cm.
5
8
o
"'0 Hepatomegaly (Hepatoma)
~
Investigations Investigations
Presentation of a Case
• a-fetoprotein increased • o-fetoproteln usually not
• Alkaline phosphatase increased
• The liver is enlarged, ... cm from the costal
slightly increased • Alkaline phosphatase highly
margin in right midclavicular line.
• Carcinoembryonic increased
• Margin is irregular. antigen normal • Carcinoembryonic antigen
• Surface is irregular and nodular. increased
• Nontender.
• Hard in consistency.
• Upper border of liver dullness in ... intercostal Q: How to suspect that hepatoma has developed in
space in right midcIavicular line. cirrhosis of liver?
• There is hepatic bruit. A: In the following way:
• Rapid deterioration of general condition.
• Pain in right hypochondrium.
My differential diagnoses are: • Increasing ascites, not responding to usual
• Hepatoma. therapy.
• Secondary deposit in liver. • Enlarging liver with appearance of nodule (hard
• Others: Hydatid cyst and polycystic liver. and irregular).
• Presence of bruit.
Q: Why is tbis hepatoma? • Biochemically increase o-fetoprotein.
A: Because the liver is hard, irregular, nodular,
non tender and there is hepatic bruit. Q: What are the causes of hepatoma?
A: As follows:
Q: Why not secondary deposit?
• Chronic hepatitis B infections. HepaoceUular
A: In secondary deposit, the nodules are usually
carcinoma (HCC) is four times common in those
multiple and small. There may be umbilication but
with HBeAgpositive than those with HBsAgalone.
no hepatic bruit. (There may be history of a primary
75-90% are associated with cirrhosis of liver.
carcinoma. )
• Chronic hepatitis C infections. Risk of HCC
Q: How to differentiate between primary carcinoma is higher in hepatitis C virus (HCV) than HBV.
(hepatoma) and secondary carcinoma? Almost always associated with cirrhosis.
A: As follows: • Cirrhosis of liver: In alcoholic cirrhosis, nonalco-
holicsteatohepatitis (NASH), haemochromatosis,
Primary Secondary Wilson disease, at-antitrypsin deficiency and pri-
History of hepatitis B or C History of primary carcinoma mary biliary cirrhosis. Macronodular variant is
infection or cirrhosis (GIT,bronchus, breast, thyroid more prone to develop hepatoma.
of liver and kidney); no primary source
• Aflatoxin produced by a fungus Aspergillus flavus
in 50% cases
(from contaminated ground nut grain stored in
Nodule usually single, Usually multiple nodules tropical condition).
may be more
• Chronic arsenicosis.
No umbilication over the There is umbilication (due to • Clonorchis sinensis (a parasitic infection).
nodule necrosis)
• Prolonged androgen therapy, anabolic steroid
Bruit present (due to No bruit (because of the necrotic and oral contraceptive pill (oestrogen) may cause
increased vascularity) lesion)
hepatoma (usually adenoma, rarely hepatoma).
Rub may be present Rub is more common • Smoking (rare).
4· ABDOMEN_

Q: What are the screening tests to detect hepatoma in


cirrhosis of liver?
A: USC and o-fetoprotein are the screening tests. USC
every 3-6 months in high-risk group to find out
Band C infection, alcoholic cirrhosis and haerno-
chromatosis.
Q: What investigations are done to diagnose
hepatoma?
A: As follows:
1. USC (shows filling defects in 90% of cases).
2. o-fetoprotein: High (60%), normal (one-third).
Levels increase with the size of tumour.
3. Others:
• CTor MRI.
• Carcinoembryonic antigen (high in
secondaries) .
• Plain X-ray of abdomen to see calcification in
liver (sun burst calcification).
4. Liver biopsy under USC control. (This is con-
firmatory but avoided in patient eligible for
transplantation or surgical resection because of
small risk of tumour seeding along the needle
tract).
5. Viral markers (HBV and HCV).
(T scan of liver (hepatoma). (A, B) Hepatoma;
N.B. Metabolic abnormalities in hepatoma are «(, 0) secondary deposit in liver
polycythaernia, hypercalcaemia, hypoglycae- Q: What are the other primary tumours of liver?
mia and porphyria cutanea tarda. A: Rarely, fibrolarnellar hepatocellular carcinoma, com-
mon in young adults, affecting equally in male and
female, in the absence of hepatitis B and cirrhosis.
The tumour is large at presentation, c-fetoprotein is
normal. This tumour is treated by surgical resection.
Other primary tumours include hemangioendothe-
lial sarcoma, cholangiocarcinoma, hepatoblastoma,
leiomyosarcoma, fibrosarcoma.
Q: What are the benign tumours of liver?
A: As follows:
• Haemangioma is the commonest. Rarely causes
symptom. No treatment is required.
• Adenoma, more in females, caused by oestrogen
containing oral contraceptives, androgen and
anabolic steroid. Surgical resection, if pressure
symptoms; also if pregnancy is desired (as size is
increased in pregnancy).
• Focal nodular hyperplasia.
• Fibroma.
• Leiomyoma.
Q: Why secondary carcinoma is more common in the
liver?
A: Because relatively blood flow in liver is more; dou-
ble blood supply (by portal vein and hepatic artery).
__ SHORT CASES IN CLINICAL MEDICINE

Q: How to treat hepatoma? highly suggestive. Level increases with size, may
A: As follows: be normal in small size.
• Surgical resection in noncirrhotic patient. 2. Others:
Recurrence is 50% at 5 years. • Chronic active hepatitis (indicates hepatocel-
• In cirrhotic patient, resection may be done with lular regeneration).
small tumour and good liver function. • Acute viral hepatitis (indicates hepatocellular
5 • Liver transplantation (5 year survival is 70% with regeneration) .
so single tumour <5 ern. or three tumours <3 em).
• Acute hepatic necrosis following paracetamol
"0
Hepatitis Band C may recur in transplanted liver.
toxicity.
~ • Percutaneous injection of ethanol, if tumour size
• Embryonic tumours of the ovary and testis.
is 3 em or less, 80% cure rate. Recurrence is 50%
• Embryonic hepatoblastoma.
at 3 years. Repeated injection may be given. It
• Rarely, in other malignancy-like carcinoma of
causes tumour necrosis.
stomach, pancreas, bile duct and gall bladder
• Transcatheter radiofrequency ablation using a
(usually with large metastasis in liver).
single electrode inserted into the tumour under
radiological guidance. • During pregnancy (in serum and amniotic
• Transcatheter hepatic arterial embolization fluid), high level indicates neural tube defect
[transarterial chemoembolization (TACE) J by (meningomyelocele or anencephaly).
Gelfoam and doxorubicin. TACE is contraindi-
cated in decompensated cirrhosis and multifocal
Fitz-Hugh-Curtis Syndrome
HCC.
• Chemotherapy: Doxorubicin may be effective in Fitz-Hugh-Curtis syndrome is caused by Chlamydia or
30% cases. Sorafenib IV, a multikinase inhibitor, Gonococcus infection, which tracks up the right paracolic
is under phase III trial. gutter to cause perihepatitis, secondarily from endocer-
vical or urethral infection. It is characterized by fever,
Q: What is o-fetoproteint What are the causes of high
pain in the right hypochondrium with radiation to right
o-fetoprotein!
shoulder, tender hepatomegaly, hepatic rub and small
A: It is a normal component of plasma protein, pro-
right pleural effusion, and so on. (Chlamydia infection
duced by the foetal liver older than 6 weeks and
is asymptomatic in 80% cases.) Investigation shows
reaches maximum concentration at 12-16 weeks of
endocervical swab for microscopy and special culture,
fetal life. It falls few weeks after birth. Reappearance
direct fluorescent antibody for Chlamydia, ELISA and
in blood in adult life is pathognomonic. Causes of
PCR may be performed.
high o-fetoprotein:
1. Hepatoma is the commonest, may be very high, Treatment: Tetracycline or doxycycline or erythromycin
>500 ngjmL in patients with liver disease is or azithromycin are used for Chlamydia infection.

Ascites with Splenomegaly (Tuberculous Peritonitis)

Presentation of a Case :1--------.

• The patient has ascites with splenomegaly.

My differential diagnoses are (causes of splenomegaly


with ascites):

• Cirrhosis of liver with p0l1al hypertension.


• Lymphoma.
• SLE.
• Disseminated TB.
• Leukaemia. Huge ascites
4. ABDOMEN __

Q: How to diagnose tuberculous peritonitis? • Laparoscopy: Tubercle may be seen over


A: As follows: the peritoneal and omental surface (biopsy
1. History: should be taken).
• History of pulmonary tuberculosis.
Q: What is the commonest site of abdominal
• General features of tuberculosis like low-
tubercu losis?
grade fever, mainly evening rise, weight loss,
A: Ileocaecal region. (Peritoneum is the second com-
sweating. ~
mon site.) 0-
• Abdominal pain, diarrhoea or features of o
Q: What are the types of tuberculous peritonitis? S
intestinal obstruction. ('l>

::l
2. Physical examination: A: Three subgroups:
• Abdomen is doughy. • Wet type: In such case, ascitic fluid protein is
• Mass in right iliac fossa (ileocaecal TB). >20 gm/L: tubercle bacilli are rarely found.
3. Investigation: • Dry type: In such case, patient presents with
• CBC, ESR. subacute intestinal obstruction, which is due to
• MT (positive in 50% cases). tubercular small bowel adhesion.
• Chest X-ray (may be primary focus in 50% • Fibrous type: In such case, patient presents with
cases). abdominal pain, distention and ill-defined
• USG of abdomen, cr scan of abdomen. irregular tender abdominal mass.
• Ascitic fluid study: Q: What is the feeling of abdomen on palpation in
o Straw colour.
tuberculous peritonitis?
o High protein >25 giL (exudative), low
A: Doughy feeling.
glucose.
o High lymphocyte. Q: How to treat?
o Adenosine deaminase (ADA) is high. A: As follows:
o Acid-fast bacilli (AFB) are rarely found. • Standard anti-TB drugs for 1. year.
Mycobacteria culture and sensitivity may • Surgery in some cases (if fistula or obstruction).
be positive for up to 50% cases. • Symptomatic treatment for pain and diarrhoea.
o PCR of ascitic fluid. • Prednisolone in some cases.

Hepatomegaly (Tender Liver, Viral Hepatitis)


Q: What history do you like to take in viral hepatitis?
Presentation of a Case :1---------, A: As follows:
1. Anorexia, nausea and vomiting.
• Liver is enlarged, ... em from the costal margin
2. Pain (usually in right hypochondrium).
in right midclavicular line.
3. High-coloured urine, yellow eye and body.
• Margin is sharp, surface is smooth, tender, soft 4. Itching, pale stool (due to intrahepatic
in consistency, upper border of liver dullness is
cholestasis) .
in right, ... intercostal space (mention location)
5. History of (for HBV):
and no hepatic bruit or rub.
• Injection or infusion of blood and blood
products, any fluid.
• Injection of contaminated needle or sharing
My diagnosis is tender hepatomegaly, which may be of syringe by others or parenteral drug
due to: abusers.
• Acute viral hepatitis. • Close contact with infected person.
• Liver abscess. • Newborn in infected mother with HBV.
• CCF. • Acupuncture or tattooing.
• Chronic constrictive pericarditis. • Homosexuality.
• Weil disease • Organ transplantation.
• Chronic haemodialysis.
Q: If it is viral hepatitis, what else do you want to see? • Travel to other parts.
A: Jaundice. • Newborn in infected mother with HBV.
_ SHORT CASES IN CLINICAL MEDICINE

N.B. Clinical features and pathological features are • Anti-Hbc (IgM type indicates acute infection).
same by all viruses. They differ in their tendency • HBVDNA.
to cause acute and chronic infections. HBV and
HCV may cause chronic hepatitis; hepatitis D Q: What are the markers of B virus replication?
A: HbeAg, HBY DNA.
virus (HDV) can cause chronic infection with
HBY. HAY and hepatitis E virus (HEV) are not Q: What are the causes of acute hepatitis?
associated with chronic infection. A: As follows:
Q: What investigations do you suggest in acute viral • Acute viral hepatitis (see below).
hepatitis? • Drugs: Paracetamol, alcohol.
A: As follows:
• Nonviral infection: Toxoplamsa gondii, Leptospira
1. CBC (there may be leucopaenia, with relative iaerohaemorrhagiae, Coxiella butnetii (Q fever).
• Others: Pregnancy, Wilson disease, poisons
lymphocytosis) .
(amanita phyllodes mushroom, aflatoxin, carbon
2. LFrs:
tetrachloride) .
• Serum bilirubin: High.
• SGPT: High. Q: What are the viruses causing hepatitis?
• Serum glutamic oxaloacetic transaminase A: As follows:
(SGOT): May be high (SGOT is raised in • Hepatitis A, B, C, D and E virus.
drug-induced hepatitis). • Other viruses are Epstein-Barr virus, CMV, herpes
• Alkaline phosphatase may be high (rarely more simplex virus and yellow fever virus.
than twice the upper limit. In cholestatic hepa-
titis, alkaline phosphatase may be very high). N.B. Hepatitis D virus is a RNA-defective virus can
1
• Prothrombin time: Prolonged in severe infect only with B virus or can superinfect in
hepatitis. those who are HBV carriers. Only B virus is
3. Viral markers: DNA type, and all others are RNA type.
• Virus A (ami-HAV, IgM indicates acute
infection ).
Q: What are the causes of chronic hepatitis?
• Virus B (HBsAg, HBeAg, anti-HBc). In acute
A: As follows:
infection, HBsAg may be cleared rapidly;
anti-HBc IgM is diagnostic. • Virus: HBV, T-lCVand combined HBV and HDV.
• Yirus E (anti-HEY, IgM indicates acute • Autoimmune.
infection), anti-HCV. • Drugs: Methyldopa, isoniazid, ketoconazole,
nitrofurantoin.
4. USG ofhepatobiliary system.
S. Others: Blood sugar, urine routine examination • Alcohol.
• Hereditary: Wilson disease, haemochromatosis,
(R/E).
<Xl-antitrypsin deficiency.
N.B. Remember the following points: • Inflammatory bowel disease (ulcerative colitis).
• HBsAg appears in the blood 6 weeks to 3
months after acute infection, then disappears. Q: How would you treat a patient with acute viral
• HBeAg rises early and declines rapidly. hepatitis?
• Anti-HBc is the first antibody to appear and A: As follows:
high titers of IgM anti-HBc suggest acute • Bed rest.
infection and continuing viral replication. It • Normal diet.
persists for many months. • Supportive and symptomatic.
• Anti-HBsAg appears late and indicates • Avoid sedative, opium and alcohol.
immunity.
Q: What is the prognosis of acute viral hepatitis?
• Anti-HBe appears after anti-HBc. Its
A: It depends on the type of the virus:
appearance indicates decreased infectivity.
1. HAY:
Q: What are the markers of HBV infection? • Good, recovery in 3-6 weeks.
A: As follows: • Rarely (0.1 %) develop acute liver failure.
• HbsAg. • Mortality in young adult is 0.1 % from
• HbeAg (indicates active replication; if persists for acute fulminating hepatic failure. Mortality
>6 months, it indicates chronic infection). increases with age.
4· ABDOMEN ~

2. HBV: Q: What are the complications of acute viral


• In 90-95% cases, full recovery occurs within hepatitis?
6 months. A: As follows:
• 5-10% develop chronic infection, which
• Acute fulminating hepatic failure (by Band
usually continues for life. Many may remain
sometimes with E viral infection in pregnancy. It
as inactive HBY infection. In some, cirrhosis
is rare by HAV).
of the lever and hepatoma may develop after
• Relapsing hepatitis, which may be clinically
many years. Cirrhosis usually develops in
evident (5-15%) Of, more commonly, only
5-20% cases in 5-20 years. This proportion
biochemically detectable. Ttsettles spontaneously.
is higher in those with e-antigen positive.
• Cholestatic hepatitis mostly by HAV; may persist
• <1% may develop fulminating liver failure.
for 7-20 weeks.
• Infection from mother to child during
pregnancy leads to chronic infection in child • Posthepatitis syndrome is seen in anxious
in 90% cases and recovery is rare. patient who complains of malaise, anorexia,
• Chronic infection is also common in Down nausea, vomiting, right hypochondrial pain
syndrome and HIV infection. or discomfort in the absence of clinical
3. HCV: or biochemical evidence of liver disease.
• Rarely causes acute infection. Reassurance is necessary.
• Commonly, it causes chronic hepatitis. • CLD (due to Band C VilUS), which may lead to
4. Combined HBV and HDV infection can cause cirrhosis of liver and ultimately to hepatoma.
more aggressive disease. • Others: Aplastic anaemia (usually reversible),
5. HEV: rarely Coombs positive haemolytic anaemia,
• Similar to HAV. polyarteritis nodosa, Henoch-Schonlein pur-
• Can cause acute liver failure with high pura, glomerulonephritis and collagen vascular
mortality in pregnancy. disease.

Brief Discussion on Weil Disease (Leptospirosis)


Q: What is Weil disease? What is its mode of infection, necrosis, manifested as oliguria or anuria)
clinical features, investigation and treatment? and haemorrhage (epistaxis, haematemesis,
A: It is a leptospiral disease caused by Leptospira utero- melaena, etc.) may occur. There may be liver
haemorrhagiae characterized by high fever, jaundice, failure, myocarditis, pulmonary haemor-
haemorrhage and renal impairment. rhage, encephalitis, aseptic meningitis may
occur. There may be cardiac failure, haemo-
Mode of infection: Spread is typically by contad
lytic anaemia, thrombocytopaenia.
with infected rat urine.
• Relative bradycardia, neck rigidity may be
Incubation period: 7-14 days. found.
Clinical features: There are two phases: 2. Immunological phase: There is development of
antibody and leptospira disappears from blood.
1. Leptospiraemic or bacteremic phase: This phase
It lasts for 2-5 days. Features are usually mild,
lasts for 1 week. Organisms may be found in
but meningism or aseptic meningitis and irido-
blood and CSF. Characterized by:
cyclitis may occur. Majority recover uneventfully.
• High fever with chill.
• Anorexia, nausea, vomiting. N.B. Any patient with high fever, jaundice, bleeding
• Headache, myalgia (mainly calf and back). manifestations and renal involvement; Weil dis-
• Conjunctival suffusion (blood-shot eyes). ease is a strong possibility.
• Skin rash (maculopapular, purpura, bruise,
Organism and animal hosts:
etc. ).
• Lymphadenopathy. • Leptospira ictetohaemorthagiae of rat.
• Jaundice, hepatosplenomegaly. • Leptospira pomona of pig.
• In severe case, renal impairment (caused by • Leptospira canicola of dog.
impaired renal perfusion and acute tubular • Leptospira hardjo of cattle.
_ SHORT CASES IN CLINICAL MEDICINE

Investigations: Treatment:
1. Antibiotic:
• CBC: Usually polymorphonuclear leucocytosis; • JVbenzyl penicillin (1.5 million units 6 hourly
thrombocytopaenia in severe case. for 1 week) or ampicillin (1 g 6 hourly for
• Blood culture (Fletcher media): Positive in first 1 week). Ceftriaxone (1 g daily for 1 week) is
week of illness (within 4 days of illness). as effective as penicillin.
• Urine RjE: Proteinuria, haematuria, RBC cast. • Doxycycline (100 mg 12 hourly for 1 week) or
• Urine culture for leptospira in second week. ampicillin (750 mg 6 hourly for 1 week) when
• LFf: High bilirubin, SGPT and PT. started within 4 days of onset of symptoms.
2. In renal failure and jaundice:
• Serum CPK is high.
• Fluid and electrolyte balance must be
• Serological test: Microscopic agglutination test
maintained.
(MAT) positive at the end of first week ELISA,
• Dialysis may be needed.
immunofluorescence technique, etc. may be
• Exchange transfusion may be needed in
done.
severe hyperbilirubinaemia.
• CSF study: Abnormal in 90% cases.
3. If anaemia and thrombocytopaenia: Blood
• PCR: Leptospira DNA by PCR in blood can be transfusion may be needed.
detected in early symptomatic disease and also in
urine from eighth day of illness; may remain for Prophylaxis: Doxycycline 200 mg weekly may have a
months thereafter. role.

Hepatomegaly (Liver Abscess)


• Haematogenous:
Presentation of a Case :1-------, o Portal pyaemia (mesenteric infection) from
intra-abdominal sepsis, suppurative appendi-
Present as in viral hepatitis.
citis and perforation.
o Septicaemia or bacteraemia (along the hepatic
My diagnosis is tender hepatomegaly, which may be artery).
due to:
• Direct extension from peripheral abscess.
• Acute viral hepatitis.
• Trauma.
• Liver abscess.
• Infection of liver tumour or cyst.
• CCF.
• Chronic constrictive pericarditis. Causal organisms are Escherichia coli, Streptococcus
• Weil disease millen, Streptococcus faecalis or other Streptococcus spe-
Q: What else do you like to see in liver abscess? cies, Staphylococcus aureus, Proteus vulgaris, anaerobic
A: As follows: organisms or bacteroids (from large bowel). Often
• Local oedema and fullness of intercostal space the infection is mixed.
(in right lower chest).
• Local intercostal tenderness (right lower chest) Q: How does the patient usually present?
called punch tenderness. A: As follows:
• Patient may have high temperature and looks toxic. Pyogenic liver abscess:
Q: What are the types of liver abscess? • Fever, may be high with chill and rigour, malaise,
A: They are of two types: anorexia and weight loss.
• Pyogenic liver abscess. • Abdominal pain, mainly in right upper abdomen,
• Amoebic liver abscess. may radiate to right shoulder.
Q: What are the causes of pyogenic liver abscess! What • Pleuritic right lower chest pain (may be small
are the causal organisms? pleural effusion, pleural rub).
A: As follows: • Jaundice is usually mild, may be severe in
• Ascending cholangitis in biliary obstruction multiple abscesses causing biliary obstruction.
(common bile duct) by stone, stricture and neo- • Sometimes, only pyrexia of unknown origin
plasm, or spreads from empyema of gall bladder. (PUO) may be present.
4. ABDOMEN __

N.B. Single abscess is common in right lobe; Points Pyogenic Amoebic


multiple abscesses are due to infection Organism Escherichia coli Escherichia histolytica
secondary to biliary obstruction. Immuno- and others
compromised patients are likely to develop History Cholangitis, Amoebiasis
liver abscess. Complications include rupture, septicaemia
secondary infection and septicaemia. Symptoms High fever with Fever mild-to-moderate,

Amoebic liver abscess (common in right lobe, usually


chill and rigour no chill and rigour 6:
0..
0
Neutrophil Common Less
single): S
(1)
leucocytosis !:l
• History of diarrhoea or intestinal disease (absent Ultrasonography Multiple lesions Usually single
in 50% cases). Chocolate or anchovy
Aspiration Frank pus
• Fever (low grade) and pain. sauce
• Toxicity is absent. Prognosis More fatal Lessfatal

N.B. Amoebic liver abscess is usually large, single


and located in right lobe. Multiple abscesses Q: What investigations do you suggest in liver abscess?
may occur in advanced disease. A: As follows:
1. FBC (leucocytosis in pyogenic liver abscess).
2. LfT (usually high alkaline phosphatase; bilirubin
is high in 25% cases. Aminotransferases are
usually normal, may be slightly high).
3. Serum albumin (low), a fetoprotein.
4. Chest X-ray (PA view) shows raised right dome
of diaphragm, small right-sided pleural effusion
or collapse of right lung.
5. Blood for culture and sensitivity (CjS) (in pyo-
genic, positive in 30% cases. In some series,
50-80% cases may be positive).
6. USG of the upper abdomen.
7. CT scan (shows multiple bull's-eye appearance)
or MIU of the upper abdomen may be done.
liver abscess (fullness of intercostal space) 8. If needed, USG-guided aspiration of pus for
CIS (in pyogenic), and amoebae is rarely found
in pus.
9. For amoebic liver abscess:
• Immunofluorescent antibody test
(positive in 95%).
• Indirect haemagglutination test
(positive in 95%).
• Complement fixation test.
• ELISA.

liver abscess (swelling in right lower chest)

Q: What is the character of pus in amoebic liver abscess?


A: Pus is in chocolate or anchovy sauce colour.
Q: What are the differences between pyogenic and
amoebic liver abscess?
A: As follows:
__ SHORT CASES IN CLINICAL MEDICINE

indicated for chronic persistent abscess or


pseudotumour.
• Treatment of the underlying cause like biliary
obstruction.
2. Amoebic liver abscess:
• Metronidazole 800 mg 8 hourly for 10
days, or secnidazole 2 g daily for 5 days, or
tinidazole or ornidazole 2 g daily for 3 days.
• Nitazoxanide 500 mg 12 hourly for 3 days.
• Diloxanide furoate 500 mg 8 hourly for 10
days or paromomycin should be given for
liver abscess (Aand B)
intestinal infection. This is given after the
Q: How to treat liver abscess? treatment of amoebic liver abscess.
A: As follows: • USG-guided aspiration should be done if
1. Pyogenic liver abscess: abscess size is large (>5 ern), there is no
• Antibiotic (amoxicillin plus gentamicin) plus response to meclical therapy or if there is a
metronidazole. Cefoperazone (1-2 g IV 12 risk of bursting.
hourly) with metronidazole (500 mg every
N.B. Remember the following points:
6 h) is an alternative. To be continued for 2-3
• Mixed infections may be found in many
weeks, sometimes up to 6 weeks.
cases. Hence, antibiotic plus antiamoebic
• If larger (>5 em) liver abscess or not respond-
drugs may be needed.
ing to antibiotic therapy, then USG-guided
• Abscess may rupture into pleural cavity,
aspiration should be done. pericardia] sac or peritoneal cavity. In such
• Surgical drainage is rarely needed. Even cases, immediate aspiration or surgical
more rarely hepatic resection may be
drainage is needed.
-

Hepatomegaly (Hydatid Cyst)


• Occasionally, jaundice due to obstruction in bile
Presentation of a Case 11-- -----,
duct.
• Rarely rupture into the abdominal cavity; pleural
• The liver is enlarged, ... em from the costal
cavity or biliary tree may occur. If it ruptures into
margin in right rnidclavicular line.
the biliary tree, ther~ may be intermittent jaun-
• Margin is rounded.
dice, abdominal pain and fever. A cyst rupturing
• Surface is nodular.
into the bronchus may cause expectoration and
• Nontender.
spontaneous cure.
• Hard in consistency.
• Upper border of liver dullness in ... intercostal • Features of cyst in other organs: A cyst in the
space in right midclavicular line. lung may be infected causing a lung abscess or
• There is no hepatic bruit. pulmonary abscess. If there is a cyst in the brain,
it may cause seizure. Renal involvement may
cause haematuria or lumbar pain.
My differential diagnoses are:
• Calcification of cyst occurs in 40% of cases.
• Hepatoma.
• Secondary deposit in liver.
Q: What are the complications of hydatid cyst?
• Hydatid cyst.
A: As follows:
• Polycystic liver.
• Pressure effect to the surrounding tissue.
• Cirrhosis of liver (macronodular).
• Rupture (abdominal or pleural cavity, biliary
Q: How does the patient present with hydatid cyst of tree); may cause anaphylactic shock.
liver? • Secondary infection.
A: As follows:
• Usually asymptomatic. Q: What are the sites of hydatid cyst?
• Mass in right hypochondrium. A: As follows:
4. ABDOMEN_

• Liver: 60%, usually right lobe (liver is the first


filter).
• Lungs: 30% (it is the second filter).
• Kidneys: 3%.
• Brain: 1%.
• Any organ (spleen, heart muscles and biliary
tree). g:
0...
o
Q: How human is infected by hydatid cyst? 3
C1>
::l
A: Close contact with infected dog, or eating under-
cooked vegetables or drinking water contaminated
with faeces of infected dog. Infection commonly
occurs during childhood. After ingestion of the eggs,
the embryo in liberated from the ovum in the smaJI
intestine and gains access to the blood stream and
Hydatid cyst in liver
is carried to other organs. A slowly growing thick-
walled cyst is formed, inside which further larval
stage of the parasite develops.

Q: Who are the definitive and intermediate hosts?


A: As follows:
• Definitive host: Dogs (common) and other
canine animals (fox, wolf and jackal).
• Intermediate host: Sheep (common), pig, cattle,
goat and man.

Q: What are the organisms of hydatid cyst?


A: As follows:
• Echinococcus granulosus of dogs.
• Echinococcus multilocularis life-cycle between fox
and vole. Man is infected accidentally.

Q: How are dogs infected? Simple cyst in liver


A: Byeating infected meat df sheep, pig, cow, etc. These
animals are infected while grazing in the field con- Q: How to treat hydatid cyst?
taminated with dog's faeces. A: As follows:
1. Surgical treatment: Cyst should be removed,
Q: What investigations do you suggest? if possible, after first sterilizing the cyst with
A: As follows: alcohol, 2.7% NaCI or 0.5% silver nitrate. Prazi-
• CBC (high eosinophils). quantel 20 mg/kg for 14 days kills protoscolices
• USG and (,1 scan: Shows cyst and also daughter perioperatively.
cysts within the parent cyst. 2. Medical treatment:
• Serology by performing complement fixation • Albendazole 15 mg/kg in two divided doses
test (CFT) (positive in 70-80% cases), for 12 weeks to 6 months. May be repeated
haem agglutination test (positive in 85%), (it cures in 30% and provides symptomatic
flocculation test, indirect fluorescent antibody relief in 50% cases).
test, immunoelectrophoresis test (Arcs-test) and • Mebendazole 400 mg twice daily for 12 weeks
ELISA(positive in 70-90% cases). to 6 months; may be repeated.
• Plain abdomen X-ray (calcification may be.seen). 3, PAIR therapy (Puncture, Aspiration, Injection
• Casoni test (nonspecific): It is done by intradermal and Re-aspiration): Percutaneous aspiration
injection of 0.2 mL hydatid fluid in forearm of cyst, followed by injection of 100% ethanol
(a control with normal saline in other arm). In into the cyst, then re-aspiration of cyst contents.
positive cases, there is formation of wheal with Mebendazole may be combined with PAIR.
pseudopodia in 20-30 min (it disappears in 1 h). 4. Calcified cyst may be left untreated.
_ SHORT CASES IN CLINICAL MEDICINE

Chronic Liver Disease or Cirrhosis of Liver


(May be isolated splenomegaly, hepatomegaly, or • Engorged veins in chest and also abdomen
hepatosplenomegaly or ascites.) (due to portal hypertension).
4. Others:
Presentation of a Case • Abdomen with splenomegaly, ascites,
engorged veins and caput medusae.
• There is generalized distension of abdomen, • Testis is small and atrophied.
flanks are full and umbilicus is everted. • Generalized pigmentation (any CLD can
• Shifting dullness is present. cause pigmentation, but commonly in
• Fluid thrill is present (mention if any). haemochromatosis and primary biliary
• Spleen is enlarged, 2 em from the left costal margin cirrhosis) .
in anterior axillary line towards the right iliac fossa. • Purpura and ecchymosis.

Q: What specific findings will you see in CLD or


My diagnosis is ascites with splenomegaly, which is cirrhosis of liver in a young patient?
more likely due to cirrhosis of the liver with portal A: As follows:
hypertension. • The eyes should be examined to see Kayser-
Q: What are the causes of splenomegaly with ascites? Fleischer (KF) ring for Wilson disease.
A: As follows: • In lung, signs of emphysema may be present in
• Cirrhosis of liver with portal hypertension. case of <Xl-antitrypsindeficiency.
• Collagen disease (SLE). Common signs in alcoholic cirrhosis:
• Lymphoma.
• Bilateral parotid enlargement.
• Leukaemia.
• Florid spider angioma.
• Disseminated TB.
• Dupuytren contracture.
Q: What else do you like to see? • Gynaecomastia.
A: I want to see the stigmata of CLD, lymph node in • Testicular atrophy with loss of body hair.
other areas, skin rash, mouth ulcer, joints (SLE).
Q: What is chronic liver disease (CLD) and chronic
Q: What are the stigmata or signs of CLD? liver failure?
A: As follows: A: As follows:
1. Hands: • CLD is defined as liver disease for 6 months or
• Palmar erythema (liver palm). longer.
• Dupuytren contracture. • Chronic liver failure means functional capacity of
• Leuconychia. liver cannot be maintained, and is characterized
• Clubbing. by encephalopathy and/or ascites. When chronic
• Flapping tremor. liver failure occurs, it is called decompensated
• Others include scratch marks, spider angi- liver disease.
oma, xanthoma, pigmentation, jaundice and
cyanosis. Q: What is cirrhosis of liver? What are the types?
2. Face: A: Cirrhosis of liver is a chronic diffuse liver disease
• Parotid enlargement (bilateral). characterized by destruction of liver cells with
• Xanthelasma. fibrosis, distortion of normal liver architecture and
• Spider angioma (also in neck, arm, forearm, nodular regeneration due to proliferation of surviv-
hand and any part above the nipple line). ing hepatocytes. The three types of cirrhosis are:
• Pigmentation. • Micronodular: Regenerative nodule is usually
• Hepatic facies. small of l-rnm size, involving every lobule,
• Jaundice. also called Laennec cirrhosis, and is common in
• Cyanosis. alcoholics.
3. Chest: • Macronodular: Large nodules, common in post-
• Gynaecomastia (also spider angioma). necrotic cirrhosis, found in HBY.
• Less hair in chest or body, scanty axillary • Rarely, mixed type (micro nodular and
(also pubic hair). macro nodular ).
4· ABDOMEN __

Q: What are the signs of portal hypertension? Q: What are the complications of cirrhosis of liver?
A: As follows: Ai As follows:
• Splenomegaly (single definite sign, mild in • Portal hypertension with rupture of oesophageal
adult but marked splenomegaly in childhood varices (haematemesis and melaena).
and adolescent). Hypersplenism is common • Portosystemic encephalopathy (hepatic precoma)
(pancytopaenia). and hepatic coma.
• Ascites. • Hepatorenal syndrome. 6:
0..
• Collateral circulation, oesophageal varices, haem- • Hepatopulmonary syndrome. o
8
orrhoid and venous hum (between xiphisternum • Hepatoma. g
and umbilicus called Cruveilhier-Baumgarten • Spontaneous bacterial peritonitis (SBP).
syndrome).
• Portosysternic encephalopathy. Q: What is the mechanism of ascites in cirrhosis of
• Endoscopy shows oesophageal varices. liver?
A: it is mainly due to renal reabsorption of sodium
N.B. Remember the following points: and water. Multiple factors are involved, such as:
• Normal portal pressure, 2-5 mmHg. If >12 1. Splanchnic vasodilatation is the main factor
mmHg, then it is a sign that symptoms and for ascites in cirrhosis ofliver. This is mediated
complications might develop. by vasodilators (mainly nitric oxide) that are
• Portal vein is formed by fusion of superior released when the portal hypertension causes
mesenteric vein and splenic vein. shunting of blood into the systemic circulation.
• Tn adults, cirrhosis of liver is the common Due to splanchnic vasodilatation, systemic arte-
cause of portal hypertension. rial pressure falls, which leads to activation of
• In children, extrahepatic portal vein renin-angiotensin mechanism with secondary
obstruction is the common cause. aldosteronism, increased sympathetic activity,
increased atrial natriuretic peptide secretion and
Q: What are the causes of cirrhosis of liver!
altered activity of kallikrein-kinin system. All
A: As follows:
these produce salt and water retention.
• Chronic viral hepatitis (B or C. or 8 and 0).
2. Other factors are:
• Chronic alcoholism.
• Nonalcoholic fatty liver disease (NAFLD). • Combination of splanchnic vasodilatation
• Immunological (autoimmune liver disease and and portal hypertension alters intestinal cap-
primary sclerosing cholangitis). illary permeability, leading to accumulation
• Biliary [primary biliary cirrhosis (PBC), secondary of fluid in peritoneal cavity.
biliary cirrhosis and cystic fibrosis]. • Portal hypertension also increases local
• Genetic (haemochromatosis, Wilson disease and hydrostatic pressure and causes increased
ai-antitrypsin deficiency). hepatic lymph production, which accumu-
• Budd-Chiari syndrome. lates into the peritoneal cavity.
• Drugs: Methotrexate. • Low albumin causes low plasma osmotic
• Idiopathic or cryptogenic. pressure, which causes extravasation of fluid.
• Failure to metabolize vasopressin by the liver
Q: What are the commonest causes of cirrhosis of that causes further retention of fluid.
liver?
A: Viral infection (8 and C), alcohol, NAFLD.In young Q: How to treat ascites in cirrhosis of liver?
patient with cirrhosis, Wilson disease may be an A: As follows:
important cause. 1. Bed rest. It improves renal flow (in horizontal
Q: What are the signs of decompensated cirrhosis? position) and increases diuresis.
A: As follows: 2. Sodium and water restriction:
• Ascites. • Sodium 88 rnmol/day (no added salt), in
• Increasing jaundice. severe case 40 mmol/day.
• Hepatic encephalopathy. • Water 0.5-1 L/day (fluid restncnon is not
• Portal hypertension with variceal bleeding. necessary, unless sodium is <l2S rnrnol/L).
• Worsening liver function (prolonged PT and low • Avoid salt-containing and salt-retaining diets
albumin). and drugs (NSAlDs, steroid and antacid).
~ SHORT CASES IN CLINICAL MEDICINE

3. Monitor weight, abdominal girth and urinary It can relieve resistant ascites but does not
output daily. Weight loss should be 0.5-1 kg/day prolong life. Following its use, frequency of
(fluid loss should not be more than 1 L daily). paracentesis and diuretic dose is reduced. It
4. Measure serum electrolyte and creatinine. can be done provided reasonable liver func-
5. Few patients will respond to the above therapy. tion without encephalopathy and minimal
6. If no response in 4 days with above therapy: disturbance of renal function (TIPSS is inef-
Diuretic should be given. Aldosterone antago- fective with intrinsic renal disease). Hepatic
nist such as spironolactone 100-400 rug/day is encephalopathy may occur following TIPSS.
given. If no response, either frusemide 40 mg This can be managed by reducing the shunt
daily is added. If no response with spironolac- diameter.
tone 400 mg plus frusemide 160 mg daily, it is • Portosystemic shunt surgery: Portocaval or
considered as refractory ascites. Prolonged use splenorenal shunt. Rarely done now-a-days.
of spironolactone can cause painful gynaeco- May cause encephalopathy.
mastia and hyperkalaemia. Eplerenone 25 mg 10. Liver transplantation may be considered, if all
once daily may be a suitable alternative (does measures fail.
not cause gynaecomastia).
Q: Will you prefer paracentesis or TIPSS in refractory
7. If no response or refractory ascites: ascites?
• Ensure that patient is not taking any salt or A: TIPSS is preferable than repeated paracentesis, as
salt-containing diet or drugs. large volume of fluid and protein are lost in para-
• If serum protein (mainly albumin) is low, centesis. Moreover, TIPSS can improve survival also.
diuretics may not respond. Then IV salt-poor
albumin followed by IV frusemide may be Q: If patient of CLD with ascites complains of fever
given. Occasionally IV dextran may be tried. and abdominal pain, what is the likely diagnosis?
S. Paracentesis: A: Spontaneous bacterial peritonitis (SBP) (other causes
• It is indicated if there is huge ascites with cardi- of fever in CLD with ascites are secondary infection,
orespiratory embarrassment or resistant ascites. hepatoma and excess pyrogen accumulation).
• 3-5 L of fluid can be removed. It is safe Q: What are the complications of ascites in CLD?
provided circulation is maintained (no fear of A: SBP and hepatorenal syndrome.
hepatic encephalopathy, thought previously).
Paracentesis is followed by IV albumin (6-S Q: If patient with CLD deteriorates, what are the
gm/L of ascetic fluid removed; usually 100 rnl possibilities?
of 20% human albumin solution for every 3 A: As follows:
L of ascitic fluid drained). Another plasma • Spontaneous bacterial peritonitis.
expander such as dextran (S gIL of ascetic • Hepatocellular carcinoma.
fluid removed) or haemaccel (125 mL/L of
Q: What are the bad prognostic signs of CLD?
ascitic fluid removed) may be used.
A: As follows:
9. Other modes of treatment (in resistant ascites):
1. Clinical:
• LeVeen shunt (peritoneovenous]: A catheter
• Increasing jaundice.
is used with one-way valve to communicate
• Ascites.
between peritoneal cavity and internal jugular
• Portosystemic encephalopathy.
vein, which allows ascitic fluid to pass directly
2. Laboratory:
into the systemic circulation (rarely done now-
• Increasing bilirubin.
a-days). Its main complications are infection,
• Sodium <120 rnmol/L (not due to diuretic).
SVC thrombosis, pulmonary oedema, bleed-
• Falling plasma albumin <30 gIL.
ing from oesophageal varices and DIC.
• Prothrombin time: Prolonged.
• TIPSS (transjugular intrahepatic portosys-
temic stent shunt): A stent is placed between Q: What investigations should be done in this case?
portal vein and hepatic vein in liver to A: As follows:
make portosystemic shunt, which is carried 1. LFT(total protein and A: G ratio, and prothrombin
out under radiological control via inter- time are the two most important tests for CLD.
nal jugular vein. It is mainly used to reduce Others are serum bilirubin, SGPT and alkaline
portal pressure and also variceal bleeding. phosphatase ).
4. ABDOMEN_

2. USG of whole abdomen (in cirrhosis, liver may mesenteric lymphatics. Most organisms are of
be small, shrunken, coarse and of high echo- enteric origin, mainly Escherichia coli, Klebsiella,
genic texture, with splenomegaly, ascites and Haemophilus, Enterococcus, other enteric Gram-
dilated portal vein). negative organisms, rarely pneumococcus and
3. Viral markers for HBV (HBsAg, HBeAg, anti- streptococcus. Anaerobic bacteria are not usually
HBc) and for HCV (anti-HCV). associated with SBP.
4. Proctoscopy (to test for haemorrhoid).
Clinical features: In a patient with cirrhosis and 5:
0-
5. Endoscopy (to test for oesophageal varices). o
ascites, may present with sudden abdominal pain, 8
6. cr scan of hepatobiliary system may be g
rebound tenderness, absent bowel sounds and fever.
performed.
There may be increasing ascites, not responding to
7. Liver biopsy under USG control (confirmatory).
diuretic. Abdominal signs may be mild or absent in
8. If CLD is due to other cause investigation should
one-third patients. In these patients, hepatic enceph-
be done accordingly, such as:
alopathy and fever are the main features.
• If haemochromatosis is suspected: Serum
iron, TlBC and ferritin. Ascitic fluid in SBPshows the following:
• If PBC is suspected: Antimitocbondrial • Fluid looks doudy (exudative is with high protein
antibody and other autoantibodies. and low sugar).
• In younger age, if Wilson disease is suspected: • Neutrophil counts of fluid >250/mm3.
Serum copper, ceruloplasmin and urinary • Ascetic culture in blood culture bottles gives the
copper. highest yield of organisms. CIS may be negative.
• If (XI-antitrypsin deficiency: Serum
Treatment of SBP: Pending the result of culture, if
(Xl-antitrypsin (which is sometimes associ-
neutrophil in ascitic fluid is high, treatment should
ated with liver disease and also pulmonary
be started immediately as follows:
emphysema, particularly in smokers).
9. Test for ascitic fluid (cytology, biochemistry, • Broad-spectrum antibiotics: Cefotaxirne 2 g 8-12
SMG). hourly for at least 5 days.
10. Other routine investigations: • Ceftriaxone plus amoxidav is an alternative.
• CBC, ESR. • Intravenous albumin may reduce mortality.
~ Serum urea and creatinine, electrolytes, blood Prognosis and prevention of SBP: Mortality is
sugar. 10-15%. SBPis an indication for referral to liver trans-
Q: What investigations are done to see the severity of plantation centre. Recurrence is common in 70% case
liver disease? within 1 year. This can be prevented by norfloxacin
A: As follows: 400 mg daily or ciprofloxacin 500 mg once or twice
• Serum albumin: If <28 gm/L, outlook is poor. daily or cotrimoxazole (1 double-strength tablet, 5
• Prothrombin time: Prolong VI' indicates severe days/week). In any patient with acute variceal bleed-
liver disease. ing, risk of bacterial peritonitis may be reduced by
• Serum electrolytes: Low sodium indicates severe giving injection ceftriaxone 1 g daily or oral norfloxa-
liver disease due to a defect in free water clearance. cin. Also, patient with high-risk group in cirrhosis (low
• Serum creatinine: High creatinine> 130 umol/L albumin, increased PI, low ascitic fluid albumin), nor-
is a marker of worse prognosis. tloxacin may be used to prevent bacterial peritonitis.

Q: What is spontaneous bacterial peritonitis (SBP)? N.B. Occasionally, there may be secondary bacterial
A: It means infection of ascitic fluid in a patient with infection other than SBP.To differentiate from
cirrhosis of liver in the absence of any apparently SBP, the following features in ascitic fluid may
primary source of infection. SBPmay develop in 8% suggest secondary bacterial infection:
cases (may be as high as 20-30%). It is usually by
• Neutrophil >10,000 (very high).
single organism (monomicrobial). Source of infec-
• Total protein: Very high (>1 g/dl, is against
tion cannot be determined usually (so it is named
SBP, highly suggestive of secondary
as spontaneous). It is suspected in any patient with
bacterial infection).
ascites who clinically deteriorates.
• Glucose <50 mg%.
Causes of SBP: The infective organism gain access • Ascitic LDH > serum LDH.
to the peritoneum by haernatogenous route or • Presence of multiple organisms in culture.
~ SHORT CASES IN CLINICAL MEDICINE

Q: What is hepatorenal syndrome? overproduction); and arterial gas analysis shows


A: If a patient with cirrhosis and ascites develops renal low oxygen saturation. HRCf may be helpful for
failure, it is called hepatorenal syndrome. It occurs detecting dilated pulmonary vessels.
in 10% cases and is of two types: Liver transplantation is indicated in hepatopul-
• Type 1 hepatorenal syndrome is characterized monary syndrome. Features improve after liver
by progressive oliguria and rapid rise in serum transplantation.
creatinine. There is no proteinuria; urine sodium Q: What is portopulmonary hypertension?
excretion is low; <10 rnrnol/day and urine/ A: It is defined as pulmonary hypertension and
plasma osmolality ratio is >1.5. Prognosis is poor; cirrhosis of the liver with portal hypertension.
without treatment median survival is <1 month. There is normal pulmonary artery wedge pressure.
• Type 2 hepatorenal syndrome occurs with refrac- It occurs in 1-2% cases of cirrhosis. It is caused by
tory ascites, characterized by moderate and stable vasoconstriction and obliteration of the pulmonary
increase of serum creatinine. Prognosis is better. arterial system due to circulating vasoconstrictors
Mechanism of hepatorenal syndrome: Initially particularly endothelin 1. This leads to breathless-
there is vasodilatation possibly due to nitric oxide, ness and fatigue. It may respond to medical therapy.
leading to hypotension. As a result, there is high Severe pulmonary hypertension is a contraindica-
plasma renin, aldosterone, norepinephrine and tion for liver transplantation.
vasopressin, leading to vasoconstriction of the renal
vasculature, causing increased preglomerular vascu- Read the Following Topics in
lar resistance. There is reduced glomerular filteration Relation to CLO
rate (GFR), and also sodium and water retention.
Other mediators like eicosanoids may also be Portosystemic Anastomosis
involved. (Between portal and systemic veins)
Renal failure is functional and tubular function is
Sites are:
intact. Hepatorenal syndrome may be precipitated
1. At the lower end of oesophagus, oesophageal trib-
by excess diuretic therapy, NSAIDs, diarrhoea or
utaries of left gastric vein (portal) communicate
paracentesis, and infection, especially spontaneous
with oesophageal tributaries of herniazygos veins
bacterial peritonitis.
(systemic).
Treatment: 2. At the lower end of rectum and anal canal, superior
• Diuretic should be stopped. rectal vein (portal) communicates with middle
• Hypovolaemia should be corrected, preferably and inferior rectal veins (systemic).
with albumin. 3. Anterior abdominal wall (around the umbilicus):
• Terlipressin or noradrenalin with intravenous • The paraumbilical vein (portal) communicates
albumin may be used. with systemic veins in epigastric, lateral thoracic,
• Liver transplantation is the best option. intercostal and lumbar veins.
• Paraumbilical vein (portal) communicates with
Q: What is hepatopulmonary syndrome? diaphragmatic veins (systemic) by a number of
A: Hepatopulrnonary syndrome is defined as hypox- small veins, called accessory portal system of
aernia occurring in patient with advanced liver Sappey.
disease. PaOl is <9.3 kPa or 70 mmHg. It is due 4. At bare area of liver, portal radicles of liver com-
to intrapulmonary vascular dilatation with no municates with diaphragmatic veins (systemic).
evidence of primary pulmonary disease. Hypoxia is 5. At retroperitoneal site, the splenic and colic veins
due to intrapulmonary AV communication. (portal) communicate with left renal veins and
The patient has features of cirrhosis with club- other tributaries of inferior vena cava (rYe) by
bing, spider angioma and cyanosis. Most patients small veins called veins of Retzius.
have no respiratory symptoms. But with more 6. At the fissure for ligamentum venosum, rarely, per-
severe disease, the patient is dyspnoeic on standing. sistent ductus venosus establishes direct portacaval
There is a characteristic reduction of arterial oxygen anastomosis (in foetal life, left branch of portal
saturation on standing. vein at the porta hepatis communicates with rvc
Transthoracic echo shows intrapulmonary via ductus venosus. After birth, ductus venosus is
shunting (which is probably due to nitric oxide fibrosed to form ligamentum venosum).
4· ABDOMEN_

Spider Angioma Venous Stars


It consists of central arteriole from which numerous These are 2-3-cm lesions that occur on dorsum of foot,
capillaries radiate, looks like spider legs. Size varies leg, back and lower chest. Caused by elevated venous
from pinhead t.o )-2 mm (sometimes 1-2 cm). These pressure and are usually found overlying the main
are found along the area of superior vena cava (SVC), tributary of large veins. Do not blanch on pressure and
commonly in neck, face, chest and dorsum of hand, and blood flow is from periphery to the centre of lesion
above the nipple lines, cause of which is not known. (opposite to spider angioma).
5:
0...
o
Blanches on pressure, may pulsate if large. Better seen S
g
with glass slide or pinhead. Palmar Erythema (Liver Palm)
Redness in thenar and hypothenar eminence, and pulp
offingers. Blanches on pressure. With glass slide, flushes
synchronously with pulse.

Spider angioma
Causes of spider angioma:
1. Physiological:
• Rarely present in normal people (2%), 1-2 in
number, common in children. If >2 in number,
Palmar erythema
it is usually pathological, especially in male than
female. Causes of palmar erythema:
• Pregnancy (usually in the third trimester, disap- 1. Physiological:
pears after 2 months of delivery). • Normal people; may be familial.
2. Pathological: • Pregnancy.
• CLD is commonly alcoholic cirrhosis (disap- 2. Pathological:
pears with improvement of liver function; • CLD (commonly alcoholic).
appearance of new spider indicates deteriora- • Thyrotoxicosis.
tion of liver function). • Polycythaernia.
• Viral hepatitis (transient). • Prolonged rheumatoid arthritis.
• Oestrogen therapy and oestrogen-containing • Chronic leukaemia.
oral contraceptive pill. • Febrile illness.
• Rarely, in rheumatoid arthritis, thyrotoxicosis.
Mechanism of palmar erythema in CLD:
Mechanism of spider angioma: • Hyperdynamic circulation.
• Due to hyperdynamic circulation. • Probably, high oestrogen (controversial).
• Excess oestrogen level (due to reduced metabolism
by the liver). Q: Why itching in liver disease?
A: Common in primary biliary cirrhosis and obstruc-
Differential diagnoses of spider angioma:
tive jaundice. Act.ualcause is unknown, probably due
• Purpura (spontaneous bleeding into skin and
to upregulation of opioid receptors and increased
mucous membrane, does not blanch on pressure
levels of endogenous opioids.
and there is progressive colour change).
• Hereditary haemorrhagic telangiectasia. Q: What are the vitamin K-dependent coagulation
• Campbell de Morgan spots. factors?
• Venous stars. A: Factors II,VII, IXand X (which are produced by liver).
_ SHORT CASES IN CLINICAL MEDICINE

Q: In which liver disease, vitamin Ktherapy is helpful? which is not deaminated by the diseased liver.
A: Usually in obstructive jaundice, as bile salt is neces- Methyl mercaptan is of intestinal origin (reduced
sary for absorption of vitamin K. Less or not helpful by defaecation or use of antibiotics). Presence of
in parenchymal liver disease, as vitamin K is not fetor hepaticus indicates severe hepatocellular fail-
used or less used by the diseased liver. ure with collateral circulation.

N.B. PT depends on factors I, H, V, VII and X. In Q: What are the causes of bad breath (halitosis)?
CLD, the PT is prolonged when these factors fall A: As follows:
below 30%. • Hepatic precoma (fetor hepaticus-like dead mouse).
• Diabetic ketoacidosis (sweetish due to ketone body).
Flapping Tremor (Asterixis) • Uraemia (fishy ammoniacal).
• Lung abscess with anaerobic infection (may be
It is characterized by irregular, flexion-extension move-
foetid).
ment of wrist and metacarpophalangeal (MCP) joints,
• Others are faulty oral hygiene and alcoholism.
abduction-adduction of fingers, produced by dorsi-
flexion of wrist and spreading of the fingers. It is called
Dupuytren Contracture
flapping because of resemblance to a bird flapping
its wings. It is demonstrated by asking the patient to It is characterized by thickening, fibrosis and shortening
stretch out arms in front, separate the fingers, dorsiflex- of superficial palmar fascia, causing flexion contracture
ion of wrist with fixed forearm by the examiner's hand. of fmgers. The ring and little fingers are commonly
affected and also other fingers are affected. Inability
If it is present, there is:
to extend the fingers fully is associated with puckering
• Jerky, irregular, flexion-extension of wrist, and
of the skin and presence of palpable nodules. Usually
MCP joint (looks like goodbye).
painless, bilateral, age related, five times common in
• Accompanied by lateral movement of fingers or male than in female, often familial with dominant
abduction-adduction of fingers.
inheritance. It may affect the sole of foot also. It is
Features of flapping tremor: slowly progressive and fasciotomy is seldom necessary.
• It is absent at rest, produced by intentional move-
ment, maximum at sustained posture.
• Usually bilateral, and not necessarily synchronous
on each side.
• Disappears during coma.
• Occasionally arms, face, neck, tongue, jaw and lids
are involved.
Causes of flapping tremor:
• Hepatic encephalopathy (the commonest cause).
• Severe cardiac failure.
• Respiratory failure.
• Renal failure.
Dupuytren contracture
• Others causes (rare): Cerebrovascular accident
(CVA) drug toxicity (phenytoin and barbiturate),
r

acute focal parietal or thalamic lesion (vascular),


and hypoglycaemia.
Mechanism of flapping tremor in CLD:
• It is due to impaired inflow of joint position sense
and other afferent informations to the brainstem
reticular formation, resulting in rhythmical lapse
of postural muscle tone.
Q: What is fetor hepaticus?
A: It is a bad smell (slightly faecal) of breath, like that
of dead mouse, due to methyl mercaptan, exhaled
in breath, derived from amino acid methionine, Diabetic cheiroarthropathy
4 • ABDOMEN ._

Causes are: • Familial (as autosomal dominant, associated with


Garrod patch on dorsum of hand).
• Cirrhosis of liver (commonly alcoholic).
• Diabetes mellitus (diabetic cheiroarthropathy,
• Alcoholism (itself, not by cirrhosis).
confused with systemic sclerosis).
• Prolonged antiepileptic drug (phenytoin).
• Peyronie disease.
• Manual worker (gardener) and chronic vibration
• Idiopathic (in many cases).
injury.
• Traumatic. N.B. It is confused with diabetic cheiroarthropathy.

Portosystemic Encephalopathy (PSE or Hepatic Precoma)


Definition: Portosystemic encephalopathy is a state of • P: Personality change (childish behaviour, abnor-
neuropsychiatric syndrome due to biochemical distur- mal behaviour, apathy, irritability).
bance of brain function caused by CLD. It may progress • I: Intellectual deterioration, from simple math-
from confusion to coma. Liver failure and portosystemic ematical calculation to organic mental function.
shunting are two important factors for PSE. It is revers- Earliest is constructional apraxia (see below).
ible, does not cause marked pathological change in • S: Speech disturbance (slow, slurred, monotonous
brain and may have cerebral oedema in advanced stage. and dysphasia) and sleep inversion (daytime
Mechanism of PSE: It is due to the nitrogenous sub- sleeping).
stances of gut origin that enters into the brain. Normally, • T: Tremor (flapping tremor).
these substances are metabolized by healthy liver. In • F: Foetor hepaticus (sweet musty odour in breath).
diseased liver, these are not metabolized and enters into Other features are convulsion, exaggerated reflex, exten-
the brain through portosysternic shunt. sor plantar response and clonus. Rarely, chronic hepatic
PSE can occur in portal hypertension with cirrhosis, encephalopathy may be associated with cerebelJar
also after surgery like portocaval or TIPS shunt. dysfunction, Parkinsonism, spastic paraplegia and
Nitrogenous substances are: dementia.
• Ammonia (produced by intestinal bacteria Q: What are the differential diagnoses of PSE?
breaking down protein). A: As follows:
• y-aminobutyric acid (GABA). • Intracranial bleeding (subdural, extradural
• Amino acid, mercaptan, fatty acids and octopamine, haematorna).
which act as false neurotransmitters. • Drug or alcohol intoxication.
Factors precipitating PSE (by increasing the availabil- • Delirium tremens (alcohol withdrawal).
ity of nitrogenous substance): • Hypoglycaemia.
• High dietary protein. • Wernicke encephalopathy.
• Gastrointestinal bleeding. • Primary psychiatric disorder.
• Constipation. • Neurological Wilson disease.
• Drugs (sedative, antidepressants and diuretics). • Postictal state.
• Infection including spontaneous bacterial Q: How to diagnose PSE in early stage?
peritonitis. A: By EEG, which shows diffuse slowing in frequency
• Fluid and electrolyte imbalance (hypokalaemia). of normal a-range (8-13 Hz) to S-range (1.5-3 Hz).
• Trauma. However, diagnosis is mostly clinical.
• Surgery (shunt surgery, TIPSS or other surgery).
Q: H.ow to treat PSE?
• Paracentesis (> 3-5 L).
A: As follows:
• Development of hepatocellular carcinoma.
• Possible precipitating factors should be identified
Clinical features of PSE (remember the mnemonic and avoided (drugs, constipation, electrolyte
DPISTF): imbalance, bleeding).
• D: Disturbance of consciousness (confusion, • No sedative and no diuretic. No protein restric-
disorientation, drowsiness, delirium, stupor and tion is recommended.
later coma). • Nutrition: Glucose (300-400 gfday) orally. If the
• Disorder of sleep (hypersomnia, inversion of sleep patient cannot take by mouth, then IVshould be
and more sleep in daytime). given.
_ SHORT CASES IN CLINICAL MEDICINE

• Lactulose: 15-30 mL 8 hourly (bowel should toxicity. Other causes are acute fatty liver in pregnancy,
move at least twice daily). If the patient is unable Wilson disease, following shock and rarely extensive
to take lactulose by mouth, it can be given per malignancy of liver.
rectally (300 mg lactulose in 700 ml saline
Clinical features are similar to PSE.
or sorbitol as retention enema). Lactitol is an
alternative to lactulose. Treatment is also similar to PSE. Liver transplanta-
• Low bowel wash (if no response to lactulose, tion may be considered.
then enema).
Q: What is constructional apraxia? How to test for it?
• Gut sterilizer: Metronidazole (200 mg 8 hourly)
A: Constructional apraxia means inability to perform
or neomycin 1-4 g, 4-6 hourly (it is ototoxic, less
a known act in the absence of any motor or sen-
or not used). Rifaxirnin 400 mg three times daily
sory disturbance. It is tested in the following way
orally is more preferable (not absorbed).
(patient will be unable to do):
• Correction of electrolyte imbalance (especially
• Ask the patient to draw a star.
hypokalaernia).
• Writing disturbance (unable to write or
• Control of infection by antibiotic. disturbance in writing).
• It is suggested that eradication of Helicobacter • Ask the patient to make triangle with three match
pylori, which is ammonia-producing may help sticks or ask to Iighten the cigarette by match
(not proved). stick.
• Chronic or refractory hepatic encephalopathy is • Reitan trail making test (it is the ability to join or
an indication for liver transplantation. connect the numbers with a pen in a certain fixed
• Other treatment: Zinc deficiency should be time). It becomes prolonged in PSE.
corrected, if present.
• If the patient is agitated: Oxazepam (10-30 mg) Normal By the patient
may be given by mouth.
• Other therapy includes flumazenil-a benzodi-
azepine receptor antagonist (may help in some
cases).

Q: What is the mode of action of lactulose?


A: Lactulose is a nonabsorbable disaccharide, reaches
the colon intact and is metabolized by colonic bac- Drawing star
teria to lactic acid. It acts in the following way:
• Osmotic laxative.
• Lactic acid reduces pH of colonic contents,
® ® @
inhibits ammonia-producing colonic bacteria @
and reduces ammonia absorption by converting
NH3-NH4, which is not absorbed.
@ @
• Promotes incorporation of nitrogen into @ @
bacteria.
® CD @
@
Q: What is fulminating hepatic failure?
A: It is a clinical syndrome of encephalopathy char- CD @ 0
acterized by confusion, stupor and coma, resulting Startfrom~
from sudden severe impairment of hepatic func-
tion, occurring within 8 weeks of onset in the
®
@0' CD
absence of pre-existing liver disease. It is also called
acute liver failure. It may be hyperacute «7 days), @ 0
acute (within 8-28 days) or subacute (29-days to 12
weeks) between the onset of jaundice and encepha- ~ Lastly 0 CD
lopathy.

The two commonest causes are viral hepatitis From 1 to 2S numbered circles can be joined within
(commonly B and E, and rarely A) and paracetamol 30 second normally; more time is required in PSE
4. ABDOMEN_

Hepatomegaly (Primary Biliary Cirrhosis)

Presentation of a Case

• The abdomen is distended, flanks are full and


skin is hyperpigmented.
• Umbilicus is everted.
6:
0-
o
• Superficial veins are visible with normal flow 8
Cb
(away from umbilicus). ::;l

• Liver is palpable, ... cm from right costal margin


in the rnidclavicular line, nontender, firm in
consistency, with smooth surface and sharp
margin. There is no hepatic bruit.
• Spleen is palpable, ... cm from the costal margin
in left anterior axillary line, towards the right Xanthoma in elbow'
iliac fossa.
• Fluid thrill and shifting dullness: Present.

My diagnosis is hepatosplenomegaly with ascites.

Q: What is the likely cause?


A: CLD with portal hypertension.

Q: What else do you like to see?


A.! Stigmata of CLD.

Q: In this middle-aged lady, what may be the cause?


A: May be due to HBV or HCV, or primary biliary Xanthoma in palm
cirrhosis.

Q: If it is PBe, what else do you like to see?


A: As follows:
• Jaundice.
• Xanthelasma.
• Xanthomatous deposit (in elbow, knee, buttock,
hand crease and tendo calcaneus).
• Pigmentation (may be generalized).
• Scratch mark of itching.

Xanthoma in buttock

Xanthelasma in PBC Xanthoma in tendo calcaneus


_ SHORT CASES IN CLINICAL MEDICINE

Q: What are the diseases associated with PBC? opioid antagonist (naltrexone) is used to control
A: PBC is an autoimmune disease; it may be associated itching in PBC.
with other autoimmune diseases such as:
Q: What investigations are done in PBC?
• Sjogren syndrome.
A: As follows:
• Thyroid disease (hypothyroidism should be
• LFr (alkaline phosphatase is very high; may be the
considered, especially with fatigue).
only finding. Aminotransferases may be elevated,
• Systemic sclerosis.
but not more than five times of the upper limit.
• Rheumatoid arthritis.
y-glutamyl trans peptidase (y-GT) is also high.
• Renal tubular acidosis.
Serum total protein and albumin is reduced. There
• Dermatomyositis.
is marked rise of 5-nucleotidases activity).
• Addison disease.
• USG (hepatomegaly with cirrhotic change, sple-
• Membranous glomerulonephritis.
nomegaly and ascites).
• Fibrosing alveolitis.
• Antimitochondrial antibody is positive in 95%
N.B. There is high incidence of coeliac disease in cases (M2 is specific). Other antibodies such as
PBC. It should be excluded when a patient antismooth muscle antibody (35%) and antinu-
presents with features of malabsorption. clear antibody (25%) may be present.
• Liver biopsy (There is infiltration of lymphocytes
Q: How the patient with PBC usually presents?
and plasma cells in portal tract, destruction
A: Common in females, middle-aged, of 40-60 years
of small bile duct with ductal proliferation,
of age:
piecemeal necrosis and cirrhosis. Granuloma
• Asymptomatic, isolated hepatomegaly on routine
may be present in 40% cases).
examination. High alkaline phosphatase in LFf.
• Endoscopic retrograde cholangiopancreato-
• Pruritus may be the early feature; it may precede
graphy (ERCP) or magnetic resonance cholangio-
jaundice by many months.
pancreatography (MRCP) (to rule out extrahepatic
• Jaundice (usually with pruritus).
biliary obstruction in doubtful cases).
• Abdominal pain or discomfort.
• Others: Serum cholesterol (high) and serum
• Features of malabsorption.
immunoglobulin (lgM) is very high.
• Fever, malaise, weakness, loss of weight.
• Others are hepatic osteodystrophy (characterized Q: What is antimitochondrial antibody (AMA)?
by bony pain or fracture due to osteomalacia or A: It is an antibody directed against mitochondrial
osteoporosis from malabsorption). pyruvate dehydrogenase complex (PDC) of mito-
Q: What is primary biliary cirrhosis? What are the chondrial enzymes. There are four types of antigens
causes! (M2, M4, M8 and M9) of which M2 is specific for
A: PBC is a chronic, progressive, cholestatic liver dis- PBC. Five M2-specific antigens have been further
ease characterized by granulomatous destruction of identified, of which E2 component of the PDC is
interlobular bile ducts, inflammatory damage with the major M2 autoantigen.
fibrosis spreading from portal tract to liver paren- AMA is presentin 95% casesofPBC, detectedby EliSA
chyma and eventual cirrhosis. (>1:160). It may be positive in autoimmune hepatitis
(20%). It is not related to severity and prognosis.
Causes: Actual cause is unknown. It is probably an
autoimmune disease occurring in a genetically pre- Q: What are the causes of granuloma in liver?
disposed person, triggered by environmental factors A: As follows:
like infections by retrovirus, bacteria including E. coli • PBC.
and mycobacteria. None are proved. • Tuberculosis.
Q: What is secondary biliary cirrhosis?
• Sarcoidosis.
A: When cirrhosis develops due to prolonged obstruc- • Brucellosis.
tion of the large biliary ducts. Causes are stone, • Parasitic (strongyloidiasis).
• Schistosomiasis.
strictures and sclerosing cholangitis.
• Drug (phenylbutazone).
Q: Why itching in PBC?
A: Actual cause of itching is unknown; but probably Q: What is the best indicator for prognosis?
it is due to upregulation of opioid receptors and A: Serum bilirubin (if> 100 mrnol/rnl, liver transplan-
increased level of endogenous opioids. This is why tation should be considered).
4. ABDOMEN_

Q: How to treat this patient? Steroid improves biochemical and histological


A: As follows: disease, but aggravates osteoporosis and
• Ursodeoxycholic acid 10-15 mg/kg (It improves other side effects, and should not be used.
bile flow, replaces toxic hydrophobic bile acid • Azathioprine, penicillamine and cydosporine
in the bile acid pool, reduces apoptosis of the have no beneficial role.
biliary epithelium. It also improves LFT, slows • In asymptomatic patient, follow-up should
down histological progress). be done. 6:
0-
• For pruritus, cholestyrarnine is given, 4-16 g/day o
orally, usually with orange juice. Main dose (8 g) Q: What are the indications of liver transplantation 3
g
is given before and after breakfast (as duodenal in PBC?
bile acid secretion is more). Alternatively, A: As follows:
rifampicin 300 mg/day or naltrexone (opioid • Advanced liver disease (increasing jaundice with
antagonist) 25 rug/day up to 300 mg/day may be serum bilirubin >100 umol/L).
given. Naloxone is an alternative. In intractable • Intractable pruritus.
itching, plasmapheresis or liver-support device
Q: How cholestyramine works?
[(molecular absorbent recirculating system
A: It is a chelating agent, acts by binding pruritogens in
(MARS)1 may be considered. Livertransplantation
intestine and increases excretion in stool. It is inef-
may be indicated in intractable pruritus.
fective in complete biliary obstruction.
• Vitamins A, D, K and calcium supplement,
alfacalcidol (1 mg/day orally). Q: What is the prognosis of PBO
• If osteoporosis: Bisphosphonate (such as risedro- A: As follows:
nate may be used). • Asymptomatic patients or patients presenting
• Liver transplantation should be considered in with pruritus: Survive for more than 20 years.
liver failure. 5-year survival is over 80%, but • Once symptoms like jaundice develop, average
recurrence of PBC after transplantation occurs in survival is around 5 years (may be 7-10 years).
one-third of cases at 10 years.
Q: What is the risk of hepatobiliary malignancy in
N.B. Remember the following points: PSO
T· Colchicine 0.6 mg orally twice daily and
methotrexate 15 mg/weekly may be helpful
A: It is increased in PBC. Risk factors for malignancy
are older age, male sex, prior blood transfusion, and
I in improving symptoms in some cases. signs of cirrhosis and portal hypertension.

Hepatomegaly (Haemochromatosis)
Q: What else do you like to see?
Presentation of a Case A: Stigmata of CLD. Also, pigmentation in other parts
of the body and arthritis.
• The abdomen is distended; flanks are full with
everted umbilicus. Q: If pigmentation and arthritis are present what is
• Skin is hyperpigmented. the likely diagnosis?
• Liver is palpable, ... cm from right costal margin A: Haemochromatosis,
in the midclavicular line, non tender, firm in Q: What are your differential diagnoses?
consistency, with smooth surface and sharp A: As follows:
margin. There is no hepatic bruit. • Decompensated cirrhosis of liver due to HVB or
• Spleen is palpable, ... em from the costal margin HCV infection or metabolic cause.
in left anterior axillary line, towards the right • Primary biliary cirrhosis.
iliac fossa.
• Fluid thrill and shifting dullness: Present. Q: IfiI)BC,what are the findings?
A: pse is more common in middle- aged female, with
longstanding generalized itching. Also, there may
My diagnosis is hepatosplenomegaly with ascites. be xanthelasma, xanthoma, etc.
Q: What is the likely cause? Q: What is the size 'Of [iver in cirrhosis?
A: CLD with portal hypertension. A: Small.
__ SHORT CASES IN CLINICAL MEDICINE

Q: Can the liver be enlarged in cirrhosis? N.B. Normal body iron is 3-4 g.ln haemochromato-
A: Liver may be enlarged if cirrhosis is due to haerno- sis, it may be 20-60 g. Mainly iron is deposited
chrornatosis, primary biliary cirrhosis and alcoholic in liver and pancreatic islets; also in endocrine
cirrhosis (early stage). glands (pituitary, adrenal), heart and skin.
Q: What investigations should be done j n
haemochromatosis? Q: Why is haemochromatosis less in female?
A: As follows: A: Females are protected by iron loss in menstruation
1. CBC. and pregnancy.
2. Liver function tests.
Q: What is the mechanism (pathogenesis) of primary
3. Iron profile: haemochromatosis?
• Serum iron (increased). A: In haemochrornatosis, mucosal absorption of iron
• Total iron-binding capacity (>70% is is more and inappropriate to the body needs. Ulti-
saturated). mately progressive accumulation of iron causes
• Serum ferritin (increased, >600 ~g/L). elevation of plasma iron, increase saturation of
4. CT scan or MRl of hepatobiliary system transferrin and high level of ferritin, which is depos-
(increased density ofliver due to iron deposits). ited in different organs of the body.
MRI has high specificity for iron overload, but
less sensitivity. Q: What are the common features of
S. Hepatic iron index (Hll): Shows quantification haemochroma tosis?
of liver iron, HII >1.9 suggests genetic haemo- A: Usually it occurs in male above 40 years. Common
chromatosis. features are:
6. Liver biopsy (iron deposition, hepatic fibrosis, • Liver involvement: Hepatomegaly, CLD (about
cirrhosis). Liverbiopsy to measure the iron stores 30% may develop HCC).
is a definitive test. • Skin pigmentation (leaden-grey skin pigmenta-
7. Others: Blood sugar, ECG, X-ray chest, echo car- tion due to melanin deposition).
diograrn. X-ray of the involved joint (shows • Diabetes mellitus.
chondrocalcinosis ). • Cardiac dysfunction (dilated cardiomyopathy,
Q: What are the causes of haemochromatosis?
CCE arrhythmia).
A: As follows: • Arthritis and chondrocalcinosis.
1. Primary: Hereditary disorder, inherited as auto- • Hypogonadism.
somal recessive.
2. Secondary: N.B. In a patient with CLD, if there are cardiac
• Haemolytic anaemia: disease, arthritis, skin pigmentation and dia-
o ~-Thalassaemia. betes, haemochromatosis is very likely.
o Sideroblastic anaemia.
o Chronic haemolytic anaemias. Q: What type of diabetes occurs in
• Exogenous iron overload: haemochromatosis?
o Repeated blood transfusion (transfusion A: This is called 'bronze diabetes' due to the bronze
siderosis) . colouration of the skin.
o Repeated iron injection. Q: What are the common causes of pigmentation?
o Prolonged oral iron. A: As follows:
• Chronic liver diseases: • Addison disease.
o Hepatitis C. • Haemochromatosis.
o Porphyria cutanea tarda. • Kala-azar.
o Alcoholic cirrhosis (in advanced stage). • Arsenicosis,
Q: What is primary haemochromatosis? • Drugs: Amiodarone, busulphan, bleomycin,
A: This is a hereditary disorder, inherited as autosomal phenothiazine, phenytoin, psoralen
recessive, characterized by excess deposition of iron • Systemic sclerosis.
in various organs, leading to fibrosis and functional • Alkaptonuria.
organ failure. It is associated with HLA-B3, B7 and • Nelson syndrome.
B14. It is more in male, less in female; but in post- • Chronic debilitating illness (malignancy,
menopausal case, it is also more in female. CLD, CRF).
4· ABDOMEN_

Q: What are the causes of raised ferritin level? • Alcohol must be avoided.
A: As follows: • Supplemental vitamin C must be avoided,
• Haemochromatosis. as pharmacological doses can accelerate iron
• Alcoholic liver disease. mobilization to a level that saturates circulating
• Hepatitis C infection. transferrin, resulting in an increase in pro-oxidant
• Nonalcoholic steatohepatitis. and free radical activity.
• As an acute phase reactant in inflammatory and • First-degree family members should be screened.
neoplastic conditions.
Q: How do you treat haemochromatosis? N.B. In primary haemochromatosis, iron is depos-
A: As follows: ited in hepatocytes; whereas in secondary iron
• Weekly or twice weekly venesection of 500 ml overload, iron accumulates in Kupffer cells.
blood (200-250 mg of iron) until the serum
ferritin is normal. It may take 2 years of more. The Q: What is the cause of death of haemochromatosis?
aim is to reduce ferritin to <50 Jlg/L. Thereafter, A: Death is usually due to cardiac failure (30%),
venesection is continued as required to keep the hepatocellular failure or portal hypertension (25%),
serum ferritin normal (usually 3-4 venesections/ and HCC (30%).
year is needed). Following venesection, most
Q: What is the risk of HCC in haemochromatosis?
of the symptoms improve or disappear, except
A: HCC usually occurs as late sequelae in patient who
testicular atrophy, diabetes mellitus and
is cirrhotic at presentation. It does not occur if the
chondrocalcinosis. Joint pain may even worsen.
disease is treated in the precirrhotic stage. In pres-
• Chelation therapy with desferrioxamine (40-
ence of cirrhosis, venesection reduces but does not
80 mg/kg/day subcutaneously) may be given.
abolish the risk of BCe.
It removes 10-20 mg of iron/day. It is mainly
used if the patient cannot tolerate venesection, Q: What is the prognosis?
especially those with cardiac disease or severe A: If it is diagnosed and treated in precirrhotic stage,
anaemia. life expectancy is normal. Even in cirrhotic patients,
• Symptomatic treatment of cirrhosis, diabetes there is a good prognosis compared to other causes
mellitus (usually by insulin), CCF and cardiac of cirrhosis. Three-fourth cases survive 5 years after
arrhythmia. the diagnosis.

:__ Ascites

Presentation of a Case

• There is generalized distension of abdomen,


flanks are full, umbilicus is everted.
• Shifting dullness is present.
• Fluid thrill is present (mention if any).

My diagnosis is ascites.

Q: What are the causes of abdominal distension?


A: Fluid, fat, flatus, faeces and foetus (five Fs). Others- Ascites
intra-abdominal mass.
Q: How to confirm ascites clinically?
Q: Why ascites? A: By eliciting shifting dullness; also fluid thrill (which
A: Shifting dullness (fluid thrill in tense ascites). is present in tense ascites).
_ SHORT CASES IN CLINICAL MEDICINE

Q: What relevants do you like to see? Q: What investigations are done in ascites?
A: As follows: A: As follows (according to suspicion of cause):
• Stigmata of CLD (page 202). 1. If CLD is suspected, LIT should be done (see
• Generalized swelling in face, leg, other parts of the CLO).
body (nephrotic syndrome, hypoproteinaemia). 2. CBC (high ESR in TB and leucocytosis in
• Neck vein, leg oedema and heart (CCF and pyogenic infection).
5 chronic constrictive pericarditis). 3. Chest X-ray (TB, cardiomegaly and chronic
so • Lymph node (lymphoma and disseminated TB). constrictive pericarditis).
"'0
4. USG of abdomen (to see liver, paraaortic lymph
~ Q: What are the causes of ascites in this case?
nodes, neoplasm and ovary in female).
A: Mention the causes of that patient in relation to age,
also sex: 5. Ascitic fluid tap for the following tests:
• Cirrhosis of liver with portal hypertension • Naked-eye examination (straw coloured,
(the commonest cause, in 80% cases). blood stained, serous and chylous).
• Intra-abdominal malignancy with peritoneal • Gram staining and CIS (in pyogenic
metastasis. infection ).
• Infection (tuberculous or pyogenic peritonitis). • Cytology (neutrophil >250 cells/rnm! or
• CCF. WBC >500 cell/rum" in SBP, high lymphocyte
• Others: Hepatoma with secondary in the peri- in tuberculous peritonitis).
toneum, hypoproteinaemia due to any cause, • Biochemistry for protein and sugar shows
Meigs syndrome (in female). high protein in exudative and low protein in
transudative. Simultaneous serum albumin
N.B. Common causes of ascites: to see serum ascitic albumin gradient (see
• Cirrhosis of liver with portal hypertension. below).
• Intra-abdominal malignancy with peritoneal • If tuberculous peritonitis is found, fluid for
metastasis (usually ovarian and GI). ADA,AFB and mycobacterial CIS, PCR.
• • CCF. • Exfoliative cytology (to see malignant cells).
6. Other tests (according to suspicion of causes):
Q: Whal is ascites? How much fluid is required to • For tuberculous peritonitis: MT.
detect ascites? • Urine for proteinuria and serum total protein
A: It is the pathological accumulation of free fluid in (nephrotic syndrome).
peritoneal cavity. Usually 2 L fluid is necessary to • Ascitic fluid amylase in acute pancreatitis
detect clinically (at least 1 L is necessary, even in (>1000 is highly suggestive).
thin person). Pleural effusion may be present in 7. CT scan or MRI (if any growth or mass
100/0cases with ascites, usually on the right side, suspected) .
mostly small, occasionally massive and unusual on
8. Laparoscopy and biopsy.
left side.

N.B. Normally, no or little fluid is present in peri-


toneal cavity. In a female, up to 20 m L may be
present, varies with menstruation.

Q: If the patient has cirrhosis and ascites, what does it


indicate?
A: Decompensated cirrhosis with portal hypertension
(a bad prognostic sign).

Q: What is the character of ascitic fluid in CLD?


A: Usually clear; may be straw or light-green coloured
and transudative.

Q: What is the single best investigation to detect ascites?


A: Ultrasonography.
4· ABDOMEN_

• Acute pancreatitis.
• Myxoedema.
• Chylous ascites.

Ascites may be exudative or transudative:


1. Transudative causes (protein <25 giL):
• Cirrhosis of liver with portal hypertension.
• Nephrotic syndrome.
• CCF.
• Meigs syndrome.
• Other causes of hypo prot einaerni a
(malnutrition and malabsorption).
2. Exudative causes (protein >25 giL):
• Peritonitis (TB and pyogenic).
Ascites • Malignancy.
• Collagen disease.
Serum-ascites albumin gradient (SAAG): It is the
• Myxoedema.
difference of albumin between serum and ascitic
fluid (calculated by serum albumin minus ascitic • Budd-Chiari syndrome.
albumin). This gradient correlates directly with • Others are acute pancreatitis and chylous ascites.
portal pressure. It is the single test to differentiate Causes of hepatomegaly with ascites:
ascites due to portal hypertension from nonportal
• BCC due to cirrhosis of liver lesion.
hypertension.
• Hepatoma with secondary in the peritoneum.
• If the gradient is >1.1 gfdL, it indicates CLD with
• Congestive cardiac failure or chronic constrictive
portal hypertension.
pericarditis.
• If <I, no portal hypertension (Ascites due to
nonportal hypertension will be noted. It is 97% • Lymphoma.
accurate.) .. Disseminated TB.
• Budd-Chiari syndrome.
N.B. Ascites protein <25 giL and SAAG>1.1 gjdL
• Decompensated cirrhosis of the liver due to
is usually suggestive of portal hypertension.
metastatic cause.
• Alcoholic hepatitis
Read the Following Points in
• Secondary in the liver with peritoneal
Relation to Ascites carcinomatosis.
Causes of ascites:
1. Liver diseases (cirrhosis of liver with portal hyper- Causes of splenomegaly with ascites:
tension, hepatoma with secondary in peritoneum, • Decompensated cirrhosis of liver with portal
Budd-Chiari syndrome). hypertension.
2. Abdominal causes (intra-abdominal malignancy • Collagen disease (SLE).
with peritoneal metastasis, such as carcinoma of • Lymphoma.
kidney, stomach, colon and ovary). • Leukaemia.
3. Peritoneal causes are peritonitis (TB or pyogenic) • Disseminated TB.
and secondaries in the peritoneum.
Causes of haernorrhagic ascites:
4. Cardiovascular (chronic constrictive pericarditis
• Traumatic.
and CCF).
5. Hypoproteinaemia (nephrotic syndrome, malnu- • Malignancy.
trition and malabsorption). • Ruptured ectopic pregnancy.
6. Others: • Acute haemorrhagic pancreatitis.
• Collagen disease (SLEand PAN). • Rupture of spleen.
• Lymphoma and leukaemia. • Any cause of bleeding disorder.
• Meigs syndrome (ovarian fibroma, ascites and • Excess anticoagulant.
right-sided pleural effusion). , Occasionally, tuberculous peritonitis.
_ SHORT CASES IN CLINICAL MEDICINE

Causes of straw-coloured ascites: Causes of chylous ascites (milky colour, high


• Tuberculous peritonitis. triglyceride and Sudan staining of ascitic fluid shows
• Occasionally, cirrhosis of liver. fat cells):
• Trauma.
Causes of cloudy ascites: • Filariasis.
• Pyogenic infection. • Tuberculosis.
5 • SBP. • Malignancy.
so
-0

~ Abdominal Mass
The mass or masses in abdomen described here exclude
Presentation of a Mass
liver, spleen and kidney.
Once a mass is palpable in the abdomen, ensure
in Anterior Abdominal Wall
whether it is intra-abdominal or extra-abdominal, • There is a mass in the right upper abdomen,
while the patient is in supine position. For this, ask the 4 x 5 ern, surface is smooth, margin is slightly
patient to keep the arms across the upper chest and raise irregular, firm in consistency, nontender and
the head upward up to halfway (rising test). Or, ask the fixed to the overlying skin.
patient to raise both the extended legs from the bed (leg
lifting test). Now look at the mass. The intra-abdominal
mass will either disappear or decrease in size; and the My differential diagnoses are (mention according to
extra-abdominal mass will be more prominent. your finding):
Once a mass is felt, the following points must be seen • Lipoma
very carefully: • Fibroma
• Neurofibroma.
• Site. Other causes of mass in the anterior abdominal wall:
• Size. • Sebaceous cyst.
• Surface (regular or irregular).
• Dermoid.
• Consistency.
• Malignant deposit.
• Tenderness.
• Melanoma.
• Margin. • Epigastric hernia.
• Mobility (fixed or mobile).
• Umbilical or paraumbilical hernia).
Present the case systematically. Examiner may ask, What • lncisional hernia.
are the differential diagnoses?' 'How to investigate?' • Rectus sheath divarication.
One must mention the possible differential diagnosis • Haematoma.
according to the site of mass and age of the patient. • Parietal abscess.

Causes of Mass in Different Sites of Abdomen


If the patient is middle-aged or elderly, the causes are:
Presentation of a Mass
1. Mass in left lobe ofliver (hepatoma, secondaries
in Epigastrium
and hydatid cyst).
• There is a mass in epigastric region (extending 2. Carcinoma of stomach.
in right or left hypochondrium). It is 6 x 7 em, 3. Lymphoma of stomach.
smooth (or irregular), nontender (or tender), 4. Carcinoma of head of the pancreas.
ill-defined margin (or round margin), firm
5. Others:
(or hard) in consistency and not freely movable.
• Due to tender mass (liver abscess).
Q: What are the causes of epigastric mass? • May be soft and cystic mass (pancreatic
A: As follows (mention according to the findings and pseudocyst).
also age of the patient): • Mass in transverse colon (carcinoma).
4. ABDOMEN_

• Pulsating mass (aneurysm of abdominal aorta). Ifthe patient is young or early-aged:


• Epigastric hernia. • Appendicular lump (tender).
If the patient is young, causes are: • Ileocaecal TB.
1. Lymphoma of stomach. • Crohn disease.
2. Mass in left lobe ofliver (hydatid cyst, hepatoma, • Lymphoma.
if tender mass- may be liver abscess). • Amoeboma (less common nowadays because of
3. Epigastric hernia.
wide use of metronidazole). 6:
0-
• Others are actinomycosis, Yersinia. infection, tubo- o
4. Pancreatic pseudocyst. :3
Q; What relevant do you like to see if there i~ epigastric
ovarian mass in female, pelvic kidney, tumour in
undescended testes, ectopic kidney.
s
mass?
If the patient is elderly or middle-aged:
A: As follows:
• Appendicular lump (tender).
• Virchow gland (due to secondaries from
stomach).
• Ileocaecal TB.
• Carcinoma of caecum (hard, irregular and
• Lymph nodes in other parts of the body
nontender).
(lymphoma).
• If hepatic mass is suspected: Look for jaundice, • Lymphoma.
evidence of CLD, primary source (e.g. testicular • Others may be as above.
mass). N.B. If there is scar mark in lumbar area, diagnosis
• Jaundice, scratch mark, pigmentation, etc. (car- is transplanted kidney.
cinoma head of the pancreas causing obstructive
Q: lIow to diagnose?
jaundice).
A: History of the patient, physical examination and
Q: Mention one investigation that will help the diag- investigation.
nosis.
Q: Mention one investigation that will help the diag-
A: USG.
nosis.
Q: What investigations should be done? A: USG.
A: As follows (mention according to the suspicion of
Q: Whitt investigations do you suggest?
causes):
A: According to suspicion of causes:
1. USG (It will give clue about mass-hepatic,
• Hb%, TG, DC and ESR (high in TB; leucocytosis
pancreatic, stomach or colon.)
2. Other investigations according to the findings in appendicular mass).
in ultrasonography: • USG of whole abdomen.
• Ifileocaecal TB (chest X-ray, MT).
• If gastric mass: Endoscopy and biopsy
(carcinoma of stomach and lymphoma). • CTor MRI.
• If hepatic mass: Investigate accordingly • FNAC (Cl-guided or USG-guided).
(hepatoma, secondaries and hydatid cyst). • Barium enema (double contrast).
• Barium meal and follow-through with spot film
• If pancreatic mass: Cf scan or MRI, ERCP.
in ileocaecal region (TB and Crohn disease).
• If colonic mass: Barium enema, colonoscopy
and biopsy. • Colonoscopy and biopsy.
• If still no diagnosis is possible, then laparoscopy
and biopsy.
Presentation of a Mass :t----------, • Occasionally, laparotomy may be required.
in Right Iliac Fossa
• There is a mass in right iliac fossa, 4 x 6 ern,
Presentation of a Mass :1---------,
smooth (or irregular), nontender (or tender), in Left Iliac Fossa
ill-defined margin (or round margin), firm (or
• There is a mass in left iliac fossa, 3 x 4 cm, smooth
hard) in consistency, and freely movable from
(or irregular), nontender (or tender), ill-defined
underlying structure and overlying skin.
margin (or round margin), soft in consistency,
and freely movable from underlying structure
Q: What are the causes of mass in right iliac fossa? and overlying skin.
A: Tell the causes according to the age of the patient:
SHORT CASES IN CLINICAL MEDICINE

Q: What are the causes of mass in left iliac fossa?


A: Causes are (mention according to the age of Causes of palpable gall bladder without
the patient): jaundice
If the patient is young (or also any age), the • Mucocoele.
causes are: • Empyema.
• Thick colon (in irritable bowel syndrome). • Occasionally, carcinoma of gall bladder.
• Faecal mass (mass indented and moulded Causes of palpable gall bladder with jaundice
by pressure).
• Occasionally, normal colon may be palpable. • Carcinoma of head of pancreas.
• Carcinoma of colon (rare). • Carcinoma of ampulla of Vater.
• Stone in common bile duct.
• Diverticulitis.
• Tubo-ovarian mass in female. • Pressure from outside on bile duct (lymphoma
and secondaries).
I
• Pelvic kidney.
• Cholangiocarcinoma.
If tile patient is elderly, the causes are: • Sclerosing cholangitis.
• Carcinoma of colon (descending or sigmoid colon).
• Diverticulitis (tender, mobile mass). Q: What investigations do you suggest?
• Faecal mass. A: As follows:
• USC of hepatobiliary system.
N.B. If laparotomy scar is present, the diagnosis is • LFT.
transplanted kidney. Examine for AV fistula • CTor MRJ.
and anaemia. • ERCP.
• Laparoscopy.
Q: What investigations do you suggest?
A: As follows: Q: Could it be gall stone with palpable gall bladder?
• USC of whole abdomen. A: Unlikely (but gall bladder is palpable, if the stone is
• Barium enema (double contrast). in common bile duct).
• Sigmoidoscopy and biopsy (if needed). Q: What is Courvoisier's law?
, Stool for occult blood. A: It is as follows:
• FNAC (Cl-guided or USC-guided). • In a jaundiced patient with palpable gall bladder,
• Laparoscopy. the cause is unlikely to be gall stones; rather it is
• Occasionally, laparotomy may be needed. due to carcinoma head of pancreas and extrinsic
pressure in bile duct.
Reverse of the law is:
Presentationof a Mass ll----------,
• Obstructive jaundice without palpable gall bladder
in Right Hypochondrium is unlikely to be carcinoma head of pancreas and
• There is a mass in right hypochondrium,S x 6 em, extrinsic pressure in common bile duct.
margin is smooth (or irregular), surface is irregular, Q: Why gall bladder is not palpable in gall stone
nontender, soft in consistency and freely movable disease?
from underlying structure and overlying skin. A: Call stone is associated with chronic cholecystitis and
gall bladder is fibrosed, which is unable to enlarge.

Q: What are the likely causes of mass in right hypo- Presentationof a Mass 11------,
chondrium?
in Left Hypochondrium
A: As follows:
• Mass in liver (hepatoma, secondaries and • There is a mass in left hypochondrium, 3 x 4 ern,
hydatid cyst). smooth (or irregular), nontender (or tender),
• Call bladder mass (carcinoma, mucocoele or ill-defined margin (or round margin), soft in
empyema of gall bladder). consistency, and freely movable from underlying
• Mass in right side of colon (malignancy). structure and overlying skin.
• Carcinoma head of pancreas.
4. ABDOMEN_

Causes of mass in left hypochondrium: • USC (investigation of choice).


• Splenomegaly. • HbOfo,TC, DC and ESR.
• Enlarged left kidney. • IfTB is suspected, MT, chest X-ray PA view.
• Carcinoma of the stomach. • CTor MRI.
• Carcinoma of the splenic flexure of colon. • FNAC (CT-guided or USC-guided).
• Carcinoma or any mass on tail of pancreas. • Barium enema (double-contrast) or barium meal
• Omental mass. and follow-through or small bowel enema (Ryle
tube is introduced through mouth and barium is
Q: What investigations do you suggest?
given).
A: As follows:
• USC of hepatobiliary system: This will show • Colonoscopy.
• Laparoscopy and biopsy (if needed) or laparot-
whether it is spleen or other mass. Then other
omy (in some cases).
investigation should be done according to
• Ifpalpable lymph node, FNACor biopsy is required.
suspicion of cause.
• If hydatid cyst suspected, Casoni test, haemagglu-
tination test.
Presentation of a Mass
in Central Abdomen
Presentation of a Mass
• There is a mass in central abdomen, 3 x 4 cm, in Lower Abdomen (Hypogastrium)
smooth (or irregular), nontender (or tender),
• There is a mass in lower abdomen, 8 x 8 ern.
ill-defined margin (or round margin), soft in
smooth (or irregular), non tender (or tender),
consistency, and freely movable from underlying
regular margin (or round margin), soft in
structure and overlying skin.
consistency, and freely movable from underlying
structure and overlying skin.
Causes of mass in central abdomen (according to the
age and sex):
Q: What are the causes of mass in lower abdomen?
If the patient is young or early-aged, the causes are:
A: As follows (mention according to your findings,
• Lymphoma. considering the age and sex of the patient):
• Tuberculosis (tabes mesentericus or • In female: Pregnancy in young, fibroid
lymphadenitis ). uterus, ovarian cyst or other ovarian mass
• Hydatid cyst. (e.g. carcinoma).
• Mesenteric cyst. • Retention of urine.
• In female, ovarian cyst and pregnancy. • Carcinoma of urinary bladder.
• Distended urinary bladder (urinary retention). • Intra-abdominal malignancy, lymphoma.
If the patient is elderly or middle-aged, the causes are:
Q: What investigations do you suggest?
• Intra-abdominal malignancy.
A: Investigation should be done according to the his-
• TB (tabes mesentericus or lymphadenitis).
tory and physical findings. Most important is USC.
• Lymphoma.
• Metastatic lymphadenitis.
I

• Hydatid cyst. Presentation of Renal r


• Mesenteric cyst. Mass (or Mass in Flank)
• Retroperitoneal growth (sarcoma).
• Distended urinary bladder (urinary retention). • There is a mass in left flank, 3 x 4 em, smooth
(or irregular), nontender (or tender), ill-defined
Q: Suggest one investigation that will help in the
margin (or round margin), soft in consistency,
diagnosis. and freely movable from underlying structure
A: USC of whole abdomen. and overlying skin.
Q: Suggest one investigationthat will confirm the
diagnosis. Q: What are the causes of unilateral renal mass?
A: CT-guided or USC-guided FNAC. A: As follows:
Q: What investigations do you suggest? • Renal cell carcinoma (middle-aged or elderly),
A: As follows (according to suspicion of cause): Wilms tumour (in children).
__ SHORT CASES IN CLINICAL MEDICINE

• Unilateral hydronephrosis OJ; pyonephrosis. Q: What are the causes of bilateral renal mass?
• Hypertrophied single kidney (if nephrectomy of A: As follows:
other kidney). • Polycystic kidney disease.
• Large renal cyst. • Bilateral hydronephrosis.
• Polycystic kidney with single palpable kidney • Diabetic nephropathy in early stage.
(due to asymmetrical enlargement). • Amyloidosis.
• Right kidney may be normally palpable. • Rarely, bilateral renal cell carcinoma.

!roO" - - - -- . Carcinoma of Stomach --

Instruction by the examiner: 8. Features of metastasis:


• Examine the abdomen. • Hepatomegaly.
• Palpate the abdomen. • Virchow gland.
• Hard nodule around umbilicus (Sister Mary
Joseph nodule).
Presentation of a Case :f----------,
• Ovarian involvement (Krukenberg tumour).
• There is a mass in epigastric region, 9 x 7 em, • Prerectal pouch: A shelf-like mass (Blumer
irregular, nontender, margin is ill-defined, firm shelf).
in consistency and not freely movable. 9. Paraneoplastic syndrome (acanthosis nigricans,
dermatomyositis, thrombophlebitis migrans).

My differential diagnoses are:


• Carcinoma of stomach.
• Hepatoma or secondaries in the liver.
• Lymphoma of stomach.
• Carcinoma of head of the pancreas.
• Carcinoma of transverse colon.
• .Pancreatic pseudocyst.
Q: What else do you like to see?
A: I want to palpate left supraclavicular gland (Virchow
gland, called Troisier sign). If carcinoma of the
stomach, there may be metastasis to left suprarenal
lymph node.
Q: Suggest one investigation.
A: Ultrasonography (this will show the nature of the
mass, whether from the stomach, liver, pancreas, etc.). VifChow gland (left)

Q: Mention one investigation to confirm carcinoma


Q: What are the types of carcinoma stomach?
stomach.
A: A" follows:
A: Endoscopy and biopsy.
1. Macroscopic observations show four types:
Q: What are the presentations of carcinoma of stomach? • Polypoid.
A: As follows: • Ulcerative.
1. Any patient above 40 years of age presenting • Fungating or cauliflower.
with 'three As' (Anaemia, Anorexia, Asthaenia). • Diffuse infiltrative (linitis plastica, rare).
2. Vomiting (if tumour in the pyloric end). 2. Microscopic observation shows two types:
3. Pain in the epigastrium. • Adenocarcinoma (95%) in turn has two
4. Dysphagia (if tumour in the cardiac end). types:
5. Haematernesis and melaena. • Intestinal, arising from areas of intestinal
6. Mass in the epigastrium. metaplasia (more common, with better
7. Only unexplained features of anaemia. prognosis)
4. ABDOMEN_

• Diffuse, arising from normal gastric mucosa Q: What is early gastric cancer?
(poorly differentiated, occurs in younger age A: It is defined as 'when carcinoma is confined to
and bad prognosis). mucosa or submucosa regardless of lymph node
• Others are squamous cell carcinoma, non- involvement'.
Hodgkin lymphoma and leiomyosarcoma.
Q: What is linitis plastica?
Q: What are the causes or predisposing factors for A: It is the diffuse submucosal infiltration by scir-
carcinoma stomach? rhous carcinoma. Stomach becomes like a rigid
A: Causes are unknown. Predisposing factors are: tube (other causes of linitis plastica are lymphoma,
sarcoidosis and secondary syphilis).
1. Diet:
• Preservatives in diet such as nitrites and Q: What investigations are done?
nitrates convert to N-nitroso compounds, A: As follows:

which are carcinogenic. Nitrate is convened • Hb%, TC, DC and ESR.


by nitrite-reducing bacteria, which colonize • Barium meal double contrast (filling defect,
in achlorhydric stomach. irregular ulcer, in infiltrating type and stomach
• Diet rich in salted, smoked or prickled food. looks like tube).
• Diet lacking fresh fruits, vegetables and • Endoscopy and biopsy.
vitamins C and A may be the contributing • USG of whole abdomen (to see any metastasis).
factors (diet with high amount of vegetables • Stool for occult blood test.
and fruits and with low salt protects • Gastric lavage for exfoliative cytology (previously
done when endoscopy was not available).
carcinoma stomach).
• To monitor recurrence, carcinoembryonic
2. Smoking.
antigen may be done.
3. Alcohol.
4. Gastric surgery (partial gastrectomy and gastro- Q: How to treat a case of carcinoma stomach?
jejunostomy. It is due to intestinal metaplasia A: As follows:

and chronic gastritis). 1. Surgery is the only curative treatment. 50-year


5. Infection by H. pylori causes chronic atrophic survival is 90% if surgery is done in early gastric
gastritis and intestinal metaplasia, which is cancer; but only 10% if done in advanced cases.
precancerous. This organism is responsible in 2. Peri operative chemotherapy: ECF (epirubici n,
60-70% cases, mostly associated with achlorhy- cisplatin and fluorouracil) has improved 5-year
dria. Chronic inflammation with generation of survival in operable gastric and lower oesophageal
reactive oxygen species and depletion of antioxi- adenocarcinoma.
dant ascorbic acid are also important. 3. Chemotherapy-not much helpful. FAM(combina-
tion of 5-Fluorouracil + adriamycin + mitomycin C)
6. Others include pernicious anaemia, adeno-
may be tried.
matous gastric polyp, familial adenomatous
4. Palliative:
polyposis, Menetrier disease, blood group A and
• Radiotherapy: Very-little role.
first-degree relatives.
• Endoscopic laser ablation of tumour tissue, if
7. Rarely, gastric cancer families, in which dif-
surgery is not possible (palliative therapy).
fuse gastric cancers occur in association with
• Endoscopic dilatation or insertion of expand-
mutations of E-cadherin gene. It is inherited as
able metallic stents may be used for relief of
autosomal dominant trait.
dysphagia or vomiting.
Q: What are the sites of carcinoma stomach?
A: As follows:
Gastric Lymphoma
• Antrum: 50%. It is the second commonest neoplasm of stomach.
• Body of stomach (greater curvature): 20-30%. Among the GIT lymphoma, 60% occurs in the stom-
ach. 95% is low-grade non-Hodgkin B-ceU type. Gastric
• Cardiac end of stomach: 20%.
lymphoma may be:
In Western countries, proximal tumours are becom-
• Primary: Arise from mucosa-associated lymphoid
ing more common.
tissue (MALT).
Q: What is the benign tumour of the stomach? • Secondary to lymph node involvement in other
A: Leiomyoma. parts of the body.
_ SHORT CASES IN CLINICAL MEDICINE

Primary gastric lymphoma may be due to H. pylori infec- Treatment:


tion. 85% are low grade and 40% are high grade, when
• Primary type: Treatment with antihelicobacter
associated with H. pylori infection. Chronic antigenic
therapy may regress the tumour. If no response,
stimulation results in monocionallymphoproliferation
other therapy for lymphoma should be given
that may cause low-grade MALTlymphoma.
(radiotherapy or chemotherapy).
Symptoms: Similar to that of gastric cancer. Patients • Secondary type: Usual therapy for lymphoma
with primary gastric lymphoma have stomach pain, (chemotherapy, radiotherapy).
ulcers or other localized symptoms; but systemic com-
Prognosis: Varies according to type. Features predicting
plaints such as fatigue or fever are rare.
a favourable prognosis are stage I or II disease, small
Diagnosis: Byendoscopy and biopsy. At endoscopy, the resectable tumour or tumour with low-grade histology,
tumour usually appears as a polypoid or ulcerating mass. and age below 60 years.

Carcinoma of Head of Pancreas (Mass in the Epigastrium)


Instruction by the examiner: • Lymphoma of stomach.
• Examine the abdomen. • Carcinoma of head of the pancreas.
• Palpate the abdomen. • Carcinoma of transverse colon.
• Pancreatic pseudocyst.
Presentation of a Case Q: What relevant do you like to see jf it is carcinoma of
the head of the pancreas?
• There is a mass in epigastric region, 7 x 7 em,
A: Jaundice, scratch marks for itching; also the patient
irregular, non tender, margin is ill-defined, firm
is emaciated, may be pigmented also.
in consistency and not freely movable (the
patient is also extremely emaciated). Q: Suggest one investigation in this case.
A: USC of upper abdomen.

Q: What other investigations do you suggest?


A: As follows:
1. Liver function test (bilirubin, SCPT, alkaline
phosphatase, prothrombin time, etc. Alkaline
phosphatase is usually very high).
2. cr scan of abdomen.
3. MRJ scan and endoscopic ultrasound are help-
ful in some cases.
4. cr or ultrasound-guided FNAC: May be done;
but not in potentially operable case. It may
spread the malignancy to the peritoneum.
5. Tumour markers especially CA 19-9 is highly
sensitive (80%); but false-positive result may be
found.
6. Others:
• Barium meal X-ray with C-Ioop (it will show
widening of the C-Ioop).
• Blood glucose.
• ERCP (to see obstruction, irregularity or
distortion of pancreatic duct. Also, helpfui to
Carcinoma head of pancreas
insert stent in obstructive jaundice.).
My differential diagnoses are: • MRCP may be done.
• Carcinoma of stomach. • Sometimes laparoscopy and laparoscopic
• Hepatoma or secondaries in the liver. USC (in very small lesion).
4· ABDOMEN __

seated, dull aching, radiates to the back, more on


lying flat, feels better with bending forward (pain
is due to involvement of coeliac plexus).
• Loss of weight, anorexia, nausea.
• Mass in upper abdomen (in 20% cases).
• Others: Diabetes mellitus, acute pancreatitis.
• Rare features are: Thrombophlebitis migrans
5:
a..
o
(arm vein is more involved than leg vein), a
(1)

venous thrombosis, portal hypertension (due p

to splenic vein thrombosis) and marantic


endocarditis.

Carcinoma head of pancreas N.B. Courvoisier's law: In a jaundiced patient with


palpable gall bladder, the cause is unlikely to
be gall stones; rather it is due to carcinoma
Q: What are the causes of carcinoma head of pancreas?
head of pancreas and extrinsic pressure in bile
A: Actual causes are unknown. Some factors are
duct. (Reverse of the law: Obstructive jaundice
responsible:
without palpable gall bladder is unlikely to be
• Age, above 70 years.
due to carcinoma head of pancreas and extrin-
• Male, predominant (twice more than female).
sic pressure in common bile duct.)
• Chronic pancreatitis.
• Alcohol.
• Smoking. Q: How to treat carcinoma head of pancreas?
• Environmental factors, such as petroleum A: As follows:
product and naphthylamine. 1. In early stage, surgical resection (Whipple oper-
• Genetic in 5-10% cases. There may be hereditary ation is performed: In this operation, pancreas,
pancreatins, multiple endocrine neoplasia duodenum, draining lymph node and part of
(MEN) and hereditary nonpolyposis colon mesentery are removed). 5-year survival is 20%
cancer (HNPCC). after surgery. Survival is improvement with
adjuvant chemotherapy (5-FU).
Q: What are the types of carcinoma of pancreas and
the sites! 2. Other treatment (usually palliative):
A: Usually adenocarcinoma (90%), which arises • Endoscopic insertion of stent to relieve
from the epithelium of pancreatic duct. Sites are as intractable itching.
follows: • For pain, analgesic, injection of alcohol in
• 60% in head. celiac plexus (USG-guided or endoscopic-
• 25% in body. USG-guided).
• 15% in tail. • Chemotherapy: 5-FU, adriamycin and cispla-
tin may be tried. Combination of 5-FU plus
Q: What are the other tumours of pancreas?
gemcitabine may help to improve the sur-
A: Some tumours may arise from islets cell
vival in advanced disease.
(insulinoma, gastrinoma, glucagonoma, sornato-
statinoma, VIPoma, etc.). May occur as a part of • Radiotherapy is not much helpful.
MEN J (parathyroid hyperplasia or adenoma, pan-
Q: What is the prognosis?
creatic tumour, pituitary adenoma).
A: Prognosis is bad; mean survival is <6 months.
Q: What are the presentations of carcinoma of the Usual 5-year survival is 2-5%. Following Whipple
pancreas? operation, 5-year survival is 5-14%. If adjuvant
A: As follows: chemotherapy is given with 5-fluorouracil, then
• Painless obstructive jaundice, with palpable gall 5-year survival becomes 21-29%. Prognosis is better
bladder: In case of carcinoma of head of pancreas. if tumour size <3 ern, no lymph node involvement,
• Carcinoma involving the body and tail: Usually negative resection of margin at surgery, ampullary
presents with pain in the epigastrium, deep or islet cell tumours.
__ SHORT CASES IN CLINICAL MEDICINE

Pancreatic Pseudocyst
Instruction by the examiner: Q: What investigations do you suggest?
• Examine the abdomen. A: As follows:
• Palpate the abdomen. • Ultrasonography.
• cror MRI.
• Serum amylase (persistently high).
Presentation of a Case • Others (LFTs, blood sugar, urea, creatinine and
serum electrolytes).
• There is a mass in epigastric region, 7 x 9 cm,
irregular, nontender, margin is ill-defined, cystic
in consistency and not freely movable.

Pancreatic pseudocyst

Pseudocyst of pancreas Causes of pancreatic pseudocyst:


• Acute pancreatitis, usually 1-2 weeks after acute
My differential diagnoses are: attack.
• Carcinoma of stomach. • Chronic pancreatitis.
• Hepatoma or secondaries in the liver. • Trauma.
• Lymphoma of stomach. Treatment of pancreatic pseudocyst:
• Carcinoma of head of the pancreas. • Small cyst,. <6 ern, no specific treatment and
• Carcinoma of transverse colon. spontaneous resolution may occur.
• Pancreatic pseudocyst. • For large cyst, conservative treatment, spontaneous
resolution in 4-6 weeks.
Q: What are the presentations of pancreatic
pseudocyst? • If no response or cyst >6 ern, or rapidly enlarging
cyst or features of obstruction of bile duct or
A: As follows:
duodenum, surgery should be done. Cyst is
• History of acute pancreatitis, trauma to abdomen.
drained to stomach, duodenum or jejunum, after
• Asymptomatic, if small cyst.
6 weeks, once pseudocyst is matured.
• Anorexia, nausea, vomiting, pain abdomen,
• Aspiration under USC may be done.
weight loss and mass in the epigastrium.

Q: What is pancreatic pseudocyst? Why it is called Q: What are the complications of pancreatic
pseudocyst? pseudocyst?
A: It is the localized peri pancreatic collection of A: As follows:
pancreatic juice and debris, surrounded by gran- • Rupture of the cyst.
ulation tissue, which usually develops in lesser • Secondary infection and abscess formation.
sac following inflammatory rupture of pancreatic • Compression of surrounding structures, obstruc-
duct. It is called pseudocyst because there is no tion of bile duct or duodenum or blood vessels
lining epithelium of cyst. causing pseudoaneurysm.
4· ABDOMEN_

Mass in the Epigastrium (Aneurysm of Aorta)


Instruction by the examiner: Q: What is aneurysm of signs and aneurysm of
• Examine the abdomen. symptoms?
• Palpate the abdomen. A: As follows:
• Aneurysm of signs: When aneurysm involves
ascending aorta, there are signs of aortic regur-
Presentation of a Case :~--------, gitation and pulsation in right side of sternum.
• Aneurysm of symptoms: When aneurysm
• There is a pulsatile mass in epigastric region, involves aortic arch and descending aorta, there
3 x 4 em, nontender, margin is well defined, soft are symptoms due to pressure (dysphagia,
in consistency. hoarseness, stridor and breathlessness).
Q: What are the complications of aneurysm?
My diagnosis is aneurysm of abdominal aorta. A: As follows:
• Rupture.
Q: What are the causes of epigastric pulsation'? • Thrombosis.
A: As follows: • Embolism,
• Normally, in lean and thin person (palpable, but • Pressure on nearest structure.
nonexpansile] , Q: Whal investigations do you suggest?
• Right ventricular hypertrophy. A: As follows:
• Pulsatile liver (in tricuspid regurgitation}. • USG of abdomen (investigation of choice).
• Mass overlying aorta (carcinoma of stomach). • Plain X-ray of abdomen (curvilinear calcification
To differentiate from aneurysm, place two fmgers may be present).
parallel at the outermost palpable margin. If aneu- • CTscan.
rysm is present, pulsation is expansile (see the fingers • Aortography.
that go apart from each other and measure the • Others to find out risk factors (blood sugar, lipid
distance). profile, VDRL and TPHA).
Q: How to treat aneurysm?
Q: What are the causes of aneurysm? A: As follows:
A: As follows: • Symptomatic: Surgery should be done.
• Atherosclerosis (the commonest cause: 90%, • Asymptomatic, but aneurysm >5.5 cm: Surgery
commonly abdominal aorta below the origin of (because of risk of rupture).
renal artery). • Small aneurysm: Follow-up (by serial USG).
• Aortitis due to syphilis (rare now-a-days), Q: What is dissecting aneurysm] What are the types?
Takayasu disease, giant cell arteritis, Reiter A: It means when there is a tear in intima of aorta,
syndrome and ankylosing spondylitis, exposing diseased media to blood and creating a
• Mycotic aneurysm (by Staphylococcus and by false lumen. It is of two types:
Streptococcus in atheromat.ous plaque).
• Type A: When involves ascending aorta; may
• Cystic medial necrosis (in Marfan syndrome, extend to involve descending aorta.
Ehlers-Danlos syndrome),
• Type B: When involves descending aorta, below
• Collagen vascular disease. the left subdavian artery.
• Trauma.
Causes of dissecting aneurysm:
Q: What are the presentations of abdomina I aneurysm? • Hypertension, the commonest cause (80%).
A: It is three times more in males, above 60 years of age: • Cystic medial necrosis in aortic wall (Marfan
• May be asymptomatic. syndrome and Ehlers-Danlos syndrome).
• Pulsatile mass. • Surgery (aortic valve replacement and coronary
• Intermittent or continuous abdominal pain that artery bypass surgery).
radiates to the back, iliac fossa or groin. • Pregnancy (in the third trimester).
• Sometimes, there may be acute severe pain due to Presentations:
rupture and can cause collapse. • Sudden severe chest pain, tearing in nature,
• Features of thromboembolism. between shoulder blades.
__ SHORT CASES IN CLINICAL MEDICINE

• Features of shock. Treatment:


• Features like acute myocardial infarction. • When it involves ascending aorta, immediate
• Asymmetrical pulse, features of AR. surgery is suggested.
• Chest X-ray (widening of upper • When it involves descending aorta, conservative
mediastinum). treatment is suggested; surgery may be necessary
• Echocardiogram (transoesophageal echocardio- later on.
gram highly specific). • Treatment of primary cause (BP must be
• CT or MRl (also highly specific). controlled).

Mass in Right Iliac Fossa (lleocaecal Tuberculosis)


Instruction by the examiner: Q: What are the causes of ileocaecal TB? Or what is the
• Examine the abdomen. pathogenesis?
• Palpate the abdomen. A: It is caused by reactivation of primary disease by
Mycobacterium tuberculosis. May be secondary to pul-
monary TB (by swallowing of sputum). Sometimes,
Presentation of a Mass primary TB due to Mycobacterium bovis (rare now-a-
in Right Iliac Fossa days). After involvement of mucosa and submucosa,
intense inflammation with necrosis occurs in the
• There is a mass in right iliac fossa, 4 x 6 em, bowel wall and lymphatic. Caseation often found.
surface is irregular, nontender, ill-defined
margin, firm in consistency, and freely movable Q: What is the type of lesion and type of ulcer in
from underlying structure and overlying skin. ileocaecaJ TB?
A: Types of lesions are ulcerative, hypertrophic or
mixed. Ulcer is transverse (in typhoid and Crohn
disease, and ulcer is longitudinal).
My differential diagnoses are (tell the causes according
Q: What are the presentations of ileocaecal 1'B1
to the age of the patient}.
A: History of pulmonary TB may be present:
If the patient is young or early-aged: • Abdominal pain is the commonest (usually in
• Appendicular lump (usually tender). right iliac fossa, occasionally generalized).
• Jleocaecal tuberculosis. • Features of intestinal obstruction (acute or suba-
• Crohn disease. cute), or peritonitis or ascites.
• Lymphoma. • Diarrhoea or malabsorption syndrome.
• Tubo-ovarian mass in female. • Mass in right iliac fossa.
• Others: Pelvic kidney, tumour in undescended • Others are fever, malaise, loss of weight and
testes. fistula formation.

If the patient is elderly 01· middle-aged: Q: What investigations are done to diagnose ileocaecaJ
TB?
• Appendicular lump (usually tender).
A: As follows:
• I1eocaecal tuberculosis.
• CBC and ESR.
• Carcinoma of caecum (usually hard, irregular and
• Chest X-ray (shows TB in 50%).
nontender).
• Mantoux test (MT).
• Lymphoma. • USC of abdomen.
• CTscan.
N.B. If there is scar mark in lumbar area, diagnosis • Barium follow-through with spot film in
is transplanted kidney. ileocaecal region.
• Colonoscopy or ileoscopy may be done.
Q: Mention one investigation. • Sometimes, laparoscopy to see tubercle in
A: Ultrasonography of abdomen. peritoneum and biopsy.
4. ABDOMEN __

Q: What are the complications of ileocaecal TB? Q: What is the treatment of ileocaecal TB?
A: As follows: A: As follows:

• Intestinal obstruction. • Standard anti-TB chemotherapy (using four


• Fistula [enteroenteric or enterocutaneous). drugs) for 1 year.
• Malabsorption. • Occasionally, surgery (if intestinal obstruction or
• Perforation (rare). fistula).
~
c...
o
Crohn Disease S
s
Instruction by the examiner: ileum. It is slightly common in female, M:F = 1: 1.2t
• Examine the abdomen. more in young (mean age is 26 years). Common
• Palpate the abdomen. presentations are as follows:
• Frequent diarrhoea.
• Abdominal pain (colicky).
Presentation of a Mass
• Weight loss.
in Right Iliac Fossa
• Failure to thrive in children.
• There is a mass in right iliac fossa, 2 x 3 ern, • Other systemic features are malaise, lethargy,
surface is irregular, nontender, margin is ill- low-grade fever, anorexia, nausea, vomiting.
defined, firm in consistency, and freely movable • Extraintestinal manifestations (see below).
from underlying structure and overlying skin. • Sometimes, it may present with acute emergency
(There may be multiple scar in abdomen due to (e.g. acute appendicitis). If laparotomy is done,
repeated surgery or fistula.) terminal ileum looks oedernatous and red.
• Sometimes, the patient may present with
recurrent aphthous ulceration of mouth, mass
My differential diagnoses are: As in ileocaecal in right iliac fossa (due to inflamed loops of
tuberculosis. bowel matted together or abscess), anal fissures
or perianal abscess.
Q: What are the causes of multiple fistula or sinuses in
abdominal wall? Q: What are the causes of Crohn disease?
A: As follows: A: Actual causes unknown, Probable factors are:
• Trauma. • Genetic and familial.
• Crohn disease. • Diet: High sugar and fat, but low residue diet.
• Abdominal tuberculosis. • Smoking.
• Faecal fistula. • Probable association with mycobacteria and
• Actinomycosis. measles virus (not proved).
• Disseminated malignancy. • Abnormal immunological response.

Q: What are the sites of Crohn disease? N.B. Remember the following points:
A: Any part of gastrointestinal tract from mouth to • Appendicectomy is protective of ulcerative colitis,
anus, may be involved, but commonly terminal but may increase the risk of Crohn disease or may
ileum is involved (hence, it was previously called result in more aggressive disease.
regional ileitis). In order of frequency. ileum and • Oral contraceptive pill increases the risk of Crohn
right side of colon, colon alone, terminal ileum disease.
alone, ileum and jejunum. Lesion is transmural (all
layers are involved). The disease can involve a small
area of the gut, or multiple areas with relatively Extraintestinal manifestations of Crohn
normal bowel in between them called 'skip lesion'. disease
Q: What is Crohn disease? What are the • Eyes: Conjunctivitis, episderitis, uveitis or iritis.
presentations? • Mouth: Aphthous ulcer and thickened lip.
A: Crohn disease is a chronic inflammatory dis- • Skin: Erythema nodosum, pyoderma gangrene-
ease of unknown aetiology involving any part of sum, fistula or scar in abdominal wall.
gastrointestinal tract, commonly the terminal
__ SHORT CASES IN CLINICAL MEDICINE

• Mesenteric or portal vein thrombosis.


• Bones and joints: Acute arthropathy or arthralgia,
• Venous thrombosis (in other veins).
ankylosing spondylitis or sacroiliitis and clubbing.
2. Morphology: By radiological or endoscopy.
• Perianal region: Perianal fistula, skin tag and
3. Laboratory:
abscess.
• Low albumin (due to protein-losing
• Liver or hepatobiliary: Fatty liver, pericholangitis,
enteropathy) .
sclerosing cholangitis (common in ulcerative
5 colitis), autoimmune hepatitis, cirrhosis of liver,
• High ESR.
Eo • High C-reactive protein (CRP).
granuloma, liver abscess or portal pyaemia, gall
• Scanning with white cell labelled with
~ stone and cholangiocarcinoma.
1IIIndium or 99mTcto locate active site.
• Kidney: Nephrolithiasis (oxalate stone), hydro-
nephrosis and pyelonephritis. Q: What investigations should be done in Crohn
• Others: Amyloidosis and venous thrombosis. disease?
-----'
A: As follows:
• CBC (anaemia is normocytic, may be megalo-
Q; What is the relation of smoking in IBD? blastic due to vitamin BI2 deficiency).
A: In smokers, the incidence of Crohn disease is high. • ESR and CRP (both high).
But there is increased risk of ulcerative colitis in • Total protein and A:G ratio (low albumin).
nonsmokers or ex-smokers. • Liver function tests (may be abnormal).
• Blood for CIS (if septicaemia is suspected).
Q: What are the types of arthritis in Crohn disease?
• Stool for RjE and CjS (to exclude infective cause
A: Peripheral arthropathy is common, which may be
like salmonella, shigella, campylobacter, E. coli,
of two types:
Clostridium difficile).
• Type 1, Pauciarticular: Usually acute, self-limiting, • USG of whole abdomen.
<10-weeks duration, occurs with IBD relapses, • Barium follow-through or small bowel enema
usually associated with other extra-intestinal (detects ileal disease; there may be narrowing of
features of IBD. Indicates active disease. the affected segment called string sign, which is
• Type 2, Polyarticular: Lasts longer (months pathognomonic of Crohn disease).
to years), not related to IBD activity; usually • Barium enema.
associated with uveitis. • Colonoscopy (in colonic Crohn disease) with
Other types are: Ankylosing spondylitis, arthralgia, ileoscopy and biopsy.
inflammatory back pain, which are not related to • Enteroscopy.
IBD disease activity. • Capsule endoscopy (in assessing small bowel
disease).
Q: What are the differential diagnoses of Crohn
disease?
• cr scan or MRI of abdomen.
A: As follows: Q: How to treat Crohn disease?
• Acute appendicitis or appendicular lump. A: Induction of remission in active disease and main-
• lIeocaecal TB. tenance of remission.
• Carcinoma of caecum. 1. Induction of remission:
• Lymphoma. a. General measures:
• Amoeboma. • Diet, with high protein, low fat and milk
• Mesenteric adenitis. free. If needed, enteral or parenteral feeding.
• Infections by Yersinia, actinomycosis. • For anaemia: Supplement of iron, B12,
Q: Ilow to assess the activity of Crohn disease? folic acid and zinc. Erythropoietin may be
A: Signs of activity are: given.
1. Clinical: • Symptomatic treatment for diarrhoea
• Eye signs: Episcleritis, conjunctivitis.and iritis. [Ioperamide. codeine phosphate or
• Mouth: Aphthous ulcer. cophenotrope). In longstanding diarrhoea,
• Skin: Erythema nodosum and pyoderma cholesryramine may be helpful.
gangrenosum. b. Drugs:
• Arthralgia of large joints. • Prednisolone: 40-60 mg/day. Budesonide
• Fatty liver or liver abscess or portal pyaemia. may be used in moderately active disease.
4· ABDOMEN_

• Combination of prednisolone and azathi- is ineffective). Infliximab helps in healing


oprine or 6-mercaptopurine (6-MP) may fistula and perianal disease. These agents
be used. are contraindicated in presence of infection
• Forperianal disease, metronidazole] 400 mg including tuberculosis. Allergic reaction may
ED for 14 days) plus ciprofloxacin may occur.
be given. 6-MP or azathioprine may be 4. Surgical management: Surgery should be
used in chronic case. Infliximab and adali- avoided, if possible, and only minimum
mumab are effective in healing fistula and resection should be done, as the disease is mul-
perianal disease. ticentric and recurrence is almost inevitable.
• In active and moderate-to-severe total Indications of surgery are:
Crohn colitis or ileo-colitis, treatment is • Failure of medical therapy, intractable disease
like active ulcerative colitis, as follows: or fulminant disease.
o Oral and per-rectal aminosalicylate • Complications like toxic megacolon, obstruc-
plus per-rectal steroid should be given. tion, perforation, massive haemorrhage,
o Oral prednisolone is indicated for more refractory fistula and abscess, etc, which are
active disease or when arninosalicylate not responding to medical treatment.
is ineffective. • Extra-intestinal complications like severe
o In more severe colitis or in patient who arthritis or pyoderma gangrenosum not
fails to maximum oral therapy, patient responding to medical treatment.
should be hospitalized and treated as • Failure to grow in children despite medical
follows: treatment.
- IV fluid and nutritional support. • Suspicion of malignancy or severe dysplasia.
- IV methyl prednisolone or hydrocor-
Recurrence is common after surgery. If second
tisone 100 mg 6 hourly.
surgery is needed, azathioprine or 6-MP should
- TopicaJ and oral aminosalicylates are
be added to prevent the chance of recurrence.
also used.
There is no strong benefit, if it is given after first
- If the patient does not respond to the
surgery.
steroid therapy, then IV cyclosporine
or infliximab may be given. Otherwise N.B. Prednisolone has no role to prevent recurrence.
urgent surgery should be done.
2. Maintenance of remission: Q: What are the differences between ulcerative colitis
• Smoking must be stopped. and Crohn disease?
• Aminosalicylates may be given, but has A: As follows:
minimal efficacy.
Features Crohn disease Ulcerative colitis
• Thiopurines (azathioprine or 6-MP) is given
Site Mouth to anus, com- Large gut,
in patient who relapses more than once a
monly ileum and right commonly rectum
year.
side of colon
• If it fails, weekly MTX should be given.
Nature Transmural (all Mucosa
• In aggressive disease, combination of immu-
layers of gut wall
nosuppressive and anti-TNF therapy should
are involved)
be given.
Type Patchy and Continuous or
3. In resistant cases (to steroid or immunosuppres- discontinuous, skip confluent
sive) or failure of above therapy, the following lesions are present
treatment may be given: Crypt abscess Less Common
• Methotrexate. Fistula, perianal Common Uncommon
• IV cydosporine. or ischiorectal
• Anti-TNF antibody, e.g. infliximab may abscess, and
be given in infusion 4-8 weekly, in three skin tag
occasions. Also, adalimumab may be used. Others Deep ulcers with Pseudopolyps
Relapse usually occurs after 12 weeks. So, fissure. Mucosa in (hypertrophy of
MTX or azathioprine or 6-MP should be between them looks mucosa)
added to maintain remission (etanercept like cobblestone
_ SHORT CASES IN CLINICAL MEDICINE

Colon cancer Less Common • Cells • Chronic inflamma- • Acute and


tory cells with chronic inflam-
Microscopy: lymphoid matory cells in
• Granuloma • In 50-60% cases, • Absent hyperplasia lamina propria
noncaseating and crypts
granuloma present. Smoking Common in smoker Common in
• Goblet cells • Slight loss or normal • Loss or depleted nonsmoker and
and distorted ex-smoker

- -

Carcinoma of Colon
Instruction by the examiner: Q: What are the sites of co Iorectal carcinoma?
• Examine the abdomen. A: As follows:
• Palpate the abdomen. • Rectum (20%).
• Rectosigmoid (10%).
• Sigmoid colon (25%).
Presentation of a Case • Caecum and ascending colon (25%).
• Transverse colon (15%).
• There is a mass in left iliac fossa, 3 x 4 em,
• Descending colon (5%).
irregular, nontender, ill-defined margin, hard in
consistency and fixed. Q: What is the most common site of carcinoma of
colon?
A: Rectosigmoid (65% cases) is the most common site.
My differential diagnoses are (mention according to Q: What are the types of carcinoma of colon?
the age of the patient): A: As follows:
If the patient is young, the causes are: 1. Macroscopically:
• Thick colon (in irritable bowel syndrome). • Polypoid and fungating.
• Faecal mass (mass indented and moulded by • Annular and constricting.
pressure). 2. Microscopically: Adenocarcinoma.
• Diverticulitis. Q: What are the causes or predisposing factors for
• Occasionally, normal colon may be palpable. carcinoma of colon?
• Tube-ovarian mass in female. A: The causes are unknown. Predisposing factors are as
• Pelvic kidney. follows:
• Carcinoma of colon (rare). 1. Dietary factors:
If the patient is elderly, the causes are: • Excess consumption of red meat, saturated
• Faecal mass. animal fat.
• Diverticulitis (tender, mobile mass). • Less dietary fibres.
• Carcinoma of colon (descending or sigmoid • Less intake of vegetables and fruits (high
colon). vegetables and fruits may be preventive for
• Thick colon (in irritable bowel syndrome). carcinoma).
• Excess and prolonged sugar consumption.
Q: What investigations do you suggest in this case? 2. Nondietary factors:
A: As follows: • Increasing age.
• usc of whole abdomen. • Cenetic factors such as benign adenornatous
• Stool for occult blood. polyp or familial adenornatous polyposis.
• Barium enema (double-contrast). • Hereditary nonpolyposis colonic cancer.
• Sigmoidoscopy and biopsy. • Family history of colon cancer.
• FNAC (CT-guided or USC-guided). • Longstanding extensive ulcerative colitis or
• Laparoscopy, in some cases. Crohn colitis, especially if associated with
• Occasionally, laparotomy may be needed. primary sclerosing cholangitis.
4. ABDOMEN_

• Personal history of breast cancer. Q: How screening and prevention are done in carci-
• Ureterosigmoidostomy. noma of colon?
• Acromegaly. A: Screening is done in the folJowing way:
• Pelvic radiotherapy. • Any person >50 years of age, stool is tested for the
• Alcohol (weak association) presence of occult blood.
• Smoking (relative risk 1.5-3.0). • Colonoscopy (gold standard).
• Obesity and sedentary lifestyle. • Flexible sigmoidoscopy is an alternative to
• Cholecystectomy. colonoscopy.
• Type 2 diabetes (hyperinsulinaemia). • CT colonoscopy may be used in screening
programme.
Factors that decrease risk of colorectal carcinoma:
• Screening for high-risk patients by molecular
• Diet: Increased fibre, fruits, vegetable, garlic, milk.
genetic analysis (very promising, but not widely
• Exercise (colon only).
available).
• Drugs: Aspirin or other NSAIDs, calcium, folic
Prevention:
acid, omega-3 fatty acids, combined oestrogen
• Chemoprevention by using aspmn, calcium,
and progesterone hormone replacement therapy.
folic acid. Cox-2 inhibitor may have some role to
Q: What are the features of carcinoma of colon? play in prevention.
A: The features of carcinoma of colon depend on the • Secondary prevention to detect early and
site (may be asymptomatic): precancer stage. It is done by screening.
• If on the left side, there may be bleeding per
~.B. Remember the following:
rectum, alteration of bowel habit, mass in left
• Colorectal carcinoma is common in the
iliac fossa.
Western world, less among Asians.
• If on the right side, there may be alteration of
• Second common cause of death.
bowel habit, intestinal obstruction, mass in right
iliac fossa. Q: How the colorectal carcinoma spreads?
A: As follows:
N.B. Any patient over 40 years of age presenting • Local infiltration through bowel wall.
with new large bowel symptoms should be • By lymphatics.
investigated. Alarming symptom~ are change • By blood.
in bowel habit, rectal bleeding, anorexia and • Transcoelomic.
weight loss, faecal incontinence, tenesmus
Q: How to treat colorectal carcinoma?
and passing mucus per rectum.
A: As follows:
Q: What investigations should be done in colorectal 1. Curative:
carcinoma? • Surgical resection of the tumour with pericolic
A: As follows: lymph nodes.
• USG of whole abdomen (to see the mass, metas- • Adjuvant postoperative chemotherapy (with
tases, lymph node involvement). 5-fluorouracil and folinic acid).
• Sigmoidoscopy or colonoscopy and biopsy (gold • Radiotherapy is not much helpful. Preopera-
standard). tive radiotherapy may be given to large fixed
• CT colonography. rectal cancer to make it resectable. Postopera-
• CT scan of whole abdomen. tive radiotherapy may be required in some
• Endoanal ultrasound or pelvic MRI (used for cases.
staging of rectal cancer). • In some cases with metastatic disease, mon-
• PET scan is useful for detecting occult metastases oclonal antibodies like bevacizumab or
and for evaluation of suspicious lesions found on cetuximab, either alone or with chemothera-
CTor MRI. peutic agents such as irinotecan, may be used.
• Barium enema (double contrast): May be helpful 2. Palliative:
to see the mass, but CT colonography is more • Palliative chemotherapy with 5-FU may
preferable. improve survival. If this fails, second-line
• Others: Complete blood count, stool for occult drug such as irinotecan may be given.
blood, CEA (to see recurrence), X-ray chest. • Endoscopic laser therapy or insertion of an
• FNAC (CT-guided or USG-guided). expandable metal stent can be used to relieve
• Sometimes, laparotomy may be needed. obstruction.
CHAPTER 5

HAEMATOLOGY
<C ••• it clearly appears that the blood is the generative part, thefountain of life, thefirst to live, the last to die, and
the primary seat of the soul"
- William Harvey

Generalized Lymphadenopathy (Hodgkin Lymphoma)

Usual instructions by the examiner:


• Perform the general examination.
• Examine the neck and relevants.

Presentation of a Case ~I-----------,

• Present as described in generalized


lymphadenopathy.

Q: What is lymphoma?
A: Group of disorders due to neoplastic proliferation
oflymphoid tissue. Majority are of B-ceUorigin. The
two types are: Bilateral cervical lymphadenopathy
• Hodgkin disease (HD).
• Non-Hodgkin lymphoma (NHL). Q: What is Hodgkin disease?
A: It is a type of lymphoma characterized by painless,
progressive enlargement of LNs associated with
Reed-Sternberg giant cell (hallmark of the dis-
ease). It usually occurs in adolescence and young
adults (20-35 years of age); also after 45 years of
age (50-70 years, two peaks of incidence). Median
age is 31 year. More in males than females (1.5:1):
Three times more with past history of infectious
mononucleosis.

Q: What is Reed-Sternberg cell?


A: It is a malignant cell of B-cell origin that is
characterized by:
• Large cell with paired mirror image nuclei that
resembles 'Owl's-eye' appearance.
• Prominent nucleoli.
Reed-Sternberg cell is the hallmark of Hodgkin
disease. Rarely, found in infectious mononu-
cleosis, recurrent Burkitt lymphoma and chronic
Right sided cervical lymphadenopathy lymphocytic leukaemia (Cl.L).
5 • HAEMATOLOGY _

Q: What are the types of Hodgkin disease? 1. A-No systemic features.


A: There are two types of Hodgkin disease: 2. B-With systemic features, such as:
1. Nodular lymphocyte-predominant Hodgkin • Weight loss.
lymphoma (HL) (5%, slowly growing, localized • Drenching sweats.
and rarely fatal). Q: What investigations are done in HD?
2. Classical HL, which is of four types: A: As follows:
• Nodular sclerosis (70%, common in young, 1. CBC: There may be normocytic normochromic
female). anaemia, lymphopaenia, high eosinophil and
• Mixed cellularity (20%, common in elderly, high ESR. Blood count may be normal.
male). 2. Chest X-ray (shows bilateral hilar lymphaden-
• Lymphocyte predominance (5%, common in opathy and widening of mediastinal shadow).
3. FNAC or biopsy (Biopsy is more preferable to
men).
see the architecture of lymph node that may not
• Lymphocyte depleted (rare, probably
be detected by FNAC; also biopsy is necessary
represents large cell or anaplastic NHL).
for staging.)
4. Ultrasonography (USG) of whole abdomen.
Q: How does the patient present in Hodgkin disease?
A: As follows:
s. cr scan of chest and abdomen including pelvis
(necessary for staging).
• Asymptomatic.
6. Others:
• Generalized lymphadenopathy starting in • PET (positron emission tomography): Used
cervical glands; then axillary, inguinal, which are for staging, assessment of response and
painless, discrete and rubbery. direction of therapy.
• Dry cough, shortness of breath (due to • Bone marrow study (involved in advanced
mediastinal lymphadenopathy ). stage).
• Systemic features: Fever, loss of weight, malaise, • Renal function tests: Mainly creatinine
weakness and pruritus (10%). (necessary prior to treatment).
• After alcohol intake, pain at the site of LN • Liver function tests: Mainly SGPT (necessary
involvement. prior to treatment).
• Serum uric acid (necessary prior to treatment).
Q: What is Pel-Ebstein fever? • Serum lactate dehydrogenase (LDH).
A: Recurrent bouts of pyrexia followed by apyrexial
N.B. Remember the following points:
period. May occur in 10% cases ofHD.
• Lymphopaenia, high LDH and lymph node
Q: What are the stages ofHD? >10 em are poor prognostic factors.
A: Ann Arbor staging classification is given below: • High ESRis an indicator of disease activity.
• Stage 1: Involvement of single LN region (I) or • High alkaline phosphatase (biliary
extralymphatic site (TAB). obstruction) indicates involvement of lymph
• Stage 2: Involvement of two or more LN regions nodes in porta hepatis.
(II), or an extralymphatic site and LN regions on
the same side of diaphragm (lIE).
• Stage 3: Involvement of LN regions on both sides
of diaphragm with (IIIE) or without (III) localized
extralymphatic involvement or involvement of
spleen (IllS) or both (IIJSE).
• Stage 4: Diffuse involvement of one or more
extralymphatic tissue with or without LN
involvement (bone marrow, liver and lung).
(Lymphatic structures include LN, spleen, thymus,
Waldeyer ring, appendix and Peyer patch.)
In the presence of systemic features, each stage may
be divided into two: Reed-Sternberg giant cell
_ SHQRT CASES IN CLINICAL MEDICINE

or doxorubicin, cyclophosphamide, oncovin or


vincristine, procarbazine, prednisolone).
S. Patient who is resistant to chemotherapy may be
considered for autologous bone marrow trans-
plantation.
Other chemotherapeutic regimen that was previously
used is as follows:
• MOPP: Mechlorethamine, oncovin (vincristine),
procarbazine and prednisolone.
• COPP: Cyclophosphamide, oncovin (vincristine),
procarbazine and prednisolone.
Reed-Sternberg giant cell Q: What is the prognosis of HD?
Q: How is staging done and why? A: Prognosis depends on the stage:
A: For staging, following tests are done: • Early-stage HD: Complete remission in >90%
• Chest X-ray. when treated with chemotherapy followed by
radiotherapy. The majority are cured.
• Bone marrow.
• Advanced-stage HD: SO-70% can be cured.
• USC of whole abdomen.
• Patients who fail to respond to initial chemother-
• cr scan (whole abdomen and chest).
apy have a poor prognosis, but some may achieve
Staging is done for selection of therapy (radiotherapy
long-term survival after autologous bone marrow
or chemotherapy) and prognosis.
transplantation.
Q: How to treat HD? • Patients relapsing within a year of initial chemo-
A: As follows: therapy have a good salvage rate with autologous
1. Majority of HD patients are now treated with BMT.
chemotherapy and adjunctive radiotherapy.
• Patients relapsing after 1 year may obtain long-
ABVD (Adriarnycin or doxorubicin, Bleomycin,
term survival with further chemotherapy alone.
Vinblastine and Dacarbazine) regimen is widely
used. ABVD chemotherapy can cause cardiac • Presence of B symptoms indicate adverse
and pulmonary toxicity due to doxorubicin and prognosis.
bleomycin, respectively. Infertility and second-
ary myelodysplasia or acute myeloid leukaemia N.B. Remember the following points:
(AML) is low with this regime. • Fertility is usually preserved afterradiotherapy.
2. Patient with early stage HD (lA, IlA, no bulk) • In young women with mediastinal disease,
is treated with 2-4 cycles of ABVD followed by radiotherapy to the breast may cause breast
radiotherapy (20-30 Cy) to the involved lymph cancer (hence, needs follow-up).
nodes. Treatment response is monitored bycr • Patients who continue to smoke after chest
scan or positron emission tomography (PET) radiotherapy, lung cancer may occur.
scan.
• Cardiac disease may occur after supradia-
3. Patient with advanced disease is usually treated phragmatic mantle field radiation.
with chemotherapy alone. Usually 6-8 cycles
• After chemotherapy, infertility may occur in
of ABVD is given. Radiotherapy at the bulk site
man (needs counsell ing and storage of sperm).
used previously is now avoided. Achieving PET-
negative remission predicts a better long-term • Infertility is less in woman. Premature
remission rate. Overall, the long-term disease menopause may occur.
control or cure rates are lower with advanced • In 5%, myelodysplasia and acute leukaemia
disease. may occur 5-10 years after alkylating agent.
4. New regimens are being tested for those who fail Q: What is disease-free or cure in HD?
with above therapy (about 25%). This includes A: If no relapse after 5 years of withdrawal of treat-
BEACOPP (bleomycin, etoposide, adriamycin ment, it is called cure or disease-free.
5 • HAEMATOLOGY __

Generalized Lymphadenopathy (Non-Hodgkin Lymphoma)


Instructions by the examiner: • A late manifestation of Hlv infection. Primary
• Perform general examination. brain lymphoma, irnmunoblastic diffuse large
• Examine the neck and relevant. B-cell lymphoma and Burkitt lymphoma may
occur in AIDS.
Q: What is non-Hodgkin lymphoma (NHL)? What • Lymphoma occurs in congenital and acquired
are the types (grading)? immunodeficiency states. Acquired immuno-
A: NHL refers to malignant proliferation of lymphoid deficiency, as a result of organ transplantation,
cells; and majority is B-cells (70%) and few T-cells is strongly associated with NHL. These are
(30%). commonlyextranodal and most frequently occur
in the first year after transplantation.
Types or grading (depending on the rate at which
• Gastric lymphomas are associated with specific
the cells are dividing):
chromosome lesions. The t(14:18) translocation
• Low grade shows the following characteristics:
in follicular lymphoma results in the dysregulated
o Low cell proliferation rate.
expression of the BCL-2 gene product, which
o Asymptomatic for many years.
inhibits apoptotic cell death.
o Slow indolent course.
• A number of familial cancer syndromes are also
o Remitting and relapsing course.
associated with NHL.
o Good response to minimal therapy.
• In autoimmune disorders, high risk of NHL is
o Incurable, but the patient survives for long time.
reponed, e.g. Sjogren syndrome and extra nodal
o Most nodular lymphoma is of low grade.
marginal-zone lymphoma.
o Small-cell disease (mature lymphocyte) is
• There may be an association with exposure to
associated with low-grade lymphoma.
pesticide, hair dyes, organic solvents, etc.
o No treatment is required, if the disease is not
advanced and asymptomatic. Q: What are the clinical features of Non-Hodgkin
o Median survival up to 10 years. lymphoma (NHLF
o Transformation to high grade is associated A: As follows:
.with poor prognosis. • NHL can occur at any age; but peak incidence is
65-70 years.
• High grade shows the following characteristics:
• It is multicentric in origin and spreads rapidly
o Cell divisions occur quicldy.
to noncontiguous areas. The disease is usually
o Early symptoms are common.
widespread at the time of diagnosis.
o Fatal, if untreated.
• Discrete, painless, firm, lymph nodal enlargement
o Responds better to treatment and patient
is the most common presentation. Waldeyer ring
may achieve a long-term remission if treated
and epitrochlear lymph nodes are frequently
properly (potentially a curable disease).
involved.
o 80% respond to initial therapy, and 35% are
• B symptoms of fever, night sweats and weight loss
disease-free for 5 years. are less prominent.
o Large-cell disease (immature lymphoid cells)
• Extranodal presentations are more common than
is of high grade.
Hodgkin disease. May involve gastrointestinal
o Most diffuse lymphomas are of high grade.
tract (stomach), lung, thyroid, skin, testes and
Stages of NHL are much similar to Hodgkin central nervous system (CNS). Skin involvement
lymphoma (HL). (T-cell lymphoma ) presents as Mycosis fungoides
and Sezary syndrome. Oropharyngeal involve-
Q: What are the causes of NHL? ment occurs rarely. Com.pression syndrome may
A: Actual cause is unknown. Probable causes are: occur, such as paraplegia due to compression
• Specific lymphoma types are associated with of spinal cord by an extradural lymphoma,
Epstein-Barr virus (EBY), human herpes virus 8 dysphagia, breathlessness, vomiting, intestinal
(HHV-8) and human T-cell Iymphotropic virus obstruction, ascites and limb oedema, SVC
(HTLV) infection. obstruction.
• Helicobacter pylori is an aetiological factor in • Bone marrow involvement is more common in
gastric MALTlymphoma. low grade (60%) and less in high grade (10%).
_ SHORT CASES IN CLINICAL MEDICINE

• Involvement ofliver and spleen results in Q: What is Waldeyer ring?


hepatosplenomegaly. A: It is a circle of lymphatic tissue in posterior part of
• Bone involvement may manifest as pathological oropharynx and nasopharynx, which includes ton-
fractures with pain. sils and adenoids. It is involved in NHL, and rarely
in HL.

Q: What are the treatments of NHL?


A: As follows:
Low-grade NHL: Not cured by any therapy. No
therapy, if the patient is asymptomatic.
Indications of treatment are:
• Marked systemic symptoms.
• Bone marrow failure.
• Features of compression [superior vena caval
(SVC) obstruction, spinal cord, gut obstruction
and ascites].
• Large lymphadenopathy causing discomfort or
disfigurement.
Treatment options include:
• Radiotherapy, for stage I.
• Chemotherapy is needed in most cases.
Chlorambucil orally may be used. More
Axillary lymphadenopathy
aggressive combination therapy may be tried in
younger age.
• In monoclonal antibody therapy rituximab
(anti-CD 20 antibody) is effective in 60%
cases. Synergistic effects are seen with standard
chemotherapy.
• Autologous stem cell transplantation can also be
given.
High-grade NHL
• Chemotherapy (R-CHOP: rituximab, cyclophos-
phamide, hydroxydaunorubicin, vincristine and
prednisolone).
• Radiotherapy-for stage I and to reduce bulk of
the disease.
• Autologous stem cell transplantation for relapse
chemosensitive disease.

Q: What is the prognosis?


Cervical lymphadenopathy A: As follows:
1. Low-grade lymphoma has a slow indolent
Q: What investigations are done in NHL? course. Median survival up to 10 years. Trans-
A: Similar to HL, with the following: formation to high grade is associated with poor
• Routine bone marrow and trephine. prognosis.
• Immunotyping of surface antigen to distinguish 2. In high-grade lymphoma, 80% respond to ini-
T-cell and B-cell tumours (done on blood, tial therapy, and 35% are disease-free for 5 years.
marrow or nodal material). CO 20.should be 3. Adverse prognostic factors in NHL includes:
done. • Age >60 years.
• Immunoglobulin determination (IgG or • Stage III or IV (advanced disease).
IgM paraprotein, which serves as markers for • High serum LOH level.
treatment response). • Performance status ECOG 2 or more.
• Uric acid. • More than one extranodal site of involvement.
4. In case of high-grade NHL, 5-year survival
of patient with adverse prognostic factors
(high-risk score) is 25%, while that of patients
without adverse prognostic factors (low-risk
score) is 75%.
5. Relapse is associated with a poor response to
further chemotherapy (<10% 5-year survival);
but in patients under 65 years, bone marrow
transplantation improves survival.
Q: What are the differences between HL and NHL?
A: As follows:

Features Hodgkin Non-Hodgkin


lymphoma lymphoma
Age Bimodal. First peak Median age 65-70
at 20-30 years and years.
second peak at
50-70 years
LN involvement Unifocal, usually Multicentric, more
localized to a single frequent involve-
axial group (cervical ment of peripheral
and mediastinal) LN
Mesenteric LN Rarely involved Commonly
and Waldeyer ring involved

Generalized Lymphadenopathy (Chronic Lymphatic Leukaemia)


Usual instructions by the examiner: • Generalized lymphadenopathy (detected on
• Perform the general examination. routine examination).
• Examine the neck and relevants. • Hepatosplenomegaly (huge splenomegaly, if
autoimmune haemolytic anaemia).

Q: What investigations do you suggest?


Presentation of a Case A: As follows:
(Elderly or Middle Aged) 1. CBC: Hb% (low), leucocytosis (50-200 x 109/
mm"), differential count (DC) shows increased
• Present as described in generalized lymphocytes.
lymphadenopathy.
2. 95%, with mostly small lymphocyte, platelet is
normal, low or slightly increased.
3. Bone marrow (increased lymphocytes).
Q: How does the patient of CLL usually present? 4. Others:
A: Common in the elderly, M:F = 2:1, involving • Reticulocyte (high in autoimmune haemolytic
B lymphocyte, after 45 years (usually 60-70 years). anaemia).
• Asymptomatic, diagnosed incidentally in routine • Coombs test (positive in autoimmune
examination. haemolytic anaemia).
• General features, such as malaise, weakness, • Paraproteins (may be increased).
fatigue, weight loss and night sweating.
• Uric acid (high).
• Features of anaemia. • Immunophenotyping of B-cell antigen
• Recurrent infection. (CD19 and CD23) and T-cell antigen (CDS).
__ SHORT CASES IN CLINICAL MEDICINE

Q: Why anaemia in CLL?


A: Anaemia is due to:
• Bone marrow infiltration.
• Autoimmune haemolytic anaemia.

Q: How to treat CLL?


A: Treatment depends on stage of disease:
• Stage A: No treatment, unless progression occurs.
The patient survives for long lime (reassurance
and follow-up). Life expectancy is normal in
older patients.
Chronic lymphatic leukaemia
• Stage B: No treatment, if the patient is
Q: What is CLL? asymptomatic.
A: It is a neoplastic disorder of lymphocyte that usually • Stage C: Usually treatment is necessary.
involves B-Iymphocytes and rarely 'l-Iymphocytes
Indications of treatment in GLL:
(5%). More in male than in female (2: 1); occurs
usually after 45 years of age. • Evidence of marrow failure indicated by worsening
Q: What are the stages of CLL? of anaemia or thrornbocytopaenia.
A: There are three stages (Binet staging): • Massive or progressive lymphadenopathy or
splenomegaly.
Stage A (60%):
• Doubling of lymphocyte count in 6 months.
• No anaemia. • Symptoms (fever, night sweating and weight
• No thrombocytopaenia. loss).
• Less than three areas of LN involvement. • Presence of haemolysis or other immune-
Stage B (30%): mediated cytopaenias.
• No anaemia. • Recurrent infection.
• No thrombocytopaenia. Mode of treatment:
• Three or more areas of LN involvement
Stage C (10%): 1. Symptomatic:
• For anaemia and thrombocytopaenia:
• Anaemia. Prednisolone, blood transfusion should be
• With or without thrombocytopaenia.
given. If it is refractory or recurrent, then
• Regardless of area of LN involvement.
splenectomy may be done, which is also
N.B. Lymphoid enlargement includes cervical, axil- indicated for hypersplenism.
lary, inguinal, also liver and spleen. • Infection: Antibiotic, immunoglobulin (y-giobulin
Another staging (Rai staging): 0.4 glkglmonth).
• Stage 0: Lymphocytosis (in blood and bone • Local radiotherapy for LN causing discomfort or
marrow) >15000/cmm, without lymphad- local obstruction and symptomatic splenomegaly.
enopathy, hepatosplenomegaly, anaemia or
2. Specific:
thrombocytopaenia.
• Chlorambucil 5 mg daily; adjust the dose
• Stage I: Lymphocytosis with lymphadeno-
according to blood count.
pathy (without hepatosplenomegaly, anaemia
• Fludarabine alone or with cyclophosphamide
or thrornbocytopaenia).
or mitoxantrone (with or without steroid) is
• Stage II: Lymphocytosis with hepatomegaly
very helpful. Fludarabine should be avoided
or splenomegaly (with or without lymphad-
in autoimmune haemolytic anaemia as it
enopathy).
aggravates anaemia.
• Stage Ill: Lymphocytosis with anaemia (Hb%
• Combination therapy with rituximab (ineffec-
<11 gjdL) with or without hepatomegaly or
tive alone). Usually rituximab plus fludarabine
splenomegaly.
with or without cyclophosphamide is the
• Stage TV: Lymphocytosis with thrornbo-
treatment of choice.
cytopaenia (<100000/ cmm) with or
• Alemtuzumab may be used in patient that
without lymphadenopathy, hepatomegaly,
progresses after fludarabine.
splenomegaly or anaemia.
5 • HAEMATOLOGY _

• Allogenic stem cell transplantation may be Q: What is the prognosis of CLL?


curative, but only used in those patient whose A: As follows:
disease cannot be controlled by standard • Median survival is about 6 years.
therapies. • Stage A may be normal life expectancy.
Q: What is Richter syndrome? • In stage C, median survival is 2-3 years.
A: When CLL is transformed to aggressive high-grade • About 50% die from infection and 30% from
lymphoma, it is called Richter syndrome. Its progno- causes unrelated to CLL.
sis is poor with median survival of less than 1 year. • Rarely, acute blastic crisis may occur.

Purpura

Usual instructions by the examiner:


• Look at the legs. What is your diagnosis? What else
do you want to see?
Look at the following points:
1. Whether they blanch on pressure or not (purpura
does not blanch).
2. Whether they are palpable and painful or not
(palpable painful purpura is of vascular origin
and nonpalpable purpura indicates thrombo-
cytopaenia).
3. Colour change (progressive colour change in
purpura from red to dark pigmented).
4. Distribution of purpura:
• Buttock ankle (Henoch-Schonlein purpura).
• Extensor surface of forearm, dorsum of hand,
leg (senile purpura and drugs).
Purpura
• Generalized purpura.
Q: What do you think are the causes in this case?
Presentation of a Case A: Describe the causes according to age and sex.
In the elderly, the causes are:
• There is multiple purpura involving both the • Senile purpura (usually on extensor surface of
legs below the knee; some are red and some are forearm and leg).
dark, and does not blanch on pressure (describe
• Drug-induced purpura (distribution is like
whether palpable or nonpalpable).
senile purpura) and the usual drugs are steroids,
NSAlDs and anticoagulant.
• Leukaemia.
• Aplastic anaemia.
• Scurvy.
• Paraproteinaemia.
In young or child, the causes are:
• Idiopathic thrombocytopaenic purpura (ITP).
• Henoch-Schonlein purpura (involving buttock
and legs).
• Drug induced.
• Acute leukaemia.
• Infections: Viral (dengue) or meningococcal
septicaemia.
~ SHORT CASES IN CLINICAL MEDICINE

Q: In a child, there is purpura in the leg. What else do


you want to see?
A: Purpura in buttock, history of arthritis, abdominal
pain, bloody diarrhoea, haematuria (Henoch-
Schonlein purpura).

6n N.B. In any age,mention the causes as follows (if present):


o
] • If Cushingoid facies, due to steroid.
E • If patient looks toxic, due to septicaemia.
::c:"" • If evidence of rheumatoid arthritis (RA), systemic
lupus erythematosus (SLE), due to disease itself
or due to NSAlDs or steroids.

Q: What else do you like to examine to find out the Campbell de Morgan spot
cause?
A: As follows: Q: What investigations should be done in purpura?
• History of fever (dengue), meningococcal septi- A: As follows:
caemia (patient is toxic) and other infection. 1. Hb%, TC, DC, ESR,platelet and PBF.
• History of fever, arthritis, bloody diarrhoea, 2. If pancytopaenia or thrombocytopaenia.
abdominal pain, haematuria (Henoch-Schonlein perform bone marrow study (dry tap in aplastic
purpura), history of drugs. anaemia, increased megakaryocyte in ITP).
• Blood dyscrasia. 3. Other investigations (according to suspicion of
causes):
• Anaemia (leukaemia and aplasia).
• Coagulation screen [bleeding time (BT),
• Bleeding gum, corkscrew hair and perifollicular clotting time (CT), prothrombin time (PT),
haemorrhage (scurvy). activated partial thromboplastin time (AJYIT)
• Any evidence of collagen disease (SLEand RA). and fibrinogen degeneration products (FDP) I
• Other disorders [uraemia, CLD, disseminated for haemophilia and Christmas disease, and
intravascular coagulation (DIC) and metabolic other coagulation factors (DIC).
disorder]. • Blood culture (septicaemia).
• Examine the LNs, liver, spleen and bony tender- • ANA, anti-doublestrand DNA (for SLE).
ness (leukaemia). • Antiphospholipid antibody.
• Liver function tests (in CLD).
Q: What are the differential diagnoses of purpura? • Renal function tests (in CRF).
A: As follows:
Q: What is Hess test?
• Drug rash. A: It is a bedside test, also called tourniquet test. In
• Spider angioma or telangiectasia (blanches on this test, a sphygmomanometer cuff is inflated over
pressure, but purpura does not). the upper arm between systolic and diastolic blood
• Erythema nodosum (painful and nodular). pressure, kept for 5 minutes and then deflated.
• Mosquito bite (usually blanches on pressure, Again after 5 minutes, look for petechiae in cubital
but sometimes may not blanch, if there is fossa and near the wrist joint. Normally, there may
extravasation of blood). be <5 petechiae. If>10 petechiae in 2.5 em- area are
observed, it indicates thrombocytopaenia. If platelet
• Campbell de Morgan spots (common in elderly):
is <60,000, it is usually positive.
These are small, nodular, reddish lesions that
do not blanch on pressure, occur on trunk and Causes of positive Hess test: It is usually positive in
upper abdomen, and resolve spontaneously. cases of thrombocytopaenia or platelet functional
These are benign angioma, common in middle- abnormality or increase in capillary fragility. (This
aged and the elderly. Malignant change occurs test is frequently done to diagnose dengue haemor-
rarely (suggested by itching, rapid increase in size rhagic fever in which, if number of petechiae is >20,
and increased pigmentation). it IS definitely positive.)
5 • HAEMATOLOGY __

Read the following topics in relation to purpura: • Collagen disease (RA and SLE).
Q: What is purpura? What are the causes? • Paraproteinaemia (purpura is due to vasculitis
A: It is the spontaneous bleeding or extravasation of or thrombocytopaenia or platelet functional
blood from the capillary in the skin and mucous abnormality) .
membrane that does not blanch on pressure, and • Amyloidosis (periorbital).
there is progressive colour change. Coagulation abnormality:
There are lWO types of purpura: • Haernophilia.
• Small and discrete of pinhead size called • Christmas disease.
petechiae. • Anticoagulant therapy.
• Large and ill-defined called ecchymosis.
Q: What are the causes of platelet functional
Causes of purpura: abnormality?
• Thrombocytopaenic. A: Platelet functional abnormality (thrombasthenia)
• Vascular. occurs in:
• Coagulation defect. • CRF.
• Chronic liver disease (CLO).
Thrombocytopaenic purpura:
• Paraproteinaernia.
1. Primary or ITP. • Myeloproliferative diseases.
2. Secondary: • Drugs: NSAIDs (aspirin, indomethacin and
• Aplastic anaemia (due to any cause). ibuprofen) .
• Leukaemia.
• Secondary deposit in bone marrow. Q: How to differentiate in bleeding or purpura,
• SLE. whether due to bleeding abnormality or coagula-
• Others: DIC, CLD and TI'P (thrombotic tion abnormality?
thrornbocytopaenic purpura), massive blood A: As follows:
transfusion.
Coagulation abnormality Bleeding abnormality
Vascular purpura:
Family history usually present Family history mayor may
1. Congenital (hereditary haemorrhagic not be present
telangiectasia, Ehlers-Danlos syndrome).
There is history of prolonged No history of prolonged
2. Acquired
bleeding bleeding
• Senility (elderly patient).
• Henoch-Schonlein purpura (in children, and Usually there is bleeding into Bleeding into the skin and
rarely in the elderly). the joint or muscle; purpura mucous membrane; purpura
• Drug-induced (NSAlDs, thiazide, steroid, is less common or rare is more common
sulphonamide, penicillin and thiouracil). Clotting time is prolonged, Clotting time is normal, but
• Infections: SBE, typhoid, meningococcal but bleeding time and plate- bleeding time is prolonged
infection, septicaemia and viral infections let count are normal and platelet count is low
(infectious mononucleosis, measles, chicken
Particular coagulation factor Coagulation factor is normal,
pox, dengue haernorrhagic fever).
is low or absent, e.g. in hae- and it is due to either low or
• Scurvy. mophilia, factor VIII is absent defect in platelet function or
• Metabolic disorder [chronic renal failure
or low vascular defect
(CRF) and Cushing syndrome].

Idiopathic Thrombocytopaenic Purpura

The usual instructions are: Q: What is idiopathic thrombocytopenic purpura?


A: It is a type of thrombocytopenic purpura due to
• Look at the patient. What is your finding? autoantibody against platelet (IgG type). The autoan-
tibody against platelet membrane glycoprotein JIb
Presentation of a Case: and IlIa is responsible for premature removal of
(Present as in purpura page 241) platelet by monocyte-macrophage system.
~ SHORT CASES IN CLINICAL MEOICINE
I

Q: What are the presentations of ITP? Q: What are the diseases to be excluded if ITP is
A: As follows: suspected?
• In child: Usually acute presentation, previous A: As follows:
history of viral infection followed by bleeding or 1. Commonly SLEis to be excluded. In 10% cases,
purpura, easy bruising, etc. Chronic ITP is rare in thrombocytopaenia may be the initial manifes-
children. tation of SLEfor many years.
• In adult: Common in female, usually insidious 2. Primary antiphospholipid syndrome may
onset without preceding viral infection. present with thrombocytopaenia.
Presents with purpura, easy bruising, epistaxis 3. Other primary haernorrhagic disorders should
or menorrhagia. Features of SLE may be present be excluded.
at presentation or may develop after long time. 4. For this purpose, following investigations
It may be associated with other autoimmune should be done:
diseases like thyroid disorder and autoimmune • ANA, anti-ds-DNA, anti phospholipid anti-
haemolytic anaemia, chronic lymphocytic body and anticardiolipin antibody.
leukaemia, solid tumour, HIV infection. • Bone marrow.
On physical examination: Apart from bleeding
points, no other physical findings. Splenomegaly is N.B. Remember these points:
very rare. • In 10% case, ITP may be associated with
autoimmune haemolytic anaemia called
N.B. Remember the following points: Evan syndrome.
• Spontaneous bleeding occurs when the • Patient >65 years old should have
platelet is <20,000/cmm. bone marrow examination to look for
• At higher count, there may be bruising, accompanying B-cell malignancy.
epistaxis and menorrhagia. • In rrp, low platelet in blood, increased meg-
• If platelet count is >50,000/cmm, there may akaryocyte in bone marrow, prolonged bleeding
not be any features diagnosed on routine test. time and normal dotting time are common.
• HIV testing should be considered, if there is
strong suspicion.
Q: How to investigate ITp7
A: As follows:
• Full blood count (FBC) (thrombocytopaenia).
• Bone marrow (increased immature
megakaryocytes) .
• Bleeding time (prolonged).
• Clotting time (normal).
• Antiplatelet antibody (present).
• Antinuclear antibody (ANA) (to exclude SLE).
• Antiphospholipid antibody (positive in 30%
Purpura in ITP (legs)
cases).
Q: What are the differences between ITP in children
and adults (or acute and chronic ITP)?
A: As follows:

Features Child (acute) Adult (chronic)


Age Usually 2-6 years 20-30 years
Sex Any Predominant in
female
Onset Acute Chronic
Previous infec- Common Unusual
tion
Platelet count <20,000/mm3 >20,OOO/mm3
Purpura in ITP (forearm and hands)
5 • HAEMATOLOGY _

Features Child (acute) Adult (chronic) is required. Dose of immunoglobulin is 1


g/kg for 3-5 days. Its effect is temporary,
Spontaneous Common Less,<20%
persists for 3-4 weeks and is quite
remission
expensive. Steroid may be added with
Duration 2-4 weeks Chronic, months
immunoglobulin.
to years

• Second-line therapy:
Q: How to treat ITP?
o If there is frequent relapse (usually two
A: Treatment is as follows:
relapses), in primary refractory disease or
1. In child: Usually self-limiting; does not
require high dose of steroid to maintain
require treatment in most cases. If there is no
safe platelet level, splenectomy should be
improvement.
done. There is complete remission in 70%
• Prednisolone (2 mg/kg) should be given
cases and improvement in 20-25% cases.
if moderate-to-severe thrornbocytopaenia
5-10% require further medical therapy.
«10,000) and bruising, epistaxis or other
bleeding. • Third-line therapy:
• If still persistent bleeding, IVimmunoglobulin o If there is failure after splenectomy, other
(IgG) should be given. therapy should be considered-corticoster-
• In some case, platelet transfusion may be oid, IV immunoglobulin, anti-D infusion,
required when there is persistent bleed- danazol, immunosuppressive therapy
ing [epistaxis, gastrointestinal tract (GIT) (azathioprine, cyclophosphamide, dapsone,
bleeding, retinal haemorrhage, intracranial vincristine, vinblastine, cidosporine,
bleeding]. mycophenolate mofetil). Also monoclonal
2. In adult: Persistent thrornbocytopaenia is com- antibody like rituximab as well as recombi-
mon. Most patients with platelet count >30 x nant thrombopoietin may be given.
109/L are stable and do not require treatment • Platelet transfusion is not usually used.
unless they are about to undergo a surgery. However, it is used only if persistent or
• First-line therapy: potentially life-threatening bleeding or where
o If spontaneous bleeding, predni.solone emergency splenectomy is done.
1 mg/kg to be given for 4-6 weeks and then
N.B. Remember the following points:
tapered. 66% will respond, but relapse is
common when the steroid dose is reduced • In nonresponder after splenectomy, think
or stopped. 10-20% usually have long- of presence of accessory spleen (confirm by
term remission. If relapse, steroid should radionuclide scan).
be started again. • After splenectomy, there is more chance of
o IV immunoglobulin may be given mainly infection by pneumococcus, meningococcus
if there is severe haemostatic failure or and Haemophilus inJluenzae (vaccination
slow response to steroid alone or surgery against these is essential).

Henoch-Schonlein Purpura
Usual instructions by the examiner: Q: What else do you want to see?
A: Buttock. In Henoch-Schonlein purpura, commonly
• Look at the patient. What is your finding? What
buttock is involved.
may be the cause?
Q: What are the differential diagnoses?
Presentation of a Case: A: As follows:
(Present as in purpura page 241) • Drug rash.
• ITP.
Q: What is the likely diagnosis? • SLE.
A: If the patient is child, may be ITP or Henoch- • Septicaemia.
Schonlein purpura (HSP). • Thrombotic thrombocytopenic purpura (TIP).
Q: What history do you like to take in Henoch-
Schonlein purpura?
A: Arthritis, abdominal pain, bloody diarrhoea and
urinary complaint (haematuria).
Q: What is Henoch-Schonlein purpura (anaphylac-
toid purpura)?
A: It is a small vessel vasculitis characterized by pur-
pura or petechial rash, polyarthritis (in big joints),
abdominal pain and glomerulonephritis. It is due
to circulating IgA-containing immune complex.
Follow for 1-3 weeks after upper respiratory tract
infection (usually viral). Other factors responsible
include food, drugs or vaccination. Hencch-Schonlein purpura (buttock)

Q: What are the clinical features of HSP? Q: What investigations should be done in HSP?
A: HSP is more common in boys of 5-15 years of age, A: As follows:
but may occur at any age. The common features are: • FBC and platelet (nonthrombocytopaenic
• Skin lesion: Purpura is common in Jegs and purpura).
buttock; face and trunk are spared. May resolves • Urine (proteinuria and haematuria).
in 2-4 weeks and fresh crops may appear. Angio- • Serum IgA is high in 50% cases (IgA-containing
oedema occurs in 50% cases. immune complex is also high).
• Polyarthritis occurs in 70%, commonly involves • Skin biopsy from normal and involved skin (it will
knee and ankle, may be fleeting type. show leucocytoclastic vasculitis with deposition
• Abdominal pain, colicky in nature, associated ofIgA and complement C3 in blood vessels).
with nausea, vorruung, bloody diarrhoea, • Kidney biopsy.
intussusception and perforation. There is Q: Suggest two investigations that are helpful for
vasculitis; bowel is oedematous and inflamed diagnosis.
causing bleeding and obstruction. This may be A: As follows:
confused with acute surgical condition. • Serum 19Ais high in 50% cases (IgA-containing
• Renal disease occurs in 30-70% cases; present immune complex is also high).
with haematuria and proteinuria. It is usually • Skin biopsy from normal and involved skin (vas-
mild. Focal necrotisingglomerulonephritis, rarely culitis with deposition of IgA and complement
renal failure may occur. In adults, 25% cases C3 in blood vessels).
develop severe crescentic or rapidly progressing
glomerulonephritis and renal failure (which are Q: Suggest one investigation that is helpful for
less in children). prognosis.
A: Renal biopsy: It shows focal and segmental prolifer-
ative glomerulonephritis, sometimes with mesangial
hypercellularity. In more severe cases, epithelial cres-
cents may be present developing rapidly progressing
glomerulonephritis and renal failure. This is less in
children. There is IgA deposition within and around
blood vessels, glomerular mesangium (it may be
confused with IgA nephropathy).
Prognosis of HSP is related to the severity of renal
involvement.

Q: How to treat?
A: .As follows:
• Self-limiting, spontaneous cure in majority of cases.
• Steroid is indicated, if there is CIT and joint
Arthritis in Henech-Schonlein purpura symptoms (does not affect the course and
5 • HAEMATOlOGY _

progression of disease). Abdominal pain may be N.B. Henoch-Schonlein purpura in adults:


improved in 24 h. • Skin involvement is common (70%).
• In renal involvement pulse IV steroid and • Gut and joint involvement occurs in 20% cases.
cytotoxic drugs should be given. • Renal involvement is more common than
• Recurrence may occur. If so, dapsone may help in children.
cutaneous recurrence. • Myocardial involvement may occur rarely.
• Prognosis is worse in adults than children.
Q: What is the prognosis?
In children:
A: Good in children, relatively bad in adults. Adverse
factors in adults include hypertension, abnormal • Gastrointestinal vasculitis is more common.
renal function and proteinuria >1.5 gfday. But only • Renal and skin involvement is less common.
1% of patients develop end-stage renal failure. • Prognosis is better.

Splenomegaly (Hereditary Haemolytic Anaemia)


Usual instructions: Q: What hereditary haemolytic anaemia this can be?
A: ~-Thalassaemia major, HbE disease, ~-thalassaemia
• Examine the abdomen and relevant.
HbE disease (double heterozygous) and hereditary
• Palpate the abdomen. What are your findings
spherocytosis.
(splenomegaly)? What relevant do you like to see?
Q: What relevant physical findings will you see in
Presentation of a Case ,f---------,
hereditary haemolytic anaemia?
A: As follows:
(The patient is usually of young or early age.) • Anaemia and jaundice.
• Spleen is hugely enlarged, ... cm from costal margin • Frontal and parietal bossing and mongoloid
in anterior axillary line towards right iliac fossa. facies with prominent malar bones.
• Short stature and retardation of growth.
• Also, family history should be taken.
Q: What are the causes of splenomegaly in this young
patient? Q: What are the features of haemolytic anaemia?
A: As follows: A: The triad features of haemolytic anaemia are:
• Malaria. • Anaemia.
• Kala-azar. • Jaundice.
• Hereditary haemolytic anaemia. • Splenomegaly.
• Lymphoma.
• CLD with portal hypertension (causes are Wilson Q: How will you confirm your diagnosis?
disease and ai-antitrypsin deficiency). A: Using haemoglobin electrophoresis.

Q: Mention one simple investigation that will help


your diagnosis.
A: Peripheral blood film (PBF) that shows microcytic
hypochromic anaemia.

Q: Mention another investigation helpful for your


diagnosis.
A: Reticulocyte count (by supravital stain)-high.

Q: What investigations do you suggest?


A: As follows:
1. Hb%, TC, DC, ESRand PBF (microcytic
hypochromic blood picture).
2. Reticulocyte count (by supravital stain)-high.
~-Thalassaemia major 3. Haemoglobin electrophoresis.
_ SHORT CASES IN CLINICAL MEDICINE

4. Others: 1. ~-Thalassaemia: In this case, there is an


• X-ray of skull, hand and other skeletal survey. inadequate production of ~-chain, causing
• Serum bilirubin: High (also urinary less production of HbA. It is of two types:
urobilinogen is high). • ~-Thalassaemia major: HbA is less, HbF is
• Serum iron profile: Ferritin (may be high, if more.
haemosiderosis). iron, TIBC. • ~-Thalassaemia minor: HbA2 is increased.
2. c-Thalassaemia: In this case, there is an inade-
quate production of a-chain, therefore less HbA,
HbF and HbA2 as all of them contain a-chain.
3. Thalassaemia intermedia, characterized by the
combination of homozygous mild ~-thalassaemia
plus c-thalassaemia. It is usually less severe and
does not require blood transfusion as anaemia is
mild-to-moderate.
Q: How to treat ~-thalassaemia major?
A: As follows:
1. Correction of anaemia: Blood transfusion
to keep Hb% above 10 g% every 4 months
(lifespan ofRBC is 4 months).
2. Folic acid 5 mg daily, to be continued.
X-ray skull of ~-thalassaemia major 3. Iron-containing drugs and diet are avoided (iron
can only be given if there is deficiency).
4. Repeated blood transfusion may cause haemo-
Causes of the microcytic hypochromic blood siderosis, which can be prevented by chelating
picture agent, desferrioxamine subcutaneously with
infusion pump overnight. Ascorbic acid 200 mg
• "on-deficiency anaemia (the commonest cause)-I
daily may be added (it causes urinary excre-
• ~-Thalassaemia (major and minor).
tion of iron). Oral iron-chelating agents such as
• Sideroblastic anaemia.
deferiprone or deferasirox may be used.
• Anaemia of chronic disease.
• Others treatment: Injection erythropoietin.
It stimulates the bone marrow, increases
Q: What are the findings in ~-thalassaemia in normal haemoglobin to some extent.
Hb-electrophoresist • Hydroxyurea 1-2 g daily may be helpful (it
A: As follows: prevents ineffective erythropoiesis).
• ~-Thalassaemia major: Hb-F is more and Hb-A is 5. Specific therapy: Allogenic bone marrow trans-
less. plantation from HLA-compatibJe sibling. Also,
• ~-Thalassaemia minor: Hb-A2 is more. gene therapy.
6. Splenectomy.
Q: What are the radiological findings in skull in 7. Genetic counseling should be offered. It is nec-
~-thalassaemia major? essary for prenatal diagnosis that is available.
A: As follows:
Indication of splenectomy:
• Widening of diploic space.
• Huge splenomegaly with pressure symptoms.
• Thinning of outer table.
• Hypersplenism: As suggested by repeated transfu-
• Thickening and coarsening of trabeculae, giving
sion in a short interval. FBCshows pancytopaenia.
rise to hair-on-end appearance.
Read the following in relation to thalassaemia: N.B. Patient with mild thalassaemia (~-thalassaemia
minor or c-thalassaemia trait) requires no
Q: What is thalassaemia?
treatment. Only avoid iron therapy.
A: It is an inherited disorder in which there is impair-
ment of haemoglobin production due to partial or Q: What are the complications of repeated blood
complete failure to synthesize the specific type of transfusion?
globin chain. It is of two types: A: As follows:
5 • HAEMATOLOGY _

• Repeated transfusion may cause haernosiderosis In adults, HbF is <1%. But in thalassaemia major,
(usually when more than 30-50 L of blood is HbF is increased.
transfused) .
• Infections such as hepatitis B, C, D and HIV. Q: How can it be diagnosed before birth?

Q: How haernosiderosis can be prevented?


A: Prenatal diagnosis is possible by obtaining
A: Haernosiderosis can be prevented by using chelat- chorionic villus material for DNA. It should be done
ing agent desferrioxarnine (1.5-2 g with each unit if both parents suffer from ~-thalassaemia minor. If
of blood). It is usually given subcutaneously in the ~-thalassaemia is found in the fetus, then termina-
anterior abdominal wall with infusion pump for 12 tion of pregnancy is indicated.
h. It may also be given with infusion drip (normal
Q: What are the presentations of thalassaemia
minor?
saline or aqua). Oral chelating agent such as defer-
What is the differential diagnosis?
iprone, 75 mg/kg in two-to-four divided doses, is
also available. Other oral chelating agent includes A: As follows:
deferasirox. Vitamin C, 200 mg daily orally also • May be asymptomatic.
helps in iron excretion. • There may be features of anaemia (microcytic
hypochromic).
Q: If the patient develops severe abdominal pain, what
is the likely cause? • Incidentally during blood count, microcytic
hypochromic blood picture.
A: Cholelithiasis (usually pigment stone, due to hae-
molysis). There may also be splenic infarction. • Haemoglobin electrophoresis shows high A2.
Acute pancreatitis may also occur. Thalassaemia minor confuses with iron-deficiency
Q: When does anaemia develops in a patient with anaemia. However, anaemia is more marked in
thalassaemia major after delivery? iron deficiency and relatively less in thalassaemia
A: Anaemia develops at the age of 4-6 months. In a nor- minor. Also, in iron deficiency, there is low iron, low
mal person, HbF disappears 4-6 months after birth. ferritin and high total iron-binding capacity.

Splenomegaly (Chronic Myeloid Leukaemia)


Usual instructions are: Philadelphia chromosome. It is common in 40-60
• Examine the abdomen. years, and peak is 55 years.
• Palpate the abdomen. Q: What are the myeloproliferative disorder?
A: There are four diseases:
Presentation of a Case • Chronic myelocytic leukaemia.
• Myelofibrosis.
(Patient is usually middle-aged or above 40 years.) • Polycythaernia rubra vera.
• Spleen is hugely enlarged, ... em from costal mar-
• Essential thrombocythaernia.
gin in anterior axillary line towards right iliac fossa. These disorders are grouped together because the dis-
ease may evolve from one form to another. All myelo-
Q: What are the causes of splenomegaly in this proliferative disorders may progress to acute myeloid
middle-aged patient? leukaemia.
A: As follows: Q: What is the common age for CMU
• Malaria. A: Usually 30-80 years, and peak is 55 years.
• Kala-azar.
Q: How does the patient usually present?
• Chronic myeloid leukaemia (CML).
A: As follows:
• Lymphoma.
• Maybe asymptomatic (25%).
• CLD with portal hypertension
• Splenomegaly (90%, may be huge in 10%).
• Myelofibrosis (if the patient is elderly).
Patient may complain of mass or discomfort, or
• Tropical splenomegaly syndrome.
heaviness or pain in left hypochondrium.
Q: What is the eML? • Unexplained anaemia.
A: It is a myeloproliferative disorder characterized by • General features of weakness, malaise, loss of
overproduction of myeloid cells and presence of weight and night sweating.
__ SHORT CASES IN CLINICAL MEDICINE

• Repeated infection.
• Bleeding (due to thrornbocytopaenia).
• Hepatomegaly (in 50% cases).

Q: Mention one investigation that will help in diagno-


sis or exclude other diagnosis.
A: FBC with PBF. It will help:
• To diagnose CML.
• Myelofibrosis (it will show leukoerythroblastic
blood picture with tear- or pear-drop
poikilocytes ).
• Pancytopaenia (due to hypersplenism).
• To diagnose malaria.
• To diagnose kala-azar: It shows leucopenia with
Chronic myeloid leukaemia in blastic transformation
high lymphocyte and monocyte; repeated blood (myeloblasts are seen)
count shows progressive leucopenia.
Q: What are the clinical phases or types of CML?
Q: What investigations do you suggest?
A: There are three phases:
A: As follows:
• Chronic phase.
1. FBC:
• Accelerated phase.
• Leucocytosis (may be very high).
• Blastic crisis.
• Differential count (DC) shows increase in
myelocyte, promyelocyte. metamyelocyte,
Q: What are the causes of death in eMU
myeloblast, 10% increase in neutrophil and
A: As follows:
also basophil and eosinophil.
• Blastic crisis.
• Platelets are increased.
• Secondary infection.
• Nucleated red cells are common.
• Myelofibrosis.
2. Bone marrow study (hypercellular marrow with
increased myeloid precursors). Cytogenetic Q: What is blastic crisis? How can you suspect blastic
analysis for Philadelphia chromosome; also crisis clinically'?
RNA analysis to see the presence of BCR-ABL A: It means the disease is transformed to acute leukae-
gene product. mia. It may be myeloid (70%) or lymphatic type
3. Other tests: (30%); and it occurs at a rate of 10%/year, relatively
• Philadelphia chromosome (positive in 95% refractory to treatment and is the cause of death in
cases). majority of cases. Prognosis is poor in myeloid type.
• LAPscore (decreases). Blastic crisis in CML can be suspected, if:
• Serum uric acid (increases).
• Serum vitamin BI2(increases). • Rapid deterioration of the patient.
• Serum LDH (increases). • Increasing splenomegaly.
• Blood picture shows increase in number of blast
cells and increasing basophil.

Q: How to treat CMU


A: Treatment depends on the phase of the disease such
as chronic phase, and accelerated or blastic crisis.
1. Treatment of chronic phase:
• Imatinib is the first-line therapy. It shows
76% (95% in some cases) response with
disappearance of Philadelphia chromosome
after 18 months. It is a tyrosine kinase
inhibitor that acts by blocking the enzymatic
Chronic myeloid leukaemia (myelocytes and action of BCR-ABL fusion protein. It reduces
promyelocytes are seen) uncontrolled proliferation of white blood
cells (WBC). Dose is 400 mg daily. In some • Busulphan controls leucocyte quickly, but
cases, 600-800 mg may be given to overcome there is greater risk of marrow depression and
the resistance. It can be continued indefinitely. rarely interstitial fibrosis of lung (Busulphan
• If failure to respond to imatinib, second- lung). It also causes increased pigmentation.
generation tyrosine kinase inhibitor such as • Hydroxyurea is preferred than busulphan;
dasatinib or nilotinib or allogenic bone mar- but controls leucocyte countless quickly than
row transplantation should be considered. busulphan; and bone marrow depression is
• Alternately, hydroxyurea or a-interferon that also less.
were previously used for initial control are • None of these drugs affect the onset of blast
still useful. However, hydroxyurea does not transformation or diminishes Philadelphia
diminish Philadelphia chromosome or affect chromosome and little effect on survival.
blastic crisis. a-Interferon was given alone • Bone marrow transplantation is indicated in
or with the chemotherapeutic agent Ara-C. whom the disease is not well controlled, in
It controls CML in chronic phase in about whom the disease progress after initial control
70% of patients and causes disappearance of or for those who have accelerated phase disease.
Philadelphia chromosome in 20% cases. It
was a first-line drug before imatinib. Q: What are the therapies that may cure CMU
• Busulphan is used previously. Not used now- A: Bone marrow transplantation (BMT), imatinib and
a-days. a-interferon.
• Bone marrow transplantation (BMT) from
allogenic-matched sibling donor (usually
Q: What is the prognosis of CML?
below the age of 40 years and in early chronic
A: As follows:
phase); 70% cure.
• With imatinib therapy, complete haematological
2. Treatment of accelerated phase and blastic remission in up to 95% cases; and 70-80% of
crisis: these have no detectable BCR-ABL transcript.
• Treatment is difficult; imatinib is indicated if Event-free and overall survival appears to be
me patient has not received it. better.
• Hydroxyurea (hydroxycarbamide) can be • Following stem cell transplantation, there is 70%
effective. cure in chronic phase in young patients.
• Low-dose cytarabine can be given. • Without treatment the median survival is 3-4
years; some may survive up to 10 years.
N.B. Remember the following points regarding chem-
otherapeutic drugs that were previously used: • if there is blastic crisis, prognosis is poor. Median
• Both busulphan and hydroxyurea: Of lessor survival is 6 months.
no use now. • CML may transform to myelofibrosis.

Splenomegaly (Myelofibrosis)

Usual instructions are: Q: What are the causes of splenomegaly in this


• Examine the abdomen. middle-aged patient?
• Palpate the abdomen. A: As follows:
• Malaria.
Presentation of a Case • Kala-azar.
• Myelofibrosis.
(The patient is usually middle-aged or elderly.)
• Spleen is hugely enlarged, ... em from costal margin • CML.
in anterior axillary line towards right iliac fossa. • Lymphoma.
• CLD with portal hypertension
__ SHORT CASES IN CLINICAL MEDICINE

Q: What is myelofibrosis and how does the patient


usually present?
A: Myelofibrosis is a disorder of unknown cause char-
acterized by bone marrow fibrosis, extramedullary
haemopoiesis and leucoerythroblastic blood pic-
ture. There is clonal proliferation of the stem cells.
Fibrosis in the marrow is due to hyperpJasia of
abnormal megakaryocyte, which releases fibroblast
stimulating factors. Tear-drop poikilocyte in myelofibrosis
It is common above 50 years. History of poly-
Q: How to treat of myelofibrosis?
cythaemia rubra vera is found in 25% cases; and 50%
have JAK-2 mutation seen in polycythaemia rubra A: As follows:
vera (PRV). Features are: • Correction of anaemia by blood transfusion and
• May be asymptomatic. Mass in the left hypo- folic acid.
chondrium or hepatosplenomegaly. • Hydroxyurea (it reduces WBCand splenomegaly).
• General features of malaise, weakness, loss of • Radiotherapy for huge spleen.
weight, night sweat, repeated infection and • Splenectomy, if huge spleen with pressure
bleeding. symptoms and hypersplenism.
• There may be peptic ulcer, pruritus after hot bath • Bone marrow transplantation (if the patient is
and gout. young).
• A new drug called ruxolitinib (inhibitor of JAK-l
Q: What investigations do you suggest? and JAK-2) may be used in some patients.
A: As follows:
Prognosis: Median survival is 4 years (ranges from
1. FBC and PBP examination
1 to 20 years).
• Leucocytosis, leucoerythroblastic blood
picture (immature nucleated RBC) and Q: What are the causes of death in myelofibrosis?
premature cells ofWBC series (myelocytes A: As follows:
and myeloblast). • Transformation to AML (10-20%).
• PBPshows tear-drop RBCs (tear-drop poikilo- • Infection.
cytes). • Bleeding (from GIT).
• Platelets are very high, and later on decreased. • Cardiovascular problem.
Giant forms may be seen.
• Anaemia is usually macrocytic, and pancyto- Q: How to differentiate myelofibrosis from CML?
paenia may occur. A: As follows:
2. Bone marrow may be dry tap; trephine biopsy • Marrow finding.
should be done (which shows increased meg- • Philadelphia chromosome (absent in
akaryocyte, increased reticulin and fibrous myelofibrosis) .
tissue). • LAPscore (increases in myelofibrosis and
3. Others: leucocyte alkaline phosphatase (LAP) decreases in CML).
score is increased, uric acid is high and folic acid • In CML,WBC count is very high with precursors of
is low; genetic test may show JAK-2 mutation. granulocytes, which are absent in myelofibrosis.

Iron-deficiency Anaemia
Usual instructions by the examiner: My diagnosis is anaemia.
• Perform the general examination. Q: What is the commonest anaemia?
A: Iron-deficiency anaemia.
Presentation of a Case Q: What simple investigation is done to diagnose
iron-deficiency anaemia?
• The patient is pale, moderately anaemic. A: Peripheral blood film (PBF) (microcytic, hypochro-
mic blood picture).
Q: What history would you take in iron-deficiency
anaemia? Causes ot microcytic hypochromic blood
A: As follows: picture
• Bleeding from any site (haemorrhoid, haemate- • Iron-deficiency anaemia (the commone~s).
mesis, melaena, gum bleeding, any bleeding • Thalassaemia.
disorder, injury). • Sideroblastic anaemia.
• Tnfemale (menorrhagia and repeated pregnancy). • Anaemia of chronic disorder.
• Drugs (NSAID in patient with arthritis).
• History of malabsorption.
Q: What is the daily requirement of iron?
• Less intake of food (anorexia, dysphagia and
A: Daily requirement of iron is 1.0 mg/day. But in
poverty).
pregnancy, extra 2.5 mg/day is needed (hence total
• Chronic illness (e.g. malignancy).
3.5 mg/day is required).
Q: Tell one peculiar symptom in iron-deficiency
Q: How to treat iron-deficiency anaemia? How long
anaemia.
will you continue the treatment?
A: Pica-it means eating of unusual items such as
A: If there is severe anaemia or haemoglobin is low,
earth, coal. ice or some foods in excess like tomato,
anaemia should be corrected by blood transfu-
sour foods. The cause is unknown.
sion. Iron therapy: Oral ferrous sulphate (200 mg
8 hourly), ferrous gluconate (300 mg 12 hourly) or
ferrous fumarate. To be given for 3-6 months after
Causes of iron-deficiency anaemia: heamoglobin is normal to replenish the iron store.
• Bleeding due to any cause: The commonest Delayed-release preparation of iron is not helpful,
from gastrointestinal tract (GTT) (haemorrhoid, as they release iron beyond the upper small intes-
colorectal carcinoma, Ca-stomach, diverticulitis, tine, where it cannot be absorbed. If the patient is
angiodysplasia.), menorrhagia in female. unable to take orally, sorbitol 1.5 mg/kg injection
• Hookworm (also schistosomiasis). intramuscularly (1M) may be given daily. It can
• Less intake of food. cause skin pigmentation. Treatment of cause should
• Malabsorption. be done (e.g. menorrhagia, haemorrhoid, etc.).
• More demand (pregnancy). Q: How to see the response to iron therapy?
A: Increase in reticulocytes after 1 week.

Q: What investigations are done in iron-deficiency


anaemia?
Related Questions-Answers about
A: As follows: Anaemia
1. Test to confirm iron-deficiency anaemia
Q: Classify anaemia.
• Full blood count with PBF (microcytic
A: Anaemia may be classified in two ways: Aetiologi-
hypochromic blood picture).
cal (based on cause) and morphological (based on
• Serum iron, TIBC and ferritin (low iron,
increased TIBC and low ferritin). morphology of RBC).
2. Test to find out the causes Aetiological:
• Stool for ova or cyst of hookworm, and occult 1. Haemorrhagic anaemia (due to blood loss):
blood test. • Acute: Trauma, postpartum bleeding, hae-
• Upper gastrointestinal (GI) tract endoscopy maternesis, melaena, epistaxis.
(oesophageal varices, peptic ulcer and carci- • Chronic: Hook worms, haemorrhoids, exces-
noma stomach). sive menstrual loss, bleeding peptic ulcer, etc.
• Proctoscopy (haemorrhoid), sigmoidoscopy 2. Dyshaemopoietic anaemia (due to inadequate
or colonoscopy (neoplasm, polyp, diverticu- production of RBC):
lum, ulcer, angiodysplasia of colon). • Deficiency anaemia: Iron, vitamin B12, folate
• USG of abdomen (any mass, fibroid uterus). deficiency.
3. Bone marrow to see stainable iron (by Prussian • Aplastic anaemia (bone marrow failure,
blue shows empty stain); not a routine, may be which may be primary or secondary to some
done in some cases. other diseases or drugs).
__ SHORT CASES IN CLINICAL MEDICINE

• Anaemia of chronic disorder (ACD): SLE, Q: What is dimorphic anaemia? What are the causes?
rheumatoid arthritis, CRF. A: When both microcytes and macrocytes are found,
• Others: Hypothyroidism, sideroblastic anae- this is called dimorphic anaemia. Causes are:
mia, malignancy. • Combined iron, B'2 and folate deficiency.
3. Haemolytic anaemia: • Sideroblastic anaemia.
• Genetic: Red-cell-membrane defect (e.g. • Treatment of anaemia.
hereditary spherocytosis, eliptocytosis,
Q: What are the causes or mechanisms of anaemia of
stomatocytosis), haemoglobin abnormality
chronic disorder?
(thalassaemia, sickle-cell anaemia) or enzyme
A: Actual mechanism is unknown. It is due to abnor-
defects (gJucose-6-phosphate dehydrogenase
mality of iron metabolism and erythropoiesis. There
deficiency, pyruvate kinase deficiency).
is less erythropoietin. Also, red cell survival is short,
• Acquired: Autoimmune, toxic, mechanical
and infectious causes. Q: What are the signs that may point to a specific cause
MorphologicaJ (depending on mean corpuscular of anaemia?
volume (MCV) and mean corpuscular haemoglobin A: As follows:
concentration (MCHC):
l. Normocytic normochromic anaemia (normal Sign Cause of anaemia

MCV and MCHC). Triad of anaemia, jaundice Haemolytic anaemia


2. Microcytic hypochromic anaemia (low MCV and splenomegaly
<76 fl, low MCHC <30 g/dL). Angular cheilitis, glossitis, Iron-deficiency anaemia
3. Macrocytic anaemia (high MCV >96 fl). koilonychia
4. Dimorphic anaemia (two-cell lines: Macrocytes
Glossitis Iron-deficiency anaemia,
and microcytes).
vitamin B'2 deficiency, folate
deficiency
Q: What are the causes of normocytic normochromic
anaemia? Splenomegaly Malaria, chronic haemolytic
A: As follows: anaemia, acute infection,
• Anaemia of chronic disorder. leukaemia, lymphoma, portal
hypertension
• Chronic infection (e.g. tuberculosis).
• Collagen disease (e.g. SLE, RA). Neurological changes Vitamin B12 deficiency (mega-
• Malignancy. (dementia, optic atrophy loblastic anaemia).
• Endocrine disease. and features of subacute
• Sideroblastic anaemia. combined degeneration
of spinal cord) and lemon-
Q: What are the causes of microcytic hypochromic yellow tint.
anaemia? Bony change (frontal and Hereditary haemolytic
A: (See above). parietal bossing) anaemia

Leg ulcer Sickle-cell anaemia, paroxys-


Q: What are the causes of macrocytic anaemia?
mal nocturnal haemoglobi-
A: As follows:
nuria (PNH)
1. Macrocytosis with megaloblastic marrow are
found in: Bony tenderness Acute leukaemia, multi-
• Vitamin B'2 deficiency. ple myeloma, lymphoma,
myelofibrosis
• Folic acid deficiency.
2. Macrocytosis with normoblastic marrow are
found in: Q: What is spurious anaemia?
• Chronic liver disease. A: When plasma volume is increased and haemoglobin
• Chronic alcoholism. is relatively low, it is called spurious anaemia. It is
• Hypothyroidism. found in pregnancy.
• Haemorrhage.
• Haemolysis. Q: What is spurious polycythaemia?
• Others: Sideroblastic anaemia, pure red cell A: Here haemoglobin is relatively increased due to low
aplasia, azathioprine therapy. plasma volume. This is found in dehydration.
5 • HAEMATOLOGY __

Q: How to investigate a patient with anaemia? Further investigations for macrocytic anaemia
A: Detailed history, physical examination and finally (high MeV):
relevant laboratory investigations are done to inves- • Bone marrow is done. If megaloblast seen, serum
tigate a patient with anaemia. B'2 and folic acid assay should be done. If nor-
History of the patient: moblast is found, further investigation should be
• Dietary history (to diagnose deficiency anaemia done according to history (see above).
such as iron, vitamin B12and folic acid deficiency).
Q: What diseases may be diagnosed from PBF?
• Malabsorption.
• Any history of bleeding (haemorrhoid, A: As follows:
epistaxis, haernaternesis, melaena, menorrhagia
in female, etc).
Finding Description Common diagnoses
• In female: Multiple pregnancies, repeated abor-
tion. Anisocytosis Variation in size of Iron-deficiencyanae-
• Drug history: NSAJDs, steroid, drugs causing RBC mia, megaloblastic
bone marrow suppression (e.g. cytotoxic drugs); anaemia, sideroblas-
drugs causing haemolysis (e.g. sulfasalazine, tic anaemia

methyldopa, etc.). Poikilocytosis Variation in shape lron-deficiencyanae-


• History of surgery: Gastrectomy or partial gastrec- ofRBC rnla, thalassaemia,
tomy, ileal surgery (responsible for vitamin B12 sideroblastic anaemia
absorption) . Microcytosis MCV <76 fL Iron-deficiency
• family history (in case of hereditary haemolytic anaemia, thalas-
anaemia). saemia, sideroblastic
• History of any chronic disease (e.g. SLE,CRf, etc). anaemia, anaemia of
chronic disorder
Clinical examination: See page 254.
Macrocytosis MCV > 100 fL Vitamin B'2and
Laboratory investigations: folic acid deficiency,
1. Fu]J blood count (Hb%, TC, DC, ESR, platelet chronic liver disease,
count): alcohol, hypothy-
• Pancytopaenia: May be due to aplastic anae- roidism, zidovudine
mia, hypersplenism, megaloblastic anaemia,
Hypochromia Central pallor of RBC Iron-deficiency
aleukaemic leukaemia. is increased in size anaemia, thalas-
2. PBF examination: Following findings may be (lower haemoglobin saemia, sideroblastic
found that indicate particular causes of anaemia content) anaemia, anaemia of
(see in the table on the right). chronic disorder
3. Reticulocyte count: High in haemolytic anaemia. Basophilic Deep-blue dots Chronic lead
4. MCV and MCHC. stippling or scattered in cyto- poisoning,
5. Bone marrow examination: Megaloblastic punctate plasm of RBC(seen dyshaemopoiesis
anaemia, aplastic anaemia, bone marrow infil- basophilia with Romanowsky
tration (secondary deposit), ring sideroblasts staining)
(in sideroblastic anaemia). Target cells Flat red cells with Iron-deflciency
G. Other investigations according to suspicion of a central mass of anaemia,
cause. haemoglobin (dense thalassaemia,

Further investigation of microcytic hypochromic area) surrounded by haemoglobin C


anaemia (low MCV and low MCHC): a ring of pallor (pale disease, CLD,
area) and an outer splenectomy
• For iron-deficiency anaemia: Serum iron, TIBC,
ring of haemoglobin
serum ferritin.
(dense area)
• For hereditary haemolytic anaemia: Haerno-
globin electrophoresis, skeletal survey. Howell-Jolly Small, round rem- Hyposplenism,
bodies nants of nuclear postsplenectomy,
• For sideroblastic anaemia: According to history,
material in RBC. dyshaemopoiesis
bone marrow examination (ring sideroblasts)
These are normally
• For anaemia of chronic disease: According to
removed by spleen.
history of the patient.
SHQRT CASES IN CLINICAL MEDICINE

Heinz bodies Formed from dena- Thalassaemia, Q: What is sideroblastic anaemia?


(Ehrlich tured, aggregated haemolysis in A: Sideroblastic anaemias are inherited or acquired
bodies) haemoglobin in RBCs glucose-6-phosphate disorders characterized by refractory anaemia, a var-
(seen with supravital dehydrogenase iable number of hypochromic cells in the peripheral
staining with brilliant deficiency, asplenia blood, and excess iron and ring sideroblasts. Blood
cresyl blue). and CLD,drug-like film usually shows microcytic anaemia; it may be
sulfasalazine, dimorphic.
dapsone
Classification:
Acanthocytes RBCwith irregular Abetalipoproteinaemia
(spur cells) spicules 1. Hereditary:
Burr cells RBCwith regularly CRF
a. X-linked disease, transmitted by female.
placed spicules 2. Acquired:
Schistocytes Fragmented RBC. Causes: DIC,
a. Primary (one of the myelodysplastic
These are found in haemolytic uraemic syndromes).
microangiopathic syndrome (HUS),TTP, b. Secondary:
haemolytic anaemia. disseminated carcino- • Inflammatory: Rheumatoid arthritis.
matosis, malignant or • Neoplastic: Lymphoma, leukaemia,
pregnancy-induced carcinoma, myeloproliferative disorders,
hypertension multiple myeloma, carcinoma.
(eclampsia) • Drugs: Isoniazid (INH), pyrazinamide,
Spherocytes Small, densely Hereditary sphere- ciclosporine,
packed RBCwith loss cytosls, autoimmune • Alcohol abuse.
of central pallor haemolytic anaemia, • Lead poisoning.
postsplenectomy • Other disorders, e.g. megaloblastic and
Sickle cell Sickle shaped Sickle-cell anaemia haemolytic anaemias, malabsorption,
Blister cells Glucose-6-phosphate severe malnutrition, erythroleukaemia.
dehydrogenase
Treatment:
deficiency
Nucleated
• Treatment of the primary cause, if present,
Normoblasts Bone marrow infiltra-
RB( tion, severe haemo-
e.g. withdrawal of drug, stop alcohol intake, etc.
lysis, myelofibrosis, • Some cases may respond to pyridoxine, folic acid.
acute haemorrhage • Correction of anaemia by blood transfusion.
Polychroma- Young red cells, Haemolysis, acute Q: What is ring sideroblast?
tia reticulocytes (bluish haemorrhage, A: It is characterized by accumulation of iron in mito-
tinge) increased red-cell chondria of erythroblast around the nucleus, giving
turnover
a ring-shaped appearance in the bone marrow.
,

I
CHAPTER 6

ENDOCRINOLOGY
'You see, but you do not observe"
- Sir Arthur Conan Doyle

Introduction
It is quite common that one may get a case related e.g. signs of thyrotoxicosis, signs of hypothyroidism,
to endocrine disorders, more frequently than thyroid examination of the eye, thyroid gland, etc.
disorders; although other diseases are also not rare. • If your diagnosis is Cushing syndrome, examine
Remember, the diagnosis of endocrine disease may be other findings in relation to this (central obesity,
obvious at your first observation of the patient. Hence, striae, proximal myopathy and blood pressure).
take a few seconds to have a quick look at the patient from • If acromegaly is suspected, then examine the face,
head to foot carefully. Diagnosis of thyroid disorders hand, visual field, voice.
(Graves disease and hypothyroidism) lies on the face After an obvious diagnosis, the examiner may ask,
of the patient. Also, Cushing syndrome and acromegaly What is the likely diagnosis? Ask some questions to the
are easily diagnosed by looking at the patient. Many patient'.
a times, it is frequently asked, 'Look at the face. What In such a case, you must have a few readymade ques-
is your diagnosis? What else do you want to examine?' tions to be asked. For example:
Underlying diagnoses by looking at the face may be:
• If Cushing syndrome is suspected, ask whether the
• Graves disease (hyperthyroid, euthyroid or patient is taking any steroid or experiencing weight
hypothyroid). gain, backache and difficulty in standing up after
• Hypothyroidism (myxoedema). sitting.
• Cushing syndrome. • If hypothyroidism is suspected, ask whether the
• Acromegaly. patient prefers warmth and has weight gain,
increased sleep, constipation and so on.
Subsequent physical examination depends on your
• If hyperthyroidism is suspected, ask whether the
diagnosis. For example:
patient has preference for cold and has excessive
• lf your diagnosis is thyroid disease, further clinical sweating increased appetite but weight loss,
examination will be related to thyroid problems, irritability and so on.

Examination of Thyroid Gland


The usual instructions are: • Look at the face (hypothyroid and exophthalmos).
What else do you want to examine?
• Examine the thyroid gland or the neck.
• Examine the thyroid status of the patient (you Proceed as follows:
must see signs of thyrotoxicosis and also signs of l. Introduce yourself. While handshaking, check
hypothyroidism) . whether the hand is warm and sweaty.
_I SHORT CASES IN CLINICAL

2. Look at the patient's face:


MEDICINE

Finally, ask the examiner, 'I want to examine the thy-


• Anxious, restless, fidgety and frightened face roid status, eyes, pretibial myxoedema and heart'.
with staring looks (thyrotoxicosis). For thyroid status, see the signs of thyrotoxicosis
• Puffy face with periorbital swelling with baggy and signs of hypothyroidism.
eyelids, loss of outer one-third of eyebrows and
1. Signs of thyrotoxicosis:
apathetic look (hypothyroidism).
• Observe tremor of outstretched hands with
50 • Exophthalmos (bilateral or unilateral: Graves
o fingers spread out. If no tremor is noted, put a
"0 disease).
paper over the fingers and note the tremor .
.S
... 3. Then look (inspection), palpate and auscultate
u • Palm: Warm and sweaty (in anxiety-palm is
o (thyroid).
-0 cold and sweaty).
c:::
W a. Inspection: Examine the neck both in front, left • Pulse:Tachycardia (better to record sleeping pulse).
and right sides. • Others: Generalized wasting, proximal myopathy
• Obvious swelling (goitre): Ask the patient and jerks exaggerated.
for deglutition (ideally patient should drink
a glass of water). If any goitre, check whether N.B. There may be atrial fibrillation (AP), ectopics
it is diffuse, single or multinodular; and then and high-volume pulse in thyrotoxicosis.
check if one side is larger than the other.
• Skin colour (redness indicates suppurative 2. Signs of hypothyroid2ism:
thyroiditis) and any scar of surgery. • Puffy face with periorbital swelling with baggy
• Other obvious swelling (lymphadenopathy). eyelids, loss of outer one-third of eyebrows and
b. Palpation (from back): Ask the patient to sit, apathetic look.
ensure the neck is slightly flexed, palpate the • Nonpitting oedema.
gland with both hands and ask the patient to • Pulse: Bradycardia.
swallow again. • Speech: Hoarse, croaky or husky voice.
If goitre is present, see whether it is unilateral • Dry and thick skin.
or bilateral and diffuse or nodular (multinodu- • Slow relaxation of ankle jerk.
lar or single); then check the following points: Q: What are the findings in the eyes? What are the
• Size (which one is large, right or left). other physical findings to be seen?
• Shape (diffuse or irregular). A: Exophthalmos, lid lag, lid retraction and eye move-
• Surface (smooth or irregular). ment (see exophthalmos on page 455).
• Consistency (soft or firm or hard).
• Tenderness (if present, indicates subacute Other physical findings:
thyroiditis and suppurative thyroiditis]. • Legs:Pretibial myxoedema (present only in Graves
• Mobility. disease), reflex(may be exaggeratedin thyrotoxicosis
• Overlying skin condition. and slow relaxation of ankle jerkin hypothyroidism).
• Lower margin (to see retrosternal extension). • Proximal myopathy (both upper and lower
c. Examine for retrosternal extension (prominent limbs) found in thyrotoxicosis (may be found in
veins in neck and upper chest; feel the lower hypothyroidism also).
limit, tracheal deviation and percuss the upper • Nails: Onycholysis in Graves disease.
part of chest for dullness. For Pemberton sign, Q~ Whal is Pemberton sign?
see below). A: It is a sign to see retrosternal extension of thyroid
4. Palpate the cervical lymph nodes (if palpable, sug- gland (also positive with any retrosternal mass). On
gest metastasis). raising both the arms above the head, the patient
5. Feel for carotid pulse (absence of pulsation indi- with retrosternal extension may develop the follow-
cates malignant infiltration). ing signs of compression:

6. Finally, ask the patient to hold breath and auscultate • Face: Suffusion or congestion and cyanosis.
for thyroid bruit (also carotid bruit and venous hum, • Respiratory distress (stridor).
which can be obliterated by gentle pressure at the root • Neck veins are engorged.
of neck). If thyroid bruit is present, check whether it • The patient may complain of dizziness and may
is actually a murmur of aortic stenosis [ejection sys- be syncope. (This sign is rare, not specific for
tolic murmur (ESM)] radiating from chest. retrosternal goitre.)
6 • ENDOCRINOLOGY _

lid lag

Onycholysis in Graves disease

N.B. Remember the following points:


• Eye signs (exophthalmos, periorbital oedema,
chemosis and diplopia), pretibial myxoedema
and thyroid acropachy are present only in Graves
disease. However, lid lag and lid retraction occurs
lid retraction in thyrotoxicosis due to sympathetic overactivity.
which supplies levator palpebrae muscle.
• Thyroid bruit present in Graves disease.
• Other findings in hands in thyrotoxicosis include
palmar erythema, onycholysis (separation of
nails from bed called Plummer sign is present
only in Graves disease) and clubbing (in case of
thyroid acropachy-only in Graves).
• Exophthalmos and pretibial myxoedema do not
correlate with toxicosis. It may be present both in
hyper- or hypo- or euthyroid state associated with
Pretibial myxoedema Graves disease.

Face in Thyroid Disease (by Looking at the Face)


The usual instructions are: Diagnosis is Graves disease with thyrotoxicosis.

• Look at the face. What is your diagnosis? What else


do you want to see? (Examine the face and also
neck to see any goitre-whether diffuse or nodular.
Examine from head to foot and observe generalized
swelling of the body and nonpitting oedema.)

Presentation of a Case
{Exophthalmos }:
Case No.1

There is bilateral or unilateral exophthalmos.


• The patient looks anxious, restless and fidgety.
• Thyroid gland is diffusely enlarged.
Case no. 1: Graves disease
_ SHORT CASES IN CLINICAL MEDICINE

Q: What else do you like to see?


A: I want to see the signs of thyrotoxicosis. I want to
examine the thyroid gland, eyes and legs (page 455).

Presentation of a Case
(Myxoedemo): Case No.2 (a)

• The face is puffy with periorbital swelling and


baggy eyelids, and appears apathetic.
• Outer one-third of the eyebrow is absent or
reduced
Case no. 2 (b): Graves disease (hypothyroid)
My diagnosis is myxoedema.

Presentation of a Case
(Myxoedema): Case No.3

• As in Case no. 2, plus thyroid gland is diffusely


enlarged, firm or rubbery, nontender, no bruit
and no exophthalmos.

Diagnosis is myxoedema due to Hashimoto thyroiditis.

Case no. 2 (a): Myxoedema (nongoitrous)

Q: What else do you like to see?


A: I want to see the signs of hypothyroidism. Also,
I want to examine the thyroid gland, eyes, legs, skin,
ankle jerk and nonpitting oedema. I want to talk to
the patient.

Presentation of a Case [f----------,


(Myxoedema): Case No. 2 (b)

• As in Case no. 2 (a) plus there is bilateral


exophthalmos.

My diagnosis is hypothyroid Graves disease. Case no. 3: Myxoedema (goitrous, Hashimoto thyroiditis)

Thyrotoxicosis
Instruction by the examiner:
Presentation of a Case [1----------,
• Look at the patient. What else do you want (Features of Thyrotoxicosis)
to see?
• Examine the thyroid gland and relevants. • The patient has tremor of outstretched hands.
• Palms of the hands are warm and sweaty.
• Pulse: 120/min (tachycardia). Pulse may be
normal, if the patient is on ~-blocker.
6 • ENDOCRINOLOGY _

Q: Ask a few questions. Or what history do you like to


take in thyrotoxicosis?
A: Ask the patient:
• Do you prefer hot or cold? (Heat intolerance)
• Are you losing weight? (Weight loss common)
• How is your appetite? (Increased appetite)
• How is your bowel habit? (Diarrhoea may occur)
• In females, ask about menstruation (Usually
amenorrhoea in thyrotoxicosis)
Others: Excess sweating, palpitation, tremor, irrita-
bility, insomnia and anxiousness or nervousness.

N.B. If any patient complains of loss of weight


despite good appetite, the likely diagnosis is
My diagnosis is thyrotoxicosis. thyrotoxicosis. (Other cause may be diabetes
mellitus (DM). However, loss of appetite in
Q: What else do you want to examine? thyrotoxicosis may occur in the elderly.)
A: I want to examine the thyroid gland, eye, cardiovas-
cular system and jerks.
Q: What is factitious thyrotoxicosis?
Q: What do you think the cause of thyrotoxicosis in A: Deliberate intake of T4 to reduce weight, usually in
this case? emotionally disturbed person. Clues for diagnosis
A: As follows (mention the causes of that case by look- are high thyroid hormones and low radioiodine
ing at face and neck): uptake. Thyroglobulin level is zero or low, high ratio
of T4:T3= 70: 1 (in conventional thyrotoxicosis, the
• In young patient with diffuse goitre and exoph- ratio is 30: 1). Combination of negligible radioio-
thalmos, the likely cause is Graves disease (even dine uptake, high T4:T3ratio and low thyroglobulin
if no exophthalmos, still it can be Graves disease). is diagnostic.
• In the middle-aged with nodular goitre (single
or multiple), the likely cause is toxic nodular or
toxic multinodular goitre. N.B. Remember the following points:
• The elderly may not have obvious features of
thyrotoxicosis. The patient may present only with
Causes of thyrotoxicosis atrial fibrillation, tachycardia or heart failure,
which masks thyrotoxicosis.
1. Gravesdisease:The commonest cause (76%).
• Children may present with excess growth,
2. Toxic multinodular goitre (14%). behaviour problem like hyperreactivity, and
3. Toxic nodular goitre (SOlo,toxic adenoma
increase in weight rather than loss.
or hot nodule called Plummer disease). • Apathetic thyrotoxicosis may be present in elderly
4. Thyroiditis (subacute thyroiditis; also called
patients with thyrotoxicosis, with features such
De Quervain thyroiditis, and postpartum as hypothyroidism. High degree of suspicion is
thyroiditis. All are transient). essential.
5. Hashimoto thyroiditis (ultimately hypo-
thyroidism develops).
Q: What investigations do you suggest in
6. factitious thyrotoxicosis (self-intake of
thyrotoxicosis?
thyroxine).
A: As follows:
7. Iodine induced (Jod-Basedow phenomenon)
and drug (amiodarone). 1. To confirm thyrotoxicosis:
8. Others (rare): • FT3, FT4 and TSH (low TSH, high T3 and T4.
• Carcinoma of thyroid (follicular). But in T3 toxicosis, TSH is low, T4 is normal
• Struma ovarii (secretes thyroid hormone). and T3is high).
• Hydatidiform mole and choriocarci- • Radioiodine uptake test (RAIU) and thyroid
noma [both secrete thyroid-stimulating scanning (technetium scintigraphy is better
hormone (TSH)]. than RAIUtest because it is quicker to perform
_ SHORT CASES IN CLINICAL MEDICINE

and requires lower dose of radioactivity). Indications of drug:


RAlU shows rapid uptake and rapid turnover.
There is high uptake in 2 or 4 and 24 hand • Usually given in first episode in patient <40 years
rapid fall after 48 h. of age.
• Small goitre.
2. To find out causes:
• Mild features of thyrotoxicosis.
• Ultrasonography of the neck (to see single,
1;0 multinodular and diffuse goitre). Disadvantages of drugs:
o
-0 • Thyroid autoantibody: For Graves disease • Relapse in 50% of the cases within 2 years of
.5 TSH receptor stimulating antibody. Also,
b stopping the drug (surgery or radioiodine or long-
o antiperoxidase and antithyroglobulin antibody term drug therapy may be needed in such case).
J (very high in Hashimoto thyroiditis, but slight-
to-moderate high in Graves disease).
• Complication such as hypersensitive skin rash and
agranulocytosis.
3. Other tests: • Compliance may be poor.
• ECG. • Costly.
• Chest X-ray [retrosternal extension of goitre
and cardiomegaly). N.B. Advise the patient with carbimazole therapy:
• Blood sugar (secondary DM). If sore throat or fever develops, which may
• Serum cholesterol (low in thyrotoxicosis). be due to agranulocytosis, stop the drug and
• Fine-needle aspiration cytology (FNAC) inform the doctor.
(if goitre is present).
• For ophthalmopathy (in Graves disease, see
Radioiodine therapy:
page 455).
It acts by destroying the functioning thyroid cells and
Causes of thyrotoxicosis with low inhibiting their ability to replicate. Depending on the
radioiodine uptake: size of goitre, 5 to 10 mCi is given orally. It is effective in
75% cases in 4-12 weeks. In this period, propranolol is
• Subacute thyroiditis (De Quervain thyroiditis). given. In severe cases, carbimazole may be given, which
• Postpartum thyroiditis. should be started 48 h after radioiodine therapy. If the
• Factitious thyrotoxicosis. drug is started before 48 h, it reduces the efficacy of
• Iodine induced (patient on iodine or
radioiodine.
amiodarone, which contains iodine).
• Ectopic thyroid tissue producing thyrotoxi- Indications of radioiodine therapy:
cosis (struma ovarii, choriocarcinoma and
hydatidiform mole). • Usually, above 40 years of age (however, some
advocate to use in young}.
Q: How to treat thyrotoxicosis? • Recurrence after surgery or drugs, irrespective of
A: Three modes of treatment: Drugs (carbimazole and age.
propylthiouracil), radioiodine therapy and surgery. • Toxic multinodular goitre or toxic adenoma or hot
nodule.
Drug therapy: • In early age, with major serious other illness.
1. Carbimazole or propylthiouracil: These drugs
• Some cases ofcarcinoma thyroid (follicular, papillary
reduce the synthesis of thyroid hormones by after surgery).
inhibiting the iodination of tyrosine. • Ablative therapy with severe atrial fibrillation; also
• Carbimazole 45-60 mg daily. When the in heart failure.
patient is euthyroid, reduce the dose; then
• Psychosis.
5-20 mg daily for 18-24 months. Periodic
• Poor drug compliance.
complete blood count (CBC) is necessary, as • Hypersensitivity to the drug.
there may be agranulocytosis. Also, Ff4 and
TSH should be measured. Contraindications of radioiodine therapy:
• Propylthiouracil 400-600 mg daily. Dose is
reduced when the patient becomes euthyroid. • Pregnancy or planned pregnancy within 6 months
2. ~-Blocker: Propranolol (up to ] 60 mg/day), It of treatment.
reduces sympathetic symptoms (such as tremor, • During lactation.
tachycardia and sweating). • Active or malignant Graves ophthalmopathy.
6 • ENDOCRINOLOGY __

Disadvantages of radioiodine therapy: persists up to 7-72 h. Treatment of thyrotoxicosis


improves the condition.
• Hypothyroidism: It occurs in 40% in first year and
80% in 15 years. Q: How to treat thyrotoxicosis with atrial fibrillation?
• Early discomfort and exaggeration of hyperthy- A: AF occurs in 10% cases, common in the elderly of
roidism may occur (due to radiation thyroiditis). >60 years, more in toxic multinodular goitre, and
Hence, the patient should be rendered euthyroid may occur in subclinical hyperthyroidism. It is M
l:l
using drug, which should be stopped 2-5 days treated in the following ways: 0-
o
before radioiodine therapy. ...,
()
.....
• p-Blocker: Propranolol (digoxin has little role). ;:l
o
• Exacerbation of ophthalmopathy.
Verapamil and amiodarone may be used, if 0"
p-blocker is contraindicated. ~
N.B. In severe thyrotoxicosis, initially carbimazole is • Antithyroid drug, followed by radioiodine therapy.
given for 4-8 weeks. Then radioiodine therapy • Anticoagulant: Aspirin in the elderly and warfarin
is given. Carbimazole is stopped 48 h before in younger.
radioiodine therapy. If no response after 12-24
• 50% AF reverts spontaneously to sinus rhythm.
weeks, a second dose of radioiodine is given.
If persistent, AF cardioversion may be done,
provided 1'4 and TS1-'Iare normal.
Surgery (subtotal thyroidectomy):
Q: How to treat thyrotoxicosis in pregnancy?
The patient should be made euthyroid by antithyroid
A: Pregnancy is unusual in thyrotoxicosis, as anovu-
drug before operation. Potassium iodide should be
added 60 mg 8 hourly, 2 weeks before operation. It latory cycles are common. Autoimmune thyroid
inhibits thyroid hormone release, reduces the size disease is also less common, as maternal immune
and vascularity of gland, making surgery technically response is suppressed in pregnancy. Because of
easier. high thyroxine-binding globulin (TBG), total T4and
T.3are high, and TSH is low. Hence, high FT4 and FT3,
Indications of surgery: and low TSH level suggest thyrotoxicosis. It is usu-
ally due to Graves disease. TSH receptor antibodies
• Large goitre or multinodular goitre. (TRAb) can cross placenta and develop thyrotoxico-
• Relapse or no response to drug. sis in foetus.
• Drug hypersensitivity.
• Noncompliance with drug. • Propylthiouracil is preferred (carbimazole
• Suspicion of malignancy. can cross placenta causing foetal goitre
• Pressure effect. and scalp skin defect in child called aplasia
• Cosmetic purpose. cutis). Propylthiouracil is given in lowest
dose; less than ISO mg daily to prevent foetal
Complications of surgery: hypothyroidism and goitre. TRAb is measured
in last trimester; if not high, the drug can be
• Hypothyroidism in 25%. stopped 4 weeks before delivery (to prevent
• Transient hypocalcaemia (10%).
neonatal hypothyroidism).
• Permanent hypoparathyroidism (1%).
• If needed, propylthiouracil can be given after
• Recurrent laryngeal nerve palsy causing hoarseness
delivery and breastfeeding should be continued,
ofvoice due to vocal cord palsy (1%).
as little is excreted in breast milk.
N.B. In toxic nodular or multinodular goitre, treat- • If surgery is necessary, it should be done in
ment of choice is radioiodine therapy or middle trimester.
surgery. Drug treatment is not helpful.
N.B. Radioiodine therapy is absolutely contraindi-
Q: What is thyrotoxic periodic. paralysis (TPPH cated in pregnancy.
A: If a thyrotoxic patient develops sudden or periodic
weakness, it is called TPP. It is due to hypokalae- Q: What is thyrotoxic crisis? How to treat?
mia (caused by entry of potassium into the cell), A: Thyrotoxic crisis is characterized by life-threatening
common in Asians. It may occur following excess increase of signs and symptoms of thyrotoxicosis
of carbohydrate or glucose or heavy exercise, and (also called thyroid storm).
_ SHORT CASES IN CLINICAL MEDICINE

Features of thyrotoxic crisis are: • Propranolol 80 mg 6 hourly (or 1-5 mg IV 6


hourly).
• High fever.
• Intravenous (IV) fluid (normal saline and
• Restlessness, agitation and irritability.
glucose).
• Nausea, vomiting, diarrhoea and abdominal pain.
• Tachycardia, AF and cardiac failure in elderly. • Carbimazole 40-60 mg daily. Or propyl-
• Confusion, delirium and coma. thiouracil 150 mg 6 hourly (if needed, through
~ Ryle tube). Carbimazole can be given per
o Precipitating factors for thyroid crisis:
"0 rectally also.
.S
to • Infection. • Other therapy: Sodium ipodate=-a radiographical
"0 • Stress. contrast media-SOO mg daily is rapidly effective.
~ • Surgery in unprepared patient. Or potassium iodide or Lugol iodine may be
• Following radioiodine therapy (due to radiation given. Dexamethasone 2 mg 6 hourly and
thyroiditis) . amiodarone are also effective.
Diagnosis: • Broad-spectrum antibiotic.
Mostly elinical and high degree of suspicion is vital. FI3, • General measures: Control of temperature and
FT4 and TSH should be done immediately. O2 therapy.
• After 10-14 days, the patient can usually be
Treatment (blood sample taken for T', T4and TSH):
maintained on carbimazole alone.
• The patient should be treated in intensive care
unit (ICU). Mortality rate in thyrotoxic crisis is 10%.

----- -- - -- -

Graves Disease

Usual instructions are: Q: What else do you want to examine?


A: Dermopathy (pretibial myxoedema).
• Look at the patient. What are your findings? What
else do you want to examine? Q: What is the natural history of Graves disease?
• Examine the thyroid gland. A: It may be hyperthyroid and euthyroid, followed by
(Remember, Graves disease may be hyperthyroid, hypothyroidism.
euthyroid and hypothyroid.)
Presentation of a Case :f----------,
(Exophthalmos): Case No.2
presentation of a CdS&
(Exophthalmos): Case No.1 • There is unilateral or bilateral exophthalmos
with no signs of thyrotoxicosis.
(See also page 455)
• There is unilateral or bilateral exophthalmos, per-
Diagnosis is euthyroid Graves disease.
iorbital oedema, chemosis, redness of eye, corneal
ulceration, lid lag, lid retraction, ophthalmoplegia,
and diplopia (mention if present).
• The face appears anxious, fidgety along with
staring looks.
• Thyroid gland is diffusely enlarged, firm in
consistency, nontender and mobile.
• There is bruit (mention the site).
• Signs of thyrotoxicosis are present (tachycardia,
tremor, warm and sweaty palm).

Diagnosis is thyrotoxicosis due to Graves disease.

Q: Why it is Graves disease?


A: As there is exophthalmos and diffuse goitre. Graves disease (thyrotoxicosis)
6 • ENDOCRINOLOGY _

Q: What is euthyroid Graves disease?


A: The patient is clinically and biochemically euthyroid,
but there is ophthalmopathy. [Thyroid releasing
hormone (TRH) stimulation test shows flat response
curve.]

N.B. Remember, ophthalmopathy in Graves dis-


ease:
• Eye problems occur in 5-10% of the cases.
• Ophthalmopathy occurs in 50% in the first
presentation, may develop after treatment of
hyperthyroidism and precedes many years
before hyperthyroidism.
• Not related to toxicosis. It may be present in
Graves disease (euthyroid) euthyroid or hypothyroid.
• If hypothyroidism develops, exophthalmos
may be aggravated, especially if treated with
Presentation of a Case radioiodine therapy.
(Exophthalmos): Case NO.3 • Lid retraction resolves when the patient is
euthyroid, but exophthalmos resolves slowly
• There is unilateral or bilateral exophthalmos.
and may take up to 2-3 years.
• Coarse, puffy facies, periorbital puffiness and
baggy eyelids. Patient looks immobile and Q: What is the mechanism of thyroid ophthalmopa-
uninterested. thy? What are the changes and how to treat?
• Skin is dry, cold and scaly. A: It is immunologically mediated. Within the
• Nonpitting oedema and slow relaxation of ankle orbit, there is cytokine-mediated prolifera-
jerk. tion of fibroblast, which secretes hydrophilic
• Voice: Coarse and croaky. glycosaminoglycans.
The following changes occur in ophthalmopathy:
Diagnosis is hypothyroid Craves disease.
• Excessive interstitial fluid and infiltration
Q: What is Graves disease! of chronic inflammatory cells in the orbit
A: It is an autoimmune thyroid disease due to stimu- (lymphocytes, plasma cells and mast cells).
lating antibody to TSH receptors characterized by • Swelling and oedema of extraocular muscles.
triad of: • Increased retrobulbar pressure and eyeball is
pushed forward (proptosis). In severe cases, optic
• Exophthalmos.
nerve compression may occur.
• Diffuse goitre.
• Dermopathy (pretibial myxoedema). Clinical features: Increased lacrimation, gritty
Q: What is the cause of Graves disease? sensation in the eye, pain due to conjunctivitis
A: Autoimmune disease due to IgG antibody against or corneal ulcer, reduced visual acuity and diplo-
TSH receptor, producing excess thyroid hormones. pia. It increases with poor control of thyroid
TRAb acts similar to TSH. Common in female, function and also following radioiodine therapy.
M:F= 1:5. TRAb is of two types: (l)TSI, with 80-95% Ophthalmopathy is common in cigarette-smok-
causing thyrotoxicosis and (2) TSH receptor- ing population. However, cigarette smoking is
blocking antibody causing hypothyroidism. weakly associated with Graves disease.

Q: What is the s-ignificance of thyroid bruit? What is Treatment:


the cause of bruit? .
• Reassurance and general treatment of the eye.
A: It is pathognomonic of Graves disease and rarely
Methylcellulose eye drops (relieves grittiness). To
occurs in other thyroid diseases. It indicates
prevent exposure keratitis, use tinted glass and
increased vascularity, probably due to an autoim-
lateral tarsorrhaphy (if corneal ulcer).
mune mechanism.
_ SHORT CASES IN CLINICAL MEDICINE

• If visual field defect, loss of visual acuity and Q: What is malignant exophthalmos?
papilloedema or progressive exophthalmos is A: Itis the severe, progressive exophthalmos, which may
present, steroid in high dose (prednisolone lead to blindness due to optic nerve compression.
60-120 mg/day) may be helpful. Treated with high dose of prednisolone up to
• Pulse methylprednisolone and cytotoxic drug 120 mg/day; it may require decompression or
(cyclophosphamide) may be helpful. orbital irradiation.
60 • In severe cases, irradiation of the orbit.
o • If no response in 7-10 days or loss of visual Q: What is pretibial myxoedema (dermopathy)?
-0
.S acuity, orbital decompression may be necessary . A: In this condition, there is firm, nodular, thickened
....
u
o • For diplopia: Correction of eye muscle by surgery or plaque-like lesion of pink or brown colour giving
'"0
(but should be delayed for 6 months, until degree a peau d' orange appearance. Usually, present in
~
of diplopia is stable). the shin of legs up to the dorsum of foot (but may
occur in any part of the body, especially at pressure
Q: Is there any evidence of cancer in Graves disease?
point). It may be pruritic and hyperpigrnented. It
A: Unusual, but highly suspicious if there is associated
is present only in Graves disease in 10%, almost
cold nodule. always associated with ophthalmopathy and is
not a manifestation of hypothyroidism (pretibial
Eponyms of eye signs in thyroid disease
myxoedema is a misnomer). Occasionally, pretibial
• Lid lag: Von Graefe sign. myxoedema develops after treatment of hyperthy-
• Absence of wrinkling of forehead on upward gaze: roidism, especially with radioiodine therapy. It is
Joffroy sign. due to the deposition of mucopolysaccharide in
• Impaired convergence of eye: Mobius sign. the dermis.
• Infrequent blinking: Stellwag sign.
• Paralysis of extraocular muscles: Iendrassik sign.
• Weakness of at least one extraocular muscle: Ballet
sign.

Q: What are the investigations and treatment in


Graves disease?
A: See section on 'thyrotoxicosis'.

Pfetibial myxoedema (in sMn)

Malignant exophthalmos (with chemosis)

Pretibial mYiKoedema(in dorsum oUoot anti shin)

Treatment is rarely necessary. Local injection of tri-


amcinolone or ointment betamethasone may be
Exophthalmos (less severe) helpful.
6 • ENDOCRINOLOGY __

Q: How treat Graves disease?


to • If hypothyroidism, treat accordingly with
A: Graves disease is an autoimmune disease, which thyroxin.
may present with hyper-. hypo- or euthyroid state. • In euthyroid cases, treatment is only symptomatic
and supportive.
• If there is thyrotoxicosis, medical, surgical or
radioiodine therapy (discussed in thyrotoxicosis).

Simple Multinodular Goitre


Instruction by the examiner:
• Examine the neck.
• Examine the thyroid gland and relevant.

Presentation of a Case

• There is multinodular goitre; left lobe is larger than


the right; and nodules are of variable size and shape,
nontender, firm in consistency and freely movable.
• There is no bruit, no retrosternal extension and
no cervical lymphadenopathy.
• The patient is clinically euthyroid (pulse-normal,
no tremor, and no warm and sweaty palm).

Multinodular goitre (small size)


My diagnosis is simple multinodular goitre.

Q: Why this is simple?


A: Because there are no features of thyrotoxicosis such
as no tachycardia, no tremor of outstretched hand,
and no warm and sweaty palm.

Q: What are the causes of simple multinodular goitre?


A: As follows:
• Iodine deficiency (the commonest cause).
• Drugs: Lithium, amiodarone and para-
aminosalicylate (PAS).
• Thiocyanate in diet.

Q: How to investigate a multinodular goitre?


A: As follows:
Multinodular goitre (huge and massive)
• RAIUtest. In iodine deficiency, there is high uptake
and slow turnover. Otherwise, RAIUis normal. Q: What are the complications of multinodular goitre?
• Thyroid scan. A: As follows:
• Ultrasonography (USG) of thyroid gland.
• May develop thyrotoxicosis (toxic multinodular
• Ff3' FT4 and TSH (all normal; TSH may be high, goitre).
due to iodine deficiency). • Compression: Such as dysphagia, hoarseness
• FNAC ofthyroid nodule (if cold nodule). (due to involvement of recurrent laryngeal nerve),
• In iodine deficiency, measurement of urinary stridor and superior vena cava (SVC) obstruction.
iodine (<10 mg/dl.) may be done. • Severe pain due to haemorrhage in nodule.
I

__ SHORT CASES IN CLINICAL MEDICINE

Q: How to treat? N.B. Remember the following points in multinodular


A: As follows: goitre:

• If small: Reassurance. Follow-up annually. • Role of thyroxine: Not much effective and may cause
• If large or mediastinal compression or cosmetic toxicosis. Chance of malignancy is rare; usually it is
reason: Partial thyroidectomy. benign. Malignancy may occur, if cold nodule.
• Common in middle-aged and elderly individuals.

Toxic Nodular or Multinodular Goitre


Instruction by the examiner: • USG of thyroid gland.
• Other tests (as described in thyrotoxicosis, such as
• Examine the neck. ECG, chest X-ray, antithyroid antibody and so on).
• Examine the thyroid gland and relevant.
Q: Which single test do you want to perform?
A: Single test is Ff3, as there may be only T3 toxicosis;
Presentation of in such case, FT4 and TSH may be normal or near-
Case No.1 normal.

Q: How to treat toxic nodular or multinodular goitre?


• There is a nodule in the right lobe of thyroid,
A: As follows:
which is 3 x 2 ern in size, firm in consistency,
nontender, smooth surface, freely movable, no • ~-Blocker (to reduce heart rate).
bruit and no palpable lymph node. • Radioiodine therapy is a treatment of choice (high
• Signs of toxicosis are present (tachycardia, tremor dose: 15-30 mCi). If severe toxicosis, antithyroid
of outstretched hand, warm and sweaty palm). drug is given, followed by radioiodine therapy.
• Occasionally, surgery may be necessary, if there
is large goitre. However, before surgery, patient
My diagnosis is toxic nodular goitre.
should be made euthyroid.

Presentation of N.B. Antithyroid drugs are given for short time.


Case No.2 Long-term treatment with antithyroid drug is
not helpful, as many nodules are autonomous
• Present as before (multinodular goitre).
and relapse is invariable after withdrawal of the
• Signs of toxicosis are present (tachycardia, tremor
drug.
of the outstretched hand, warm and sweaty palm).

My diagnosis is toxic multinodular goitre.

Q: What is the natural history of toxic multinodular


goitre?
A: It is permanent and there is no spontaneous remission.

Q: Bow to investigate such a case?


A: As follows:

• Ff3, FT4 and TSH (high T3 and T4, 10wTSH).


• IWU test and thyroid scan (which shows rapid Toxic multinodular goiter
uptake and rapid turnover. There is high uptake Q: What are the difference between Graves disease and
after 2 and 24 h, falls after 48 h). toxic multinodular goitre?
A: As follows:
Differences between Graves disease and toxic multinodular goitre
Features Graves disease Toxic multinodular goitre
Age 20-40 years Middle-aged or elderly
Past history Not specific Nodular goitre for long time
Thyroid gland Diffusely enlarged Multinodular goitre
Cause Autoimmune disease Usually iodine deficiency
6 • ENDOCRINOLOGY _

Differences between Graves disease and toxic multinodular goitre


Ophthalmopathy Common Rare
Dermopathy Common Absent
Heart complications Lesscommon More common
Thyroid acropachy May occur Absent
Hypothyroidism May occur Absent

Simple Diffuse Goitre


Instruction by the examiner:

• Examine the neck.


• Examine the thyroid gland and relevant.

Presentation of a Case
• Thyroid gland is diffusely enlarged, 5 x 4 em,
nontender, firm in consistency, freely mobile,
has no bruit, no retrosternal extension and no
palpable lymph node.
• There are no signs of toxicosis.

My diagnosis is simple diffuse goitre.

Q: Why is it simple? Puberty goitre (simple)


A: Because there is no sign of toxicosis.
Q: What do you think are the causes in this case? Q: What are the causes of diffuse goitre?
A; As follows: A: As follows:
• Iodine deficiency. • Physiological: Puberty and pregnancy.
• If the patient is young, it may be puberty goitre. • Iodine deficiency (endemic goitre).
• Goitrogens. • Autoimmune: Hashimoto thyroiditis, Graves
Q: What are the presentations of simple diffuse goitre? disease and postpartum thyroiditis.
A: Simple diffuse goitre is common in the second and • Goitrogens: Drugs (amiodarone, lithium, para-
third decade, 15-25 years, more in females (F:M= 3:1). aminosalicylate (PAS) and phenylbutazone).
The patient presents with swelling in the neck. Or Thiocyanate in diet (cabbage, cauliflower, turnips,
only goitre is found during routine examination. soya beans, brussels sprouts).
• Iodide in large doses.
Sometimes, goitre is evident in pregnancy.
• Subacute thyroiditis (de Quervain thyroiditis).
• Dyshormonogenesis.
• Riedel thyroiditis.
• Suppurative thyroiditis.
• Infiltrative disease: Amyloidosis and sarcoidosis.

Q: How to treat simple diffuse goitre?


A: As follows:
• Mild-to-moderate: No treatment. Reassurance and
follow-up. T4is sometimes helpful to shrink goitre.
• Treatment of primary cause.
• If large goitre is associated with pressure effect or
cosmetic reason, surgery (partial thyroidectomy)
Diffuse goitre (simple) maybe done.
~ SHORT CASES IN CLINICAL MEDICINE

N.B. Clue for the diagnosis of different types of goitre: • Diffuse and fum or rubbery hard goitre, with no
exophthalmos: Suggestive of Hashimoto thyroiditis.
• Diffuse and soft goitre, with exophthalmos:
• Diffuse and soft: Suggestive of simple diffuse goitre.
Suggestive of Craves disease.
• Diffuse, soft and tender: Suggestive of subacute
thyroiditis.

Solitary Nodule or Simple Nodular Goitre


Instruction by the examiner: 2. 5-10% of the thyroid nodules are hot nodules.
Treatment is:
• Examine the neck.
• Surgery or radioiodine therapy.
• Examine the thyroid gland and relevant.
• Ethanol injection into the nodule may be given.
Presentation of a Case :1----------. 3. Cystic nodule is usually benign. Aspiration is effec-
tive in 50% of the cases. Ethanol injection into the
• There is a solitary nodule in the right (or left nodule is helpful. Thyroxin therapy is ineffective.
lobe), 2 x 2 cm, firm in consistency, nontender. Large cyst (>4cm) or cyst with bloody fluid usually
smooth surface, freely movable, no bruit and no recur and surgical excision should be considered.
palpable lymph node.
• No signs of toxicosis.

My diagnosis is simple nodular goitre.


Q: What are the causes of solitary thyroid nodule?
A: As follows:

• Simple nodular goitre.


• Palpable nodule on diffuse or multinodular goitre.
• Thyroid cyst.
• Thyroid adenoma or toxic adenoma.
• Malignancy (carcinoma and lymphoma).
Q: What is the nature of solitary nodule?
A: Majority benign (80-90%); and some cases may be
malignant (5-10%). Solitary nodule (right lobe)

Q: Mention one investigation.


A: USC of thyroid.
Q: What investigations do you suggest?
A: As follows:
• USC (to see whether cystic or solid).
• RAIU and thyroid scan.
• FI3, FI4 and TSH.
• FNAC and open biopsy (if suspected mal ignancy).

N.B. FNAC is the first investigation to be done in


single thyroid nodule; accuracy is 70-97%.

Q: How to treat solitary thyroid nodule?


A: As follows: Solitary nodule (isthmus)

1. 80% of the thyroid nodules are solid, 'cold, non-


Q: What are the types of benign adenoma?
toxic and colloid. Treatment is:
A: They are of three types:
• Reassurance and follow-up.
• Surgery for cosmetic purpose; or if suspicion • Follicular (the commonest).
of malignancy. • Papillary.
• Thyroxin may be given in some cases. • Hurthle cell type.
6 • ENDOCRINOLOGY _

Q: How to treat benign adenoma? no pretibial myxoedema). Thyroid scan shows hot
A: As follows: nodule. It occurs in 5% of the cases of hyperthyroid-
ism and is common in females of >40 years.
• If nonfunctioning: Reassurance and follow-up.
• Thyroxin may be given (if <4 em, FNAC shows
Treatment: Radioiodine therapy or surgery.
benign lesion and no risk factor for malignancy).
Dose 2-3 mg/kg/day for 6 months. Re-evaluate. Q: How would you investigate a patient presenting
If no response, stop the therapy. ~
with solitary thyroid nodule? 0-
o
• Surgery: For cosmetic purpose or suspicion of A: See the scheme below:
o
,_,
malignancy. S·
o
0-
Q: What is toxic adenoma? ~
A: Thyrotoxicosis due to thyroid adenoma (usually
follicular) called Plummer disease (no eye sign and

Clinical + Hormone + Scan + USG

• RAI-therapy • Ethanol Observation • Aspiration


• Ethanol • Surgery for 6 • Surgery
• Follow up months • Ethanol
• Surgery

FNAC, fine-needle aspiration cytology; U5G, ultrasonography cytology. Courtesy: Dr Md. Farid Uddin.
_ S~ORT CASES IN CLINICAL MEDICINE

Q: What is the fate of untreated thyroid nodule? is associated with euthyroidism; occasionally
A: As follows: hyperthyroidism.

• May persist for long time. All are common in females except MCf, which
• Spontaneous regression in 30%. occurs equally in both sexes. The various types are:
• Malignancy (5-10%). • Papillary: 70-80% (the commonest).
• Cystic changes due to haemorrhage within the • Follicular: 10%.
6Q
o nodule. • Anaplastic or undifferentiated: 5%.
-0
.S • Secondary infection . • MGT: 5-10% .
....
u
o Q: ITow to suspect malignancy in a single thyroid • Others: S% (lymphoma and secondary deposit
'"0
c: nodule? in thyroid).
~
A: As follows:
Papillary carcinoma
• Age is elderly.
• The commonest type; usually in young, 20-40
• History of recent and rapid growth.
years. May be in later life (bimodal).
• Hoarseness of voice.
• It is slowly growing and multifocal, and makes
• History of radiation therapy in childhood
up 90% of irradiation-induced thyroid cancer.
(in head and in neck).
• Local lymph node metastasis is common;
• Family history [medullary carcinoma of thyroid
haematogenous spread is less common.
(MCT)].
• Patient may present with cervical
• Gland is solitary, hard, irregular and fixed to the
lymphadenopathy without thyroid enlargement.
underlying structures.
• FNAC is very sensitive and specific test.
• Associated palpable lymph node.

Q: What are the types of nodules seen in thyroid scan? Treatment and prognosis:
A: They are of two types: • Total thyroidectomy followed by high-dose
1. Nonfunctioning: Cold nodule. radioiodine therapy (to destroy remaining
2. Functioning: thyroid tissue and metastatic site).
• Hot nodule: Only functioning nodule takes • Lifelong T4 (to suppress TSH, as it is TSH
radioiodine; other parts of thyroid are dependent).
suppressed. • Prognosis is good; survival is almost the same
• Warm nodule: Functioning nodule and part as in normal person, if the lesion is localized;
of surrounding nodule show uptake. 20 years' survival in 95% (a potentially benign
lesion).
• Isofunctioning nodule and other parts of
• If distant metastasis, 40% survival up to 10 years.
surrounding nodule show equal uptake; and
both cannot be differentiated. Follow-up (following tests are done periodically):
Q: How to treat functioning nodule? • Serum thyroglobulin. If raises, indicates
A: As follows: recurrence or metastasis (normally, thyroglobulin
1. Hot nodule: is undetectable).
• Radioiodine therapy: Better, especially after • Periodic whole-body scanning is done to check
40 years of age. any metastasis.
• Surgery. If there is recurrence, high-dose radioablation
• Local injection of alcohol: 1-2 mL weekly, should be done.
4-8 injections (80-90% effective).
2. Warm nodule: Follow-up. Local injection of Follicular carcinoma
alcohol may be considered.
• Common in middle-aged, 40-60 years of age.
3. Isofunctioning nodule: Follow-up. LOcal injec-
• Usually, a single encapsulated lesion.
tion of alcohol may be considered.
• Blood-borne metastasis is common (to lung,
Q: What are the types of thyroid malignancy! brain and bone). Lymph node metastasis is rare.
A: Thyroid carcinoma is less common-1 % of • Diagnosis is done by open biopsy (FNAC is less
the malignancies. Usually, thyroid malignancy specific).
6 • ENDOCRINOLOGY _

• Treatment and follow-up: Exactly like papillary. hyperparathyroidism) and -Ilb (as in type-Ila
Prognosis is good. plus multiple neuroma and Marfanoid body).
• Both follicular carcinoma and its secondary May be sporadic too.
metastasis takes-up and responds to radioiodine • Serum calcitonin is high (but no hypocalcaemia)
therapy. and carcinoembryonic antigen (CEA) may be high.
• Also secretes histamine, serotonin, slow-reacting
Anaplastic or undifferentiated carcinoma: substance of anaphylaxis (SRS-A),prostaglandin
• Usually in the elderly, >60 years, more in women and adrenocorticotropic hormone (ACTJ-I).
and highly malignant. • May cause carcinoid syndrome and Cushing
• Rapid thyroid enlargement of over 2-3 months. syndrome.
Goitre is hard. • Lymph node metastasis is common and
• There may be hoarseness of voice due to recurrent haematogenous spread is rare.
laryngeal nerve palsy and stridor due to tracheal • Treatment: Total thyroidectomy with removal of
compression. affected lymph nodes and T4 therapy. External
• If possible, perform surgery (total thyroidectomy radiotherapy may be given after surgery.
and T4therapy). • Does not take radioiodine; chemotherapy is not
• Resistance to almost any other treatment. effective.
Radiotherapy may be given.
• Survival is 6 months after diagnosis, if surgery is Prognosis: Variable and relatively good. Some may sur-
not possible. vive up to 20 years, but some <1 year.

Medullary carcinoma ofthyroid (MCT): Lymphoma of thyroid:

• Arises from parafollicular C-cells, usually • Rarely, may occur as primary or as a part of
multifocal. May be inherited as autosomal generalized lymphoma.
dominant (AD) disease. • Generally occurs in thyroid gland affected by
• Common in middle-aged and elderly, but may Hashimoto thyroiditis. A rapidly enlarging mass
occur in young age when there is family history. in thyroid in patients with Hashimoto thyroiditis
should arouse suspicion of lymphoma.
• Associated with multiple endocrine neoplasia
(MEN) type-Ila (phaeochrornocytoma and Treatment: External irradiation plus chemotherapy.

- ---- --------- ---

Hypothyroidism
Usual instructions are: • Instruction 3: Examine the thyroid gland
systematically. Then mention, 'I want to examine
• Look at the face. What are your findings? What else the face, talk with the patient, and examine legs
do you want to examine? and skin'.
• Examine the leg. What are your findings? What
else do you want to examine?
• Examine the thyroid gland.

Proceed as follows:
• Instruction 1: Mention the findings of the face
(see below). Then mention, 'I want to examine
the legs (nonpitting oedema and slow relaxation
of ankle jerk) and skin', talk with the patient
(hoarse and croaky voice) and examine the
thyroid gland.
• Instruction 2: Mention the findings in legs. Then
mention, 'I want to examine the face, talk with
the patient, and examine the skin and thyroid
gland'. Myxoedema
__ SHORT CASES IN CLINICAL MEDICINE

• Following radioiodine therapy.


• After surgery (check for any scar mark).
• Thyroid aplasia.
• TSH deficiency.

Presentation of (I Case rl--------.


(Myxoedema with GOitre):
Case No.2 Co)
• Above findings in Case no. 1 plus there is a
nodular goitre involving the isthmus of the
thyroid gland, nontender, firm in consistency,
freely movable and no bruit.
Nonpitting oedema in myxoedema

My diagnosis is myxoedema, goitrous.


Presentation of a Case 11-- -.
(Only Myxoedema, No Goitre): Presentotion of (] Case
Case No.1 (Myxoedema with Goitre):
• The face is puffy with periorbital swelling, baggy Case No.2 (b)
eyelids and malar flush. There is also loss of
• Above findings in Case no. 1plus thyroid gland is
outer one-third of eyebrows. diffusely enlarged, nontender, firm and rubbery,
• Skin: Dry, rough, cold and thick (there may be
freely movable and has no bruit.
yellow skin due to carotinaemia, vitiligo and
erythema ab igne).
• Nonpitting oedema in leg and generalized My diagnosis is myxoedema.
swelling of whole body.
Q: What do you think of the causes (goitrous
• Voice: Low pitched, slurred, hoarse and croaky
hypothyroidlsm] ?
(hypothyroidism is diagnosed by talking over
A: Hashimoto thyroiditis.
telephone) .
• Ankle jerk (slow relaxation). Q: What ate the other causes of hypothyroidism in this
• Pulse: SO/min (bradycardia). case?
• Thyroid gland (not enlarged). A: Other possibilities are:

• Drugs: Lithium, amiodarone and iodide.


My diagnosis is myxoedema. • Endemic iodine deficiency (less common).
• Rarely, dyshormonogenesis and infiltrative disease
Q: Ask some questions to the patient. Or, what history
(amyloidosis and sarcoidosis).
would you like to take?
A: When the patient answers, hear the voice and com-
ment on it.
• Do you prefer hot or cold? (Cold intolerance)
• Are you gaining weight? (Increased weight gain)
• Tell me about your bowel habit (Usually
constipation)
• In females: Ask about menstruation (Usually
menorrhagia)
(Other history: Drug, thyroid surgery and radioiodine
therapy to find out causes.)
Q: What do you think of the cause in this Case
(nongoltrous hypothyroidism)1
Ai More likely autoimmune or spontaneous atrophic.
Other possibilities are: Cas~no. 1: Myxoe(!el'nlt (flongoitrOLlS)
6 • ENDOCRINOLOGY __

My diagnosis is Graves disease with hypothyroidism


(for details see Graves disease).

Q: Why hypothyroidism in Graves disease?


A: Natural history of Graves disease is hyperthyroidism,
followed by euthyroidism and hypothyroidism.
(This may occur following radioiodine therapy or trJ
;:l
c,
after surgical treatment.) o
...,
(")


Q: What are the causes of hypothyroidism? o
0-
A: As follows: ~
1. Autoimmune:
• Hashimoto thyroiditis.
Case no. 2 (a): Myxoedema • Spontaneous atrophic hypothyroidism.
(goitrous-nodular)
• Graves disease (associated with TSH receptor-
blocking antibody).
2. Iatrogenic:
• Radioiodine therapy for thyrotoxicosis.
• After surgery (thyroidectomy).
• Postradiotherapy in neck.
• Drugs: Lithium, amiodarone and antithyroid
drug therapy.
3. Others:
• Endemic iodine deficiency.
• Postpartum thyroiditis.
Case no. 2 (b): Myxoedema • Rarely, dyshormonogenesis.
(Hashimoto thyroiditis-diffuse goitre) • Secondary to hypopituitarism and hypotha-
lamic disorders (rare).

Q: What are the causes of goitrous hypothyroidism?


A: As follows:
• Hashimoto thyroiditis.
• Graves disease (in such case, there is also
exophthalmos and diffuse goitre with
dermopathy).
• Endemic iodine deficiency (less common).
• Drugs: Lithium, amiodarone, iodide.
• Rarely, dyshormonogenesis.
Case no. 3: Myxoedema Q: What are the causes of nongoitrous hypothy-
(Graves disease showing exophthalmos)
roidism?
A: As follows:
• Autoimmune or idiopathic (spontaneous
Presentation of a Case atrophic )-commonest cause.
(Myxoedema with Graves Disease): • Following radioiodine therapy for thyrotoxicosis.
Case No.3
• Postradiotherapy in the neck.
• Above findings in Case no. 2 plus there is bilateral • After surgery (thyroidectomy).
exophthalmos and pretibial myxoedema. • Secondary to hypopituitarism, hypothalamic
disorders.
_ SHORT CASES IN CLINICAL MEDICINE

• Delayed developmental milestones such as


delayed dentition, delayed crawling, etc.
• Hoarse voice.
• Short stature.
• Thick and short neck with presence of
supraclavicular pad of fat.
• Lethargy and hypotonia.
3. In older children, typical features of hypothy-
roidism are present.
In the early age, high degree of suspicion is essen-
tial for the diagnosis. Routine screening of neonates
using a blood spot for 1'SH is done for diagnosis.
T4 should be started immediately. If treatment is
delayed, permanent neurological and intellectual
Juvenile hypothyroidism damage may occur.

Q: What investigations should be done to diagnose


myxoedema?
A: As follows:
1. Serum Fry FT4 and TSH (both T3and T4are low,
TSH is high).
2. Autoantibody (for Hashimoto thyroiditis):
Antiperoxidase and antithyroglobulin (both are
very high).
3. Other tests (not for diagnosis, but to check other
effects):
• ECG (low-voltage tracing, sinus bradycardia
and T-wave inversion).
• Chest X-ray (cardiomegaly due to pericardial
effusion and heart failure).
• Serum cholesterol and triglyceride (high).
Cretinism
• Creatinine phosphokinase (CPK), lactate
Q: What is cretinism? dehydrogenase (LDH) and serum glutamic
A: It is defined as hypothyroidism due to congenital oxaloacetic transaminase (SCOT). (All may
deficiency of thyroid hormone, also called congeni- be high; these are not done routinely.)
tal myxoedema. Features are as follows:
Q: What single investigation should be done in
1. Features in neonates: myxoedema?
• Prolonged physiological jaundice. A: Single test is TSH and other one is 1'4' But T3 some-
• Hoarse cry. times may be normal, as it is converted from 1'4'
• Lethargy.
• Constipation. Q: What are the thyroid functions in secondary
• Feeding problem. hypothyroidism?
• Hypotonia. A: Low Fry Ff4 and TSH (usually all thyroid hormones
2. Features in older babies are: are low.)
• Characteristic facies: Dull and so-called idiotic
look, large head, sparse hair, broad flat nose with Q: If a patient has low T3, T4 and TSH, what are the
big nostrils, widely set eyes (hypertelorism), causes? How to investigate in such case?
thick everted lips with macroglossia. A: Causes may be in the pituitary or hypothalamus.
• Pot-belly with umbilical hernia. TRH stimulation test should be done.
• Skin is dry, scaly, rough, cold and pale yellow • After giving TRH, if TSH is high, the cause is in
(carotenaemia). Hair is sparse, coarse and rittle. the hypothalamus.
6 • ENDOCRINOLOGY _

• If there is no or little rise of TSH, the cause is in • Most sensitrve investigation is TSH, which is
the pituitary. high. Also, FT3 and FT4 should be done (total T3
and T4 may be high in normal pregnancy due to
Q: What is the treatment of hypothyroidism?
increaseTBG).
A: Thyroxine: It should be started with low dose. The
• Treatment: Thyroxine should be given (100-150
dose should be increased gradually after 3 weeks.
pgm once daily). Requirement of thyroxine is
Single dose is preferable and should be taken before
relatively high (40-50%) in pregnancy because of
breakfast. TSH should be repeated after 6-8 weeks.
increased metabolism of thyroxine by the placenta
Once TSH is normal, maintenance dose should be
and also increased serum TBG in pregnancy, which
continued as a single daily therapy. For follow-up,
binds thyroxine, resulting in less FT3and :FT4.
annual thyroid function test should be done.
• Dose of thyroxine should be adjusted to maintain
normal TSH (serum TSH and FT4 should be
N.B. If the patient has deficiency of cortisol (as measured during each trimester).
in hypopituitarism or Addison disease), cor-
ticosteroid should be given first and then Q: How to treat an elderly patient with hypothyroidism?
thyroxine. Otherwise, if thyroxine is given first A: Treatment is the same. But one should take care
without correcting cortisol deficiency, there whether the patient is suffering from any ischaernic
will be severe Addisonian crisis. heart disease. Following thyroxine, it may precipi-
tate angina and myocardial infarction. Treatment is
Q: Why thyroxine should be started in low dose? the same as above.
A: Because if high dose is given, it may precipitate
anginal attack.
Read the Following Topics in Relation
Q: How long will you continue the treatment?
to Hypothyroidism
A: Lifelong.
Q: What bedside test will you do in myxoedema?
A: Ankle jerk. There is slow relaxation called hung-up
Q: If the patienthas ischaemic heart disease with
reflex (other jerks may show slow relaxation).
hypothyroidism, how to treat?
A: As follows: Q: How is slow relaxation best elicited in the ankle?
• Thyroxine should be given in low dose (25 pgm). Why slow relaxation?
Dose should be increased slowly up to the A: Best elicited by kneel-down position on a chair or
optimum dose. bedside. It is due to decreased muscle metabolism.
• P-Blocker (propranolol) should be added. Q: Why non pitting oedema 7
• Coronary dilator, calcium antagonist may he added. A: Due to deposition of mucopolysaccharide sub-
• Coronary angiography followed by angioplasty stances, hyaluronic acid and chondroitin sulphate.
or coronary artery bypass surgery may be needed. These are also responsible for hoarse voice, carpal
Q: What are the causes of transient hypothyroidism? tunnel syndrome and body swelling.
How to treat? Q: What is the difference between myxoedema and
A: Causes are: hypothyroidism?
• Subacute thyroiditis. A: Myxoedema is severe form of hypothyroidism due
• Postpartum thyroiditis. to deposition of mucopolysaccharide substances; but
• Drug induced. all hypothyroidism may not be myxoedematous.
Temporary T4 therapy may be given. Follow-up
Q: What are the types of anaemia in hypothyroidism?
with measurement ofTSH should be done.
A: Anaemia may be:
Q: How to investigate and treat hypothyroidism in
• Usually, normocytic normochromic.
pregnancy?
• Iron deficiency, if menorrhagia (in female).
A: Hypothyroidism is difficult to diagnose in preg-
• May be macrocytic due to associated pernicious
nancy, as normal pregnancy may be associated with
anaemia.
many features of hypothyroidism such as cold skin,
cold intolerance, weight gain, constipation. High Q: What are the causes of anaemia in hypothyroidism?
degree of suspicion is essential. A: Causes of anaemia:
_ SHORT CASES IN CLINICAL MEDICINE

• Anaemia of chronic disorder. 1. Carpal tunnel syndrome (or tarsal tunnel


• Iron deficiency. syndrome).
• Vitamin BI2 deficiency. 2. Slow relaxation of ankle jerk.
• Folate deficiency. 3. Psychosis (myxoedema madness).
• Other factors responsible: Menorrhagia in female, 4. Myxoedema coma.
anorexia. 5. Cerebellar syndrome.
~ 6. Deafness (Trotter syndrome).
c N.B. Macrocytosis in peripheral blood; but normob-
-0 7. Others:
.E
.... lastic bone marrow occurs in hypothyroidism. • Epileptic fit (due to SIADH) .
o
C
-0
• Peripheral neuropathy.
Q: What is the difference between primary and sec- • Myotonia (Hoffman syndrome).
~
ondary hypothyroidism? • Proximal myopathy.
A: As follows: • Pseudodementia.
• Primary hypothyroidism involves thyroid gland • Drop attack
associated with myxoedema.
Q: What is sick euthyroid syndrome?
• Secondary hypothyroidism is due to the
A: In any severe acute nonthyroidal illness or after
involvement of pituitary or h.ypothalamus. (n
surgery, there may be abnormal thyroid function
such cases, myxoedema is rare; there are other
test; although the patient is euthyroid-called sick
features of hypopituitarism.
euthyroid syndrome. It may occur after myocardial
Q: What is subclinical hypothyroidism (borderline infarction, pneumonia, cerebrovascular disease
hypothyroidism or compensated euthyroidism)? (CVD) and drugs (dopamine and steroids). Usually,
A: In this condition, T3and T4are in the lower limit of there is JowTSH, high 1'4' and normal or low L. Lev-
normal and TSH is slightly high. The patient may be els are usually mildly below normal and are thought
clinically euthyroid. This may persist for many years, to be mediated by interleukins (lL-l and lL-6). Test
though overt hypothyroidism may occur. Conver- should be repeated after recovery of systemic illness.
sion to overt hypothyroidism is more common in Biochemical thyroid function should not be done
men or when thyroid peroxidase CfPO) antibody is in patients with acute nonthyroidal iJlness, unless
present or when TSH level is more than 10 ml.Ij'L. there is good evidence of thyroid disease (such as
goitre and exophthalmos).
Treatment: Thyroxine therapy may be given ifTSH is
persistently raised above 10 mUlL or when there are Mechanisms of sick euthyroid syndrome:
symptoms or high titre of thyroid antibodies or lipid • Reduced production or affinity of TBG to T,
abnormalities. If only TSH is marginally high with and T3.
vague symptoms, thyroxine may be given sometimes. • Reduced peripheral conversion of T4 to T,;
However, in female TSH should be normalized dur- occasionally, more rT3(inactive reverse TJ
ing pregnancy to avoid any adverse effect in foetus. • Reducedbypothalamic pituitaryTSH production;
If TSH is marginally raised, the test should be hence low T3and T4•
repeated after 3-6 months.
Q: What is Hoffman syndrome?
Q: What are the cardiovascular problems in A: In myxoedema, there may be myotonia with pain
myxoedema? and swelling in the muscles after exercise called
A: As follows: Hoffman syndrome.
• Sinus bradycardia. Q: What is Pendred syndrome?
• Pericardial effusion. A: It is an inherited disorder [autosomal recessive
• Congestive cardiac failure. (AR)] associated with sensorineural deafness and
• lschaemic heart disease. goitre. It is due to the inborn error of thyroid
• Hypertension hormone synthesis.
• Atherosderosis(becauseofhypercholesterolaemia).
Q: What is myxoedema coma? What are the mecha-
Q: What are the neurological features 10 nisms? How to treat?
hypothyroidism? A: Myxoedema coma is characterized by depressed
A: As follows: level of consciousness or even coma. Convulsion
6 • ENDOCRINOLOGY _

may occur. It is rare; may occur in severe hypo- • The goitre is usually diffuse, moderately enlarged,
thyroidism, usually in elderly. Cerebrospinal fluid and firm or rubbery. Sometimes, it may be soft-
(CSF) studies show high pressure and high protein to-hard.
content. There is 50% mortality. • Antithyroid antibody (very high, >1000 lUlL):
Antimicrosomal (antiperoxidase) in 90% and
Causes of myxoedema coma:
anti thyroglobulin antibodies (no rise of TSH-
• Syndrome of inappropriate ADH secretion receptor antibody).
(SIADH). Coma is due to hyponatraernia. • About 25% patients are hypothyroid at
• Hypoxaemia. presentation. In the remaining patients, serum
• Hypercapnia. T4 is normal and ISH is normal or raised. There
• Hypothermia. is a risk of developing overt hypothyroidism in
• Hypoglycaemia. future. Initially, the patient may present with
• Other factors: Cardiac failure, infection, use of features of toxicosis called Hashitoxicosis.
sedative. • In young patients «20 years), ANF may be positive.
Treatment of myxoedema coma: It is better to be • Since this is an autoimmune disease, it may be
treated in ICU. Before starting treatment, blood is associated with other autoimmune diseases like
taken for Ff3, Ff4, ISH and cortisol. Addison disease, DM, premature ovarian failure,
rheumatoid arthritis, Sjogren syndrome, ulcerative
1. 1'3 (rapidly acting) 20 ugm, 8 hourly usually IV colitis, autoimmune haemolytic anaemia.
given (parenteral T4 is not available; also slow- • Treatment: Thyroxine (it reduces the size of
to-start action). If parenteral T3 is not available, goitre also).
oral thyroxine through Ryle tube should be
given. Q: What is the radioiodine uptake in Hashimoto
2. IV hydrocortisone: 100 mg 8 hourly (especially thyroiditis?
if suspicion of hypopituitarism). A: It shows the following:
3. Other treatment:
• Initially: Increased (toxic phase).
• Slow rewarming. • After few days or weeks: Norma) uptake.
• High-flow 02 therapy. • Later on: Less uptake (hypothyroid phase) .
•. IVfluid and glucose.
• Antibiotic. Q: What are the histological findings in Hashimoto
• Assisted ventilation may be necessary (as in thyroiditis 1
any unconscious patient). A: As follows:

Q: What is myxoedema madness? • Lymphocyte infiltration; also monocyte and


A: It may occur in severe hypothyroidism in the eld- plasma cell.
erly. There is dementia or psychosis, sometimes • Hyperplasia and fibrosis.
with delusion. Sometimes, these features may occur • Hurthle cell.
shortly after starting thyroxine replacement. Depres-
Q: What are 'the autoimmune diseases associated with
sion is common in hypothyroidism.
thyroid disorder?
Q: What is Hashimoto thyroiditis? A: Thyroid disorders may be associated with autoim-
A: It is an autoimmune thyroiditis characterized by mune diseases like pernicious anaemia, Addison
destructive lymphoid infiltration of thyroid lead- disease, Sjogren syndrome, DM, autoimmune
ing to atrophic change with regeneration and goitre haemolytic anaemia, systemic lupus erythemato-
formation. It is more common in middle-aged sus (SLE), autoimmune hepatitis, primary biliary
woman. cirrhosis (PBC) and premature ovarian failure.

-Acromegaly
Usual instructions are: • Examine the hands of the patient. What is your
diagnosis? Perform the general examination of the
• Look at the patient. What is the diagnosis? What
patient.
else do you want to examine?
_ SHORT CASES IN CLINICAL MEDICINE

Presentation of a Case 11---------.


(by looking at the Patient):
Case No.1
• Large coarse facies with prominent supraorbital
ridge, more wrinkling of forehead and baggy
eyelids.
• Large jaw, mainly lower; large lips, nose and ears.
• Teeth show malocclusion and are widely apart
with prognathism (protrusion of lower jaw
forward; hence the lower teeth overbites the
upper teeth).
• Large tongue (macroglossia).
• Skull enlarged (look for any frontal craniotomy
scar).

Acromegaly

My diagnosis is acromegaly.
Q: What else do you want to examine in this case?
A: As follows:
• Hands (shake hands): See below.
• Also examine the legs: Large feet.
• Eyes: Bitemporal hemianopia (see visual field).
• Skin: Thick, greasy and sweaty.
• Other parts of body: Enlarge as a whole.
• Talk with the patient: Husky and cavernous voice
(due to the enlargement of larynx).
• Axilla:Acanthosis nigricans and skin tag (molluscum
fibrosum).
• Others: Kyphosis, arthritis, gynaecomastia, coarse
hair, cardiomegaly and thyromegaly.
• Check BP (high).
• Examine the urine for sugar (secondary DM may
occur). Large face and jaw

Presentation of a Case
(by Examining the Hands):
Case No.2
• Both hands are large, warm and sweaty, doughy
feeling, spade-like fingers.
• Carpal tunnel syndrome (if asked to examine the
hands, always test for it).
• Clubbing present.

My diagnosis is acromegaly. Spade-like hands


6 • ENDOCRINOLOGY __

Q: Why it is called acromegaly?


A: Because of the enlargement of peripheral (acral)
parts of body (aem means periphery or limbs and
megaly means big).

Q: Can acromegaly and gigantism exist together?


A: Yes, if excess growth hormone starts in adolescence
and persists in adult life, the two conditions may be
present together.

Large foot

Widely apart teeth with prognathism (front view)

Gigantism

Q: What is the cause of acromegaly?


A: Eosinophilic adenoma of pituitary (macroad-
enoma) causing excess growth hormone (GH)
secretion after fusion of epiphysis. Rarely, ectopic
production of growth hormone-releasing hormone
Prognathism (side view) (GHRH) may cause acromegaly (pancreatic islet cell
tumour, oat cell carcinoma of bronchus and medul-
Q: What are the changes in the eyes in acromegaly? lary carcinoma of thyroid).
A: Bitemporal hemianopia (due to pressure on optic
chiasma). Also, optic atrophy, papilloedema and Q: What are the causes of prominent supraorbital
angioid streaks in retina. ridge?
A: As follows:
Q: What is acromegaly?
A: Acromegaly is characterized by generalized enlarge- • Rickets.
ment of the whole body. It is due to excess growth • Paget disease.
hormone secretion from pituitary macroadenoma • Achondroplasia.
(>10 mm) after union of epiphysis. If occurs before • Hydrocephalus.
the union of epiphysis, it is called gigantism. • Hereditary haemolytic anaemia.
SHORT CASES IN CLINICAL MEDICINE

Q: What are the causes of macroglossia? • Tumour expansion (mass effect).


A: As follows: • Pituitary apoplexy: Rapid expansion of pituitary
tumour due to infarction or haemorrhage within
• Acromegaly.
the tumour. The patient complains of sudden
• Hypothyroidism.
severe headache and loss of consciousness
• Amyloidosis.
(require immediate neurosurgical intervention).
• Down syndrome.

Q: What are the causes of baggy eyelids? Q: What are the investigations done in acromegaly?
A: As follows: A: As follows:

• Old age. 1.Radiology:


• Myxoedema. • Skull X-ray: It shows enlarged sella turcica,
• Acromegaly. erosion of clinoid process, enlarged skull,
• Alcoholism. mandible and sinuses.
• Nephrotic syndrome or acute glomerulonephritis. • X-ray of the hands: There is large soft tissue,
bones, widening of joint space and tufting of
Q: What are the presentations of acromegaly? terminal phalanges.
A: As follows: • X-ray of the foot: To see heel pad (normally,
• Progressive increase of the size of the body (there in male up to 21.5 mm and in female: 18 mm;
may be a history of change in size of rings, shoes, if>25 rnm, highly suggestive). Other changes
hats). like hand.
• Weight gain but weakness. • Other X-rays: Knee joint and chest X-ray.
• Visual field defect (the patient gives history of 2. GH assay (radioimmunoassay): Normally,
collision with doors, persons because of the <1 mU in adult (except in stress).

defective temporal field of vision). 3. Glucose tolerance test (CIT) with simultaneous

• Headache (common). measurement ofGH (more diagnostic): Normally


• Excessive sweating. during CIT, there is suppression of GH <2 mU. In
• Features of hypertension, OM. acromegaly, there is failure of suppression of GH;
• Patient may give history of frequent visit to the occasionally there is paradoxical rise of GH.
dentist. 4. Measurement of insulin-like growth factor-1
• Sleep apnoea syndrome. (IGF-I) (also called somatomedin-C) usually
increases.
Q: What are the signs of active acromegaly? 5. CT scan or MRl of the skull (MRl is more prefer-
A: Signs of activity: able).
6. Others:
• Progressive increase in the size of the body.
• Excessive sweating. • Assessment of other anterior pituitary
hormones.
• Increasing visual field defect.
• Large skin tags (Molluscum fibrosum). • Comparison with old photographs.
• Presence of glycosuria (OM). • Perimetry (to see bitemporal hemianopia).
• Blood sugar: OM in 10% cases; IGT in 25%
• Hypertension.
cases.
• Progressive headache.
• Enlarging thyroid. • ECG.
• Serum calcium (increases in MEN).
N.B. Long-term complications of acromegaly:
Q: What is the treatment of acromegaly?
• Increased incidence of large bowel carcinoma. A: As follows:
• Increased atherosclerosis.
1. Surgery:
• Trans-sphenoidal removal of microadenoma
Q: What are the causes of death in acromegaly?
is the treatment of choice (there is high success
A: As follows:
rate, rapid reduction of growth hormone and
• Heart failure. low incidence of hypopituitarism). Cure
• Complications of hypertension. rate is 80% in microadenoma and 40% in
• Degenerative vascular disease. macroadenoma.
6 • ENDOCRINOLOGY _

• After 3 months postoperative, measure • Clinical improvement (decreased facial puffiness,


growth hormone and pituitary-function tests. body size, less sweating, improvement of
If growth hormone remains high, adjuvant hypertension and DM).
medical or radiotherapy may be needed. • Progress can be assessed by GH and insulin-like
• Occasionally, transfrontal surgery is done growth factor (IGF-l) measurement.
in large rnacroadenoma with suprasellar
extension. Total removal of tumour may
Brtief Ois'(,uss.ionRega'rding
not be possible due to more complications.
Postoperative radiotherapy should be given.
Hyperprolactinaemia
2. Radiotherapy: Q: WhaLare the causes ofhYPel1Pro!lactinaemia?
• It is used as a second-line therapy. External ;1\: As follows:
irradiation by linear accelerator is given in
acromegaly, which persists after surgery to 1. Physiological: Severe stress, pregnancy, lacta-
stop the tumour growth and to lower growth tion, exercise, coitus and sleep.
hormone levels. However, growth hormone 2. Drugs:
level falls very slowly over many years. (Previ- • Dopamine antagonist group of drugs:
ously implantation of Yttrium was used.) o Antipsychotic (phenothiazine, butyroph-
• Radiotherapy can be used in combination enones).
with somatostatin analogue or dopamine o Antiemetic (metodopramide, dornperidone].
agonist because of slow biochemical response o Antidepressant.
to radiotherapy. • Dopamine-depleting drugs (methyldopa).
3. Drugs: Given if surgery is not possible or there is • Oestrogen therapy (e.g. oral contraceptive
persistent acromegaly after surgery. pill)
• Somatostatin analogue (octreotide or 3. Pathological:
lanreotide) may be used as a slow-release • Prolactinoma (usually microadenoma
injection, every 2-4 weeks. <10 mm).
• Bromocriptine: It is a dopamine agonist, • Pituitary macroadenoma.
given in high dose, which reduces GH level • Macroprolactinaemia (there is high
and the size of tumour. But it is less potent prolactin without clinical features of
in lowering growth hormone and recurs after hyperprolactinaemia) .
withdrawal of drug. Its main side effects are • Primary hypothyroidism.
nausea, vomiting and postural hypotension. • Polycystic ovarian syndrome.
Alternatively, cabergoline 0.5 mg/day may be • Rarely: Renal failure, liver failure, hypotha-
given or quinagolide may be used. lamic tumour, ectopic tumour, postictal state,
• A peptide GH receptor antagonist chest wall injury or reflex (e.g. postherpes
(pegvisomant) may be used. zoster).
4. Other treatment: • Idiopathic.
• Control of hypertension and DM (both Clinical features of hyperprolactinaemia:
improve with the treatment of acromegaly).
• Galactorrhoea, hypogonadism (commonest
• Cardiac problems: Whether it improves with
symptoms).
the treatment of acromegaly is not clear.
• In male: Decreased libido, impotence,
lethargy.
N.B. Aim of treatment is to reduce the growth hor- • In female: Amenorrhoea, oligomenorrhoea,
monelevelbelowS mUlL, which shows reduced menorrhagia, infertility.
mortality. A normal IGF-l level is also a goal
Investigations:
of therapy. So, the progress can be assessed by
monitoring growth hormone and IGF-l 'level. • Serum prolactin (very high).
• CT or MRI of brain.
q: How eo assess me response of therapy Q(fi! • Other investigations according to the
acrom figa,ly'? suspicion of cause, e.g. thyroid function,
A~ As follows: renal function.
__ SHORT CASES IN CLINICAL MEDICINE

N.B. Remember the following points: • Trans-sphenoidal surgery: May be done in


microadenoma. It is also done in macroadenoma;
• If serum prolactin is high, repeat measurement
though complete removal may not be possible.
is indicated to reconfirm.
• Radiotherapy: If macroadenoma fails to shrink
• If serum prolactin is in the range of 500-1 000
following dopamine agonist drugs or total surgical
mtl/L, it is more likely due to stress or drugs.
removal is not possible.
• If serum prolactin is in the range of
56 • In pregnancy, tumour size may be enlarged and
o 1000-5000 mUlL, it may be due to stress or
o may cause headache and visual field defect. In such
drugs or microadenoma .
.5
;.... case, dopamine agonist therapy should be started,
u • Serum prolactin >5000 mUfL is highly
o if there are symptoms.
"0 suggestive of macroprolactinoma.
~
Treatment: Indications of surgery:
• Treat the primary cause and stop the responsible • Intolerance to drugs.
drugs, if any. • Resistance to drugs.
• Dopamine agonist drugs (such as bromocriptine, • Rapid expansion causing mass effect like visual
cabergoline and quinagolide) are usually given as field defect.
a first-line therapy. • Large cystic macroadenoma.

Cushing Syndrome
Usual instructions are: My diagnosis is Cushing syndrome.
• Perform the general examination of this patient.
• Look at the face. What are your findings? What
else do you want to examine? (Describe the face as
written below. Then mention, 'I want to examine
neck (see below), abdomen (striae), proximal
myopathy, bony pain due to osteoporosis, BP and
urine for sugar test').
• Look at the abdomen. What are your findings?
What else do you like to examine? (There are
multiple striae and distended abdomen. I want to
examine face and other things as above).

Presentation of a Case Cushing syndrome showing plethoric


moon face with stria on arm
• The patient has moon-like, puffy, plethoric face
with acne and hirsutism.
• There is deposition of fat at the root of neck
(buffalo hump and increased fat above the
supraclavicular fossa).
• The patient is obese. There is truncal obesity
with relatively lean and thin limbs (lemon on a
match stick appearance).
• There are multiple pink or purple striae in
abdomen and other parts in skin.
• Skin is thin, with multiple purpura or bruise.
• Proximal myopathy is present.
• There is kyphosis or scoliosis, and tenderness in
spine (due to osteoporosis).
• BP is high. Cushing syndrome (showing hirsutism
and plethoric moon face)
6 • ENDOCRINOLOGY _

Striae in abdomen

Cushing syndrome {bunate. hump-1eft arrow, Q: What history do you ~~Ik('(0 take? Or ask one
supradavicular fat right arrow, hirsutism) question to the patient.
A: I want to know about prolonged use of steroid.

Q~ what are your differential diagnoses?


A: As follows:
• Simple obesity.
• Hypothyroidism.
• Metabolic syndrome.
• Polycystic ovarian syndrome (in early age).

Q: What al-e the causes of periorbiital oedema?


A: As follows:

• Nephrotic syndrome.
• Acute glomerulonephritis.
• Myxoedema.
• Angioedema.
• Dermatomyositis.
• Orbital cellulitis.
• Malignant exophthalmos.

Q: What ate the causes 'Of moon faoe or puffy fa<x?


Cushing syndrome (large ot'east, stlliae~ 1\: As follows:

• Cushing syndrome (plethoric moon face, with


hirsutism, acne).
• Myxoedema (puffy with baggy eyelids, fall of
lateral eyebrows, malar flush).
• Nephroticsyndromeandacuteglomerulonephritis
(puffy with periorbital oedema).
• Superior vena caval obstruction (engorged
and nonpulsatile veins, plethoric face with
subconjunctival effusion).
• Angioedema (localized, swollen lip or face).
• Chronic alcoholism (plethoric, puffy face).
• Simple obesity.
• Surgical emphysema. (History of trauma; also
swelling is extended up to the neck and chest.
Str~al!in thigh There are multiple crepitations on palpation.)
_ SHORT CASES IN CLINICAL MEDICINE

Q; What is the commonest cause of Cush ing syndrome? • Ectopic ACfH syndrome (oat cell carcinoma
A: Prolonged use of steroid. of bronchus, bronchial adenoma, bronchial
carcinoid and carcinoma of pancreas).
Q: Name some diseases where steroid is used for pro- • ACTH therapy.
longed period.
2. Non-ACTH dependent:
A: Addison disease, SLE, pemphigus vulgaris, der-
• Steroid therapy: The commonest cause (even
matornyosius, severe rheumatoid arthritis,
topical or inhaled steroid for long time in
hypopituitarism, diffuse parenchymal lung disease
susceptible cases may be responsible).
(DPLD), etc.
• Adrenal adenoma and adrenal carcinoma
(common in women).
Q: Mention one absolute indication of steroid therapy.
A: Pemphigus vulgaris (also Addison disease). 3. Others: Pseudo-Cushing syndrome (due to
alcohol, depression and obesity).
Q: Why backache in Cushing syndrome?
A: Osteoporosis (may cause vertebral collapse and Q: What is Pseudo-Cushing syndrome?
kyphosis). A: Cortisol excess due to other illness without
involvement of the pituitary adrenal axis is called
Q: What is the character of striae in Cushing syndrome? Pseudo-Cushing syndrome. There is increased
A: Striae are pink- or purple-coloured lesions in the urinary excretion of steroid, absent diurnal
skin of abdomen and other parts of body. variation of cortisol and failure of suppression by
dexamethasone.
Q: What is Cushing syndrome? What are the common • It may occur in chronic alcoholism, severe
features? depression and in simple obesity. All the
A: It is defined as chronic glucocorticoid excess, what- features of Cushing syndrome revert to normal
ever is its cause, which leads to constellation of after removal of the cause (features in favour of
symptoms and signs, commonly: Cushing syndrome are bruise, myopathy and
• Weight gain but weakness. hypertension, all of which are usually absent in
• Proximal muscular weakness (characterized by Pseudo-Cushing syndrome).
. difficulty in combing, raising the hands above • To differentiate from Cushing syndrome: Insulin-
the head, standing from squatting). induced hypoglycaernia is helpful. In Cushing
• Hirsutism in female. syndrome, there is almost no response. But,
• Amenorrhoea or oligomenorrhoea. in Pseudo-Cushing syndrome, there is excess
• Loss of libido. cortisol secretion.
• Backache, pathological fracture (due to
osteoporosis), collapse of the vertebra with N.B. Remember, if there is history of alcohol intake,
reduction of height. advice the patient to stop taking alcohol.
• Easy bruising, purple abdominal striae. Repeat the cortisol or dexamethasone sup-
• Hypertension, OM (30%) or impaired glucose pression test. It may be normal. Then further
tolerance (leT). test is not recommended.
• Frequent infection, especially fungal infection,
Q: How to differentiate clinically different types of
slow wound healing.
Cushing syndrome?
• Mood disturbance like depression, insomnia,
A: By history, physical examination and investigation:
irritability, lethargy.
• On examination: Moon face, buffalo hump, 1. In Cushing syndrome due to adrenal cause:
truncal obesity, hirsutism, acne on face, pink • In adrenal adenoma: Clinical features of
striae, growth retardation in children. glucocorticoid excess are present; but andro-
genic effect like hirsutism and virilization are
Q~ What are the causes of Cushing syndrome!
absent and there is no pigmentation.
A: The common cause is steroid therapy. Other causes:
• In adrenal carcinoma: Clinical features of
1. ACTH dependent: glucocorticoid excess are present and andro-
• Pituitary microadenoma <10 mm called genic effects like hirsutism and virilization
Cushing disease (80%). Common in women. are rapidly progressive.
6 • ENDOCRINOLOGY _

2. In ectopic ACTH syndrome: Usually there is rhythm. Normally, serum cortisol is high
short history, excess pigmentation due to high in morning and low in midnight (called
ACfH level, weight loss (rather than obesity) circadian rhythm).
and severe hypokalaemic alkalosis. Hyperten- o Low-dose dexamethasone suppression test:
sion and oedema are more common. Classical 0.5 mg 6 hourly for 2 days. Measure serum
features of Cushi ng syndrome are usually absen t. cortisol at 9 am on days 0 and 2. Failure
3. In Cushing disease: There are classic features of suppression of cortisol «60 nmol/L
of Cushing syndrome. If there is pituitary mac- on second sample) indicates Cushing
roadenoma, visual disturbance and features of syndrome due to any cause. Or, 24-h
hypopituitarism may be present. There may be fea- urinary free cortisol <100 nmol/day also
tures of raised intracranial pressure like headache. excludes Cushing syndrome.
4. Marked hypokalaernia suggests ectopic ACfH 2. Tests to find out the cause (to localize the site of
syndrome. lesion):
5. History of alcoholism and depression or simple • Serum ACfH:
obesity suggests pseudo-Cushing syndrome. o If ACTH is low or undetectable, adrenal
cause is likely. Then USC, CT or MRJ of
Q: What is the difference between Cushing disease and
abdomen is done to find adrenal tumour. If
Cushing syndrome?
no mass is seen, then adrenal vein sampling
A: As follows:
or adrenal scintigraphy should be done.
• Cushing disease shows increased ACTH from the o If ACTH is high: Likely cause is pituitary
pituitary gland that stimulates adrenals. lesion (Cushing disease) or ectopic ACTH
• Cushing syndrome is caused by excess steroid syndrome. Now high-dose dexamethasone
due to any cause. suppression test or corticotrophin releasing
hormone test is done to differentiate
Q: What are the causes of death in Cushing syndrome?
between these two.
A: As follows:
High-dose dexamethasone suppression
• Hypertension. test: 2 mg 6 hourly for 2 days. Plasma
• Myocardial infarction. cortisol is measured at 9 am on days
•. Heart failure. o and 2. Plasma cortisol on day 2 less
• Infections. than 50% of that in day 0 suggests
Cushing disease (in 90% cases). Fail-
Q: How to investigate Cushing syndrome?
ure of suppression occurs in ectopic
A: Initial tests are done to confirm the diagnosis and
ACTH and adrenal tumour. Urine cor-
further tests are done to find out the cause. tisol <50% of basal suggests Cushing
1. Tests to confirm Cushing syndrome: disease and >50% of basal suggests
• First line screening test: ectopic ACTH syndrome.
o 24-h urinary free cortisol measurement. Corticotropin releasing hormone
o Overnight dexamethasone suppression (CRH) test: 100 ug bovine CRH IV is
test: 1 mg dexamethasone is given given. Measure serum ACfH and cor-
orally at 11 prn. Blood sample is taken tisol for 2 h. It increases in Cushing
at 9 am in the next morning to measure disease, but no response in ectopic
serum cortisol. Normally, almost total ACTH (peak plasma cortisol >20%
suppression of cortisol (<100 n mol/L). and/or ACTH >150% of basal values
Failure of suppression indicates Cushing suggests pituitary disease).
syndrome due to any cause. This test is • If Cushing disease is present: CT or MRI of
simple, can be done as an outpatient skull. MRJwill show pituitary microadenoma
screening test, but gives some false- in 70% cases. If no mass is seen, selective
positive results. catheterization of inferior petrosal sinus to
• Second-line screening test (if above tests are measure ACTH for pituitary lesion .
abnormal): • If ectopic ACTH syndrome is the cause: Chest
o Serum cortisol level (8 am and at 12 X-ray, cr scan of chest (to see carcinoma of
midnight): Shows loss of circadian bronchus or bronchial carcinoid).
~ SHORT CASES IN CLINICAL MEDICINE

3. Other tests (to see the effect): 2. Adrenal tumour:


• Electrolytes (hypokalaemia). • In adrenal adenoma or carcinoma, surgical
• Blood sugar. resection is done (adrenalectomy).
• Bone mass density to see osteoporosis. • In carcinoma, there is chance of recurrence.
Then radiotherapy or chemotherapy or adre-
N.B. Remember the following points: nolytic drugs like mitotane may be given.
• Plasma cortisol levels are highly variable. So, • Other drugs: Metyrapone or ketoconazole
random measurement of daytime plasma may be used (which inhibits biosynthesis of
cortisol level is of no value. cortisol).
• In pituitary tumour, there is high ACTH and 3. Ectopic ACTH:
high cortisol. • If possible, the primary lesion should be
• In ectopic ACTH syndrome, there is high surgically removed (like bronchial carcinoma
ACTH and high cortisol. or carcinoid). Other treatment of the primary
• In adrenal tumour, there is low or undetectable
cause like radiotherapy or chemotherapy
ACTH and high cortisol.
should be considered.
• Cushing disease and cortisol secreting adrenal
• If surgery is not possible, medical therapy
tumour are four times common in women as above or bilateral adrenalectomy may be
than men. But ectopic ACTH syndrome is considered.
more in men.
Q: What is Nelson syndrome?
Q: Why dexamethasone is used in suppression test"? A: Nelson syndrome is characterized by ina-eased
Why not other steroids like prednisolone? pigmentation due to excess ACTH associated with
A: Because dexamethasone does not cross-react in enlarging pituitary tumour, which occurs after bilat-
radioimmunoassay for cortisol. But other steroids eral adrenalectomy for Cushing syndrome. It occurs
can cross-react. in around 20% cases.
Q: How to treat Cushing syndrome? The tumour is locally invasive. It can be prevented
A: It depends on the cause. by pituitary radiotherapy soon after adrenalectomy.

1. Cushing disease: Treatment: Surgical removal of the tumour. Occa-


• Transphenoidal removal of microadenoma. sionally radiotherapy (if not given previously).
• If surgery is not possible or unsuccessful,
bilateral adrenalectomy should be done.
Later, the patient may develop Nelson
syndrome (see below).
• If surgery is not possible, sometimes only
pituitary irradiation may be given. External
irradiation is given. It is slowly effective in
50-60% cases; response in children is better
than adults; 80% may be cured.
• To reduce ACTH production: Bromocriptine
or cyproheptadine is rarely effective.
• Drugs such as metyrapone and ketoconazole Nelson syndrome [Bilateral adrenalectomy scar
may be given. (arrow below); striae, pigmentation (arrow above)]

Addison Disease
Usual instructions are: Proceed as follows:
• Perform the general examination (emaciation and 1. Look for pigmentation (dull, slate-color-red or
pigmentation) . grey-brown) in the following sites:
• Examine the patient (pigmentation). What else do • See the whole body (may be generalized
you want to see? pigmentation).
6 • ENDOCRINOLOGY _

• Face and neck (exposed parts). • Sarcoidosis.


• Mucous membrane of mouth (opposite the • Haemochromatosis.
molar), lips and conjunctiva. • Bilateral adrenal haemorrhage: Following
• Skin crease (palmar crease), knuckles and meningococcal septicaemia (Waterhouse-
nipples. Friedrichsen syndrome) and trauma.
• Pressure points (elbow and knee). • Lymphoma.
• Recent scar.
2. Check for vitiligo.
3. Monitor BP: Both standing and lying (BP is low
and there may be postural hypotension).
4. The patient looks emaciated.

Presentation of a Case
• The patient has generalized pigmentation, more
marked in face, neck, mucous membrane of
mouth, palmar crease, knuckles, knee and elbow.
• There is also vitiligo (mention where).
• The patient looks emaciated; BP is low and there
is also postural hypotension.

Pigmentation in crease
My diagnosis is Addison disease.
Q: What are the causes of pigmentation?
A: See in the chapter "General Examination".

Q: Mention one investigation in this patient.


A: Serum cortisol and ACTH level.

Q: What do you expect in Addison disease?


A: Low cortisol and high ACTH.

Q: What investigation will you do after this finding?


A: Short synacthen test (see below).

Q: What is Addison disease?


A: It is the primary adrenocortical insufficiency
resulting in glucocorticoid and mineralocorticoid Generalized pigmentation
insufficiency. There is destruction of the entire
adrenal cortex.

Q: What are the causes of Addison disease?


A: As follows:
1. Common causes:
• Autoimmune mechanism: 80% of the cases
(more in female).
• TB of adrenal gland: 10%.
• Secondary deposit in adrenals.
• HlV infection.
• Bilateral adrenalectomy.
2. Other causes (less common or rare):
• Amyloidosis. Pigmentation in lips and gum
Q: Why does pigmentation OCruT in Addison disease?
A: Due to excess ACfH that stimulates excess melanin
production.

Q: What are the sites of pigmentation in Addison


disease?
A: As follows:

• May be generalized.
• Exposed parts (face, neck).
• Skin crease (palmar crease) and knuckles.
• Pressure points (elbow, knee).
• Recent scar.
Pigmentation tn palate
Q: What investigations should be done to diagnose
Q: What are the diagnostic criteria in Addison Addison disease?
disease? A: As follows:
A: Triad of: 1. Routine tests:
• Weakness or emaciation (100% cases). • CBC (shows high eosinophil, lymphocyte
• Pigmentation (90% cases). and ESR). Anaemia may be present, specially
• Hypotension (88%). associated with pernicious anaemia.
• Blood glucose (low or lower limit, especially
(Other features: Gastroenteritis in 56%, postural
during Addisonian crisis).
symptoms 12%, salt craving 19%).
• Electrolytes. (Hyponatraemia and hyper-
kalaernia. Hyponatraemia is more important
Q: What is the size of heart in Addison disease? than hyperkalaernia. Mild acidosis may be
A: Small heart. present.]
• Other tests: Serum renin (increases),
Q: Why does postural hypotension occur in Addison aldosterone (low) and serum calcium (may
disease? be high).
A: It is due to hypovolaernia and sodium loss.
2. Test to confirm:
Mineralocorticoid deficiency is responsible for
• Plasma ACfH and 'cortisol measurement is
hypotension.
confirmatory (there is high ACfH, >80 ng/L,
and low or lower normal cortisol).
Q: What are the diseases associated with Addison
• Short synacthen test "should be" tlone. If
disease?
cortisol level does not rise, it indicates
A: It is an autoimmune disease; therefore it may be
. primary or secondary adrenocortical
associated with other auto-immune diseases, such '(.)(1'::)1) f)1, deficiency.' Then plasma ACfH should be
as Graves disease, Hashimoto thyroiditis, pernicious
blo» done. ACfH is high in Addison disease and
anaemia, primary ovarian failure, myasthaenia
,).,; low or undetectable in ACfH deficiency. If
gravis and type-I DM, etc.
ACfH test is unavailable, long synacthen test
can be done to differentiate between primary
Q: What.are the presentations of Addison disease?
and secondary adrenocortical deficiency
A: As follows:
(see below).
• Chronic adrenocortical insufficiency (weakness, 3. Tests to find out causes:
pigmentation, hypotension). 'Ii • Che~'i X-ray (to diagnO'sduberculo1Sis).
• Addisonian crisis. • ....._"-.LOif~~ c.
f...... .y~. • Plain X-rav ~dom.en
lcifi .
{';)IS(IJ~t ft.'to.siorn
(,,(10,' . ~61t.{ l"n~'I)S 10 aT •
see adrena
I
~(k~ ca Cl canon 10 . '
.' '''nhs r't.! ?Q0,sb ,(IBbno)sG •
Q: Why does vitiligo occur in Addison disease? • Adrenal autoantlboay. . •. VI] i e
A: It is due to autoimmunity. Vitiligo is present in • USG or Cf scan ofna~f~r1ciU rtQ look for
Tff'l)f)<)I snsrbs 161SJslj(i •
10- 20% of the cases. calcification in IB or malIgnancy .
,-......./_
'! .r
"11 , !-.,r~ {) r-

::iwbiol'{IlIA •
6 • ENDOCRINOLOGY _

4. Other tests: According to some authority, 10 mg after


• Screening for pernicious anaemia and other waking up, 5 mg at 12 noon and 5 mg
autoimmune disorders. al6 pm.
• Thyroid screening. o Or, if hydrocortisone is not available,
• Other tests according to suspicion of cause (e.g. prednisolone 5 mg on waking in morning
sarcoidosis, amyloidosis, haemochrornatosis, and 2.5 mg at 6 pm in afternoon.
HTV, histoplasmosis, metastatic carcinoma, etc). • Mineralocorticoid: Fludrocortisone 0.05-
0.1 mg (50-300 ugm) daily.
Q: How to perform synacthen test?
• Androgen: Dehydroepiandrosterone (DHEA)
A: Short synacthen test may be done during anytime of
50 mg/day may be given in female. It
the day, but better at 9 am, nonfasting. It is usually
increases libido and sense of well-being; but
done for:
complications like acne and hirsutism may
1. Diagnosing Addison disease. occur.
2. Screening test for ACfH deficiency. 2. Treatment of the cause, e.g. antitubercular
Procedure: therapy in tuberculosis.

• Short synacthen test: 250 J.lgACfH (synacthen or


General advice to the patient:
tetracosactrin) 1M or IV is given. Serum cortisol
is measured at 0 and 30 min. If cortisol rises • The patient should always carry a bracelet and
to >460 nrnol/L, it rules out Addison disease. steroid card, which should contain informations
Failure to rise may indicate primary or secondary regarding the diagnosis, dose of steroid and
adrenocortical insufficiency. doctor's contact address.
• Long synacthen test: 1 mg ACfH 1M daily for • Good nutrition, regular meal, high carbohydrate
3 days. Serum cortisol is measured at 0, 4, 8 and and sufficient salt.
24 h on each day. Progressive rise of cortisol • The patientshouldkeep ampoules ofhydrocortisone
indicates secondary adrenocortical insufficiency. at home. If oral therapy is impossible, the patient
Failure to rise indicates Addison disease (cortisol should take injection by himself, family members
remains <700 nrnol/L 8 h after last injection). or general, practitioner (GP).
. I J lJ lj!)iqtd I _" "J!,,' , ,1)1 I ,I
J

~ The oarient should know how to ingease steroid


jd(
srn !he 1£11'1'iOIl TO 'J11n If, l'9V,}~ ,Y[JIl:WP<;l?,OU.., -I rum
N.B,:T~meIWer the following: replacement dose for ,jnterc;urrem illness. During
. t1 .. ~ rrr D:;nlIljJ'n 'H1 ,("'(f]
intercurrent. stress (fever, cold and uauma),~ the
«0 1fol trhe3 patient is In(fiWl!ijexarfi'MlYas@n'e'S9r ,9Z0)lJlg 0\ \J I V J
dose should be doubled. . b I \/1
J~ \flS'h~thasone, it will not intei~te6"'w1!th -0'1 b vn II9([1 .J&J~ gm Ou r '.:l(T021J1m01 '~r •

',Oi ,o2l:~ffi!ia1'assay (as these do not cross-react). . rbtiffw 'Vf~~Jrf (). 1v'\1 grn 00 [ 9f102i1'l0)
om ormg 0 e patient:
10j91ji1falfdfu cortisol is usually low in Addison ~
::l sr ns» briG "l 6ta7.i tn'.:lilGtj '.:IliJ lirrtu bsununoo
Il:01oVJfl91)iT9b b . some cases, It . may bee wi
'IQls~ase; ut JD WIthito '{I 1.£ ilpP6pB?RMbij t1igc7?d~rlg'bv&aTiI~ru'"19ghg~1
)~rn'~!''(i6JIU1IQ. . ..
normal or inappropriately -low in sefiOusly OJ. 8rM~~urer8klI.t001 IW ~frd Ur~igl).¥?2ihO)Olb'UI
I'11 pauetl,t-0o,
woJ c
ran d omrI(>iH . 1 measurement
coruso HT)A
2'(*bS~\Ifefel8?0lY¥'e?b bsbivib ni ~[fl Oc-O~
Im10lis'{IJafl2\10 itt!p'&Wan~_v"Ftbwever, ~~l'c\Jt~fu sd bluorie '{(jf>19CiJ lrt9rftcl)f,kp=11 lsnigno [I'9[lJ)
( .
ctifflliMi is <10CJ>fii1.ffi~}kll,l4tis high:l~gg@t-i<t\e N.6. Remember the fQllovying points d&fiK~stress:
of Addison dise§S'e?'R§8jl:if the serum !tb"rmal ,fTOtl5ri1n I ,g.S) 921J£J gnr'{h~o(TJJ 10 j[l9m1f)9'1~ II

is. >460_nmol/L, 1t9ffi{-([s2tfut Addison disease, 1. IIJf~J:§t,I{J;~Jit>RV:tl~s-M~..in;


\~Q-IslI~n9ilriJ.Uma):
• Single test for diagnosis of Addison disease is the Double dose of steroid.
simultaneous measurement of ACfH and serum 2. During surgery:
cortisol level (ACfH is high, cortisol is low). • Minor surgery: Hydrocortisone 100 mg
1M or TV premedication.
Qi How to lr,e~'\.A.ddisondisease!
• M,<ijor surgery: Hydrocertisone ,~00 rng
A; As foll~ws:
1M or IV 6 hourly for 24' h:' then SO mg 6
1. Replacement of hormones: hourly. It shouldbe c6ndm.i'ed&GhlfItmJ
I'll, ·"Gluco€orticojd~, I i, <l"I'Tr;;~<)(rrJA J pa.tJ..~okil' ;«i1pa~ l~~.p£·~akingl;),y, ll19:J.1P1"
,15. mg in m~r'f).ing,a after
"fit-! < iv1 "'rPIfM-y<h.·9~RTli~.o,Iil.ej., 3d) 1&ilv..3!!lffg<tW9J~l)~{l~it~)lYn(');f.JMllw<Uqc,QI;1:js0l1e
waking) and 5 mg in aftern0.0f,l,(:Q~Zpm). should replace oral.therapy.-,» . "-'If nh
_ SHORT CASES IN CLINICAL MEDICINE

Q: What is Addisonian crisis? N.B. Remember the following:


A: It is an acute severe adrenocortical insufficiency
• In severe hyponatraemia «125 mmoljL),
characterized by circulatory shock with severe
hypertonic saline is unnecessary; plasma Na
hypotension. It is often precipitated by inter-
should not be increased >10 mmoljLjday.
current disease, surgery or infection. The patient
This may cause central pontine myelinolysis
presents with muscle cramps, nausea, vomiting,
(osmotic demyelination syndrome).
Sri diarrhoea, acute abdomen, collapse and uncon-
o • During crisis or acute illness, mineralocorti-
"0 sciousness. There may be unexplained fever.
coid such as fludrocortisone is unnecessary, as
.S
.... Laboratory findings include hyponatraemia, hyper-
o high dose of steroid provides sufficient minera-
o kalaemia, and, in some cases, hypoglycaemia and
"0 locorticoid activity. It can be started later on.
~ hypercalcaemia.
• For hyperkalaernia, volume replacement is
sufficient. No extra treatment is usually neces-
Causes: sary, but occasionally requires specific therapy.
• Sudden withdrawal of steroid (common cause, if Q: What drug is avoided in acute abdominal pain in
patient is on steroid for long time). Addison disease?
• Stress (severe infection and operation). A: Morphine, as patients are more sensitive to this drug.
• Bilateral adrenal haemorrhage (meningococcal
Q: How to differentiate between primary and second-
septicaemia, injury and anticoagulant).
ary adrenocortical insufficiency?
• Thyroxine therapy in a patient with hypopi-
A: As follows:
tuitarism without steroid therapy.
Topic Primary Secondary
Treatment: Site Primary Cause in pituitary

• Blood is taken to measure cortisol, glucose and involvement of or prolonged use


adrenal gland of steroid
electrolytes.
• Three problems are present: Shortage of salt, Pigmentation Present Usually pallor

sugar and steroid (3S). BP Low Normal because


• N fluid, normal saline rapidly (1 L in 30-60 aldosterone secre-
min). Subsequently, several litre of normal saline tion is not depend-
enton ACTH
may be required in 24 h.
• IV 10% glucose. Secondary sex Normal Early lossof
• IV hydrocortisone 100 mg stat. Then hydro- character secondary sex char-
acters. Also, there
cortisone 100 mg 1M 6 hourly, which is
are features of
continued until the patient is stable and can take
deficiency of other
by mouth. Then oral steroid is started. Initially
pituitary hormones.
hydrocortisone 20 mg 8 hourly, reducing to
ACTH High Low
20-30 mg in divided doses over a few days
Electrolytes Low Na+, high K+ Usually normal
(then original replacement therapy should be
given). Autoimmune Autoimmune Unlikely
• Treatment of underlyi ng cause (e.g. infection, disease diseases are
associated
adrenal or pituitary pathology, etc).

Tall Stature
Usual instructions are: Proceed as follows:

• Perform the general examination. 1. Appearance: If tall and obese, it may be Klinefelter
• Look at the patient (patient looks tall). What else syndrome. If tall, lean and thin, it may be Marfan
do you want to do? syndrome.
6 • ENDOCRINOLOGY _

• Secondary sexual characters (small genitalia, and


absence of axillary and pubic hair and beard).
4. If the above are excluded:
• Examine for evidence of thyrotoxicosis.
• Examine for evidence of gigantism (tall and
proportionately large body).
• Look for anosmia (Kallmann syndrome).
• Take family history (familial tall stature and
constitutional).
In the examination, usual cases oftall stature are:
• Klinefelter syndrome.
• Marfan syndrome.

Q: What are the causes of tall stature?


A: As follows:
Tall in Marfan syndrome 1. Constitutional.
2. If Marfan syndrome is suspected, then see the 2. Genetic (familial).
following: 3. Marfan syndrome.
• Measure the height of the patient (from crown 4. Homocystinuria.
to heel). 5. Chromosomal abnormalities (Klinefelter syn-
• Measure the length of arm span. drome and Kallmann syndrome).
• Measure the length of upper segment (pubic 6. Endocrine (gigantism and thyrotoxicosis).
symphysis to vertex) and lower segment (pubic 7. Miscellaneous:
symphysis to sole). Normally, lower segment • Cerebral gigantism.
is larger than the upper segment (in adults the • Extra Y syndrome (more Y-chromosome,
ratio is 0.8:1). taller but no excess GH).
• Also look for arachnodactyly, high-arched • In prepubertal hypogonadism due to any
palate, eyes (lens dislocation) and heart [aortic cause: Tall stature is common because of
regurgitation (AR)). failureoffusion of epiphysis. But, postpubertal
hypogonadism is not associated with tall
3. If Klinefelter syndrome is suspected, then see the
stature because of fusion of epiphysis.
following:
• Gynaecomastia. N.B. Most common cause of tall stature: Constitu-
• Testis (small). tional, hereditary or early development.

Klinefelter Syndrome
Usual instructions are: My diagnosis is Klinefelter syndrome.
• Look at the patient. What is your diagnosis? What Q: What is Klinefelter syndrome?
else do you want to see? A: It is a chromosomal abnormality in which there is
• Perform general examination of this patient. an extra X-chromosome associated with hypogo-
nadism (due to small testis). It is characterized by:
Presentation of the Case
• Tall stature (eunuchoid body proportion: Arm
• The patient is tall and obese. There is bilateral span greater than height and leg is more long;
gynaecomastia. lower extremity is greater than upper extremity).
• Both testes are small, pea-sized and firm; the • Obese.
penis is also small and there is absence of pubic • Gynaecomastia (carcinoma of breast may develop
and axillary hair, and beard. in 20% cases).
• The voice is high pitched. • Absence or rudimentary external genitalia (small
penis and testis: volume <5 ml).
__ SHORT CASES IN CLINICAL MEDICINE

• Absence of secondary sexual characters (axillary, • High-pitched voice.


pubic hair and beard). • Small and poorly developed external genitalia.
• High-pitched voice. • Immature personality.

N.B. In mosaic, there may be normal puberty. Diag-


nosis is done during routine investigation for
@ infertility.
.....o
.gs Q: What are the usual
syndrome?
presentations of Klinefelter

-0 A: As follows:
~
• Poor sexual development.
• Infertility.
• Gynaecomastia.
• Small or undescended testis.

Klinefelter syndrome

Klinefelter syndrome
.9HTO-'t '? t' ·'·\''1~i!1i 'i~l."tg<;jb (M .srs U dI1 an 1l'iIJ2LJ
Q: What is the abnormality of the testis? Q: Why patient is tall in Klinefelter syndrome?
4: Both testes are small and firrn. These .show sern- jA[lWn\ffb~nf;d~fll:¥eI~c§- Wfthll/lat:f<'7bNpiS1\y~ea1)dosltre
el 'n9i:!} (blli"" m <l';f~,n.h"ldb 11:'''<-i..t)1l1~),·>J
imrerous tuou es dysgenesis. riya jjuza {',ci Ion JIan d in puberty. ~,)'32 OJ 3ft!';W !Jov ob '-)? .,
-og(',.{,IbV.fl d IlW b'j1b'_".t,,·d.:.• J? ... '~· .orn,», 1. Y. ()U,,:3.!_ Ill) I .Jfl"3ill)(l ziriJ to noirsuirncxa ()-r~)J' ,g ,
J fl[ roSIS are presen WIUlm e semiruterous LUuU es .
. "'-Ii9alk&il'funcM'ri'istimpiil~(i, resuliih i'fi!h_¥pog- g ~tig~O"rl'S"'dO y~st in Rnne'teTte~
I1/onadism,ictMflojIAlieie: :-;is")i3ZolDSp_eIrmia> lilil~ high syndrome?
, II (r.r'J - '::J A: (J'}t$_:,fpP8'-t~:'L i'l .~G'),.) I qr !;, ',)i (j < rl" .

Q: What
J11,,' .( "f.-I

are the features


c r

of eunuG~W)
')1"

body
IH(' 'J"t..,·I')V,OI
f • Serum testosterone
'c!;hdttrlOnes-(fIideased
(low),
FS'H and LH) .
gonadotrophic

.- '.'P,rqp~,nj.Qa'},hJd 10 I' ,-~


).d)$'eit:lnY6e-stfl1>g~ri"'(l!Fi'Qr@ases)
.
A: Features are apparent from chilc;lh.Qoo~\'O_:;:,1. • Azoospermia is uhlivetHlfl
f!h~z}f~.Hi'st~tb!r~~li:h'-<it>hg>j~g:" I~UJ IJ(L:.lf,,·J • Chromosomal analysi~ (two or lrncH.-d
r -." ,)f " .,. ,.

• Hairless face (also sparse body hair). X-chr-omosome, one or-more Y-chromosome}-J
6 • ENDOCRINOLOGY _

Q: What is the common karyotype abnormality? • Carcinoma of breast in male is more in Klinefelter
AI Usually, 47 XXY, which results from nondysjunction syndrome.
during meiosis in one of the parents; may be 46 XY
Q: How to treat Klinefelter syndrome?
or 47 XXY mosaic.
A: As follows:
Q: What are the associations in Klinefelter syndrome? • No specific treatment.
A: The usual associations are: • Androgen (testosterone may be used moral, ~
0-
injection, patch, gel and pellet). o
• OM type-2. o
....,
• Plastic surgery for gynaecomastia. .....
::I
• LowT4· • Life span is usually normal. o
• Bronchial asthma. 0-
~

Short Stature
Usual instructions are: Causes of short stature:
• This patient is 25 years old. Look at the patient. 1. Constitutional (the commonest cause).
What is your diagnosis? What else do you want 2. Familial or genetic.
to see? 3. Physiological growth delay.
• Perform the general examination of this patient, 4. Chronic systemic disease (heart, kidneys and
who is 28 years old. respiratory).
5. Endocrine diseases:
Proceed as follows:
• Hypopituitarism.
1. History to be taken: • Isolated GH deficiency.
• Family history (parents and relatives). • Cretinism (hypothyroidism).
• Pregnancy record (growth retardation and • Cushing syndrome.
weight at birth, and any congenital disease). • Pseudohypoparathyroidism.
6. Nutritional (protein energy malnutrition, e.g.
• Rate of growth.
• Systemic disease (respiratory, cardiac, Cl'I' and kwashiorkor, marasmus, rickets).
7. Chromosomal abnormalities:
renal).
• Nutrition (less intake and malabsorption). • Turner syndrome.
• Age of appearance of secondary sexual • Noonan syndrome (male Turner).
characters. 8. Skeletal dysplasia:
• Use of steroid during childhood. • Short limb and normal trunk (achondroplasia).
• Psychosocial deprivation. • Short limb and short spine: Mucopolysaccha-
ridoses (Hurler syndrome).
2. Physical examination: 9. Psychosocial deprivation.
• Height and weight chart (if height is below third 10. Others: ~-Thalassaemia major, cystic fibrosis,
percentile, it is considered as short stature). juvenile idiopathic arthritis (JIA).
• Arm span and height (achondroplasia). Features of achondroplasia:
• Shortlimbscompared to trunk (achondroplasia).
• Short stature.
• Reduction of weight and height (malnutrition
• Large head. Face is small; nasal bridge is flat.
and systemic disease).
• Short and broad limbs (both upper and lower).
• Moreweight but short height indicates endocrine
Trunk has normal growth.
disease (hypothyroidism, Cushing syndrome),
• Normal development: Dental, endocrine, sexual, etc.
and genetic syndrome (Prader-Willi syndrome,
• Normal intelligence.
Laurence-Moon-Biedl syndrome).
• Look for evidence of systemic disease (heart, It results from defect in the fibroblast growth factor
kidney and respiratory). receptor-S gene. There is decrease in proliferation of the
• Others (Turner syndrome, Noonan syndrome cartilage present in growth plate. It is inherited as auto-
and pseudohypoparathyroidism). somal dominant disease.
__ SHORT CASES IN CLINICAL MEDICINE

Twenty·year-old girl with JIA Achondroplasia

Constitutional short stature


Pseuc!ohypoparathyroldlsm

Q: How to investigate short stature? • Lateral skull X-ray (may show calcification).
A: Afterexclusion of systemic disease, proceed as follows: • Thyroid screening.
• GH assay (better do the stimulation test).
• Bone age (constitutional delay, hypothyroidism
• Other tests according to suspicion of causes.
and GH deficiency).

----~~ ~~ ----

Diabetes Mellitus

Common short cases selected in any clinical examina- • Diabetic neuropathy (page 330)
tion related to OM are already described in this book • Diabetic retinopathy (page 445).
(Chapter 1). Usual cases are:
In a diabetic patient, examination involves multiple
• Diabetic foot (page 39) systems of the body. General inspection may give a
• Leg ulcer (page 36) clue to any underlying cause of OM. Look at the face
• Diabetic amyotrophy (page 41) (e.g. Cushing syndrome and acromegaly, which are the
• Lipodystrophy of thigh (page 42) causes of secondary OM). Generalized pigmentation
• Necrobiosis lipoidica diabeticorum (page 43) occurs in haemochromatosis called bronze diabetes.
6 • ENDOCRINOLOGY _

Also, general examination may show any complication 11. At the end, examine the urine for sugar, proteinu-
(e.g. diabetic foot, nephropathy, neuropathy and so on). ria and ketone bodies.
However, there may be other instructions by the
examiner, described as follows: Usual Instruction is (Case No.2): ,1--------,

• Examine the lower or upper limbs of this diabetic


USlJollnstruction is (Case No.1): 't-----,
patient.
• Perform the general examination of this diabetic
patient. Proceed as follows (lower limbs):
1. Inspection:
• Necrobiosis lipoidica diabeticorum (central
Proceed as follows:
yellow scar with surrounding red margin, found
1. Look at the patient for: over the shins, due to atrophy of subcutaneous
• Cachexia (in type-I DM) or obesity (in type-2 DM) collagen).
• Generalized pigmentation (found in haemo- • Ulceration and infection (boils, abscess,
chromatosis called bronze diabetes). gangrene, carbuncle and cellulitis). Look at the
• Kussmaul breathing or air hunger (found in sole and between the toes.
diabetic ketoacidosis). • Diabetic dermopathy (small rounded plaques
2. Look at the face for: with raised border over the shins in a linear
orientation). Look for any pigmented scars (late
• Signs of Cushing syndrome, thyrotoxicosis or
acromegaly, which may cause DM. diabetic dermopathy).
• Ear infection, xanthelasma, corneal arcus, • Injection marks on thighs and fat atrophy or
ocular palsy and other obvious cranial nerve hypertrophy.
palsy (e.g. Bell palsy). • Hair loss and skin atrophy (found in small
vessel disease).
3. Examine the oral cavity for oral candidiasis. • Muscle wasting (due to neuropathy or diabetic
4. Examine the neck for thyromegaly (DM may occur amyotrophy, or as a part of generalized wasting).
in th yrotoxi cosis). • Examine both knee joints (swollen joints-
5. Examine the neck and axilla for acanthosis nigri- deformity indicates Charcot joints).
cans (associated with insulin resistance).
2. Palpation:
6. Look for dehydration (in diabetic ketoacidosis)
• Temperature (may be cold and blue due to
and sweating (in hypoglycaemia) or oedema (dia-
peripheral vascular disease and associated skin
betic nephropathy).
atrophy or absent pulses).
7. Examine the skin for hypopigmentation (vitiligo
• Pulsation (absent due to peripheral vascular
may indicate autoimmune cause of DM), diabetic
disease or atherosclerosis) and bounding pulse
dermopathy, ulcers, infections (boils, carbuncle,
(in neuropathy).
cellulitis), necrobiosis lipoidica diabeticorum, fat
• Pitting oedema (nephropathy and autonomic
atrophy or hypertrophy in injection sites, granu-
neuropathy) .
loma annulare, hair loss and skin atrophy. Look
3. Auscultate over femoral artery (for bruit).
for fungal infection in nails.
4. Perform neurological examination of lower limbs
8. Look for any obvious muscle wasting (in thigh
(see page 313).
called diabetic amyotrophy) or joint deformity
(Charcot joint) and Dupuytren contracture or trig-
ger finger. Also in lower limbs, look for calluses, Usuallnstfuction is (Case No.3): '1-------.
nail change, ankle reflex and foot deformities such
• Examine eyes of this diabetic patient.
as hammer or claw toes.
9. Feel the peripheral pulses (reduced or absent in ath-
erosclerosis), look for fixed heart rate or loss of sinus Proceed as follows:
dysrhythmia (autonomic neuropathy) and auscul- • Look for xanthelasma and corneal arcus (occurs in
tate carotid arteries (for bruit in atherosclerosis). hyperlipidaemia in association with DM).
10. BP (lying and standing to see postural hypoten- • Acuity of vision.
sion, found in autonomic neuropathy). • Argyll Robertson pupil.
_ SHORT CASES IN CLINICAL MEDICINE

• Rubeosis iridis (new vessels on the anterior surface called IGT. The patient who has ICT is at increased
of iris). risk of developing frank DM type-2 with time and
• Cataract. also macrovascular complications are more (mainly
• Examine the cranial nerves Ill, N and VI (especialJy cardiovascular) .
the third nerve palsy; if it occurs in DM, it spares Lifestyle modification for type-2 DM and annual
the pupil). check-up for glucose are recommended for this
50 • Finally, perform fundoscopic examination for patient. Cardiovascular risk factors should be treated
o diabetic retinopathy. aggressively.
(5
.S
b Q: What is impaired fasting glucose (IFGF
o Q: What are the causes ofloss of vision in DM?
A: fPC is defined as the fasting glucose level between
J A: As follows:
6.1 and 6.9 mmol/L (110-125 mg/dl.) according
• Diabetic retinopathy. to WHO. However, American Diabetes Association
• Cataract. (ADA) defines it as fasting glucose level between 5.6
• Age-related macular degeneration. and 6.9 rnrnol/L (100-125 mg/dL). These patients
• Retinal vein occlusion. are prone to develop frank DM and cardiovascular
• Retinal artery occlusion. disease. The patient is advised for weight reduc-
• Nonarteritic ischaemic optic neuropathy. tion of about 5-10% of their body weight, regular
• Glaucoma. exercise and follow-up. Usually no drug therapy is
recommended.
Read the Following Topics in Re'ation to
N.B. Patients with IGTand/orlFC are now regarded
Diabetes Mellitus
as prediabetics.

Q: What are the criteria for the diagnosis of OM?


Q: What is latent diabetes?
A: Criteria for diagnosis ofDM are:
A: It means blood glucose is usually normal, but may
• Fasting plasma venous blood sugar level be high under certain stressful conditions. Examples
>7.0 mrnol/L (or 2-h postprandial blood sugar are: pregnancy, infection, obesity, stress or drugs
level >11.1 mrnol/L). like steroid, thiazide diuretics, etc.
• Random blood sugar> 11.1 mmol/L.
• During OCTf, >11.1 mmol/L 2 h after 75 g Q: What is potential diabetes?
glucose. A: It means blood sugar usually is normal, but the
patient has increased risk of developing OM in
N.B. Remember the following points: future due 10 genetic reasons like:

• Random means without regard to time since • Both parents are diabetic.
the last meal. • One parent is diabetic and the other has a family
• Fasting means no calorie intake for 8 h at history of diabetes.
least (not more than 16 h). • Has a diabetic sibling.
• Fasting blood sugar <5.6 mmol/L is normal. • In a twin, if one is diabetic.
• In symptomatic patient, one abnormal
finding is diagnostic of DM. Q: What is blliWlrediabetes?
• In asymptomatic patients, two values are A: Brittle diabetes (or unstable diabetes or labile dia-
required. betes) refers to uncontrolled insulin-dependent DM
• For OGlT, only fasting glucose and 2 hours with recurrent, dramatic, large swings in blood glu-
after 75 g of glucose are sufficient for diagnosis. cose levels, often without any apparent reason. This
• OCTr should be done only for borderline leads to irregular and unpredictable hyperglycae-
cases (fasting glucose 6.1-7.0 mmol/L or rnia, frequently with ketosis, and sometimes serious
random glucose 7.8-11.0 mmol/L) and also hypoglycaemia.
for the diagnosis of gestational diabetes Brittle diabetes occurs in 1-2% of diabetics, usu-
mellitus (CDM). ally in young (15-30 years) patient with type-1 DM,
but may also be found in elderly patient with type-I
Q: What is impaired glucose tolerance (lG'f)? or type-2 OM. It often develops after total pan-
A~ When fasting glucose is <7 mmol; but during OG11", createctomy. It may be caused by gastrointestinal
2 h after the glucose load is 7.8-11.0 mmol. it is absorption problems including delayed stomach
6 • ENDOCRINOLOGY _

emptying (gastroparesis), drug interactions, and skeletal malformations, characteristically


problems with insulin absorption or hormonal caudal regression syndrome, neural tube defect.
malfunction. • Foetal macrosomia (if OM is present in later
pregnancy).
Q: Classify OM aetiologically. • Neonatal hypoglycaemia (as maternal glucose
A: Aetiological dassification of DM: crosses the placenta, but insulin cannot. As a
1. Type-I DM: result, fetal islet cells secrete excess insulin, which
• Idiopathic. may cause neonatal hypoglycaemia).
• Immune mediated. • Increased risk of polycythaernia. hyperbili-
rubinaernia and hypocalcaemia.
2. Type-2 OM.
3. Gestational OM. • Hyaline membrane disease.
4. Other specific types: Q: What is the cause of macrosomia in OM?
• Genetic defects of ~-cell function. A: Macrosomia means large or big baby (birth weight
• Genetic defects of insulin action. >90 percentile for gestational age). It is due to
• Genetic syndromes such as Down syndrome, persistent maternal hyperglycaemia leading to
Klinefelter syndrome, Turner syndrome and foetal hyperglycaemia and prolonged foetal hyper-
DIDMOAD (Diabetes Insipidus, Diabetes insulinism. This stimulates excessive somatic growth
Mellitus, Optic Atrophy and Deafness) mediated by insulin-like growth factors (IGFs).
syndrome. Macrosomia affects all organs except the brain.
• Pancreatic diseases such as chronic
Q: What are the complications of DM?
pancreatitis and haernochrornatosis.
A: As follows:
• Endocrine diseases such as acromegaly,
Cushing syndrome, glucagonoma and thyro- A. Acute complications:
toxicosis. • Hypoglycaemia.
• Drug induced (e.g. corticosteroid). • Diabetic ketoacidosis.
• Viral infections (e.g. congenital rubella, • Hyperosmolar non ketotic diabetic coma.
mumps and Coxsackie virus B). • Lactic acidosis.
• Uncommon form of immune-mediated OM. • Infections: Boils, carbuncle, abscess, cellulitis,
tuberculosis.
Q: What is gestational diabetes mellitus (GDM)?
B. Long-term complications:
How is it diagnosed?
1. Microvascular:
A: It is defined as any degree of glucose intolerance with
• Neuropathy: Peripheral neuropathy
the onset or first recognition during pregnancy. It
(sensory, motor or mixed), mononeuritis
constitutes 90% of women with pregnancy compli-
multiplex, mononeuropathy, autonomic
cated by diabetes. More than 50% women ultimately
neuropathy.
develop diabetes in the next 20 years and this is
• Nephropathy (CKD).
linked with obesity. Mostly they develop type-2 OM.
• Eye complications (retinopathy, cataract).
Oral glucose tolerance test (OCTI) with 75 g of
• Foot complications (ulcers, gangrene,
glucose is used as a screening test for gestational dia-
arthropathy).
betes mellitus (GOM) for women between 24 and
28 weeks of gestation. Blood glucose is measured at 2. Macrovascular:
fasting, land 2 h after glucose load. GOM is diag- • Coronary circulation: Myocardial ischaemia,
nosed ifblood glucose is: infarction.
• fasting >5.1 mmol/L • Cerebral circulation: Transient ischeamic
• after 1 h 10 mmolfL attack (TIA), cerebrovascular diseases (CVD).
• after 2 h 8.5 mrnol/L, • Peripheral circulation: Ischaemia,
claudication.
Q: What are the complications of foetus in DM during • Foot complications (ulcers, gangrene,
pregnancy? arthropathy).
A: As follows:
N.B. Diabetes mellitus can cause painless or silent
• Teratogenicity (if OM is present in earlypregnancy, myocardial infarction. Foot complications are
in the first 6 weeks), there may be cardiac, renal due to both macro- and microvascuJar.
_ SHORT CASES IN CLINICAL MEDICINE

Q: What are the types of neuropathy in OM? Q: What are the causes of sudden death in OM?
A: As follows: A: It is more likely due to autonomic neuropathy.
Patient usually dies from sudden cardiorespiratory
• Sensory neuropathy (common).
arrest.
• Mixed motor and sensory neuropathy.
• Asymmetrical motor neuropathy (diabetic Q: What is insulin resistance syndrome or metabolic
amyotrophy) . syndrome or syndrome X?
~ • Autonomic neuropathy.
o A: Presence of type-2 DM, central obesity, hyper-
"0 • Mononeuropathy.
.5 tension and dyslipidaemia (elevated LDL and
....o • Mononeuritis multiplex .
o triglyceride, low HDL) is called metabolic syn-
-0
Mechanism of neuropathy in OM: drome. Other features are hyperinsulinaemia.
~ microalbuminuria, elevated fibrinogen and plasmi-
• Axonal degeneration.
nogen activator inhibitor I, plasma uric acid and
• Patchy or segmental demyelination.
increased sympathetic activity. The primary defect is
• Involvement of intraneural capillaries.
insulin resistance. This predisposes to an increased
Q: What are the features of autonomic neuropathy in risk of cardiovascular disease.
DM?
A: In both type-I and -2 OM, there may be autonomic Q: What investigations do you suggest in this case?
neuropathy, which involves multiple systems of the A: As follows:
body. Features are:
• Urine R/M/E.
• CVS: Postural hypotension, fixed heart rate, • Blood sugar (fasting and 2 h after breakfast)
resting tachycardia and, sometimes, sudden • HbA1c'
death. • CBC with ESR.
.. GIT: Gastroparesis and nocturnal diarrhoea, • Blood mea and serum creatinine.
constipation. • Serum lipid profile.
• Genitourinary: Urinary incontinence, difficulty in • USG of whole abdomen.
micturition, erectile dysfunction and retrograde • CXR PA view.
ejaculation. • Plain X-ray abdomen to see pancreatic
• Sudomotor: Gustatory sweating, nocturnal calcification.
sweating without hypoglycaemia, hyperhidrosis • BCG.
of upper extremity and anhydrosis of lower
Q: How to diagnose clinically a case of hypoglycaemic
extremity, and anhydrosis of foot can cause
coma and hyperglycaemic coma?
cracked skin and ulcer.
A: Typical features of hypoglycaemic coma are:
• Vasomotor: Cold feet due to loss of vasomotor
response, dependent oedema due to loss • Excessive sweating.
of vasomotor tone and increased vascular • Tachycardia.
permeability. • Tremor.
• Autonomic neuropathy can reduce • Other: Jerks may be brisk, planter-bilaterally
counterregulatory hormone release, leading to an extensor.
inability to sense hypoglycaemia appropriately. Typical features of hyperglycaernic coma are:
• Pupillary: Decreased pupil size, resistance to
• Severe dehydration.
mydriatic drugs, and delayed or no reflex to light
• Pulse: Weak, BP-low.
(called pseudo-Argyll Robertson pupil).
• Air hunger is present: Kussmaul breathing and
Q: What are the causes of painless myocardial acetone in the breath are present.
infarction? • Others: Reflexes are reduced; planter will be flexor.
A: As follows:
Q: What are the differences between hypoglycaemic
• OM with autonomic neuropathy. coma and diabetic coma (coma with ketoacidosis)?
• Elderly patient with dementia.
A: As follows:
• CVD.
6 • ENDOCRINOLOGY _

Points Hypoglycaemic coma Diabetic coma (DKA)


History Excessinsulin, but no or insufficient food intake Too little or no insulin, concurrent infection or
following heavy exercise digestive disturbance
Onset Rapid onset, usually after taking insulin. Patient Slow onset, patient has ill health for several days
is in good health prior to this.
Symptoms Weakness, tremor, sweating, palpitation, hunger, Intense thirst, polyuria, dehydration, vomiting, air
occasional vomiting from depot insulin hunger and abdominal pain
Signs
- Skin and tongue - Moist - Dry
- Eyes - Normal - Sunken
- Pulse - High volume, tachycardia -Weak
- BP - Normal or raised -Low
- Breathing - Shallow or normal - Kussmaul breathing
- Acetone smell - Absent - Present
- Reflexes - Brisk - Diminished
- Plantar response - Often extensor - Usually flexor
- Intraocular pressure - Normal - Decreased
Urine
- Ketonuria - Absent - Present
- Glycosuria - Absent - Present
Blood
- Glucose - Hypoglycaemia «60 mg/dL) - Hyperglycaemia (>300 mg/dL)
- Bicarbonate - Normal - Reduced
-pH - Normal -Low
- Acetone - Normal -Hiqh
Treatment Oral or IV glucose Insulin and others
Prognosis Good Bad
CHAPTER 7

NEPHROLOGY
"One of the first duties of the physician is to educate the masses not to take medicine"
- Sir William Osler

Usual instructions by the examiner are: • General features: Pale, pigmented, puffy face with
• Examine the abdomen. baggy eyelids, generalized oedema.
• Examine the abdomen and see the relevants, • Skin lesion like scabies (may be related to
• Palpate the abdomen. What are your findings? poststreptococcal glomerulonephritis).
• Anaemia [indicates chronic kidney disease
N.B. For details, see Chapter 4 'Abdomen'. (CKD) or plethoric face (indicates secondary
polycythaemia in polycystic kidney disease) I.
Once asked to examine the abdomen in relation to • Blood pressure [hypertension in CKD or acute
nephrology, very likely findings are: glomerulonephritis (AGN)).
• Unilateral or bilateral renal mass. • Arteriovenous (AV) fistula in the wrist or below the
clavicle (haemodialysis).
• Mass in the right or left iliac fossa (transplanted
• Fundoscopy (hypertensive retinopathy).
kidney).
After finishing the physical examination, examiner may
• Abdominal distention due to ascites (as in neph-
ask, 'do you like to perform any investigation?" Answer
rotic syndrome).
with "yes, I want to perform bed side urine examination
Once a renal mass is present, the following relevant for albumin (in nephrotic syndrome) and sugar (may
findings should be seen: be related to diabetic nephropathy)'.

Mass in Flank (Renal)


Usual instruction by the examiner: • Hypertrophied single kidney (if nephrectomy of
• Examine the abdomen. other kidney or congenitally one kidney is absent).
• Palpate the abdomen. • Renal cyst.
• Polycystic kidney with single palpable kidney (due
Presentation of Case fl--------, to asymmetrical enlargement).
(Unilateral Mass): Case No.1
N.B. Right kidney may be normally palpable.
• There is a mass in right (or left) loin or flank,
10 x 9 em, round in shape, nontender, surface is Q: Suggest one investigation.
smooth or irregular, moves with respiration. A: Ultrasonography.
• We can get above the swelling. Q: What investigations do you suggest?
• On percussion, there is resonance over it (colonic A: As follows:
resonance). • Ultrasonography (USG) of abdomen.
• Complete blood count (CEC), erythrocyte
My differential diagnosis is a renal mass, which may be sedimentation rate (ESR).
due to (causes of unilateral. renal mass): • Urine routine and microscopic examination.
• Renal cell carcinoma (in elderly) or Wilms tumour • Blood sugar.
(in children). • Blood urea, serum creatinine.
• Unilateral hydronephrosis or pyonephrosis. • Serum electrolytes.
____________________________ 1__•__NEPHROLOGY ~

• Computed tomography scan (CT) or magnetic Q: What are the causes of bilateral renal mass?
resonance imaging (MRl). A: As follows:
• Intravenous pyelogram (IVP). • Polycystic kidney disease.
• Bilateral hydronephrosis.
Presentation of Case • Diabetic nephropathy in early stage.
(Bilateral Mass): Case No.2
• Amyloidosis. z
('1>
"'0
• There are bilateral masses in both right and left • Rarely, bilateral renal cell carcinoma.
8.'o"
flanks; right (or left) is larger, surface is irregular, 0-
margin is round or irregular, nontender and Q: What investigations do you suggest? ~
freely movable from underlying structure. A: As above.
• We can get above the swelling.
• On percussion, there is resonance over the mass.

Bilateral Renal Mass (Polycystic Kidney Disease)


Usual instruction by the examiner: • Cystic liver in 30% cases (common in infantile
• Examine the abdomen. type), but hepatic dysfunction is rare. There may
• Palpate the abdomen. be cyst in spleen, ovary and pancreas.
• Berry aneurysm in circle of Willis 10% (may
Presentation of a Case: See in bilateral renal mass. rupture causing subarachnoid haemorrhage).
Q: What is polycystic kidney disease (PKD)? • Polycythaemia (due to increased erythropoietin
A: It is an inherited cystic disease of the kidney, and is secretion).
of two types: • Renal stone in 10% cases (usually calcium
oxalate).
• Adult polycystic kidney disease (APKD),
autosomal dominant, usually common one. • Renal neoplasm rarely.
Males and females are equally affected. Q: What are the causes of acute pain in PKD?
• Infantile polycystic kidney disease (IPKD), A: Pain is due to:
autosomal recessive, rare and associated with • Acute haemorrhage in the cyst.
hepatic fibrosis; fatal in first year due to hepatic • Infection in the cyst.
or renal failure. • Renal stone.
• Renal cell carcinoma, rarely.
Q: What are the presentations of adult PKD?
A: As follows: Q: If the patient is unconscious, what is likely diagnosis?
• May be asymptomatic, detected mass on routine A: Subarachnoid haemorrhage (due to rupture of berry
examination. aneurysm). Other causes may be:
• Discomfort, pain or heaviness in the loin. • Cerebral haemorrhage as a complication of
• Recurrent painless haernaturia (due to rupture of hypertension.
cyst in renal pelvis or infection). • Sometimes, hypernatraernia (due to salt-loosing
nephropathy).
• Recurrent urinary tract infection (UTI).
• Acute loin pain or renal colic. Q: What are the causes of death in PKD?
• Features of hypertension (after 20 years of age). A: Death may be due to:
• Features of renal failure. • Chronic renal failure (in one-third cases).
• intracerebral haemorrhage.
• Cerebrovascular accident (CVA) (usually
• Myocardial infarction.
subarachnoid haemorrhage, due to rupture of
berry aneurysm. Or cerebral haemorrhage as a Q: What investigations do you suggest?
complication of hypertension). A: As follows:
Q: What are the other features of PKD? • usc (always mention the first investigation).
A: As follows: • Urine RjE (haematuria) and CjS.
__ SHORT CASES IN CLINICAL MEDICINE

• Full blood count (FBe) (polycythaemia may members, if anyone of the family members with
occur). PKD has history of subarachnoid haemorrhage.
• Renal function tests (urea, creatinine).
• Serum electrolytes (some cases salt looser). N.B. Remember the following points:
• High-resolution cr or MRI. • PKD is always bilateral (may be unilateral, if
• Intravenous urograms (IVU). other kidney is absent).
• PKD is a misnomer; cyst occurs in many other
organs (liver, spleen).
• More common in sickle cell disease, cystic
fibrosis, Huntington disease.
• May be associated with mitral valve prolapses
(25%) causing mitral regurgitation, and aortic
regurgitation (rarely severe).
• Colonic diverticulas may occur.
• Abdominal wall hernia.
• Polycystic kidney disease is not premalignant.
• Hypertension is present in 75% cases.
• Usually there is polycythaemia due to high
Polycystic kidney disease
erythropoietin level. There may be anaemia; there
is chronic renal failure. (However, haemoglobin
Q: What are the findings in NU in polycystic kidney level is higher than expected for the degree of
disease? renal failure.)
A: NU shows: Q: What are the features of infantile PKD?
• Enlargement of both kidneys. A: It is rare, inherited as autosomal recessive and is
• Stretched, distorted and elongated pelvicalyceal associated with cyst in other organs and hepatic
system (giving rise to spidery appearance). fibrosis. It is fatal in the first year of life; death is
due to renal failure or hepatic failure.
Q: How to manage the patient of PKD?
A: As follows:
Q: What are the cystic diseases of kidney?
• Control of hypertension.
A: As follows:
• Control of urinary tract infection.
• Simple cyst, usually congenital.
• Plenty of fluid.
• Salt (there may be salt looser) in some cases. • Acquired cyst, after dialysis (in chronic renal
• Largecyst:Maybeaspirated underultrasonography failure).
guidance. Or, laparoscopic cystectomy may be • Polycystic kidney disease.
done. • Medullary sponge kidney, with unknown cause,
• Treatment of renal failure: Either dialysis, not genetic. Cyst is confined to the papillary
sometimes renal transplantation. collecting ducts. Age 40-60 years; prognosis good.
• Genetic counseling. Usually no hypertension or no renal failure).
• Family screening (for any family member after • Medullary cystic disease (cyst small in cortical
20 years of age, USG is done to detect cyst). or corticomedullary junction. Renal failure is
• Magnetic resonance (MR) angiography to detect common; hypertension may occur. Patient usually
berry aneurysm may be considered in some family has polyuria, increased thirst and salt loser).

Unilateral Mass (Renal Cell Carcinoma)


Usual instruction by the examiner: Q: What is renal cell carcinoma?
A: Renal cell carcinoma (or hypernephrom.a) is an
• Examine the abdomen.
adenocarcinoma arising from. proximal tubular
• Palpate the abdomen.
epithelial cells. It is usually unilateral. But in 10%
Presentation of a Case: See in unilateral renal mass. cases, it may be bilateral.
_________________________ 7 • NEPHROLOGY ~

Renal cell carcinoma is highlyvascular, microscopically


composed of large cells containing clear cytoplasm.
Haemorrhage and necrosis give the cut surface a
characteristic mixed golden yellow and red appearance.
In Von Hippel-Lindau syndrome, inherited as auto-
somal dominant, there may be bilateral renal cyst, renal
adenoma and renal cell carcinoma.

Q: How the patient presents in renal cell carcinoma?


A: It is common in elderly; males are affected twice
more than females. The patient may be asympto-
matic. It usually presents with a triad of:
• Painless haematuria (in 50% cases).
• Loin pain or heaviness.
• Palpable mass in loin.
Other features: Renal cell carcinoma (right side)

• In 20% cases, pyrexia of unknown origin (PUO)


may be the only manifestation.
• Features of anaemia and hypertension,
• Malaise, anorexia, weight loss.
• In 5% cases, there may be polycythaernia.
• Some patients may present with features of
metastasis. It may spread along the renal vein
to inferior vena cava. In renal cell carcinoma of
left kidney, it may spread along the left renal
vein, which may obstruct the left testicular vein
leading to left-sided varicocele. Direct invasion
of perinephric tissue is also common. Lymphatic
spread occurs to para-aortic lymph node. Also,
there may be blood-borne metastasis to any
CTscan showing renal cell carcinoma in left kidney
distant organ.

Q: Why PUO in renal cell carcinoma? Q: What may be the haemoglobin status of this
A: It is due to secretion of pyrogen by tumour. patient?
A: Usually, there is normocytic and normochromic
Q: If the patient has fever with unilateral renal mass, anaemia. But, there may be polycythaemia in 5%
what other diagnosis is possible? cases.
A: Renal tuberculosis (or disseminated tuberculosis).
Q: Why polycythaemia?
A: It is due to excess erythropoietin secreted by
Q: What investigations do you suggest?
neoplastic cells.
A: As follows:
• Urine routine and microscopic examination Q: What peptide hormones may be produced by renal
(haematuria may be present). cell carcinoma?
• Complete blood count (CBC), ESR (anaemia, A: Erythropoietin, renin, ADI-I, parathyroid hormone
may be polycythaemia). (PTH)-related peptide.
• USC. Q: If the patient has hypercalcaemia,
what may be the
• CT or MRI (MRI is better for tumour staging).
cause?
• TVP. A: It may be due to bony metastasis or secretion of
• USC or CT-guided fine-needle aspiration cytology
parathorrnone-like substance.
(FNAC).
• Other investigations for metastasis, e.g. X-ray Q: What electrolyte abnormality may occur?
chest, liver function tests, isotope bone scan, etc. A: Hypokalaemia.
_ SHORT CASES IN CLINICAL MEDICINE

Q: How to treat renal cell carcinoma? 3. Medroxy progesterone acetate may be used in
A: As follows: metastatic disease.
1. Surgery: 4. Some benefit may be found with immuno-
• Radical nephrectomy including perirenal therapy using interferon and interleukin-2.
fascial envelope and ipsilateral para-aortic 5. New drugs are:
lymph nodes should be done, if possible. • Tyrosine kinase inhibitors, e.g. sorafenib and
This should be performed even if metastasis sunitinib.
is present, as it reduces the systemic features
• mTOR (mammalian target of rapamycin)
and regresses the metastasis.
inhibitors, e.g. temsirolimus and everolimus,
• Partial nephrectomy may be done if there are
• Bevacizumab, an antibody against vascular
bilateral tumours or the contralateral kidney
endothelial growth factor (VEGF), may slow
has poor functional capacity.
the progression in metastatic disease.
2. If surgery is not possible due to high operative
or other risk (e.g. single functioning kidney), Prognosis: 5-year survival rate is 60-70%, if tumour
small tumours may be treated percutaneously is confined to the renal parenchyma; 15-35%, if
by radiofrequency ablation or by tissue-sparing there is lymph node involvement; and 5%, if there is
surgery or cryoablation. distant metastasis.

Mass in Left or Right Iliac Fossa (Transplanted Kidney)


Usual instructions are: • Renal artery stenosis (5%).
• Examine the abdomen. • Congenital and inherited (5%), e.g. polycystic
• Palpate the abdomen. kidney disease, Alport syndrome.
• Unknown (5-20%).

Q: From where kidney is collected for transplantation?


Presentation of a Case A: It is collected from:
• There is a mass in right (or left) iliac fossa, • Matched cadaver donor.
5 x 5 ern, nontender, round in shape, surface is • Relatives who are HlA-identical and ABO-
regular, with clear margin. matched.
• There is a laparotomy scar (look for AV fistula • Brain-dead person who are on ventilator.
in arm). Q: Where the kidney is placed?
A: In iliac fossa (either right or left). Donor renal
vessels are anastomosed with recipient's external or
My diagnosis is transplanted kidney. internal iliac artery and vein.

Q: What are the indications of kidney transplantation? Q: How do you diagnose acute rejection?
A: End-stage renal disease [when glomerular filtration A: In the following way:
rate (GFR) is < 5 ml./rnin]. • Pain in the transplanted area.
• Reduction of urine output.
Q: What are the causes of CKD?
• Renal area is tender.
A: As follows:
• Increased serum urea and creatinine.
• Glomerular diseases (30-40%), e.g. 19A neph-
• Urine shows plenty of red blood cell (RBC).
ropathy, mesangiocapillary glomerulonephritis
• USG shows reduced echogenicity.
(MCGN)
• Isotope scan (99TcM or DTPA) shows reduced
• Diabetes mellitus (20-40%).
uptake.
• Hypertension (5-20%).
• Graft biopsy may be required.
• Obstructive uropathy:
• Chronic pyelonephritis Q: How to manage after rejection?
• Tubulointerstitial diseases (5-10%). A: Short course of very-high-dose steroid is given.
• Systemic inflammatory diseases (5-10%), Other therapies, namely antilymphocyte antibody
e.g. systemic lupus erythematosus (SLE),vasculitis. or plasma exchange, are used in resistant case.
7 • NEPHROLOGY _

Q: Is there any bad effect of repeated blood transfusion Q: What are the complications of immunosuppression?
before kidney transplantation? A: As follows:
A: Repeated transfusion should be avoided as it car- • Infection (see below).
ries a risk of HlA sensitization. But pretreatment • Malignancy (see below).
with multiple transfusions from donor tends to
increase graft survival (in contrast to bone marrow Q: What are the complications after renal
transplantation). transplantation?
A: As follows:
Q: What is CKD and what is ESRD? • Acute rejection characterized by rising of
A: As follows: creatinine, fever, loin pain, hypertension, swelling
• Chronic kidney disease (CKD) is the irreversible of the graft. Urine shows protein, lymphocyte,
deterioration of renal function classically devel- and renal tubular cells. Occurs in 10-30% cases
oping over a period of years. In CKD, glomerular within 6 months. Graft biopsy shows immune
filtration rate (GFR)> 10 ml./rnin, medical treatment cell infiltrate and tubular damage. Treatment:
is still possible to maintain renal function. High-dose methylprednisolone; in resistant cases
• End-stage renal disease or failure (ESRD) is a stage antithymocyte globulin (ATG), antilymphocyte
when renal replacement therapy is compulsory globulin (ALG) or OIT3 may be used.
by either dialysis or renal transplantation, • Chronic rejection: Usually occurs after 6
without which death is likely. Here GFR <5 mLI months. The patient presents with gradual rise
min and medical treatment cannot maintain of creatinine and proteinuria. Graft biopsy shows
renal function. vascular change, fibrosis and tubular atrophy. It is
not responsive to increased immunosuppression.
Q: What are the contraindications of renal
• Infection: Cytomegalovirus (CMV), Pneumocystis
transplantation?
jiroveci, oral candidiasis, polioma virus. Bacterial
A: As follows:
infection is common in first few months.
1. Absolute:
• Complication of immunosuppressive drugs
• Active malignancy: A period of at least 2 years including steroid.
of complete remission is recommended for
• Acute tubular necrosis (ATN): It is the commonest
. most tumours.
cause of cadaveric graft dysfunction (40-50%).
• Active vasculitis or recent anti-GBM (anti- It is associated with a worse long-term outcome
glomerular basement membrane) disease. and increases the risk of graft rejection.
• Severe heart disease or any severe comorbid
• Technical failures: Occlusion or stenosis of the
condition. arterial anastomosis, occlusion of the venous
• Severe occlusive aortoiliac vascular disease. anastomosis and urinary leaks.
2. Relative: • Post-transplantation lymphoproliferative
• Age: While practice varies, transplants are disorder: Epstein-Barr VilUS(EBV)-associated
not routinely offered to very young children malignancies (such as lymphoma) are common
«1 year) or older people (>75 years). in patients who receive biological agents and in
• High risk of disease recurrence in the children.
transplant kidney. • Chronic allograft nephropathy: Most common
• Disease of the lower urinary tract: In patients cause of late graft failure.
with impaired bladder function, stricture • Malignancy: Skin tumour (including basal and
urethra. (An ileal conduit may be considered.) squamous cell carcinoma), renal, cervical and
• Significant comorbidity. vaginal.
Q: What drugs are used to prevent rejection (or man- • Hypertension.
agement after transplantation)? • Atherosclerosis.
A: Usually a combination of: • Recurrence of renal disease.
• Prednisolone. Complication of renal transplantation (Remember
• Cyclosporine or tacrolimus. the formula, 'TROPICAL'):
• Azathioprine or mycophenolate rnofetil/ • T-Thrombosis of graft kidney artery and vein.
sirolimus or everolimus. • R-Rejection of graft kidney.
_ SHORT CASES IN CLINICAL MEDICINE

• O-Obstruction of graft ureter with perinephrichae- Prognosis after kidney transplantation:


matoma, seroma, urinoma, or lymphocoele.
Survival in transplant from living donor:
• P-Primary disease recurrence. The most
common recurrence is MCGN type-Il (80-100%). • I-year survival 85-90%.
• I-Infection: Bacterial [any, tuberculosis (TB)], • 3-year survival 70-75%.
Viral (CMV, chicken pox, polioma VilUS), fungal, • 5-year survival 60-65%.
Cryptococcus neoformans), Parasite (Pneumocystis jiraveci, • 10-year survival 50-55%.
isospora, Cydosporidium, Microspora, Giardia).
• C-Cyclosporine toxicity and other immunosup- Survival in transplant from cadaver donor:
pressive drug toxicities. • 96% patient survival and 92% graft survival at
• A-Acute tubular necrosis. 1 year.
• L-Leakage of graft ureter due to error or • 87% patient survival and 82% graft survival at
ischaemia. 5 year.

Nephrotic Syndrome
Instructions by the examiner:
• Look at the face of the patient.
• Do the general examination.

Presentation of a Case if-------,

• The patient is grossly oedematous.


• The face is puffy with baggy eyelids.
• Pitting oedema is present.

My diagnosis is nephrotic syndrome (NS).

Q: What are your differential diagnoses?


Puffy face with baggy eyelids in nephrotic syndrome
A: As follows:
• Acute glomerulonephritis.
• Congestive cardiac failure.
• Cirrhosis of the liver.
• Hypoproteinaemia due to malnutrition or
malabsorption.

Ascites and oedema in nephrotic syndrome

Q: What history would you like to take in NS?


A: As follows:
• Diabetes mellitus.
• Malignancy (lymphoma, leukaemia).
• Drugs, e.g. CaptopriJ, NSAlDs, penicillamine, gold.
Nephrotic syndrome • Skin rash, arthritis, arthralgia, alopecia (SL£).
7 • NEPHROLOGY __

• History of other diseases like malaria, leprosy, 1. Primary renal disease: All glomerulonephritides
syphilis, hepatitis B virus (HBV), hepatitis C virus (80%):
(HCV), amyloidosis, vasculitis. • Minimal-change glomerular disease
• Family history of sickle cell disease, Alport (commonest in children).
syndrome, nail patella syndrome. • Membranous CN (commonest in adult).
• Mesangiocapillary and proliferative
Q: What bedside test do you like to do? glomerulonephritis. z
(1)
A: Bedside urine examination (shows massive • Focal and segmental glomerulosclerosis. "0

proteinuria).
~
• 19A nephropathy. o
0-
2. Secondary to other disease: ~
Q: What investigations should be done in nephritic
• Diabetic nephropathy.
syndrome?
• Collagen disease, mainly SLE; also rheumatoid
A: As follows:
arthritis (by amyloidosis).
1. Urine R/E: Shows gross proteinuria. Red cells
• Amyloidosis.
and red cell casts are absent. (Also look for urine
• Drugs: Penicillamine (common), captopril,
sugar to exclude diabetic nephropathy.)
gold, mercury.
2. 24-h urinary total protein (more than 3.5 g/24 h
• Neoplastic: Carcinoma (bronchial
is suggestive of nephritic syndrome). carcinoma), lymphoma.
3. Serum total protein, serum albumin and A:C • Infection: Malaria (quartan malaria),
ratio (hypoalbuminaemia). bacterial endocarditis, HBV, HCV, human
4. Serum lipid profile [high cholesterol and high immunodeficiency virus (HIV), secondary
triglycerides (TC) may be present]. syphilis, leprosy.
S. Blood sugar, blood urea, serum creatinine, • Allergies: Bee stings, snake bite, antisnake
serum electrolytes should be done. venom, pollens.
6. USC of the whole abdomen to look for renal Q: What is the commonest cause of nephrotic
pathology. syndrome in children and adult?
7. To find out causes: A: Commonest cause in children is minimal-change
• Blood sugar (to exclude diabetic glomerulonephritis: and the commonest cause in
nephropathy) . adult is membranous glomerulonephritis.
• Chest X-ray (to exclude bronchial carcinoma, Q: What are the lipid abnormalities in NS? What are
lymphoma; also to see bilateral pleural
the mechanisms?
effusion, pericardial effusion).
A: Lipid abnormalities are:
• Antinuclear antibody (ANA), anti-double- • Hypercholesterolaernia.
stranded DNA (anti-dsDNA) (if the history is • High low-density lipoprotein (LDL), very-low-
suggestive of SLE). density lipoprotein (VLDL) and intermediate-
• Perinuclear antineutrophil cytoplasmic density lipoprotein (1OL).
antibodies (P-ANCA) and cytoplasmic anti- • HDL shows no change, or it may be low.
neutrophil cytoplasmic antibodies (C-ANCA)
The mechanisms are:
(if the history is suggestive of vasculitis).
• Increased synthesis of lipoproteins by the liver,
• Hepatitis B surface antigen (HBsAg) and anti- secondary to hypoalburninaernia.
HCV screening. • Reduced clearance of triglyceride-bearing
• Complement C3 and C4. lipoprotein (chylomicron and VLDL) in direct
• Renal biopsy [to see the type of glomerulone- response to albuminuria.
phritis (CN), whether minimal, membranous
As a result, there is high rate of atherosclerosis.
or membranoproliferative]. (This will guide
for diagnosis, therapy and prognosis.) Q: What are the mechanisms of proteinuria?
A: There is increased permeability of the glomerular
Q: What is nephrotic syndrome? What are the causes? capiJlarywall due to:
A: Nephrotic syndrome is characterized by • Reduction of fixed negatively charged protein
generalized oedema, massive proteinuria and molecules in glomerular capillary wall, which
hypoalbuminaemia (with or without hyper- repels negatively charged protein molecules and
lipidaemia]. Causes are: allows proteins to pass through the pores.
__ SHORT CASES IN CLINICAL MEDICINE

• Damage to the glomerular basement membrane • If there is frequent relapse or there is a need for
that leads to increase in the size and number of high-dose steroid or incomplete response to
pores allowing passage of larger molecules. steroid: Cyclophosphamide (2.0 rug/kg/day
for 8-12 weeks) and mycophenolate mofetil
Q: Why oedema in NS?
with low-dose steroid should be given.
A: Previously it was thought that reduction of albumin
causes low plasma oncotic pressure leading to 7. In membranous glomerulopathy, the following
oedema. But recent view is that oncotic pressure is treatment may be given:
not changed in NS; oedema is due to sodium reten- • lnj. methylprednisolone 500-1000 mg
tion in the extracellular compartment. Also, there is intravenous (IV) for 3 days followed by oraL
change in molecular barrier, which causes oedema. prednisolone 0.5 mg/kg/day for 27 days in
first, third and fifth months and tab. cyclo-
Q: What is the blood pressure in NS? phosphamide 2 mg/kg/day or chlorambucil
A: Usually blood pressure is normal, even low. If 0.2 mg/kg/day for 30 days in second, fourth
there is hypertension, usually it is secondary to and sixth months.
underlying diseases like SLE with renal involve- • Chlorambucil (0.2 mg/kg/day in months
ment, polyarteritis nodosa. diabetic nephropathy or 2, 4 and 6 alternating with oral prednisolone
terminal stage of nephrotic syndrome. 0.4 rug/kg/day in months I, 3 and 5) or
Q: How to treat nephrotic syndrome?
cyclophosphamide (1.5-2.5 rug/kg/day for
A: As follows: 6-12 months with 1 mg/kg/day of oral
1. Fluid restriction: Depending on previous day's
prednisolone on alternate days for the first
output and patient's oedema status (average - 2 months) are equally effective. However, this
treatment is reserved for patients with severe
500-1000 mL/day).
2. Salt restriction. or prolonged nephrosis (proteinuria >6 g/day
3. High-protein diet (2 g/kg/day). In severe case, for >6 months), or renal insufficiency and
intravenous salt-poor albumin may be given hypertension.
in diuretic resistant patients and in those with • Cyclosporine and mycophenolate mofetil
oliguria and uraemia in the absence of severe with oral steroid may be used.
glomerular damage, e.g. in minimal change • Anti-CD20 antibodies (rituximab) have been
nephropathy. It helps in diuresis. Protein intake shown to improve renal function, reduce
should be restricted in patient with impaired proteinuria and increase the serum albumin.
renal function. • Ora] high-dose corticosteroid and azathio-
4. Diuretics: Loop diuretics (frusemide, prine are ineffective.
bumetanide). Potassium-sparing diuretics 8. Focal and segmental glomerulosclerosis:
(spironolactone) may be added. • Symptomatic and supportive treatment.
5. ACE inhibitor or angiotensin II receptor • Steroid is effective in 40% cases. Tab.
antagonist is used in all types of GN (for their prednisolone 1 mg/kg/day for 3 months and
antiproteinuric properties. These drugs reduce then tapered. Total duration of treatment is
proteinuria by lowering glomerular capillary at least about 6 months to 1 year. Most cases
filtration pressure). progress to renal failure.
6. In case of minimal-change disease: • If no response: Mycophenolate mofetil
• Prednisolone: 60 rng/m" body surface area 1-2 g/day or cyclosporine 5-6 mg/kg/day for
(maximum 80 mg/ day) is given for 4-6 weeks, 3 months and then tapered and maintained
followed by 40 mg/m? every alternate day for up to 15 months.
a further 4-6 weeks. More than 95% responds • Tacrolimus 0.05 mg/kg/day may be tried
(in children). Alternately, prednisolone 1 mg/ (occasionally effective).
kg/day up to response (urine protein free) or • Renal transplantation can be done in renal
3 months followed by tapering the dose in failure, but may relapse after transplantation.
next 3 months. 9. Mesangiocapillary or membranoproliferative
• If there is relapse after withdrawal of steroid, it GN:
should be given again with gradual withdrawal. • Only symptomatic and supportive treatment.
Some patients may require low-dose mainte- • No specific treatment.
nance dose (5-10 mg/day) for 3-6 months. • Aspirin may be given.
7 • NEPHROLOGY _

10. Treatment of complication: • Increased synthesis of dotting factors:


• If infection: Antibiotic is given. Pneumococcal Factors V, VIII and fibrinogen.
vaccine is recommended. • Other factors: Thrombocytosis and over diu-
• Venous thrombosis: To prevent, prolong bed resis resulting in dehydration, reduced renal
rest should be avoided. Prophylactic heparin blood flow and increased viscosity, prolonged
if immobile (enoxaparin may be given), bed-ridden condition.
followed by oral anticoagulant. 2. Also, there is hyperlipidaernia, commonly high
• For hyperlipidaemia: Statin may be added. LOL, VLOL, cholesterol and triglyceride. So,
11. Treatment of underlying cause, if any. there is more atherosclerosis.
Q: What is the prognosis in NS? These predispose to increased venous thrombosis
A: It depends on the type of NS. that occurs especially in renal vein.
• Prognosis of minimal-change disease in children In nephrotic syndrome, if there is loin pain, hae-
is excellent. Remission and relapse may occur maturia and deterioration of renal function, it is
most commonly in children and less in adult. highly suggestive of renal vein thrombosis. It is more
CKO does not occur. common in membranous nephropathy, mesangi-
• In membranous nephropathy, one-third may ocapillary glomerulonephritis and amyloidosis.
remit spontaneously, one-third remains in neph- To diagnose renal vein thrombosis, Doppler
rotic syndrome and one-third shows progressive ultrasound, CT or MRI, sometimes renal angiogram
loss of renal function. (venous phase) may be done.
• In focal segmental glomerulosclerosis (FSGS)
and mesangiocapillary GN: Prognosis is bad. Treatment: Anticoagulant heparin for 5-7 days; then
• IgA nephropathy: Course of the disease is indo- warfarin for 3-6 month.
lent. ESRO occurs in 20 years.
Q: What are the complications of nephrotic
Read the Following Topics in Relation to
syndrome?
A: As follows:
Nephrology
• Hypercoagulability leading to venous thrombosis Types of haematuria:
[especially renal vein thrombosis, also deep vein • Initial haematuria: Presence of blood at the
thrombosis (OVT)) and pulmonary embolism. beginning of micturition, usually due to penile
• Infections such as pneumococcal infection (may urethral cause.
cause peritonitis and septicaemia), cellulitis, • Terminal haematuria: Presence of blood at the
streptococcal infection, etc. due to loss of end of micturition, usually due to bladder neck or
immunoglobulin (lgG deficiency) complements. prostatic urethral cause.
• Hyperlipidaemia leading to atherosclerosis. • Total haematuria: Presence of blood throughout
• Oliguric renal failure. micturition, usually due to bladder or urinary
• May cause bilateral pleural effusion, pericardia I tract disease or blood dyscrasia or excess
effusion. anticoagulant.
• Loss of thyroxin-binding globulin that causes low
Urinary incontinence: It means urine leaks involuntar-
Ff3 and FT4, which leads to hypothyroidism.
ily. It is of four types:
• Loss of transferrin and iron, resulting in iron-
• Stress incontinence: Loss of urine with activity
deficiency anaemia.
such as coughing, sneezing, lifting any object,
• Loss of vitamin D-binding protein, leading to
exercise, etc.
osteomalacia.
• Urge incontinence: Uncontrolled loss of urine
Q: What are the mechanisms of renal vein thrombosis preceded by strong urge to void urine. It is
in nephrotic syndrome? due to inflammatory condition or neurogenic
A: Mechanisms are as follows: bladder.
1. In nephrotic syndrome, there is hypercoagulable • Overflow incontinence: Chronic urinary retention.
state due to: associated with this condition.
• Loss of inhibitors of coagulation in urine such • Total incontinence: Patient loses urine at any
as antithrombin Ill, protein C and S, and also time and at any position due to loss of sphincter
loss of fibrinolytic factor (plasminogen). efficacy.
SHORT CASES IN CLINICAL MEDICINE

Causes of red or dark urine: Causes of sterile pyuria (urine shows pus cells but is
• Haernaturia (red). negative on culture):
• Haemoglobinuria (dark). • Renal tuberculosis.
• Myoglobinuria (in rhabdomyolysis). • Nongonococcal urethritis (Chlamydia,
• Food dye (beet root). ureaplasma, etc.).
• Drugs: Rifampicin (orange), L-dopa (dark on • Schistosomiasis.
standing), phenolphthalein (pink) and selma • Tubulointerstitial nephritis.
(orange). • Papillary necrosis.
• Acute intermittent porphyria (urine is dark, if kept • Polycystic kidney.
for long time). • Haemorrhagic cystitis (due to
• Alkaptonuria (urine becomes black, if kept for cyclophosphamide) .
long time). • Inadequately treated UTI.
• Prostatitis.
Causes of painless haematuria:
• Renal cell carcinoma. Q: How to differentiate between haematuria and
• PRO. haemoglobinuria?
• Papilloma of urinary bladder. A: As follows:
• Schistosomiasis.
Points Haematuria Haemoglobinuria
• Benign hypertrophy of prostate.
Urine Red Dark
• Bleeding disorder.
• Heparin, antiplatelet-drug therapy. Microscopy RBC present RBC absent
CHAPTER 8

NEUROLOGY
"The man who confesses his ignorance shows it once, he who tries to conceal it shows many times"
-AJapanese Proverb

Introduction
Neurological cases are the most feared in the exami- Occasionally, examiner may ask, 'Examine the legs or
nation. However, these cases are probably the most lower limbs'.
straightforward in terms of diagnosis and defining One must remember that there may be nonneuro-
the site of lesion. To attain efficiency and skillfulness, logical cases. Candidates often mistakenly perform the
a good deal of practice is required that will suffice to neurological examination only. Hence, never forget the
score highly. nonneurological cases and examine properly keeping
Candidates are expected to know: 'What is the these in mind.
lesion? What is the site of lesion? What is the cause
Nonneurological cases may be (look very carefully
of lesion'? The signs need to be elicited carefully for
and diagnosis may be obvious):
exact anatomical localization of any lesion.
• Swelling of knee joints (arthritis or effusion).
Candidates are usually asked:
• Erythema nodosum.
• Perform the neurological examination of lower • Pretibial myxoedema.
limbs (or upper limbs). • Systemic sclerosis.
• Look at the leg. What is your finding? (wasting, • Diabetic foot.
fasciculation or pes cavus). • Unilateral or bilateral leg swelling.
• Examine the hands (wasting or claw hands).
• Vasculitis.
• Talk to the patient (dysarthria, dysphasia or hoarse
• Necrobiosis lipoidica diabeticorum.
voice).
• Bowing tibias (rickets or Paget disease or congenital
• Look at the patient. What is your diagnosis?
(Parkinsonian face or involuntary movement, anomaly).
e.g. tremor, chorea.) Even if asked to perform neurological examination,
• Examine the cranial nerves or examine the facial quickly look carefully. Aftergood visual survey, if nothing
nerve. is obvious, proceed with the neurological examination.

Neurological Examination of lower limb

Most likely cases in the examination are: N.B. If urinary catheter is present, it indicates spinal cord
compression or disease, mostly multiple sclerosis
• Spastic paraplegia or flaccid paraplegia. (MS).
• Peripheral neuropathy.
• Friedreich ataxia. Inspection:
• Motor neuron disease (MND). • Wasting (mention the location-right or left or
• Old poliomyelitis. both or thigh or leg).
Proceed as follows: • Skin change (shiny, pigmented, rough, ulceration,
Introduce yourself and ensure that lower limbs are well hair loss and gangrene). See the tip of the toes
exposed (with permission). The patient should be lying including sole.
in supine position. • One legis smaller than the other (old poliomyelitis).
__ SHORT CASES IN CLINICAL MEDICINE

• Pes cavus (right or left or both feet). • Dorsiflexion of ankle: 'Push your foot upwards
• Fasciculation (if not visible, tap the muscle with against my hand'. (Prime movers are tibialis
your fingers). anterior, extensor digitorum longus and
• Any swelling in calf muscles (pseudohypertrophy extensor hallucis Iongus=-La and LS.)
of calf muscles). • Inversion of foot: 'Push your foot inwards
• Joint abnormality (deformity or swelling, the signs against my hand'. (Prime movers are tibialis
Gl of inflammation). anterior and posterior=-Ls and Sl.)
o • Eversion of foot: 'Push your foot outwards
-0 Palpation:
8 against my hand'. (Prime movers are peroneus
Q)
1. Bulk of the muscles (measure with tape from a
Z longus and brevis-LS and 81.)
particular point):
• Extension of great toe: 'Push your great toe
• Unilateral wasting (old poliomyelitis).
upwards and do not let me push it down'.
• Generalized wasting [MND, polyneuropathy,
(Prime mover is extensor hallucis longus-LS.)
lower motor neuron (LMN) lesion].
S. Reflexes:
• Isolated anterior wasting in thigh (diabetic
• Knee (L3 and L4).
amyotrophy).
• Ankle (Sl and 82).
• Wasting in the leg that stops suddenly at a
• Plantar (LS, S1 and S2): Mention to the patient,
certain level (Charcot-Marie- Tooth disease).
'T am going to tickle the bottom of your foot',
2. Tone: Tell the patient, 'Keep your limbs relaxed':
with an orange stick at the outer portion of the
• Lift the leg and allow it to fall.
sole. Report according to your finding, 'Plantar
• Palpate the muscle and perform side-to-side
is extensor or flexor or equivocal'.
movement of the limb.
• Lastly, passive movement of the limb (in N.B. Remember the following points:
irregular fashion-flexion and extension). • If reflexes are absent, never forget to do
3. Test for clonus [ankle and patella). reinforcement. Ask the patient to pull his or her
4. Muscle power (against resistance): If any weak- clasped hands outwards (Iendrassikmanoeuvre),
ness, mention grading of weakness. To test, ask or clench the teeth and see the reflex.
the patient to follow your instructions as follows • Examine for Oppenheim sign (by pressing
(press by your hand): heavily the shin along the inner border of tibia
.• Hip flexion: 'Raise your leg straight; do not let going down) and Gordon sign (by squeezing
me push it down'. (Prime mover is iliopsoas- the Achilles tendon). If these signs are positive,
11 and L2.) there will be extensor plantar response
• Hip extension: 'Push your leg down; do not let (indicates extensive pyramidal lesion).
me pull it up'. (Prime mover is glutei muscles-
L4 and LS.) 6. Superficial reflexes:
• Hip adduction: 'Push your thighs inwards; do • Abdominal reflex (T6-Tll): Elicited by lightly
not let me move them apart'. (Prime movers are stroking the abdominal wall diagonally towards
adductors of thigh, such as adductor longus, umbilicus in each of the four quadrants of
brevis and magnus-L2, L3 and L4.) abdomen. If positive, reflex contraction of
• Hip abduction: 'Push your thigh outwards. Do abdominal wall occurs. Absent in upper motor
not let me push them inward'. (Prime mover- neuron (UMN) lesion. Early loss in MS.
gluteus medius and minim us, sartorius and • Cremasteric reflex (11-L2): Stroke the inner
tensor fasciae latae-L4, LS and Sl.) part of thigh in downward direction. Normally,
• Knee flexion: 'Bend your knees; do not let me contraction of cremasteric muscles pulls up the
straighten them'. (Prime mover is hamstrings scrotum and testis on the side stroked.
such as biceps femoris, semimembranosus and 7. Coordination (Explain and show it to the patient.):
semitendinosus-LSi Sl and S2.) • Heel-shin test: 'Please raise your leg, put your
• Knee extension: 'Straighten your knees: do not heel upon the knee of other leg and run it
let me stop doing it'. (Prime mover is quadriceps smoothly along the shin'. (Repeat the same for
femoris-L3 and L4.) other leg.)
• Plantar flexion of ankle: 'Push your foot down- • Foot taping test: Keep your hand at a little distance
wards against my hand'. (Prime mover is from ball of patient's foot and ask the patient to
gastrocnemius, plantaris and soleus-S1 and S2.) tap it rapidly on your hand (dysdiadochokinesia).
8 • NEUROLOGY __

8. Sensory test: Explain to the patient with light touch


by cotton-wool in normal area such as forehead. Signs of upper motor neuron (UMN) lesion
Ask the patient 'Can you feel it'? Now touch the • Weakness or paralysis.
leg or foot. Ask the patient, 'Can you feel it'? Tf no, • Increased tone (hypertonia, spastic, may be clasp
continue to touch above, until the patient can feel knife).
to find out the level of sensory loss. • Exaggerated tendon reflex, may be clonus and
• Light touch (cotton-wool). absent abdominal reflex. z
• Pin prick. • Plantar response: Extensor. ~
go
• Vibration sense: Always explain the patient • No wasting (occasionally due to disuse). 0-
first, should be tested on medial malleolus; ??J
• Normal electrical excitability of muscles.
if impaired, it may also be tested in knee and • Upper limb drift: Present (see below).
anterior superior iliac spine.
• Position sense (in great toe; always explain this
to the patient.)
Q: What are the sites of UMN lesion?
Perform the test according to the nerve distribution A: Tn the:
• Outer thigh L2 (upper thigh). • Cerebral cortex
• Inner thigh L3 (also around knee). • Internal capsule
• Outer leg LS (up to medial foot). • Brainstem
• Inner leg L4. • Descending tracts up to anterior horn cells of
• Medial foot LS. spinal cord.
• Lateral foot S1.
9. Gait:
• Ask the patient to walk, look for any abnormali- Signs of LMN lesion
ties and also ask to turn quickly (in Parkinsonism,
• Weakness or paralysis (flaccid).
unable to turn quickly).
• Hypotonia.
• Ask the patient to walk heel-to-toe (to exclude
• Loss of all reflexes
midline cerebellar lesion-ataxia).
• Ask the patient to walk on toes (S1 lesion will
make it impossible).
• Ask the patient to walk on heel (L4 and LS



Wasting of involved muscles.
Fasciculation of affected muscles.
Plantar: Normal or absent.
_j
lesion will make it impossible).
10. Test for Rombergism (Ask the patient to stand Q: What are the sites of LMN lesion?
with legs together and close the eyes. In positive A: In the motor pathway from anterior horn cells (or
case, the patient tends to sway or fall. Be careful to cranial nerve nucleus) via peripheral nerve to motor
prated the patient from falling.). If positive, indi- end plate.
cates sensory ataxia ldue to subacute combined
degeneration (SCD) and tabes dorsalis].
11. Finally, look at the spine to see any deformity, scar, Signs of extrapyramidal lesion
gibbus and local tenderness.
• Rigidity (leadpipe or cogwheel).
Read the following topics carefully. • Hypokinesia or bradykinesia (poverty of mov-
ement) or akinesia (no movement).
Grading of muscular weakness, Medical
• Involuntary movements (tremor, chorea,
Research Council (MRC) criteria athetosis, dystonia and hemiballismus).
• Grade 0: Complete paralysis. Causes of hypertonia
• Grade 1: A flicker of contraction only.
• Grade 2: Power detectable, when gravity is • UMN lesion (spastic, may be clasp knife).
excluded by postural adjustment. • Extrapyramidal lesion (lead pipe or cogwheel).
• Grade 3: Limb can be held against gravity, but not • Conversion disorder (rigidity continues to increase
against examiner's resistance. with more and more passive movement).
• Grade 4: There is some degree of weakness. • Others: Catatonic state, tetanus and strychnine
• Grade 5: Normal power. poisoning.
__ SHORT CASES IN CLINICAL MEDICINE

Q: What are the causes of loss of tendon reflex?


Causes of hypotonia A: As follows:

_j
• LMN lesion (due to any cause). • LMN lesion.
• Cerebellar lesion (knee jerk may be pendular). • Peripheral neuropathy.
• Dorsal column lesion. • Dorsal column lesion.
• Polyneuropathy. • Hypokalaemia or hyperkalaemia.
• Hypokalaemia or hyperkalaemia. Isolated loss of jerk indicates radicu)opathy of the
• Drug: Any muscle relaxant. affected segment (such as disc prolapse):
• Loss of biceps (CS,61esion).
Q: What is upper limb drift?
• Loss of triceps (C6,7Iesion).
A: Normally, outstretched hands in front are held sym-
• Loss of ankle jerk (SIlesian).
metrically even when the eyes are closed. In UMN
lesion, when the upper limbs are outstretched with
Q: What are the causes of extensor plantar response?
the palm uppermost, affected limb drifts down-
A: As follows:
wards and medially, forearm tends to pronate and
• first year of life (due to lack of myelination of
hands flex slightly at fingers.
pyramidal tract. Myelination occurs in 6-12
Q: How reinforcement helps or acts? months).
A: Reinforcement acts by increasing the excitability of • Sleep.
anterior horn cells and by increasing the sensitiv- • UMN lesion.
ity of muscle spindle primary sensory endings to • Deep coma (due to any cause).
stretch by increased gamma fusimotor drive. • Postepileptic period (after grand mal seizure).

Spastic Paraplegia (Spinal Cord Compression)


Usual instructions are:
• Vibration and position senses are normal.
• Examine the lower limbs or perform the neurolog-
• There is no spinal deformity or tenderness or any
ical examination of the lower limbs.
surgical scar over the spine.
Proceed to examine the lower limbs as described before. • Gait: Scissor gait.
If there is any sensory loss, find out the level. Always • Rombergism is absent.
look at the back to find any scar, bony deformity, and
gibbus or local tenderness in spine. Later, examine the
upper limbs also.
My diagnosis is spastic paraplegia, more likely due to
spinal cord compression.
Presentation of a Case
Q: What are the causes of spastic paraplegia?
• The patient has slight wasting of right or left, or A: As follows (remember the age and also sensory loss):
both thigh or leg (mention, if any). • Spinal cord compression due to any cause (see
• There is hypertonia in both lower limbs (may be below).
clasp knife). • Demyelinating disease (in MS).
• Both flexor and extensor muscles of lower limbs • Motor neuron disease (in middle-aged or elderly).
are weak (mention the grade). • Friedreich ataxia (in early age).
• Knee and ankle jerks are exaggerated on both sides. • Hereditary spastic paraplegia.
• Plantar response is extensor on both sides. • SCD.
• There is ankle clonus in right or left or both
(mention, if any). Q: What are the cardinal signs of spinal cord
• Coordination is impaired in right or left or both compression?
(mention, if any). A: As follows:
• Loss of sensation in right or left or both (mention, • UMN signs (spastic paraplegia).
up to where). • Segmental sensory loss (sensory loss up to a
particular segmental level).
8 • NEUROLOGY __

Q: What are the other features of spinal cord Q: What are the causes of paraplegia of sudden onset?
compression? A: As follows:
A: As follows: • Trauma (to vertebral column).
• Sphincter disturbance: Common (urinary reten- • Collapse of vertebra due to any cause.
tion and loss of bladder control). • MS.
• Root pain: Frequent at the site of compression. • Anterior spinal artery occlusion.
• Pain radiates in a band around the chest (thoracic • Dissecting aneurysm.
compression) . • Haematomyelia.
• Postinfectious or postvaccinal myelitis
Q: Could it be due to MND?
(transverse myelitis).
A: Unlikely, because in MND there will be:
• Paraplegia in flexion: Due to partial transection
• No sensory loss (very important sign).
of spinal cord where the limbs are involuntarily
• Fasciculation. flexed in hips and knees (because extensor
• Age (usually above 40 years).
muscles are more paralysed than flexors).
Q: Could it be due to Friedreich ataxia? • Paraplegia in extension: Due to complete transec-
A: In Friedreich ataxia, there will be: tion of spinal cord. (Flexors are more paralysed
• Early age. than extensors. There is coincidental involvement
• Pes cavus and kyphosis. of spinal extrapyramidal tracts.)
• Loss of knee and ankle jerk, plantar is extensor.
• Signs of cerebellar lesion. Q: What are the commonest causes of spastic
Signs of posterior column lesion (loss of vibration paraplegia?
and position sense). A: As follows (7 Ts):
• Trauma.
Q: What are the other findings in Friedreich ataxia?
• Tuberculosis (Pott disease).
A: Optic atrophy, high arched palate, deafness and • Tumour (meningioma, neurofibroma,
cardiomyopathy. lymphoma, leukaemia, myeloma, glioma).
Q: Could it be due to subacute combined • Transverse myelitis.
degeneration? • Tabes dorsalis.
A: Unlikely, as in SCD there will be: • Twelve (B12 deficiency).
• Peripheral neuropathy. • Thrombosis.
• Posterior column lesion.
• Knee and ankle jerk absent (knee jerk may be brisk; Q: What are the causes of spinal cord compression?
plantar may be extensor). A: As follows:
• Romberg sign is positive. 1. Extradural (lesion in vertebral column):
• Others: Anaemia and smooth shiny tongue with • Trauma.
atrophy of papilla. • Tuberculosis (TE) of spine (Pott disease).
N.B. Remember to diagnose the cause of spastic • Lymphoma.
• Secondary deposit (elderly).
paraplegia:
• Multiple myeloma (elderly).
• IJ spastic paraplegia with sensory loss with a
definite upper limit: It is likely to be due to spinal • Abscess (paravertebral).
cord compression. 2. Intradural (extramedullary and intramedullary):
• If plantar response is extensor, sensory loss in a. Extramedullary causes (within dura):
glove and stocking pattern with loss of vibration • Meningioma.
and position sense: It is likely to be due to SCD. • Neurofibroma.
• If cerebellar signs are present: The diagnosis is MS • Secondary deposit (elderly).
or Friedreich ataxia. • Lymphoma.
• In young patient with pes cavus, dorsal colurnn • Leukaemia.
lesion and cerebellar signs: The likely diagnosis is b. Intramedullary causes:
Friedreich ataxia. • Glioma.
• If spastic paraplegia, but no sensory loss: The • Ependymoma.
likely diagnosis is MND (amyotrophic lateral • Syringomyelia.
sclerosis; also LMN lesion in upper limb). • Haematomyelia.
__ SHORT CASES IN CLINICAL MEDICINE

Q: What are the features of cerebral lesion causing


paraplegia?
A: As follows:
• There is bladder disturbance (urinary retention).
• Cortical type of sensory loss.
• Other features: Headache, vomiting, convulsion,
Jacksonian fit.
N.B. Lower limbs and micturition centre are repre-
sented in the paracentral lobule. Hence, lesion
Gibbus in tuberculous spondylitis in this area produces paraplegia associated
with bladder dysfunction.
Q: What are the noncompressive causes of spastic
paraparesis or paraplegia?
A: As follows:
• MND (e.g. amyotrophic lateral sclerosis).
• Subacute combined degeneration.
• Transverse myelitis.
• MS.
• Friedreich ataxia.
• Lathyrism.
• Syringomyelia.
• Vascular disease of the cord.
• Hereditary spastic paraplegia.
Cerebral palsy-spastic diplegia • Tropical spastic paraplegia.
• Postvaccination.
Q: How to find out the sensory level in spinal cord
• Syphilitic amyotrophy.
compression?
• Nonmetastatic manifestation of malignancy.
A: In the following ways:
• Radiation myelopathy.
Vertebral level Spinal cord segment • Functional.
Cervical vertebrae Add 1 According to the type ofIesion:
Upper-thoracic vertebrae (T1-T6) Add 2 • Demyelinating: MS, acute demyelinating enceph-
Mid-thoracic vertebrae (T7-T9)
alomyelitis (ADEM).
Add 3
• Inflammatory: Sarcoidosis, postviral,
Xth thoracic vertebra Lumbar 1 and 2
postvaccinal.
Xlth thoracic vertebra Lumbar 3 and 4
• Infective: HlV, Herpes zoster, Herpes simplex,
Xllth thoracic vertebra Lumbar 5 syphilis.
First lumbar vertebra Sacral and coccygeal • Degenerative: MND, familial spastic paraplegia
cord segments (FSP), SCD, Friedreich ataxia (FA).
• Vascular: Anterior spinal artery infarction,
Q: What are the causes of spastic paraplegia due to
intramedullary haemorrhage, spinal arteriov-
cerebral lesion ?
enous malformation (AVM).
A: As follows:
• Toxic: Lathyrism, radiation.
• Parasagittal meningioma (usually involving falx
meningioma). Q: What are the differences between compressive and
• Thrombosis of superior longitudinal sinus. noncompressive paraplegia?
• Thrombosis of unimpaired anterior cerebral A: As follows:
artery.
• Multiple cerebral infarctions. Topic Compressive Noncompressive
• Hydrocephalus. Onset Gradual May be acute
• Trauma. Symmetry Asymmetrical Usually symmetrical
• In children, cerebral palsy (usual lesion is bilat- Root pain Present Absent
eral parasagirtal cortical lesion).
8 • NEUROLOGY __

Bony deformity Present Absent Position of lower Extended Flexed

Present Absent limbs


Bony tenderness
Absent Deep reflexes Exaggerated Lessexaggerated
Hyperaesthesia or Present
girdle-like sensation Clonus Present Absent

Upper level of Present Absent Mass reflex Absent May be present


sensory loss Bladder Precipitancy Automatic bladder
Flexor spasm Common Uncommon
Bladder and bowel Usually early Usually late feature; Q: What is mass reflex?
involvement feature may be early in acute A: Following stimulation of skin of the lower limbs or
transverse myelitis. lower abdominal wall, there is reflex flexion of the
lower trunk muscles and the lower limbs, evacua-
tion of the bladder, bowel and semen, and sweating
Q: What investigations are done in spastic paraplegia?
called mass reflex. This reflex indicates severe spinal
A: As follows:
cord lesion.
• X-ray oflumbodorsal spine Ianteroposterior (AP)
and lateral view]. Q: What are the causes of absent ankle jerk, but extensor
• cr scan or MRJ of spine. plantar response?
• Lumbar puncture and cerebrospinal fluid (CSF) A: As follows:
study. • Subacute combined degeneration of the spinal
• Other tests on the basis of cause (e.g, tuberculosis cord.
and multiple myeloma). • Friedreich ataxia.
• MS.
Q: What is paraplegia in flexion and paraplegia in • Taboparesis.
extension? • Diabetes mellitus with cervical myelopathy.
A: As follows:
Q: What are the findings of spinal cord compression at
• Paraplegia in extension: It indicates an increase in
different levels of spinal cord?
the extensor muscle tone leading to an extension
A: As follows:
of the lower limbs (hip and knee extended;
• Lesion above C5: UMN lesion in both upper
feet plantar flexed). It is due to involvement of
and lower limbs with loss of sensation in all four
pyram idal tracts from partial transection of spinal
limbs.
cord, but the extrapyramidal tracts (especially
• Lesion at C5: LMN lesion in proximal muscles
vestibulospinal tracts) are intact. May change to
of upper limbs (rhomboid, deltoid, biceps,
paraplegia in flexion if the damage to the spinal
brachioradialis) with segmental loss of sensation
cord is more extensive and the vestibulospinal
and UMN lesion in rest of the upper limbs and in
tracts are destroyed.
lower limbs. Biceps reflex is lost and triceps jerk
• Paraplegia in flexion: Lower limbs are flexed (in
is exaggerated.
hips and knees; feet are dorsiflexed). It is due
• Lesion at C8: LMN lesion and wasting of the
to complete transection of spinal cord. Both
intrinsic muscles of the hand and UMN lesion in
pyramidal and extrapyramidal tracts are affected.
lower limbs. There is segmental loss of sensation.
Paraplegia in extension and paraplegia in flexion • Lesion in thoracic cord: Spastic paraplegia
follows severe injury to the spinal cord. with segmental sensory loss. (Loss of upper
abdominal reflexes at 17 and T8. Loss of lower
Q: What are the differences between paraplegia in abdominal reflexes and upward displacement of
extension and flexion? the umbilicus atTIO and T'l l}.
A: As follows: • Lesion at L1: UMN lesion in lower limbs and
cremasteric reflex is lost (normal abdominal
Paraplegia in Paraplegia in
reflex).
Points extension flexion
• Lesion at L4: LMN lesion and wasting of
Cause Pyramidal lesion Pyramidal and quadriceps; loss of knee jerk but hyperreflexia of
extrapyramidal
ankle jerk and extensor plantar response.
Hypertonia More in extensor More in flexor • Lesion in L5 and Sl: LMN weakness of knee flex-
group of muscles group of muscles ion and hip extension (S1) and abduction (L5),
_ SHORT CASES IN CLINICAL MEDICINE

and calf and foot muscles. Knee jerk is present, • Lead pipe in which resistance is uniform
but no ankle jerk or plantar response. Anal reflex throughout the passive movement (better
is present. seen in elbow and knee).
• Lesion in S3 and S4: No anal reflex, saddle • Cog wheel in which continuous resistance is
sensory loss, normal lower limbs. interrupted by tremor (better seen in the wrist
and ankle joints).
Q: What is the difference between rigidity and

-
~
o
o
....
::l
~
.A;
spasticity?
As follows:
1. Spasticity means increased resistance during the
Q: How to treat spastic paraplegia?
A: As follows:
1. General measures:
Z initial part of passive movement, followed by • Physiotherapy.
lessening of the resistance. • Nutritional support.
• It may be clasp-knife type, in which there is • Care of bowel and bladder (if needed,
more resistance at the onset of movement catheterization) .
followed by sudden loss of resistance. It • Change of position to prevent bedsore.
is due to pyramidal lesions. Spasticity is Special bed may be used.
better felt with attempting extension of • For spasm: Baclofen, tizanidine, gabapentin,
upper limbs and flexion of lower limbs. It is botulinum toxin, etc. Intrathecal baclofen or
associated with other signs of UMN lesion. phenol may be tried.
It involves only the antigravity muscles 2. Specific measures:
(extensors of the upper limbs and flexors of • Neurosurgical intervention: For any tumour,
the lower limbs). selective dorsal rhizotomy. (Cutting selective
2. Rigidity means sustained uniform resistance nerve roots between L2 and S2; it is useful in
during passive movement. Rigidity is found in cerebral palsy patient.)
extrapyramidal lesion and involves all groups of • Orthopaedic measures.
muscles. It may be: • Treatment of primary cause.

Monoplegia (Brown-Sequard Syndrome)


Usual instructions are: Q: What is Browri-Sequard syndrome? What are the
• Examine the lower limbs. Or perform the neuro- causes?
logical examination of the lower limbs. A: It is due to the damage on one side or hemisection
of the spinal cord characterized by:
1. On the side of lesion:
• At the level: Band of hyperaesthesia and LMN
Presentation of a Case It-------, lesion.
(Supposing Right Lower Limb) • Below the level: Loss of vibration and position
sense (posterior column), and UMN lesion
• There is hypertonia and weakness of extensors
(ipsilateral).
and flexors of knee and ankle.
2. Contralateral (opposite) side: Loss of pain and
• Knee and ankle jerks are exaggerated; plantar is
temperature (spinothalamic tract lesion) below
extensor. the level of lesion, giving rise to dissociated sen-
• No sensory impairment, but loss of vibration sory loss.
and joint position sense.
Causes of Brown-Sequard syndrome:
• In the left lower limb: Loss of pain and
temperature extending up to just below umbilicus • Compression of spinal cord tumour (glioma or
(T8,9), but vibration and joint position sense angioma).
and light touch are intact. • MS.
• Myelitis.
• Trauma.
• Radiation myelopathy.
My diagnosis is hemisection of spinal cord in the right The patient complains of numbness of one side, where-
side (Brown-Sequard syndrome). as weakness, heaviness and stiffness on other sides.
N.B. Iflesion is high up (C5,6), there may be hemi- • MRI of the spinal cord.
T plegia as well. • Others: According to suspicion of cause, e.g. CSF
Q: What investigations should be done? study, viral serology, vitamin B12level, syphilitic
A: As follows: serology (if any).
Dorsal
column

Cortico-spinal
tract
Spino-thalamic
tract

Below the lesion


(loss of vibration and ----\
position sense, UMN lesion)

Loss of pain and ----\--1--


temperature

Monoplegia
Usual instructions are: Q: What is the sensory abnormality in
• Perform neurological examination oflower limbs. poliomyelitis?
A: Usually, no sensory abnormality.
Presentation of a Case ~I--------
Q: What is the cause of poliomyelitis?
(Supposing Right Side)
A: It is caused by polio virus, which is an enterovirus in
• Right lower limb is short and pes cavus is present. the picornaviridae family. It is of three types:
• There is wasting of muscles with hypotonia. • Type I: Brunhilde.
• Muscle power is diminished, grade 3/5. • Type II: Lansing.
• Knee and ankle jerks: Diminished (or absent). • Type III: Leon.
• Plantar: Normal (or equivocal).
Q: What are the types of vaccine for polio myelitis?
• There is no sensory abnormality.
A: Two types:
• Vibration and position senses are normal.
• Oral live attenuated polio virus (OPV) or Sabin
• Coordination: Impaired.
vaccine.
• Inactivated polio virus (IPV) or Salk vaccine.
My diagnosis is monoplegia of the right lower limb.
Q: What are the causes? Q: What are the types of polio?
A: As follows: A: They are offour types:
• Old poliomyelitis. • Abortive: Characterized by fever, myalgia, sore
• Hemiatrophy. throat, etc. Self-limiting.
• Nonparalytic: Above features plus signs of
Q: What is the site oflesion in poliomyelitis? meningeal irritation. Complete recovery occurs.
A: Anterior horn cell of the spinal cord (commonly in • Paralytic: Above features that subside in 4-5 days
lumbar region). recur with signs of meningeal irritation, muscular
Q: What is the site oflesion of weakness of one limb or pain followed by asymmetric flaccid paralysis.
monoplegia? • Bulbar polio: Characterized by cranial nerve
A: Brachial plexus (upper limb), lumbosacral plexus involvement and respiratory muscle paralysis.
(lower limb) and central lesion (in motor cortex or Also, soft palate, pharyngeal and laryngeal muscle
partial internal capsule lesion). paralysis is common.
_ SHORT CASES IN CLINICAL MEDICINE

Hemiatrophy Old poliomyelitis

Multiple Sclerosis (Presenting as Spastic Paraplegia)


Usual instructions are: Q: Why not spinal cord compression?
• Perform neurological examination of lower limbs. A: No definite upper limit of sensory loss.
• Examine the eye or fundoscopy (optic atrophy and
Q: If it is MS, what relevants do you want to see?
nystagmus: See Chapter 10, 'Examination of the
A: As follows:
Eye').
• Eye: Nystagmus, and optic neuritis or optic atrophy.
• Test for cerebellar signs (see in the 'Cerebellar
• Signs of cerebellar lesions (see in 'Cerebellar
Lesions' in this chapter).
Lesion' in this chapter).
• Abdominal reflex (early loss).
Presentation of a Case • Urinary incontinence, impotence and constipa-
tion (triad of Steinberg).
• Presents as described in spastic paraplegia; plus
there may be slight sensory abnormality, but no Q: What are the presentations of MS?
definite upper limit. A: Twice more common in females. Age of onset is
20-45 years. Before puberty and after 60 years, it is
My diagnosis is spastic paraplegia, more likely multi- rare. The patient usually presents with:
ple sclerosis (MS). • Weakness of one or more limbs.
• Optic neuritis (patient complains of blurring of
Q: What are the differential diagnoses?
vision).
A: As follows:
• Features of spastic paraplegia (confused with
• Spinal cord compression.
spinal cord compression).
• Subacute combined degeneration.
• Features of cerebellar signs (ataxia and tremor, etc).
• Neurosyphilis.
• Features of brainstern dysfunction (vertigo,
• Neurosarcoid.
diplopia, nystagmus, facial numbness or weak-
Q: What are the causes of bilateral upper motor ness, dysarthria, dysphagia and pyramidal signs
neuron lesion involving the lower limbs? in limbs).
A: As follows: • Bladder dysfunction (incontinence, dribbling
• Spinal cord compression. and hesitancy).
• MS. • Sensory disturbance: Tingling of the extremities
• MND (amyotrophic lateral sclerosis). and light banding sensation around the trunk or
• Hereditary spastic paraplegia. limbs (due to posterior column involvement).
• Transverse myeliltis. • Others (rarely): Epilepsy, trigeminal neuralgia,
• Subacute combined degeneration. facial palsy (may be recurrent), VIth nerve palsy,
• Friedrich ataxia. tonic spasm or brief spasm of limbs, dementia,
• Bilateral cerebral infarction. neuropsychiatric dysfunction, depression.
• Cervical myelopathy. • Euphoria despite disability.
8 • NEUROLOGY _

Common mode of onset: MRI in MS is helpful for the following:


1. Optic neuritis (25%). 1. Reveals asymptomatic plaques in cerebrum,
2. Transverse myelitis (like spinal cord brainstem, optic nerves and spinal cord.
compression) . 2. MS plaques are hyperintence on T2W and even
3. Cerebellar ataxia. more strikingly obvious on fluid attenuated
4. Various brainstern syndromes: Vertigo, facial inversion recovery (FLAIR).
pain or numbness, dysarthria, diplopia. 3. On T2W, several asymmetrical, well-demarcated z
(l)

...,'"
Signs and symptoms of MS (remember the
mnemonic: WATSON)
lesions immediately adjacent to ventricular
surface (periventricular lesion) usually denote
MS. Especially diagnostic are oval or linear
-
o
o
~
regions of demyelination oriented perpendicu-
• W: Weakness. larly to the ventricular surface. When viewed
• A: Ataxia (cerebellar). on sagittal images, they extend outward from
• T: Tremor (cerebellar).

_j
corpus callosum in fimbriated pattern termed
• S: Speech (scanning). 'Dawson fingers'.
• 0: Optic neuritis. 4. Another characteristic pattern is C-shaped par-
• N: Nystagmus. tial ring of enhancement created by rounded
lesion interrupted by gyrus.
Q: What investigations should be done in MS? 5. Lesions that have undergone some degree of
A: As follows:
cavitation are hypo intense on Tl-weighed
1. MRJ of brain and spinal cord: Investigation of
(TlW) images termed 'black holes'.
choice. It shows multiple plaques, hyperintense
6. Serial MRJ can demonstrate progress of disease,
in T2W and FLAIRmainly in the periventricular
e.g. increasing number of lesions.
region, corpus callosum, cerebellar peduncles,
7. MRI changes assume maximum diagnostic sig-
juxtacortical posterior fossa, brainstem and
nificance when they are consistent with the
subpial region of spinal cord. CT scan is not
clinical findings.
sensitive.
2. Lumbar puncture and CSF study. [There is slight Q: What are the clinical courses (or types) of MS?
increase in lymphocyte, increase in total pro- A: As follows:
tein in (40%) cases, and oligoclonal band in
• Relapsing and remitting MS (80-90%): The
(70-90%) cases, mainly IgG on electrophoresis.]
patient suffers from episodes of acute worsening
3. Evoked potential: Mainly YEP (visual evoked
with recovery and remains stable between relapses.
potential) is usually delayed, if there is optic
• Primary progressive MS (10-20%): Gradual neu-
nerve involvement.
rological deterioration from the onset. It usually
4. To exclude other conditions:
begins after 40 years (late onset).
• Chest X-ray (to exclude bronchial carcinoma).
• Secondary progressive MS:Some cases of relapsing
• X-ray of spine (to exclude cord compression).
and remitting course show gradual neurological
• Serum angiotensin converting enzyme (to
deterioration. There may be superimposed acute
exclude sarcoidosis).
relapses.
• Serum BI2 (to exclude subacute combined
degeneration of spinal cord). • Fulminating MS « 10%).
• ANA (to exclude SLE). Q: What is MS? What is the natural history of the
• Antiphospholipid antibodies. disease?
Q: What is the role of MRI? A: It is a demyelinating disorder of central nervous
A: MRI is the single most investigation to confirm the system (CNS) characterized by multiple plaques
diagnosis of MS, positive in 80% cases. It shows of demyelination within the brain and spinal
hyperintense focal periventricular white matter cord, gliosis and varying degrees of inflammation.
lesions in T2-weighted (T2W) images. Although, Usually there is relative preservation ofaxons.
these are typical, but not pathognomonic and Natural history is extremely variable-+may be
may be found in small infarcts, disseminated acute, subacute, insidious, relapsing and remitting,
metastases, Moyamoya disease and inflammatory chronic progressive, spontaneous recovery, and
diseases. rapidly progressive and secondarily progressive.
_ SHORT CASES IN CLINICAL MEDICINE

It is also called disseminated sclerosis, as the weakness after hot bath. It is due to heat-induced
plaques are disseminated both in time and space. conduction block of partially demyelinated fibres.
Presence of two neurological lesions in anatomically Q: What is Lhermitte sign?
unrelated sites or at different times indicates MS. A: When the neck is flexed, there is tingling or electric
Q: What are sites of involvement in MS? shock-like sensation or funny sensation that passes
A: As follows: down in the upper limb, trunk and perhaps lower
~ • Optic nerve. limbs called Lhermitte sign (also called barber's
o
'0 • Brainstem. chair sign). It indicates that the disease is near the
a~ • Cerebellum. dorsal column nuclei of higher cervical cord (indi-
z • Periventricular region. cates cervical cord compression). Causes are:
• Spinal cord (posterior column and corticospinal • MS (the commonest cause, especially in acute
tract). exacerbation) .
Q: What are the prognostic factors in MS?
• Cervical spondylosis.
A: As follows: • Cervical cord compression (by tumour).
1. Good prognostic factors:
• Subacute combined degeneration.
• Radiation myelopathy.
• Early age of onset.
• Relapsing and remitting form of disease. • Cervical spondylotic myelopathy.
• Visual or sensory symptoms alone at N.B. A similar sensation provoked by neck exten-
presentation. sion may occur called reverse Lhermitte sign.
• Minimum neurological impairment 5 years It strongly suggests cervical spondylosis.
after onset.
Q: What happens during pregnancy in MS?
• More benign course in women than in men.
A: Mild protective effect during pregnancy. Exaggera-
• Little residual disability 5 years after onset.
tion may occur in puerperium.
2. Poor prognostic factors:
• Old age >40 years. Q: What are the causes of MS?
• Frequent relapse in first 2 years. A: Unknown, the following factors may be associated:
• Short interval between first two relapses. • Environmental factors: More in temperate zone,
• Pyramidal, brainstem and cerebellarsymptoms. and rare in tropical country. Greater among rural
• Primary progressive disease. than urban dwellers.
• Poor recovery from relapse. • More frequent in the higher socioeconomic group.
• MRI shows many lesions. • Genetic: Ten times more in first-degree relative.
• Immunological: There is an increase in activated
Q: What are the other demyelinating disorders?
T-lymphocyte in CSF,increasein immunoglobulin
A: As follows:
in CNS and increase in antibody to some virus
• Devic disease (acute necrotising myelitis with
(measles).
optic neuritis).
• Diet: More in those who eat animal fat.
• Tuberous sclerosis (patchy demyelination).
• HLA association - DR2, DR3, B7, A3.
• Leukodystrophies.
• Schilder disease (diffuse cerebral sclerosis). There Q: How to treat MS?
may be cortical blindness if occipital cortex is A: As follows:
involved. Other features are cerebral deafness, 1. During acute attack:
quadriplegia, hemiplegia, dementia, pseudobulbar • Intravenous methylprednisolone: 1 g for
palsy. 3 days or oral 500 mg for 5 days. It shortens
• Acute disseminated encephalomyelitis (ADEM). the duration of relapse but does not affect
the long-term outcome, followed by oral
Q: What are the features of end-stage MS?
prednisolone 40 mg daily for 10 days, then
A: In end-stage disease, the patient is severely disabled 20 mg for 2 days and then 10 mg for 2 days.
with spastic paraplegia, tetraplegia, ataxia, optic
• Or, high-dose prednisolone: 40-60 mg daily
atrophy with blindness, pseudobulbar palsy, urinary
for 10 days, then tapered over for 2 days (It
incontinence, brainstem dysfunction and dementia.
has no role for long-term use for prevention.)
Q: What is Uhthoff phenomenon? • Plasmapheresis is sometimes helpful in
A: Exaggeration of symptoms after hot bath is called patient with severe relapse and unresponsive
Uhthoff phenomenon. The patient feels extreme to corticosteroid.
8 • NEUROLOGY II!IIIIII
2. To prevent relapse (disease-modifying drugs • For fatigue: Amantadine, modafinil or
may be given): amitriptyline.
• Immunosuppressive drug: Azathioprine • For impotence: Sildenafil may be used.
may be helpful. (cyclophosphamide, • Control of infection.
sometimes helpful in aggressive disease, • Prevention of pressure sore.
is not recommended for widespread use.) • Rehabilitation, occupational therapy, walking
Mitoxantrone may be helpful. aids, visual aids, etc.
• Subcutaneous or intramuscular P interferon • Counselling, patient's education.
(1a or Ib) reduces number of relapse (30%).
Q: What is the role of steroid in MS?
• Glatiramer acetate has similar effect. It has
A: In MS with optic neuritis, high-dose intravenous
immunomodulatory effect.
methylprednisolone for 3 days followed by short
• Monoclonal antibody to p-integrins
course of prednisone may be given. It reduces the
(natalizumab) Or to lymphocyte epitopes
rate of relapse of MS over 2 years.
(campath-lH) or alerntuzumab may be
helpful in severely affected patient. Q: What is the role of exercise in MS?
• N immunoglobulin may be helpful in A: The patient should be active during remission and
aggressive cases. avoid excessive physical exercise during relapses.
3. Supportive and symptomatic treatment for
complication and disability: Q: What is the indication of interferon p-therapy?
• For incontinence: Intermittent self-cathete- A: It is used only in patient with relapsing remitting
rization drugs like oxybutynin, tolterodine, type of MS who had at least two relapses in the
etc. previous 2 years and who are able to walk unaided.
• Urgency or frequency: Intermittent self-
N.B. Remember the following points in MS:
catheterization is advised if postmicturition
• Common in women of 20-40 years of age (rare
residual urine is >100 mL. If it is <100 rnl,
before puberty and after 60 years, F:M = 2:1).
oxybutynin or tolterodine may be given.
• It is called disseminated sclerosis, as it is dissemi-
• For spasticity: Physiotherapy and drugs like
nated in time and place.
baclofen (oral or intrathecal), tizanidine,
• Patient may complain of blurring or loss of
benzodiazepine or dantrolene may be used.
vision. Using fundoscopy nothing is seen. ('If
Local intramuscular injection of botulinum
doctor sees nothing, patient sees nothing'. This is
toxin or chemical neuronectomy is other
due to retrobulbar neuritis.)
option. Cannabis extracts and synthetic
• Peripheral nervous system is spared; only CNS
cannabinoids are also used.
involvement.
• For dysaesthesia: Carbamazepine, gabapentin,
• Causes of death are uraemia and
phenytoin or amitriptyline may be helpful.
bronchopneumonia.
• For ataxia: Isoniazid (INH) or clonazepam.

Flaccid Paraplegia (GBS)


[Common cases in flaccid paraplegia are Cuillain-Barre
• Both knee and ankle jerks: Diminished or
syndrome (GBS) and peripheral neuropathy.]
absent (ensure that you have done
Usual instructions are: reinforcement).
• Examine the lower limbs. Or, perform the neuro- • Plantar: Normal or equivocal on
logical examination of the lower limbs. both sides.
• There is no sensory abnormality (may be some
Presentation of a Case sensory abnormality; mention if any).
• Vibration and position senses are normal.
• There is wasting of muscles in both lower limbs
• Coordination: Could not be elicited due to
(mention, up to where) with hypotonia.
weakness.
• Muscle power is diminished, grade 2/5
(mention where). • Gait: Unable to walk due to weakness.
_ SHORT CASES IN CLINICAL MEDICINE

My diagnosis is flaccid paraplegia, which is more likely • Loss of all reflexes.


due to CBS. • Bilateral facial palsy in 50% cases, unilateral
facial palsy in 25% cases.
Q: What are the causes of flaccid paraplegia?
• Sensory loss: Minimum or absent.
A: As follows:
• Sphincter involvement (rare).
• CBS.
• Motor neuropathy due to any cause. NJ? Remembe.r the following points:
• Tabes dorsalis. • Diffuse weakness with loss of all reflexes:
• Friedreich ataxia. A very striking finding in GBS.
• Progressive muscular atrophy (one type of MN D). • CBS may be associated with Hodgkin
• Acute inflammatory demyelinating polyradicu- lymphoma.
lopathy (ATOP). • Papilloedema may develop.
• Hysterical conversion reaction (HCR). • May develop syndrome of inappropriate
antidiuretic hormone (SIADH).
Q: What are the causes of predominant motor
neuropathy?
A: As follows: Q: What are the dangerous complications of GBS
• CBS. (cause of death)?
• Charcot-Marie- Tooth disease. A: As follows:
• Acute intermittent porphyria. • Respiratory muscle paralysis. The patient may
• Chronic lead poisoning. develop respiratory failure within hours.
• Diabetic amyotrophy. • Bulbar palsy (dysphagia, nasal regurgitation).
• Diphtheria. • Cardiac conduction block.
• Paraneoplastic syndrome. • Cardiac arrhythmia.

Q: If it is CBS, what else do you want to examine? Q~ What is GBS?


A: As follows: A: lt is a post-infective demyelinating neuropathy
• In the upper limb: Both may show features of of unknown cause, usually 1-3 weeks after res-
flaccid weakness (all four limbs may be paralysed piratory infection, diarrhoea, and occasionally
at the same time). after vaccination or surgery. There is demyelina-
• Loss of all reflexes (an important due). tion of peripheral nerve or spinal root, which is
• Cranial nerve: Bilateral facial palsy may occur. immunologically mediated. This may follow after
• Fundoscopy: Papilloedema may be present. infection with cytomegalovirus or Mycoplasma or
Q: How does the patient of CBS usually present? Campylobacter jejuni. CBS is monophasic and does
A: As follows: not recur.
• History of upper respiratory tract infection
Q: What is the cause of CBS? What is the mechanism?
(URTI) or gastroenteritis (viral or bacterial).
A: GBS develops 1-3 weeks after respiratory infection
• After 1-3 weeks, there is weakness of lower
or diarrhoea (mainly by Campylobacter) in 70%
limbs that ascends over several weeks (ascending
cases. Triggering factors may be Campylobacterjejuni,
paralysis). It may advance quickly thereby
cytomegalovirus (CMV), Mycoplasma, herpes zoster,
affecting all the limbs at once and can lead to
human irnmounodeficiency virus (HTV), Epstein-
paralysis (quadriplegia).
Barr virus (EBV) infection.
• Respiratory paralysis in 20% case. Progressive
respiratory involvement and paralysis is the main There are two mechanisms:
problem. 1. Demyelinating [acute inflammatory demyeli-
• Paraesthesia and pain in back and limbs may occur. nating neuropathy (AlDP)].
• Facial and bulbar weakness. 2. Axonal, which may be:
• Autonomic dysfunction: Change of blood pres- • Motor [acute motor axonal neuropathy
sure, tachycardia, increased sweating; dysrhythmia (AMAN)].
may occur. • Sensorimotor [acute motor and sensory
axonal neuropathy (AMSAN)],
Brief clinical findings in GBS:.
• Flaccid paralysis involving lower limbs and may Q: What investigations do you suggest in GBS?
involve all four limbs. A: As follows:
• CSF analysis: Typical finding is 'albuminocyto- Indication of ventilation:
logical dissociation' (albumin may be very high, • Impending respiratory failure: Tachypnoea,
> 1000 mg%, lymphocytes are slightly raised or decrease in arterial 02 tension <85 mmHg,
normal, <20/mm3• If lymphocyte is >50, CBS forced vita] capacity (PVC) <20 mL/k& maximum
is unlikely. CSF protein may be normal in first inspiratory pressure <30 cm H20, maximum
10 days). expiratory pressure <40 cm H20.
• Frequent monitoring of respiratory function tests • PVC <1.5 L.
[forced vital capacity (PVC), forced expiratory • Pa02 <10 kPa.
volume in 1 s (FEV1), peak expiratory flow rate • PaC02 >6 kPa.
(PEFR)J. • Rapid progression of disease.
• Arterial blood gas analysis (as respiratory failure • Bulbar dysfunction.
may occur at any time).
• Bilateral facial palsy.
• Nerve conduction study. (It shows slow
• Autonomic involvement.
conduction or conduction block. Demyelinating
neuropathy, usually found after 1 week.)
Q: What is the prognosis of CBS?
• Investigation to identify CMV, Mycoplasma or A: As follows:
Campylobactet should be done.
• 80% recovery, may take several months (3-6
• Serum electrolyte. months). If axons have been damaged, the
Note: Triad of acute symmetrical ascending paralysis regeneration may require 6-18 months or
of limbs, areflexia and albuminocytological dissocia- longer.
tion in CSF is highly suggestive of CBS. • 10% residual disability.
• 3-5% die (in some study up to 10% deaths).
Q: What is Miller-Fisher syndrome?
A: It is a variant of CBS characterized by triad of Q: What are the bad and good prognostic factors?
ophthalmoplegia, ataxia and areflexia. It is a rare A: As follows:
disease.
Adverse prognostic factors:

Q: How. to treat CBS? • Preceding CI infection.


A: As follows: • >60 years of age.
• Ideally the patient should be treated in ICU • The most severe or rapidly evolving form of the
and respiratory function should be monitored disease (maximum disability within 7 days).
regularly (vital capacity and arterial blood gases). • Descending paralysis.
The patient may require artificial ventilation. • Bulbar weakness.
• High-dose intravenous y-globulin should be • Autonomic dysfunction.
given to all patients (it reduces the duration • Asymmetrical weakness.
and severity). Dose is 400 rngjkg/day for 5 days. • Very high CSF protein.
It is helpful, if given within 14 days. It may • Evidence of widespread axonal damage.
precipitate angina or myocardial infarction. In • Those requiring early and prolonged mechanical
congenital IgA deficiency, it may cause allergic ventilatory assistance.
reaction.
Good prognostic factors:
• Plasma exchange if given within 14 days is equally
• Symmetrical involvement.
effective in reducing the severity and duration of
CBS. • Slowly progressive.
• Steroid has no proven value (may worsen). Meth- • Facial nerve involvement.
ylprednisolone with immunoglobulin has no • Young age.
proven benefit. • Preceding H/O respiratory infection.
• Plasmapheresis may be required. • Demyelinating disease.
• Physiotherapy is the mainstay of therapy. • No respiratory involvement.
• Prevention of pressure sore and venous • Ascending paralysis.
thrombosis. • Early treatment.
• Other symptomatic treatment. • Early recovery.
__ SHORT CASES IN CLINICAL MEDICINE

Polyneuropathy
Usual instructions are: Q: What are the causes of predominantly sensory
• Perform neurological examination of lower limbs neuropathy?
or upper limbs. A: As follows:
• Diabetes mellitus.
Presentation of • Leprosy.
Case No.1 • Deficiency of vitamins B" Bo and B12.
• Chronic renal failure.
• Present the case described in flaccid paraplegia • Paraneoplastic neuropathy (in bronchial
page 325, plus: carcinoma).
• There is bilateral symmetrical sensory loss in • Drugs (INH, vincristine).
stocking pattern with light touch and pin prick • Hereditary sensory neuropathy.
(mention up to where), and also loss of vibration • HIV.
and position senses. • Multiple myeloma.
• Gait is ataxic with wide based and high steppage.
• Rombergism is positive. Presentation of
Case No.3
My diagnosis is peripheral neuropathy (mixed motor
• There is wasting of all the muscles oflower limbs.
and sensory type).
• Muscle power is diminished, grades 3/5 and 4/5.
Q: What do you think is the cause in this case? • Knee and ankle jerks are diminished (or absent).
A: As follows: • Plantar response: Equivocal.
• Diabetes mellitus. • Sensory is normal.
• Nutritional deficiency (BI, BG, BI2, folic acid,
pantothenic acid and vitamin E).
• Malignancy, e.g. bronchial carcinoma (in elderly My diagnosis is peripheral neuropathy (predomi-
patients). nantly motor type).
~ Drugs (isoniazid, vincristine, phenytoin, Q: What are the causes of predominantly motor
amiodarone, statins, cisplatin and dapsone). neuropathy?
• Alcoholism. A: As follows:
• Infections (leprosy). • Cuillain-Barre syndrome, chronic inflammatory
• Idiopathic. demyelinating polyneuropathy (CIDP)
Q: Could it be MND? • Charcot-Marie-Tooth disease.
A: No. In MND, there is no sensory loss (fasciculation • Acute intermittent porphyria.
is present). • Chronic lead poisoning.
• Diabetic amyotrophy.
• Diphtheria.
Presentation of I • Paraneoplastic syndrome.
Case No.2 • POEMS (Peripheral neuropathy, Organomegaly,
Endocrinopathy, M-protein, Skin changes).
• Muscle power and tone are normal; reflexes are
absent.
Presentation of :1-------,
• There is bilateral symmetrical sensory loss in
stocking pattern with light touch and pin prick
Case No.4: Upper Limb
(mention up to where); and also loss of vibration • There is wasting of the small muscles of hands,
and position senses. extending up to the middle part of both forearms.
• Gait is ataxic with wide based and high steppage. • All the muscles are weak.
• Rombergism is positive. • There is bilateral symmetrical sensory loss in
gloves pattern with light touch and pin prick
My diagnosis is peripheral neuropathy (predominant (mention up to where).
sensory type).
8 • NEUROLOGY __

My diagnosis is peripheral neuropathy (mixed type). Treatment: Tricyclic antidepressant, phenytoin,


carbamazepine and topical capsaicin.
Q: What else do you want to examine in this case?
A: I want to examine the lower limbs to see any Q: What are the mechanisms of neuropathy?
evidence of neuropathy. A: As follows:
• Demyelination.
Read the following topics in relation to polyneuropathy. • Axonal degeneration.
• Wallerian degeneration (after section of nerve,
Q: What are the causes of peripheral neuropathy?
axonal and myelin sheath degeneration).
A: (Mention the causes according to the age of the
• Compression (called entrapment neuropathy);
patient):
and there is segmental degeneration at the site of
• Diabetes mellitus.
compression.
• Nutritional deficiency (B1, B6, Bn, folic acid,
• Infarction of nerve: microinfarction of nerve due
pantothenic acid and vitamin E).
to arteritis of vessels supplying nerve (diabetes
• Malignancy (bronchial carcinoma, lymphoma
mellitus and polyarteritis nodosa).
and multiple myeloma).
• Infiltration in nerve (leprosy, sarcoidosis and
• Drugs [isoniazid (IN H), vincristine, phenytoin,
malignancy).
amiodarone, statins, cisplatin and dapsone].
• Alcoholism. Q: What are the causes of demyelination and axonal
• Cuillain-Barre syndrome. degeneration?
• Infections (leprosy, HIV,typhoid and diphtheria). A: As follows:
• Collagen disease [systemic lupus erythematosus 1. Causes of demyelination:
(SLE), polyarteritis nodosa (PAN) and • Cuillain-Barre syndrome.
rheumatoid arthritis (RA)). • CJDP.
• Others: Chronic renal failure (CRF), chronic • Hereditary sensory motor neuropathy.
inflammatory demyelinating polyneuropathy • Diphtheria.
(ClOP) and idiopathic (in many cases). • Diabetes mellitus.
• Refsum disease.
Q: What are the investigations done in • HIV.
polyneuropathy! 2. Causes of axonal degeneration:
A: As follows: • Toxic neuropathy (alcohol and drugs).
• Full blood count (FBC) and peripheral blood film • Diabetes mellitus.
(PBF) examination [evidence of megaloblastic • Para neoplastic.
anaemia in subacute combined degeneration • IgG paraproteinaernia.
(SCD)- macrocytosis]. • Hereditary.
• Blood sugar. • Vitamin deficiency.
• Chest X-ray (bronchial carcinoma).
Q: How to differentiate between demyelination and
• Serum BI2and folate assay.
axonal degeneration?
• Renal and hepatic function.
A: By NCS and electromyography (EMC):
• Bone marrow (megaloblast in SCD).
• In demyelination, there is slowing of nerve
• Other investigations according to suspicion of
conduction, amplitude of nerve action potential
causes [antinuclear antibodies (ANAs) and RA
(CMAP) is normal.
test).
• In axonal degeneration, the conduction velocity
• Nerve conduction study [(NCS) (axonal or
is normal as axonal continuity is maintained in
demyelinating)] .
surviving fibres, amplitude of CMAP is reduced.
Q: What are the causes of painful neuropathy? Using needle EMC, denervation on the affected
A: As follows: muscles may be demonstrated.
• Diabetes mellitus. Q: Which part is first involved in peripheral
• Deficiency of vitamins B, and Bu' neuropathy? Why?
• Alcohol. A: Usually distal part of the limbs is commonly involved,
• Carcinomatous neuropathy. because longer the nerve fibre earlier is the involve-
• Porphyria. ment. Since the nerve fibres supplying the distal parts
• Arsenic or thallium poisoning. of the limbs are longer, they are first affected.
_ SHORT CASES IN CLINICAL MEDICINE

Q: What is mononeuritis multiplex and what are the Q: What are the types of neuropathy in DM?
causes of it? A: As follows:
A: Separate involvement of more than one peripheral • Commonly sensory neuropathy.
nerve or cranial nerve by a single disease is called • Mixed motor and sensory neuropathy.
mononeuritis multiplex. It is due to involvement of • Asymmetrical motor neuropathy (diabetic
vasa nervorum or malignant infiltration of nerves. amyotrophy) .
Causes are: • Autonomic neuropathy.
• Diabetes mellitus. • Mononeuropathy.
• Leprosy. • Mononeuritis multiplex.
• Rheumatoid arthritis. Mechanism of neuropathy in DM:
• Vasculitis (SLE,polyarteritis nodosa). • Axonal degeneration.
• Amyloidosis. • Patchy or segmental demyelination.
• Malignancy (carcinomatous neuropathy). Involvement of intraneural capillaries.
• Sarcoidosis. Q: What are the causes of thickening of nerves?
• HlV infection. A: As follows:
• Wegener granulomatosis. • Leprosy.
• Acromegaly. • Neurofibroma.
• Paraproteinaemia. • Amyloidosis.
• Lyme disease. • Acromegaly.
• Idiopathic multifocal motor neuropathy. • Sarcoidosis.
• Charcot-Marie-Tooth disease (hereditary motor
Causes of acute mononeuritis multiplex (usually and sensory neuropathy).
vascular): • Repeated friction or trauma.
• Polyarteritis nodosa. • Refsurn disease.
• Diabetes mellitus. • Dejerine-Sottas disease (hypertrophic peripheral
• Collagen disease (SLEand RA). neuropathy).

Polyneuropathy [Subacute Combined Degeneration-(SCD)]


Usual instructions are: Q: What else do you want to see, if it is SCD?
• Perform neurological examination of lower limbs. A: As follows:
• Anaemia (may be lemon yellow pallor in
Presentation of a Case: pernicious anaemia).
Lower Limb • Glossitis (smooth tongue).
• Examination of abdomen may show: Mass of
Present the case as described in "peripheral carcinoma stomach, or scar mark in the abdomen
neuropathy" (page 328). However, knee jerk is (if gastrectomy).
brisk (ankle jerk is lost or absent) and planter is • Eye shows optic atrophy.
extensor. • Evidence of dementia.
N.B. Anaemia is usually present, but sometimes
My diagnosis is peripheral neuropathy. vitamin B12 neuropathy may not be associated
Q: What do you think is the cause in this case and with anaemia and there may be normal blood
why? picture with normal marrow. Serum BI2 should
A: T think this is a case of SCD because there are: be measured.
• Peripheral neuropathy.
• Signs of posterior column lesion (loss of vibration Q: What history do you like to take in SCD?
and position sense). A: As follows:
• Signs of pyramidal lesion (plantar is extensor, • Family history of pernicious anaemia.
knee jerk is brisk and ankle jerk is absent). • History of gastrectomy, resection of ileum.
• Rombergism is positive. • Frequent diarrhoea (Crohn disease).
8 • NEUROLOGY __

• Patient's dietary habit (if the patient is • Age: 40-60 years, equal sex involvement.
vegetarian) . • Sensory symptoms: Paraesthesia, tingling or
• History of chronic pancreatitis. numbness, starting in toes and fingers. Lower
• History of infection with Diphyllobothrium latum. limbs are more commonly affected than the
upper limbs.
Q: Why it is called combined degeneration? (Or, what
• Motor symptoms: Weakness, ataxia and loss of all
is the lesion in SCD?) reflexes. May be exaggerated knee reflex with loss
A: Because there are: of ankle jerk but extensor plantar.
• Posterior column lesion (degeneration of the • Bladder involvement: Urinary incontinence and
ascending tract of the posterior column). dribbling (usually in late stage).
• Pyramidal lesion (degeneration of the descending • Eye: Optic atrophy.
pyramidal tracts in the lateral column). • Mental change: Dementia, impaired memory,
• Demyelination of peripheral nerve (peripheral confusion and depression.
neuropathy).
Q: What investigations are done in SCD?
Q: What are the presentations ofSCD? A: As follows:
A: The patient usually complains of tingling or numb- 1. PBC and PBP examination (macrocytosis and
ness, or buming sensation and weakness in the hypersegmented neutrophil).
Legs. 2. Bone marrow (to see megaloblast).
Q: Why polyneuropathy in SCD? 3. Serum BI2 assay.
A: It is due to demyelination.
4. Other investigations according to the suspicion
of cause:
Q: What is the type of anaemia in SCD? • For Addisonian pernicious anaemia:
A: Macrocytic (megaloblastic) anaemia due to vitamin Antiparietal cell and anti-intrinsic factor
BI2 deficiency. antibody, endoscopy and biopsy to see gastric
atrophy, Schilling test.
Q: What happens if blood transfusion is given in vita-
• Investigation for Crohn disease.
min BI2 neuropathy with severe anaemia?
A: Blood transfusion or packed cell should be avoided N.B. Macrocytosis in blood and megaloblastic mar-
without correcting vitamin B12;otherwise neurolog- row are invariable in SCD.
ical manifestation may be aggravated. Q: How to treat SeD?
A: As follows;
Q: What is the daily requirement of Vitamin B12?
• Injection of vitamin B12:1000 ugm intramuscular
How long does it take to develop deficiency of
(1M), 5 doses, 2-3-days apart, then every
vitamin B12?
3 months for lifelong.
A: 1-2 mg daily. To develop deficiency, it takes 3 years.
• Following therapy, iron deficiency may occur in
Q: What are the causes of vitamin B12deficiency? the first few weeks. So, oral iron therapy should
A: As follows: be given.
• Addisonian pernicious anaemia. • Also, following therapy there may be
• Total gastrectomy (B12 injection should be given hypokalaernia, which may need correction.
every 3 months) or partial gastrectomy (10-20% • Bl2 orally 2 mg/day may be given. 1-2% is
develop Bl2 deficiency within 5 years; BI2injection absorbed by diffusion without intrinsic factor.
should be given every year). Sublingual BI2 may be effective.
• Ileal disease (Crohn disease, ileal resection). • Treatment of primary cause.
• Stagnant loop syndrome.
Q: Will you transfuse blood in this patient?
• Pancreatic insufficiency (failure to transfer
A: Blood transfusion is avoided as it may precipitate
vitamin BI2 from R-protein to intrinsic factor).
heart failure. Also, before replacing B12, if blood
Vegetarian diet.
transfusion or packed cell is given, it may aggravate
• Chronic tropical sprue. neurological manifestations. However, blood (or
• Fish tapeworm (D. latum).
packed cell) transfusion may be considered if there is:
• Congenital intrinsic factor deficiency.
• Angina.
Q: What are the clinical features of SCD? • Heart failure.
A: As follows: • Cerebral hypoxia (confusion, dizziness, etc.).
_ SHORT CASES IN CLINICAL MEDICINE

Q: How to see the response?


N.B. Neurological features of vitamin BI2 deficiency
A: Clinical improvement may occur within 48 h; reticulo- should be excluded in any patient with any of
cytosis may be seen after 2-3 days of starting therapy. the following unexplained diseases:
Haemoglobin level rises by] g/dl, every week.
• Peripheral sensory neuropathy.
Q: What is the response of neurological lesion to vita- • Spinal cord disease (posterior column lesion
min 812 therapy? and corticospinal tract lesion).
A: Response is variable. It may improve, remain • Optic atrophy (rare).
unchanged or may even deteriorate. Sensory abnor- • Dementia (rare).
malities improve more than the motor and peripheral • Autonomic neuropathy.
neuropathy responses, better than myelopathy. Sen-
sory neuropathy takes 6-12 months for recovery;
Q: What are the features of tabes dorsalis?
longstanding polyneuropathy may not be improved.
A: As follows:
Q: What are the causes of posterior or dorsal column • Signs of dorsal column lesion (loss of vibration
lesion? and position sense). Squeezing of tlle calf muscles
A: As follows: and the Achilles tendon produces no pain (deep
• Subacute combined degeneration. sensation is lost).
• Tabes dorsalis. • Bladder involvement: Common.
• Friedreich ataxia. • Eye signs: Argyll Robertson pupil and bilateral
• MS. ptosis.
• Brown-Sequard syndrome (ipsilateral leg). • Rombergism: Positive.

Motor Neuron Disease (MND)


Usual instructions are:
Remember, for the diagnosis of MND, if any patient
• Neurological examination of lower limbs or upper presents with:
limbs.
• Wasting of muscles (weak also).
• Look at the legs. What is your finding? (Fascicula- • But exaggerated reflex.
tion.) What else do you want to examine? • No sensory loss and (but fasciculation is
• Examine the hands (wasting). present).
• Diagnosis should be motor neuron disease, until
proved otherwise.
Presentation of }I---- __ -,
Case No.1: Lower Limb N.B. During examination, if the patient looks emo-
• There is wasting of all the muscles of lower limbs tionally upset, and there is dribbling of saliva, it
is suggestive of pseudobulbar palsy.
and fasciculation (mention the location).
• Muscle power is diminished (mention the Q: What else do you like to examine?
location), grades 3/5 and 4/5. A: As follows:
• Knee and ankle jerks are exaggerated, also 1. Upper limbs: To see signs of LMN lesion in the
patellar or ankle clonus (mention, if any). upper limb (then the diagnosis is amyotrophic
• Plantar response: Extensor on both sides. lateral sclerosis).
• Sensory is normal. 2. Tongue:
• Coordination: Difficult to elicit because of • Wasting and fasciculation (bulbar palsy).
weakness.
Then talk with the patient (nasal voice) and
• Gait: The patient is unable to walk (may be ask about nasal regurgitation.
spastic gait). • Spastic (pseudobulbar palsy).
3. Jaw jerk (exaggerated in pseudobulbar palsy).
My diagnosis is motor neuron disease. 4. Emotionally upset (pseudobulbar palsy).
8 • NEUROLOGY __

Presentation of
Case No.2: Examination of Hands
(Look at the hands. What are your findings?)
• There is generalized wasting of the small muscles
of hands involving the thenar, hypothenar and
interossei muscles with dorsal guttering and claw
zg
....
hand [flexion of interphalangeal (IP) joints and o
0-
extension of metacarpophalangeal (MCP) joints]. ~

Q: Mention one physical examination you would like


to do. Or, what else do you like to do?
Wasting In MND
A: I want to see the sensory function. If there is no loss

- of sensation, my diagnosis is MND. If there is loss of


sensation, J have some differential diagnoses:
• Peripheral neuropathy due to any cause.
• Cervical spondylosis.
• Cervical rib.
• Cervical cord compression (neurofibroma and
men ingioma).
• Leprosy.
• Syringomyelia (dissociated sensory loss and
trophic changes in hand).

Q: What are the causes of wasting of the small


muscles of the hand?
MND (wasting of thenar and hypothenar
A: As follows:
muscles-claw hand) • MND.
• Syringomyelia.
Q: What are the differential diagnoses? • Charcot-Marie-Tooth disease.
A: As follows: • Cervical spondylosis.
1. Diabetes mellitus. • Cervical rib.
2. Syphilitic amyotrophy or syphilitic cervical • Pancoast tumour.
pachymeningitis. • Following arthritis.
3. Paraneoplastic syndrome (commonly due to • Peripheral nerve lesion (peripheral neuropathy
bronchial carcinoma). or combined ulnar and medial nerve lesion).
4. Sarcoidosis. • Myopathy (dystrophia myotonica).
5. Spinal diseases:
Q: What investigations should be done in this case?
• Cervical cord compression.
A: No specific test, diagnosis is usually clinical. Investi-
• CASMA(chronic asymmetrical spinal
gations are done to exclude other diseases:
muscular atrophy).
• Blood sugar (to exclude diabetic amyotrophy).
• Old poliomyelitis (confuses with progressive
• Venereal Disease Research Laboratory (VDRL)
muscular atrophy).
test or Treponema pallidum haemagglutination
• Spinal muscular atrophy of juvenile-onset
(TPHA) to exclude neurosyphilis.
type-3 (onset in childhood, mild form, affects
• Chest X-ray (to exclude bronchial carcinoma).
mostly proximal muscles).
• X-ray of the cervical spine.
• Motor neuropathy due to any cause (confuses
• Ultrasonogram of whole abdomen (to see any
with progressive muscular atrophy).
neoplasm).
Q: Could it be syringomyelia? • EMC (to confirm fasciculation and denervation).
A: Unlikely, as in syringomyelia there is dissociated • Nerve conduction velocity (NCV) (normal motor
sensory loss. and sensory conduction).
_ SHORT CASES IN CLINICAL MEDICINE

• Lumbar puncture and CSFstudy (no abnormality, 2. Cerebral lesion:


only slightly raised protein). • Progressive bulbar palsy: Medullary lesion.
• cr or MRI (brain and spinal cord). • Pseudobulbar palsy: Cortical lesion.
N.~: There are certain 'No's in MND: According to type of lesion:
• No sphincter disturbance (rarely involved in 1. Pure UMN lesion: PLS, pseudobulbar palsy.
late case). 2. Pure LMN lesion: PMA, bulbar palsy.
• No sensory involvement. 3. Mixed lesion: ALS.
• No loss of awareness till death.
• No dementia. Q: How to treat MND?
• No ocular involvement. A: As follows:
• No cerebellar or extrapyramidal lesion. 1. No curative treatment.
• No abnormality of CSF usually. 2. Supportive treatment:
• Physical rehabilitation.
• Psychological support.
Read the Following Topics in Relation to • Occupational rehabilitation.
MND • Nutritional care: Change the form and texture
of the food, high calorie food supplement,
Q: What is MND? What are the causes of MND?
enteral feeding, PEG (percutaneous endo-
A: It is a progressive disease of unknown cause, char- scopic gastrostomy).
acterized by the degeneration of motor neurons in • Speech and communication therapy.
the spinal cord, cranial nerve nuclei and pyramidal • Respiratory therapy: NIPPV.
• Home and hospice care, end-of-life palliative
neurons in the motor cortex.
MND is common in middle-aged and elderly, care.
rare before 30 years of age. Males are commonly 3. Symptomatic treatment:
affected than females (pseudobulbar palsy is more • Fatigue: Pyridostigrnine, amantadine.
in females). There is no remission and the disease • Depression: SSRI,venlafaxine, amantadine.
is fatal within 3-5 years. Young patients and • Emotional lability: Same.
thosewith bulbar symptoms show rapid progression. • Cramps: Quinine sulphate, vitamin E,
clonazepam.
·Causes are unknown; possible factors are:
• Fasciculation: Carbamazepine
• Familial: 5-10% of the cases may be inherited as
• Spasticity: Baclofen, tizanidine.
autosomal dominant.
• Sialorrhoea: Hyoscamine sulfate,
• May follow viral infection, trauma, exposure to
scopolamine patch.
toxin and electric shock.
• Joint pain: Analgesics, NSAlDs
• Glutamate toxicity has been implicated as a factor
• Insomnia: Zolpidem tartrate.
in amyotrophic lateral sclerosis (ALS).
• Respiratory failure: Bronchodilators.
4. Neuroprotective agents: Riluzole, TGF-I, ASO,
Q: What is the pathology of MND?
vitamin E, Coenzyme Q-lO, neuroprotective
A: Degeneration of Betz cells, pyramidal tract, cranial
factors. Riluzole is a glutamate antagonist that
nerve nuclei and anterior hom cells. Both UMN
may retard progression and prolong the survival.
and LMN may be involved, but there is no sensory
involvement. Q: What is the prognosis of MND?
A: MND is a progressive disorder and its remission
Q: What are the types of MND? is unknown. It is usually fatal within 3-5 years.
A: According to the site of lesion: Younger patient with early bulbar syndrome tend
l. Spinal cord lesion:
to show a more rapid course. Prognosis is relatively
• Progressive muscular atrophy (PMA): LMN better in progressive lateral sclerosis and progressive
lesion. muscular atrophy.
• Amyotrophic lateral sclerosis (ALS): Com-
bined UMN and LMN (LMN lesion in upper Q: What is the cause of death?
limbs and UMN lesion in lower limbs). A: Bronchopneumonia, respiratory failure resulting
• Primary lateral sclerosis (PLS): Pure UMN from diaphragmatic paralysis and complication of
lesion (rare). immobility.
8 • NEUROLOGY __

Features and diagnosis of individual MND • Gag reflex is present.


• The patient is emotionally labile (crying and
Progressive muscular atrophy: laughing).
• Weakness, wasting and fasciculation of distal limb • Siteoflesion: Bilateral UMN lesion (supranuclear)
muscles, usually starting in small muscles of one involving the pyramidal tract (supranuclear lesion
or both hands. of lower cranial nerves: IX,X, Xl, XII).
• Tendon reflex is lost (due to involvement of • Causes:
zg
...,
anterior horn cell). o Bilateral repeated cerebrovascular accident o
0-
Amyotrophic lateral sclerosis:
(CVA) involving internal capsule (multi-infarct I ~
dementia).
• Weakness, wasting, fasciculation and loss of all o Demyelinating disease (MS).
reflexes (LMN lesion) in upper limb plus spastic o Motor neuron disease.
weakness with exaggerated reflexes and extensor
plantar response in lower limb (UMN lesion), or
commonly there is generalized hyperreflexia.
• Bulbar and pseudobulbar palsy may follow Q: What are the neurological nonmetastatic
eventually. syndromes of malignancy? (Also called
paraneoplastic syndrome.)
Primary lateral sclerosis: A: As follows:
• Only UMN lesion (upper limb and lower limb). • Motor neuron disease.
• Progressive tetraparesis with terminal pseudobul- • Sensory neuropathy.
bar palsy may occur. • Mononeuritis multiplex.
• Cranial polyneuropathy.
Progressive bulbar palsy: • Eaton-Lambert syndrome.
• Presents with 3 'Ds' -dysarthria, dysphonia and • Spastic paraparesis.
dysphagia. There is nasal regurgitation, dribbling • Cerebellar syndrome.
of saliva. • Dementia and encephalopathy.
• Speech is nasal, indistinct and slurred. • Progressive multifocalleucoencephalopathy.
• Tongue: Wasted, wrinkled and fasciculating.
Cause of neurological nonrnetastatic syndromes is
• There is palatal palsy.
unknown. It may precede the clinical manifestation
• Gag reflex is absent.
of malignancy. Usually, it is associated with small-cell
• Site of lesion: Nucleus of lower cranial nerves in
carcinoma of the lung and lymphoma.
medulla (IX, X, Xl and XII). Lesion is bilateral and
LMN type. Q: What is the difference between fasciculation and
• Common causes: fibrillation of muscle?
o Motor neuron disease. A: As follows:
o Cuillain-Barre syndrome. • Fasciculation: Contraction of groups of muscles.
o Syringobulbia. It is visible.
o Brainstern infarction. • Fibrillation: Contraction of single muscle fibre or
o Poliomyelitis. unit. It is not visible. Diagnosed by EMG.
o Neurosyphilis.
Q: What is fasciculation? What are the causes? What is
o Neurosarcoid. the mechanism?
Pseudobulbar palsy: A: Random spontaneous twitching of a group of muscle
fibres or a motor unit that produces movement of the
• This is more common in women.
overlying skin or mucous membrane or digits. Fascicu-
• Speech: Nasal, slurred, indistinct and high pitched
lation may be coarse or fine, usually present at rest, but
(so called Donald Duck or hot potato dysarthria
not during voluntary movement. It is usually sponta-
due to tight immobile tongue). .
neous and may be elicited by tapping with finger or
• Tongue: Small and tight, spastic, unable to
hammer over the muscle (this procedure is controver-
protrude, but no wasting or fasciculation.
sial, as it is not accepted by some neurologists because
• Jaw jerk is exaggerated.
it should be spontaneous). If fasciculation is present
• Palatal movement is absent.
with weakness and wasting, it indicates LMN lesion.
_ SHORT CASES IN CLINICAL MEDICINE

Mechanism of fasciculation: It is due to spontane- • Syphilitic amyotrophy.


ous firing of surviving axons that strive to innervate • Neuralgic amyotrophy.
the muscle fibres that have lost their nerve supply.
2. Metabolic:
Causes of fasciculation: • Tetany.
1. Neurogenic: • Thyrotoxic myopathy.
• MND. • Anticholinergic drugs.
• Charcot-Marie- Tooth disease.
• OPC poisoning.
• Spinal muscular atrophy.
• Radiculopathy (cervical spondylosis, cervical rib). 3. Normal:
• Syringomyelia. • Benign fasciculations (in anxiety or tension. It is
• Peripheral neuropathy. usually found around the shoulder joint).
• Creutzfeldt-Iakob disease (CJO) • After exercise in fit adults.
• Acute stage of poliomyelitis (rarely in old • After tensilon test
polio). • Muscle cramps.

Friedreich Ataxia
Usual instructions are: • Heart shows cardiomyopathy (hypertrophic
cardiomyopathy, may cause sudden death) and
• Examine the lower limbs. Examine for cerebellar
ECG change in 50% cases.
signs in the lower limbs.
• Sensory neural deafness (in 10% cases).
• Look at the legs. What are your findings? What else
do you want to examine? Q: What are the causes of high-arched palate?
A: As follows:
• Friedreich ataxia.
Presentation of a Case: '1----------., • Marfan syndrome.
lower Limbs (the Patient is • Homocystinuria.
Usually Young) • Turner syndrome.
• Tuberous sclerosis.
• The patient has bilateral pes cavus and cocking
Q: Why absent tendon reflex, but extensor plantar
of toes.
response?
• There is wasting of muscles of leg.
A: Because of combination of pyramidal lesion, dor-
• Muscle tone is diminished in both lower limbs.
sal column and dorsal root lesion. Also, there is
• Muscle power is diminished in both lower limbs.
involvement of the peripheral sensory fibres that
• There is loss of knee and ankle jerks.
leads to sensory disturbance in the limbs and
• Plantar extensor on both sides.
depressed tendon reflex.
• Sensory test is normal, but loss of vibration and
position senses. Q: What are the causes of pes cavus?
• Coordination is impaired on both lower limbs. A: As follows:
• Ataxic gait (cerebellar). • Congenital.
• Rornbergisrn is positive. • Friedreich ataxia.
• Charcot-Marie-Tooth disease.
• Hereditary motor and sensory neuropathy.
My diagnosis is Priedreich ataxia. • Spinocerebellar degeneration.
• Peripheral neuropathy in childhood.
Q: What else do you want to see? Old poliomyelitis (usually unilateral).
A: As follows:
• Palate (high arched). Q: What are the differential diagnoses of Friedreich
• Spine (kyphoscoliosis). ataxia?
• Signs of cerebellar lesion (see page 338). A: As follows:
• Eye (nystagmus in 25%, fundoscopy shows optic • MS.
atrophy in 30%, retinal atrophy and retinitis • Tabes dorsalis.
pigmentosa). • Spinocerebellar degeneration.
8 • NEUROLOGY __

Q: What is Friedreich ataxia? What is the cause? What Q: What are the sites of lesion in Friedreich ataxia?
are the features? A: As follows (there is progressive degeneration):
A: It is the most common type of hereditary ataxia • Cerebellar lesion.
inherited as autosomal recessive trait; and in some • Spinocerebellar tract.
cases it is inherited as autosomal dominant. • Posterior column lesion (loss of vibration and
Cause: Unknown. Mutation of FRDA gene in chro- position sense) and dorsal root ganglia lesion.
mosome 9. The mutation is abnormal expansion • Degeneration of peripheral sensory fibres. zg
of trinucleotide repeat within a gene that codes • Corticospinal tract lesion (lateral column lesion). '"
o
• Eye (primary optic atrophy). 0'
for protein 'Frataxin'. whose function is to prevent
~
intramitochodrial iron overloading.
Q: What are the different types of hereditary ataxias?
Features of Priedreich ataxia are:
A: It may be of different types, with different patterns
• Family history: May be present.
of inheritance:
• Usual onset: Young, <15 years (8-16 years).
• Presentations: Progressive difficulty in walking 1. Aut.osomal recessive:
(truncal ataxia and ataxia of lower limbs), • Friedreich ataxia.
weakness of lower limbs and dysarthria. • Ataxia telangiectasia.
• Ataxia with vitamin E deficiency
• Signs are:
o Cerebellar signs (dysarthria, nystagmus, 2. Autosomal dominant:
intention tremor, ataxic gait, etc.). • Spinocerebellar ataxia type 1-28
o Posterior column: Absent vibration and • Episodic ataxia
position sense, positive Rhornbergism. • DRPLA (Dentatorubropallidoluysian
o Corticospinal tract sign: Plantar extensor, atrophy).
weakness. 3. X-linked:
o Peripheral nerve: Absent reflexes in lower • Fragile X-associated tremor/ataxia syndrome
limb, wasting of muscles. (FXTAS)
4. Mitochondrial:
• Diabetes mellitus (common).
• Associated with kyphoscoliosis, pes cavus, • Mitochondrial encephalomyopathy, lactic
cocking of toes, optic atrophy, spina bifida acidosis, and stroke-like episodes (MELAS)
and hypertrophic cardiomyopathy (may cause • Myoclonic epilepsy with ragged-red fibres
sudden death). Hearing loss. (MERRF)
• Normal mentation (may have mild dementia). • Kearns-Sayre syndrome (KSS)
• Prognosis: Usually progresses slowly, death
Q: What are the causes of combined cerebellar, pyram-
occurs before 40 years of age (usually 20 years
idal and dorsal column signs?
after the onset of symptoms due to cardiac and
A: As follows:
respiratory complications).
• Become chair: Bound 9-15 years after onset of • MS.
symptoms. May be static and survive up to 60 • Friedreich ataxia.
years. • Spinocerebellar degeneration.
• Syphilitic meningomyelitis.
N.B. In young patients with pes cavus plus
• Arnold-Chiari malformation.
combination of cerebellar lesion (bilateral),
UMN lesion (extensor plantar) and posterior
Q: What investigations should be done in Freidreich
column lesion (loss of vibration and position
ataxia?
senses) is highly suggestive of Fried reich ataxia.
A: As follows:
• CBC, ESR.
• Blood sugar (high in 10%).
• Chest X-ray (cardiomegaly).
• ECG (arrhythmia).
• MRI of brain and spinal cord (shows atrophy of
cerebellum and spinal cord).
• NCS (shows that conduction velocity in motor
fibres is normal or mildly reduced, but sensory
Pescavus action potentials are small or absent).
_ SHORT CASES IN CLINICAL MEDICINE

Q: How to manage this patient? • Ankle or foot orthoses.


A: There is no specific therapy. However, the following • Walking aids.
measures should be taken: • Occupational therapy.
• Visual aids.
• Physiotherapy and exercise. • Hearing aids.
• Orthopaedic surgery to correct scoliosis, pes • Treatment of related conditions like diabetes
60 cavus and other deformities. mellitus, hypertrophic cardiomyopathy, etc.
o
o
3
Z Cerebellar Lesion
Usual instructions are:
Presentation of a Case I

• Examine for cerebellar signs.


• Talk with the patient. What isyour finding? (Scanning • There is titubation (mention, if any), and tilting
speech). What else do you want to examine? (J want of shoulder towards the right or left side.
to examine to see the cerebellar signs.) • Speech is scanning or staccato (jerky, slurred and
explosive; the patient talks syllable-by-syllable).
Proceed as follows:
• There is horizontal and coarse nystagmus (right
• Look carefully: Head nodding (called titubation > left).
either to-and-fro movement-like 'yes-yes' or • Finger-nose test is positive (right, left or both)
rotatory movement like 'no-no'). and intention tremor is present, which is worse
• Shoulder is tilted towards the site of lesion (lower as the finger approaches the target.
down). • Dysdiadochokinesis: Present.
• Talk with the patient: Scanning or staccato speech • Heel-shin test: Impaired (incoordination).
(with bilateral lesion). • There is hypotonia in the right (or left) lower limb.
• Eye: Nystagmus (jerky, horizontal and coarse with • Knee jerk is pendular (mention, if any).
ina-ease in amplitude on looking towards the site • Gait: Reeling or drunken.
of lesion). Skew deviation of the eyes: Ipsilateral • Rombergism is absent.
.down and inwards, and contralateral up and out.
• Finger-nose test: Positive (look for past point and My diagnosis is cerebellar lesion (mention the side-
intention tremor that increases on approaching
right or left).
the target).
• Oysdiadochokinesis (rapid alternating pronation Q: What do you think are the causes in this case?
and supination of forearm); tap one hand with A: Mention the causes according to the age of the patient:
its palmar, and dorsal aspect on the other hand If the patient is young, the causes are:
alternately. • Friedreich ataxia.
• Rebound phenomenon: Askthe patient to hold both • MS.
the arms out and front, and keep it. Push both arms • Wilson disease.
and release suddenly. Ipsilateral arm will fly past the • Drugs (phenytoin and carbamazepine).
starting point (it is due to the failure of reflex arrest). • Others: Hypothyroidism and trauma.
• Also ask the patient to extend the arms and look If the patient is middle-aged or elderly, the
for arm drift due to hypotonia of agonist muscles. causes are:
Now, examine the lower limbs for cerebellar signs: • MS.
• Brainstern vascular lesion.
• Heel-shin test (incoordination).
• Paraneoplastic syndrome.
• Ipsilateral hypotonia (it is due to the loss of
• Alcohol and drugs.
facilitatory influence on spinal motor neuron) and
• Secondary deposit in cerebellum.
diminished muscle power.
• Degenerative lesions:
• Knee jerk (pendular}.
o Progressive cerebellar degeneration.
• Dysdiadochokinesia of foot (ask the patient to tap
o Olivopontocerebellar degeneration.
your hand by his/her foot).
o Shy-Drager syndrome.
• Gait (reeling or drunken: Patient will stagger
o Creutzfeldt-Iakob disease.
towards the affected side).
8 • NEUROLOGY __

Q: What are the drugs causing cerebellar syndrome? 6. Infection (cerebellar abscess from otitis media,
A: Phenytoin, carbarnazepine. lithium, phenobarbi- HTVand Kuru).
tone, sometimes chemotherapy agents. 7. Inherited (Friedreich ataxia and other hereditary
ataxias).
Q: Mention one investigation to confirm cerebellar
8. Cerebellar syndrome of malignancy (paraneoplas-
lesion. tic syndrome).
A: MRl.
9. Cerebellar syndrome:
Q: What is the nature of the cerebellar tremor? • Shy-Drager syndrome.
A: Intension tremor, which is absent at rest but appears • Steele-Richardsori-Olszeweski syndrome.
during voluntary activity, especially when approach- • Creutzfeldt-Iakob disease.
ing to a target. • Wilson disease.
10. Others:
Q: What are the findings, if there is vermis lesion? • Hypothyroidism.
A: As follows: • Arnold-Chiari lesion.
• Ataxia is usually truncal, causes difficulty in • Trauma (punch-drunk syndrome).
standing and sitting unsupported with broad- • Cerebral palsy.
based gait. • Hydrocephalus.
• Only lower limbs are affected. When limbs are
tested separately on bed, usually little or no sign. Q: What are the functions of cerebellum?
• Romberg sign may be positive (same on dosing A: It is concerned with the control of voluntary move-
and opening of the eye-to differentiate from ments and maintenance of posture and balance.
sensory ataxia).
Q: What are the malignancies causing paraneoplastic
Q: What is the cause of vermis lesion? syndrome of cerebellum?
A: Alcohol, which causes atrophy of anterior part of A: Carcinoma of ovary, uterus, breast, small-cell
vermis (sparing the upper limb). carcinoma of the lung, Hodgkin lymphoma, etc. This
is probably immune mediated. Cerebellar lesions
Q: When to suspect only midline lesion? What are the
are usually bilateral (unilateral lesion is against
causes?
paraneoplastic cerebellar lesion). Two antibodies are
A: When only truncal ataxia is present, there is
recognized for different malignancies:
abnormal speech and heel-toe walking is positive.
• Anti-yo (anti-Purkinje cell antibody): Related to
The causes are:
carcinoma of ovary, uterus and breast.
• Paraneoplastic syndrome.
• Anti-Hu (antineuronal cancer antibody): Related
• Midline space-occupying lesion. to small-cell carcinoma of lung, carcinoma of
Q: What are the findings if there is lesion in the cer- prostate, sarcoma and neuroblastoma.
ebellar hemisphere? N.B. Remember that signs of cerebellar lesion occur
A: There is ipsilateral limb ataxia. on the same side of lesion, because most cerebel-
lar fibres cross twice in brainstern while entering
Read the Following Topics in Relation to and exiting the cerebellum. Summary of the
Cerebellum signs of cerebellar lesions:
• Titubation.
Causes of cerebellar lesion:
• Tilting towards the site of lesion.
1. Vascular [cerebellar haemorrhage or infarction, • Nystagmus (horizontal).
arteriovenous (AV) malformation and brainstem • Scanning speech.
vascular lesion]. • Intention tremor.
2. Demyelinating eMS). • Incoordination.
3. Drugs (phenytoin and carbamazepine) and alcohol.
• Dysdiadochokinesis.
4. Toxins (carbon monoxide poisoning, solvent
• Past-pointing (dysmetria).
abuse and lead poisoning).
• Ataxia.
5. Neoplasm (haemangioblastoma, medulloblastoma,
• Hypotonia.
astrocytoma, secondary deposit and compression by
• Diminished tendon reflex (knee jerk may be
acoustic neuroma).
pendular).
_ SHORT CASES IN CLINICAL MEDICINE

Parkinsonism

Usual instructions are: I


Presentation of
• Look at the face. What are your findings? What else Case No.1
do you want to examine?
• Look at the hands (resting tremor). What else do (By looking at the face):
you want to examine? • The patient has titubation of head (mention, if
Proceed as follows: any).
1. In the face: • There is mask-like, expressionless face with less
• Titubation of head. blinks of the eyes, staring look and dribbling of
• Mask-like, expressionless, less blinking face saliva.
with staring looks (frequency of spontaneous
blinking is reduced, called serpentine stare).
Blepharodonus (tremor of eyelids, when eyes My diagnosis is parkinsonism.
are gently closed).
Q: What else do you like to examine?
• Dribbling of saliva.
2. Talk to the patient: Speech-slow initiation, husky, A: Comment on resting tremor, if present. Then tell,
slurred, indistinct, lacking intonation, low volume "I want to examine the following":
and monotonous (or mutism). Palilalia is present • Glabellar tap (positive).
(repetition of the end of a word). • Muscle tone (rigidity-cogwheel or lead-pipe).
3. Glabellar tap: Positive (tap the forehead above the • Speech (talk to the patient).
bridge of the nose repeatedly). In normal person, • Hypokinesia (see the fastening of button and
blinking will stop after three to five blinks, but in writing by the patient).
parkinsonism, the patient continues to blink. This • Gait (see below).
sign is unreliable.
4. Repetitive tapping over the bridge of nose (two
per second): Produces a sustained blink response
Presentation of I

(Myerson sign).
Case No.2
5. Look at the tremor (see below): Present at rest,
tremor disappears or reduces with activity or (As described in Case no. 1) plus
holding something. • There is resting tremor with pill-rollingmovement
6. Rigidity: Lead pipe (better seen in elbow) or cog-
of right (or left) thumb, which disappears after
wheel (better seen in wrist).
voluntary movement or holding something.
7. Tests for hypokinesia:
• Rigidity is present (which is cog wheel or lead
• Ask the patient to do fastening of button.
pipe).
• Ask to write (micrographia and handwriting is
• There is hypokinesia.
tremulous and untidy).
• Speech is husky, slurred, monotonous and low
• Ask to touch tip of all fingers with thumb
successively or ask to count (slow initiation, volume.
unable or can do slowly or progressive reduction • Gait (ask the patient to raise from chair, walk,
of amplitude of each movement). turn quickly, stop and start):
• Ask the patient to do rapid fine fj nger movement o Difficulty in starting to walk. Once started,
(like piano playing): It becomes indistinct, rapid, small, shuffling steps occur (hardly
slurred and tremulous. raising the foot from ground), as if trying to
• Ask the patient to perform two different simulta- keep up with his own centre of gravity.
neous motor acts (patient is unable to do so). o Stooped or flexed attitude with less swinging
8. Ask the patient to stand and see the position of arms.
(flexed and stooped attitude). o Rapid walking and difficulty in stopping
9. Gait: Ask to walk and to turn quickly (difficulty in himself with tendency to run (festination).
starting to walk, called freezing, paucity of move- o He has difficulty in rapid turning (fractionated
ment, less swinging of arms and flexed attitude; gait).
inability to turn rapidly, called fractionated turn).
8 • NEUROLOGY _

My diagnosis is Parkinsonism. • History of head injury (punch-drunk syndrome).


Q: What do you think are the causes of Parkinsonism • Drugs (phenothiazine, butyrophenone, metochlo-
in this case? prarnide, reserpine, tetrabenazine, a-methyldopa,
A: Mention the causes according to the age: lithium, valproic acid, fluoxetine).
• Any history of fever, convulsion, headache, coma
In elderly patients, the causes are:
(postencephalitic) .
• Idiopathic or paralytic agitans (the commonest • Jaundice, chronic liver disease (CLD) (Wilson
cause). disease).
• Drugs • Any history of headache, vomiting, convulsion
• Postencephalitic Parkinsonism. (cerebral tumour).
• Neurosyphilis.
• Trauma. Q: Describe the tremor in Parkinsonism.
• Cerebral tumour. A: Tremor is involuntary, coarse (4-6 Hz), present at
In young patients, the causes are: rest, disappears or reduces during voluntary activity
and sleep, and ina-eases with emotion or anxiety.
• Postencephalitic Parkinsonism.
Initially, the tremor is characterized by pill-rolling
• Drugs. movement between thumb and index finger, flexion
• Wilson disease.
and extension of fingers, abduction and adduction
of thumb, and pronation and supination of forearm.
Later, tremor may affect arms, legs, feet, jaw and
tongue. Tremor is absent in one-third cases at
presentation and throughout its course in some cases.
Commonly, the patient presents with unilateral
resting tremor in hand.

Q: What are the types of rigidity in Parkinsonism?


A: They are of two types:
• Lead pipe: Uniform rigidity in flexors and
extensors oflimbs (better seen in elbow or knee).
• Cog wheel: Rigidity is interrupted by tremor
(better seen in wrist joint). It is due to exaggerated
stretch reflex interrupted by tremor.
N.B. Sometimes, rigidity of the examining limb is
Face in a case of Parkinsonism increased with simultaneous active movement
of the opposite limb. (It is one type of rein-
forcement).

Q: What is the difference between rigidity and


spasticity?
A: As follows:
1. Spasticity means increased resistance during the
initial part of passive movement, followed by
lessening of the resistance.
• It may be clasp-knife type, in which there is
more resistance at the onset of movement,
followed by sudden loss of resistance. It is
due to pyramidal lesions. Spasticity is better
felt with attempting extension of upper limbs
and flexion of lower limbs. It is associated
with other signs of UMN lesion. It involves
Parkinsonism (flexed attitude) only the antigravity muscles (extensors of the
upper limbs and flexors of the lower limbs).
Q: What specific past history would you like to take? 2. Rigidity means sustained uniform resistance
A: As follows: during passive movement. Rigidity is found in
extrapyramidal lesion and involves all groups of • Rapid small shuffling step (festination) in order
muscles. It may be: to avoid falling. The patient hardly raises the foot
• Lead pipe in which resistance is uniform from the ground.
throughout the passive movement (better • The patient seems to catch up with his own centre
seen in elbow and knee). of gravity (flexed attitude).
• Cog wheel in which continuous resistance is • There is less swinging of the arms during walking.
interrupted by tremor (better seen in the wrist • The patient has difficulty in SLopping himself.
and ankle joints). • He has difficulty in rapid turning (fractionated
gait) and turns en block.
Q: What are the features of hysterical rigidity?
• Obstacles cause the patient to freeze in place.
A: In hysterical rigidity, muscle tone increases more
and more with increasing manoeuvre of the affected Q: What are the other abnormal gaits in Parkinsonism?
limb. The more the limbs are moved or examined, A: As follows (these tests should not be done at the
the more rigid it gets. bedside):
Q: What is dyskinesia or akinesia? Describe dyskinesia • Propulsion: ]fthe patient is pushed from behind,
or hypokinesia in Parkinsonism. he is unable to stop himself and may fall forward.
A: Dyskinesia is the difficulty in initiating motor • Retropulsion: If the patient is pushed from
activity or poverty or slowing of movement (brady- front, he is unable to stop himself and may fall
kinesia). In Parkinsonism, it may be: backward.
• Difficulty in initiating movement. • Kinesia paradoxica.The patient is unable to initiate
• Slowness of movement (bradykinesia). The patient a movement; but once started, can complete the
is slow and ineffective in attempt to deliver a quick, whole act (may run down the stairs, but cannot
hard blow; he cannot complete a rapid ballistic stop at the bottom) or the patient is unable to
movement. initiate a movement, but during emotion or fear
• Poverty of movement. (e.g. fire in the house), can perform the movement
• Alternative movement progressively impeded, (even run out from the house).
finally blocked completely.
• Difficulty in executing two motor acts Q: What are the reflexes and plantar response in
simultaneously. Parkinsonism?
• Bradykinesia is reflected also by slowness in A: All the reflexes and plantar responses are normal. It
chewing, limited capacity to make postural may be difficult to elicit because of rigidity. Plantar
adjustment. is flexor. It may be extensor if it is associated with
Q: What are the differences between essential tremor the following disorders:
and Parkinsonian tremor? 1. Postencephalitic Parkinsonism.
A: As follows: 2. Other diseases (called atypical Parkinsonian
syndrome):
Parkinsonian • Shy-Drager syndrome
Features Essential tremor tremor • Steele-Richardson-Olszeweski syndrome
Stimulus Occurs with action Occurs with rest (progressive supranuclear palsy).
Family H/O Yes No • Olivopontocerebellar atrophy (OPCA)
tremor • Corticobasal degeneration (CBO).
Body parts Hands, head Hands, legs, rarely
involved head Q: What is Parkinsonism?
Distribution Bilateral and Unilateral and A: It is a syndrome consisting of tremor, rigidity, brady-
at onset symmetric asymmetric kinesia and loss of postural reflexes.
Sensitivity Yes No
Q: What is Parkinson disease?
to alcohol
A: Parkinson disease (paralysis agitans) is the primary
Course Stable and slowly Progressive
progressive
or idiopathic Parkinsonism. It is a neurodegen-
erative disorder due to involvement of the basal
Q: Describe the typical gait in Parkinsonism. ganglia, characterized by slowness of movement,
A: Festinate gait, characterized by: rigidity, tremor and loss of postural reflex.
8 • NEUROLOGY __

Q: What is Parkinsonian plus? • Stage IV:Stage 3 plus severe postural abnormality


A: It characterized by features of Parkinsonism requiring substantial help.
with other degenerative disease like progressive • Stage V: Severe, fully developed disease. The
supranuclear palsy (Steele-Richardson-Olszeweski patient is restricted to bed and wheel chair.
syndrome), olivopontocerebellar degeneration,
nigrostriatal degeneration, primary autonomic Q: What are the causes of Parkinsonism?
failure (Shy-Drager syndrome). A: Unknown, multiple factors are responsible: z
(1)

Features that may indicate a diagnosis of Parkin- 1. Paralysis agitans (idiopathic, also called Parkin- E;
o
son-plus syndrome include: son disease). Itusually occurs in middle-aged or 0-
elderly. ~
• Symmetrical features, especially at an early stage.
• Early onset of features like postural instability, 2. Postencephalitic (encephalitis lethargica and
fall, dementia, hallucinations, autonomic Japanese B encephalitis).
dysfunction, etc. 3. Drugs: Phenothiazines (chlorpromazine,
• Presence of pyramidal signs (not due to previous prochlorperazine), butyrophenones (haloperi-
stroke or other pathology), cerebe.llar signs or dol), metoclopramide, sulpiride, cisapride,
ocular signs (e.g. nystagmus, gaze palsy). tetrabenazine and methyldopa.
• Poor response to levodopa. 4. Neurosyphilis.
5. Poisoning: Carbon monoxide, manganese and
Q: What are the diagnostic criteria for Parkinsonism?
MPTP (methyl-phenyl-tetrahydropyridine) may
A: Triad of:
occur in drug addicts.
• Tremor at rest (4-6 Hz)
6. Herbicide (paraquat, may be related to MPTP).
• Rigidity
• Hypokinesia 7. Trauma (punch-drunk syndrome and repeated
(Remember the formula TRH. According to Brain head injury).
Bank criteria, postural instability is another criterion). 8. Genetic (Wilson disease and Huntington dis-
ease).
Q: What is the pathological change in Parkinsonism? 9. Cerebral tumour (involving basal ganglia).
A: In idiopathic Parkinsonism, there is progressive 10. Parkinsonian plus (when associated with fea-
degeneration of the pigmented dopaminergic tures or pathology of other disease):
neurons of substantia nigra and formation of eosin-
• Shy-Drager syndrome.
ophilic cytoplasmic inclusions in neurons (Lewy
• Steele-Richardsori-Olszeweski syndrome
bodies, which is the pathological hallmark). Hence,
(progressive supranuclear palsy, characterized
there is a deficiency of dopamine (and melanin)
by inability of the movement of eye vertically
with relative increase in cholinergic transmission
or laterally and dementia).
(imbalance between dopamine and acetylcholine).
• Olivopontocerebellar degeneration.
Q: What is the mental status in a patient with 1l. Creutzfeldt-Iakob disease.
Parkinsonism? 12. Hypoparathyroidism.
A: As follows: 13. Normal pressure hydrocephalus (triad of urinary
• Initially, intellect and memory are normal. There incontinence, gait apraxia and dementia).
may be slowness of thought and memory retrieval 14. Atherosclerotic Parkinsonism (stepwise progres-
(bradyphrenia) and subtle personality changes. sive broad-based gait and pyramidal signs).
• Depression occurs in one-third of the patients.
• Global dementia (20%) and psychosis. Q: What investigations should be done in
• Drug treatment may precipitate acute confusion. Parkinsonism?
A: Diagnosis is usually clinical. Investigations are done
Q: What are the stages of Parkinsonism?
for specific cases or to exclude other diseases:
A: As follows:
1. cr scan or MRI (may be done if there is pyrami-
• Stage I: Unilateral involvement (hemiplegic
dal, cerebellar and autonomic involvement or
Parkinsonism ).
• Stage II: Bilateral involvement but no postural doubtful diagnosis).
abnormality. 2. In patient <50 years, screening for Wilson disease:
• Stage Ill: Bilateral involvement with mild postural • Serum ceruloplasmin (low).
abnormality. • Serum copper (high serum free copper).
__ SiiORT CASES IN CLINICAL MEDICINE

• 24 h urinary copper (high). Following Q: What are the treatment modalities in Parkinsonism?
penicillamine therapy, 24 h urinary copper A: As follows:
>25 mmol is confirmatory. • Treatment of the cause and withdrawal of
• Liver function tests may be done. offending drugs, if any.
• Liver biopsy with quantitative measurement • Symptomatic treatment of tremor, rigidity and
of copper (less done). bradykinesia.
Q: How to differentiate between postencephalitic • Physiotherapy and speech therapy.
Parkinsonism and paralytic agitans? • Surgical treatment.
A: As follows: • Occupational therapy and rehabilitation.

Q: What is the treatment of Parkinson disease?


Postencepha- Paralytic
A: Drug treatment is usually not started in mild case
Criteria litic agitans
because of untoward side effects. It should be started
Age Any, commonly Elderly or
when there is significant disability and when the
young late-middle age
symptoms begin to interfere with work and social
Onset Sudden Insidious
life, or falling becomes a threat.
Previous history Encephalitis, No particular
1. Combination oflevodopa and dopa decarboxylase
fever and history
inhibitor is the treatment of choice. Available
headache
combinations are levodopa and carbidopa
Complaints Mainly rigidity, Mainly tremor
(co-careldopa 110 or 275 mg), and levodopa and
also impaired
benserazide (co-beneldopa 62.5 mg). Treatment
higher functions,
excess salivation should be started with lowest possible dose and
(autonomic gradually increased as needed.
features), little or 2. Tremor and rigidity may be controlled by
no tremor anticholinergic drugs (such as trihexiphenidyl,
Symmetry Fairly symmetrical Asymmetrical benztropine, orphenadrin, benzhexol, biperiden).
rigidity and 3. Other drugs: Amantadine, selegilin, catechol-
hypokinesia O-methyl transferase (COMT) inhibitors
Eye signs: (entacapone, tolcapone), dopamine agonist
• Oculogyric crisis Present Absent (pergolide, bromocriptin, lisuride, ropinirole
• Ophthalmoplegia Present Absent and pramipexole) may be used.
• Pupil Abnormal No abnormality 4. General measures:
(dilated, irregular)
• Physiotherapy and speech therapy.
Neurological features: • Occupational therapy and rehabilitation.
• Dystonia, dementia, Present Absent 5. Other measures:
chorea, hemiparesis
• Cognitive impairment and psychiatric
• Rigidity Usually lead pipe Usually
symptoms may be helped by rivastigmine.
(due to absence cogwheel
(Selective serotonin reuptake inhibitory
of tremor)
• Tendon reflexes Brisk Normal
(SSRI) are drugs of choice for depression.
• Plantar response Extensor Flexor These drugs may aggravate Parkinsonian
symptoms. Trazodone is helpful in treating
Levodopa Lesssensitive Sensitive
depression and insomnia.)
Prognosis Not good Slowly
• For psychosis, confusion or hallucination:
progressive
Atypical antipsychotic can be given.
Lewybody Absent Present
Q: What is the role of surgery in Parkinsonism?
Q: What is oculogyric crisis? A: Surgery may be considered in some particular cases
A: It is an involuntary, sustained upward and outward of Parkinson disease. However, it is rarely done
deviation of the eyes. Causes are: because medical treatment is available. Options are:
• Postencephalitic Parkinsonism. • Stereotactic thalamotomy (ventrolateral nucleus
• Drugs (commonly phenothiazines). of thalamus): Usually unilateral. Bilateral
• Petit mal epilepsy. thalamotomy is not recommended. It relieves
8 • NEUROLOGY __

tremor on the contralateral side (but little effect patient complains of freezing and rigidity before
on bradykinesia, rigidity, motor fluctuation and the next dose of levodopa. This may be managed
dyskinesia). by dividing levodopa into smaller and frequent
• Pallidotorny: Destruction of a part of the doses. Also by using slow-release preparations or
globus pallidus interna is done. It helps in the adding dopamine agonist or adding amantadine.
improvement of contralateral features like tremor, • On-off phenomenon: After prolonged use, the
bradykinesia and rigidity. Also reduces dyskinesia. drug may become less effective. There is sudden,
Bilateral pallidotomy is not recommended. unpredictable change in response in which periods
• Subthalamotomy: Involves destruction of a part of severe Parkinsonism (freezing and immobility-
of subthalamic nucleus. It improves contralateral the off period) alternate with periods of
features like tremor, bradykinesia and rigidity. dopamine-induced dyskinesias, agitation, chorea
• Deep brain stimulation: It is a procedure that has and dystonic movements (the on period). This can
replaced the surgery. A lead is implanted into the be managed by lowering the dose of levodopa or
targeted brain structure such as thalamus, globus adding seJegiline with levodopa. COMT inhibitor
pallidus interna or subthalamic nucleus. Then it or dopamine agonist may be added.
is connected to an implantable pulse generator,
Q: What are the extrapyramidal effects of phenothi-
usually in subclavicular area, that delivers high-
azi ne group of drugs (or antipsychotic drugs)?
frequency electrical discharge. It can be used to
A: As follows:
stimulate bilateral lesion.
• Foetal midbrain or adrenal tissue implantation in • Parkinsonism (tremor is Jess and responds to
basal ganglia. This is helpful in younger patients. anticholinergic drugs than L-dopa).
Cannot be used regularly because of ethical issues. • Dystonia (by prochlorperazine and
metoclopramide) .
Q: What are end of dose deterioration and on-off • Akathisia (it is the uncontrolled restlessness with
phenomenon? repetitive and irresistible need to move).
A: After 3-5 years of levodopa therapy, there may be • Tardive dyskinesia: Characterized by orofacial
fluctuating response to levodopa in up to half of the dyskinesia including lip smacking, chewing,
patients. These include: pouting and grimacing. It is usually due to use of
• Bnd of dose dyskinesia (wearing-off effect): Due phenothiazines and butyrophenones for at least
to progression of the disease and loss of capacity . 6 months. May be worse or may persist after the
to store dopamine, duration of action oflevodopa withdrawal of drug. Tetrabenazine may help.
becomes progressively shorter. As a result, the • Chorea.

Chorea

Usual instructions are:


• There may be writhing or dancing movement of
• Look at the patient. What is your diagnosis? What the limbs.
else do you want to see?

Presentation of a Case My diagnosis is chorea (or choreoathetosis).


(by Looking) Q: What else do you want to examine?
A: As follows:
• There is an involuntary, nonrepetitive.
quasipurposive, irregular, jerky movement of • See muscle tone (there is hypotonia).
the arm, head and whole body, which is present • Ask the patient to protrude the tongue: Patient
at rest and increases after activity. is unable to keep the tongue protruded out. (It
• The movements are randomly distributed and darts in. and out called serpentine movement or
moves from one part of the body to other. jack in the box.)
• The patient looks clumsy and restless (may keep • Ask the patient to raise both arms above the
dropping object). head opposing the palmar side (hands will sway
outward; there is pronation of forearm).
• Perform handshake with the patient. (There is Q: What history should be taken?
alternate squeezing and gripping of patient's A: As follows:
finger or lack of sustained hand gripping called • Family history (in Huntington chorea and Wilson
milkmaid's grip.) disease).
• Ask the patient to raise both arms in front. (There • Drugs (neuroleptics, phenothiazine, tricyclic
may be flexion of wrist joint and hyperextension antidepressant, oral contraceptive pill, phenytoin
of metacarpophalangeal and finger joints called and L-dopa).
dinner fork deformity or choreic hand.) • In females: Oral contraceptive pill and pregnancy.
• History of encephalitis.
• In young patients or children: History of sore
throat or rheumatic fever (rheumatic chorea).
• Others: History of thyrotoxicosis, polycythaemia
rubra vera (PRV) and SLE.
Read the following in relation to chorea:

Q: What is chorea?
A: It is the involuntary, nonrepetitive, quasi purposive,
irregular and jerky movements of one or more parts
of the body due to extrapyramidal lesion. Chorea
may be unilateral or generalized; sometimes patient
attempts to disguise this by completing the involuntary
movements with a voluntary movement. It worsens
with anxiety or activity and disappears during sleep
Chorea (chorea-means dance, from a Greek word),

Q: What is the site of lesion in chorea?


A: Caudate nucleus of basal ganglia, Also, due to exces-
sive activity in striatum due to dopaminergic drugs
used to treat Parkinsonism. Dopaminergic path-
ways dominate over cholinergic transmission.

Q: What are the causes of chorea?


A: As follows:
1. Rheumatic chorea (poststreptococcal called
Sydenham chorea or St. Vitus dance).
2. Senile chorea.
3. Hereditary (Huntington chorea, Wilson
disease, benign familial chorea, paroxysmal
choreoathetosis, spinocerebellar ataxia and
neuroacanthocytosis) .
Chorea 4. Drug induced (see above).
5. Pregnancy (called chorea gravidarum).
6, Encephalitis Jethargica.
7. Following stroke.
8. Others (rare):
• Polycythaemia rubra vera and other
myeloproliferative disorders.
• SLEand anti phospholipid syndrome.
• Endocrine (thyrotoxicosis, idiopathic
hypoparathyroidism and hypoglycaemia).
• Kernicterus.
• Cerebral birth injury.
• Cerebral trauma.
Chorea • Creutzfeldt-Iakob disease,
8 • NEUROLOGY __

• Henoch-Schonlein purpura. lower limb more than upper limb. Occasionally,


• Vascular (lacunar infarction and arteriovenous there is juvenile onset where Parkinsonism is the
malformation). main feature.
• Carbon monoxide poisoning.
Huntington chorea is diagnosed by:
Q: How to treat chorea? • Family history.
A: As follows: • Chorea followed by progressive dementia,
• Reassurance and treatment of primary cause producing a dancing sort of gait.
(if any). Other features of Huntington disease, chorea may
• Drugs that may be helpful are phenothiazine,
be associated with the following:
buryrophenones (haloperidol), tetrabenazine
and sodium valproate. • Bradykinesia.
• Myoclonus.
Q: What is Sydenham chorea? • Dystonia, dysarthria, dysphasia.
A: It occurs in rheumatic fever due to the involvement • Ataxia.
of CNS that develops after streptococcal infec- • Slow, saccadic eye movements.
tion and there is diffuse mild encephalitis. History • Cognitive impairment.
of rheumatic fever may be present in one-third of • Psychiatric disturbance.
the cases. • Rigidity.
• Common in children and adolescents, more in
Pathological changes in Huntington chorea:
female, age 5-15 years.
• Chorea is associated with emotional instability, • Cerebral atrophy With neuronal loss in caudate
irritability, inattentiveness, confusion and fidgety. nucleus and putamen.
Speech is often affected. Changes of neurotransmitters:
• It manifests for long time, may be 6 months after
• Reduction of acetylcholine transferase and
the initial infection. glutamic acid decarboxylase (GAD) in the
• Other evidences of rheumatic fever may be absent corpus striatum.
when chorea is present. • Depletion ofy-aminobutyric acid (GABA),
• Carditis may be first manifestation and rheumatic substance P, angiotensin-converting enzyme and
heart disease may occur. metencephalin in substantia nigra.
• Fever is unusual, and erythrocyte sedimentation
• High somatostatin level in corpus striatum.
rate (ESR), antistreptolysin 0 (ASO) titre and
C-reactive protein (CRP) are usually normal. Investigations:
• It is usually self-limiting and recovers within • cr scan or MRJ:Atrophy of caudate nucleus and
weeks or 1 month (may be 5-15 weeks). also cerebral atrophy.
Recurrence may occur in 20% cases. Occa- • DNA analysis.
sionally, it may relapse during pregnancy (called • Haloperidol or phenothiazine for dyskinesia.
chorea gravidarum) or in those who use oral Tetrabenazine may be given.
contraceptive pills. • Psychological support.
Treatment: • Institutional care for dementia.
• Genetic counselling is essential.
• No treatment in most cases as recovery is
spontaneous. Q: Name some involuntary movements.
• In patient with severe chorea: Benzodiazepine, A: As follows:
haloperidol, tetrabenazine or val pro ate may be • Tremor.
given. • Chorea.
• Penicillin prophylaxis up to the age of 20 years • Athetosis.
(as in rheumatic fever). • Hemiballismus.
Q: What is Huntington chorea? • Myoclonus.
A: It is a disorder, inherited as autosomal dominant, • Tic.
in which chorea is associated with progressive • Torsion dystonia (it may be generalized or local-
dementia. Gene responsible is on the short arm ized such as spasmodic torticollis, writer's cramp,
of chromosome 4. Usually present in adult, dur- oromandibular dyskinesia, blepharospasm and
ing the third to fourth decade. Chorea involves hemiplegic dystonia).
__ SHORT CASES IN CLINICAL MEDICINE

Tremor
Usual instructions are: • Resting tremor (typical of Parkinsonism).
• Look at this patient. What are your findings (resting • Action tremor or postural tremor (present on
tremor)? What else do you want to examine? outstretched hands).
(See other signs of Parkinsonism.) • Intention tremor.
• The patient has tremor. Now examine. According to the amplitude or nature, it may be fine
or coarse.
Proceed as follows:
Q: What are the causes of action tremor?
1. If the tremor is present at rest, see abduction- A: As follows:
adduction of the thumb (pill-rolling movement)
• Anxiety.
and flexion-extension of fingers. Likely diagnosis • Thyrotoxicosis.
is Parkinsonism (then examine for other signs of • Senile tremor.
Parkinsonism). • Benign essential tremor.
2. If no resting tremor, ask the patient to outstretch • Cerebellar tremor (increases near the target).
the hands in front. If tremor is present, it is called • Drugs (see above).
action tremor. Then examine according to suspi- • Familial.
cion: Check for thyrotoxicosis, history of taking • Idiopathic (in many cases).
drugs and family history.
3. Ifno resting tremor or no tremor with outstretched Q: What is intention tremor?
hands, then test for intention tremor (cerebellar A: Tremor that comes on voluntary movement, but
lesion); and if present, check for cerebellar signs. disappears on rest is called intention tremor. It is
4. If still no tremor, see for flapping tremor. caused by cerebellar lesion due to any cause.
Q: What are the causes of fine and coarse tremor?
Q: What is tremor? A: As follows:
A: It is the involuntary, oscillatory and rhythmical 1. Causes of fine tremor:
movement of one or more parts of the body due • Anxiety or nervousness.
to alternate contraction of a group of muscles and • Thyrotoxicosis.
their antagonists. • Senile tremor.
Q: What are the causes of tremor? • Benign essential tremor.
A: As follows: • Drugs (e.g. salbutamol, terbutaline).
• Functional (anxiety, hysterical conversion • Familial.
reaction and nervousness). • GPr.
2. Causes of coarse tremor:
• Endocrine (thyrotoxicosis, phaeochromocytoma
and hypoglycaemia) • Parkinsonism.
• Intention tremor.
• Parkinsonism.
• Cerebellar tremor (also called intention tremor). • Flapping tremor in hepatic precorna.
• Wilson disease.
• Benign essential tremor.
• Sometimes in senile tremor.
• Senile tremor.
• Drugs: Salbutamol and other ~-agonist, Q: What is the nature of tremor of GPE
phenothiazines, butyrophenones, methyldopa, A: Tremor is usually present in the tongue, is seen
lithium intoxication, anticonvulsant (phenytoin, during attempting protrusion, and manifested as
carbamazepine and sodium valproate), backward and forward movement of the tongue
amphetamine, theophylline and caffeine. called trombone tremor (other features of GPI are
• Alcohol (chronic alcoholism and alcohol dementia, Argyll Robertson pupil and bilateral
withdrawal). UMN lesion signs).
• Toxin (mercury, arsenic and lead).
Q: What is benign essential tremor?
• General paresis of insane (GPI).
A: Itis a familial tremor that is inherited as anautosomal
• Flapping tremor.
dominant and is usually present in outstretched
Q: What are the types of tremor? hands and also when hands adopt a posture such
A: They are of three types: as holding a glass or spoon. Occasionally, present
8 • NEUROlOG __

at rest. Worse in upper limbs. Often, there is tituba- Treatment:


tion. Other features are: • Propranolol is helpful in small dose.
• Slowly progressive, though benign, rarely • Alcohol may relieve the tremor; but there is
produces severe disability. No rigidity and no chance of addiction.
hypokinesia. • Primidone is sometimes helpful.
• Tremor is not aggravated by movement. • Rarely in severe cases, injection of botulinum
• It is common in elderly (but may occur at any age). toxin may be helpful. z
~
,...
• Handwriting is shaky and untidy, but no • Rarely, in intractable cases, stereotactic o
m icrographia. thalamotomy. 0-
~
• Anxiety increases the tremor.
Q: What are the differences between benign essential
Site of lesion is patchy neuronal loss in cerebellum tremor and Parkinsonian tremor?
and cerebral connection. A: (See in 'Parkinsonism').

Speech
Usual instructions are: • See whether comprehension is good (expressive
1. Instruction 1: Talk with the patient or ask some or motor type).
questions (to test whether a candidate can diagnose • Comprehension is impaired (sensory type).
by talking with the patient). Speech disorders are: Ask the following questions:
• Dysphasia.
• What is your name?
• Dysarthria.
• Dysphonia. If unable to answer, then ask:
• Any voice change: Husky and croaky • Close your eyes.
(myxoedema). • Put out your tongue.
2. Instruction 2 (other than speech disorder): Look • Raise your right or left hand or both hands
at the patient. Now, ask some questions. above head.
For instruction I, proceed as follows: If the patient can perform all these, it is motor
• Once you talk to the patient, try to find out the type of dysphasia.
nature of speech.
If comprehension is not good, then ask:
• Is it dysphasia, dysarthria or dysphonia?
• What is your name?
Next question will be according to the nature of • What is your address?
speech, as follows:
The patient answers fluently, but speech is mean-
If there is dysarthria, ask some questions to find
ingless or incoherent (not related with the ques-
out type of dysarthria:
tion). There may be inappropriate words or new
• Try to ask a question with long sentence
words or nonexisting words. Hence, it is likely to
(e.g. Would you please tell me your name and
be sensory type of dysphasia.
address' ? or What breakfast did you take this
morning' ?) If the patient neither answers nor responds to
your instructions, such as:
Ask the patient to speak 'British constitution or
West Registrar Street.' Ascertain whether the speech • What is your name? (no answer).
is cerebellar or pseudobulbar or bulbar palsy: • What is your address? (no answer).
• Is it scanning or staccato? (Slow, slurred, Then hold a pen. Ask the patient, 'What is it'? If no
explosive or speech broken into syllable-by- answer, then ask:
syllable-all suggestive of cerebellar lesion.)
• 'Is it a key'? The patient answers, 'No'.
• Is it spastic? (Pseudobulbar palsy)
• 'Is it a pen'? The patient answers, Yes'.
• Is it nasal and indistinct? (Bulbar palsy)
If there is dysphasia, try to find which type of Then the diagnosis is nominal dysphasia (the
dysphasia (motor or sensory): patient answers either yes or no).
SHORT CASES IN CLINICAL MEDICINE

Q: What are the component parts of speech? Q: Where are the sites of lesion in different types of
A: There are three component parts of speech: dysarthria?
• Phonation (abnormality is called dysphonia). A: As follows:
• Articulation (abnormality is called dysarthria). • UMN lesion of cranial nerves.
• Cerebellum.
• Language (abnormality is called dysphasia).
• Brainstem.
Q: What is dysphasia? • Extrapyramidal system.
• At the periphery in the muscles of face, lip, tongue
A: It is the disordered use of language with or without
and pharynx.
impaired comprehension of received speech.
N.B. For details about dysarthria, see in the topic
Q: What are the types of dysphasia? "Dysarthria" later in this chapter.
A: As follows:
Q: What is dysphonia?
• Motor dysphasia (expressive).
A: It is the alteration of the quality of voice with reduc-
• Sensory dysphasia (receptive). tion in volume as a result of vocal cord disease. It is a
• Nominal dysphasia. disorder of vocalization; the fault is in the vocal cord.
• Global dysphasia (both motor and sensory).
Q: What are the causes of dysphonia?
N.B. For details about dysphasia, see in the topic A: It is found in laryngitis, tumour of the vocal cord or
"Dysphasia" later in this chapter. bilateral adductor paralysis. Sometimes hysterical.

Q: What is dysarthria? For instruction 2, proceed as follows: There is


A: It means the defect in articulation and enunciation likely to be obvious diagnosis by looking at the
of speech. It may result from the lesion of the patient. Now talk to the patient to find out typical
muscles, myoneural junctions, or motor neuron of speech change in that disease. Examples are:
lips, tongue, palate and pharynx. • Hypothyroidism - croaky or husky voice.
• Parkinsonism - slow initiation of voice, dysphonia.
Q: What are the types of dysarthria?
A: As follows: Q: What is the difference between dysphasia and dys-
• Scanning (cerebellar lesion). anhria?
A: Dysphasia is a disorder of language; but dysarthria
• Spastic (pseudobulbar palsy).
is a disorder of speech.
• Paralytic (bulbar palsy).
• Extrapyramidal (slow, slurred, monotonous, Q: Which cerebral hemisphere is responsible for lan-
husky, dysphonia or aphonia). guage function?
• Variegated (hypothyroidism, multiple ulcer or A: The dominant hemisphere. Left hemisphere is dom-
thrush in the mouth, amyloidosis due to large inant in 97% right-handed person and 50-60%
tongue and temporomandibular arthritis). left-handed person.

Dysphasia
Usual instructions are:
• Or, the patient is unable to talk, but can talk
• Talk with the patient. Or, ask some questions.
a few words after sometime. There is lack of
fluency, has difficulty in finding some words, but
Presentation of a Case comprehension is good.
(Motor Dysphasia): Case No.1
• The patient can understand, but is unable to talk
or answer. My diagnosis is motor (expressive) dysphasia (also
called Broca dysphasia).
8 • NEUROLOGY __

Q: What else do you want to examine?


Presentation of a Case
A: Hemiplegia, more likely right sided (ask the patient
(Nominal Dysphasia): Case No.3
to raise the hands, legs). (In a right-handed person,
left hemisphere is dominant; and it is also dominant • The patient has difficulty in naming the object;
in 50% of left-handed persons.) but comprehension is good and the speech
production is relatively good.
Q: Where is the site of lesion in motor dysphasia?
A: Broca area (posterior part of inferior frontal gyrus)
in dominant hemisphere (along the middle cerebral My diagnosis is nominal dysphasia.
artery or of its frontal branch).
Q: What is nominal dysphasia?
Q: What are the causes? How is the prognosis? A: It is a type of motor dysphasia in which there is
A: Cerebrovascular accident (such as cerebral infarction difficulty in naming the familiar object (although
or haemorrhage). Prognosis is good. naming difficulty may occur in all dysphasia), but
other aspects of speech are normal. The patient
usually talks less, answer is either 'yes' or 'no' and
Presentation of a Case comprehension is good.
(Sensory Dysphasia): Case No.2
Q: Where is the site of lesion in nominal dysphasia?
• The patient speaks fluently, but cannot A: Dominant posterior part of the temporoparietal
understand the spoken (auditor dysphasia) or area. It is rare in its pure form. Sometimes, the
written word (alexia). patient uses long sentences to overcome the failure
• Speech is unintelligible, which is incorrect to find the correct word (circumlocution).
word (hand for foot), inappropriate words
(paraphasia, incorrect syllable in word (e.g. tooth
brush as tooth smooth), new words (neologism)
Presentation of a Case rl---------,
and nonexisting words (Jargon dysphasia). (Global Dysphasia): Case No.4

• The patient has marked disturbance in


comprehension and in expression.
My diagnosis is sensory (receptive) dysphasia.

Q: What is the cause?


A: Cerebrovascular accident (such as infarction or My diagnosis is both motor and sensory (global
haemorrhage). It may be also due to space-occupying dysphasia).
lesion. Q: Where is the site of lesion in global dysphasia?
How is the prognosis?
Q: Where is the site of lesion?
A: Extension of infarction in the left cerebral hemi-
A: Posterior part of dominant superior temporal gyrus
sphere along the territory of left middle cerebral
(called Wernicke area).
artery. Patient has difficulty in reading, writing and
Q: What else do you like to examine? also there is:
A: As follows: • Right-sided hemiplegia.
• Homonymous hemianopia. • Right homonymous hemianopia.
• Sensory (impaired or diminished on right side of • Marked intellectual deterioration.
the body). Prognosis is poor.
• Dyslexia (d ifficulty in read ing).
• Dysgraphia (difficulty in writing). Q: What is conductive dysphasia?

• Dyscalculia (difficulty in calculation). A: In this type of dysphasia, the patient follows the
command, but repeating the statement and naming
Q: What happens in repetitive speech? the object poorly is present.
A: Severely impaired (it is better in expressive type).
Q: Where is the site of lesion?
Q: What is the prognosis? A: Arcuate fibre or the fibres communicating between
A: Recovery is usually good. Wernicke and Broca area.
__ SHORT CASES IN CLINICAL MEDICINE

Dysarthria
Usual instructions are:
Presentation of a Case
• Talk with the patient.
(Pseudobulbar Palsy): Case No.3
• The speech is indistinct, slurred and high pitched
Presentation of a Case
(the so-called Donald Duck or Hot Potato
(Dysarthria): Case No. 1 dysarthria due to tight immobile tongue).
• The tongue is spastic (small and tightened),
• The patient has slow, slurred, scanning speech or
unable to protrude.
explosive in nature.
• There is no wasting and no fasciculation.
• Palatal movement is absent.
My diagnosis is cerebellar speech. • Gag reflex is present.
• Jaw jerk is exaggerated.
Q: What else do you want to see? • The patient is emotionally labile (laughing and
A: Cerebellar signs (see cerebellar lesion, page 338.) crying).

Presentation of a Case My diagnosis is pseudobulbar palsy.


(Bulbar Palsy): Case No.2
Q: Where is the site of lesion in pseudobulbar palsy?
• The speech is nasal, indistinct, slurred. There is
A: Bilateral liMN lesion (supranuclear) involving the
(may be) dribbling of saliva.
pyramidal tract (supranuclear lesion of lower cra-
nial nerves: IX, X, xi. XII).

My diagnosis is bulbar palsy.


Q: What else do you want to examine?
N.B. For diagnosis of bulbar palsy, remember the 3 A: As follows:
'Ds' (Dysarthria, Dysphonia and Dysphagia). • Sensory change (no abnormality).
• Signs of liMN lesion (bilateral generalized
Q: What else do you want to see? What is the site of
spasticity) .
lesion in bulbar palsy?
A: As follows: • Dysphagia.
• Palate: Absent movement (ask the patient to say • Aphonia or dysphonia (in severe cases).
'aah' and see the palate).
• Tongue: Wasting, wrinkled and fasciculation. Q: What are the causes of pseudobulbar palsy?
• Gag reflex: Absent. A: As follows:
• The patient has nasal regurgitation. • Bilateral repeated eVA involving internal capsule
(multi-infarct dementia).
Site of lesion: Nucleus of lower cranial nerves in me-
dulla (IX,X, XI and XII). Lesion is bilateral and LMN • Demyelinating disease (MS).
type. • Motor neuron disease.
• Others: Brainstern tumour, trauma.
Q: What are the causes of bulbar palsy?
A: As follows: Q: Why pseudobulbar palsy is seen in bilateral lesions
• Motor neuron disease. only?
• Cuillain-Barre syndrome. A: Because most cranial nerve nuclei receive bilateral
• Syringobulbia. innervations from corticobulbar tract.
• Brainstem infarction.
• Poliomyelitis. Q: What are the differences between bulbar and
• Neurosyphilis. pseudobulbar palsy?
• Neurosarcoid. A: As follows:
8 • NEUROLOGY __

Criteria Bulbar palsy Pseudobulbar palsy Nasal Present Absent

Type of lesion LMN UMN regurgitation

Bilateral, internal Tongue Wasted, Small, stiff or spastic


Site of lesion Medulla
capsule fasciculation

Slow, slurred and Jaw jerk Absent Brisk


Speech Nasal
indistinct Emotion Normal Labile
z
go
0-
Examination of Hands (Wasting of Small Muscles of Hands) ~

Usual instructions are: • Neck (for cervical spondylosis, cervical rib and
• Look at the hands. What is the diagnosis? supraclavicular bruit).
• Examine the hands or examine the upper limb. • Dissociated sensory loss for syringomyelia (for
details see page 354).
By looking, the obvious diagnoses may be possible (for • Thickening of nerve (at elbow):"Leprosy.
details see 'Examination of Hands' in Chapter 1). In this • Trophic change, ulcer, gangrene and burn.
chapter, wasting of small muscles of hands, related to
neurological diseases, are described. Q: What are the causes of wasting of small muscles of
hand?
A: As follows:
Presentation of a Case :1--------,
• Motor neuron disease.
(Hand): Case No. 1 • Charcot-Marie-Tooth disease.
• There is generalized wasting of the small muscles • Syringomyelia.
of hands involving the thenar, hypothenar and • Cervical spondylosis.
interossei muscles with dorsal guttering and claw • Cervical cord compression.
hand [flexion of interphalangeal (IP) joints and • Cervical rib.
extension of MCP joints]. • Pancoast tumour.
• Peripheral nerve lesion (polyneuropathy,
• No sensory abnormality.
combined ulnar and median nerve Jesion).
• Leprosy.
My diagnosis is motor neuron disease (for details, see
• RA.
inMNO). • Myopathy (dystrophia myotonica).
Q: What are the causes of wasting in one hand?
Presentation of
A: As follows:
Case No.2 • Cervical cord compression or cervical rib.
• As in Case no. 1 plus sensory abnormality • Brachial plexus lesion (trauma and tumour).
(mention whether median, ulnar or both). • Pancoast tumour.
• Old poliomyelitis.
• Cerebral palsy.
My differential diagnoses are (wasting of small mus-
• Leprosy.
cles of hand with sensory loss):
• Peripheral neuropathy due to any cause.
• Cervical spondylosis.
• Cervical lib.
• Cervical cord compression (neurofibroma and
meningioma).
• Leprosy.
Syringomyelia (dissociated sensory loss and trophic
changes in hand).

Q: What else do you want to examine?


A: As follows: Wasting of thenar and hypothenar
__ SHORT CASES IN CLINICAL MEDICINE

Claw hand-like deformity (pseudoclaw hand) may


occur in:
• Dupuytren contracture.
• Volkmann ischaemic contracture.
• Postburn contracture.
• Diabetic cheiroarthropathy.

Wasting of dorsal interossei guttering

Q: What is claw hand? What are the causes?


A: It is a deformity of hand characterized by flexion of
all interphalangeal (H') joints and hyperextension
of MCP joints. It is due to weakness of the lumbri-
cals and interossei, and unopposed action of long
extensors of finger.
Only LMN lesions of C8 and Tl nerves, C8 and Tl
nerve root lesion, and brachial plexus lesion will pro- Claw hand (dorsal surface)
duce localized wasting of lumbricals and interossei
causing claw hand. A combined ulnar and median
nerve lesion will produce true claw hand. Ulnar claw
hand involves fourth and fifth fingers because radial
lumbricals are supplied by median nerve.

Causes of claw hand:


• Combined ulnar and median nerve lesion:
Trauma, leprosy.
• Brachial plexus lesion (C8 and Tl): Cervical rib,
thoracic inlet syndrome, Klumpke paralysis.
• Neurological disease: MND, Charcot-Marie-
Tooth disease, syringomyelia, intramedullary
tumour and polio. Claw hand (palmar surface)

Syringomyelia
Usual instructions are:
• Joints: Hyperextension of MCP and flexion of
• Examine the hands or perform the neurological
interphalangeal joints giving rise to claw-hand
examination of the upper limbs (for examination
appearance.
of hands, see in Chapter 1).
• Muscle tone and muscle power are diminished.
• Biceps, triceps and supinator reflexes are
Presentation of a Case diminished.
• There is dissociated sensory loss (loss of pain
• There is a scar mark or burn or painless ulcer on
the index finger of right side. and temperature, but intact light touch, vibration
and position sense) in the arms, shoulder and
• There is wasting of the small muscles of the
neck (classically in 'cape' distribution).
hands and forearm; also dorsal guttering of the
hands is present. Few fasciculations are present
(mention, if any).
My diagnosis is syringomyelia.
8 • NEUROLOGY __

Q: What are the causes of syringomyelia?


A: As follows:
• Developmental anomaly (at the foramen
magnum).
• Obstructions of fourth ventricle by congenital
defect of the base of the skull or cervical spine
(as in Arnold-Chiari malformation).
• There may be arachnoiditis in the region of
foramina of Magendie and foramina of Luschka.

Q: How the patient usually presents?


A: Presents in third or fourth decade, rarely in early
age. The features are:
Bilateral claw hand and wasting of small muscles • Wasting of muscles of hands, forearms, shoulder
of hands in syringomyelia girdles.
• Loss of pain and temperature sensation. Patient
may complain of painless burn.
• Difficulty in walking.

Q: What are the physical findings in syringomyelia?


A: Triad of:
• Dissociated sensory loss in neck, shoulder and
arm.
• LMN lesion signs in the upper limb (muscle
atrophy and loss of reflex).
• UMN lesion signs in lower limbs.

Hand wasting in syringomyelia

Q: What else do you want to examine?


A: As follows:
• Eyes-for Homer syndrome (syringobulbia).
• Kyphoscoliosis, spina bifida and pes cavus.
• Charcot joints of shoulder and elbow.
Examination of lower limbs (may be UMN lesion
signs in lower limb, spastic paraplegia).

Q: What is syringomyelia?
A: It is a developmental anomaly in which there are
cavities filled with fluid surrounded by glial tissue Hand in syringomyelia
near the centre of spinal cord, mostly originating at
C8 and Tl segment, but may occur anywhere in the Q: What are the investigations done?
spinal cord. A: As follows:
• X-ray of the neck (congenital anomaly of foramen
Expanding cavity may disrupt:
and widening of cervical canal).
• Anterior horn cells of spinal cord. • MRI (investigation of choice) or CT scan.
• Lateral spinothalamic tract.
• Corticospinal tract. Q: What are the CSF findings?
May extend upwards to involve the brainstem A: It may show high protein, which is higher if there is
(syringobulbia). CSF blockage.
_ SHORT CASES IN CLINICAL MEDICINE

Q: What are the features of syringobulbia?


A: As follows:
• Dissociated sensory loss in the face.
• Horner syndrome.
• Palatal palsy.
• Dysarthria.
• Nystagmus.
• Cranial nerve involvement (V, Vll, IX and X).

Q: What is dissociated sensory loss? What are the


causes?
A: Loss of pain and temperature, but intact light touch,
Charcot joint in syringomyelia
vibration and position sense.

Q: How to treat syringomyelia? Causes of dissociated sensory loss:


A: As follows: 1. Lesion at the centre of spinal cord (sensory loss is
• Supportive: Regular activity, physiotherapy; avoid always bilateral and segmental). The causes are:
burn, trauma or hot water. • Syringomyelia (the commonest).
• Surgical decompression may be necessary. • Intramedullary neoplasm of the spinal cord.
Q: What surgeries are possible? • Haematomyelia.
A: As follows: 2. Hemisection of spinal cord (Brown-Sequard
• Cervical decompression: In which suboccipital syndrome).
craniectomy, CI-C3IaminectomyanddurapJasty. 3. Lesion at the anterior half of the spinal cord:
This is done in Arnold-Chiari malformation. Anterior spinal artery occlusion (LMN lesion at
• Dorsolateral myelotomy: In which syrinx is the site, and bilateral UMN lesion and lateral
drained into the subarachnoid space, usually spinothalamic lesion below).
done following decompression. 4. Lateral medullary syndrome: Thrombosis of
• Shunt: Such as syringoperitoneal shunt, syrin- posterior inferior cerebellar artery (PICA).
gosubarachnoid shunt ventriculoperitoneal shunt. Syringobulbia also produces same type of lesion.

Myotonic Dystrophy
Usual instructions are: • Look at the face of the patient (signs in face-see
• Examine the hands. What else do you want to see? below).
• Look at the patient. What is your diagnosis? What • Percussion may be done on the tongue (dimples or
else do you want to see? depressions are seen).

Proceed as follows (first instruction):


• Look at the hands: front and back (apparently no Presentation of a Case
abnormality) . (Hands): Case No.1
• Tell the patient 'Close and open your hands • The patient has myotonia, as suggested by the
repeatedly as quickly as possible.' The patient is inability to open the hands after dosing and also
unable to open the dosed hands (myotonia). to relax the hands after handshaking.
• Shake hands with the patient (unable to relax the • There are also dimples on the percussion of
hands-grip myotonia). thenar eminence.
• Percuss on thenar eminence with your fingers.
There are dimples or depressions that fill up slowly
(percussion myotonia). My diagnosis is myotonia.
8 • NEUROLOGY __

Presentation of a Case :~-------,


(by Looking at Face and Other Parts):
Case No.2

1. In face:
• Frontal baldness (patient may be wearing
wig).
• Long, lean, triangular, sad and expressionless
face.
• Wasting of temporal is and masseter.
2. Eyes:
• Partial ptosis (usually bilateral, may be
Myotonia
unilateral) with smooth forehead.
• Cataract (stellate cataract). May be posterior
subcapsular fine deposit.
• Difficulty in opening the eyes after firm
closure.
3. Neck: Wasting of sternomastoid and shoulder
girdle muscles. There is weakness of flexion
and normal extension.
4. Others:
• Wasting of distal muscles of arms (forearm
wasting first) and legs.
• Testes (atrophy).
• Cynaecomastia.

My diagnosis is myotonic dystrophy.

Q: What are the other features of myotonia


dystrophica?
A: As follows:
• Inherited as autosomal dominant.
Myotonia dystrophica • Males are affected more than females.
• Age of onset may be any.
Q: What is myotonia? • Diabetes mellitus and impaired glucose tolerance
A: It is the continued contraction of the muscles after (IGT) may occur.
cessation of voluntary contraction. • Intellect and personality may have mild
deterioration.
Q: What is myotonia congenital • Small pituitary fossa and hypogonadism may
A: It is an inherited disorder (autosomal dominant) that occur.
is characterized by failure to relax the muscle after • Low serum IgG levels.
forceful contraction. Present at birth with feeding
difficulty, inability to open the eyes and a pecu- Tolerate anaesthesia poorly.
liar cry. It is a mild disease that improves with age. Heart (cardiomyopathy, valvular heart disease, mitral
Other features of myotonia dystrophica are absent. valve prolapsed, and arrhythmia or conduction
There may be diffuse muscular hypertrophy due to defect).
continuous involuntary isometric contraction.
Q: What do you mean by myopathic facies?
Prognosis: Normal life expectancy.
A: It is characterized by expressionless, sad-like facies
Treatment: Procainamide or quinidine may be with frontal baldness and ptosis. Sometimes looks
helpful. like sleepy appearance.
_ SHORT CASES IN CLINICAL MEDICINE

Q: What is the gait in myotonic dystrophy? Q: How to treat myotonia dystrophica?


A: There is high steppage gait. Foot drop is present. A: Only symptomatic. No specific treatment.
• Myotonia may be treated by phenytoin
Q: What are the types of myotonic dystrophy? (procainamide or quinidine may be used, but
A: They are of two types: may worsen cardiac conduction).
• Type 1: Classical type associated with distal • Genetic counselling.
muscular wasting and weakness.
5ri
o Q: What are the causes of myotonia?
-0 • Type 2: Similar features, but there is proximal
A: As follows:
g wasting and weakness.
• Myotonia dystrophica.
Z
Q: What are the EMG findings? • Myotonia congenita.
A: High-frequency activity that varies repeatedly to • Others: Hyperkalaemic periodic paralysis,
cause a characteristic sound on loud speaker (waxing myxoedema (Hoffman syndrome), hereditary
and waning of potentials called Dive-Bomber paramyotonia (autosomal dominant), cold-
effect). induced myotonia and drug (clofibrate).

Myopathy (Muscular Dystrophy)

Usual instructions are:


• Examine the upper limbs or lower limbs.
• Look at the face. What are your findings? Examine
the relevants.

Presentation of a Case 11---------.


(Limb-Girdle Myopathy): Case No.1

• There is wasting of muscles of both upper limbs


with muscular weakness, mainly involving the
proximal (proximal myopathy).
• Muscle tone is normal. Reflexes are normal or
slightly reduced.
• No sensory abnormality.
limb-girdle myopathy (back)

My diagnosis is limb-girdle myopathy.


Q: What else do you want to see?
A: I want to examine the lower limbs (findings like
upper limbs).

Q: What is limb-girdle myopathy?


A: It is a type of muscular dystrophy characterized by the
involvement of shoulder and pelvic girdle muscles.
• Type 1 (10%): Autosomal dominant; slower and
later onset.
• Type 2 (90%): Autosomal recessive; occurs in
childhood or early adulthood.
• Age of onset is 10-30 years; male and female are
equally affected.
• May involve cardiac muscle (may cause
conduction abnormality or heart failure).
• Calf pseudohypertrophy sometimes occurs.
Limb-girdle myopathy (front) • Face and hands are spared.
8 • NEUROLOGY __

• Intelligence is normal. • Onset is 10-40 years of age.


• Muscle enzymes are normal or slightly elevated. • Course is variable, but usually relatively benign .
• Prognosis is poor; chair-bound at 20-25 years of • There is wasting of muscles of face, neck and
age (10-20 years after the onset of disease). shoulder girdle (lower trapezzi, pectoralis, biceps,
triceps). Hypertrophy of the deltoid.
• Winging of scapula (due to the involvement of
Presentation of a Case serratus anterior muscle).
(Fascio-Scapulo-Humeral Dystrophy): • Pain in shoulder girdle is common.
Case No.2 • Face looks dull, expressionless; lips open and
slack; inability to whistle and puff the cheek.
• There is wasting of muscles of face, neck and
• Eyes: Bilateral partial ptosis.
shoulder girdle. • There may be distal lower-limb weakness.
• Face looks dull, expressionless; lips open and
• Lower abdominal muscles are specifically
slack; inability to whistle and puff the cheek. involved with sparing of upper abdominal
• Eyes: Bilateral partial ptosis. muscles (Beever sign-ask the patient to lie flat;"
• Winging of scapula (due to the involvement of then ask him to raise the head. This results in
serratus anterior muscle). contraction of the upper abdominal muscles,
• Pectoralis and trapezius are also wasted. with displacement of the umbilicus upwards).
• Lower limb muscles are affected in 50% cases,
which usually affects at later age. Then foot drop
My diagnosis is fascio-scapulo-humeral dystrophy.
occurs.
• Fundoscopy: Retinal telangiectasia in 60% cases.
• Intelligence is normal.
• Prognosis: Normal life span and slowly progressive.
• Muscle enzymes are usually normal or slightly
elevated.
• Inflammatory and perivascular inflammation
may be seen in muscle biopsy. Also, retinal
microvascular abnormality may be fouod.

Presentation of a Case rl--------.,

(Becker Muscular
Fascio-scapulo-humeral dystrophy
Dystrophy): Case No.3

(Male young)
• There is pseudohypertrophy of calf muscles (also
deltoid, muscles of buttock and infraspinatus).
• Proximal myopathy is present, mainly lower
limb (face is normal).

My diagnosis is Becker muscular dystrophy.


Q: What is Becker muscular dystrophy?
A: It is inherited as X-linked disorder, only males are
affected and features are same as Duchenne type
Pseudohypertrophy of calf muscles with the exception of:
• Onset is late (5-25 years).
Q: What is fascio-scapulo-humeral dystrophy? What • Less severe, less rapid progression and less
are the features? cardiomyopathy. Mental retardation and
A: It is a type of muscular dystrophy that is inherited as kyphoscoliosis are uncommon. Respiratory
an autosomal dominant, which is characterized by involvement is a late feature.
the involvement of muscles of the face and shoulder • Chair bound at about 25 years after the onset.
girdle. • Survival up to fourth to fifth decade.
_ SHORT CASES IN CLINICAL MEDICINE

Causes of pseudohypertrophy of calf muscles: • Limb-girdle myopathy.


• Duchenne muscular dystrophy. • Fascio-scapulo-humeral dystrophy.
• Becker muscular dystrophy. • Myotonia dystrophica.
• Myotonia congenita.
Q: What is Duchenne muscular dystrophy? What are
• Others are oculopharyngeal or ocular myop-
the features? athy, and congenital muscular dystrophy.
A: It is inherited as X-linked recessive disorder (30% 2. Congenital myopathy (rare):
spontaneous mutation). Duchenne gene is on the
• Central core.
short arm of X-chromosome, Xp21; and its product • Nemaline myopathy.
called dystrophin is absent (diagnosed by western • Myotubular myopathy.
blot anaLysisof muscle biopsy). 3. Secondary myopathy (endocrine disease and
• Affects only male, age of onset is 3-4 years. drugs).
• The child presents with difficulty in walking or
getting up from sitting or lying position. There is Q: What are the differences between neuropathy and
history of frequent fall and delayed motor activity myopathy?
(e.g. walking). A: As follows:
• Gower sign is positive (while the child gets up from • Myopathy usually involves proximal muscles
lying position, he uses the hands to climb up). (except myotonia dystrophica, which involves
• There is pseudohypertrophy in early stage involv- distal muscles).
ing calf and deltoid muscles. Later there is • Neuropathy usually involves distal muscles
weakness; it first involves the proximal muscles. (except diabetic amyotrophy), GBS, amyloidosis,
• Gait: Waddling (duck like). Lyme disease.
• Other features include dilated cardiomyopathy,
kyphoscoliosis and mental retardation. There is Q: What investigations should be done in myopathy?
early respiratory involvement. A: As follows:
• Prognosis is poor, chair bound by the age of • Creatine phosphokinase (CPK) is very high, up
10 years and few survive up to 20 years. to 40-fold in Duchenne type (in other types,
• Causes of death are dilated cardiomyopathy and slightly raised).
respiratory failure or inanition. • EMG shows short duration, low amplitude,
spiky polyphasic action potential (spontaneous
Q: What is myopathy?
fibrillation is also seen occasionally), reduction
A: It means disease of the skeletal muscle (voluntary)
in motor unit amplitude and duration with
muscle.
normal number of units activated during effort.
Q: What is muscular dystrophy? What are the types of • ECG (cardiomyopathy and dysrhythmia),
muscular dystrophy? echocardiography, blood sugar.
A: It is a group of hereditary muscular disorder char- • Muscle biopsy (shows variation of muscle fibre
acterized by progressive degeneration of groups of size, degenerative changes, regeneration and
muscles without the involvement of nervous system replacement by fat. On immunological staining,
with absence or reduced dystrophin. The types are: there is absence of dyst.rophin. This is commonly
1. Hereditary muscular dystrophy: found in Duchenne muscular dystrophy).
• Duchenne type (pseudohypertrophic). • Lactic acid (to exclude mitochondrial myopathy).
• Becker muscular dystrophy. • Molecular genetic testing.

Carpal Tunnel Syndrome

Usual instructions are: 3. Motor functions: Examine the thumb (weakness of


• Examine the hands or look at the hands ..What are abduction, flexion and opposition of thumb) and
your findings? also examine for interossei (Chapter 1).
• Pen touching test (to assess for the weakness of
Proceed as follows: abductor pollicis brevis): Ask the patient to lay
1. Look at palm (wasting of thenar muscles). the hand flat with palm upward. Ask to abduct
2. Perform sensory test (loss of sensation along lateral the thumb vertically to touch the pen held
three-and-half fingers). above it. It is impossible, if median nerve palsy.
8 • NEUROLOGY __

4. Elicit the following signs: Q: What else do you want to see?


A: 1want to see the evidence of primary cause
• Tinel sign: Percussion over the flexor aspect of
(see below).
the wrist (flexor retinaculum) or tap the median
nerve in forearm, the patient may experience Q: What are the nerves involved in entrapment
paraesthesia along the distribution of the nerve. neuropathy?
• Phalen sign: Flexion or extension of the wrist for A: As follows:
1 min produces paraesthesia along the distri- • Median nerve (at wrist).
bution of nerve (lateral three-and-half fingers). • Ulnar nerve (at elbow or at wrist in Guyon
• Tourniquet test: Raise BP above systolic for canal-bounded proximally by pisiform bone
2 min (produces paraesthesia). and distally by the hook of the hammet).
• Durkan test: Direct pressure over carpal tunnel • Radial nerve (at spiral groove of humerus
for 30 s may produce paraesthesia. following fracture).
• Closed fist test: Flexion of the fingers into a • Meralgia paraestherica (at inguinal ligament).
dosed fist for 60 s produce paraesthesia in the • Common peroneal nerve or lateral popliteal
median nerve distribution. nerve (at the neck of fibula).
Tarsal tunnel syndrome (at the flexor retinaculum at
Presentation of a Case ankle joint, there is compression of posterior tibiaJ
nerve).
• There is wasting of thenar muscles.
N.B. ln entrapment neuropathy, there is demyeli-
• Weakness of abduction, flexion and opposition
nation, may be axonal degeneration.
of thumb, and weakness oflateral two lumbricals.
• Loss of sensation along the radial three-and-half Q: What is carpal tunnel syndrome? What are the
fingers. causes?
A: It is a type of entrapment neuropathy due to com-
My diagnosis is carpal tunnel syndrome. pression of median nerve under flexor retinaculum
of wrist causing wasting, tingling, numbness and
pain along the distribution of nerve. It is more com-
mon in female, less in male.
Causes of carpal tunnel syndrome
Wasting 01
thenar muscles • Pregnancy (due to fluid retention, usuaJly in the
third trimester).
• Obesity.
• RA.
• Acromegaly.
• Myxoedema.
• CRF on long-term dialysis (due to deposition of
~2-mjcroglobulin-an amyloid).
• Tuberculous tenosynovit.is.
• Primary amyloidosis.
• Tophaceous gout.
• Drug (oral contraceptive pill).
• Osteoarthrosis of carpus (related to old fracture).
• Idiopathic (common in female, middle-aged and
obese. May occur in male with unaccustomed
hand use, e.g. house painting).

Q: What is the usual presentation of carpal tunnel


syndrome?
A: Nocturnal pain, numbness, and paraesthesia in
palm and fingers often occurs at night, awaken-
ing the patient from sleep. Pain may be referred to
Carpal tunnel syndrome whole arm and shoulder.
_ SHORT CASES IN CLINICAL MEDICINE

Q: How to confirm the diagnosis? It is usually self-limiting. Occasionally, may be treated


A: By nerve conduction study. with corticosteroid and local anaesthetic injection at
the anterior superior iliac spine. Causes are:
Q: How to treat?
• Obesity.
A: As follows:
• Pregnancy.
• Treatment of primary cause.
• OM.
&l
o • Splint in wrist, especially at night; local steroid
<5 • Idiopathic.
injection (in intracarpal tunnel) and diuretic may
a
Q) help. • May be tight jeans.
z In severe cases: Surgical decompression of carpal
tunnel may be required.
Muscles supplied by median nerve,
Q: What is meralgia paraesthetica? mnemonic: LOAF
A: It is a type of entrapment neuropathy due to the
• L: Lateral two lumbricals.

_j
compression of lateral cutaneous nerve of thigh on
• 0: Opponens pollicis.
leaving the pelvis, just medial to the anterior superior
• A: Abductor pollicis brevis.
iliac spine. There is pain and paraesthesia over the
• F: Flexor pollicis brevis.
upper and outer thigh, with reduction of sensation.

Ulnar Nerve Palsy


Usual instructions are:
• There is also wasting of the medial side of forearm
• Examine the hands or look at the hands. What are (due to involvement of flexor carpi ulnaris and
your findings? What else do you want to do? medial half of flexor digitorum profundus).
• Froment sign is positive.
Proceed as follows:
1. Look at palmar and dorsal aspect of hands. If
generalized wasting, perform the neurological My diagnosis is ulnar nerve lesion or palsy.
examination of hands (Chapter 1).
Q: What are the causes of ulnar nerve lesion?
2. To find out causes: A: As follows:
• Evidence of fracture or dislocation of the elbow • Fracture of ulna or dislocation of elbow.
(injury, any scar or deformity). • Injury at the wrist or palm.
• Evidence of osteoarthrosis at elbow. • Mononeuritis multiplex due to any cause (OM,
• Injury at the wrist or palm. PAN, RA,SLE, amyloidosis and leprosy).
• Thickened nerve.
• Osteoarthrosis of elbow.
• Any causes of mononeuritis multiplex.
• Occupation: With constant leaning of elbows
(clerk) or constant flexion or extension at elbow
(carpenter, painter, decorator) and wrist (screw
Presentation of a Case driver, drills).
• There is generalized wasting of the small muscles
of hands (except thenar) with dorsal guttering. Q: What are the motor and sensory supplies of ulnar
• Ulnar claw hand (extension of MCP and flexion nerve (C8 and 1'1)?
of IP joint of fifth and fourth fingers). A: As follows:
• Loss of sensation along fifth and half of the • Motor supply to small muscles of hand (except
fourth finger. LOAF), flexor carpi ulnaris and medial half of
• There is weakness of abduction and adduction flexor digitorum profundus.
of fingers. • Sensory supply to the fifth finger and half of the
fourth finger.
Radial Nerve Palsy

Usual instructions are:


• Examine the hands or look at the hands. What are
your findings? What else do you want to do?
Proceed as follows:
1. Ask the patient:
• To raise both arms in front (obvious wrist drop).
• To extend the wrist (weakness of wrist
extension).
• To extend the elbow against resistance; weakness
of elbow extension (triceps involvement). Wristdrop
2. Test for brachioradialis: Ask to flex the elbow with
forearm halfway between pronation and supination Q: What is Saturday night palsy?
(failure to flex; brachioradialis does not spring up). A: In this disorder, the patient is heavily sedated with
3. Pronation and supination (impaired). alcohol, sleeps with the arms hanging over the
4. The patient is unable to straighten the fingers. back of chair and radial nerve is compressed at the
However, if the wrist is passively extended, the middle-third of the humerus causing paralysis of
patient is able to straighten fingers at IP joints the nerve. Brachioradialis, supinator and extensors
(due to action of interossei and lumbricals), but is of forearm are involved. Usually, complete recovery
unable to extend MCP joints. occurs within weeks.
5. Grasp is weak, but grip improves if wrist is
N.B. Radial nerve is vulnerable to be involved at three
extended.
sites:
6. Check sensation over the anatomical snuffbox for
dorsal aspect of thumb (there is loss of sensation). • Axilla (incorrect use of crutch).
7. Triceps reflex (absent). • Spiral groove of humerus (mid-shaft fracture and
saturday night palsy).
Q: What are the causes of radial nerve palsy?
• Damage to the posterior interosseous nerve
A: According to the site:
at proximal forearm, where it penetrates the
• Axilla: Trauma, radiation, compression by
supinator muscle.
improper use of crutch, axillary growth.
• Sensory branch may be compressed at the wrist.
• Spiral groove or mid-shaft of the humerus:
Trauma, compression (e.g. saturday night palsy). Anatomy of the radial nerve: It is the termination
• Proximal forearm: Trauma, subluxation of the of the posterior cord of brachial plexus, derived from
radius, repetitive forearm supination. the CS-8 and Tl spinal nerve. In elbow, it gives two
• Wrist: Trauma, compression by tight bracelet or branches-superficial radial (entirely sensory) and
handcuff. posterior interosseous (entirely muscular). It gives
• Chronic lead poisoning. motor supply to brachioradialis, supinator and
• Mononeuritis multiplex (due to any cause). extensors of the forearm.

Cranial Nerves
Usual instructions are: • Now examine the individual cranial nerve as follows:
• Examine the cranial nerves. First Cranial Nerve
(Remember, the examiner may stop you at any
point and you must be ready to answer.) • Examine the nose quickly with a torch light to see
any deviated nasal septum (DNS), polyp.
Proceed as follows: • Ask the patient, 'Do you have any difficulty in your
• Ask the patient to sit at the edge of bed, face-to-face. sense of smell'?
See any obvious finding (ptosis, squint, asymmetry (Ideally use a perfume, put in each nostril and ask,
of face and dribbl ing of saliva). 'Do you get the smelJ'?)
__ SHORT CASES IN CLINICAL MEDICINE

Second Cranial Nerve IIIrd, IVth and Vlth Cranial Nerves


• Acuity of vision. • Look for any obvious ptosis and squint. Next
• Colour vision. examine eye movement.
• Field of vision. • Ask the patient 'Look at my finger. Follow it with
• Fundoscopy (should be done at the end). your eyes with head fixed'.
~ • See movements in horizontal and vertical
-g
o
o Ask the patient whether he uses any glass. If so, ask to
wear it and do the examination. (Remember, each eye
directions like the pattern 'H'.
• See nystagmus. At extreme gaze, ask, 'Do you see
Z should be examined separately.) one or two fingers (diplopia)'?
Acuity of vision (examine both distant and near • Next examine pupil: Size, shape, light and
vision): Better use a Snellen chart (a mini Snellen chart accommodation reflex (see in Chapter 10,
'Examination of the Eye').
may be used). If not available, proceed as follows:
• Distant vision: Ask the patient, 'Look at the wall If diplopia is present, further test is necessary as
clock. What is the time now' ? "'Look at the window. follows:
How many rods are there in the window' ? • Ask to describe false image (it is pale, less distinct
• Near vision: Ask, 'Read the newspaper or any small and more peripheral than real one). Ask, 'Whether
object'. two images lie side-by-side or one above the other' .
• If images are side-by-side, it indicates medial or
Colour vision (ideally it should be done with Ishihara lateral rectus palsy. If images lie on one above the
chart): other, then either oblique muscles or superior or
• Show different colours to the patient and ask, inferior recti are involved.
'What colour is it'? • To decide which pair of muscles are responsible,
ask, 'In which direction, there is maximum image
Field of vision (Sit opposite to the patient, 1-m apart separation ?'" (Separation is greatest in the direction
at a same plane. Test each eye separately): in which the weak muscle has its purest action.)
• Now at the point of maximum separation, cover
• To examine the right eye, ask the patient 'Cover
one eye and find which image disappears. Loss
your left eye with left hand gently, look steadily at
of lateral image indicates that covered eye is
my left eye' (you should cover your right eye). No
responsible.
one should move the eye and should look each
• If diplopia persists after covering one eye, it may be
other's tip of the nose.
due to astigmatism, dislocated lens or conversion
• Hold your index finger midway and from disorder.
periphery, bring towards the centre until you see
it. Ask the patient, 'Do you see my finger? Tell me
when you see it.' Vth Cranial Nerve
• If the patient fails to see, continue to bring the
Perform both motor and sensory tests:
finger and ask when the patient can see.
Motor test (see any wasting of masseter and temporalis
• Tn this way, see in horizontal, upper and lower muscles):
quadrant (temporal field).
• Ask the patient, 'Clench your teeth tightly.' Palpate
• Then, see nasal field in the same way.
the muscles.
• Change your hand and repeat in other eye in the
• Test for pterygoids: Ask the patient, 'Open your
same manner. mouth and stop me from closing it'. (If paralysis is
Test of central scotoma: on one side, muscle of that side is less prominent
and jaw deviates towards the side of lesion.)
• Use a red-headed pin; move it from temporal • Test for jaw jerk.
side to nasal side in the midway. Ask the patient,
'Do you see it? Tell me, when it disappears'. Sensory test: Test along the three divisions of nerve on
each side using cotton and pin with eye dosed.
Finally perform fundoscopy (see in Chapter 10, Test corneal reflex with wisp of cotton. Touch the
'Examination of the Eye') cornea (not conjunctiva); see if there is reflex blinking.
8 • NEUROLOGY __

N.B. Remember the following points: Cochlear division:


• Corneal reflex: Afferent (sensory) passes through • Look at the external ear and meatus (wax and rash).
ophthalmic division of Vth nerve and efferent • Rub your finger or keep watch near ear, ask the
(motor) through VTIthnerve. patient, 'Can you hear it'?
• If blinking on contralateral eye, but absent in • 'Rinne test' and 'Weber test' may be necessary
tested eye, it indicates VIIth nerve lesion in (normally, air conduction is better than bone
ipsilateral eye (eye tested). 0 nly one or three conduction).
divisions ofVth nerve may be lost:
o If all sensory are lost, then lesion in ganglia or IXth and Xth Cranial Nerves
sensory root (acoustic neuroma).
o [f only one branch of sensory is lost, then • While talking with the patient, observe any nasal
lesion is preganglionic. voice or hoarseness (hoarseness indicates bilateral
o If dissociated sensory loss in face, then lesion paralysis of superior laryngeal branch of vagus). If
in brainstem or upper cervical cord (occlusion it is unilateral, then it is usually asymptomatic.
of posterior inferior cerebellar artery). • Ask the patient about any nasal regurgitation.
• 'Open your mouth and say aah': See palatal
Vllth Cranial Nerve movement (arching of palate). If one side remains
flat and immobile, it indicates paralysis of that side
Perform both motor and sensory tests: Look at the face
(soft palate is pulled to the normal side).
and see the following (present in VIIth nerve palsy)
• Ask to cough: If bovine cough, indicates recurrent
• Obvious facial asymmetry; and the affected eye
laryngeal nerve palsy.
appears open and wide.
• Palatal reflex (gag reflex, lXth nerve): Touch back
• Unilateral drooping of the corners of mouth.
of pharynx and see constriction.
• Nasolabial fold is less pronounced.
• Taste sensation in posterior one-third of tongue
Motor test: (1 Xth nerve).
• Tell the patient, Look at the ceiling, keep head
fixed' . See wrinkling of forehead-unilateral or Xlth Cranial Nerve
bilateral (frontal belly of occipitofrontalis). Ask the patient, 'Shrug your shoulder against resistance'
• 'CIQse your eyes tightly; do not let me open' (trapezius muscle).
(orbicularis oculi). If it fails to close, look for Bell 'Turn your head to the other side against resistance; feel
phenomenon. sternocleidomastoid' (test on both sides).
• 'Ask to whistle' (orbicularis oris).
• 'Puff your cheek out' (buccinator). If paralysis, air Xllth Cranial Nerve
escapes easily on the affected side.
• 'Show me your teeth and have a smile' (levator Look at the tongue to see any:
angul i oris and risorius). If paralysis is present, face • Wasting.
is drawn to the healthy side. • Fasciculation.
• Small and spastic or tight.
Sensory test:
Ask the patient: 'Put out your tongue' . Observe the
• Taste sensation in anterior two-thirds of the following:
tongue (nerve fibres pass along the lingual nerve
• If small and spastic (unable to protrude).
to chorda tympani, then to geniculate ganglion of
• Deviation (towards the weak side).
facial.nerve and then to medulla oblongata).
• Posterior one-third of tongue is through Ask the patient, 'Waggleyour tongue' . Feel the weak side.
glossopharyngeal nerve. N.B. Remember the following points:
Finally, look at external auditory meatus and palate to • In unilateral UMN lesion, there may not be any
see rash (Ramsay Hunt syndrome). Also, test for hyper- change in tongue.
acusis (nerve to the stapedius muscle). • In bilateral UMN lesion, tongue is small, spastic
and unable to protrude (found in pseudobulbar
Villth Cranial Nerve palsy, IXth, Xth and XIIth cranial nerve palsy.)
Vestibular division: • In LMNtype oflesion. there is wasting, fasciculation
• Ask the patient about any vertigo or dizziness or and weakness of tongue (found in bulbar palsy,
giddiness. IXth, Xth and XIIth cranial nerve palsy.)
_ SHORT CASES IN CLINICAL MEDICINE

Facial (Vllth) Nerve Palsy


Usual instructions are:
• Look at the face. What are your findings? What else
do you want to examine?
• Examine the face.
~ • Examine the facial nerve.
o
"0
~ Presentation of If---------,
Z Case No. 1 (Instruction 1-
by Looking Face, Suppose Left)
Failure of wrinkling of forehead (left)
• Facial asymmetry.
• Drooping of the left corner of mouth.
• Left eye appears wide and open.
• Nasolabial fold: Less pronounced.

Q: What else do you want to examine?


A: 1want to examine VIIth cranial nerve.

Presentation of
Case No.2 (Instruction
2 or 3-Suppose Left Side)
• There is failure of wrinkling of forehead of left left eye-wide and open
side.
• The left eye cannot be closed; and on attempting
to close, there is Bell phenomenon (eyeball is
rolled upwards and outwards).
• There is weakness of left side of face on puffing
the cheek.
• Failure to whistle and smile.
• While showing the teeth, lips are drawn to right
side.

My diagnosis is left-sided Bell palsy.


Q: Why it is Bell palsy?
A: Because Bell phenomenon is present along with Showing teeth-lips drawn to right
facial pal.sy.

left-sided Bell palsy (Bell phenomenon) Failure to whistle


8 • NEUROLOGY __

Q: Why it is called Bell palsy? • Protection of eye during sleep (shut with tape, or
A: Due to the presence of Bell phenomenon. even tarsorrhaphy), artificial tears or ointment.
• Residual paralysis may occur in 10% cases.
Q: What else do you want to examine? Cosmetic surgery may be helpful.
A: A"c, follows: During recovery, aberrant reinnervation may occur
• Taste sensation in anterior two-thirds of the tongue producing unwanted facial movement and tear dur-
(chorda tympani). ing salivation (called crocodile tear).
• Palate and external auditory meatus to see any
Q: What is the prognosis of Bell palsy?
vesicle (Ramsay Hunt syndrome).
A: As follows:
• Evidence of hyperacusis. • Spontaneous improvement begins in 2nd
week, 70-80% cure within 12 weeks. May take
Q: What is Bell palsy?
12 months. Less than 10% may have residual
A: It is an LMN type of facial palsy.
weakness.
Q: What are the causes of Bell palsy? • Prognosis can be detected by EMG: Reduction of
A: Idiopathic in 95% cases; it may be related to viral amplitude of facial muscle action potential in the
infection due to reactivation oflatent herpes simplex 1st week indicates slow or incomplete recovery.
virus 1. Q: What is Ramsay Hunt syndrome? How to treat?
A: It is the herpes zoster of geniculate ganglia charac-
Q: Where is the site of lesion in Bell palsy?
terized by:
A: Facial canal. in perrous part of temporal bone • Ipsilateral VIlth cranial nerve palsy (lower motor
(in stylomastoid foramen). neuron).
• Rash (herpetic vesicles) in the external auditory
Q: What is Bell phenomenon?
meatus and soft palate.
A: On attempting to dose the eyes, the eyeball rolls
• Ipsilateral loss of taste and buccal ulceration.
upwards and outwards, and is called Bell phenome-
• Deafness and Vth nerve lesion may also occur.
non. It is a normal phenomenon, but cannot be seen
because eyes are dosed. It is only seen in Bell palsy. Treatment: Antiviral like famcidovir may be given.
However, complete recoveryis lesslikelythan Bellpalsy.
Q: What is the relation between diabetes mellitus and
Bell palsy?
A: Diabetes mellitus is thought to be responsible for
10% of the cases of Bell palsy.

Q: What are muscles tested in Bell palsy?


A: As follows:
• Frontal belly of occipitofrontalis (wrinkling of
forehead).
• Orbicularis oculi (closer of eyes).
• Corrugator supercilii (frown).
• Buccinator (puffing of cheek).
Bilateral Bell palsy (Bell phenomenon in both eyes)
• Orbicularis oris (whistling).
• Levator anguli oris and risorius (showing the
teeth and smiling).
• Stapedius muscle (hyperacusis).

Q: How to treat Bell palsy?


A: As follows:
• Prednisolone 40-60 mg daily for 1 week should
be given within 72 h.
• In severe case, prednisolon plus antiviral (acyclovir
or valaciclovir) may be given within 72 h.
• Physiotherapy: Facial exercise and
electrostimulation within 14 days. Bilateral facial weakness in myasthaenia gravis
_ SHORT CASES IN CLINICAL MEDICINE

[Causes-Bell palsy, Ramsay Hunt syndrome,


Causes of bilateral facial palsy trauma and chronic suppurative otitis media
• Cuillain-Barre syndrome (GBS). (CSOM)].
• Sarcoidosis. 4. Lesion in face:
• Bilateral Bell palsy (rare). • Parotid tumour, sarcoidosis, and trauma or
• Bilateral parotid disease. surgery of parotid gland.
@ • Lyme disease.
o
"0 • Any cause of mononeuritis multiplex (rare).
3
v
Z Causes of bilateral facial weakness

• Bilateral facial palsy.


• Myasthaenia gravis.
• Myopathy (fascio-scapulo-humeral
d
myopathy
Presentation of a Case
(UMN Type-Supposing on Right Side):
Case No.4

• Asymmetry of face on the right side. Wrinkling


and myotonic dystrophy). of forehead is normal on both sides.
• The patient can close the eyes properly (upper
N.B. Myasthaenia gravis and some myopathy may part of face is not involved).
mimic bilateral facial palsy. However, these are • There is involvement of lower part of the face
not due to nerve involvement. (as evidenced by impaired puffing of the cheek,
smiling, whistling and showing the teeth).
Presentation of a Case
(LMN Type of Facial Palsy): Case No.3
My diagnosis is right-sided UMN type of facial palsy.

Q: Why UMN lesion? What else do you want to see


• Just like Case 1, but there is no Bell phenomenon.
and Why?
A: It is UMN lesion because only the lower part of
My diagnosis is LMN type of facial palsy.
face is affected and upper part is spared. I want to
Q: Why LMN type of facial palsy? see if the patient has hemiplegia (ask the patient to
A: Because both upper and lower parts of face are raise the arms and legs) because common cause of
affected. UMN facial palsy is CVA,involving internal capsule,
which is associated with hemiplegia.
Q: Why not UMN type of facial palsy?
A: In UMN of facial palsy, only lower part of face is Q: Is there any isolated liMN facial palsy without
involved, as muscles of upper part of face are bilat- hemiplegia?
erally innervated.
A: Yes, if the lesion occurs in the contralateral frontal
Q: How to locate the site of lesion in LMN facial palsy complex (involving inferior frontal branch of the
and what are the causes? middle cerebral artery).
1. Lesion in pons:
Q: What are the differences between UMN and LN1N
• Associated VIth nerve palsy (lateral rectus type of facial palsy?
muscle of eye) and Vth nerve palsy.
• If corticospinal tract is involved, there is A: As follows:
contralateral hemiplegia. (Causes-vascular,
demyelinating disease, tumour and Topic UMN LMN
syringobulbia. ) Site of lesion Above the In the facial nucleus
2. Cerebellopontine angle lesion: facial nucleus, and distal to the
• Usually associated with Vth, Vlth and commonly in nucleus
VIith nerve palsy and also cerebellar signs. internal capsule
(Causes-acoustic neuroma, neurofibroma Involved area Lower part of Both upper and lower
and meningioma.) face part of face
3. Lesion in petrous part of temporal bone: Bell phenomenon Absent May be present
• Associated with loss of taste in anterior two-
Taste sensation Not affected May be affected
thirds of the tongue and also hyperacusis.
8 • NEUROLOGY __

Hyperacusis Absent May may occur, Associated Usually Not so (contralateral


if nerve to the feature associated with hemiplegia, if
stapedius is involved hemiplegia pontine lesion); other
Facial wasting or Absent May be present findings according to
atrophy site of lesion

z
g
o
0-
Vlth Nerve Palsy ~
Usual instructions are:
• Diabetes mellitus.
• Examine the eyes or look at the eyes. What else do • Others: Sarcoidosis, giant cell arteritis, Lyme
you want to examine? disease, acoustic neuroma and nasopharyngeal
• See the movement of eyes. carcinoma.

Q: What are the causes of bilateral Vlth nerve palsy?


Presentation of a Case A: As follows:
(supposing right sided) • Trauma.
• Wernicke encephalopathy.
• The patient has convergence squint in right eye
• Mononeuritis multiplex.
at rest.
• There is no lateral movement of right eye; and on • Raised intracranial pressure (VIth nerve palsy is
attempting to look on that side, there is diplopia often associated with VIIth nerve lesion also).
(outermost image comes from the affected eye).
• Now test for diplopia with covering the eye; the
outer image disappears on covering the affected
eye.

My diagnosis is right-sided Vlth nerve palsy.

Q: What does diplopia look like in Vlth nerve palsy?


A: Image looks parallel and horizontal to each other
(if the image is oblique or angular, it indicates supe-
rior oblique palsy).

Right-sided Vlth nerve palsy


Q: What are the causes ofVIth nerve palsy?
A: As follows:
• Raised intracranial pressure is the commonest
cause, due to stretching of the nerve in its long
intracranial course (a false localising sign).
• Brainstem lesion (vascular, neoplastic and MS).
• Cavernous sinus lesion (tumour, thrombosis,
infection, aneurysm).
• Trauma.
• Any cause of mononeuritis multiplex.
• Idiopathic (common).
• Subacute meningitis (due to fungal, tuberculous,
lymphomatous, carcinomatous). Right-sided Vlth nerve palsy-failure of abduction
_ SHORT CASES IN CLINICAL MEDICINE

Multiple Cranial Nerve Palsy

Presentation of a Case :f-------~


(Supposing IIIrd, IVth and Vlth
Nerve Palsy): Case No. 1

• There is ptosis.
• No movement of eyeball in any direction.
• On attempting to look at the right or left there
is diplopia.
• Pupil is dilated, no reaction to light, both direct
and consensual.

My diagnosis is IIIrd, IVth and Vlth nerve palsy.


Q: What are the causes?
Left-sided ptosis due to IIIrd nerve palsy
A: As follows:
1. Raised intracranial pressure due to any cause:
• Meningitis.
• Encephalitis.
• Sarcoidosis.
• Neoplasm.
• GBS.
2. Brainstem lesion:
• Vascular (CVA).
• Neurosyphilis.
• MS.
• Syringobulbia.
3. Mononeuritis multiplex (due to any cause).
4. Cavernous sinus thrombosis or
carotid-cavernous fistula.
Left eye-failure of adduction and dilated
pupil (IIIrd nerve palsy)
Presentation of a Case II------~
(IIIrd and Vlth Nerve Lesion-Supposing
Right Side): Case No.2

• There is ptosis on right side.


• Pupil is dilated; no reaction to light (both direct
and consensual).
• There is failure of lateral movement of eye; also
there is failure of upward and downward (eye
in abduction) and upward movement (eye on
adduction), and medial movement.
• Diplopia is present on movement of the lateral
gaze.

My diagnosis is right-sided IIlrd and VIth cranial


nerve lesion. Left-sided ptosis and failure of wrinkling due to facial palsy
8 • NEUROLOGY __

z
@
o
0'
~

Failure of abduction (left) due to Vlth nerve palsy

Left IIIrd nerve palsy and right Vllth nerve palsy

Q: What are the causes?


A: As follows:
• Raised intracranial pressure due to any cause.
• Mononeuritis multiplex.
• Meningitis.
• Encephalitis.
• Trauma.

Presentation of a Case :f-------------,


(IXth, Xth and Xllth Cranial Nerve
Palsy): Case No.3
Failure of elevation and depression of eyeball
(inferior and superior oblique palsy-eyeball medially) • There is hoarseness of voice, nasal regurgitation
and nasal voice.
• Palate movement is absenst.
• Gag reflex is absent.
• Tongue is spastic, wasted and weak. There is
fasciculation in the tongue.

My diagnosis is IXth, Xth and XIIth cranial nerve palsy.


Q: What are the causes of IX, X, XI and XIlth cranial
nerve palsy?
A: According to the site of lesion:
1. Within brainstem:
• Infarction.
• Syringobulbia.
• MND.
• Poliomyelitis.
2. Around skull base:
Failure of elevation and depression of eyeball • Carcinoma of nasopharynx.
(superior and inferior rectus palsy-eyeball laterally) • Glomus tumour.
• Neurofibroma.
• Jugular venous thrombosis. Remember the site of lesion involving

• Trauma. multiple cranial nerves

3. Within neck and nasopharynx: • Unilateral lIIrd, IVth, Vth and VIth nerve
• Carcinoma of nasopharynx. involvement suggests lesion in cavernous sinus.
• Metastases. • Unilateral Vth, VIlth and VIIIth nerve invo-
• Carotid artery dissection (XII). lvement suggests lesion in cerebellopontine
angle.
• Polyneuropathy.
• Unilateral IXth, Xth andXIth nerve involvement
• Trauma.
suggests lesion in jugular foramen.
• Lymph node biopsy in posterior
• Combined bilateral Xth, XIth and XIlth nerve
triangle (XI).
involvement suggests bulbar palsy, if LMN
4. Others:
lesion signs are present; and pseudobulbar
• CBS. palsy, ifUMN lesion signs are present.
• Tubercular meningitis. ---__.
• Carcinomatous meningitis. Actions of extraocular muscles:
• Encephalitis. • When eye is abducted, elevator is superior rectus
• Brainstem lesion, vascular (CVA) and and depressor is inferior rectus (both supplied by
neoplastic. IIIrd nerve).
• Bulbar and pseudobulbar palsy. • When eye is adducted, elevator is inferior oblique
• Neurosyphilis. (supplied by Illrd nerve) and depressor is superior
• Mononeuritis multiplex. oblique (supplied by IVth nerve).

(VO with Hemiplegia


Usual instructions are: Q: What would you look for in the heart?
• Examine the lower limb or examine the upper limb. A: Any arrhythmia like atrial fibrillation and any
• Perform the neurological examination of upper or valvular lesion.
lower limbs.
Q: What would you look for in the neck?
A: Carotid bruit (thrombus from carotid may be dis-
Presentation of a Case ~f---------,
lodged and cause cerebral thrombosis).
• There is difficulty in raising the right leg and Q: What is the cause of CVD in this patient?
right arm. A: I would like to take the history of hypertension,
• Muscle tone is increased in both right upper and any cardiac problem and also the onset, history of
lower limbs, but normal in left limbs. unconsciousness, headache, etc. The causes may be:
• Motor power is diminished; grade 0/5 in right
• Cerebral infarction: Commonest cause.
upper and lower limbs, but normal in left side.
• Deep tendon reflexes are exaggerated in right • Cerebral haemorrhage.
limbs and normal in left limbs. Q: What is the likely site of lesion in your case?
• There is ankle clonus on the right side. A: Left internal capsule due to involvement of lenticu-
• Plantar response is extensor on the right side and lostriate branch of middle cerebral artery.
flexor on the left side.
• Sensory functions - normal. Q: What are the diseases included in CVA?
• There is a urinary catheter in situ. A: As follows:
• Cerebral haemorrhage.
My diagnosis is right-sided hemiplegia more likely • Cerebral thrombosis.
with CVD. • Cerebral embolism.
Q: What else do you want to examine? • Subarachnoid haemorrhage.
A: I want to examine the speech and cranial nerves. • Hypertensive encephalopathy.
Also, I want to examine the blood pressure, the • Cerebellar haemorrhage.
heart and the neck (bruit) to find out the cause. • Cerebellar infarction.
8 • NEUROLOGY __

Q: What investigations do you suggest? 2. Subcortical:


A: As follows: • Usually monoplegia.
1. CT scan of the head (first investigation to be done). • May be contralateral hemiplegia.
2. Complete blood count with ESR. • Speech disturbance may be present.
3. Blood sugar. • There may be loss of postural sensibility,
4. Blood urea and serum creatinine. tactile localization and discrimination of
5. Fasting lipid profile. the affected limbs due to involvement of
thalamocortical fibres.
6. Serum electrolytes.
7. Chest X-ray PA view. 3. Internal capsule:
• Contralateral hemiplegia.
8. ECG.
9. Source of the event: • Hemianaesthesia due to damage of the
sensory fibres and homonymous hemianopia
• Doppler study of extra-rintracranial vessels.
due to damage of visual fibres-both of
• Echocardiography: Transthoracic or
which lie posterior to the pyramidal tract in
transoesophageal.
internal capsule.
• MRA or CTA of the cerebral vessels.
• DSA of the cerebral vessels (gold standard to • Global aphasia in left-sided lesion.
find out AVM or aneurysm). • VII th cranial nerve palsy on the side of palsy-
UMN type.
Q: What is stroke? What are the types of stroke? 4. Brainstern:
A: Stroke may be defined as sudden development • Symptoms are: Vertigo, nausea, vomiting.
of focal neurological deficit due to nontraumatic • Crossed hemiplegia, brainstem syndrome,
vascular cause, lasting for more than 24 h. It is of pu p illary abnormality, cerebellar invo lvemen t,
following subvarieties: gaze paralysis, Horner syndrome.
• Transient ischaernic attack (TIA): Sudden
• If pons is involved: There is deep coma,
neurological dysfunction due to cerebral
pin-point pupil. hyperpyrexia, decerebrate
ischaemia lasting less than 24 h; and the patient
or decortical rigidity, absence of lateral eye
recovers completely within 24 h.
movement on head turning.
• Stroke in evolution: The symptoms worsen
• If involvement of midbrain and medulla:
gradually or in a step-wise pattern over hours
There is loss of consciousness, quadriplegia,
or days, and the neurological deficit persists for
Cheyne-Stokes breathing, decerebrate rigidity.
more than 24 h.
• Completed stroke: Clinical signs of neurological Q: How to treat CVD?
deficit are persistent. A: As follows:
• Reversibleischaemic neurological deficit (RIND):
1. General measures:
Neurological deficit persists for more than 24 h,
• Nasogastric tube feeding.
but recovers completely within 3 weeks.
• Regular change of posture (2 hourly).
• Partial non progressive stroke (PNS): Neurological
• Care of bowel, bladder (catheterization),
deficit persists for more than 3 weeks, but is either
mouth (to prevent fungal infection), and
partial or ends up with minimal residual deficit.
eyes (tear naturale or taping of the affected
Q: What are the features of CVD according to the site eye shut).
of involvement of different parts of brain? 2. Control of risk factors: Hypertension, diabetes
A: As follows: mellitus, hyperlipidaemia, etc.
1. Cortical: 3. If cerebral oedema: Dexamethasone or mannitol.
• Usually monoplegia. 4. Specific treatment according to the type of
• If les~on is extensive, contralateral hemiplegia stroke (after CT scan):
may occur. • Cerebral infarction: Antiplatelet drugs
• Speech disturbance may be present, if the (e.g. aspirin, dopidogrel). Cerebral vasodiJator
lesion involves the dominant hemisphere. like vinpocetin should be given. If atrial
• Iacksonian convulsions and headache may fibrillation, heparin followed by warfarin
occur. should be considered.
• There may be cortical type of sensory loss, • Cerebral haemorrhage: For massive
e.g. astereognosis. haemorrhage, neurosurgical intervention
__ SHORT CASES IN CLINICAL MEDICINE

may be required. Other treatment is of naturally occurring anticoagulant (protein C,


symptomatic and supportive. protein S, antithrombin lIJ, factor V Leiden). Tn
• Subarachnoid haemorrhage: Nimodipine can all these conditions, there is increased tendency
be given, whereas neurosurgical intervention of thrombosis.
is essential. • Vasculitis, Behcet disease.
5. To reduce morbidity and improve quality of life: • Vascular malformation: AVM, berry aneurysm
Physiotherapy, speech therapy, occupational causing SAH.
therapy, etc. • Dissecting aneurysm.
• In female: Oral contraceptive pill, eclampsia.
Q: What is the prognosis of CVA?
• Homocystinuria.
A: Prognosis depends on the type of lesion, site, extent
• Syphilis.
of involvement and associated primary risk factors.
• Premature atherosclerosis may occur in familial
• Mortality rate is higher in intracerebral
hyperlipidaemia.
haemorrhage than embolic stroke.
• Rarely, migraine may cause cerebral infarction.
• In cerebral infarction, immediate prognosis IS
• Drugs like amphetamine, cocaine.
better, long-term prognosis depends upon extent
of damage. Q: What investigations should be done in a young
• Tn cerebral haemorrhage, immediate prognosis is patient with stroke?
worse. A: See as above. For young patient, specific investiga-
• In haemorrhagic stroke, 25% die within first tions are as follows:
2 years, 10% die within first month. However, • Chest X-ray, ECG and echocardiography (to
prognosis is better in the long run, when the exclude cardiac problem like MS with AF, other
haematoma resolves. cardiac problem like PFO, TOF).
• One-third returns to independent mobility; one- • CBC, ESR: It will also exclude polycythaemia
third becomes disabled. rubra vera.
• Serum lipid profile: To exclude juvenile
Q: What are the causes of recurrent hemiplegia?
hyperlipidaemia.
A: As follows:
• For collagen vascular disease: ANA, anti-dsDNA,
1. CVD due to:
anticardiolipin and anti phospholipid antibody.
• Atrial fibrillation.
• Perinuclear antineutrophil cytoplasmic antibody
• Hyperviscosity syndrome.
(P-ANCA), cytoplasmic anti neutrophil cytoplasmic
• Homocystinuria.
antibody (C-ANCA).
• Amyloidosis.
• Coagulation screening, serum antithrombin III,
• Deficiency of protein C, S or antithrombin Ill.
protein C and protein S level.
• Polycythaemia rubra vera.
• Others: Red cell mass (PRV), chromatographic
• Antiphospholipid syndrome.
test in serum and urinary level of homocystine or
• Polyarteritis nodosa.
methionine (homocystinuria), TPHA and VDRL
• Wegener granulomatosis.
(syphilis).
• Takayasu arteritis.
2. Other causes: Q: What are the risk factors for stroke?
• MS. A: Risk factors are variable for ischaemic stroke and
• Hemiplegic migraine. haemorrhagic stroke:
• Epilepsy (Todd palsy). Risk factors for ischaemic stroke:
• Hysteric hemiplegia.
• Nonmodifiable:
Q: What are the causes of CVD in a young patient? o Age.
A: As follows: o Gender.
• Mitral stenosis with atrial fibrillation (cerebral o Ethnicity or race.
embolism from cardiac source). o Genetics.
• Other cardiac cause: PFO, VSD, TOF. o Family history.
• Antiphospholipid syndrome. • Modifiable:
• SLE. o Hypertension.
• HaematologicaJ disease: Sickle cell anaemia, o Smoking.
polycythaemia rubra vera, inherited deficiency o Lifestyle.
~ 8__• NEUROLOGY IIIIIIII
o Diabetes mellitus. 2. Millard-Gubler syndrome: Paralysis of VIth
o Obesity. cranial nerve (LMN type) with or without
o Heart disease (atrial fibrillation, ischaemic VIIth cranial nerve palsy (LMN type) with
heart disease, cardiomyopathy, etc.). contralateral hemiplegia (crossed hemiplegia).
o Dyslipidaemia. Lesion is at pons.
o Oral contraceptive pill. 3. Lateral medullary syndrome (posterior inferior
o Alcohol. cerebellar artery thrombosis, also called Wal-
o Previous history of stroke or TIA. lenberg syndrome): The patient presents with
o Carotid vessel atherosclerosis. acute vertigo, nausea, vomiting and diplopia,
o Atheromatous aortic arch. cerebellar and other signs. Features depend on
o Diseases like homocystinuria, hyperfibri- the precise structures damaged and may include:
nogenaemia, deficiency of protein C, Sand
• Ipsilateral:
antithrombin III, polycythaernia rubra vera,
i. Trigeminal: Diminished pain and
anti phospholipid syndrome, thrombotic
temperature (due to the involvement of
throm bocytopenic purpura, sickle cell anaem ia,
descending tract and nucleus of trigemi-
protein V Leiden syndrome, vasculitis (like
nal nerve).
polyarteritis nodosa, Wegener granulomatosis,
11. Cerebellar sign (due to the involvement
Takayasu arteritis), migraine.
of cerebellum and its connection).
o Abuse of cocaine, use of COX-2 inhibitor
iii. Horner syndrome (due to the involve-
(slightly increased incidence of stroke).
ment of descending sympathetic tract).
Risk factors for haernorrhagic stroke: iv. Palatal paralysis and diminished gag
• Hypertension. reflex (there may be hoarseness and
• AVM. dysphagia due to vocal cord paralysis
• Aneurysm. because of IXth and Xth nerve lesion).
• Amyloid angiopathy. v. Diplopia (VIth nerve involvement).
• Cavernous angioma.
• Contralateral:
• Anticoagulant therapy.
1. Loss of pain and temperature due to
• Hypercoagulable disorder.
spinothalamic tract involvement (in the
• Drugs: Cocaine, amphetamine.
trunk, limbs, may be in face). It is called
• Vasculitis: SLE, polyarteritis nodosa (PAN),
dissociated sensory loss.
isolated CNS vasculitis.
• When there is isolated occlusion of PICA,
• Septicaemia.
pyramidal pathways escape and there is no
• Moyamoya disease.
hemiplegia. However, in majority of the cases,
• Haemorrhage into brain tumour.
there is vertebral artery involvement and
Q: How to prevent stroke? pyramidal signs (hemiplegia) are present.
A: As follows: 4. Medial medullary syndrome:
• Risk factors like hypertension, diabetes mellitus, • It is due to occlusion oflower basilar artery or
obesity, etc. should be identified and controlled. vertebral artery or one of its medial branches.
• Smoking and alcohol should be stopped. It is characterized by contralateral hemiplegia,
• Antiplatelet drug, e.g. aspirin. which spares the face, contralateral loss of
• Life style modification: Regular physical exercise, vibration, and joint position sense, ipsilateral
dietary modification. paralysis and wasting of tongue.
• Statin should be given to all patients.
• If there is atrial fibrillation: Treatment of primary N.B. Remember the following points:
cause and anticoagulation. • Damage to brainstem reticular activating
• Treatment of primary cause. system leads to coma.
Brainstem syndrome • Upper brainstem infarction (ventral pons)
1. Weber syndrome: Ipsilateral paralysis of Wrd leads to the locked-in syndrome.
cranial nerve (LMN type) with contralateral hemi- • Pseudobulbar palsy may occur after
plegia (crossed hemiplegia). Paralysis of upward lower brainstem infarction (medullary
gaze is usually present. Lesion is at midbrain. infarction, bilateral cerebral infarction).
_,. SHORT CASES IN CLINICAL MEDICINE

Q: What are the causes of coma or unconsciousness? • Diabetes mellitus [ketoacidosis, hypoglycaemia,
A: As follows (remember the mnemonic lactic acidosis, hyperosmolar non-ketotic diabetic
AEIOU-DAMH): coma (hyperglycaemic hyperosmolar state), drug
• Apoplexy: Cerebral haemorrhage, subarachnoid poisoning (sedativej].
haemorrhage, etc. • Alcohol.
• Epilepsy. • Metabolic: Metabolic acidosis.
• Infection (e.g. encephalitis, meningitis, cerebral • Hypoglycaemia, hypoxaernia, hypertensive
malaria, severe septicaemia). encephalopathy, hepatic coma, hypothyroidism
• Opium poisoning. (rnyxoedema coma), hyponatraernia,
• Uraemia (renal failure). hypothermia, hyperpyrexia, head injury.

Gait Abnormality
Usual instruction: • Stamping gait: The patient raises the foot
suddenly and tends to throw forward, bringing it
• Look at the gait. What is your diagnosis?
to the ground with a stamp, often heel first. It is
found in sensory ataxia.
Presentation of a Case :1------. • High stepping gait: The patient lifts his foot high
to avoid scrapping the toes. It is found in foot
• The patient has broad-based reeling or drunken gait. drop.
• Hemiplegic gait: The patient has difficulty in
bending the knee and drags the hemiplegic
My diagnosis is cerebellar gait.
limb in a semicircle-like motion with the toes
Q: What else do you like to see? scrapping the floor and the forefoot flops to
A: I would like to see other signs of cerebellar lesion, the ground before the heel. This is found in
such as: hemiplegia.
• Scissor gait: During walking, one leg crosses in
• Titubation.
front of the other. It is found in spastic paraplegia,
• Tilting towards the site of lesion.
usually due to cerebral palsy.
• Nystagmus (horizontal).
• Waddling gait: The patient walks on a wide
• Scanning speech.
base with trunk moving side-to-side and
• Intention tremor.
pelvis drooping on each side. At each step toes
• Incoordination. touch the ground before the heel. It is found
• Dysdiadochokinesis. mostly in myopathy, osteodystrophy, bilateral
• Past-pointing (dysmetria). hip joint disease, etc. There is increased lumbar
• Ataxia. lordosis.
• Hypotonia. • Marche a petits pas: There is slow movement.
• Diminished tendon reflex (knee jerk may be The patient walks with very short, shuffling and
pendular). irregular steps with loss of associated movements.
The gait is similar to that of Parkinson disease.
Q: What are the causes of cerebellar lesion? It is seen in normal pressure hydrocephalus.
A: See in 'cerebellar lesion'.
Q: What is astasia abasia?
Q: What are different types of gait? A: This is seen in patient with psychogenic distur-
A: As follows: bance. The patient is unable to walk to stand,
• Cerebellar gait: Broad-based reeling or drunken falls far to the side on walking but usually regains
gait. It is found in cerebellar lesion. balance before hitting the ground. The legs may be
• Parkinsonian gait: Festinate gait mainly (for thrown out widely or the patient may kneel with
details, see in 'Parkinsonism'). each step.
8 • NEUROLOGY __

Orofacial Dyskinesia
Usual instruction: TD is due to altered dopamine receptor sensitivity
• Look at the face. Or look at the patient. induced by the drug.

Q: What are the drugs that may cause tardive


Presentation of a Case I-
1------, dyskinesia?
A: It is usually due to use of neuroleptics like phenothi-
• There is orofacial dyskinesia with chewing,
azines (e.g. chlorpromazine) and butyrophenones
pouting, lip smacking, grimacing and tongue
(e.g. haloperidol) for at least 6 months. Other drugs
movement.
include:
• There is choreoathetosis of the limbs and trunk.
• Antiemetic: Metoclopramide (D2 dopamine
receptor antagonist).
My diagnosis is Orofacial dyskinesia.
• Antiepileptic: Phenytoin, carbamazepine,
Q: What history do you like to take? phenobarbital, ethosuximide.
A: I would like to take drug history. Usually, there is • Others: Reserpine, tetrabenazine, antihistamines,
history of prolonged intake of neuroleptic drugs. fluoxetine, amoxapine.
Q: If there is history of intake of neuroleptic drugs,
N.B. Quetiapine and olanzapine causes less TD.
what will be the diagnosis?
A: Tardive dyskinesia. Tardive means after chronic Q: What other extrapyramidal complications may
exposure to dopamine receptor blockers (such as occur due to neuroleptics?
antipsychotic, antiemetic). A: As follows:
Q: What is tardive dyskinesia? • Acute dystonia: Occurs soon after starting
A: Tardive dyskinesia (TD) is a drug-induced disorder the drug. Usually due to metoclopramide,
characterized by orofacial dyskinesia such as lip prochlorperazine. This is treated by
smacking, chewing, pouting, grimacing, rhythmic IV benzatropine or procyclidine. Also, the
tongue movement and choreoathetoid movement. offending drug should be stopped.
Involuntary movements involve the tongue, lips, • Akathisia (uncontrollable restlessness, repetitive
face, trunk and extremities that have persisted for at and irresistible need to move).
least 4 weeks and that began during treatment with • Parkinson syndrome.
neuroleptics or within 4 weeks of discontinuing
Q: HowtotreatTD?
neuroleptics.
A: The drug should be stopped. Tetrabenazine may help.
Diagnosis of neuroleptic-induced 'I'D generally
requires history of taking neuroleptics for at least Q: What are the different types of dyskinesias!
3 months. It may persist or even get worse after the A: As follows:
withdrawal of drug. • Chorea.
TD is most common in patients with schizophre- • Athetosis.
nia, schizo affective disorder or bipolar disorder who
• Tic.
have been treated with antipsychotic medication for
• Tortion dystonia.
long periods; but occasionally occur in other patients
• Hemiballismus.
as well.

Foot Drop
Usual instruction: • Dorsiflexion and eversion of right foot is weak.
• Examine the lower limb of the patient.
• Ankle jerk is normal.
• Sensation over the lateral aspect of right leg and
Presentation of a Case
dorsum of right foot is impaired.
• There is wasting of anterior tibial and peroneal
group of muscles on the right lower limb.
My diagnosis is right-sided foot drop.
__ SHORT CASES IN CLINICAL MEDICINE

Q: What are the causes of common peroneal nerve


palsy?
A: As follows:
• Trauma (to the nerve, fibular fracture, total knee
arthroplasty or proximal tibial osteotomy).
• Compression by plaster or tourniquet around the
knee, ganglion arising from superior tibiofibular
joint.
• Leprosy.
• Old polio.
Foot drop (right) • Any cause of mononeuritis multiplex: diabetes
mellitus, polyarteritis nodosa, collagen vascular
Q: Where is the lesion?
disease, etc.
A: Common peroneal (lateral popliteal) nerve palsy.
Q: What are the branches of common peroneal nerve? Q: What investigation should be done to diagnose
A: There are two branches: common peroneal nerve palsy?
• Superficial peroneal nerve: Sensory to lateral calf A: Nerve conduction study (local conduction block
and dorsum of foot; also responsible for eversion or slowing in the region of the head of the fibula).
of foot (motor).
• Deep peroneal nerve: Dorsiflexion of foot-and Q: How to treat common peroneal nerve palsy?
toe, sensation to the web space between first and A: As follows:
second toes. • Surgery, if the nerve is severed.
Q: Can it be due to deep peroneal nerve palsy? • Use of splint and calliper shoes for intact and
A: In deep peroneal nerve palsy, the sensory deficit will concussed nerve.
be lirnited to the area between the first and second toes.
Q: What are the reflex and planter responses in com- Q: What are the causes of foot drop?
mon peroneal nerve palsy? A: As follows:
A: Both normal. • Peripheral neuropathy.
Q: What else do you like to examine? What findings • Common peroneal nerve palsy.
do you expect? • Sciatic nerve palsy.
A: I want to examine the gait. There will be high- • Motor neuron disease.
stepping gait (the patient lifts his foot high to avoid • lA, L5 root lesion.
scrapping the toes, and there is an audible 'clop' • Lumbosacral plexus lesion.
of the foot as he walks). Also, the patient will be
unable to stand on the right heel. Q: What are the causes if a patient is unable to walk on
heel?
Q: What is the site of the lesion of common peroneal
nerve palsy? A: As follows:
A: The nerve is usually injured at the head of the fibula • Common peroneal nerve palsy
due to fractureor compression by a tourniquet or splints. • Charcot-Marie- Tooth disease

Charcot-Marie- Tooth Disease


Usual instruction:
• Bilateral pes cavus and clawing of toes are present.
• Examine the lower limb of this patient. Or perform
the neurological examination of the lower limb. • Dorsiflexion: Weak in both feet.
• Ankle jerk: Absent bilaterally.
Presentation of a Case • Plantar response: Bilaterally equivocal or absent.
• Sensory: Mild impairment of both superficial and
• There is wasting of distal muscles of both lower deep sensation up to mid thigh (even marked
limbs up to the middle of legs, giving rise to
sensory loss with trophic ulcer may be found).
inverted champagne-bottle (stork or spindle legs)
• Gait: High steppage gait with foot drop.
appearance.
8 • NEUROLOGY __

My diagnosis is distal motor and sensory neu- 2. Cause of unilateral pes cavus:
ropathy, most likely due to Charcot-Marie- Tooth • Old poliomyelitis.
disease. • Spinal trauma.
• Spinal cord tumour.
N.B. Always look for scar in the neck of the fibula. If • Malunion of calcaneus or talus fracture.
present, it is suggestive of trauma or fracture that
may cause common peronial nerve lesion. Pal- Q: What is the mechanism of pes cavus in Charcot-
pate the nerve that may be thickened. Marie-Tooth disease?
A: There is weakness of anterior tibialis and per-
oneus muscles. The posterior tibialis and peroneus
longus antagonize these muscles resulting in pes
cavus.

Q: What is the mode of inheritance?


A: It is variable, may be autosomal dominant or
recessive or X-linked. Affected family members may
have forme fruste with only pes cavus and absent
ankle jerks.

Q: What are the types of peroneal muscular


atrophy?
A: As follows:
• Hereditary motor and sensory neuropathy type-I:
Inverted-champagne-bottle appearance of lower
limb in Charcot-Marie-Tooth disease There is demyelinating neuropathy. It is the
commonest type (70%), inherited as autosomal
dominant. NCV is reduced.
Q: What else do you want to see?
A: I want to examine the upper limbs. There may be • Hereditary motor and sensory neuropathy type-Il:
There is axonal neuropathy. There is prominent
wasting of small muscles of hands and arm muscles.
sensory involvement with pain and paraesthesia.
There may be claw hand.
It is inherited as autosomal dominant (AD)
and autosomal recessive (AR). NCV is relatively
Q: How does the patient present?
normal.
A: The patient usually presents with foot deformities
• Hereditary motor and sensory neuropathy
or gait disturbance in early childhood or early
type-III (also called distal spinal muscular
adult life. Slow progression leads to features
atrophy): There is demyelinating neuropathy.
of polyneuropathy with distal weakness and
NCV is reduced.
wasting that begins in the leg, associated with
• Other types: CMT with optic atrophy, deafness,
distal sensory loss, depressed or absent tendon
retinitis pigmentosa and spastic paraparesis.
reflexes.
• Rarely another type: CMTX-an X-linked
dominant hereditary motor sensory neuropathy
Q: What are the causes of pes cavus? (HMSN).
A: It may be unilateral or bilateral.
1. Cause of bilateral pes cavus:
Q: What investigation do you suggest?
• Charcot-Marie-Tooth disease.
A: Nerve conduction velocity.
• Friedreich ataxia. • In HMSN type-I: There is marked reduction in
• Muscular dystrophies. motor and sensory conduction velocity.
• Spinal muscular atrophy.
• In HMSN type-Il: Motor conduction velocity is
• Cerebral palsy. normal or slightly reduced, sensory nerve action
• Syringomyelia. potential may be absent, and signs of chronic
• Hereditary spastic paraparesis. partial denervation are found in affected muscles
• Spinal cord tumour. electromyographically (EMG).
__ SHORT CASES IN CLINICAL MEDICINE

Q: How to treat? Q: What is the prognosis of this disease?


A: No curative treatment, only symptomatic. A: It is slowly progressive, arrests in middle life. Lifes-
• Reassurance and education to the patient. pan is usually normal. Abortive cases are common.
• Regular exercise and physiotherapy. Degree of disability is less in spite of marked
• Walking aids. deformity.
• Occupational therapy.
• Orthopaedic measures for correction.
CHAPTER 9

RHEUMATOLOGY
"The rheumatism is a common name for many aches and pains, which have
yet got no peculiar appellation, though owing to very different causes"
- William Heberden

Introduction
Rheumatological diseases are commonly selected in
any clinical examination. Of these, rheumatoid hand is
a very popular short case, though other cases are also
common.
Most of the diagnoses are straightforward that can
be diagnosed by looking at the patient. It is frequently
asked either to examine a particular part of the body
or to look at a part for a spot diagnosis. However, even
if asked to examine a particular part, "look quickly
from head to foot". A good clue for diagnosis may
be obvious, such as systemic sclerosis, systemic lupus
erythematosus (SLE), dermatomyositis and rheuma-
toid arthritis (RA). During examination of joints,
"always 'look, feel, move, measure and compare with Heberden node in DIP joint and Bouchard node
in PIP joint in primary osteoarthrosis
the other side".
Diseases included are: seropositive and seronegative
arthritis, rheumatic fever, fibrositis, neuralgia, myositis, Remember some hallmarks in rheumatolog-
bursitis, gout and other conditions producing somatic ical disease:
pain or soreness and stiffness.
• Heberden node in distal interphalangeal
(DIP) joint and Bouchard node in proximal
interphalangeal (PIP) joint are hallmarks of
primary osteoarthrosis.
• Rheumatoid nodule is the hallmark of rhe-
umatoid arthritis.
• Tophus is the hallmark of gout.
• Heliotrope rash and Gottron sign are pathog-
nomonic of dermatomyositis.
• Butterfly rash usually indicates SLE.
• Osteophyte (radiologically) is the hallmark of I
osteoarthrosis.
• Syndesmophyte (radiologically) is the hal-
Tophus in gout with chalky discharge
lmark of ankylosing spondylitis.
_ SHORT CASES IN CLINICAL MEDICINE

Systemic Sclerosis
Usual instructions are: • There is sclerodactyly (tapering of fingers) and
• Look at the face. What are the findings? What else infarction (or ulcer or gangrene) at the tip of
do you like to examine? finger, and atrophy of pulp is present. May be
• Examine the hands or legs. What else do you like resorption, beaking of the nails (pseudoclubbing
to examine? due to resorption), and amputation of finger
(may be).
Presentation of a Case :~-------, • Telangiectasia at the base of nails.
(by Looking at the Face): Case No. 1 • There is hard nodule over one finger (calcinosis).
• The face is smooth, shiny, tight, and immobile
with hypopigmented and pigmented areas (salt My diagnosis is systemic sclerosis.
and pepper appearance).
N.B. If presence of calcinosis, telangiectasia and sclero-
• Lips are thin, pursed and with puckering.
dactyly, it is called CREST syndrome (calcinosis,
• Nose is pinched up and tapered (beaking of
Raynaud phenomenon, oesophageal involve-
nose: bird beak face).
ment, sclerodactyly and telangiectasia) or CRST
• There is loss of wrinkling of forehead and
syndrome (calcinosis, Raynaud phenomenon,
multiple telangiectasia (mention the location).
sclerodactyly and telangiectasia).
• Puckering of the skin around mouth, and orifice
of mouth is small (microstomia).
• The patient has difficulty in opening the mouth
(ask to open the mouth).

My diagnosis is systemic sclerosis.

Flexion of fingers with skin change (typical


hands in systemic sclerosis)

Face in systemic sclerosis

Q: What else do you want to see?


A: I want to examine the hands, legs and skin of other
parts of the body (see below).
Infarction at the tip of the fingers
with flexion contracture
Presentation of a Case
(Hands): Case No.2 Presentation of a Case 11-------.
• The skin of both hands is smooth, shiny, tight (by Looking and Palpating the Skin):
and thick and oedernatous with pigmented and Case No.3
hypo pigmented area.
• Interphalangeal (IP) joints are swollen with • Skin is pigmented, thick and tight, and also some
flexion contracture. vitiligo is present.
9 • RHEUMATOLOGY _

My diagnosis is systemic sclerosis. • Is there any difficulty in breathing?


• Is there any bowel abnormality? (Occasional
diarrhoea or constipation.)

Q: What is systemic sclerosis? What are the


presentations?
A: It is a connective tissue disease characterized by
fibrosis and degenerative changes in skin, vasculature
and internal organs. Common in females (F:M = 4:1),
age 30-50 years. Usual presentations are:
• Raynaud phenomenon-the commonest
90-97%, may precede by months or years before
other symptoms.
• Tightening and thickening of the skin of hands
Gangrene in left second toe and other parts of the body, arthralgia and
arthritis (non-erosive inflammatory), heartburn
(reflux oesophagitis due to hiatus hernia),
dysphagia, odynophagia, occasional diarrhoea
and constipation (blind-loop syndrome) and
shortness of breath (DPLD).

Two types of systemic sclerosis are:


• Diffuse cutaneous systemic sclerosis (DCSS):
This involves the skin of face and trunk. Also
above the knee and elbow (initially, skin is
oedematous, thick and tight, and several months
later Raynaud phenomenon).
• Limited cutaneous systemic sclerosis (LCSS):This
involves skin below the knee and elbow (initially,
Infarction of great toe
Raynaud phenomenon, followed by skin change,
Q: What is CREST or CRST syndrome? What questions also called CRESTsyndrome).
are asked in CRESTsyndrome? Other varieties (localized):
A: It is a syndrome characterized by: • Scleroderma sine scleroderma: Involves internal
• C: Calcinosis. organ without skin lesion.
• R: Raynaud phenomenon. • Morphoea: Localized, well demarcated, indurated
• E: Esophageal involvement (dysphagia). or plaqued, with central hypopigmentation and
• S: Sclerodactyly. tethering of skin, usually in extremities and face.
• T: Telangiectasia. • Linear: If skin involvement is in a linear pattern,
Ask about Raynaud phenomenon and dysphagia. usually in lower limbs.
CREST syndrome is also called limited cutaneous
Pathology or pathogenesis in systemic sclerosis:
systemic sclerosis, with a better prognosis. Anticen-
• Vascular change: Widespread vascular damage in
tromere antibody is present in serum in 70-80% of
arteries, arterioles and capillaries. There is intimal
the cases (if diffuse deposition of calcium in subcuta-
proliferation, vessel wall inflammation, and
neous tissue with presence of acrosclerosis, it is called
endothelial damage with release of cytokines and
Thibierge- Weissenbach syndrome).
endothelin-I, the latter causes vasoconstriction.
Q: In systemic sclerosis, what questions do you like Also, platelet activation.
to ask? • Fibrotic change: Fibroblast synthesizes collagen
A: The following questions may be asked: I and III, fibronectin, glycosaminoglycans, pro-
• Do your fingers change colour on exposure to ducing fibrosis in dermis and internal organs.
cold? (Raynaud phenomenon). • Humoral immunity: Increased during
• If yes, what colours are they? (Pallor, cyanosis T-lymphocyte and complement activation,
and redness.) autoimmunity and autoantibody production to
• Is there any difficulty in swallowing: nuclear antigen.
Q: What are the histological findings of skin biopsy in 2. Ioints and muscles: Flexion contracture or
systemic sclerosis? deformity. Proximal or distal muscle weakness.
A: As follows:
3. Gastrointestinal:
• Thinning or absence of epidermis.
• Oesophagus: 50% involvement. May be reflux
• Excess collagen and fibrosis of dermis, loss of
oesophagitis, sliding hiatus hernia, constric-
appendages in dermis, perivascular infiltration
tion or secondary achalasia, and dysmotility
of chronic inflammatory cells (lymphocytes
or reduction of peristalsis. Oesophageal man-
and plasma cells). Blood vessels show intimal
ometry shows abnormal and reduced
proliferation and obliteration.
peristalsis in 90%.
Q: What investigations you would perform in systemic • Stomach: Early satiety, gastric outlet obstruc-
sclerosis? tion and recurrent occult upper gastrointestinal
A: Diagnosis is usually clinical. The following investi- tract (CIT) bleeding causing "watermelon
gations may be done: stomach".
1. Complete blood count (ESR is high, but CRP • Intestine: Hypomotility, bloating, distension,
is usually normal unless there is severe organ intestinal obstruction or pseudo-obstruction,
involvement or coexisting infection). blind-loop syndrome, diarrhoea and wide-
mouth diverticula in colon. Rarely, a serious
2. Serology:
disorder called pneumatosis cystoides
• RA test (positive in 20-30% cases).
intestinalis, in which there is radiolucent
• ANA (positive in 70% cases).
cyst or streaks in the wall of small intestine
• Antitopoisomerase 1 or anti-Scl-70 (positive
due to air in the intestinal wall (detected by
in 30% in diffuse type).
plain x-ray abdomen) can occur. The patient
• Anticentrornere antibody (positive 60% in
presents with severe abdominal pain.
CREST and 10% in diffuse systemic sclerosis)
with a speckled nucleolar pattern. 4. Liver: Primary biliary cirrhosis (PBC) may be
associated.
3. Skin biopsy for histopathology.
4. Others: 5. Respiratory:
• Urine RjE (may be proteinuria). • DPLD, pulmonary hypertension and cor
• X-ray of the hands (deposition of calcium pulmonale. Pulmonary hypertension may
around the fingers, erosion, resorption of occur without parenchymal lung disease due
phalanges and disorganization of joints). to pulmonary vessel involvement. Pulmonary

• Chest x-ray (DPLD and honeycomb lung). hypertension is six times more common in
• cr scan of the chest (to detect D PLD). limited type than diffuse type.
• Lung function tests (restrictive lung disease). • Others: Pneumonitis, rarely pleural effusion,
• Barium swallow (dysmotility or reduction of and alveolar ceU carcinoma.
peristalsis, narrowing and dilatation. Hiatus 6. Heart: Dysrhythmia, conduction defect, heart
hernia may be present, detected by barium failure, cardiomyopathy and pericardial effusion.
swallow x-ray in Trendelenburg position).
• Barium follow through. 7. Kidneys (20% involvement):
• Motility study may be done. • Renal failure in advanced stage (often fatal).
• Electrocardiogram (ECG). • Malignant hypertension (difficult to control,
• IgG level (raised). may respond to angiotensin-converting
enzyme inhibitors).
Q: What are the changes in the different systems of the
body? 8. Endocrine:

A: As follows: • Hypothyroidism (due to thyroid gland


fibrosis).
1. Skin changes:
• Impotence.
• In hands and face (see above).
• Skin in other parts of the body (thick, tight, 9. Neurologic:
hyper- or hypopigmented and vitiligo). • Entrapment neuropathy.
• Skin of the chest is tight and thick (looks like • Facial nerve palsy.
Roman breast plate). • Autonomic dysfunction.
9 • RH.EUMATOLOGY _

Q: How to manage systemic sclerosis? • For reflux oesophagitis-proton pump


A: There is no specific therapy. inhibitor and prokinetic drugs.
1. General management for Raynaud phenomenon: • For blind-loop syndrome-broad-spectrum
• Exposure to cold should be avoided (by the antibiotic is useful.
use of gloves or mittens), lubricants should • If renal involvement-ACE inhibitor may be
be used (to avoid dryness). helpful.
• Smoking should be stopped, regular exercise • Myositis-corticosteroid and cytotoxic drugs
and massage of skin, cleanliness of digital ulcer. may be needed.
• Drugs to be avoided (beta-blocker, ergotamine,
oral contraceptive and sympathomimetic). Q: What is the role of steroid?
• Antiplatelet (aspirin) may be given. A: Steroid has little or no role in the treatment of scle-
• Calcium antagonist (diltiazem and nifed- roderma. It can be given in low doses (10 mg/day)
ipine). ACE inhibitor, angiotensin II receptor in DPLD due to PSS with cyclophosphamide.
blocker (valsartan) may be effective. Prognosis of systemic sclerosis: Depends on type,
• If no response or in severe case, prostacyclin age, sex, involvement of internal organ and extent of
analogue epoprostenol infusion intermittently.
skin involvement Bad prognostic factors are:
• If still no response, then surgery (digital • Diffuse cutaneous systemic sclerosis.
sympathectomy and microsurgical revascu-
• Elderly patient.
larization). lumbar sympathectomy, thoracic
• Male sex.
sympathectomy under video-assisted thoracic • Tnvolvementofinternal organs (especially kidney,
surgery. If needed, amputation of fingers lung and heart). Proteinuria, high ESR low gas
or toes. transfer of carbon monoxide and pulmonary
• Antibiotic may be needed if there is infec-
hypertension.
tion on ulcerated skin lesions. Higher doses
should be given since tissue penetration is Limited cutaneous systemic sclerosis (CRST or
poor in scleroderma. CREST):
2. For arthritis, non-steroidal anti-inflammatory • Prognosis is relatively better.
drugs (NSAIDs). • Usually mild.
3. Steroid and cytotoxic drugs may be used, if • 70% show lu-year survival.
myositis or alveolitis. • Pulmonary hypertension may develop later on.
4. Other drugs that have shown no proven benefit, Diffuse type:
but may be tried are: • 70% show S-year survival.
• Penicillamine-reduces cross-linking of colla- • Death due to cardiac, renal and pulmonary
gen (acts as antifibrotic). involvement.
• Methotrexate (given in a weekly dose of
7.5-15 mg). Q: What are the differential diagnoses of systemic
• Cyclosporine or interferon-y. Recombinant sclerosis or thickened skin?
human relaxin therapy (subcutaneously). A: Scleroderma-like thickening of skin, secondary to
S. Other therapy, according to the involvement of some other diseases is called pseudoscleroderma,
the organ: The causes are:
• Physiotherapy. • Scleroederna.
• If DPLD-cyclophosphamide or azathioprine • Sderornyxoedema.
combined with low-dose steroid. • Eosinophilic fasciitis or eosinophilic myalgia
• Hypertension-ACE inhibitor may be given. syndrome.
• Pulmonary hypertension-oral vasodilator, • Amyloidosis.
warfarin and oxygen. In advanced case, pro- • Graft-versus-host disease (GVHD).
stacyclin therapy (inhaled, subcutaneous • Diabetic cheiroarthropathy.
or intravenous) or oral endothelin receptor • Acromegaly.
antagonist (bosentan, sitaxentan) sbould be • Porphyria cutanea tarda.
given. • Carcinoid syndrome.
• Right heart failure-symptomatic treatment • Toxic oil syndrome.
with diuretic, digoxin etc. Heart lung or single • Chemically induced (pentazocine, bleomycin
lung transplantation in selected cases. and vinyl chloride).
_ SHORT CASES IN CLINICAL MEDICINE

Eosinophilic Fasciitis • Marked fibrosis of SC fascia.


• Infiltration of inflammatory cells.
It is a scleroderma-like syndrome characterized by pain,
• Infiltration of plenty of eosinophil.
swelling, and induration of skin and subcutaneous tis-
sue followed by sclerosis of dermis and subcutaneous Treatment:
tissue.
• Self-limiting, spontaneous resolution may occur in
• Skin is thick, but not tight. 2-5 years.
• Common in adult, involves the skin of forearm • Good response to steroid.
and leg first, then peripheral parts. • H2-receptor blocker, such as cimetidine, may help.
• No Raynaud phenomenon and no internal organ • Diet containing L-tryptophan should be avoided.
involvement.
• Carpal tunnel syndrome is an early feature, more Scleroedema
marked after exercise.
It is a disease characterized by painless, oedernatous
• It is associated with eosinophilia, hypergam-
induration of face, scalp, neck trunk and upper part of
maglobulinaemia, thrornbocytopaenia. aplastic
the extremities. There is no involvement of the hands
anaemia and myelodysplastic syndrome.
and feet. Common in children, may be associated with
Microscopical examination of skin in eosinophilic streptococcal infection, usually self-limiting, resolves in
fasciitis shows: 6-12 months.

Raynaud Phenomenon or Disease


Usual instructions are:
• Look at the hand. What are the findings? What is
your diagnosis?

I
Presentation of a I

Case (Hands)

• The fingers are.pale, some are cyanosed and cold.


• There is sclerodactyly (tapering of fingers),
infarction (or ulcer or gangrene) at the tip of
finger and atrophy of pulp. May be resorption,
beaking of the nails (pseudoclubbing due to
resorption) and amputation of finger (may be).
Pallor on exposure to cold (palmar aspect)
Diagnosis is Raynaud disease.

Pallor on exposure to cold (dorsal aspect) Infarction at the tip of fingers


9 • RHEUMATOLOGY _

proposed as a cause for vasoconstriction in some


patients with Raynaud phenomenon.

Q: What is Raynaud disease and Raynaud syndrome?


What are the causes?
A: As follows:
• When the disease is primary, it is called Raynaud
disease.
• When the disease is secondary to other disease, it
is called Raynaud syndrome.
Causes:
1. Primary or idiopathic: Also called Raynaud
disease (50-60%).
• Five times more common in females than in
Infarction at the tip of fingers, males.
pulp atrophy • Usually in young patients, 20-40 years of age,
may be associated with angina.
Q: What is the commonest underlying disease? • Thumbs often spared. Usually runs benign
A: Systemic sclerosis. course.
2. Secondary causes:
Q: What is Raynaud phenomenon? • Systemic sclerosis (the commonest cause).
A: Raynaud phenomenon is a syndrome of episodic • Collagen disease (SLE, mixed connective
digital ischaemia manifested by pallor, cyanosis and tissue disease [MCTDj, RA, dermatomyositis
redness of the fingers and toes on exposure to cold. and Sjogren syndrome).
Vibration and emotional activity may precipitate • Occupation (pneumatic drills and vibrating
the attack. It is due to episodic vasoconstriction or tools ).
vasospasm of small arteries, arterioles and capillar- • Obliterative arterial disease (atherosclerosis
ies of fingers and toes. and Buerger disease).
• Drugs (beta-blocker, ergotamine and
Q: What are the signs of Raynaud phenomenon? What cytotoxic drug: vinblastine, bleomycin and
are the mechanisms? cisplatin) .
A: Three signs or stages (remember PCR): • Neurogenic (cervical rib and spinal cord
• First: Pallor (due to vasoconstriction of peripheral disease).
arterioles). • Haematological (cryoglobulinaemia or cold
agglutinin disease, myeloproliferative disease
• Second: Cyanosis (due to deoxygenated blood
and Waldenstrom macroglobulinaemia).
following ischaemia).
• Last: Redness (due to reactive hyperaemia, there Q: What are the differential diagnoses of Raynaud
is increased blood flow following vasodilatation). phenomenon?
Mechanism: A: As follows:
• Acrocyanosis.
There are two theories:
• Erythromelalgia.
• Exaggerated reflex sympathetic vasoconstriction • Chilblain.
(Raynaud original theory). • Cold agglutinin disease.
• Abnormalities in the vascular wall resulting in
hyper-responsiveness and vasoconstriction to Q: How to treat Raynaud disease?
cold. A: See in systemic sclerosis.
• Recent opinion suggests that accelerated
destruction of platelet and release of agents such N.B. Women who develop toxaemia of pregnancy are
as serotonin and thromboxane Az have been more likely to have history of Raynaud disease.
__ SHORT CASES IN CLINICAL MEDICINE

Rheumatoid Hand
Usual instructions are:
Presentation of
• Examine the hands of this patient. Case No. 1 (Rheumatoid Hand)
Proceed as follows: 1. Inspection:
(Look at the dorsum and palm of hands, then proceed.) • There is generalized wasting of small
Inspection: (There is bilateral, symmetrical polyarthrop- muscles of hands with dorsal guttering. No
athy). Look carefully the following points: atrophy, ulcer, infarction, gangrene or rash.
• Wrist joints are swollen. PIP joints of both
1. Wasting-generalized, involving thenar, hypoth- hands are spindle shaped. There is swan
enar and all the small muscles of the hand. There neck deformity in right little and ring
is dorsal guttering in both hands. fingers, Z deformity in right thumb and
2. Any ulcer, infarction, gangrene, rash, rheumatoid ulnar deviation of the right hand. There is
nodule and palmar erythema (mention if present). mild flexion deformity of fingers on right
3. Joints of hands: hand as evident by prayer sign.
• There is dorsal subluxation of the ulnar
• Flexion deformity or contracture.
styloid.
• PIP-spindle-shaped swelling and boutonniere • Movement is restricted in wrists and fingers
deformity. of both the hands with slightly impaired
• DIP-swan neck deformity. functional activity.
• Z-deformity of thumb, ulnar deviation and 2. Palpation:
dorsal subluxation of ulna at the carpal joint.
• The joints are tender.
4. Wrist joint-any swelling, synovial thickening and • Synovial thickening is present in both wrists.
subluxation. • There is a rheumatoid nodule in the extensor
5. Ask the patient to make apposition of both hands surface of the right forearm.
together (prayer sign), flex and extend the fingers
alternately (see any deformity).
My diagnosis is rheumatoid arthritis.
Palpation:
1. Ask the patient, if any pain in hands (be careful Presentation of
not to hurt the patient). Case No.2 (Vasculitis)
2. Palpate the joints for tenderness. • Mention as above.
3. Feel for rheumatoid nodule on palmar surface • There is a small ulcer and few infarctions in the
(head of metacarpals), dorsal surface of fingers tip of fingers and toes (mention which one).
(also seen in extensors offorearm, elbow, occiput,
• Radial pulse is feeble.
scapula and shin and tendo-Achilles).
4. Neurological examination of hands (see page 333
and 353). My diagnosis is rheumatoid arthritis with vasculitis.

5. Finally, assess the functional activity of hands:


Presentation of
• Ask th.e patient to use a glass and drink, open
Case No.3 (Foot)
the button, write with a pen, grip strength (ask
to squeeze your fingers) and key grip (give a • All the metatarsophalangeal )0111tsand inter-
key, ask the patient to move the hand with the phalangeal joints are swollen and deformed in
key, as in unlocking). both feet.
• Lateral deviation with dorsal subluxation of toes.
Q: At this point, the examiner may ask, "What else do • Plantar subluxation of the metatarsal head.
you like to see?" • Hallux valgus deformity.
A: Yes, I want to examine other joints, eyes, chest and • Feet are flat with plantar erythema.
abdomen to see extra-articular manifestations.
9 • RHEUMATOLOGY _

My diagnosis is rheumatoid arthritis.

Vasculitis in rheumatoid arthritis


Swan neck deformity (DIP)

Boutonniere deformity (PIP)

Vasculitis in rheumatoid hand

Q: Could it be seronegative arthritis or ankylosing


spondylitis?
A: Unlikely, since in seronegative arthritis there is
usually asymmetrical involvement of the bigger
joints. But in my case, there is bilateral symmetrical
involvement of small peripheral joints.

Q: Which joint is spared in RA?


A: Distal interphalangeal joint (may be involved in
secondary osteoarthrosis).
Z-deformity of thumb, ulnar deviation
and swan neck deformity Q: What is RA?
A: It is a chronic autoimmune inflammatory destruc-
tive and deforming polyarthritis characterized by
bilateral symmetrical involvement of small and
large joints, with systemic involvement and extra-
articular features having a prolonged course with
intermittent exacerbation and remission.
Age 30-50 years, common in females. Before men-
opause, it is 3 times more in female than male, but
after menopause, sex ratio is almost equal.

Q: What are the mechanisms of wasting of muscles


in RA?
A: Multiple factors are responsible: disuse, vasculitis,
polyneuropathy, mononeunus multiplex and
Severe rheumatoid arthritis (Dorsal guttering) entrapment neuropathy.
~ SHORT CASES IN CLINICAL MEDICINE

Q: What is boutonniere deformity? What is the


mechanism?
A: Fixed flexion of PIP joint and extension of DIP
joint. Because of chronic synovitis of PIP joint,
there is rupture of central slip of extensor tendon,
allowing to protrude the joint between two lateral
slips of extensor tendon.

Q: Why it is called boutonniere or button-hole?


A: Because of rupture of central slip of extensor
tendon, it looks like gap of button hole.

Q: What is swan neck deformity? What are the


mechanisms? Rheumatoid nodule (elbow)
A: Fixed flexion of DIP joint and extension of PIP joint
(reverse of boutonniere).
Mechanism: Because of chronic synovitis:
• Rupture or stretching of extensor tendon on
dorsum of DIP joint.
• Secondary to stretching of volar plate at PIP joint.
• Shortening of intrinsic muscles of hands, which
exerts tension on dorsal tendon sheath leading to
hyperextension of PIP joint.

Q: Why radial deviation?


A: Weakness of extensor carpiulnaris leads to radial
deviation at wrist as the carpal bone rotates.

Q: Why ulnar deviation? Rheumatoid nodule (forearm)


A: It occurs in response to radial deviation to keep
the tendons to the phalanges in normal line to the Q: What is Baker cyst?
radius. A: Cyst on the back of knee joint communicates with
the joints but fluid is prevented from returning to
Q: Why Z-deformity of thumb? the joint by a valve-like mechanism. May rupture
A: Because of chronic synovitis, there is hyperexten- during knee flexion and fluid enters into the calf.
sion of IP joints and fixed flexion and subluxation After rupture, severe pain, swelling and tenderness
of metacarpophalangeal (MCP) joints of thumb. in the calf. Confuses with deep venous thrombosis
(DVT).
Diagnosis: USC of knee joint and cr scan or
magnetic resonance imaging (MRI) and arthrogram.
Treatment after rupture of Baker cyst:
• Relief of pain by NSAID.
• Elevation of the limb.
• Intra-articular injection of steroid (methylpre-
dnisolone or triamcinolone).
• Surgical toileting and removal of cyst ifnecessary.

Q: What are the causes of Baker cyst?


A: RA, osteoarthrosis and rarely congenital.

Q: What is rheumatoid nodule? Where is it found?


A: These are painless, firm, subcutaneous nodule,
invariably associated with positive rheumatoid factor.
Baker cyst It is present in 20-30% cases of rheumatoid arthritis.
Sites: Pressure points such as elbow, extensor surface • Level of disability at the onset.
of forearm and hands (fingers), scapula, scalp, sa- • Female sex.
crum, shin, Achilles tendon, toes, sclera, pleura, lungs 2. Blood tests:
and pericardium. • High titres of anti-CCP antibodies and RA.
Significance of rheumatoid nodule: • High CRP.
• It is one of the diagnostic criteria. • Normochromic normocytic anaemia.
• Associated with high titre of rheumatoid factor 3. X-rays:
(positive RAtest). • Early erosive damage
• Associated with active and aggressive RA.
• A bad prognostic sign. N.B. Ultrasound and MRI can show cartilage and
bone damage prior to conventional X-rays.
Histologically three zones:
• Central zone of necrotic material including Q: What is malignant RA?
collagen fibril, noncollagen filament and cellular A: Severe and progressive RA associated with severe
debris. extra-articular manifestations, systemic features and
• Mid zone of palisading macrophages. vasculitis.
• Outer zone of granulation tissue.
Q: What are the extra-articular manifestations of RA?
Nodules may ulcerate and become infected. Treat- A: As follows:
ment with methotrexate may increase the number of 1. Eye:
rheumatoid nodule in some patients. • Episcleritis.
Nodules resolve when the disease is under control. • Scleritis.
If it causes problem, it may be removed surgically or • Scleromalacia.
by local injection of corticosteroid. • Scleromalacia perforans.
N.B. Rheumatoid nodules may be confused with • Keratoconjunctivitis sicca.
tophi of gout, xanthomata, olecranon bursa in 2. Respiratory:
elbow, sarcoid nodule or neurofibroma. • Pleurisy,
• Pleural effusion (may be bilateral).
Q: What are the diagnostic criteria of RA? • Fibrosing alveolitis.
A: AM (American Rheumatism Association) criteria: • Nodules in the lungs (Caplan syndrome).
• Morning stiffness (>1 hour). • Obliterative bronchiolitis (rare).
• Arthritis of three or more joint areas. 3. Cardiac:
• Arthritis of hand joints and wrist. • Pericarditis (30%).
• Symmetrical arthritis. • Pericardial effusion (rare).
• Rheumatoid nodule. • Chronic constrictive pericarditis (rare).
• Positive RAfactor. • Rarely myocarditis, endocarditis, heart block,
• Typical radiological changes (erosion or dysrhythmia, cardiomyopathy, aortic regur-
periarticular osteopaenia), gitation and coronary artery occlusion.
• Duration is 6 weeks or more. 4. Vasculitis: Digital arteritis and nail fold infarct,
N.B. When four or more criteria are present, there is Raynaud phenomenon, visceral arteritis, monon-
93% sensitivity and 90% specificity. euritis multiplex and pyoderma gangrenosum.
5. Neurological:
Q: What are the bad or poor prognostic factors of RA? • Entrapment neuropathy (compression of
A: As follows: nerves by hypertrophicsynovium], commonly
1. Clinical: carpal tunnel syndrome (compression of
• Insidious rather than explosive onset. median nerve) and tarsal tunnel syndrome
• Early development of rheumatoid nodule. (compression of posterior tibial nerve).
• Extra-articular manifestations • Peripheral neuropathy.
• Severe functional impairment. • Mononeuritis multiplex.
• One-year active disease without remission. • Cervical cord compression (due to atlanto-
• Increasing number of peripheral joints axial subluxation).
involvement. • Progressive cervical myelopathy.
_ SHORT CASES IN CLINICAL MEDICINE

6. Haematological:
• Anaemia: Usually normocytic normochromic,
occasionally macrocytic due to folate
deficiency or associated with pernicious
anaemia and microcytic hypochromic due to
bleeding from NSAID.
• Thrombocytosis.
• Eosinophilia.
• Pancytopaenia (due to hypersplenism in Felty
syndrome).
7. Musculoskeletal:
• Muscle wasting. Scleromalacia
• Tenosynovitis.
• Bursitis. Q: What is rheumatoid factor?
• Osteoporosis. A: Rheumatoid factor is an antibody, directed against
8. Others: Fc portion of IgG. It may be IgM or IgG type. Rheu-
• Lymphadenopathy. matoid factor is positive in 75% cases, but 100%
• Splenomegaly. positive in RAwith extra-articular manifestations. It
• Osteoporosis. is detected by latex slide test (RA test is more sensi-
• General features (malaise, fever, weakness, tive, but less specific, and is used for screening) and
loss of weight and wasting). Rose Waaler test (RW test, the sheep cell agglutina-
• Amyloidosis. tion test, is less sensitive, but more specific). Other
causes of positive rheumatoid factor are:

• Collagen diseases: Sjogren syndrome (90%),


SLE (30%), systemic sclerosis, dermatomyositis,
fibrosing alveolitis and mixed essential
cryoglobulinaemia.
• Infections: Infectious mononucleosis, infective
endocarditis, tuberculosis, leprosy, kala-azar,
hepatitis B, syphilis, malaria, filariasis and
schistosomiasis.
• Liver disease: PBC (50%).
• Sarcoidosis.
• Temporarily after vaccination and blood
transfusion.
• Over 65 years (20% in normal population).
Episcleritis High titres of rheumatoid factor indicate:
• Severe erosive disease.
• More extra-articular manifestations.
• Poor prognosis.
• Associated with rheumatoid nodule, vasculitis
and Felty syndrome.
Q: What are the mechanisms of anaemia of RA?
A: As follows:
• Anaemia of chronic disorder.
• Megaloblastic anaemia (due to either folate
deficiency or if associated with pernicious
anaemia).
• Hypersplenism (Felty syndrome).
• Haemolytic anaemia (Coombs test may be
Scleritis positive).
9 • RHEUMATOLOGY __

• CIT bleeding (due to NSAID or vasculitis: causing Q: What are the investigations done in RA?
iron-deficiency anaemia). A: As follows:
• Marrow suppression (gold and penicillamine, • FBC (ESR is high, pancytopaenia may occur in
though less used nowadays). Felty syndrome).
• RA test and RW test.
Q; What is palindromic RA? • Anti-CCP antibody (cydic citrullinated peptide).
A: Recurrent acute episode of monoarthritis lasting • X-ray of hands and other involved joints, chest
24-48 hours. Knee and finger joints are most com- X-ray.
monly affected, but any peripheral joint may be • Others: CRP (high) and urine ana.lysis (proteinuria,
involved. Fever may occur, but no other systemic may occur in amyloidosis).
features. It may be confused with acute gouty
Q: What is anti-CCP antibody? What is its significance?
arthritis and atypical onset of rheumatoid arthritis.
A: It is the antibody to cydic citrullinated peptide
There may be many attacks without any permanent
(CCP). It binds to peptides in which the amino
articu.lar damage. However, one-third to half cases
acid arginine is converted to citrulline by pepti-
may develop typical RA. This can be treated with
dylarginine deaminase, an enzyme abundant in
NSAID during pain. Hydroxychloroquine may be
inflamed synovium.
used in preventi ng recurrent attack.
Anti-CCP antibody is highly specific in RA (95%),
Q: What is Caplan syndrome? present in 70% patients with rheumatoid arthritis.
A: Rheumatoid lung nodules with pneumoconiosis is Un.like rheumatoid factor, it is not positive in other
called Caplan syndrome. Common in coal-workers' autoimmune diseases. It is helpful for early diag-
pneumoconiosis, may occur in any pneumoconio- nosis and is also a predictor of an aggressive disease.
sis. Nodules are rounded, 0.5-2.5 em, present at In patient with undifferentiated arthritis, anti-CCP
periphery of the lung. These may rupture causing antibody is helpful for early diagnosis of RA. It may
pneumothorax or may cavitate and cause haerno- be detected in asymptomatic patient several years
ptysis. It may be confused with tuberculosis or before the development of RA.
neoplasm. Q: How to treat RA?
A: As follows:
Q: What is Felty syndrome?
• Relief of symptoms by rest and NSAID. Also
A: Rheumatoid arthritis with splenomegaly and
intra-articular injection, splinting, hydrotherapy
neutropaenia is called Felty syndrome. There may
etc. may relieve symptoms.
be hypersplenism (anaemia, leucopaenia and
• Suppression of activity and progression of
thrombocytopaenia). It occurs in long-standing,
disease by disease modifying antirheumatic drug
seropositive, deforming, but inactive arthritis, in
(DMARD).
<1 % of the cases. Females are affected more than
• Other measures: Physiotherapy, orthopaedic
males, age 50-70 years. Leg ulcers or sepsis are
measures. Synovectomy of the wrist or finger
complications due to nutropaenia, Splenectomy
tendon sheaths of the hands may be required for
may be necessary for hypersplenism. It is associated
pain relief or to prevent tendon rupture when
with high titre of rheumatoid factor, subcutaneous
medical interventions have failed. Osteotomy,
nodules and other manifestations of systemic rheu-
arthrodesis or arthroplasty may be needed later.
matoid disease.
• Patient education is important.
Q: What precaution is necessary before general anaes- Q: What are the surgical treatments in RA?
thesia or upper CI endoscopy? A: As follows:
A: X-ray of the cervical spine should be taken to ru.le • Synovectomy of wrist or finger tendon sheath
out atlanto-axial subluxation. (Otherwise during (for pain relief or tendon rupture).
anaesthesia, cervical cord compression may lead to • In advanced or severe cases, arthrodesis,
sudden death). arthroplasty and osteotomy may be done in
selected cases.
Q: If the patient develops nephrotic syndrome (or
proteinuria in urine examination), what is the Q: What DMARDs are used?
likely cause? A: As follows:
A: Renal amyloidosis. • First choices are sulphasalazine and methotrexate.
__ SHORT CAHS IN CLINICAL MEDICINE

• Other drugs: Chloroquine, hydroxychloroquine, • Mechanism of action: Competitively inhibits


leflunomide, azathioprine, and cydosporine dihydrofolate reductase, interfering with DNA
(gold and penicillamine are not used any more). synthesis and cell division.
• Anti-TNF-alpha and IL-l, also rituxirnab. are Chloroquine:
more effective than other DMARD in preventing
• It is a weak with slow DMARD effect.
joint erosions. If disease activity persists despite
• Dose: 250 mg daily as a single dose.
an adequate trial of two DMARDs, anti-TNF
• Side effects: Anorexia, nausea, vomiting, skin
therapy should be considered.
rash. Prolonged use may cause neuromyopathy
DMARD should be started from the beginning, may and ocular toxicity. In eye, corneal microdeposit
take 4-12 weeks forresponse. If no effect in 6-12 weeks, (reversible after drug withdrawal), retinopathy
combination with methotrexate and sulphasalazine (may cause blindness), bull's eye macula and
may be given. Prednisolone 7.5-] 0 rng may be added optic neuritis.
with DMARD to give symptomatic relief. • Periodic examination of eye is essential. To
reduce ocular toxicity, the drug can be given for
Brief note about DMARDs: 10 months in a year.

Sulphasalazine: Hydroxychloroquine:
• Dose: 200-400 mg daily.
1. When taken by mouth, it is broken to sulp-
• Used alone in mild disease or as an adjuvant with
hapyridine and 5-aminosalicylic acid (ASA).
other DMARD.
Sulphapyridine is effective in RA (ASA is effec-
• Retinopathy is the serious complication, but this
tive in inflammatory bowel disease). It acts
is rare before 6 years of treatment.
probably by inhibiting cycle-oxygenase and
other enzymes responsible for synthesis of Lefiunomide:
prostaglandin. • [t can be used if methotrexate fails to respond.
Also can be used together with methotrexate.
2. Dose: Stan with low dose, increase the dose
• Dose: 100 mg daily for 3 days, then 10-20 mg
weekly.
daily.
• Initially, 250 mg (112 tablet) BID for 1 week. • Side effects: Skin rash, diarrhoea, reversible alo-
• Then 500 mg (1 tablet) BID for 1 week. pecia, hepatotoxicity, carcinogenic and teratogenic.
• Then 1,000 rng (2 tablets) BID to be continued It needs a washout of 2 years before conception
(maximum dose 2-3 g daily). (3 months in man and 2 years in woman), so
avoid in women who want to be pregnant.
3. Side effects: GIT upset (anorexia, nausea, vom-
iting, diarrhoea), skin rash, Stevens-Johnson • Periodic blood check up is mandatory.
syndrome, reversible sterility in males, blood Biological agents:
dyscrasias (agranulocytosis, megaloblastic anae- These agents block specific immune factors respon-
mia and haemolytic anaemia). sible for RA. Drugs are anti-TNF-aJpha and IL-1
receptor antagonist. These are highly expensive, used
4. Periodic check up: FBC, liver function test
only when there is failure of two DMARDs.
(SGPT) and renal function tests (creatinine),
every 1-3 months. Anti- TNF-alpha:
Methotrexate: 1. It is more effective, rapid onset of action, greater
• Very effective DMARD. May take 4-6 weeks to act. clinical efficacy and sustained benefit than
• Dose: 7.5-10 mg. in a fixed day weekly (up to standard DMARD.
25 mg). Folic acid 5 mg/day should be given on 2. Anti-TNF drugs are:
the next day (folinic acid is more preferable). • Infliximab (given in infusion every
• Side effects: Anorexia, nausea, vomiting (pre- 1-2 months)
vented by using anti-emetic before starting the • Etanercept (given SC every 2 weeks)
drug). Rarely, may cause bone marrow depression. • Adalimumab (given SC every 2 weeks)
Prolong use may cause hepatic fibrosis and DPLD. 3. Common side effects: Hypersensitivity reaction,
• Periodic check up: CBC, liver function tests headache, hypotension. Reactivation of latent
(SGPT) and renal function test (creatinine) tuberculosis may occur. Sometimes lymphoma
should be done. may occur.
9 • RHEUMATOLOGY __

Anti-interleukin (IL-I) receptor: Diagnosed by Two major or one Four of seven


• Anakinra may be used in RAwhen anti-TNF-a is major and two criteria
unsuccessful (less effective than anti-TN F). It is minor criteria plus
used in combination with methotrexate. signs of previous
Streptococcus
Anti-interleukin 6 (IL-6) receptor:
infection
• Tocilizumab: It is a humanized monoclonal
antibody against IL-G receptor. It is used for Morning stiffness Absent Present ~
(l)
~
treatment of moderate to severe active rheumatoid RAtest Negative Positive 8

arthritis in adult patient alone or in combination Subcutaneous Present Rheumatoid
M
0
with methotrexate and/or other DMARDs. Dose 0"
nodule nodule ~
8 mg/kg as TVinfusion, once in every 4 weeks.
Erythema Present Absent
Rituximab: marginatum
• It is an antibody directed against the CD-20 Present
Joint deformity Absent
receptor, expressed on B lymphocytes.
• It produces significant improvement in RA Cardiac involve- Common Less
ment
positive patients. May be used alone or in
combination with steroid or methotrexate. Chronic constrictive Never May cause
• It is given by two TVinfusions (1 g each) 2 weeks pericarditis
apart. Chorea May be present Absent

Tocilizumab: Treatment Prophylactic DMARD


• It is an antibody directed against the IL-Greceptor. penicillin
• It acts similar to those ofTNF blockade. Sequelae Rheumatic Not so
valvular lesion
Q: What are the criteria for remission or response to (it licks the joints,
therapy of RA1 kills the heart)
A: As follows:
• No joint pain
Q: What is Sjogren syndrome? How will you
• No fatigue
confirm it?
• No joint tenderness
A: It is an autoimmune disorder characterized by dryness
• Morning stiffness <15 minutes.
of eye (keratoconjunctivitis sicca) and dryness of
• No soft tissue swelling.
mouth (xerostomia) with nonerosive arthritis. Fibro-
• ESR less than 30 mm in 1 hour. 51

sis and atrophy of the salivary glands occur. There is


At least 5 criteria must be present for at least 2
infiltration of lymphocytes and plasma cells in lacri-
consecutive months.
mal and salivary glands. It is of two types:
Q: What are the differences between rheumatic fever
• Primary: Not associated with collagen disease
(sicca syndrome).
and rheumatoid arthritis?
A: As follows: • Secondary: Associated with collagen disease
(commonly RA).

Points Rheumatic fever RA Presentation of primary Sjogren syndrome:


common in females, FM = 9: I, 40- 50 years.
Age 5-15 years 20-40 years
• Dryness of mouth and eyes.
Cause Sequelae of im- Autoimmune • Arthralgia and non-progressive arthritis.
mune response
• Raynaud phenomenon.
to Streptococcus
• Dysphagia.
beta-haemolyticu5
sore throat
• In lung-pulmonary diffusion defect and fibrosis.
• Renal (renal tubular acidosis, nephrogenic
Joints Large Peripheral small
diabetes insipidus).
joints
• Vasculitis.
Type of arthritis Fleeting Bilateral, • Others (fever, weakness, lymphadenopathy and
symmetrical
neuropathy, fit, depression).
SHORT CASES IN CLINICAL MEDICINE

• Lymphoma-increased incidence of non-Hodgkin • Anti-Ro (SS-A, positive in 70%. It can cross


B cell lymphoma. (It is associated with 40-fol.d placenta causing congenital complete heart
risk). block).
• May be associated with other autoimmune • Anti-La (SS-B).
disease like thyroid disease, myasthenia gravis,
Treatment:
PBe, autoimmune hepatitis.
• Artificial tear (hypromellose). contact lens, oral
Investigations: hygiene, artificial saliva, oral gel and chewing
1. CBC (high ESR). gum.
2. Schirmer test (a strip of filter paper is placed • Stimulation of saliva flow by sugar-free chewing
inside lower eyelid. In normal people, at least gum or lozenges may be helpful.
6 rnm is wet in 5 minutes. In Sjogren syndrome • Oral candidiasis should be treated promptly.
<5 mm). • Vaginal dryness is treated with lubricants such as
3. Biopsy of lip or salivary gland (lymphocytic and K-Yjelly.
plasma cell infiltration). • Hydroxychloroquine (2-3 mg/kg daily, may
4. Rose Bengal staining of eyes shows punctate or improve the flow of tear).
filamentary keratitis. • Treat the primary cause.
5. Antibody test: • Extraglandular and MSK manifestations may
• RA test (positive in 90%). respond to steroid. Immunosuppressive drugs
• ANA (positive in 60-70%). can be added.

Knee Joint Arthritis (Examination of Knee Joint)


Usual instructions are: o If small effusion, see Bulge sign (as follows: left
• Examine the knee joints. hand compresses the suprapatellar pouch at the
• Examine the lower limbs. What are your findings? lower end of quadriceps and the finger of right
hand is run along the groove on either side of
Proceed as follows: patella alternately; bulging, if present due to
(With permission, expose the knee and thigh, the small effusion, will not be compressed).
patient is lying in a supine position. Look carefully both • See the movement slowly and passively, ask if
front and back of the knee.) there is any pain. See the range of movement and
feel for crepitus.
1nspection: • Now examine the ligaments:
• Obvious joint swelling (right or left, localized or o Medial and lateral collateral ligaments: By just
generalized). flexing the knee and holding the leg with one
• Deformity of the joint. hand, palpate with other hand in lateral and
• Skin (red, shiny, pigmentation and scar mark). medial movement of leg.
• Back of knee (Baker cyst). o Cruciate ligaments: Flex the knee at 90 then
0
,

• Wasting of muscles of thigh or leg, swelling in calf sit over the foot of the patient lightly to fix the
(rupture Baker cyst). leg. Now pull and push the leg anteriorly and
Palpation: posteriorly by keeping the fingers on the back of
knee and feel the ligaments. Increased anterior
• Feel the temperature (compare with normal side).
movement of the leg suggests anterior ligament
• Tenderness. laxity, and increased posterior movement of leg
• Synovial thickening (suprapatellar part). suggests posterior ligament laxity.
• Examine for effusion:
o Patellar tap (fluid from suprapatellar bursa is N.B. If asked to examine the lower limbs and if obvi-
forced in joint space by squeezing the lower part ous finding is arthritis, then examine the joints
of quadriceps; then patella is pushed posteriorly mainly. If nothing obvious, perform the neuro-
by tip of two or three fingers). logical examination.
9 • RHEUMATOLOGY _

Presentation of a Case
(Painful Left Knee Joint with Effusion):
Case No.1
• The left knee joint is swollen, more marked
above the patella.
• Skin is red and shiny.
• There is a cystic swelling on the back of right
knee (Baker cyst).
• Local temperature is raised, and the joint is tender.
• Patellar tap is positive (indicates effusion).
Knee swelling (bilateral)
• There is restricted movement on left knee.

My differential diagnoses are (mention the causes Presentation of a Case


according to age): (Painful Knee Joint, Both Knee Joints):
Case No.2
In young patient, the causes are:
• Traumatic. • Describe as above in both knee joints.
• Infective arthritis (pyogenic and tuberculous).
• Juvenile idiopathic arthritis (JJA).
My differential diagnoses are (mention according to
• Reactive arthritis or Reiter syndrome.
the age):
• Haemophilic arthritis.
• Other sero nega tive arthritis (ankylosing spo ndyl itis In young patient, the causes are:
and psoriatic arthritis). • Rheumatic fever (if deformity, it is against
In middle aged or elderly patient, the causes are: rheumatic fever).
• Reactive arthritis or Reiter syndrome.
• Traumatic. • JIA.
• Infective arthritis (pyogenic and tuberculous). • Seronegative arthritis (ankylosing spondylitis and
• Gout. enteropathic arthritis).
• Pseudogout. • Rheumatoid arthritis.
• Reactive arthritis. • Psoriatic arthritis (check for skin lesion and nail
• Osteoarthrosis. change).
• RA.
In middle aged or elderly patient, the causes are:
• Osteoarthrosis.
• Gout.
• Pseudogout.
• Rheumatoid arthritis.
• Psoriatic arthritis.
• Seronegative arthritis (ankylosing spondylitis and
enteropathic arthritis).
Q: How will you investigate this patient?
A: As follows:
1. FBC: Leucocytosis (septic arthritis) and high
ESR.
2. X-ray of knee joint.
3. RAtest.
4. Serum uric add.
5. Aspiration of joint fluid and analysis:
• Physical character (straw, purulent and
haemorrhagic) .
Knee swelling (left) • Gram staining and cytology.
__ SHORT CASES IN CLINICAL MEDICINE

• Biochemistry (protein and sugar). Q: What are oligoarthritis and polyarthritis?


• CIS. A: As follows:
• Acid-fast bacillus (AFB). • When there are less than five joints or joint
• If gout suspected, uric acid crystal (to be seen groups are involved, it is called oligo arthritis
under polarising light). (pauciarticular arthritis)
• When more than five joints or joint groups are
6. Others:
involved, it is called polyarthritis.
• Blood for CIS (if septic arthritis).
• If SLE suspected (ANF, anti-double-stranded
Q: What is Charcot joint? What are the causes,
DNA).
features and treatment?
• Synovial biopsy. A: It is a chronic progressive degenerative arthropathy
characterized by deformity, osteoarthrosis and new
Read the Following Topics in Relation to bone formation, resulting from disturbance of sen-
Arthritis sory innervation of affect.edjoint. Causes are:
• Diabetes mellitus (usually involves the joints of
Q: What are the causes of acute monoarthritis? foot).
A: As follows (remember the mnemonic: GRASP-TH): • Tabes dorsalis (usually involves the joints of
• G: Gout. lower limbs-knee, ankle and hips).
• RA: Reactive arthritis. • Syringomyelia (usually involves joints of upper
• S: Septic arthritis (pyogenic). limbs-elbow, wrist and shoulder).
• P: Pseudogout. • Others: Meningomyelocele, repeated intra-
• T: Trauma, tuberculous. articular injection of steroid, leprosy and
• H: Haemophilia (in early age). amyloidosis.

N.B. In children, septic arthritis and JIA are the Features:


common causes. Also, haemophilic arthritis, • Painless joint with deformity.
leukaemia and osteomyelitis may occur. • Enlargement of joint with bony outgrowth.
• May be effusion.
Q: What are the causes of polyarthritis? • Loose body.
A: As follows: • Joint (unstable, subluxation and crepitus).
1. Infective (bacterial and viral).
2. Inflammatory: Treatment:
• Rheumatic fever. • Treatment of primary cause.
• RAand its variants. • Braces and splints may be used.
• Seronegative arthritis (ankylosing spondylitis,
Reiter syndrome, enteropathic arthritis and Differences between mechanical and inflammatory
arthritis
psoriatic arthritis).
• JIA (<16 years). Collagen disease (SLE, Mechanical Inflammatory
dermatomyositis, systemic sclerosis and Features arthritis arthritis
polyarteritis nodosa). Age Any Below 40 years
3. Degenerative (osteoarthrosis).
Onset Acute Insidious,
4. Metabolic (gout, pseudogout).
>3 months
5. Neuropathic arthropathy (Charcot joint).
6. Haematological (haemophilic arthritis and Family history Absent Usually positive

Henoch-Schonlein purpura). Exercise Worse Improved


7. Others (polymyalgia rheumatica, sarcoidosis, Rest Worse
Better
haernochrornatosis, acromegaly and hyper-
trophic osteoarthropathy). Morning stiffness Absent Present

Sleep disturbance Absent Present


N.B. In any patient with mono- or polyarthritis,
history of annular skin rash, facial palsy, diag- Neurological signs May be present Absent
nosis may be Lyme disease.
9 • RHEUMATOLOGY __

Range of movement Asymmetrical Symmetrical Maximum pain After After rest


movement
Straight leg raising Positive Normal
(SLR) Crepitus in joint Coarse crepitus Fine crepitus

Swelling and warm Absent Present ESRand CRP Normal High

Juvenile Idiopathic Arthritis (JIA/JeA)


Usual instructions are:
• Examine the legs or knee joints (patient is usually
a child or young adult).

Presentation of a case: (as described previously in


knee joint)

Q: What are your differential diagnoses?


A: As follows:
• Rheumatic fever.
• Seronegative arthritis.
• SLE.
• Viral arthritis.
• Acute leukaemia.
Deformity in elbow and small chin in JIA
Q: What relevance do you like to see?
A: As follows:
• I want to examine other joints and also spine.
• For systemic onset JIA (Still disease):
Hepatosplenomegaly, lymphadenopathy,
erythematous skin rash (present during fever
called Salmon rash), pleurisy, pericarditis and
eye (to see iri tis) .
• Detailed history to exclude other causes of arthritis
in children.

Spindle-shaped PIP in JIA

Deformity in knee and joints of upper limbs in JIA Deformity of joints in hands in JIA
~ SHORT CASES IN CLINICAL MEDICINE

Q: What investigations should be done in JIA?


A: As follows:
• Complete blood count (leucocytosis in Still
disease, may be lymphocytosis, thrombocytosis
and high ESR).
• CRP (high).
• RA test (usually negative, positive in 10% cases).
• ANA, Anti-ds-DNA if SLE is suspected.
• X-ray of the involved joint.
• Other investigations to exclude other diseases
(according to history).

Q: What is JIA?
A: It may be defined as onset of arthritis before 16 years
of age and persisting for more than 3 months.
Swollen knee joint in JIA
Q: What are the types of JlA?
A: As follows:
1. Systemic onset (Still disease)-10-15% cases.
2. Oligoarthritis (pauciarticular): It is of 2 types:
• Oligoarthritis (persistent): Common (50-60%),
four or less joints are affected, mainly knee,
ankles and wrists, in asymmetrical pattern.
Common in girls, 3 years of age. Uveitis may
OCCUI. Relatively good prognosis.
• Oligoarthritis (extended): Occurs in 25%
cases, arthritis of many joints may develop
after 6 months. This can be very destructive.

3. Polyarticular JIA: Involvement of more than four


Salmon rash in JIA
joints, 30-40% cases of JIA. It is of two types.
Q: Could it be rheumatic fever? • RA test is positive: Affects girls older than
A: Unlikely, because in rheumatic fever, there is no 8 years. Small joints of hands, wrist, ankle,
deformity of joints and wasting of muscles. More- feet etc. are involved; later larger joints are
over, rheumatic fever is diagnosed by major and involved. Can be very destructive.
minor criteria (rheumatic fever licks the joints and • RA test is negative: Affects more in girls
kills the heart). younger than 12 years, but may be in any
age. Joints involvement like RA positive type
Q: Why is there wasting of muscles in JIA? but cervical spine, temporomandibular joint
A: Disuse and release of cytokines (interleukin I, 6 and elbows may be involved. ANA may be
and TNF). positive with chronic uveitis.

Q: What are the causes of acute arthritis in children? 4. Other types of juvenile arthritis:
A: As follows: • Enthesitis-related JIA.
• Rheumatic fever. • Psoriatic arthritis.
• JIA. • Unclassified.
• Infections (bacterial and viral).
• Acute leukaemia. Q: What are the features of systemic onset JIA
• Henoch-Schonlein purpura. (Still disease)?
• Haemophilic arthritis. A: As follows:
• Reactive arthritis. • Arthritis: Involving knee, wrist and ankle. Other
• Others are sickle cell disease, psoriatic arthritis, joints may be involved.
SLE, osteomyelitis and hypermobility syndrome. • High fever: Intermittent type, may be continuous.
9 • RHEUMATOLOGY _

• Skin rash: Appears with fever and disappears Commonly occurs in young adults between 16 and
when fever subsides. These are macular or 35 years of age, rarely after 60 years.
maculopapular, Salmon pink colour rashes
(Salmon rash). Diagnostic criteria of Adult Still disease:
• Extra-articular features: Hepatosplenomegaly Each of the 4 criteria:
and lymphadenopathy (common). There may
• Quotidian fever, more than 39°C.
be pericardial effusion, pleural effusion and
• Arthralgia or arthritis (knee, wrist and ankle).
disseminated intravascular coagulation (DIC).
• In chronic cases: Micrognathia (small mandible), • Rheumatoid factor is negative.
fusion of cervical spine, and retardation of growth. • ANF is negative.

Q: How to treat JIM Plus 2 of the following:


A: As follows: • Leucocytosis >15 x 109fL (usually very high, may
be >40,000).
1. To relieve pain, NSAID.
• Evanescent macular or maculopapular rash,
2. General measures:
Salmon coloured, nonpruritic (common in chest
• Rest during pain and passive movement of
and abdomen).
the limb to prevent contracture.
• Serositis (pleurisy or pericarditis).
• Physiotherapy.
• Explanation and reassurance to the parents, • Hepatomegaly.
also to the patient. • Splenomegaly.
3. Tn severe cases: Steroid, preferably alternate • Lymphadenopathy (usually cervical, may be
generalized) .
days. Pulse methylprednisolone may be given,
followed by methotrexate (steroid may cause High fever with chill and sweating is very common. Ini-
early fusion of epiphysis resulting in short stat- tially, arthritis may be mild. Other features-abdominal
ure. Even 3 mg prednisolone daily may cause pain, myalgia and sore throat.
this effect.)
Investigations:
4. Disease modifying drugs should be given in all
cases: • CBC (low haemoglobin, high ESRand
• Methotrexate-5 mg weekly (increase the leucocytosis) .
dose gradually). • CRP (is high).
• Sulphasalazine-30-50 mg/kg, effective in • RAand ANF (are negative).
enthesitis related JlA. • Serum ferritin is very high (>10,000).
• Others-IV immunoglobulin, cydosporine • Fibrinogen level (high).
and cytotoxic drugs (cyclophosphamide,
• ASTand ALT(high).
chlorambucil and azathioprine) may be tried.
• If methotrexate fails, anti-TNF may be given Treatment:
(helpful in all cases, except in systemic onset
• NSAID (high-dose aspirin 1 g 8 hourly). Other
type, where results are variable). Etanercept
NSAIDs-indomethacin, ibuprofen may be help-
may be used.
ful. NSAID may be effective in 50% cases.
5. Orthopaedic surgery, if needed.
• If no response-high-dose prednisolone (60-100
N.B. If aspirin is used for fever or arthritis, associ- mg/day). When the fever subsides, reduce the dose
ated with viral infection like influenza below slowly. Steroid is necessary in 50% cases.
12 years of age, it may cause Reye syndrome. • Other drugs-chloroquine, hydroxychloroquine,
So, it should be avoided. methotrexate, sulphasalazine, azathioprine and
cyclophosphamide may be tried.
• In chronic case, tumour necrosis factor (TNF) antag-
A Brief Note on Adult Still Disease onists such as etanercept or anakinra (interleukin-I
It is a disease of unknown cause, characterized by high receptor agonist) may be considered.
fever, seronegative arthritis, skin rash and polyserositis.
Usually, it is diagnosed by exclusion of other diseases. N.B. Recurrent episode occurs in one-third cases.
SHORT CASES IN CLINICAL MEDICINE

Septic Arthritis
Usual instructions are: • CBC (leucocytosis).
• Examine the legs or knee joints. • CRP.
Presentation of a case: (present as in knee joint • Blood CjS.
arthritis) • X-ray of the joint involved.
• Joint fluid aspiration: See the physical character
Q: What are the differential diagnoses? (turbid), biochemistry, cytology (>S,000/mm3),
A: See in monoarthritis in knee joint arthritis (mention Gram staining and CIS.
the causes according to the age of the patient).
Q: How to treat septic arthritis?
A: As follows:
• Complete rest.
• For pain, give NSAJD.
• Antibiotic mainly flucloxacillin (2 g IV G hourly)
for 2-3 weeks, then oral (9-10 weeks). Other
antibiotic may be added.
• Joint aspiration may be necessary. Occasionally,
surgical drainage.
• Early regular passive movement.

Q: What are the causes of infective arthritis?


A: As follows:
• Bacterial: Staphylococcus aureus (causing native
Septic arthritis
arthritis), Staph. epidermidis (prosthetic arthritis),
Streptococcus, Pneumococcus, Meningococcus, H.
Q: 'What are the causes of septic arthritis? inj1.uenzae etc.
A: As follows:
• Viral: Parvovirus B19, rubella, HBV, HIV.
• Haematogenous spread from skin or other site of
infection. • Gonococcal arthritis: There is purulent arthritis
(may be fleeting) associated with pustular skin
• Direct puncture or joint aspiration or trauma.
lesion, tenosynovitis, urethral discharge and
Q: What are the risk factors for septic arthritis?
history of sexual exposure.
A: As follows: • Tubercular arthritis. One specific of tubercular
• Pre-existing joint disease (e.g. RA). arthritis is called Poncet disease, which is
• DM. associated with tenosynovitis and arthritis.
• Irnmunocomprornised state. • Lyme disease: Caused by Borrelia burgdorferi
from the bite of infected tick. Initially, there is
Q: What are the presentations of septic arthritis? skin rash called erythema chronicum migrans
A: As follows: accompanied by fever, malaise, headache,
1. Features of joints: arthralgia, lymphadenopathy. After weeks or
• Severe acute or subacute monoarthritis, may months, some patients develop features of
be polyarthritis (if there is septicaemia). meningoencephalitis, cranial or polyneuropathy,
• Joint is swollen; hot and red with severe pain cardiac problem like conduction block,
and restricted movement. myocarditis and arthritis. Confirmed by detection
2. Systemic features: High fever, malaise and of antibody titre-igM in the first month, JgG
weakness. in later months. Amoxicillin or doxycycline is
given early in the disease. Late disease should be
Q: What are the investigations done in septic arthritis? treated with IV ceftriaxone or benzyl penicillin
A: As follows: for 2-4 weeks.
9 • RHEUMATOLOGY __

Haemophilic Arthritis
Usual instructions are:
• Examine the legs or knee joints.

Presentation of a Case
(Patient is a Child or Young)

1. Inspection:
• Both knee joints are swollen, erythematous
and deformed, right one more than the left.
• Muscle wasting is present around both knee
joints, more over the right knee.
• There are few ecchymosis of various sizes
over the right thigh. Haemophilic arthritis (knee joint)

2. Palpation:
Q: What is the cause of joint deformity in
• Both knee joints are warm and tender. haemophilia?
• Patellar tap is positive. A: It is due to secondary osteoarthrosis and wasting of
• Movement is restricted. the muscles around the joint.

Q: What are the presentations of haemophilic


My differential diagnoses are:
arthritis?
• Juvenile idiopathic arthritis OIA).
A: Haemarthrosis occurs when plasma level of factor
• Rheumatic fever.
VIIJ-C is <1 %. Arthritis may be spontaneous with-
• Haemophilic arthritis (due to haemophilia A).
out trauma or may follow even minor trauma.
• Christmas disease with arthritis.
• Acute leukaemia. • Initially, tingling, abnormal sensation, stiffness
• Septic arthritis. and instability of the joint.
• Later on, the joint is red, hot, swollen and painful.
Q: Ask one history to the patient.
A: I would like to ask any history of prolonged bleed- Progression of arthritis depends on repeated haernar-
ing following any trauma, injury or tooth extraction. throsis, which leads to:
Q: Can it be rheumatic fever? • Synovium hypertrophy.
A: It can be. It is diagnosed by major and minor criteria. • Fibrotic change in synovium.
Q: Which joints are commonly involved in • Destruction of cartilage.
haemophilic arthritis? • Reduction of joint space.
A: Commonly knee, elbow, ankle and hip joints are
• Subchondral cyst formation.
involved.
• In infants, hip joint is commonly involved. • Bone shows erosion, marginal sclerosis,
• In older children, knee joint is commonly involved. osteophyte formation and ankylosis of joint.

Q: What are the radiological signs in haemophilic


arthritis?
A: As follows:
• Initially, joint space is increased and widening
of intercondylar notch occurs (in knee joint,
indicates long-standing haemorrhage).
• Later on reduction of joint space, periarticular
osteopaenia, marginal sclerosis, subchondral
cyst formation, secondary osteoarthrosis (with
osteophyte) and ankylosis of joint.
__ SHORT CASES IN CLINICAL MEDICINE

• Once acute stage is over, the patient should be


mobilized, and physiotherapy should be started
(isometric exercise, followed by active movement,
hydrotherapy).
• Arthrocentesis (aspiration from joint) is rarely
necessary.

Q: What is haemophilia?
A: It is an X-linked inherited disorder due to deficiency
of factor VIII or antihaernophilic factor, character-
ized by prolonged bleeding. Usually, female is the
carrier and maJe is the sufferer. There is high rate of
new mutations, in 30% there is no family history.

Q: What are the types ofhaemophilia?


A: Normal factor VIII level is 50-150 IU/dL. According
to severity, it is of 3 types:

! • Mild-Factor VIII is >5 IU/dL (bleeding occurs


following major injury or surgery).
• Moderate-Factor VIII is 1-5 IU/dL (bleeding
occurs following minor injury or surgery).
• Severe-Factor VIII is <1 IU/dL (spontaneous
bleeding into the joints, muscles).

Q: What is the pedigree of haemophilia?


A: As follows:
1. If father is affected:
• All daughters are carriers.
Haemarthrosis in haemophilia (wide intercondylar • All sons are normal.
notch and joint space) 2. If mother is carrier:
• 50% daughters are carriers.
Q: What are the causes of haemarthrosis? • 50% sons are sufferers.
A: As follows:
N.B. Remember the following points:
• Trauma.
• Haemophilia. • In female carrier, 1 son is affected, 1 son is
• Christmas disease. normal, 1 daughter is carrier and 1 daughter
is normal.
• Von Willebrand disease.
• Sickle cell disease. • In a female carrier, factor VIII is <50%
• Excess anticoagulant. norma], because of randomized inactivation
• Rarely, malignancy. of one X-chromosome.

Q: How to treat haemophilic arthritis? Q: Can a female be the sufferer in haernophilia?


A: As follows: A: Yes, rarely a female can suffer, because of the
• Complete rest, elevation of the affected limb and following reasons:
immobilization by splinting. • If her mother is a carrier and father is a sufferer of
• Analgesic may be given (paracetamol or haemophilia.
acetaminophen or codeine). Aspirin or other • Turner syndrome (45 XO).
NSAlDs are contraindicated (as they interfere • According to lyonization theory, there is
with platelet function and may cause excess randomized inactivation of one X chromosome
bleeding). in the developing fetus. Then the number of
• Factor VlIJ transfusion 20-30 IU/kg. Repeated affected X chromosome may be predominant.
after 12 hours and also after 24 and 36 hours Female may be affected, if normal X chromosome
(higher dose is required, if treatment is delayed). is disproportionately inactivated.
9 • RHEUMATOLOGY __

Q: How does a patient with haemophilia usually • For severe bleeding, factor VIn level should
present? be raised to at least 50 IU/dL. Treatment to
A: Depends on whether factor V(J I deficiency is mild, be continued for a period of 1 week or more.
moderate or severe. • For major surgery, factor VIII level should
• Prolonged and persistent bleeding after trauma be raised to 100 IU/ dL preoperatively and
or injury, tooth extraction. maintained above so IU/dL until healing.
• Sometimes, spontaneous bleeding may occur in Continuous infusion may be needed.
severe cases. Treatment to be continued for 7-10 days.
• Bleeding into the large joints and muscles (psoas 2. If factor Vlll is not available, cryoprecipitate,
and calf muscle) is also common. fresh frozen plasma or fresh blood can be given.
Q: What is the common site of muscular bleeding? 3. To prevent recurrent bleeding into joints and
A: Commonly in calf muscles and psoas. subsequent joint damage, factor VIII infusions
should be given regularly thrice a week starting
Q: What happens if the patient has bleeding into the from early childhood (around 2 years of age).
psoas muscle? 4. Synthetic vasopressin (desrnopressin, an ana-
A: As follows: logue of vasopressin) is given intravenous,
• Severe pain in lower abdomen. subcutaneous or intranasally. It produces 3-5-
• Paraesthesia in thigh and weakness of quadriceps fold rise in factor V1II:C and is very useful in
due to compression of femoral nerve. patient with a baseline level of factor VIII >] 0
Q: What investigations are done in haemophilia? Il.l/dl., It prevents the complications associated
with blood products. It is useful for treating
A: As follows:
bleeding episodes in mild haemophilia and as
• FBC, platelet (usually normal).
prophylaxis before minor surgery, ineffective in
• Bleeding time (normal).
severe haemophilia. It is also given forvWD, but
• Prothrombin time (normal).
not in Christmas disease.
• Clotting time (prolonged).
• APTI (prolonged). N.B. Remember the following points:
• FactorVUI:C assay (deficient or absent). • 1 unit/kg factor VIIl will raise blood level by
• vWF (normal). 2%. So, the dose of factor VIII is calculated
• Serum fibrinogen (normal). as follows:
• X-ray of involved joint (i n haemophilic arthritis). • FVIII dose = Desired factor VIII level - FVIll
baseline level x Body weight (kg) x 0.5
N.B. APIT is prolonged, which is corrected by
unit/kg.
addition of normal plasma. If not corrected
• Previously, factor VIII was prepared from
after the addition of normal plasma, more
plasma. It is now prepared by recombinant
likely there is antibody formation or the pres-
DNA technology (so, there is less risk of
ence of antiphospholipid antibody.
transfusion transmitted infection, but more
Q: Is antenatal diagnosis possible? expensive) .
A: Yes. Antenatal diagnosis may be done by molecular • Advice to the patient-trauma should be
analysis of fetal tissue obtained by chorionic villus avoided and precaution should be taken
biopsy at 11 to 12 weeks of pregnancy. before tooth extraction and surgery. The
patient should carry a special medical card in
Q; Ilow will you manage haemophilia?
which details of the disease and its treatment
A: As follows:
must be recorded.
1. Management of bleeding episode: • Half-life of factor VTIl:C is 12 hours.
• Factor VllI concentrate is given by intravenous
Q: What are the complications of haemophiliaj
infusion, twice daily as its half-life is 12 hours
A: A,> follows:
and the blood level should be maintained for
3-5 days. 1. Due to repeated haemorrhage:
• For minor bleeding, factor VIII level should be • Arthropathy due to repeated bleeding in joint
raised to 20-30 lU/dL. However, desmopressin (e.g. knee, elbow).
0.3 microgram/kg 12 hourly infusion over 20 • Atrophy or wasting of muscles secondary to
minutes may be given, which. raise factor VIII. haematoma in muscle.
~ SHORT CASES IN CLINICAL MEDICINE

• Mononeuropathy due to pressure by plasma in 1:1 ratio. But if factor Vl1I antibody
haematoma. develops, APTTis not corrected with normal plasma
• Death may occur due to intracerebral in this ratio. This case is very difficult to neat. Fol-
haemorrhage. lowing options are available:
2. Due to therapy: • High dose and frequent infusion of factor VIII
• Infections: Hepatitis A, B, C, D. Also, HIV. may be given.
• FactorVlIl antibody (up to 30% patients with • Changing the species such as porcine factor VIn
severe haemophilia). may be used.
• Factor IX may be used. It helps bypassing the
N.B. Remember the following points: inhibitors.
• Risk of viral transmission is eliminated • Recombinant factor VIla helps bypassing the
because of prior screening of donors. inhibitors.
• Infectious agents that can cause Creutzfeldt- • Factor eight inhibitor bypassing activity (FEIBA,
Jakob disease may be transmitted by blood an activated concentrate of factors II, IX and X),
and blood products. Of, prothrombin complex concentrate (PCC) ,
• All patients should receive vaccination for which contains factor VII, IXand X may be used.
HAVand HBV. • Sometimes, immunosuppressive therapy such as
• Use of recombinant factor VIII effectively steroid, azathioprine or cytotoxic drugs may be
eliminate transfusion transmitted infection. given.
• In long term, management is to eradicate inhibi-
Q: What are the causes of death? tory antibody. This can be done by using immune
A: As follows: tolerance induction (ITI). Recently, anti-CD20
• Bleeding, mainly intracerebral. monoclonal antibody (rituximab) as coadjuvant
• HIV related. is promising.
• Hepatitis due to HCV.
Christmas Disease
Q: If factor VIII antibody develops, how can it be
suspected and treated? It is also called haemophilia B. It is due to deficiency of
A: It is suspected if no response to factor VIII in factor IX. Features are like haemophilia A. It is treated
therapeutic dose. AP1T is prolonged. In normal hae- with factor IXconcentrate, half-life is 24 hours. Prophy-
mophilia, APlT is corrected by addition of normal laxis is given twice a week. Desmopressin is ineffective.

Reiter Syndrome
Usual instructions are: • Circinate balanitis and keratoderma bien nor-
• Examine the legs or knee joints. rhagica (in palm, sole and toes).
Presentation of a case: (as in knee joint, the patient is
young adult)

Q: What history do you like to take from the patient?


A: History of urethritis, diarrhoea or dysentery (Shig-
ella, Campylobacter and Yersinia), and sexual exposure
(Chlamydia) .

Q: What else do you want to examine?


A: I want to examine:
• Eyes (conjunctivitis, usually bilateral, may be
iritis in 10%).
• Back (evidence of sacroiliitis, SI).
• Foot (heel pain, Achilles tendinitis and plantar
fasciitis) . Circinate balanitis (glans penis)
Circinate balanitis (involving skin) Keratoderma blennorrhagica (palm; more blisters)

Keratoderma blennorrhagica

Conjunctivitis

Q: What is the triad of Reiter syndrome?


A: Triad:
• Arthritis.
• Conjunctivitis.
• Urethritis (non-specific).
Reiter syndrome is common in males (M:F = 15: 1),
16-35 years. Male with human leucocyte antigen
(HLA) B27 has 20% risk of suffering the disease after
Keratoderma blennorrhagica (severe)
Shigella dysentery.

Q: What is Reiter syndrome and reactive arthritis?


A: As follows:
• Reiter syndrome is a seronegative arthritis
characterized by arthritis, conjunctivitis and
urethritis.
• Reactive arthritis means when only arthritis
follows after an attack of diarrhoea, and
dysentery or sexual exposure (pathology in
one site, but affecting the joints). It is actually
a variety of Reiter syndrome. Organisms are
Keratoderma blennorrhagica Salmonella, Shigella, Campvlooacter, Chlamydia
(palm; a few blisters and thickening) and Yersinia.
__ SHORT CASES IN CLINICAL MEDICINE

Q: How does a patient with Reiter syndrome usually Q: What is keratoderma blennorrhagica?
present? A: Skin lesion characterized by vesiculopapules with
A: As follows: desquamated margin, which coalesces to form
1. History of diarrhoea, dysentery or sexual exposure. crusty plaques. Usually found in palm and sole, also
2. After 1-3 weeks, asymmetrical oligoarthritis scrotum, scalp and trunk. It may disappear and recur
involving the bigger joints (knee and ankle), also. Nail dystrophy and subungual hyperkeratosis
conjunctivitis and urethritis. may occur. It is confused with pustular psoriasis.
Treated with methotrexate or azathioprine.
3. Extra-articular features:
• Conjunctivitis and iritis. Q: How to treat Reiter syndrome?
• Achilles tendinitis and plantar fasciitis. A: As follows:
• Circinate balanitis. • Rest and N5AID for pain. Sometimes, local
• Skin rash (macular, vesicular or pustular). steroid injection.
• Keratoderma blennorrhagica (in palm, sole • Antibiotic (tetracycline and erythromycin for
or toes). non-gonococcal urethritis).
• Nail dystrophy and buccal erosion. • Usually, single attack, which settles.
• In some cases of recurrent and remitting arthritis,
• Others (rare) include pericarditis, aortic
disease-modifying drugs, such as sulphasalazine
regurgitation, conduction defect, pleurisy,
or methotrexate or azathioprine, should be given.
peripheral neuropathy and meningoencep-
• In severe cases, steroid may be given.
halitis.
• Anti-TNF therapy may be helpful in some severe
cases.
Q: What investigations should be done in this patient?
A: As follows: • Physiotherapy.
• Hb%, total count (TC), differential count (DC) Q: What are the differences between gonococcal arthri-
and ESR (high). tis and Reiter syndrome?
• Urine analysis shows pus cells, sterile on routine A: As follows:
culture (sterile pyuria).
Features Gonococcal arthritis Reiter syndrome
• CRP(high).
Involvement of Mainly of upper and Mainly lower
• RAtest and ANF (negative).
joints lower extremities extremity
• X-ray of the joints involved and 51 joint Vesicular lesion Common Uncommon
(ankylosing spondylitis). Backache Unusual Common
• If joint effusion is present, aspiration of fluid and (ankylosing
analysis (complement is high in synovial fluid). spondylitis)
• HLA-B27 positive in 70% of the cases (in affected Organism Gonococcus isolated No definite
persons). from smear organism found

Psoriatic Arthropathy
Usual instructions are:
• Examine the lower limbs or knee joints. Presentation of Case
• Examine the hands. (Hand): Case No.2
• There is swelling with gross deformity of right
Presentation of a Case (or left), second and third DIP joint and first
(Lower Limbs or Knee): Case No. 1 PIP joint. There is also (may be) deformity of
MCP (mention which one). Arthropathy is
Present as in knee joint arthritis. Look for skin rash
asymmetrical.
and nail changes.
• In nails, pitting, thickening of nail plate,
hyperkeratosis and onycholysis are seen.
Q: What else do you want to examine? • There is a small psoriatic patch at the dorsum of
A: 1want to examine the skin to see psoriatic patch and right (or left) hand.
nail changes.
9 • RHEUMATOLOGY _

My diagnosis is psoriatic arthritis.

Psoriatic arthritis with psoriatic patch


Psoriatic arthritis (DIP)with patch and nail lesion

Typical skin lesion in psoriasis

Psoriatic arthritis (DIPand MCP)

Q: What are the sites of psoriatic patch?


A: Wrist, elbow, scalp, hairline, back of ear, natal cleft,
around umbilicus, shin, knee and extensor surfaces
of limbs and scrotal region.

Q: What are the other possibilities?


A: As follows:
• Gout.
• Osteoarthrosis.

Q: Why not this is a case of RA?


Nails pitting A: Because in RA:
• PIP joints are involved. No involvement of DIP.
• Arthritis: bilateral and symmetrical.
• Skin lesion and nail changes are absent in RA.

Q: What are the types of arthritis in psoriasis?


A: There are five types:
• Asymmetrical inflammatory oligoarthritis
(of hands and foot): 40%.
• Symmetrical seronegative polyarthritis (like
rheumatoid): 25% (no rheumatoid nodule and
involvement of PIP, DIP, Mep joints, and nail
changes help to diagnose; 50% of the cases
Nails in psoriasis develop arthritis mutilans).
_ SHORT CASES IN CLINICAL MEDICINE

• SI or spondylitis: 15%. More in males, psoriatic


lesion before arthritis, and nail changes are
usually present.
• Predominant DTP joint arthritis: 15% (typical),
nail dystrophy is invariable.
• Arthritis mutilans: 5%.

Q: What are the nail changes in psoriasis?


A: Nail change is present in 85% of the cases of
psoriasis:
• Nail pitting.
• Onycholysis.
• Subungual hyperkeratosis.
X-ray in psoriatic arthritis (DIP)
• Horizontal ridging and thickening of nail.
(arthritis of DIP in fourth and fifth finger)

Q: What are the other diseases causing nail change?


A: As follows:
• Fungal infection.
• Reiter syndrome.

Q: What are the diseases affecting DIP joint?


A: As follows:
• Psoriasis.
• Gout.
• Osteoarthrosis.

X-ray in psoriatic arthritis (DIP)


Q: What are the causes of arthritis rnutilans? (severe polyarthritis of DIP and PIP)
A: As follows:
1. Rheumatoid arthritis. Q: How to treat psoriatic arthritis?
2. Psoriatic arthritis. A: As follows:
3. Juvenile idiopathic arthritis. 1. Treatment of psoriasis: Gene.ral measures, local
therapy and systemic therapy (for details see
chapter on Dermatology).
Q: What investigations do you suggest in psoriatic
arthritis? 2. Treatment of arthritis:
A: Diagnosis is clinical. For exclusion of other diseases, • NSAID (however, some NSAlDs may aggra-
following tests are done: vate psoriasis).
1. CBC (ESR may be high). • In persistent and progressive: Sulphasalazine
2. CRP (high). and methotrexate or azathioprine (these
3. RA and ANF (negative). drugs will help in both skin lesion and
4. Radiology: X-ray of the joint involved (hand, arthritis). Cyclosporine may be used.
foot and SI joint) may be normal: • Biological agents: Monoclonal antibody
• X-ray of the hand shows destruction of DIP that produces dramatic response in psoriasis
joint with deformity, punched out lytic (e.g., infliximab, etanercept or adalimumab
lesion, pencil-in-cup appearance (narrow may be given when all other drugs fail.
distal end of the proximal bone fits into the Rituximab has no role in psoriatic arthritis).
splayed out proximal end of the distal bone) • In retinoid, acitretin 20 mg daily is effective
and extensive bone resorption resulting in in both arthritis and skin lesion (avoid in
"opera glass hand" (one bone enters into young female as it is teratogenic).
its neighbouring bone like a telescope, thus • Prednisolone may be needed (sometimes
giving rise to this appearance). steroid is given intra-articularly).
• X-ray of the SI joint (ankylosing spondylitis). • PUVA is mainly for skin lesion. Sometimes
o Serum uric acid (may be high) can cause helpful in arthritis when synchronous skin
secondary gout. lesion and arthritis are present.
9 • RHEUMATOLOGY __

N.B. Remember the following points: Q: What are the common complications of anti-TNF-
• 7% of psoriasis patients develop arthritis. alpha therapy?
• Arthritis occurs in 20% cases before the onset A: As follows:
of psoriasis. Arthritis is present with current or • Flare of tuberculosis.
previous psoriasis in 70% cases. In 5% cases, • Haematological malignancy.
synchronous onset of skin lesion and arthritis.
• Worsening of heart failure.
• No skin lesion in 5% cases.
• Blood dyscrasia-anaemia, thrombocytopaenia,
• Age: Third or fourth decade (25-40 years).
leucopaenia. aplastlc anaemia.
• Equally present both in males and females (but
ankylosing spondylitis is twice more in males
Q: What are the contraindications of anti-TNF-alpha
than in females).
therapy?
• 50% of the patients with ankylosing spondylitis
A: As follows:
are HLA-B27 positive.
• Active tuberculosis (2 months anti-TB therapy
• Spontaneous remission of arthritis may occur.
should be given before starting anti-TNF-alpha
• Avoid the following drugs (which may aggravate
therapy)
psoriatic skin lesion, the exfoliative lesion):
Chloroquine and hydroxychloroquine, lithium, • Latent tuberculosis.
beta-blocker, ACE inhibitor and alcohol. • Active bacterial infection.
• CCF.
Q: Name some drugs that are effective both in psoriasis
• Septic arthritis in previous 1 year.
and arthritis.
A: Methotrexate, sulphasalazine, cydosporine, azathi- • Demyelinating disease.
oprine and biological agents. • Pregnancy and breast feeding.

Ankylosing Spondylitis
The usual instructions are: • Ask the patient to look up (patient is unable to do).
• Examine the back and relevant parts. • Ask the patient to turn either side (whole body
• Look at the patient and examine (patient may turns when the patient attempts).
be standing or lying). • Ask the patient to stand along the side of wall with
Proceed as follows (patient should be examined the back (inability to make contact of the body
in lying and in standing position): against the wall).
• See the range of movement of spine by flexion,
During lying: extension and lateral bending of body (see any
• See SLR (ask the patient to raise the straight leg. The restriction) .
patient may complain of pain on the affected side,
• Perform Schober test as follows:
which indicates root pressure due to disc prolapse.)
o Mark two points 10 cm above and 5 cm below
• Abdomen (appears protruded).
a line joining the dimple of Venus on the sacral
• Examine for evidence of sacroiliitis (compressing
promontory (the line passes along Ls' and
iliac bones).
dimple indicates the site of posterior superior
• Assess the movement of hip joint.
iliac spine).
• See the chest expansion, and examine lungs for
o Ask the patient to bend forward as far as
apical fibrosis.
possible.
• Examine heart for aortic regurgitation (AR).
o Now measure the distance between upper and
• Examine the eyes (for iritis).
lower markings.
• Examine the foot for Achilles tendinitis and plantar
fasciitis. Normally, it increases by >5 em below 50 years of age.
Now, ask the patient to stand up and perform the If <5 em, it indicates limitation of mobility of spine.
following points: Modified Schober test: Only the upper mark-
• Observe whether there is fixed thoracic kyphosis, ing is taken, which is sufficient for the test, as the
loss of lumbar lordosis and compensatory hyper- lower part is remaining fixed. It is called modified
extension of neck. Schober test.
_ SHORT CASES IN CLINICAL MEDICINE

Q: Ask some questions to the patient?


A: Low back pain with morning stiffness, any skin
lesion (psoriatic arthritis) and history of frequent
loose motion or bloody diarrhoea (inflammatory
bowel disease).

Presentation of a Case

• There is loss of lumbar lordosis, thoracic


kyphosis and compensatory hyperextension of
neck (in advanced stage, question mark 'T' or
stooped posture).
• The patient is unable to look up and unable to turn
to any side without movement of whole body.
• There is restricted movement of spine in all direc-
tions (forward, lateral and backward bending).
• SLRis positive. Protruded abdomen, thoracic kyphosis, lumbar lordosis
• The patient is unable to mak~ contact between
the occiput and the wall when standing with Q: What are your differential diagnoses?
heel and back against the wall (ask the patient A: Any cause of spondyloarthropathy may be consid-
to stand along the side of the wall with heel ered as differential diagnosis.
and back against the wall, now measure occiput • Reiter syndrome or reactive arthritis.
to wall distance. Gap indicates limitation of • Enteropathic arthritis (Crohn disease and
thoracic and cervical spine). ulcerative colitis).
• Sacroiliitis is present. • Psoriatic arthritis.
• Achilles tendinitis and plantar fasciitis are present. • Juvenile chronic arthritis (if the patient is
• Right knee joint is mildly tender and swollen <16 years old).
with slight reduction of movement. Q: What else do you want to see in ankylosing
• Schober test is positive. spondylitis?
• Abdomen is protruded. A: I want to see extra-articular features:
• Eye (to see iritis).
• Heart (AR).
My diagnosis is ankylosing spondylitis.
• Chest and lungs (restricted movement of chest,
apical fibrosis, pulmonary hypertension and cor
pulmonale).
• Foot (Achilles tendinitis and plantar fasciitis).
Q: What are the other causes of sacroiliitis?
A: Any cause of spondyloarthropathy.

Q: How to differentiate AR of rheumatic heart disease


and of ankylosing spondylitis?
A: From history and echocardiogram:
• In rheumatic heart disease, echocardiogram
shows the involvement of valve cusps (shortening,
thickening and fusion).
• In ankylosing spondylitis, echocardiogram shows
involvement of aorta or aortic root dilatation due
to aortitis.
Q: What is spondyloarthropathy or spondarthritis?
A: It is a group of inflammatory arthritis characterized
Juvenile ankylosing spondylitis by:
1. Seronegative rheumatoid factor (RA test is
negative).
2. Sacroiliitis and inflammatory spondylitis.
Q: What is the cause of chest pain in ankylosing
spondylitis?
A: Chest pain is due to costochondral junction
-
3. Asymmetrical inflammatory oligoarthritis involvement.
(lower> upper limbs, bigger joints are involved).
Q: What is the nature of arthritis in ankylosing
4. Inflammatory enthesitis.
5. Absence of nodules and other extra-articular spondylitis?
features of rheumatoid arthritis. A: Inflammatory.

6. Typical overlapping extra-articular features: Q: What are the extra-articular manifestations of


• Mucosal inflammation (such as conjunctivitis, ankylosing spondylitis?
buccal ulceration, urethritis, prostatitis, bowel A: As follows:
ulceration) . 1. Eyes: Uveitis (iritis) 25%, conjunctivitis20%
• Pustular skin lesions and nail dystrophy. (the patient may present with painful red eye
• Anterior uveitis. and photophobia).
• Aortic root fibrosis (AR, conduction defects). 2. Heart:
• Erythema nodosum. • AR (4%, due to aortitis), mitral regurgitation.
7. Familial association (high in HlA-B27). • Conduction defect (first-degree block, left
bundle branch block [LBBBJ),and atrioven-
Q: What are the diseases in spondyloarthropathy?
tricular block (AVblock), and pericarditis.
A: As follows:
3. Chest and lungs:
• Ankylosing spondylitis.
• Reiter syndrome or reactive arthritis. • Chest pain (pleuritic) and reduced chest
• Enteropathic arthritis (Crohn disease and expansion (costovertebral joint involvement).
ulcerative colitis). Apical pulmonary fibrosis, cavitation and
• Psoriatic arthritis. later on aspergilloma may occur.
• Pulmonary hypertension and cor pulmonale.
Q: What are the types of ankylosing spondylitis?
4. Prostatitis (80%), usually asymptomatic.
A: There are two types:
5. Neurological: Cauda equina syndrome (there is
• Pr.imary: Without any other association.
• Secondary: When associated with other seronega- weakness of lower limb, loss of sphincter and
tive arthritis. rectal control and saddle sensory loss).
6. Others include plantar fasciitis, Achilles
Q: What is enthesopathy? tendinitis, amyloidosis and osteoporosis.
A: Inflammation at the ligamentous attachment with
Q: What are the diagnostic criteria for AS?
erosion of adjacent bone. Healing of such lesion
A: The presence of 3 of following 4 in adults <50 years
at the junction of intervertebral disc and vertebral
with chronic back pain indicates AS:
bodies causes new bone formation, called syn-
• Morning stiffness >30 minutes.
desmophyte (hallmark of ankylosing spondylitis).
• Improvement of back pain with exercise but
Q: What is ankylosing spondylitis? What are the usual not rest.
presentations? • Awakening because of back pain during second
A: It is a chronic inflammatory seronegative spondar- half of the night only.
thritis characterized by progressive stiffening and • Alternating buttock pain.
fusion of axial skeleton. Age is second and third Alternately, Modified New York Criteria (1984)
decade, M:F = 3:1. is used:
The patient usually presents with low back pain
• Lowback pain at least 3 months duration increased
with morning stiffness, worse in the morning and
by exercise relieved by rest.
with inactivity, and pain improves after exercise.
• Limitation of lumbar spine movement in at least
Peripheral arthritis occurs in 10% cases.
2 planes.
Q: Is there any peripheral arthritis in ankylosing • Decreased chest expansion less than normal.
spondylitis? • Bilateral sacroiliitis grade 2-4.
A: May occur in juvenile onset disease, also in 20-30% • Unilateral sacroiliitis grade 3-4.
of adult cases. Hip and knee are commonly affected. Definite AS:Sacroiliitis with any clinical criteria.
__ SHORT CASES IN CLINICAL MEDICINE

Q: What are the investigations done in ankylosing • In osteitis condensans ilii, there is sclerosis on
spondylitis? the iliac site of sacroiliac joints, confuses with
A: As follows: sacroiliitis. It is a postpartum radiographic
1. X-ray of Sl joints and spine (lumbosacral, dorsal finding.
and cervical).
2. MRI of lumbosacral spine may be done in some
6ri cases (more sensitive than X-ray).
0
-0 3. CBC (ESR may be high).
....
'"S 4. CRP (may be high).
::l
(1)
5. Rheumatoid factor (negative).
~ 6. HLA-B27 (measured in blood lymphocytes; is
positive in 90% of the cases).
7. Others according to cause (barium enema
and follow through for inflammatory bowel
disease).

N.B. HLA-B27 is neither necessary nor sufficient


for diagnosis of AS. However, presence of
HLA-B27 increases the probability of AS. In
typical AS with absence of HLA-B27, there is
probability of IBD. X-ray shows calcification of longitudinal ligament and
bamboo spine appearance
X-ray shows:
• Sacroiliitis: Often the first abnormality,
beginning in the lower synovial parts of the
joints with irregularity and loss of cortical
margins, widening of the joint space and
subsequently sclerosis, narrowing and fusion.
• In thoracolumbar spi ne: Squaring of vertebrae
due to erosion. Bridging syndesmophytes
formation atthe outermost fibres ofthe annulus
results in bamboo spine appearance. There
may be ossification of anterior longitudinal
ligament or interspinous ligament and facet
joint fusion.
Radiographically, ankylosing spondylitis may be
confused with alkaptonuria, DISH, lumbar spon-
dylosis and osteitis condensans ilii. Features are as X-ray of lumbar spine shows syndesmophyte formation
follows:
• In alkaptonuria, calcification looks like AS. Q: What are the differences between syndesmophyte
But there is calcification of intervertebral disc and osteophyte?
in alkaptonuria, never in AS. A: As follows:
• DISH (Diffuseidiopathicskeletalhyperostosis):
Syndesmophyte Osteophyte
Florid new bone formation along the
Secondary to inflammatory disease Degenerative
anterolateral aspects of at least four contiguous
vertebral bodies. In DISH, calcification gives It grows longitudinally It grows horizontally
(above or below) causing outwards
the appearance of flowing wax on the anterior
bridging of adjacent vertebra
body of vertebrae. Sacroiliac and apophyseal
Due to endochondral New bone at the
joints are normal in DISH. It occurs in middle
calcification of annulus fibrosus corners of vertebra
aged and elderly, usually asymptomatic.
• In lumbar spondylosis, there is disc space Hallmark of ankylosing Hallmark of
spondylitis osteoarthrosis
narrowing and marginal sclerosis.
9 • RHEUMATOLOGY_

Q: What is the natural history of ankylosing • In patient with persistent active inflammation,
spondylitis? anti-TNF drug therapy (etanercept, adali-
A: Up to 40% may develop severe spinal restriction, murnab, infliximab) may be helpful for both
20% may have significant disability. Early periph- spinal and peripheral arthritis.
eral joint disease, especially hip joint involvement • Local steroid injection may be given for per-
indicates poor prognosis. sistent plantar fasciitis, other enthesopathies
and peripheral arthritis.
Q: How would you perform genetic counselling with
• Oral steroid may be needed for acute uveitis.
this patient?
• Other drugs: Thalidomide, pamidronate
A: If the patient is HLA-B27 positive, there is 30%
(may be used in resistant cases).
chance for the siblings to develop ankylosing
spondylitis. 3. Orthopaedic measures: May be needed for
severe hip, knee or shoulder restriction.
Q: How to treat ankylosing spondylitis?
A: As follows: Q: What is the prognosis?
A: With appropriate treatment, the prognosis is usually
1. General measures:
excellent and there is minimum disability unless
• Patient's counselling and education.
the hip joints are involved. Around 80% patients
• Exercise is the mainstay. The patient must
remain fully employed.
remain active. Swimming is the best activity.
• Prolong sitting or inactivity should be N.B. Remember the following points:
avoided. • Disease is mild in women. Peripheral arthritis is
• Physiotherapy. more in women. Spinal arthritis is less.
• Occupational therapy. • May be severe, when it affects in early age, also
2. Drug treatment: worse prognosis.
• NSAID to relieve pain. • Hip joint involvement is more in teen age.
• DMARDs-sulphasalazine or methotrexate • In the past, radiotherapy was given, which is not
(MTX) are helpful in peripheral arthritis, but used nowadays, because there is to-fold increased
has no effect on axial disease. incidence of leukaemia (MIL).

Dermatomyositis or Polymyositis
Usual instructions are: • Finger shows periungual erythema, dilated nail-
• Look at the patient or face. What are your findings? fold capillary (telangiectasia), ragged cuticles and
• Examine the hands and relevants of this patient. haemorrhage.
• Cottron sign (see below).
In the face, see the following points:
• Joint changes (swelling, tenderness and
• Skin rash (macular, maculopapular, scaly and
contracture) .
erythematous) over the cheeks and forehead. There
may be butterfly rash and erythema-resembling For relevants, see the following points:
sunburn. • Skin rash in other pa11sof the body.
• Heliotrope rash around the eyelid (commonly in • Look for evidence of proximal myopathy (both in
the upper eyelid). upper and lower limbs).
• Periorbital oedema. • Tenderness of muscles (present in 50% of the cases
• Puffy face. of polymyositis).
• Telangiectasia.
• Erythematous rash may be present on the neck and I

upper chest (in the shape ofY - called Y sign), or


Presentation of
on the shoulders (Shawl sign). Case No. 1 (Face)

In the hands, see the following points: • In the face, there is erythematous, maculopapular,
• Skin rash (macular, maculopapular, scaly and scaly rash over the cheeks and forehead.
erythematous) on the dorsum of hands (usually • There is heliotrope rash in upper eyelids of both
spares the phalanges, unlike SLEthat involves the eyes and periorbital oedema.
phalanges, but spares the knuckles).
.... SHORT CASES IN CLINICAL MEDICINE

My diagnosis is dermatomyositis. Q: What are the differential diagnoses?


A: As follows:
Q: What else do you like to see?
A: I want to see skin lesion in other parts of the body, • Drug rash.
proximal myopathy and joints. Also, I want to • Systemic lupus erythematosus (or discoid lupus
exclude other primary causes, especially malignancy erythematosus, DLE).
(such as bronchial carcinoma). • Mixed connective tissue disease (MCID).
• Systemic sclerosis.
Presentation of • Sarcoidosis.
Case No.2 (Hand) • Acne rosacea.
• Lepromatous leprosy.
• There are multiple maculopapular, scaly,
erythematous rash on the dorsum of both hands. Q: What is polymyositis or dermatomyositis?
Also, rashes with flat topped macules over the
A: Polymyositis is the non-suppurative, non-infective
knuckles (Gottron sign) are present. inflammation of skeletal muscle characterized
• In the finger, there are periungual erythema,
by necrosis, fibrosis and regeneration of mus-
ragged cuticles, haemorrhage and telangiectasia
cles. When associated with skin rash, it is called
(dilated nailfold capillary).
dermatomyositis.
• Joints are swollen, tender with contracture
deformity. Q: How do you classify dermatomyositis/polymyositis?
A: Five types:
My diagnosis is dermatomyositis. 1. Adult polymyositis.
2. Adult dermatomyositis.
3. Dermatomyositis or polymyositis associated
with malignancies.
4. Childhood dermatomyositis or polymyositis.
5. Dermatomyositis or polymyositis associated
with other autoimmune rheumatic disease
(ARD, e.g. SLE,rheumatoid arthritis and SSc).

Q: What are the causes of dermatomyositis?


A: Actual cause unknown. Probable factors are:
• Autoimmune mechanism (presence of anti-le-I
antibody and lymphocyte infiltration in muscle).
• Viral (coxsackie B, rubella and influenza have
been suggested).
• Associated with malignancy (commonest cause
Face in dermatomyositis is bronchial carcinoma in male and ovarian
carcinoma in female). Dermatomyositis can
precede any malignancy of lungs, ovary, breast,
stomach.
• Drugs causing polymyositis-penicillamine and
zidovudine.

Q: If the patient is elderly, what disease should be


excluded?
A: Malignancy (bronchial carcinoma, also carcinoma
of the breast, GIT and ovary).

Q: What are the presentations of dermatomyositis?


A: Common in female, F:M = 2:1, peak incidence is in
fifth and sixth decades.
• Muscular weakness, mainly involving the
Dermatomyositis proximal muscles (shoulder and pelvic girdle).
9 • RHEUMATOLOGY _

• Skin rash-typically affects upper eye lids with Q: If the patient is elderly, what do you think is the
erythema called "heliotrope rash" (photo- underlying cause?
sensitive, erythematous and scaly). Fiat, red rash A: May be associated with neoplasm (commonly bron-
on face and upper Hunk, erythematous rash of chial carcinoma, also carcinoma of the breast, GIT
knuckles with raised violaceous scaly eruption and ovary).
(Gottron sign). Erythematous rash also may
occur in knee, elbow, malleoli, neck and anterior Q: What is Gottron sign?
A: It is scaly, purple-red, raised, flat-topped, and
chest (often a V sign) or back and shoulder
vasculitic patches over the extensor surface of joints
(shawl sign).
and knuckles of hands. It may be found on the
• Pharyngeal. laryngeal and respiratory muscles
elbow and knee.
involvement may cause dysphagia, dysphonia
and respiratory failure. Q: What other skin lesion may resemble Gottron
• Others: Arthralgia, arthritis, myalgia and Raynaud papule?
phenomenon. A: SLE, psoriasis, lichen planus.

N.B. Remember the following points:


• In some patients, only skin rash may be present
without muscle involvement called dermatomy-
ositis sine myositis.
• Ocular involvement is rare.
• Malignancy is more in dermatomyositis, less in
polymyositis.
• The patient may present with acute renal
failure due to myoglobinuria secondary to
rhabdomyolysis.

Q: What is the pathognomonic sign in dermatomyositis?


A: Heliotrope rash. Gottron sign

Types of dermatomyositis or polymyositis

There are five types:


• Primary idiopathic polymyositis.
• Primary idiopathic dermatomyositis.
• Dermatomyositis or polymyositis associated with
neoplasia.
• Childhood dermatomyositis or polymyositis
associated with vasculitis.
• Dermatomyositis or polymyositis associated with
collagen vascular disease.

Q: What are the presentations of dermatomyositis?


Heliotrope rash A: Common in female, F:M = 3:1, and with age in
fourth and fifth decade.
Q: What is heliotrope rash? • Gradual and progressive muscular weakness and
A: It is a violaceous, purple or lilac coloured rash, wasting, mainly involving the proximal muscles
present usually over the eyelids. It may also be (shoulder and pelvic girdle).
present on the cheeks, nasal bridge and knuckles. • Skin rash (photosensitive, erythematous and scaly).
Presence of heliotrope rash is highly suggestive of • Pharyngeal, laryngeal and respiratory muscles
dermatomyositis. It is derived from the name of the involvement may cause dysphagia, dysphonia
shrub Heliotropium, which has fragmented, purple and respiratory failure.
flowers. It is more common in childhood dermato- • Others include arthralgia, arthritis, myalgia,
myositis, also occurs in adults. dysphagia and Raynaud phenomenon.
~ SHORT CASES IN CLINICAL MEDICINE

Skin rash in dermatomyositis Heliotrope rash and butterfly rash

Skin rash in dermatomyositis (leg) Dermatomyositis (contractu re)

Q: What investigations should be done in dermato-


myositis?
A: As follows:
1. FBC (ESR is high, but may be normal even in
active disease).
2. Muscle enzymes:
• Creatine phosphokinase (CPK) is very high
(the most sensitive test).
• Other enzymes-serum aldolase, SCOT,
SCPT and LDH (may be high).
3. Electromyography-abnormal almost in eVeIY
Skin rash in dermatomyositis (body) case, normal in 10%.
4. Muscle biopsy shows the following findings:
• Necrosis, swelling, vacuolation, disruption
and fragmentation of muscles.
• Infiltration of lymphocyte, plasma cells,
eosinophils and macrophages.
• Fibrosis.
• Perivascular inflammatory cells infiltration
and thickening of vessel wall.
5. Other tests:
• To exclude malignancy (chest x-ray, USC,
cr scan, MRl, PET scan, mammogram,
Skin rash in dermatomyositis (hands) gastrointestinal tract imaging).
9 • RHEUMATOLOGY _

• Other x-rays (limbs or hands) to see soft tissue


calcification. It is common in childhood
dermatomyositis.
• RA test and ANF (positive in 50% cases).
• Anti-le-I antibody is more specific. Positive
in 30% cases of dermatomyositis and 50%
cases of polymyositis (if anti-le-I antibody
is present, respiratory muscles involvement
may occur).
• MRl of muscles, to detect abnormal muscles
(helpful to take biopsy).
• Urine for myoglobin in acute cases.
Measurement of CPK is important for the
following reasons:
• Very high in dermatomyositis.
• Indicates active disease. Childhood dermatomyositis
• To see therapeutic response (reduces after
therapy).
Occasionally, CPK may be normal in derma-
tomyositis:
• If dermatomyositis is associated with internal
malignancy.
• Due to long-standing disease with atrophy of
muscles.
• Due to the presence of inhibitors in blood.

Q: What are the causes of high CPK?


A: As follows:
• exercise.
• Intramuscular injection.
• Muscle trauma or road traffic accident.
• Convulsion.
• Alcoholism.
X-ray showing calcification in dermatomyositis
• Dermatomyositis or polymyositis.
• Acute myocardial infarction (CPK-MB).
Diagnostic criteria of dermatomyositis
• Myopathy.
• Rhabdomyolysis. There are four criteria:
• Chronic liver disease (CLD). • Typical clinical history.
• Drugs-statins, busulfan, narcotics, colchicine • Increased muscle enzyme (CPK).
and chloroquine. • EMG findings.
Iso-enzymes of CPK: • Muscle biopsy.
• MM (mainly in skeletal muscle). • In a typical case with skin lesion, muscle biopsy
• MB (mainly in cardiac muscle). is not essential. But in idiopathic polymyositis,
• BB (in brain). muscle biopsy is necessary.

Q: What are the EMG findings in dermatomyositis? Q: How to treat dermatomyositis or polymyositis?
A: As follows: A: As follows:
• Spontaneous fibrillation (at rest). 1. General measures:
• Small amplitude, short duration and polyphasic • Patient education.
action potential (after voluntary activity). • Physiotherapy.
• Salvos of repetitive potential on mechanical • Bed rest in severe case.
stimulation of the nerve. • Avoidance of sunlight, use of sunscreen.
2. Medicine: • High-dose N immunoglobulin may help in
• Prednisolone 0.5-1 mg/kg daily to induce refractory cases.
remission. Continued for at least 1 month • Rituximab may be used if all fail.
after myositis is clinically and enzymatically 3. Treatment of underlying malignancy may
inactive. Then taper the dose slowly. improve the condition.
Maintenance dose is 5-7.5 mg daily. May be
Q: What is the prognosis?
required for months, even years.
A: As follows:
• If severe respiratory or pharyngeal weakness- • 5-year survival rate in treated patient is >95%;
Methylprednisolone 1 g daily for 3 days. to-year survival is 84%.
• If no response to steroid-methotrexate or • Worse prognosis if severe in presentation, delay
azathioprine may be given. of treatment, severe dysphagia or respiratory diffi-
• If methotrexate and azathioprine fail- culty, older patient and if underlying malignancy.
cyclosporine or cyclophosphamide or myco- • Dermatomyositis responds better than polymyo-
phenolate mofetil may be tried. sitis. So better prognosis.

Proximal Myopathy
Usual instructions are:
• Test for proximal myopathy.
• Examine the lower limb or upper limb.
Proceed as follows:
1. In the upper limb:
• Ask the patient to raise the arms above head
(patient is unable to do so).
• Ask the patient to sit, abduct the arm (patient is
unable), outstretch the arm in front (patient is
unable), keep the arm on side at 90° and press
against resistance.
2. In the lower limb:
• While the patient is lying flat, ask him to raise the
lower limbs (patient may be unable to do so).
• Press the knee and ask the patient to raise the
limbs against resistance.
• Ask the patient to sit on the floor in squatting
position. Then ask him to stand up (patient is
unable to do so). Proximal myopathy (wasting shown, early stage)
• Ask the patient to walk (see the gait).

Presentation of a Case rl-----------.

• There is proximal myopathy involving both the


upper and lower limbs.

My diagnosis is proximal myopathy.

Q: Ask some questions to the patient (or what are the


complaints by the patient)?
A: The patient may complain of difficulty in combing,
elirobing up stairs and rising from the chair (or ask
Proximal myopathy (wasting shown, severe) the patient about these).
Q: What are the causes of proximal myopathy? Q: What is Eaton-Lambert syndrome?
A: As follows: A: It is a para neoplastic syndrome characterized by
• Dermatomyositis or polymyositis. proximal muscle weakness and wasting due to
• Myasthenia gravis. defective acetylcholine release at the neuromuscular
junction. It commonly involves the lower limb, but
• Myasthenic myopathic syndrome (Eaton-
may involve any muscle.
Lambert syndrome).
• Myopathy (limb girdle, fascioscapulo humeral Cause:
and mitochondrial), except myotonic dystrophy.
• Cushing syndrome. • There is defect in acetylcholine release at the
neuromuscular junction, which is thought to be
• Diabetic amyotrophy.
due to an auto-antibody against prejunctional
• Thyrotoxicosis (also hypothyroidism). voltage gated calcium channel on the motor
• Polymyalgia rheumatica. nerve terminal. Small cell carcinoma of the lung
• Osteomalacia. may trigger the auto-antibody reaction.
• Hyperparathyroidism. • It is commonly due to small ceil carcinoma of the
• Periodic paralysis. lung, may be associated with or may precede the
• Alcohol and drugs (steroid, chloroquine, manifestations of carcinoma.
amiodarone, lithium and zidovudine).
• McArdle syndrome (myophosphorylase Features:
deficiency, there is stiffness and cramps of • Proximal weakness, commonly in the lower
muscle after exercise, which is hard and painful limbs but any muscle can be involved. Ptosis and
on movement). diplopia may occur in 70% cases.

Q: What are the causes of painful muscle? • Reflexes are absent or diminished.
A: As follows: • Muscle power may be increased and tendon
• Physiological (after exercise). reflexes may return after repeated activity or
sustained contraction of the relevant muscle
• Polymyositis or dermatomyositis.
(reverse of myasthenia gravis).
• Polymyalgia rheumatica.
• Patients may have autonomic dysfunction (dry
• Fibromyalgia syndrome.
mouth, constipation, impotence).
• Viral infection.
• Chronic alcoholism N.B. Abnormality of reflex, muscle power and
--r-
• Following convulsion autonomic disturbance are absent in myasthe-
• Associated with rheumatological disease. nia gravis.
• Functional.
Investigations:
Q: What are the causes of acute or sudden muscular
weakness? • EMG shows progressive incremental response
A: As follows: (reverse of myasthenia gravis where there is
decrernental response).
• Cuillain-Barre syndrome.
• Hypokalaemia and hyperkalaemia. • Antibody to P/Q type voltage gated calcium
channel (anti-VGLC)-found in 90% cases.
• Familial periodic paralysis.
• Thyrotoxic periodic paralysis.
Treatment:
• Functional (hysterical conversion reaction
[HCR)). • 3,4 diaminopyridine may be given.
• IV immunoglobulin may be helpful.
Q: What are the drugs causing myopathy?
• Pyridostigmine may give symptomatic relief.
A: Steroid, penicillamine, hydroxychloroquine or
chloroquine, statins (lovastatin), zidovudine and • Plasmapheresis.
clofibrate. • Treatment of the primary cause.
_ SHORT CASES IN CLINICAL MEDICINE

Gout
Usual instructions are:
• Examine the hands or foot.
• Look here (examiner may point a site). What is it?
(Tophus.)

Look at the following points carefully (in the hands):


• Any deformity of the joints or swelling (symmetrical
or asymmetrical).
• Presence of tophi (with skin necrosis, chalky or
pasty material).
• Look for tophi in other parts (helix of ear, extensor
Tophi in hands
and ulnar sUlface of the forearm, olecranon bursa,
dorsum of hands and fingers, and Achilles tendon).

I
Presentation of a I

Case (in Hands)

• The patient has deformity with swelling of DIP


joints, involving the fingers of right (or left)
hand and also other small joints (mention
which one).
• There is a tophus with necrosis over it with whitish
or chalky discharge. Diagnosis is gout (if tophus is
present, diagnosis is chronic tophaceous gout).
Gout affecting elbow

My diagnosis is gout.

Podagra

Acute gout

Tophus (confuses with Heberden node)


Chronic tophaceous gout
9 • RHEUMATOLOGY _

(MSUM) crystals giving rise to arthritis, tenosyno-


vitis, bursitis and tophaceous deposit. It is of two
types:
• Primary: common in males, above 40 years.
• Secondary: common in females, above 65 years.

Q: What is pseudo gout?


A: It is a crystal arthropathy caused by deposition of
calcium pyrophosphate dihydrate (CPPD). It is also
called chondrocalcinosis. Examination of synovial
fluid under polarized light shows rhomboid shaped
and positively birefringence CPPD crystals.

Q: What is tophus?
Tophus (helix of ear)
A: Nodular, hard and irregular swelling due to the
deposition of urate with inflammatory cells sur-
rounding them [tophus means chalk stone). It
Q: What relevant findings do you want to see in gout?
indicates chronic gout. Patients with severe topha-
A: As follows:
ceous disease appear to have milder or less-frequent
• Liver and spleen (lymphoma, myeloproliferative
acute attack than non-tophaceous patients.
and lymphoproli ferative disease).
Tophus may have area of necrotic or ulcerated skin
• Lymph nodes (lymphoma). over it and may exude chalky material containing
• Evidence of psoriasis (skin lesion and nail monosodium urate crystal. Tophus resolves slowly by
change). treatment of hyperuricaernia.
• Evidence of chronic renal failure (CRF and
hypothyroidism) . Tophus confuses with:
• History of hypertension, DM, ischaernic heart • Rheumatoid nodule.
disease (IHD) and drugs (see below). • Tendon xanthoma.
• Neurofibroma.
Q: Wh~ch joints are involved in gout?
• Lipoma.
A: As follows:
• MTP (metatarsophalangeal) joint of the great toe Causes of hyperuricaemia and gout:
>50% of the cases (called podagra).
1. Defect in renal excretion:
• Other joints are ankle, midfoot, knee, small joints
• Renal failure.
of hand, wrist and elbow.
• Drugs include diuretic (thiazides), low-
Q: What are the differential diagnoses of gout? dose aspirin, pyrazinamide, ethambutol,
A: The differential diagnoses of acute gout are: nicotinic acid, ethanol, cytotoxic drug and
• Traumatic. cyclosporine.
• Septic arthritis. • Hyperparathyroidism.
• Pseudogout. • Myxoedema.
• Psoriatic arthritis. • Chronic lead poisoning.
• Reiter syndrome. • Down syndrome.
• Lactic acidosis (alcohol, starvation, toxaemia
The differential diagnoses of chronic gout are: of pregnancy and type I glycogen storage
• Pseudogout. disease).
• Osteoarthrosis.
2. Excess production of uric acid:
• Psoriatic arthritis.
• Myeloproliferative disease.
• RA.
• Tuberculous arthritis. • Lymphoproliferative disease.
• Psoriasis.
Q: What is gout? What are the types? • Excess purine synthesis (hypoxanthine-
A: Gout is a disorder of purine metabolism char- guanine phosphoribosyltransferase [HGPTJ
acterized by hyperuricaemia associated with the deficiency, glucose-e-phosphatase deficiency).
deposition of monosodium urate monohydrate • Idiopathic (common)
__ SHORT CASES IN CLINICAL MEDICINE

Q: Could uric acid be normal in acute gout? 3. In severe arthritis with effusion, aspiration and
A: Yes, hyperuricaemia is common but not invariable. intra-articular steroid (methylprednisolone)
Usually, 5% of the hyperuricaernic patients develop may be given.
gout. 4. Long-term treatment:
Q: What are the precipitating factors of acute gout? • Dietary restriction: Avoid uric acid containing
A: Acute attack may be precipitated by: diet (liver, kidney, brain, red meat, cabbage,
• Sudden rise of uric add: Dietary excess or severe caul iflower, carrot and spinach). Severe calorie
dietary restriction, alcohol and diuretic (thiazide). restriction should be avoided. However, no
• Sudden fall of uric acid by allopurinol or need of strict specific dietary restriction.
uricosuric drug. • Avoid alcohol and starvation.
• Others: Trauma, surgery and severe exercise. • Reduction of weight in obese patients (slow
reduction).
Q: How does the patient usually present with acute
• Avoid precipitating drugs.
gout? How to investigate?
A: In typical acute attack: • Allopurinol or other hypouricaemic drugs.
• Severe excruciating pain, mainly in metatar- • Febuxostat, a non-purine analogue inhibitor
sophalangeal (MTP) joint of great toe, usually in of xanthine oxydase.
early morning or late night, awaking the patient • Hypouricaemic drugs should not be given
from sleep. Other joints are also involved. in acute attack may be started after several
• The joint is red, swollen and tender. weeks of acute attack.

Q: What investigations should be done in gout? Indications of hypouricaemic drug therapy:


A: Diagnosis is clinical. • Recurrent attack of gout.
• CBC: ESR (high), and leukaemia should be • Presence of tophi.
excluded. • Radiological evidence of joint damage.
• Serum uric acid.
• CRP (high). Uricosuric drugs: Probenecid, sulphinpyrazone
• To be confirmed-aspiration from joint, bursa (high-dose aspirin and indomethacin may be urico-
or tophus to see MSUM crystals under polarized suric). Uricosuric drugs may be helpful if:
microscope. It looks needle shaped and negatively • No renal failure.
birefringence crystals (but in pseudogout, calcium • Creatinine clearance >80 ml/rnin (if, <80 ml/
pyrophosphate dihydrate [CPPD I crystals are min, less effective; if, <30 ml/rnin, it has no
rhomboid and positively birefringence). It is effect).
intracellular (in leucocytes) in acute cases, may • Urine uric acid <700 mg/day on general diet.
be extracellular.
Uricosuric drugs are contraindicated in:
• Other investigations-urea and creatinine (to
exclude CRF), blood sugar, lipid profile, thyroid • Gout with overproduction of uric acid and
screening (to exclude hypothyroidism). gross uricosuria.
• Renal failure.
Q: What is the role of serum uric acid level to diag-
• Urate lithiasis.
nose acute gout?
A: It has little value as in acute gout, serum uric acid
may be normal. Asymptomatic hyperuricaemia is Brief Note on Allopurinol
more common.
Drug of choice. It inhibits the production of uric acid
Q: How to treat acute gout? by inhibiting xanthine oxidase, responsible for conver-
A: As follows: sion of xanthine and hypoxanthine to uric acid. Dose is
1. For pain: NSAlD (indomethacin, naproxen and 100-300 mg/day. In renal failure or old age, start with
diclofenac oral or suppository). low dose (100 mg). Allopurinol should not be given
2. If no response, colchicine 1 mg loading dose, in acute attack and is usually given several weeks after
then 0.5 mg every 6 hours till relief of symp- the acute attack. If given, it may precipitate acute attack.
toms. (Side effects include nausea, vomiting, To prevent, NSAJD or colchicine (0.5 mg 12 hourly)
diarrhoea and abdominal pain.) should be used together.
9 • RHEUMATOLOGY _

After allopurinol, uric acid should come down to


Asymptomatic Hyperuricaemia
half of the normal stage. Dose should be gradually
increased, until this value is achieved. Drug should be 1. It is 10 times more common.
continued indefinitely. 2. No treatment is required in majority of the cases.
3. A search should be made to find out the secondary
Q: What is uric acid nephropathy? cause of hyperuricaemia.
A: It means renal impairment is secondary to hyperu- 4. Reduction of weight, avoidance of uric acid
ricaemia. There may be chronic renal failure, acute containing diet, alcohol and smoking.
renal failure (due to obstructive uropathy) and 5. Indication of treatment in asymptomatic
nephrolithiasis. Nephropathy is due to: hyperuricaernia:
• If it is symptomatic.
• Urate deposition in renal parenchyma.
• 24 hours urine uric acid >1,100 rug/day.
• Renal tubular obstruction and uric acid stone • Strong family history of gout, urate stone or renal
formation. failure or persistent high uric acid >10.1 mg.

Systemic Lupus Erythematosus (SLE)


Usual instructions are:

• Look at the face. What are your findings? What else


do you want to see?
• Perform the general examination of the patient.

Presentation of a Case 11-----------,

• There are erythematous and scaly rash along the


butterfly distribution and also in forehead.
• Some areas of scarring or depigmentation and
telangiectasia are present.
• Face looks puffy and plethoric (due to steroid).
Butterfly rash in SLE

My diagnosis is SLE. Q: What else do you want to see?


A: 1 want to see the following features:
1. Skin rash in the other parts of body.
2. Mouth ulcer (sharply defined white patch with
red margin).
3. Alopecia (non-scarring).
4. Arthropathy or arthritis.

Q: What history do you like to take in SLE?


A: 1 want to take the history of:
• Whether the rash is aggravated on exposure to
sunlight.
• Drugs (see below) and oral pill.
• Repeated abortions in female.
• Convulsion, depression and unconsciousness or
paralysis or paresis.
• Raynaud phenomenon (present in 20% of the
cases ).
• History ofDVf or thromboembolism.
Discoid lupus erythematosus • Family history of SLE.
_ SHORT CASES IN CLI.NICAL MEDICINE

Q: What are your differential diagnoses?


A: As follows:
• SLE(orDLE).
• Dermatomyositis.
• Mixed connective tissue disease.
• Systemic sclerosis.
• Sarcoidosis.
• Drug rash.

Alopecia

Mouth ulcer in SLE (multiple)

Skin rash (SLE)

Mouth ulcer in SLE

Arthritis of hand (SLE)

Q: What is SLE?
A: It is an autoimmune chronic multisystem disease
characterized by product jon of multiple autoanti-
bodies, immune complexes and widespread
immune-mediated organ damage.
Q: From where is the name SLEderived?
A: Lupus means wolf. SLE is named because of the
erosive nature of the condition that looks like the
Cytoid body in SLE damage caused by hungry wolfs bite.
9 • RHEUMATOLOGY __

Q: What are the manifestations of SLE? • Ascites.


A: SLE is more in females, F:M = 9:1; age-in second • Others-nausea, vomiting, diarrhoea,
and third decades. Sex ratio is equal in children and abdominal pain, mild hepatosplenomegaly.
the elderly. It involves any organ of the body. 7. Haematological:
1. General features: • Anaemia (normocytic normochromic) and
• Fever. Coombs positive autoimmune haemolytic
• Arthralgia, arthritis (non-erosive), myalgia. anaemia.
• Fatigue, tiredness, weakness, malaise. • Thrombocytopaenia (often confused with
• Weight loss. ITP), leucopaenia, neutropaenia and
• Skin manifestation (80%)-malar rash, lymphopaenia.
discoid rash, photosensitivity, ulcer, vasculitic 8. Central nervous system (CNS):
lesions on the finger tips and around the nail • Fatigue, headache, poor concentration.
folds, livedo reticularis. (In discoid lupus, • Visual hallucination.
only skin is involved).
• Chorea.
• Scarring alopecia. Lupus hair (short broken
• Epilepsy.
hair above the forehead).
• Migraine.
2. Heart (involved in 25%): • Cerebrovascular disease (CVD).
• Pericarditis, pericardial effusion and • Organic psychosis.
myocarditis.
• Paralysis.
• Rarely, non-infective endocarditis called • Depression.
Libman-Sacks endocarditis (common in
• Transverse myelitis.
anti-phospholipid syndrome). Mitral regurgi-
• Lymphocytic meningitis.
tation may occur.
• Cerebellar ataxia.
• There is increased frequency of IHD and
• Cranial nerve palsy.
stroke in patients with SLE.
• Peripheral neuropathy.
3. Vascular: Raynaud phenomenon, vasculitis,
9. Kidneys:
anerial and venous thrombosis (in anti-
• Clornerulonephritis (commonly proliferative,
phospholipid syndrome) and atherosclerosis.
may be mesangial, focal or diffuse, and
4. Lungs (involved in up to 50% cases): membranous).
• Pleurisy, pleural effusion (may be bilateral) • Nephrotic syndrome.
and pneumonitis. • Renal vein thrombosis.
• Atelectasis and shrinking lung with elevation
10. Others: Lymphadenopathy, infertility, abortion,
of diaphragm (shrinking lung syndrome).
menorrhagia.
• Pulmonary fibrosis (rare, found in overlap
syndrome).
Q: What are the types of SLE?
• Increased risk of thromboembolism.
A: As follows:
• Intrapulmonary haemorrhage with vasculitis
(rare but dangerous). 1. According to presence of ARA criteria:
5. Eyes: • Possible SLE-when 2 criteria are present.
• Episcleritis, scleritis, conjunctivitis and optic • Probable SLE-when 3 criteria are present.
neuritis (blindness is uncommon). • Definite SL£-when 4 or more criteria are
• Secondary Sjogren syndrome (dry eye and dry present.
mouth) in 15% cases. • Classic SLE-when many criteria are present.
• Retinal vasculitis can cause infarct (fundos- 2. According to the severity:
copy shows white, hard exudate called cytoid • Mild:
body). o Fever.
G. CIT (rarely involved): o Arthralgia, arthritis.
• Mouth ulcer (usually painless, but may be o Rash, headache.
painful if secondary infection). o Mild pericarditis or mild pericardial
• Mesenteric vasculitis (causing small bowel effusion.
infarction or perforation). o Mild pleural effusion.
__ SHORT CASES IN CLINICAL MEDICINE

• Severe: by 'Ireponema pallidum immobilization or


o Massive pleural effusion. fluorescent treponemal antibody absorption
o Massive pericardial effusion. test).
o Renal involvement. 11.. ANA positive (in the absence of drugs causing
o CNS involvement. lupus syndrome).
o Acute vasculitis.
N.B. Presence of four or more criteria at a time
o Myocarditis.
or sequential appearance is diagnostic. Posi-
o Lupus pneumonitis.
tive anti-double-stranded DNA and anti-Srn
o Haemolytic anaemia.
(Smith) antibodies are diagnostic of SLE.
o Thrombocytopaenic purpura.
Q: What is 'disseminated lupus erythematosus'
Q: What are the diagnostic criteria of SLE? (DLE)7
A: As follows: A: It is a variant of SLE in which the disease is mainly
1. Malar rash. limited to skin, characterized by photosensitivity,
2. Discoid rash. discoid rash, erythema, scaling, follicular plug-
3. Photosensitivity. ging and telangiectasia. M:F = 1:2 (in SLE, the
ratio is 1:9). SLE may occur in 5% cases (more in
4. Oral ulcer (oral or nasopharyngeal ulcer, which
children). ANAis positive (30% cases), anti-double-
may be painless).
stranded DNA is negative and complements are
5. Arthritis (non-erosive arthritis involving two or normal.
more peripheral joints with tenderness, swelling
or effusion). Treatment:
6. Serositis: • Hydroxychloroquine 200-400 mg/day.
• Pleurisy or pleural effusion. • In severe cases, prednisolone.
• Pericarditis or pericardial effusion. • Use of sunscreen and topical steroid may help.

7. Renal involvement: Q: What is seLE (subacute cutaneous lupus


• Persistent proteinuria >0.5 g per day (in erythematosus) ?
24-hoUI urinary protein) or A: It consists of scaly red patch or circular, flat, red
• Greater than 3 + proteinuria (if total urinary lesions (confused with psoriasis), highly photosen-
protein is not performed) or sitive. Most patients have anti-Ro (SSA).
• Cellular casts (red cell, granular and tubular).
Q: What is the type of arthritis in SLE?
8. Neurological disorder: Seizure or psychosis in A: Usually non-erosive and non-deforming. Rarely,
the absence of offending drug or metabolic deformity may occur similar to RA called Jaccoud
derangement (uraemia, ketoacidosis and arthritis. Aseptic necrosis of hip and knee may occur.
electrolyte imbalance).
Q: What are the atypical features of SLE?
9. Haematological disorders:
A: As follows:
• Haemolytic anaemia or • Raynaud phenomenon.
• Leucopaenia «4,000/mm3 at two or more • Chorea.
occasions) or • Only repeated abortion.
• Lymphopaenia «1,000/mm1 at two or more • Epilepsy or cerebrovascular accident (CVA) in
occasions) or young age.
• Thrombocytopaenia (<lOO,000/mm3 in the • Psychiatric disorder.
absence of offending drug).
Q: What are the drugs causing SLE?What are the fea-
10. Immunological disorders:
tures of drug-induced SLE?How to treat such case?
• Anti-DNA antibody in abnormal titre or A: As follows:
• Anti-Sm antibody (the presence of antibody • Hydralazine (common [90% J and slow acetylator).
to Sm nuclear antigen) or • Procainamide (rapid acetylator).
• Positive anti-phospholipid antibody. • Anticonvulsant (carbamazepine and phenytoin).
• False-positive serological tests for syphilis • Phenothiazine.
(positive for at least 6 months and confirmed • Isoniazid (INH).
9 • RHEUMATOLOGY __

• Oral contraceptive pill Q: What is the type of arthritis in SLE?


• ACE inhibitor. A: Usually non-erosive and non-deforming. Rarely,
• Penicillamine. deformity may occur similar to RA called Iaccoud
• Methyldopa. arthritis. Aseptic necrosis of head of femur may
• Minocycline. occur.

Features of drug-induced SLE: Q: What is the cause of avascular necrosis of head of


• Sex ratio: equal. femur in SLE?
A: It is due to:
• Lung involvement is common, but renal and
neurological involvements are rare. • Vasculitis in SLE.
• Prolonged use of steroid therapy (actual cause
• ANA is usually positive.
of necrosis due to steroid is unknown. Probably
• Anti-double-stranded DNA is negative.
steroid causes hypertrophy of lipocytes, which
• Complements are normal. compresses blood vessels, leading to ischaemic
• Anti-histone antibody is positive in 95% of the necrosis of bone).
cases (characteristic, but not specific).
• Drugs causing SLE-like syndrome usually do not Q: What is the cause of backache in SLE?
aggravate primary SLE. A: Avascular necrosis, which is due to vasculitis and
• Clinical features and laboratory abnormalities steroid therapy. Avascular necrosis is the ischaernic
revert to normal when the offending drugs are necrosis of bone. It commonly involves head of
withdrawn. femur, also knee, shoulder and so on.

Treatment: Withdrawal of drugs. Short course of ster- Q: What investigations should be done in SLE?
oid is necessary. A: As follows:
1. CBC (anaemia, leucopaenia, thrombocytopae-
Q: What is the relation of pregnancy with SLE? nia, lymphopaenia and high ESR).
A: Pregnancy is not contraindicated. Fertility is usually 2. Urine (proteinuria, haernaturia, and RBC or
normal except in severe disease. Repeated abor- granular cast). If proteinuria is present, 24 hour
60n (due to the presence of anti-phospholipid urinary protein should be done.
antibody). still birth and intrauterine growth retar- 3. CRP (It is normal. If CRP is increased, it indi-
dation may occur. cates infection).
The disease should be in remission and pregnancy 4. ANA (positive in 95% cases. It is the most sensi-
should be avoided if there is neurological, renal and tive screening test).
cardiac abnormality. Relapse may occur in the first 5. Anti-double-stranded DNA (positive in 30-50%
trimester and in puerperium. ofthe cases. lt is highly specific for SLE).
If the mother bas 'anti-Ro antibody' (SSA), there 6. Anti-Srn (Smith) antibody (positive in 10-25%
may be congenital complete heart block of the baby cases).
due to transplacental transfer of antibody. With anti- 7. Complements (C3 and C4 are low in active
Ro or anti-La antibodies in mother, there is 2% risk of disease).
giving birth to a baby with neonatal lupus syndrome. 8. Immunoglobulin (high titre of IgM and IgG).
This is characterized by skin rash, hepatitis and fetal 9. Serum anti-phospholipid antibody (detected by
heart block. enzyme-linked immunosorbent assay, ELISA).
10. Others (these indicate presence of antiphos-
Treatment of SLE during pregnancy: Prednisolone pholipid antibody):
should be continued (avoid dexamethasone and • VORL (false positive}.
betamethasone, which are broken by placental • Platelet (low).
enzymes). • Prothrombin time (prolonged).
• APTT (prolonged, not corrected by addition
Q: What is the cause of abdominal pain in SLE? of normal pJasma).
A: As follows: "
11. Skin biopsy (from normal skin and skin lesion
• Mesenteric vasculitis or perforation of gut. for histopathology and DIF):
• Peptic ulcer as a complication of steroid. • Normal skin biopsy-immunofluorescence
• Perisplenitis or splenic infarction. test shows deposition of immune complex
• Pancreatitis. at dermoepiderrnal junction (lupus band).
__ SHORT CASES IN CLINICAL MEDICINE

Biopsy specimen of normal skin for DlF • Rituximab (anti-CD-20) may be used in some
should be sent in normal saline soaked gauge patients.
(not in formalin, as the protein is denatured • Symptomatic treatment for hypertension and
by formalin). For histopathology, biopsy oedema.
specimen is taken in formalin.
N.B. Remember the following:
12. If renal involvement: 24 hours urinary protein,
• Focal glomerulonephritis respond well to
serum urea and creatinine, creatinine clearance treatment with prednisolone 40-60 mgfday.
rate (CCR), renal biopsy.
• Diffuse and membranous lesions do not
13. lfCNS involvement: EEG, cr or MRl. respond well to steroid only. Pulse therapy
with methylprednisolone for 3 days followed
Q: What are the features of renal involvement in SLE? by maintenance with prednisolone is
rd
A: Renal involvement occurs in ] /3 of SLE; of these, necessary. Sometimes azathioprine 2-3 mg/kg
251}/o develop end-stage renal failure within 10 years. bodyweightorcyclophosphamidelOO-150mg
Lupus nephrit.is is classified histologically by WHO daily with prednisolone may be given.
criteria. • Pulse therapy: IV cyclophosphamide is more
effective than pulse methylprednisolone
Type Histology Clinical features alone. Also, combination therapy is
Type I Minimal mesangiallupus Asymptomatic sometimes more effective. Continuing TV
nephritis (LN), normal on treat.ment, quarterly for 1 year, after renal
light microscopy remission decreases the risk of renal flare.
Mild renal disease
• Prognosis is better in type I, nand v.
Type II Mesangial proliferative LN
with mesangial Q: How to diagnose renal involvement in SLE?
hypercellularity and matrix A: The patient may present with nephrotic syndrome
expansion.
or acute nephritic illness or renal failure. Renal
Type III Focal LN (involving <50% Haematuria and involvement is diagnosed by:
glomeruli). Subepithelial proteinuria 1. Urine:
deposits seen. • Proteinuria (+++), haernaturia, and granular
Type IV Diffuse LN (involving Progression to or RBC cast.
:2:50%glomeruli). There. nephrotic syndrome, • UTP (24 hours urinary total protein) >0.5g.
are segmental and global hypertension and 2. Blood:
lesions as well as active renal insufficiency. • Urea and creatinine (high) and creatinine
and chronic lesions. It is the most clearance rate (CCR): low.
Subendothelial deposits are common and most 3. Renal biopsy.
present. severe form of LN.
Q: How to treat SLE?
TypeV Membranous LN, occurs in Heavy proteinuria
A: As follows:
10-20% patients. Can occur (NS),haematuria,
in combination with type hypertension. Good 1. General measures:
III and IV. prognosis. • Explanation and education regarding the
nature of the disease.
Type VI Advanced sclerosing LN. Progressive renal
• Reassurance.
It causes sclerosis of >90% failure. Severe form.
• Psychological support.
glomeruli.
• Avoidance of UV light exposure, use of sun
blocks.
Treatment of LN: • Cardiovascular risk factors like hypertension
• Type I: No treatment. and hyperlipidaernia should be controlled.
• Type II: Benign. But sometimes, steroid may be Smoking should be stopped.
needed. 2. Drug therapy:
• Type III, TV and V: Immunosuppressive • Mild cases-NSAlD (if fever, arthralgia and
therapy with steroid and cyclophosphamide or headache).
mycophenolate mofetil is used for induction. • Chloroquine or hydroxychloroquine-
Then azathioprine and mycophenolate mofetil effective in cutaneous lesion, arthritis, arthral-
are used for maintenance. gia and serositis without organ involvement.
9 • RHEUMATOLOGY _

• Rash, synovitis or pleuropericarditis-short Methylprednisolone:


course steroid.
Indication: Severe SLE,when there is carditis, renal and
• Severe and active disease-steroid should be cerebral involvement, poor general condition.
given. Dose: 500 mg to 1 g mixed with 200 ml, 5% DA N daily
• Acute life-threatening SLE affecting kidney, for 3-5 days, followed by prednisolone 1 mg/kg for
CNS or CVS,haemolytic anaemia, thrombotic 6-10 weeks, then gradual tapering.
thrombocytopaenia-high-dose steroid and
immunosuppressive drug. Pulse methyl- Cyclophosphamide:
prednisolone (500 mg to 1 g N) daily for 3-5
Indications:
days. Cyclophosphamide (2 mg/kg N) may
be given 2-3 weekly on 6-8 occasions. • Renal lupus (classes Ill, IV and V).
• In renal involvement-mycophenolate mofetil • Severe renal disease as evidenced by (UTP > 1 g,
may be used instead of cyclophosphamide. raised serum creatinine and decreased creatinine
• In thrombocytopaenia not responding to clearance).
immunosuppressive therapy, danazol may • CNS involvement.
be used.
Dose:
Steroid (prednisolone):'
• 0.5-1 g/m2 of body surface area. It is given
Indications: monthly as a pulse therapy IV with 500 mL fluid
(every month for six cycles, then every 3 months
• In mild disease, not responding to chloroquine
for another six cycles or till the disease is inactive
and NSAlD.
for 1-2 years). Then hydroxychloroquine or
• Severe and active disease with 'involvement of
azathioprine can be started.
organ (heart, kidney, CNS and haernatological
• Alternately, cyclophosphamide 250 mg/rn" of body
abnormality).
surface may be given every 15 days for 12 doses,
Dose: followed by prednisolone as maintenance therapy.
• Oral cyclophosphamide can be given.
• Without major organ involvement-0.25-0.5
• After 12 weeks of cyclophosphamide, oral
mg/kg/day.
azathioprine (2-2.5 mg/kg daily) may be given.
• Major organ involvement-1-1.5 mg/kg/day.
• Advantages of azathioprine-no gonadal toxicity
Dose schedule of steroid (one suggested regimen): and no adverse effect on pregnancy.
• Cyclophosphamide can cause gonadal toxicity,
• Initially, full dose (45.0-60.0 mg daily) for amenorrhoea, sterility, premature menopause and
4-8 weeks, then
sometimes, leukaemia may occur. Moreover, it
• Reduce 10.0 mgweekly till 30.0 mg, then, cannot be continued in pregnancy.
• 25.0 mg/day for 1 week, then, • Cyclophosphamide is usually given with high-
• 20.0 mg/day for 1 week, then, dose prednisolone.
• 15.0 mg for 1 month, then,
(Periodic blood count should be done initially
• Reduce 2.5 mg every 2 weekly as follows-
2 weekly, then monthly.)
o First day 15.0 mg, second day 12.5 mg-
2 weeks, then
Q: What is the mechanism of haemorrhagic cystitis?
o First day 15.0 mg, second day 10.0 mg -
How to prevent this?
2 weeks, and so on.
A: Cyclophosphamide can cause haemorrhagic cystitis
o This is continued until 2nd day regimen is (more by oral route). It is due to production of
completed, then acrolein, a metabolite from cyclophosphamide
• 15.0 mgon alternate day (continue and folio,:",up). that is highly toxic to the mucosa of the urinary
When the disease activity (both clinically and biochem- bladder.
ically) disappears, steroid should be reduced slowly This can be prevented by:
over months and can be withdrawn (may be needed to • Plenty of fluid (3-4 L).
continue for 2-3 years. For lupus nephritis, treatment is • Using mesna (0, 4 and 8 hours of therapy).
usually given for 5 years). • Advise the patient to micturate frequently.
_ SHORT CASES IN CLINICAL MEDICINE

Prognosis in SLE: It is of two types:


• lu-year survival rate is 90%, but this is lower if 1. Primary: Thromboembolism without features
vital organs are involved (heart, kidney, lung, ofSLE.
CNS).
2. Secondary: Causes are-
• Mortality shows bimodal pattern. In early
• Rheumatological-SLE, DLE, RA, systemic
age, death is usually due to infection (mostly
sclerosis, Sjogren syndrome, psoriatic
opportunistic), renal or cerebral disease. In later
arthropathy, dermatomyositis.
age, accelerated atherosclerosis is common,
• Vasculitis-Behcetsyndrome, temporal arteri-
incidence of myocardial infarction is 5 times
tis, Takayasu arteritis.
more than in general population (so, risk factor
• Malignancy-solid tumours like bronchial
for atherosclerosis should be avoided, such as
carcinoma, hypernephroma, prostatic carci-
avoid smoking, control hypertension, obesity,
noma, thymoma, oesophageal carcinoma.
hyperlipidaemia etc.).
• In the long term, some patients may develop • Haernatological disease-leukaemia,
cancers, especially lymphoma. lymphoma, myelofibrosis, polycythaernia,
• Rarely, there may be deforming arthritis, myeloma, monoclonal gammopathy.
chronic progressive destruction of joints and • Infections-HN, infectious mononucleosis,
osteoarthrosis. rubella, parvovirus, hepatitis, TB, leprosy,
infective endocarditis, Klebsiella, syphilis.
Q: What is anti-phospholipid syndrome? • Drug induced-vprocainamide, phenothiazine.
A: Anti-phospholipid syndrome is characterized by • Miscellaneous-diabetes mellitus, dialysis
the presence of anti-phospholipid antibody, associ- patient.
ated with recurrent arterial or venous thrombosis,
recurrent fetal loss or thrornbocytopaenia. Some In <1% cases of anti-phospholipid syndrome, a
patients with anti-phospholipid antibody may not severe type called catastrophic anti-phospholipid
get anti-phospholipid syndrome. It is positive in syndrome may occur. In such case, there may be
SLE and may be found in many other cases, but diffuse thrombosis, thrombotic microangiopathy
may be positive without any cause, called primary and multiorgan failure. It is treated with intrave-
anti-phospholipid syndrome. nous heparin, high-dose steroid and intravenous
immune globulin and plasmapheresis.
Two types of anti-phospholipid antibody:
• Anticardiolipin antibody-may be IgG or IgM. Investigations:
IgG is more pathogenic.
1. Serum ami-phospholipid antibody (anti-cardi-
• Lupus anticoagulant-it interferes with
olipin and lupus anticoagulant).
phospholipid-dependent coagulation tests.
2. Indirect tests for anti-phospholipid antibody
In some patients, only one of these is positive and in
(anticoagulant):
others, both are positive.
• Thrornbocytopaenia.
Presence of anti-phospholipid antibody is associ-
• False positive VDRL.
ated with:
• Thrombosis, venous or arterial: 17% of stroke • Prolonged prothrombin time.
<45 years of age is thought to be due to anti- • Prolonged AMT, which is not corrected by
phospholipid antibody. addition of normal plasma.
• Recurrent abortion or intrauterine growth
Treatment anti-phospholipid syndrome:
retardation.
• Haematological-thrombocytopaenia and • With thrombosis: Warfarin should be continued
autoimmune haemolytic anaemia. for life long.
• Neurological-epilepsy, TIA, stroke, migraine • Without history of thrombosis, but with anti-
and chorea. phospholipid antibody only: Aspirin or clopidogrel
• Heart-sterile endocarditis (Libman-Sacks). may be given. Rarely warfarin may be needed.
• Renal involvement with proteinuria and lupus • In pregnancy: Low-dose aspirin and subcutaneous
nephritis (membranous). heparin should be started early in gestation.
• Skin-ulceration, livedo reticularis, gangrene and This reduces the chance of miscarriage, but pre-
necrosis of digit. eclampsia and poor fetal growth remain common.
9 • RHEUMATOLOGY _

Vasculitis
Usual instructions are: Q: What are the differential diagnoses?
• Perform the general examination. A: As follows:
• Look at the foot or hand. • Drug reaction.
• Collagen disease (SLE, rheumatoid arthritis,
Wegener granulomatosis, polyarteritis nodosa).
Presentation of a Case [1----------,
• Microscopic polyarteritis.
• There are few infarctions at the tip of the great • Cryoglobulinaemia.
and second toe of right foot, also one small • Buerger disease.
infarction at the tip of small toe of left foot. • Henoch-Schonlein purpura.
• Multiple purpuric spots and few papular lesions • Sarcoidosis.
are present on the front of both legs. • Cutaneous amyloidosis.
• There is one small ulcer above the medial
malleolus. Q: What are the diseases that mimic vasculitis?
A: As follows:
• Infective endocarditis.
My diagnosis is vasculitis.
• Meningococcaemia.
• Atrial myxoma.
• Anti-phospholipid syndrome.
• Malignancy.
• Cholesterol emboli.

Q: What is vasculitis?
A: Vasculitis syndrome is a heterogeneous group of
disorders characterized by inflammation and
necrosis of the walls of affected blood vessels, with
associated damage to the skin, kidney, lung, heart,
brain and gastrointestinal tract. Vasculitis may be
Vasculitis (toes) primary in the absence of any cause or secondary to
many inflammatory or infective diseases.

Q: What are the common causes of vasculitis?


A: Causes of vasculitis depend on the type of artery
affected.
1. Involvement of large artery:
• Giant cell arteritis, polymyalgia rheumatic.
• Takayasu arteritis.
• Behcet disease.
2. Involvement of medium-sized artery:
• Classical polyarteritis nodosa.
Vasculitis (fingers) • Buerger disease.
• Kawasaki disease.
Q: What else do you like to see?
A: I want to see any evidence of collagen disease like 3. Involvement of small artery:
rheumatoid arthritis. SLE (butterfly rash, alopecia), • Immune complex mediated-
history of fever, hypertension, heart (infective endo- Henoch-Schonlein purpura, essential
carditis), history of drug therapy. cryoglobulinaemia, cutaneous
leucocytoclastic vasculitis.
N.B. If vasculitis is associated with deformity of • ANCA associated-Wegener granulomatosis,
hand or foot joint, likely diagnosis is rheuma- Chug-Strauss syndrome, microscopic
toid arthritis with vasculitis. polyangiitis.
__ SHORT CASES IN CLINICAL MEDICINE

4. Other conditions associated with vasculitis: • Cardiac=heart failure, angina or myocardial


• Drug-induced vasculitis. infarction.
• Infective, e.g. subacute bacterial endocarditis, • Gastrointestinal-abdominal pain (due to
chronic hepatitis Band C. chronic ischaemia or mucosal inflammation),
• Collagen disease-rheumatoid arthritis, mouth ulcer, diarrhoea.
SLE, systemic sclerosis, polymyositis or • Renal-proteinuria, haernaturia, cast, acute
dermatomyosi tis. or chronic renal failure.
• Goodpasture syndrome. • Neurological-mononeuritis multiplex,
• Hypocomplementaemia. sensory or motor neuropathy, fit, psychoses,
• Serum sickness. confusion, stroke.
• Vasculitis with malignancy (paraneoplastic 3. Secondary vasculitis: Features of the primary
syndrome)-lymphoma, hairy cell disease.
leukaemia.
• Inflammatory bowel disease. Q: What investigations should be done?
Q: What are the common clinical features of vasculitis? A: As follows:
A: As follows: • CBC, ESR.
• CRP.
1. Constitutional symptoms: Fever, weight loss,
• Urine for routine examination.
malaise, arthralgia, arthritis, myalgia.
• ANA, anti-ds-DNA, pANCA, cANCA.
2. Features due multiple systems of the body: • Biopsy of involved organ for histology and
• Skin-painful palpable purpura, immunofluorescence.
vesiculobullous lesions, urticaria, cutaneous • Angiography in some cases.
nodules, ulcers, livedo reticularis, digital
gangrene. Q: How to treat vasculitis?
• Eye-episcleritis, ulcer, visual loss. A: As follows:
• Ear, nose, throat-epistaxis, nasal crust, • According to the cause.
recurrent sinusitis, deafness. • High-dose steroid and immunosuppressive drugs
• Respiratory=haernoptysis, cough, dyspnoea. are given in many cases.

Osteoarthrosis
Usual instruction:
• Look at the patient's hands; Or, examine the hands
of this patient.

Presentation of a Case I~------

• There are bony swellings at the proximal


(Bouchard nodes) and distal (Heberden nodes)
interphalangeal joints of both hands.
• DIP joints of both hands are swollen.

My diagnosis is osteoarthrosis,

Osteoarthrosis of knees

Q: Why osteoarthrosis?
A: Because Bouchard and Heberden nodes are
hallmarks of osteoarthrosis.

Q: What are Bouchard and Heberden nodes?


A: As follows:
• Bouchard node is the new bone formation at the
Osteoarthrosis of hands proximal interphalangeal joint.
9 • RHEUMATOLOGY _

• Heberden node is the new bone formation at the Q: What are the complications of osteoarthrosis?
distal interphalangeal joint. A: As follows:
• Entrapment of nerves (like ulnar nerve), carpal
Q: What else do you want to see? tunnel syndrome.
A: I want to examine other joints, especially the knee • Cervical spondylosis.
joints, ankles, cervical and lumbar spine. • Deformity.

Q: What are the x-ray findings? Q: What typical deformity may occur in the hand?
A: Osteophyte, also marginal sclerosis may be seen. A: Square hand due to subluxation of the first metacar-
pophalangeal joint.
Q: What is osteoarthrosis?
Q: In the hand, which joint is typically involved in
A: It is a degenerative disease of the synovial joint char-
osteoarthrosis?
acterized by focal loss of articular hyaline cartilage
A: Distal interphalangeal joints (DIP).
with proliferation of new bone and remodelling of
joint contour. It is not inflammatory. Q: What are the diseases that involve DIP?
A: As follows:
Q: What are the types of osteoarthrosisi
• Osteoarthrosis.
A: Two types: • Psoriatic arthritis.
• Primary. • Gout.
• Secondary-usually asymmetrical, commonly
involve the weight-bearing joints. Causes are Q: How to treat osteoarthrosis?
trauma, rheumatoid arthritis, septicarthritis, gout, A: As follows:
neuropathic joints, acromegaly, hyperparathy- • Explanation and reassurance.
roidism, chondrocalcinosis, haemochromatosis. • Patient education.
• Regular exercise.
Q: What is nodal osteoarthrosisi • Reduction of adverse mechanical factors,
A: It is a primary general ized osteoarthrosis, which e.g. weight reduction if obese, use of appropriate
is autosomal dominant and occurs mainly in footwear, walking stick etc.
middle-aged women. There may be bilateral sym- • Pain relief-paracetamol, topical or oral NSAID
metrical involvement of many joints. Bouchard and and capsaicin are used. Intra-articular C011ico-
Heberden nodes are typically found in nodal oste- steroid may be needed.
oarthrosis. There is good functional outcome ofthe • Physiotherapy may be helpful.
hands despite marked deformity. • Surgery includes osteotomy and joint replacement.

Charcot Joint
Usual instruction:
• Look at the patient's knee or ankle joint or examine
the lower limb.

Presentation of a Case rl------~

• The right ankle joint is swollen and deformed.


• Movement of the right ankle produces crepitus
sounds.

My diagnosis is Charcot joint. Charcot joint of both knee


__ SHORT CASES IN CLINICAL MEDICINE

Q: What else do you like to see or what history do you • Others-meningomyelocele, hereditary sensory
like to take? neuropathy, peripheral nerve injury etc.
A: As follows:
• Sensory examination. Q: What are the x-ray findings?
• In the upper limb, dissociated sensory loss A: As follows:
(to exclude syringomyelia). • Reduction, destruction and disorganization of
• History of lancinating pain, signs of posterior the joint.
column lesion and Argyll Robertson pupil • Loose bodies.
(to exclude tabes dorsalis). • Marginal sclerosis.
• History of diabetes mellitus.
Q: What is Charcot joint? What are the causes?
A: It is the complete destruction and disorganization
of the joint, usually secondary to loss of proprio-
ception of the joint sense.
Causes are:
• Syringomyelia (usually involves joint of upper
limb-elbow, shoulder joint).
• Tabes dorsalis (usually involves joint of lower
limb-knee, ankle).
• Diabetes mellitus (usually involves the joint of
foot).
• Leprosy.
• Repeated intra-articular injection of steroid. X-ray of Charcot joint of knee
CHAPTER 10

EXAMINATION OF THE EYE


"To all students of medicine - who listen, look, touch and reflect: may they hear, see,feel and comprehend"
-Barlow

Introduction
Usual instructions are: 7. Look for:
• Examine the fundus. What are your findings? • Acuity of vision (for distant and near vision).
• Examine the eye of this patient. • Colour vision.
• Look at the eyes. What are your findings? (May be • Field of vision.
ptosis, exophthalmos, squint, xanthelasma and • Corneal reflex.
heliotrope rash.) 8. Finally, perform fundoscopy.
For fundoscopy, proceed as follows:
Proceed as follows: • The patient should be examined either in sitting
Before examining the eyes, patient should sit at the edge or lying down in a dark room.
of the bed facing the examiner. • Ask the patient to look straight and keep his or
her eyes open.
Inspection:
• Candidates should use his or her right or left
1. Look at the face to see any facial asymmetry (in eye to examine the patient's corresponding eye.
hemiplegia and Bell palsy), myasthenic, myotonic,
• Look at the eye from at least 50 cm and check
tabetic face, and thyrotoxic or hypothyroid face.
for red reflex (start with 201 red numbers).
2. Ptosis (complete or partial), squint, exophthalmos, Opacity in media of the eye (cornea, anterior
eyebrows (fall of lateral one-third), xanthelasma, chamber, lens and vitreous) will appear as black
lid retraction, puffy face with baggy eyelids and specks or lines against red reflex.
heliotrope' rash. • Look at the following points by gradually
3. Cornea (arcus, Kayser-Fleischer [KF] ring, band coming from red numbers to black (negative)
keratopathy, ulceration and Bitot spot). and ophthalmoscope brought close to the
patient's eye:
4. Sclera (blue sclera, pigmentation, redness and che-
mosis). o Optic disc (comment on colour, margin and
cup).
5. Pupil: o Macula (one or two discs away from and a
• Size (dilatation or constriction).
little below the temporal margin of the disc).
• Shape (irregular or unequal). It appears darker than the surrounding retina
• Light reflex (direct and consensual). and in young individuals has a central yellow
• Accommodation reflex. point called "fovea centralis".
6. Movement of the eyeball: o See the nasal and temporal halves of fundus
• Keep the head of the patient fixed. Tell the and then the whole fundus.
patient: "Follow my finger". o Look for retinal vessels (remember, retinal
• See both horizontally and vertically like "H". artery has four main branches and normal
• During the movement of eyes: Look for nys- ratio of artery to vein is 2:3).
tagmus, diplopia (by asking the patient at the o Look for transparency of vessels (arteries
extreme gaze), lid lag and lid retraction. usually have a shiny central reflex stripe),
SHORT CASES IN CLINICAL MEDICINE

presence of arteriovenous (AV) nipping and


tortuosity of veins.
o Look for exudate and haemorrhage.
Present your findings systematically.
Normal optic disc: Rich in yellow colour, rest of fundus
is rich in red colour.

Normal fundus Bilateral ptosis

Optic Atrophy
Usual instructions are: • Margin: indistinct.
• Examine the fundus. What are your findings? • Some changes in retina may be present
(e.g., exudate and haemorrhage).
Presentation of Q: What is the mechanism of OA?
Case No.1 A: Degeneration of optic nerve fibres.

(Mention in which eye-right or left or both) Q: What do you think are the causes in this patient?
• The disc is pale with clear margin. A: As follows:
• There is reduction of number of capillaries in • Intracranial space-occupying lesion (SOL).
the disc (normally 7 -10 capillaries cross the disc • Secondary to optic neuritis (multiple sclerosis).
margin). • Glaucoma.
• No other changes in retina. • Optic nerve compression (by tumour and
aneurysm).

My diagnosis is primary optic atrophy (OA). Presentation of I

Case No.2
• The disc is greyish-white with indistinct margin.
• There are few exudates and haemorrhage in the
retina (mention the location).

My diagnosis is secondary OA.

Primary optic atrophy

Q: Why primary 0/\ 1


A: Because, the disc is pale with clear margin and no
change in retina.
Q: Why not secondary OA?
A: In secondary OA:
• Disc: greyish white. Secondary optic atrophy
10 • EXAMINATION OF THE EYE _

• Neurosyphilis.
• Nutritional amblyopia (vitamin B12
deficiency, tobacco and alcohol).
4. Hereditary (Priedreich ataxia, Leber OA and
DIDMOAD syndrome).
5. Ischaemic optic neuropathy (in giant cell arteritis).
6. Others: Trauma in optic nerve, Paget disease and
retinal artery occlusion.

Secondary optic atrophy Q: What is DIDMOAO syndrome?


A: It is the combination of:
Q: Why secondary OA? • DI: Diabetes insipidus.
A: Because: • OM: Diabetes mellitus.
• Disc is greyish-white with indistinct margin. • OA: Optic atrophy.
• There is exudate and haemorrhage in the retina. • 0: Deafness.
Q: What do you think is the cause of secondary OA? Q: What is glaucomatous OA?
A: Secondary to papilloedema. A: Glaucomatous OA denotes loss of disc substance;
Q: How do you investigate in a case of OA?
hence, cup is enlarged and emerging vessels appear
to bend sharply outwards.
A: As follows:
• Full blood count (FBe) and peripheral blood film
(macrocytic anaemia in vitamin B12 deficiency).
• Serological test for syphilis (venereal disease
research laboratory [VORL] and Treponema
pallidum haemagglutination TPHA).
• X-ray of the skull.
• Computed tomography (CT) or magnetic
resonance imaging (MRI) of brain.
• Other investigations according to suspicion of
causes.
Q: What are the types of OAl
Glaucomatous optic atrophy (advanced, severe)
A: There are three types:
• Primary (due to optic neuritis, compression in
the optic nerve and glaucoma).
• Secondary (to papilloedema, as in intracranial
SOL).
• Consecutive (secondary to the disease of retina:
retinitis pigmentosa, choroidoretinitis and Tay-
Sach disease).
(The term consecutive is controversial; it is actu-
aJly a secondary OA.)

Q: What are the causes of ON


A: As follows:
Glaucomatous optic atrophy (early)
1. Raised intracranial pressure due to intracranial
SOL (neoplasm, abscess and cyst).
2. Long-standing papilloedema. Q: What is the difference between optic neuritis and
3. Secondary to optic neuritis, which may be due to: optic atrophy?
• Demyelinating disease. A: Optic neuritis is an acute inflammatory process
• Drugs (ethambutol. quinine and chloroquine) affecting the optic nerve. Optic atrophy is the degen-
and toxins (methyl alcohol poisoning, lead, eration of the optic nerve head, sometimes a sequel
arsenic and cyanide poisoning). to optic neuritis.
_ SHORT CASES IN CLINICAL MEDICINE

Papilloedema (Pure)
Usual instructions are: Q: What is papilloedema? What are the stages of
papilloedema?
• Examine the fundus. What are your findings?
A: It is the swelling of the optic nerve head. Stages are:
• Early sign: Absence of spontaneous pulsation of
Presentation of a Case If-----------,
retinal veins and increased pink or red coloura-
tion of the disc.
(Mention in which eye, right or left or both)
• Blurring of disc margin, first starting in nasal side.
• There is bilateral papilloedema, more marked in
• Filling of physiological cup.
right or left eye.
• Fullness of optic disc, then elevation.
• Vessels on the disc become curved over its edge.
• May be haemorrhage surrounding the disc.
Q: What do you think is the cause in this case?
A: Possible causes are (in the absence of exudate or
Q: What are the differences between papillitis (optic
haemorrhage or other changes in retina):
neuritis) and papilloedema?
• Raised intracranial pressure due to SOL A: As follows:
(neoplasm, abscess and haematoma).
• Benign intracranial hypertension. Papillitis (optic
Features neuritis) Papilloedema
Q: Could it be malignant hypertension 1
Eye Painful, specially No pain
A: Unlikely, because in malignant hypertension there
on movement
will be other changes of hypertensive retinopathy,
such as, AV nipping, exudate and flame-shaped Visual acuity Markedly reduced Slightly reduced in
haemorrhage. advanced cases

Colour vision Affected (red) Normal


Q: Which disease is confused with papilloedema?
A: It is confused with: Central scotoma Present Absent, and there
is peripheral
• Papillitis (optic neuritis).
constriction of
• Pseudo papilloedema (developmental anomaly
visual field
of the optic nerve and there is no oedema. It has
no clinical significance). Blind spot Normal Increased
• Drusen (hyaline bodies): Pale yellow spots, Pupil Dilated, less reac- Normal
mostly in macula. tion to bright light
• Myelinated nerve fibres.

N.B. If confusion arises, it may be confirmed by


fluorescence angiography. In papilloedema,
leaking of the dye in capillaries occur.

Q: What is the visual abnormality in papilloedema?


A: Transient obscurations of vision due to temporary
impairment of retinal blood flow.

Q: What is the field defect in papilloedema'?


A: Peripheral constriction of visual field and blind
spot is enJarged.

Q: What is the important sign in papilloedema due to


raised intracranial tension?
A: Absence of venous pulsation. Papilloedema (early)
10 • EXAMINATION OF THE EYE __

Q: What is benign (idiopathic) intracranial hyperten-


sion (BIH)? What are the causes?
A: It is a syndrome of raised intracranial tension in the
absence of intracranial SOL, localizing neurological
sign or cerebrospinal fluid (CSF) out-flow obstruc-
tion. Common in females of 18-40 years of age,
obese and rarely familial. Its causes are:
• Drugs: Tetracycline, nalidixic acid, oral contracep-
tive pill, nitrofurantoin, sulphur drugs, phenytoin
and steroid (both therapy and withdrawal).
• Endocrine disorders: Hyperthyroidism, hypothy-
roidism, hypoparathyroidism, Addison disease
Papilloedema (advanced case showing blurring and Cushing syndrome.
of margin and bleeding)
• Others: Hypervitaminosis A, pregnancy,
menarche, puerperium, middle ear disease,
Q: What are the causes of papilloedema?
dural sinus thrombosis or obstruction, head
A: As follows:
injury, systemic lupus erythematosus (SLE),
1. Raised intracranial pressure due to any cause
and cryoglobulinaemia.
(such as intracranial space occupying lesion,
intracranial haemorrhage, venous sinus Q: What is the commonest presentation of BTH?
thrombosis, cerebral oedema, hydrocephalus, A: Frequent headache.
meningitis, encephalitis).
2. Hypertensive retinopathy (malignant Q: What is the complication of Bn-f1
hypertension) . A: Visual loss.
3. Benign (idiopathic) intracranial hypertension.
Q: How to diagnose Bll-I? What investigations do YOLI
4. Central retinal vein occlusion.
suggest?
5. Others:
A: Diagnosis is done by exclusion of other diseases.
• CO2 retention (hypercapnia).
Investigations are:
• Subarachnoid haemorrhage or
• X-ray of the skull (increased size of sella turcica).
cerebrovascular accident (CVA).
• Lumbar puncture shows increased CSF pressure
• Secondaries in brain.
and no other changes in CSF.
• Cuillain-Barre syndrome.
• CT or MRI of brain (to exclude other pathology).
• Graves disease.
• Sometimes, MR angiography or cerebral venog-
• Cavernous sinus thrombosis.
raphy may be done to exclude cerebral venous
• Severe anaemia. sinus thrombosis.
• Hypoparathyroidism or hypocalcaemia.
• Hypervitaminosis A. Q: How to treat BIH?
A: As follows:
Q: What are the causes of unilateral papilloedema? 1. Treatment of predisposing factors (reduce obes-
A: As follows: ity and avoid precipitating drug).
• Central retinal vein occlusion.
2. Frusernide or acetazolamide, steroid may be
• Foster-Kennedy syndrome. Infiltration in one given.
eye (lymphoma, leukaemia and sarcoidosis).
3. Frequent lumbar puncture (may be done on
• Early stage of bilateral papilloedema.
alternate days).
Q: What is Foster-Kennedy syndrome? 4. If no response:
A: It is characterized by a frontal lobe turnout com- • Lumboperitoneal shunt, especially, if
pressing optic nerve causing ipsilateral optic atrophy progressive visual loss.
but contralateral papilloedema due to raised intra- • Optic nerve sheath fenestration may be
cranial pressure. done.
~ SHORT CASES IN CLINICAL MEDICINE

Central Retinal Vein Occlusion (CRVO)


Usual instructions are: Q: What are the complications of CRVO?
• Examine the fundus. What are your findings? A: As follows:
• Neovascularization of retina (may require laser
therapy).
Presentation of I
• Rubeosis iridis.
Case No.1
• Glaucoma (rubeotic).
(Mention in which eye, right or left) • Optic atrophy.
• There are multiple scattered haemorrhages over • Permanent loss of vision.
the whole retina.
Q: What is the prognosis of CRVO?
• The veins are dilated and tortuous.
A: As follows:
• Few soft exudates and papilloedema are present.
• In mild case without macular involvement, there
is no visual disturbance.
My diagnosis is central retinal vein occlusion (CRVO). • In severe case, there is marked visual deterioration.
Q: What is the appearance of haemorrhage in CRVO? Secondary glaucoma may occur due to new
A: Stormy sunset appearance. vessels formation.

L
Presentation of I

Case No.2

(Mention in which eye, right or left)


• There are multiple scattered haemorrhages over
the right lower part of the retina.
• The veins are dilated and tortuous.
• Few soft exudates and papilloedema are present.
Central retinal vein occlusion (bleeding
with multiple exudate)
My diagnosis is branch retinal vein occlusion.

Central retinal vein occlusion (more


severe-with multiple bleeding)
Branch retinal vein occlusion (involving the upper part;
Q: What are the causes of CRVO? bleeding with some exudate)
A: As follows:
• Hypertension (the commonest cause, 70% cases).
• Atherosclerosis.
• Diabetes mellitus (DM).
• Hyperviscosity syndrome (causes are multiple
myeloma, Waldenstrom macroglobulinaemia
and polycythaemia rubra vera).
• Others: Leukaemia, hyperlipidaemia, nephrotic
syndrome, SLE,anti phospholipid syndrome,
hypercoagulable state (antithrombin III
deficiency, protein S or C deficiency, factor V Branch retinal vein occlusion (involving the lower part;
Leiden mutation) etc. only bleeding, no exudate)
10 • EXAMINATION OF THE EYE _

Q: What is the prognosis in branch retinal vein • Full blood count.


occlusion? • Blood sugar.
A: Prognosis is good, if haemorrhage does not involve • Bone marrow (to exclude multiple myeloma).
macula. • Plasma viscosity.
N.B. Superior temporal vein is commonly involved. • Plasma protein electrophoresis.
There may be quadran tic field defect. • Lipid profile.
Q: What investigations do you suggest in central Q: How to treat?
retinal vein occlusion? A: Treatment of primary cause.
A: As follows:

Hypertensive Retinopathy
Usual instructions are:
• Examine the fundus. What are your findings?

Presentation of
Case No.1
(Mention in which eye, right or left)
• Retinal arterioles are irregular, tortuous and
narrow.
• There is silver wiring with increased light reflex.
Flame-shaped haemorrhage, cotton wool spot,
• Veins are engorged, and AV nipping is present. macular star, arteriolar straightening
• There are flame-shaped haemorrhages and
cotton-wool spots and few hard exudates (if Q: Why not this is diabetic retinopathy?
hard exudate occurs around macula or fovea, it A: In diabetic retinopathy:
is called macular star). • Haemorrhage: usually dot and blot.
• No AV nipping.
My diagnosis is hypertensive retinopathy. • Hard exudate.
Q: What are the causes of AV nipping?
I
Presentation of I A: As follows:
Case No.2 • Hypertensive retinopathy (grade TI).
• Atherosclerosis.
• As in Case 1 plus papilloedema.
Q: What is cotton-wool spot and what are the causes?
A: It indicates area of retinal infarction or ischaemia.
My diagnosis is hypertensive retinopathy (malignant Its causes are:
hypertension) .
• Hypertension.
• Occasionally DM.
• Central retinal vein occlusion.
• Central retinal artery occlusion.
• Severe anaemia.
• SLE (called cytoid body).
• Others include leukaemia, infective endocarditis
(Roth spot), HIV infection.
Q: What is malignant hypertension? What does it
indicate?
A: It means papilloedema with blood pressure (BP);
mainly the diastolic> 130 mmHg with or without
Flame-shaped haemorrhage, cotton wool spot,
papilloedema
renal impairment. It indicates cerebral oedema.
__ SHORT CASES IN CLINICAL MEDICINE

Q: What is the pathogenesis of malignant • Grade III: Grade II plus cotton-wool exudate and
hypertension? flame-shaped haemorrhage.
A: Unknown, probable mechanisms are: • Grade IV: Grade III plus papilloedema (Grades
• Fibrinoid necrosis of the wall of small artery and III and IV indicate malignant hypertension).
arteriole, which results in end-organ damage.
• Dilatation of cerebral arterioles (due to auto-
regulation), which may result in cerebral
infarction or haemorrhage.
It may be reversible after treatment. In malignant
hypertension, papilloedema may occur in the
absence of haemorrhage and exudate.
Q: What are the grades of hypertensive retinopathy?
A: Four grades (Keith-Wagener-Barker classification):
• Grade I: Thickening of arterial wall, increase
tortuosity, narrowing of arteriole and increased
light reflex (silver wiring). Grade IVretinopathy
• Grade II: Grade I plus AV nipping and reduction
of arterial calibre in comparison to vein (normal Q: What are the ocular complications of hypertension 7

ratio ofV:A = 3:2). A: Retinal vein occlusion and retinopathy.

. Retinal Haemorrhage
Usual instructions are:
• Examine the fundus. What are your findings?

Presentation of the Case :f----------,

• There are few haemorrhages (mention the


location), some are flame shaped and some are
irregular in outline.
• The fundus looks pale. Retinal and preretinal haemorrhage

Q: What are the causes of retinal haemorrhage?


My differential diagnoses are (mention the causes of A: As follows:
that patient according to age): • Hypertension.
• Aplastic anaemia. • Diabetes mellitus.
• Leukaemia. • Atherosclerosis.
• Thrombocytopaenia. • CRVO due to any cause.
• Haemophilia. • Subarachnoid haemorrhage (subhyaloid with
• Christmas disease. crescentic-shaped or straight horizontal border).
• Severe anaemia due to any cause. • Raised intracranial pressure due to any cause.
• Anticoagulant drug therapy. Q: How to treat retinal haemorrhage?
A: Treatment of underlying cause.

Q: What is Roth spot? What are the causes?


A: Retinal haemorrhage with a white spot in the centre
is called Roth spot. Its causes are:
• Infective endocarditis (due to autoimmune
vasculi tis).
• Leukaemia.
• SLE.
Retinal haemorrhage in diabetic retinopathy • Severe anaemia.
10 • EXAMINATION OF THE EYE __

Roth spot with haemorrhage


Roth spot

Diabetic Retinopathy
Usual instructions are:
Q: Why this is not hypertensive retinopathy?
• Examine the fundus. What are your findings?
A: In hypertensive retinopathy, there will be:
• AV nipping.
Presentation of • Soft or cotton-wool exudate.
Case No.1 • Flame-shaped haemorrhage.

(Mention in which eye, right or left) Q: What is microaneurysm? What are the causes of
• There are microaneurysm of capillaries (mention microaneurysm of capillaries?
the location). A: Microaneurysms are the out-pouching of capillary
• Few haemorrhages (dot and blot), which are walls due to pericyte loss, appears as small red dots.
small and round shaped. It is the early sign of diabetic retinopathy. Causes are:
• Also there are hard exudates (yellowish with • Diabetes mellitus.
clear margin). • Hypertension (usually in malignant
• Disc is normal. hypertension) .
• Atherosclerosis.
My diagnosis is diabetic retinopathy, simple • Collagen diseases (SLE and polyarteritis nodosa,
background. PAN).
• Hyperviscosity syndrome (macroglobulinaemia).
• Otbers include sickle cell anaemia, leukaemia,
mycotic aneurysm and retinal vein occlusion.

N.B. Microaneurysm is always along the vessel wall,


and it may be confused with haemorrhage. It
is confirmed by fluorescence angiography.

Q: Why haemorrhage?
A: It is due to rupture of microaneurysm, resulting in
Dot and blot haemorrhage (soft exudate)
dot and blot haemorrhage.
Q: What are hard exudates?
A: These are lipid and protein residues of serous
leakage from the vessels, yellowish in colour and
irregular in outline with sharply defined margin.
Q: in simple background retinopathy, what are the
symptoms of the patient?
A: Usually asymptomatic, as the macula is spared.
Q: H ow to treat such a case?
Dot and blot haemorrhage (hard exudate) A: As follows:
_ SHORT CASES IN CLINICAL MEDICINE

• Control of diabetes mellitus, stop smoking and


control of hypertension (if any).
• Annual fundal examination.
• For leaking microaneurysm: Argon laser
photocoagulation may be done.

N.B. Simple retinopathy is rare in young if DM


< 10 years duration, but common if> 20 years
duration. In older diabetes mellitus, early
development may occur.

Preproliferative retinopathy

Presentation of a Case
(Maculopathy): Case No.2

• Findings as in Case 1 plus


• Haemorrhage and hard exudates encroaching
upon the macula (maculopathy or macular
oedema).

My diagnosis is background diabetic retinopathy with


maculopathy.
Proliferative retinopathy (advanced stage, severe)
Q: What are the symptoms of the patient?
A: Visual impairment, mainly central vision (reduc-
tion of visual acuity). Diabetic maculopathy is one
of the common causes of loss of vision in patient
with non-proliferative retinopathy.

Q: In which diabetes it is common?


A: It is common in the elderly, and is of non-insulin-
dependent diabetes mellitus (NIDDM).

Q: Ilow to treat such a case?


A: As follows: Proliferative retinopathy (early stage)
• Good control of diabetes mellitus.
• Photocoagulation may improve visual acuity in
50% cases. Presentation of I

Case No.3

(Mention in which eye, right or left)


• As in case no. 2 plus
• Multiple large blot haemorrhages and cotton-
wool spots.
• Venous dilatation, venous beading and venous
loops are also present.

My diagnosis is preproliferative diabetic retinopathy.


Q: What are the features of preproliferative
retinopathy?
Maculopathy A: As follows:
10 • EXAMINATION OF THE EYE __

• Multiple cotton-wool spots (the earliest sign).


• Venous abnormality (irregular, beading, redupli-
cation and loops).
• Multiple haemorrhage and intraretinal microvas-
cular abnormality.

Q: How to treat preproliferative retinopathy?


A: As follows:
• Good control of DM.
• Photocoagulation in some cases.
• Regular follow-up should be done.

Retinal photocoagulation
Presentation of a Case
(Proliferative Type): Case No.4

• Findings as in case no. 3 plus


• Venous dilatation, tortuosity and beading.
• There are some new vessel formations around
the disc.
• There are few vitrious haemorrhage.

My diagnosis is proliferative diabetic retinopathy.

Presentation of I

Case NO.5 Panretinal photocoagulation

• As in case no. 4. plus


Q: What are the features of proliferative retinopathy?
• There are multiple photocoagulation scars
A: As follows:
(appears like exudate, with areas of small brown
or yellowish spot of variable size and shape). • Exudative maculopathy.
• Preretinal haemorrhage.
• New vessels formation and fibrous proliferation.
My diagnosis is proliferative diabetic retinopathy, Q: What are the symptoms of the patient in prolifera-
treated with photocoagulation. tive retinopathy?
A: As follows:
• Asymptomatic without macular involvement.
• Blurring of vision with macular involvement.
Q: What is the treatment of proliferative retinopathy?
A: As follows:
• Good control of DM.
• Photocoagulation.
Q: What is the prognosis?
A: Prognosis of proliferative retinopathy is variable if
untreated. 50% may develop blindness within 5 years.
Q: In which type of OM, proliferative retinopathy is
more common?
Panretinal photocoagulation A: It is common in IDDM of long duration.
_ SHORT CASES IN CLINICAL MEDICINE

N.B. Remember the following points of retinopathy Q: What are the causes of retinal neovascularization?
inDM: A: As follows:
• The overall prevalence is 25%. It occurs in 40% • DM.
cases of IDDM and 20% of NIDDM. • Rarely malignant hypertension, sickle cell anae-
• Depends on the duration of DM. mia, sarcoidosis, hyperviscosity syndrome and
• If DM occurs before 30 years, the incidence of Behcet syndrome.
retinopathy is 50% after 10 years and 90% after
30 years. Q: What is the pathogenesis of new vessel formation?
• It is unusual for retinopathy to develop within A: Unknown, probably there is production of angio-
5 years of the onset of DM. However, 5% patients genic factors from the area of ischaemic retina.
with NIDDM have background retinopathy at Recently, a substance called "vascular endothelial
presentation. growth factor" (VEGF) has been isolated from ocu-
lar fluid, which is angiogenic.
Q: What are the complications of proliferative These new vessels are very fragile and leaking,
retinopathy? liable to rupture causing haemorrhage (intraretinal,
A: As follows: preretinal or vitreous). Serous protein leakage
• Vitreous haemorrhage. from these vessels stimulates connective tissue
• Retinal detachment. reaction called retinitis proliferans. Later on, retinal
• Glaucoma (rubeotic glaucoma). detachment may occur.
• Rubeosis iridis (new vessel formation in iris).
Q: What are the eye problems in diabetes mellitus?
A: As follows:
• Diabetic retinopathy.
• Cataract.
• Central retinal vein occlusion.
• Glaucoma.
• Rubeosis iridis.
• Retinal artery occlusion.

Q: What is the cause of visual loss in diabetes


mellitus?
A: As follows:
• Macular oedema.
• Proliferative retinopathy.
Vitreous haemorrhage
• Retinal detachment.

Q: What are the indications of laser photocoagulation


therapy in diabetic retinopathy?
A: As follows:
• Macular oedema.
• Preproliferative.
• Proliferative retinopathy.

Q: What are the complications of laser photocoagula-


tion therapy?
A: As follows:
• Vitrious haemorrhage.
• Retinal vein occlusion.
• Constriction of visual field.
Rubeosis iridis • Headache.
10 • EXAMINATION OF THE EYE _

Retinitis Pigmentosa
Usual instructions are:
• Examine the fundus. What are your findings?

Presentation of a Case :1-----------,


• There are multiple areas of black pigmentation
like bone corpuscles with variable size and shape, Retinitis pigmentosa (advanced stage, severe)
some in criss-cross pattern, at the periphery of
fundus.
• Macula is normal, and the disc is also normal.
• No exudate or haemorrhage.

My diagnosis is retinitis pigmentosa.

Q: What is the differential diagnosis?


A: Old choroidoretinitis. Retinitis pigmentosa (early stage)

Q: What is retinitis pigmentosa? What are the Q: What are the causes of retinitis pigmentosa?
features? A: As follows:
A: It is a progressive degenerative disease of the retina • Isolated or congenital. Laurence-Moon-Biedl
with pigmentary epithelium in a bone spicule syndrome (features: dwarfism, polydactyly, obes-
pattern. ity, hypogonadism and mental retardation).
• It starts in early childhood and affects • Hereditary ataxia (Friedreich ataxia).
both eyes. • Refsum disease.
• [~herited as X-linked, autosomal recessive or • Abetalipoproteinaemia.
autosomal dominant. • Usher disease.
• May occur sporadically as an isolated ocular • Alstrom syndrome.
disorder and also associated with other • Familial neuropathies.
disease.
• Degeneration primarily affects the rods, and also Q: What is the common complaint or presentation of
may involve cones. retinitis pigmentosa?
• Loss of vision by middle or advanced age. Visual A: Night blindness (due to reduction of rods at the
acuity is normal initially, and patient becomes periphery of retina).
blind by 20-30 years.
• There may be constriction of peripheral field of Q: How to treat?
vision and OA (consecutive). A: As follows:
• Cataract is a common complication causing • Regular and periodic check-up.
further visual impairment. • Treatment of primary cause.

. Ptosis
Usual instructions are:
Presentation of a Case ~I----------'
• Look at the face. What else do you want to see?
(Unilateral Ptosisof the Ilird
• Examine the eyes.
Nerve Palsy): Case No. 1
(If there is obvious ptosis, is it bilateral or unilateral,
complete or partial? Ask the patient to open the eyes, • There is complete ptosis in the right (or the left) side.
but the patient is unable.)
_ SHORT CASES IN CLINICAL MEDICINE

Q: Why ptosis and dilated pupil in the IIIrd nerve


palsy?
A: As follows:
• Ptosis is due to paralysis of levator palpebrae
superioris (supplied by ITIrd nerve).
• Dilated pupil is due to paralysis of the constrictor
of pupil (supplied by IlIrd nerve).
(Dilator of pupil is supplied by sympathetic nerve,
which when paralyzed, pupil is constricted as in
Horner syndrome).

Unilateral ptosis - IIIrd nerve palsy (left) Q: What are the causes of IIIrd nerve lesion 7
A: As follows:
1. Nuclear lesion (infarction, haemorrhage,
neoplasm and multiple sclerosis).
2. Midbrain with CVA (Weber syndrome: ipsilateral
IIIrd nerve palsy with contralateral hemiplegia
due to thrombosis of a branch of posterior
cerebral artery}.
3. Unruptured aneurysm of posterior communi-
cating artery (painful ophthalmoplegia).
Right-sided IIIrd nerve palsy 4. Others:
• Mononeuritis multiplex (DM, SLE, PAN,
Q: What else do you want to see? sarcoidosis, amyloidosis and leprosy).
A: As follows (raise the upper eye lid): • Su bacute meningitis (carci nornatous,
• Divergent squint (eyeball is fixed in downward lymphomatous, fungal, tuberculous and
and outward position). meningovascular syphilis).
• Pupil (dilated and fixed, and no reaction to direct • Raised intracranial pressure (because the
and consensual light). nerve has long and tortuous course, so likely
• No movement of eyeball in upwards, downwards to be compressed by any displacement of
and medially. brain stem).
• Loss of accommodation. • Ophthalmoplegic migraine and Guillain-
• May be diplopia (angulated, mention in which Barre syndrome.
direction) .
Q: What are the signs of I1Ird nerve lesion?
Q: What is your diagnosis? A: As follows:
A: IIIrd nerve palsy. • Ptosis (complete).
• External squint.
Q: What are the causes of unilateral ptosis? • Pupil: Dilated, no reaction to direct and
A: As follows: consensual light.
• Congenital. • Inability to move the eye upwards, downwards
• Traumatic. and medially.
• Senility. • Loss of accommodation reflex.
• Complete IIIrd nerve palsy.
Q: What investigations do you suggest in IIIrd nerve
• Horner syndrome (partial ptosis). Myasthenia
palsy?
gravis (may cause unilateral or bilateral ptosis).
A: As follows:
• Hysterical conversion reaction (HCR) ..
• Blood sugar.
N.B. IlIrd nerve palsy may be partial: pupil may be • FBC (erythrocyte sedimentation rate [ESR] is
spared. Occurs in DM and vasculitis, which high in vasculitis).
causes infarction of the nerve. Parasympa- • CT or MRJ of brain.
thetic fibres supplying the pupil remain intact • Occasionally, cerebral arteriography (if aneurysm
and spare the pupil. is suspected).
10 • EXAMINATION OF THE EYE _

Presentation of a Case rl----------,


(Bilateral Ptosis):Case No.2

• There is bilateral ptosis. Eye movement is normal.


• Pupil: Normal in size and shape, and reacts to
direct and consensual light.
• Accommodation is normal.
(See frontal baldness, wasting of temporalis and
myotonia in myotonia dystrophica.)

Bilateral ptosis (ocular myopathy)

Q: What are the causes of bilateral ptosis?


A: As follows:
• Myasthenia gravis. Tabes dorsalis (bilateral ptosis
with frontalis overaction. Argyll Robertson [AR]
pupil, OA and dorsal column lesion). Myopathy
(myotonia dystrophica and fascioscapulohumeral
myopathy).
• Ocular and oculopharyngeal myopathy.
• Congenital (rare). Bilateral Horner syndrome
Bilateral ptosis (congenital) (rare, may occur in syringomyelia).

N.B. If the patient has myopathy, no wrinkling of


forehead. In other cases, there is overaction of
frontalis with wrinkling offorehead.

Presentation of a Case
(Ocular Myopathy): Case NO.3

• There is bilateral ptosis, no compensatory wrin-


kling of forehead (may be wasting of muscles of
face and neck).
• No movement of eyeball in any direction (com-
plete ophthalmoplegia).
Bilateral ptosis (myasthenia gravis)
• Pupil is normal in size and shape, reacts to direct
and consensual light.
• Accommodation is normal.

My diagnosis is ocular myopathy or oculopharyngeal


myopathy.

Q: What is ocular myopathy?


A: It is a hereditary disorder, inherited as autosomal
dominant (AD) or sporadic, common in young,
characterized by bilateral ptosis with complete
ophthalmoplegia. It may be due to absence of soft
tissue in eyelid and periorbital region, also causes
Bilateral ptosis (senile) mild facial and neck weakness.
_ SHORT CASE'S IN CLINICAL MEDICINE

Q: What is oculopharyngeal myopathy? My diagnosis is myotonia dystrophica (see page 356).


A: Like ocular with dysphagia due to cricopharyngeal
achalasia, common in the elderly. N.B. During examination for ptosis, remember the
following points:
Presentation of a Case :1-------------, • If complete ptosis with divergent squint and
dilated pupil, Illrd nerve palsy.
(Myotonic Dystrophy): Case No.4
• Partial ptosis with apparent enophthalmos and
• There is bilateral ptosis with frontal baldness, constricted pupil, Horner syndrome.
wasting of temporalis, masseter and • Absence of movement in any direction, ocular or
sternomastoid. oculopharyngeal myopathy.
• Face is long, lean, sad and expressionless. • Frontal baldness and temporal wasting, myotonic
dystrophy.

Horner Syndrome
Usual instructions are: Q: What else do you want to see in this patient?
A: As follows:
• Examine the eye of this patient.
• Neck: Lymph nodes, scar, thyrornegaly, aneurysm
• Look at the eyes. What are your findings?
(carotid and aortic).
• Chest: Apical signs (Pancoast tumour).
Presentation of a Case • Hands: Clubbing, nicotine stain, wasting of small
(Horner Syndrome): Case No. I muscles of hands and pain.
• Evidence of syringomyelia (dissociated sensory
(Mention in which eye)
loss).
• There is partial ptosis with enophthalmos
• Evidence of lateral medullary syndrome.
(eyeball looks shrunken and inwards).
• Pupil constricted (miosis), and reacts to direct • Absence of sweating (affected side of face, whole
upper limb and upper part of trunk).
and consensual light.
• Movement of the eyeball is normal.
Q: What is Horner syndrome?
A: It is a syndrome due to lesion in the sympathetic
My diagnosis is Horner syndrome (right or left sided). pathway characterized by:
• Partial ptosis.
• Miosis (pupillary constriction), reacts to direct
and consensual light.
• Enophthalmos.
• Anhydrosis (absence of sweating in affected side
of face, whole upper limb and upper part of
trunk).

Q: Why partial ptosis, miosis and enophthalmos in


Horner syndrome (bilateral) J lorner syndrome?
A: As follows:
• Partial ptosis is due to paralysis of the upper
tarsal muscles.
• Miosis is due to paralysis of the dilator of pupil.
• Enophthalmos is due to paralysis of Muller
muscle.

Q: What are the causes of Horner syndrome (accord-


ing to the site of lesion)?
Horner syndrome (left) A: As follows:
10 • EXAMINATION OF THE EYE __

l. Tl lesion: Q: What investigations should be done in Horner


• Pancoast tumour. syndrome?
• Trauma to brachial plexus. A: As follows:
• Cervical rib. • Chest x-ray.
2. Neck (sympathetic lesion): • X-ray of cervical spine.
• Trauma. • CT scan or MRJ of brain.
• Neck surgery. • Other investigations, according to suspicion of
• Cervical sympathectomy. causes.
• Lymphoma. Q: Discuss the cervical sympathetic pathway.
• Thyroid carcinoma. A; It originates from the sympathetic nucleus in
3. Brain stem lesion: hypothalamus and passes through the brain stem
• Vascular (lateral medullary syndrome). to the lateral horn of C, and Tl segment of spinal
• Multiple sclerosis. cord. From there, preganglionic fibres emerge and
4. Cervical cord lesion (bilateral Horner syndrome pass to sympathetic ganglia (usually superior cervi-
may occur): cal ganglia). Then the post-ganglionic fibres pass in
• Syringomyelia. the carotid sheath with internal carotid artery (lCA),
enter the skull along with it and in the cavernous
• Spinal cord tumour (glioma and
sinus, and join with the ophthalmic division of the
ependymoma) .
Vth nerve. Then it enters into the orbit, via short
5. Migraine (temporary Horner syndrome may
ciliary nerve, and supply the dilator pupillae, Muller
occur).
muscle and sweat gland on the side of face.

Argyll Robertson Pupil (AR Pupil)


Usual instructions are: • Neurosyphilis, as in tabes dorsalis, and general
paralysis of the insane (CPI), the commonest
• Examine the eye of this patient.
cause.
• Look at the eyes. What are your findings?
• DM.
Q: What are the features of ARpupil?
Presentation of a Case
A: As follows:
(AR pupil is always bilateral) • Pupil is small, irregular and unequal.
• Both the pupils are irregular, small and unequal. • Loss of light reflex, but persistence of
• There is loss of light reflex (both direct and accommodation reflex.
consensual) . • Impaired response to mydriatic drug.
• Accommodation reflex is normal. • Iris shows patchy atrophy and depigmentation.
• Iris is patchy atrophy and depigmentation.
Q: What is site of lesion ofAR pupil?
A: Tectum of midbrain proximal to oculomotor
My diagnosis is AR pupil. nucleus, around aqueduct of sylvius.
Q: The patient has AR pupil. What else do you want
to see?
A: Other features of tabes dorsalis need to be noted:
• Wrinkling of forehead with bilateral ptosis (due
to compensatory overaction of frontalis).
• Flaccid paraplegia.
• Loss of knee and ankle jerks, plantar is flexor (but
in taboparesis, may be extensor).
ARpupil
• Posterior column lesion (loss of vibration and
Q: What are the causes of AR pupil? position sense).
A: As follows: • Loss of deep pain in Achilles tendon.
_ SHORT CASES IN CLINICAL MEDICINE

• Hypotonia. • Charcot joints.


• Bladder sensation is lost (urinary retention and • Fundoscopy shows OA.
overflow incontinence).
Q: What is the lesion in heart linked with AR
• Gait may be stamping or steppage gait. pupil?
• Romberg sign is positive. A: Aortic regurgitation.

Different Types of Pupil


Q: wha; are the causes of constricted pupil (miosis)? Q: What are the causes of absent accommodation
A: As follows: reflex, but normal light reflex?
• Horner syndrome. A; As follows:
• AR pupil. • Cortical lesion (cortical blindness).
• Pontine haemorrhage. • Parkinson disease by encephalitis lethargica.
• Senility (pupil in old age tends to be small, and (Parkinson disease due to encephalitis lethargica,
may be irregular}. the pupils react to light, but not to accommodation,
called reverse AR pupil).
• Morphine.
• Miotic drugs (pilocarpine and physostigmine). Q: What is the afferent and efferent pathway of light
• Poisoning (organophosphorous, opium and reflex!
trichloroethanol). A: Afferent is through optic nerve and efferent is
through the IIIrd cranial nerve.
Q: What arc the causes of dilated pupil (mydriasis)?
A: As follows: Q: What is Marcus-Gunn phenomenon or pupil?
• IIlrd nerve palsy. A: In this disorder, direct reflex is brisk on exposure
• Holmes-Adie pupiL to light. However, when light is alternately focused
from one eye to other, the pupil on the affected
• Optic nerve lesion (optic neuritis or retrobulbar
side dilates slowly, when exposed to light. Found in
neuritis).
optic neuritis.
• Mydriatic drugs (atropine and homatropine).
• Other drugs (tricyclic antidepressant, belladonna The mechanism is as follows:
and amphetamine). • When light is focused on healthy eye, a rapid
• Datura poisoning. pupillary constriction occurs in both eyes. When
light is focused again on the affected eye, the
• Fixed dilated pupil (occurs in brain death, also in
eye fails to transmit the message to continue the
deep coma).
constriction as quickly as normal. As a result,
Q: what are the causes of unequal pupil (anisocoria)? pupils have time to recover and dilate, despite the
A: As follows: light shining on abnormal eye.

• Physiological anisocoria in normal eye (20%). N.B. Read the following points in relation to pupil of
• Iritis. unconscious patients:
• Syphilis. • One pupil is dilated, fixed to light: Indicates
• Holrnes-Adie pupil. herniation of uncus of temporal lobe (coning)
• Mydriatic drug in one eye. and compression of Illrd nerve.
• Blindness or amblyopia in one eye (pupil is large • Pinpoint fixed pupil: Indicates pontine
in affected eye). haemorrhage.
• Cerebrovascular accident. • Midpoint and slightly dilated pupil indicates
damage to the midbrain with interruption of
Q: What are the causes of absent light reflex, but pupillary light reflex.
present accommodation reflex? • Midpoint pupil that reacts to light indicates coma
A: As follows: of metabolic origin and central nervous system
• AR pupil. (CNS) depressant drugs.
• Midbrain lesion. • Fixed dilated pupil (bilateral): Indicates brain
• Ciliary ganglion lesion. death and deep coma.
10 • EXAMINATION OF THE EYE __

Holmes-Adie Pupil
Usual instructions are: pupil may constrict slowly. If then exposed to dark
for long time, the pupil dilates slowly. During
• Examine the eye of this patient.
accommodation, after some delay, abnormal pupil
• Look at the eyes. What are your findings?
constricts slowly, may be smaller than normal.
it is also called myotonic pupil, which is a benign
Presentation of a Case ~I----------, condition, common in young women, usually
unilateral (80%), and rarely bilateral. Hence, pupil
• The pupil of right (or left) eye is dilated than appears unequal. It may be associated with loss of knee
other, regular (or circular). and ankle jerk on the same side. Its cause is not known.
• It does not react to light immediately, but when
light is focused for a long time, it constricts Q: What is the site of lesion?
slowly. A: Ciliary ganglia (due to parasympathetic
• Eye movements: Normal, no diplopia. denervation) .

Q: What is the significance of Holrnes-Adie pupil?


My diagnosis is Holmes-Adie pupil. A: It is a benign condition, should not be confused
with AR pupil.

Q: What are the differences between /\R pupil and


Holmes-Adie pupil?
A: As follows:

AR pupil Holmes-Adie pupil


Bilateral Usually unilateral
Holmes-Adie pupil (dilated tight pupil)
Pupil: small, irregular and Regular, one pupil dilated
unequal
Q: what else do you want to examine?
A: Tendon jerk (knee and ankle) on the same side, and Loss of light reflex, Slow reaction to light and
both may be lost. but persistence of accommodation
accommodation reflex
It is then called Holmes-Adie syndrome.
Impaired response to Normal response to mydriatic
Q: What is Holmes-Adie pupil? mydriatic
A: It is an abnormality characterized by absent or
Occurs in neurosyphilis Unknown cause (benign
delayed pupillary constriction to light or accom-
condition)
modation. If exposed to light for prolonged period,

Exophthalmos
Usual instructions are: 2. Look at the eyes from behind to confirm propto-
sis (eyeball may be visible above the supraorbital
• Look at the face. What are your findings? What else
ridge), and may require to place a paper between
do you want to examine?
the supraorbital ridge and maxillary prominence
• Examine the eyes.
(note the space between this).
Proceed as follows: 3. If obvious exophthalmos, note the following
1. Look at the eyes from front, look and comment points:
about sclera that is visible between the upper eye- • Lid retraction (by inspection, ask to look
lid and the upper limbus of cornea. Observe for straight): The upper eyelid is retracted and sclera
any swelling of eyelids, congestion of sclera, chem- above the upper margin of corneal limbus is
osis (oedema of conjunctiva), corneal ulceration visible (normally one-third of the cornea is
and thyroid stare (a frightened expression). covered by the upper eyelid).
_ SHORT CASES IN CLINICAL MEDICINE

• Lid lag (ask the patient to follow your finger N.B. For details, see page 264.
down): The upper eyelid fails to follow the
Q: What are the causes of exophthalmos?
finger (called von Graefe sign).
A: As follows:
• Ask the patient to.wrinkle the forehead (may be In unilateral exophthalmos, the causes are:
absent, called Joffroy sign). 1. Graves disease (the commonest cause).
• See the movement of the eyeball both 2. Orbital cellulitis.
horizontally and vertically (if movement is 3. Retro-orbital deposition found in:
absent with exophthalmos, it is called
• Lymphoma.
exophthalmic ophthalmoplegia).
• Leukaemia.
• During movement: Ask about diplopia.
• Secondary deposit.
• Test for convergence (impaired convergence is
• Hydatid cyst.
called Mobius sign).
4. Tumours of the orbit:
4. Now examine the following points: • Neurofibroma.
• Signs of thyrotoxicosis (warm sweaty hands, • Sphenoidal ridge meningioma.
tremor and tachycardia).
• Osteoma.
• Signs of hypothyroidism (puffy face, baggy eye- • Glioma (from optic nerve sheath).
lid and loss of outer one-third of the eyebrows,
• Dermoid.
coarse dry skin, non-pitting oedema and slow
In bilateral exophthalmos, the causes are:
relaxation of ankle jerk).
1. Graves disease (the commonest cause).
• Examine the thyroid gland (di ffuse enlargement 2. Cavernous sinus thrombosis.
in Graves disease). 3. Caroticocavernous fistula.
4. Others:
Presentation of a Case ~I----------, • Hand-Schuller-Christian disease.
(Bilateral or Unilateral Exophthalmos) • Craniostenosis.
• Hypertelorism.
• There is bilateral (or unilateral) exophthalmos • Apparently in severe myopia (eye is longer
(as evidenced by sclera above, the upper limbus than normal).
is visible, and more marked on right or left).
• Bilateral retro-orbital deposition (lymphoma
• Eyelids are swollen; there is chemosis, redness
and leukaemia).
and congestion of conjunctival vessels, and cor-
neal ulceration. Eye movement is impaired of the
right (or left) eye, (mention in which direction),
this indicates ophthalmoplegia.
• There is diplopia on looking (mention in which
direction) .
• Impaired convergence (Mobius sign).
• Mention about lid lag, lid retraction and wrin-
kling offorehead (if any).

My diagnosis is Graves disease. Bilateral exophthalmos

Bilateral exophthalmos (Graves disease) Unilateral exophthalmos (right)


10 • EXAMINATION OF THE EYE ~

Q: What investigations do you suggest? Read the following topics carefully:


A: As follows: • Cavernous sinus thrombosis: Usually follows
• Thyroid function tests (for Graves disease) (See infection of nose, orbit or face. The patient is
chapter on Endocrinology). very toxic, with high temperature. Eyeballs are
• Orbital ultrasonogram (USG). very painful and congested. Urgent treatment is
• cr scan of orbit (enlargement of extraocular mus- necessary.
cles, compression in optic nerve and proptosis). • Caroticocavernous fistula: There is pulsating exoph-
thalmos and conjunctival congestion. A bruit is
Q: What is the difference between proptosis and heard over the orbit, the intensity of which may be
exophthalmos? reduced by pressing over the carotid artery. Carotico-
A: It is synonymous; it means protrusion of the eye- cavernous fistula is due to rupture of infradinoid
ball (according to some authority: If unilateral, it is part of the rCA to the cavernous sinus, mostly due
called proptosis and if bilateral it is called exoph- to trauma or spontaneous. This may require neu-
thalmos). It is assessed by Hertel exophthalmometer rosurgical intervention. In 10% cases, there may be
(normally <20 mm). small fistula, which resolves spontaneously.

Nystagmus
Usual instructions are:
Presentation of a Case [1---------,
• Examine the eyes. Or, examine the movement of
eye. What are your findings? (Jerky Nystagmus): Case No.1
• There is nystagmus in the right (or left) eye on
Proceed as follows:
lateral movement, faster component towards
1. Ask the patient to sit, look straight in front and see the right (or the left) side (horizontal, jerky
whether nystagmus is present or not: nystagmus).
• If present in central gaze, likely to be ocular • No other abnormality in eye movement.
'nystagmus (fixation nystagmus).
• Diplopia is absent.
• If absent, then see in lateral gaze, called gaze
nystagmus.
2. Now see any nystagmus by the movement of My diagnosis is horizontal, jerky nystagmus.
eyeball (horizontal and vertical):
• Keep your finger straight in front of the eye (not Q: What do you think is the cause in this case?
below), 2-3 feet from the patient. A: May be cerebellar lesion (on affected side) or ves-
• Move the finger laterally, patient should follow tibular lesion (on contralateral side).
the finger up to 30 to the left and the right and
0

keep your finger for 5 seconds (a guide of 30 0


: Q: What else do you want to e-xamine?
limbus and caruncle meet in adducting eye). A: As follows:
• Do not move beyond 45 from central, and
0
• Cerebellar signs (see chapter on Neurology).
beyond that nystagmus may be physiological. • History of vertigo, deafness and tinnitus (suggests
3. Observe, whether it is horizontal, vertical or vestibular nystagmus).
rotatory. • History of drugs, namely phenytoin and other
anticonvulsants, barbiturates, alcohol, and
4. Whether it is jerky or pendular and faster
benzodiazepines (all these may cause nystagmus).
component in which direction.
• Nystagmus may present by movement of head to
5. During movement, observe whether nystagmus one side (positional nystagmus, and is found in
is present in abducting eye; at the same time, benign positional vertigo).
observe the failure of adduction of other eye
(called internuclear ophthalmoplegia, which is Q: What is the cause of vertical nystagmus?
also called ataxic nystagmus). A: Due to brain stem lesion, causes are:
SHORT CASES IN CLINICAL MEDICINE

• Nystagmus on downward gaze: Lesion in foramen 1. According to the direction:


magnum (involving medulla). • Horizontal.
• Nystagmus on upward gaze: Lesion on the floor • Vertical.
of fourth ventricle (involving midbrain). • Rotatory.
Q: what are the sites of lesion that produce 2. According to the site oflesion:
nystagmus? • Cerebellar nystagmus (towards the site of
A: As follows: lesion).
• Cerebellar lesion. • Vestibular nystagmus (away from the site of
• Vestibular lesion (central and peripheral). lesion).
• Brain stem. • Brain stem lesion (usually vertical nystagmus,
and may be in other direction).
Presentation of a Case 3. Others:
(Internuclear Ophthalmoplegia): • Positional nystagmus (associated with benign
Case No.2 positional vertigo).
• Ocular or fixation nystagmus (usually
• On lateral movement, there is nystagmus in the pendular type).
abducting eye and failure of adduction of other • Optokinetic.
eye (dissociation of movement of other eye). • See-saw nystagmus.
• On covering the abducting eye, adduction of
other eye is normal. Q: What is jerky nystagmus?
• No other abnormality. A: Jerky or phasic nystagmus is characterized by eye
movement faster in one direction than other.
My diagnosis is internuclear ophthalmoplegia (ataxic Usually seen in horizontal direction, elicited on
or dissociated nystagmus). lateral gaze in one or both directions.
The causes are cerebellar lesion, vestibular lesion or
Q: What is the site of lesion? lesions of their connection in the brain stem.
A: In medial longitudinal fasciculus (MLF) in the brain
stem. MLF connects the VIth nerve nucleus on one Q: What is pendular nystagmus?
side to the IlIrd nerve nucleus, which is on the A: In this type, oscillations are equal in speed and
opposite side of brain stem. The causes are: amplitude in both directions of the eye movement.
• Multiple sclerosis (the commonest cause). It is usually seen in central gaze.
• Pontine glioma. The cause is poor visual acuity (in severe refractive
• Vascular cause (CVA). error or macular disease), usually congenital and
asymptomatic.
• Wernicke encephalopathy (also there is ocular
palsy, loss of pupillary reflex, ataxia and Korsakoff
Q: What is ataxic nystagmus!
psychosis) . A: In this type, on looking to one side, nystagmus
• Encephalitis. is present in the abducting eye and there is fail-
• Phenytoin toxicity. ure of adduction of the other eye. It is also called
Read the following topics carefully: "dissociated nystagmus" and is present in internu-
clear ophthalmoplegia.
Q: WhJ.! is nystagmus?
A: It is the involuntary, rhythmical and oscillatory move- Q: What are the causes of horizontal nystagmus?
ment of the eyes due to inability to maintain the A: As follows:
posture of eyes, owing to the lack of balance of the 1. Cerebellar.
opposing ocular muscles. It is defined by the direction 2. Vestibular nystagmus.
of fast phase and is exaggerated on gaze to that side. 3. Brain stem lesion.
Nystagmus must be sustained for more than a few 4. Others:
beats to be significant. Nystagmus may be jerky or pha- • Ocular or fixation nystagmus (usually
sic, pendular or ataxic (internuclear ophthalmoplegia). pendular type).
Q: What are the types of nystagmus? • Optokinetic.
A: As follows: • In normal person, in extreme lateral gaze.
10 • EXAMINATION OF THE EYE __

Q: What are the causes of vertical nystagmus? Q: What is optokinetic nystagmus?


A: As follows: A: It occurs when the patient follows a rapidly mov-
• Brain stem lesion up-beating (midbrain lesion) ing scene (as during travelling in a train, eye
and down-beating (meduJla associated with remains fixed to a telegraph pole). It is a normal
foramen magnum lesion). phenomenon.
• Rarely, ocular nystagmus.
Q: What is see-saw nystagmus?
Q: What are the causes of vestibular nystagmus? A: In this condition, one eye raises and tUlTIS in and
A: Vestibular nystagmus is usually horizontal or rota- the other eye falls and turns out. It is due to parasel-
tory, and not vertical. It is of two types: peripheral lar tumour.
and central.
1. Peripheral: Lesion in labyrinth or vestibular Q: What is positional nystagmus?
nerve. Fast component of nystagmus is contral- A: In this condition, nystagmus is present in certain
ateral to the site of lesion, may be associated position and rapid movement of the head.
with cochlear lesion. Its causes are: • It can be detected by Hallpike test, in which the
• Labyrinthitis (may be viral). patient should lie on the back, support the head
• Meniere disease. and allow to fall below horizontal plane and turn
• Acoustic neuroma. to one side.
• Head injury. • After a few seconds, rotatory nystagmus and
• Middle ear disease. vertigo develop. If the position is maintained,
• Vestibular neuronitis (presents with acute fatigue occurs after 10-20 seconds, nystagmus
vertigo, tinnitus and deafness). and vertigo disappear.
2. Central: Lesion is in vestibular nuclei. Its causes • If the test is repeated immediately, there is little
are: response (adaptation). It is usually associated
• Cerebrovascular accident. with benign positional vertigo.

• Multiple sclerosis. Causes: Calcific degeneration of utricle and saccule


• Neoplasm. of the inner ear causes small particles to fall on to
• Alcohol. the cupola of semicircular canal during the move-
• AnticonvuJsant drugs. ment of head.

Subhyaloid Haemorrhage
Usual instruction: Q: What are the typical findings in subhyaloid
• Perform fundoscopy. haemorrhage?
A: Sharply demarcated preretinal (subhyaloid) haem-
Presentation of a Case :1---------, orrhage with a fluid level (crescentic or upward
concavity).
• There is haemorrhage with crescentic shape or If the patient is in supine position, then the fluid
upward concavity in the right eye. level is not seen. Retinal haemorrhage and mild
• Other part of the retina is normal (there may be papilloedema may be seen. The haemorrhage may
bleeding spots in the retina). extend into the vitreous humour, which is called
Terson syndrome.
My diagnosis is subhyaloid haemorrhage.
Q: What is the presentation of subhyaloid
haemorrhage?
A: The patient presents with sudden painless loss of
vision. There may be blurring or floaters or black
spots with or without flashing lights. There may be
features of subarachnoid haemorrhage.

Q: What are the causes of vitreous haemorrhage?


Subhyaloid haemorrhage A: As follows:
_ SHORT CASES IN CLINICAL MEDICINE

• SAH. • Lumbar puncture and CSF study may be done.


• Diabetes mellitus. • Cerebral angiography will be needed later to find
• Hypertension. out the source.
• Blood dyscrasia. Q: What (Ire the findings in CSF?
• Trauma. A: As follows:
Q: What is the cause of subhyaloid haemorrhage? • High pressure.
A: Subarachnoid haemorrhage is the commonest • Frankly haemorrhagic fluid.
cause (subhyaloid haemorrhage is pathognomonic • Xanthochromia (when kept for some hours).
of SAl-I). Q: How will you differentiate the CSF finding of SAH
Q: What are the features of SAH? from trauma?
A: As follows: A: In case of trauma, the initial sample is mixed with
blood, but subsequent samples show clear fluid or
• Sudden severe headache, usually occipital
less blood. But in SAH, all the test tubes have fluid
(thunder-clap headache or struck by a hammer).
mixed with blood. Xanthochromia is absent in trau-
• Nausea, vomiting.
matic blood.
• Loss of consciousness.
• Neck rigidity. N.B. Remember the following points regarding SAH:
• Kernig sign may be positive. • More common in women.
• Immediate mortality is 30%.
Q: What are the causes of SAH? • Chance of recurrence is 40% in 4 weeks and 3%
A: As follows: annually.
• Rupture of berry aneurysm. • Berry aneurysm arises from bifurcation of circle
• Arteriovenous malformation (AVM). of Willis.
Q: What investigations do you suggest for SAI-E • If the patient with SAH deteriorates, it indicates
A: As follows: either rebleeding or cerebral infarct due to reflex
vasospasm of the cerebral vessels (which can be
• cr scan of head (preferable over MRJ).
prevented by using nimodipine).
• cr angiogram may be done if needed.

Choroidoreti nitis
UsuaJ instruction: Q: What are the causes of choroidoretinitis?
• Perform fundoscopy. A: As follows:
• Toxoplasmosis.
Presentation of a Case 11--------, • Tuberculosis.
• Sarcoidosis.
• There are multiple pigmented patches with • Syphilis.
whitish or greyish areas within these, seen on the
• Toxocariasis.
upper and temporal side of the right eye.
• Behcet disease.
• There is no haemorrhage, but exudates are present.
• Vessels are normal. • CMV infection.
• HN (AIDS).

Diagnosis is choroidoretinitis. Q: Why there is pigmentation?


A: It indicates exposed sclera due to old choroiditis,
causing atrophy of the choroidoretina.
Q: What are the presentations with choroidoretinitis?
A: Patient may complain of reduced vision, floaters
and redness of eye. Features of primary disease may
be present.
Q: What are the typical findings of CMV retinitis?
A: There may be haemorrhage, oedema and exudate.
Chorioretinitis in toxoplasmosis Typically, it looks like cottage cheese and ketchup.
10 • EXAMINATION OF THE EYE _

Q: I low would you treat CMV retinitis? Q: What is the prognosis?


A: Ganciclovir or foscarnet (intravenous). A: Poor, if fovea is involved. Also depends on
aetiology.
Q: What are the other causes of pigmentation in retina?
A: As follows:
Q: What are the retinal changes in AIDS?
• Photocoagulation. A: As follows:
• Retinitis pigmentosa.
• Due to CMV (see above).
• Malignant melanoma.
• Due to HlV-cotton wool spots.
• Racial (tigroid fundus).

Retinal Detachment
Usual instruction: • Uncontrolled diabetes mellitus.
• Perform fundoscopy. • Retinal surgery (loss of vitreous following cataract
surgery).
• Trauma (perforating eye injury).
Presentation of a Case :1----------, • Chronic inflammation.
• Posterior vitreous detachment.
• The retina is opaque and grey in the left eye (no
pink colour). • Severe myopia.
• There is ballooning detachment with numerous • Systemic disease: Hypertension, toxaemia of
folds (indicates large collection of subretinal pregnancy, chronic glomerulonephritis, retinal
fluid). venous occlusive disease, retinal vasculitis.

Q: What is the presentation?


My diagnosis is retinal detachment. A: As follows:
• Flashes of bright light (photopsia) in the
peripheral part of vision.
• Floaters in the eye.
• Feeling of heaviness in the eye.
• Blurring of vision.
• Blindness or shadow starting peripherally and
progressing centrally. The patient typically
describes a curtain or veil being drawn over the
visual field.
• Loss of central vision.

Q: What is the mechanism of retinal detachment?

Retinal detachment
A: Separation within the retina between the pho-
toreceptors and retinal pigmented epithelium,
Q: What are the causes? characterized by collection of fluid and blood in
A: As follows: this space.

Bitemporal Hemianopia
Usual instruction: • Look at the patient (acromegaly). With this
• Examine the field of vision. Or, examine the optic diagnosis what would you like to examine in the
nerve. eyes.
_ SHORT CASES IN CLINICAL MEDICINE

• Pituitary tumour.
Presentation of a Case • Craniopharyngioma.
• There is loss of vision on the temporal side in • Sarcoidosis.
both eyes. • Suprasellar meningioma.
• Central vision is intact.
Q: What arc the presentations?
A: May not be any complaint, but sometimes diplopia.
Diagnosis is bitemporal hemianopia. Patient may complain of repeated collision on the
Q: What is the site of lesion 7 sides with another person or door etc.
A: Centre of the optic chiasma, damaging the fibers
Q: What investigations do you suggest?
from nasal half of retina, as they decussate at
A: As follows:
chiasma. This will result in loss of both temporal
• Perimetry.
half of visual field.
• X-ray skull.
Q: What are the causes? • MRI of brain.
A: As follows: • Pituitary hormone assay.

Miscellaneous
Yellowish thickening of conjunctiva, may be on either
Corneal Arcus
side of cornea, progresses towards cornea, but does not
cover it. It is due to the hyaline degeneration of elastic
tissue. Its causes are:
• In the elderly, exposure to dust and fume.
• Gaucher disease.

Bitot Spot

Corneal arcus
It is a crescentic whitish opacity, like a line near the
periphery of cornea. Usually, starts at lower part, ulti-
mately completes the circle.
• Corneal arcus indicates annular infiltration of lipid
in the peripheral rim of cornea.
• A normal finding in the elderly (arcus senilis).
• Sometimes, it is associated with hypercholes-
terolaernia. especially in young patients (arcus
juvenilis). May also be present in type IV hyperli-
poproteinaemia.

Pingueculae

Bitotspot
Pingueculae
10 • EXAMINATION OF THE EYE __

• Hypercalcaemia due to any cause.


• Iridocyclitis in children.
• Idiopathic in children.
• Phthisis bulbi.
Treatment:
• Chelating agents, such as sodium versenate, are
helpful.

Kayser-Fleischer Ring

Keratomalacia
These are white plaques of desquamated, thick
conjunctival epithelium. These are triangular in shape,
usually found in young patients, due to vitamin A
deficiency. Sometimes, may occur due to exposure to
dust and glare.
Q: What are the eye problems that occur due to
vitamin ,\ deficiency?
A: As follows:
1. Night blindness. Kayser-Fleischer ring
2. Xerophthalmia: It is a greenish-brown discolouration at the corneal
• Initially, xerosis conjunctivae (dIY,thick, there margin, usually appears first at the upper periphery, and
is pigmented bulbar conjunctiva associated then encircles the whole cornea. It is due to the depo-
with smoky appearance), then Bitot spot. sition of copper in Descemet membrane of cornea. It
• Later, when dryness spreads over the cornea, may not be seen by naked eye, and requires slit-lamp
it is dull, hazy and lacks lustre due to examination.
. keratinization-caUed xerophthalmia. Presence of KF ring is pathognomonic of Wilson dis-
3. Keratomalacia: Corneal opacity, ulcer and disso- ease. Found in adult (60%), almost always present in
lution leading to blindness. neurological Wilson disease. It may be absent or less in
4. Treated with vitamin A, otherwise blindness young children. It is rarely found in primary biliary cir-
may occur. rhosis. KF ring disappears with treatment.

Band Keratopathy Blue Sclera

Bluesdera
Band keratopathy Its causes are:
It is the subepithelial deposition of calcium salt in the • Isolated.
cornea with a clear zone separated from limbus. Its • Osteogenesis imperfecta.
causes are: • Marfan syndrome.
__ SHORT CASES IN CLINICAL MEDICINE

• Homocystinuria. Methanol Poisoning


• Ehlers-Danlos syndrome.
Methanol is a component of varnishes, paint remover,
• Pseudoxanthoma elasticum.
wind shield washer solutions and copy machine fluid.
Q: What are the causes of sudden blindness? Methanol is metabolized by alcohol dehydrogenase to
A: Sudden blindness may be in one eye or both eyes. formaldehyde and formic acid. It is mainly metabo-
lized in liver (90%), only 10% is excreted unchanged by
Causes of mono-ocular (one eye) blindness:
lungs and kidneys.
• Trauma to the eye.
Clinical features of methanol poisoning are:
• Central retinal artery occlusion.
• Early manifestations (by methanol): Nausea,
• Central retinal vein occlusion.
vomiting, abdominal pain, headache, vertigo,
• Giant cell arteritis. . dizziness, convulsion, confusion, stupor and
• Optic neuritis or retrobulbar neuritis. coma.
• Acute ischaemic optic neuropathy. • Later on (by metabolite formic acid): Retinal
• Migraine. injury leading to blindness. There is metabolic
• Others include acute glaucoma, retinal detach- acidosis. Ocular toxicity occurs 15-19 hours after
ment and Leber OA, functional. ingestion.
Causes of binocular (both eyes) blindness: • In severe cases: Bradycardia, myocardial depression
and shock
• Bilateral occipital lobe infarction.
• Bilateral occipital lobe trauma. Treatment:
• Bilateral occipital nerve damage. • Gastric lavage.
• Methyl alcohol poisoning, may be in quinine • Supportive measures: N fluid, oxygen.
and ethambutol toxicity. • Correction of acidosis: Sodibicarb in large dose
• Functional (hysteria). (alkalinization enhances formic acid excretion).
• In early stage: Ethanol is given (it inhibits methanol
Q: What are the causes of gradual blindness? oxidation by competing the inhibition of enzyme).
A: As follows: Ethanol is given 10 mL/kg of 10% ethanol IV or
Causes of bilateral blindness of gradual onset 1 mL/kg of 95% ethanol orally.
• Thiamine (100 mg QID), pyridoxine (50 mg QID)
• Cataract.
and folate (50 mg QlD).
• Macular degeneration.
• Folinic acid 30 mg IV every 6 hourly. It reduces ocular
• Diabetic retinopathy (vitreous haemorrhage).
toxicity (accelerates metabolism of formic acid).
• OA. • Dialysis: Indicated, if ingestion of methanol is
• Bilateral optic nerve or chiasmal compression. >30 g or metabolic acidosis or blood methanol
• Bilateral optic nerve damage (tobacco amblyopia). >500 mgfL.
CHAPTER 11

DERMATOLOGY
"This is a very testing part. It is more difficult than a written test"
- Talley & O'Connor

Introduction
In any clinical examination of medicine, a few common • Stratum granulosum (granular layer).
cases related to dermatology are frequently selected. • Stratum spinosum (prickle cell or Malpighian
Most of the diagnosis is obvious on visual impression. layer).
The concept of inspection is always a valuable starting
• Stratum basale (basal layer).
point during examination of a patient in dermatological
diseases. 2. Dermis.
With the patient's permission, undress the patient 3. Hypodermis or subcutis.
and remove the make-up, jf possible. A magnifying
Epidermis is an avascular stratified squamous epithe-
lens is helpful. Good light, preferably natural, is more
lium, attached to dermis by basement membrane. Basal
appropriate. Feeling the skin provides diagnostic clue.
cells move outwards towards superficial horny layer,
Examine the patient very carefully and gently. Describe
and the time taken is four weeks; 95% cells of epider-
the lesion precisely as follows: distribution, colour, size
mis are keratinocytes: the remaining 5% are Langerhans
and shape, oozing, pattern of lesion (linear, ring-like,
cells and melanocytes. There are a few Merkel cells.
reticulated, annular and so on). After inspection, pal-
Dermis contains blood and lymphatic vessels, nerves,
pate to see tenderness, consistency, temperature and
muscle, appendages (sweat glands, apocrine gland,
mobility.
sebaceous glands and hairfollicles) and immune cells
Usual instructions are: such as mast cells and lymphocytes. It also contains col-
lagen, elastin and ground substance.
• "Perform the general examination of this patient"
or "look at here" (examiner may indicate a part). Functions of the skin:
• What is your diagnosis?
• What else do you want to examine? 1. Protection against chemicals, ultraviolet radiation
(UVR), antigens and microbes.
In this chapter, a few common dermatological diseases
are included with related questions and answers and a 2. Physical' barrier against friction and shearing
brief discussion. Candidates are advised to go through forces.
a lot of dermatological cases to develop their skill for 3. Preservation of a balanced internal environment
spot diagnosis. (by preventing loss of water, electrolytes and
macro-molecules) :
Skin consists of three layers:
4. Sensation (pain, touch and temperature).
1. Epidermis: It has 5 layers; from top to bottom, the
5. Synthesis of vitamin D and testosterone.
layers are:
• Stratum corneum (horny layer). 6. Temperature regulation.
• Stratum lucidum. 7. Body odour and psychosocial factors.
SHORT CASES IN CLINICAL MEDICINE

Psoriasis
Usual instructions are:

• Look at here, what is your diagnosis?

Presentation of a Case

• There are multiple, well-circumscribed, erythe-


matous plaques with silvery white scales at the
knee, scalp and natal cleft (mention the site).

My diagnosis is psoriasis.

Q: What else do you want to examine?


A: I want to see:
Guttate psoriasis (front)
• Nails: Pitting, oil spot, cracking of free edges,
onycholysis (separation of nail plate from its bed),
and thickening and subungual hyperkeratosis.
• Joints: Arthropathy (see page 408).
• Eye: Iritis (may be blepharitis, kerati tis and
conjunctivitis) .
• Tongue: Geographical tongue.

Plaque psoriasis Guttate psoriasis (back)

Q: What are the sites of psoriatic skin lesion? Q: What are the differential diagnoses of psoriasis?
A: Extensor surfaces of knee, elbow, wrist, back of ear, A: Psoriasis may be confused with the following
scalp, hairline, extensor of limbs, sacrum, around diseases:
the umbilicus, intergluteal cleft and flexures (natal • Dermatomyositis (heliotrope sign, atrophy and
cleft, axillary fold), submammary fold and nails. poikiloderma).
• Lichen planus (usually on flexor surface, Wick-
ham striae, violaceous flat-topped papules and
adherent scale).
• Seborrhoeic dermatitis (greasy, yellowish scale
on eye brows, nasolabial crease, gluteal crease,
ears, sternal region, axilla, sub mammary folds,
umbilicus and groin).
• Pityriasis rosea (short duration, herald patch and
cellarette scaling on trunk upper arms and thighs).
Scalp psoriasis • Subacute lupus erythematosus.
________________ ,_,_. DERMATOLOGY I!IIIIII
• Secondary syphilis. • Psoriasis.
• Dermatophytosis (tinea corporis, cruris and pedis). • Lichen planus.
• Viral warts (verruca plana).
Q: What is Auspitz sign and Koebner phenomenon in • Others: vitiligo, pityriasis rubra pilaris, mollus-
psoriasis? cum contagiosum, nummular eczema and Darier
A: As follows: disease.
• Auspitz sign: On removing the scales forcibly,
there are occurrences of capillary bleeding points. Q: What are the factors that aggravate psoriasis?
• Koebner phenomenon: Psoriatic lesion is A: The aggravating factors are:
produced when the normal skin of a psoriatic 1. Trauma.
patient is scratched or injured (may occur in 2. infections: ~-haemolyticstreptococci (aggravates
surgical scar). guttate psoriasis) and HIV infection.
3. Psychological factors: emotion and anxiety.
4. Drugs:
• ~-Blocker.
• Antimalarial (chloroquine and
hydroxychloroq uine).
• Lithium.
• Systemic steroid: the condition aggravates
after withdrawal of steroid (rebound
phenomenon) and also after stopping of
the prolonged usage of local steroid.
• Angiotensin-converting enzyme (ACE)
inhibitor.
• Alcohol.
5. Rarely, sunlight (UVR may worsen).
Koebnerphenomenon 6. Metabolic (hypocalcaemia and dialysis).

Pustular psoriasis

Auspitz sign

Q: What is Koebner phenomenon? What are the


causes of Koebner phenomenon?
A: Koebner phenomenon is the appearance of
iso-
morphic skin lesions at the site of trauma, burn or
scratch mark The causes are: Nail pitting
_ SHORT CASES IN CLINICAL MEDICINE

Read the Following Topics in Relation to


Psoriasis

Q: What is psoriasis? What are the various types of


psoriasis?
A: It is a chronic inflammatory disease of skin
characterized by well-defined erythematous plaque
with silvery white scales, involving commonly the
extensor surface, elbows, knees and sacral regions
associated with recurrence and remission. It affects
1-2% of the population.

There are four types:


Plaque psoriasis
• Chronic plaque psoriasis (Common. Well
demarcated, red with a dry silvery white scale. It Q: What investigations should be done in psoriasis?
commonly involves elbow, knee and lower back, A: As follows:
but may also involve scalp, nails, flexures, palms). 1. Routine:
• Complete blood count (CBC).
• Guttate psoriasis (raindrop like psoriasis is a
• Liver function test (LFf)
variant, common in children and young adults.
• Serum creatinine.
An explosive eruption of very small circular or
• Lipid profile.
oval plaques appears over the trunk about 2 weeks
• X-ray chest.
after a streptococcal sore throat. Majority of the
• Urine R/M/E (to see proteinuria).
patients develop plaque psoriasis in later life).
• Serum electrolytes (hypokalaernia.
• Pustular psoriasis (It may be localized involving hyponatraemia, hypochloraemia).
palm and sole or rarely generalized, which may • Serum uric acid level.
be serious). • Serum IgE (to differentiate from atopic
• Erythrodermic psoriasis (>90% of body surface dermatitis).
area becomes red and scaly). 2. To establish the diagnosis, the following proce-
dures may be performed:
Q: What is the pathology of psoriasis? • Skin biopsy for histopathology (definitive).
A: Rapid proliferation and abnormal differentia-
• Antistrept.olysin 0 (ASO) titre (high in guttate
tion of epidermis (due to hyperproliferation of
psoriasis) .
keratinocyte) and infiltration of inflammatory cells • Throat swab culture and sensitivity (in guttate
(polymorph, T-lymphocyte and other inflammatory
psoriasis) .
cells). Accelerated epidermopoiesis is considered to • X-ray of the affected joints (to examine
be the fundamental pathological event in psoriasis. psoriatic arthritis).
3. To exclude other causes, the following tests may
be performed:
• VDRL,TPHA, antinudearantibody(ANA), anti
SS-A, anti SS-B, direct immunofluorescence
(DIF), uric acid and skin scraping for fungus.

Q: What are the histological findings?


A: As follows:
• Hyperkeratosis, parakeratosis, orthokeratosis.
• Neutrophilic micro-abscess of Munro in the
stratum corneum.
• Granular layer is absent or thin over the dermal
papilla.
• Spongiform pustule of Kogoj in stratum
Exfoliative dermatitis in psoriasis spinosum.
11 • DERMATOLOGY .-

• Regular elongation of rete ridges, which are dub Q: How to treat psoriasis?
shaped. A: As follows:
• Dermal capillary dilatation and tortuosity 1. General measures:
surrounded by a mixed neutrophilic and
• Explanation and reassurance.
lymphohistiocytic perivascular infiltrate.
• Avoid trauma, precipitating drugs and anxiety.
Q: What are the complications of psoriasis?
A: As follows: 2. Specific treatment:
• Psoriatic arthropathy. • Local therapy.
• Exfoliative dermatitis. • Systemic therapy.
• Secondary infection. • Combination therapy.
• Hyperuricaemia and gout.
• Others: amyloidosis, renal failure, hepatic failure
Local therapy (topical therapy on the lesion):
and congestive cardiac failure (CCF).
• Emollient: Common emollients are petrolatum,
paraffin, urea (up to 10%), olive oil etc.
• Salicylic acid (;::;5%): It is keratolytic, used to
soften and remove scale from psoriatic plaques.
• Crude tar (3-5%): It inhibits DNA synthesis.
• Dithranol: It inhibits DNA synthesis.
• Calcipotriol: It is a vitamin D3 analogue. It
inhibits epidermal proliferation and restores
normal horny layer. It is very effective in the
treatment of plaque type and scalp psoriasis. It
may cause hypercalcaemia and hypercalciuria.
• Tazarotene: Third-generation topical retinoid. It
acts by modulating keratinocyte differentiation
Psoriasis vulgaris and hyperproliferation, also by suppressing
inflammation.
• Topical steroid (mildly potent to super potent
according to severity of disease).
• lNR therapy: Narrow band UVB (peak emission
around 311 om) has been proved more effective
than broadband l1VB. However, it may cause
burning.
• Tacrolimus and pimecrolimus: Helpful for thin
lesions in areas prone to atrophy or steroid acne.
• Excimer laser: Indicated for patients with stable
Psoriasis in surgical scar recalcitrant plaques particularly in the elbow and
knee region.

Systemic therapy:

• PlNA (psoralen and UVA): Long-term use


may cause squamous cell carcinoma, basal cell
carcinoma and melanoma.
• Retinoid (acitretin): May help for arthritis and
psoriasis (especially pustular and also plaque).
Avoid in young female patients (it is teratogenic).
• Methotrexate, azathioprine or cyclosporine may
Psoriatic arthritis also be used.
~ SHORT CASES IN CLINICAL MEDICINE

• Biologic agents: Commonly used drug is anti-TNF-a For nail disease:


such as infliximab, etanercept, adalimumab and • Systemic agents
efulizumab. • Topical retinoids
• Other drugs: Tacrolimus, mycophenolate mofetil, • Local triamcinolone injections
hydroxyurea and thioguanine. • Topical fluorouracil.
Combination therapy:
• MTXplus topical agent. Q: Name the drugs that will help cure both psoriasis
• MTXplus retinoid. and arthritis.
• Retinoid plus PUVA. A: Methotrexate, azathioprine and acitretin.
• MTXplus infliximab,

Erythema Multiforme and Stevens-Johnson Syndrome


Usual instructions are:
• Look at here, what is your diagnosis?

Presentation of a Case ~f---------,

• There are multiple erythematous, maculopapular,


urticarial, vesicular and bullous lesions involving
the skin of palm, leg and foot (mention the site),
with few target lesions.

My diagnosis is erythema multiforme. Stevens-Johnson syndrome (body)


Q: What else do you want to examine? Why?
A: J want to see the oral cavity for mouth ulcer. Q: What is the important skin lesion in erythema
If mouth ulcer is present associated with skin multiforme?
lesion, the diagnosis is Stevens-Johnson A: Target lesion (also called iris lesion or Bull's eye
syndrome (SIS). lesion). In this lesion, there is central pallor or
dusky purpura with oedema and peripheral redness.

Q: What are the differential diagnoses?


A: As follows:
• Drug reaction.
• Systemic lupus erythematosus (SLE).
• Pemphigus vulgaris or bullous pemphigoid.
• Dermatitis herpetiform is.
• Urticaria or urticarial vasculitis.

Q: What is erythema multiforme? What is SJS?


A: As follows:
• Erythema multiforme is an acute inflammatory
reaction in the skin and mucous membrane,
characterized by multiple erythematous skin
Stevens-Johnson syndrome (face) lesions, such as macules, papules, vesicles, bullae
11 • DERMATOLOGY _

and target lesions involving the extensor surfaces • Drugs: Sulfonarnides, carbarnazepine,
of limbs. It is due to circulating immunocomplex thiacetazone, barbiturate, penicillin, phenytoin
that follows 7-14 days after precipitating factors and phenylbutazone.
(infections and drugs). It is usually self-limiting, • Idiopathic (50% cases).
resolves in 3-6 weeks, may recur. • Others: Malignancy (carcinoma and lymphoma),
• SJS is the severe form of erythema multiforme collagen disease (SLEand dermatomyositis),
with widespread bullous lesion in skin and Wegener granulomatosis, and sensitivity to
mucous membrane of mouth, eyes, and genitalia vaccination (polio and BCG).
associated with severe constitutional symptoms.
Q: What history do you like to take?
A: As follows:
• History of drugs.
• Infections.
• Any malignancy.
• Collagen disease.

Q: What investigations should be done in erythema


rnultiforme?
A: As follows:
• Full blood count (FBC).
Erythema multiforme (target lesion on hands)
• ASO titre.
• Antibody to herpes simplex type l.
• Antimycoplasma antibody.
• Other investigations according to suspicion of
causes.

Q: How to treat?
A: As follows:
• Offending drugs should be stopped.
• Symptomatic (lV fluid, antipyretic and
antibiotic).
Erythema multiforme (target lesion on buttock) • Local care of eyes and mouth.
• Treatment of primary cause.
N.B. Remember the following points: • In severe cases, especially in SJS, IV
• SIS has <10% body surface area (BSA) immunoglobulin can be given.
involvement. • Aciclovir (for recurrent herpes simplex infection).
• 10-30% BSA involvement is called SJS-TEN • Steroid: Its use is controversial. However, it
(toxic epidermal necrolysis) overlap cases. can be used and should be tapered rapidly
• When there is more than 30% of BSA because once skin loss occurs, it may aggravate
involvement, it is called TEN. It is most morbidity and mortality of the disorder due to
commonly induced by the same medications. immunosuppression.
Patients who initiaJly present with SJS may
progress to TEN. Q: What is bullous lesion? What are the causes of
bullous lesion of the skin?
Q: What are the causes of erythema multiforme?
A: Bulla is a circumscribed, fluid-filled elevation of
A: As follows:
skin more than 1 cm in diameter. The causes are:
• Infections: Herpes simplex virus type 1 (30%
cases) and Mycoplasma pneumoniae (common). 1. The commonest causes are:
Other infections include Streptococcus and • Erythema multifonne.
Histoplasma. • Pemphigus vulgaris.
• Bullous pemphigoid. Q: What is Nikolsky sign?
• Dermatitis herpetiforrnis. A: Rubbing of uninvolved skin results in the separation
2. Others: of epidermis of the skin and this condition is called
• Bullous impetigo. Nikolsky sign. The causes are pemphigus vulgaris
(the commonest cause), pemphigus foliaceus, TEN,
• Insect bite.
SSSSand epidermolysis bullosa (dystrophic type).
• Congenital, epidermolysis bullosa.
• Porphyria cutanea tarda.
• Staphylococcal scalded skin syndrome (SSSS)
and toxic epidermal necrolysis.
• Diabetic bullous lesions of skin.

Toxic epidermal necrolysis Nikolsky sign in pemphigus

Dermatitis Herpetiformis

Presentation of a Case

• There are multiple grouped, symmetrical,


erythematous, polymorphous, papular,
papulovesicular, vesiculobullous or bullous,
urticarial and excoriated skin lesion on the
extensor surface of knee, elbow, buttock, scalp,
upper back and sacrum.

My diagnosis is dermatitis herpetiforrnis,


Q: What is the commonest symptom?
A: Severe itching. Dermatitis herpetiformis (hand)
Q: What history do you like to take and why?
A: Diarrhoea or malabsorption after taking gluten-
containing diet (oat, rye, wheat, barley). There may
be coeliac disease, 10% of them may have dermati-
tis herpetiformis.
Q: What are the differential diagnoses?
A: As follows:
• Drug reaction (bullous).
• Erythema multiforme.
• SLE (bullous).
• Pemphigus vulgaris.
• Bullous pemphigoid.
• Others: scabies, contact dermatitis or atopic
dermatitis. Dermatitis herpetiformis (back)
N.B. Remember the following points:
• Dermatitis herpetiform is is common in male,
and the male to female ratio is 2: 1.
deposition alone or with C3 at the dermo-
epidermal junction with accentuation
in dermal papillae (IgM and IgG are
--
• 80% of the patients have HlA B8/DRw3. occasionally observed).
• Taking iodide and gluten-containing diet may 2. Anti-endornysial and tissue transglutaminase
precipitate an attack. antibody may be present in the serum.
• Can occur at any stage of adult life, rare in Antireticulin antibody may be present.
childhood.
3. Endoscopic biopsy from duodenojejunal flex-
• Once occurred, it is recurrent.
ure to diagnose coeliac disease shows:
• Oral mucosa: Involved rarely, mostly when
bullae are numerous. • Short and wide villi (partial villous atrophy).
• Higher incidence of developing malignancy than • Total flat mucosa (subtotal villous atrophy).
in general population (small bowel lymphoma). • Reduced height of epithelial cells and
Hypothyroidism may also occur. increased plasma cells in lamina propria and
• There may not be overt symptoms of malabsorp- intraepithelial lymphocytes.
tion, though an abnormal D-xylose absorption 4. HLA-typing (HLA-B8 and HLA-DR3 are positive
may be found in 70% patients. in 80% cases).
• Almost all patients have partial villous atrophy
5. Therapeutic trial with dapsone may reduce
in jejunal biopsy, even if no gastrointestinal tract
itching.
(GIT) symptoms are present.
Q: How to treat dermatitis herpetiforrnis?
A: As follows:
• Dapsone: 100-150 mg/ day (start with 100 mg
and increase the dose gradually to an effective
level). May be required for life-long usage.
Rapid relapse occurs once the drug is stopped.
Dramatic clinical response occurs in 72 hours
(a therapeutic test) and itching reduces quickly.
Dose is decreased gradually. Maintenance dose
prescribed is 50 mg/day.
• Gluten-free diet (avoid wheat, rye, oat and barley).
Dermatitis herpetiformis (leg and foot)
• Other drugs used: Sulpha pyridine, sui phasalazine,
colchicine, and tetracycline with nicotinamide.

Complications of dapsone therapy:


• Blood dyscrasia (haemolytic anaemia,
agranulocytosis and methaemoglobinaemia).
• GIT upset.
• Hepatitis.
• Neuropathy.
• Skin rash or exfoliative dermatitis.

Dermatitis herpetiformis (hands) N.B. Dapsone should be avoided in glucose-o-phos-


phate dehydrogenase deficiency.
Q: What investigations should be done?
A: As follows: Q: What are the associations in dermatitis
herpetiform is?
1. Skin biopsy shows:
A: As follows:
• Infiltration of neutrophils, eosinophils, fibrin
at dermal papilla and subepidermal vesicle. • Coeliac disease.
• Direct immunofluorescence in normal skin • Malignancy (lymphoma).
biopsy (perilesional) shows granular IgA • Thyroid disorder (hypothyroidism).
_ SHORT CASES IN CLINICAL MEDICINE

Herpes Zoster (Shingles)

Presentation of
Case No.1

• There are multiple vesicles or pustules, papules


and few crusted lesions (in thoracic or lumbar
region), which have not crossed the midline.

My diagnosis is herpes zoster.

Presentation of
Case No.2 Herpes zoster (healed)

• Same lesion along the distribution of the Q: What is the percentage of involvement of
ophthalmic division of trigeminal nerve. dermatome?
• There is redness and ulceration in the cornea A: As follows:
(right or left eye). • Thoracic: 55%.
• Cranial: 20% (trigeminal nerve commonly
involved).
• Lumbar: 15%.
• Sacral: 5%.

Q: Why does reactivation occur?


A: As follows:
• Spontaneous.
• Immunocompromised state: Multiple myeloma,
malignancy (lymphoma and leukaemia), HIV
infection and steroid and cytotoxic drug therapy.

N.B. High-risk groups include the elderly and


immunocompromised patients.
Herpes zoster

My diagnosis is herpes zoster ophthalrnicus.

Q: What is the cause?


A: It is due to reactivation of varicella zoster virus
that lies dormant in dorsal root ganglion of sensory
nerves, following chicken pox in childhood. Tho-
racic dermatome is commonly involved.

Q: What are the sites of lesion?


A: As follows:
• Dorsal root ganglia (when thoracic or lumbar
dermatome is involved). -
• Gasserian ganglia (trigeminal ganglia): in herpes
zoster ophthalmicus.
• Geniculate ganglia (in Ramsay Hunt syndrome). Herpes zoster (buttock)
______ ,_,_- DERMATOLOGY __

(may occur in Hodgkin lymphoma, leukaemia,


HN and bronchial carcinoma).
• In ophthalmic herpes, corneal ulcer and iritis
occur.
• Ramsay Hunt syndrome.
• Pleurisy, myocarditis and hepatitis.
• Others: Phrenic nerve palsy and muscle wasting.
Rarely, purpura and necrosis of affected segments
occur (purpura fulminans).

Q: How to treat herpes zoster?


A: As follows:
1. Local treatment:
• Antiseptic powder (povidone-iodine), drying
Herpes zoster (thigh)
solution, calamine lotion and local aciclovir
cream.
Q: What history do you like to take? • Topical idoxuridine: 5% solution during first
A: As follows: 36 hours of emption may help.
• History of chicken pox in childhood. • Local application of heat and gentle pressure
• Immunocompromised disease (multiple may help.
myeloma, lymphoma and leukaemia).
2. Systemic:
• Diabetes mellitus.
• Oral aciclovir (800 mg, five times daily for
• Drugs (steroid and cytotoxic drug].
1 week) or valacyclovir 1 g 8 hourly or farnci-
Q: What are the presentations of herpes zoster? dovir 500 mg TDS.
A: As follows: • In immunocompromised patient: Intravenous
• Initially, burning discomfort or pain along the (IV) aciclovir 10 mg/kg 8 hourly plus anti-
involvement of dermatome and dysaesthesia is varicella zoster immunoglobulin may be given.
present (when thoracic dermatome is involved • In severe cases, prednisolone 30-60 rug/day
0)1 right side, it confuses with acute cholecystitis). may be given (it does not cause dissemina-
• After 3-4 days, redness followed by papule, tion). Systemic steroid is contraindicated in
vesicle or pustules occurs (lesions occur in immunocompromised host.
cluster). Crust formation occurs after few days. 3. Interferon may help in limiting herpes zoster in
patients with cancer.
Q: What investigations are done to detect herpes
zoster? 4. A 4-week course with prednisolone may help to
A: Diagnosis is usually clinical. Investigations are reduce post-herpetic neuralgia.
rarely necessary.
• Raising antibody titre.
• Viral culture (44% positive).
• Polymerase chain reaction (peR; 97% positive).
• Tzanck smear from vesicle (75% positive).
• Histopathology shows intra-epidermal vesicle
with large swollen cells called balloon cells.

Q: What are the complications of herpes zoster?


A: As follows:
• Post-herpetic neuralgia.
• Secondary infections: Meningoencephalitis,
myelitis and motor radiculopathy (lumbar and
brachial).
• Bowel and bladder dysfunction occurs, if sacral
root is involved. Generalized herpes zoster Herpes zoster ophthalmicus (right side)
_ SHORT CASES IN CLINICAL MEDICINE

• Rash in the external auditory meatus and palate.


• Ipsilateral VIIth cranial nerve palsy (lower motor
neuron).
• Ipsilateral loss of taste and buccal ulceration.

Herpes Zoster Ophthalmicus


5'i5
o This commonly involves ophthalmic (first) division of
8 trigeminal nerve and occurs in 20% cases. Pain, tingling
'"§
Q)
and numbness around the eye followed by vesicular
c lesions are present.

The usual features are:


• Mucopurulent conjunctivitis.
• Episcleritis. scleritis or iritis.
Herpes zoster ophthalmicus (left side) • Keratitis, corneal ulcer or panophthalmitis.
• Pupillary distortion.
Q: How to treat post-herpetic neuralgia? • Occasionally, optic atrophy and visual loss.
A: It occurs in 10% cases, common in the elderly.
• Choroidoretinitis, secondary glaucoma and
• Gabapentin and amitriptyline (separately or in
cicatricial lid scarring.
combination).
• Topical: 10% lidocaine gel or 5% lidocaine- Treatment:
prilocaine in patch may be given.
• if no response, transcutaneous nerve stimulation 1. Local care of the eye:
(TENS) may help. • Local 3% aciclovir ointment (five times daily)
• A 3-week course of systemic prednisolone and idoxuridine 0.1% drop in eye.
40-60 rug/day, taper in 3 weeks may be given, if • Local antibiotic eyedrop.
there is no contraindication. • Eye protection using sterile pad.
• If there is still no response and in severe cases, 2. Oral aciclovir 800 mg, five times daily for 1 week.
ablation of ganglia may be tried (permanent
(Consult with an ophthalmologist.)
anaesthesia of that pa11may occur).
(Usually there is no response to analgesics. It may
Q: Which cranial nerves are involved in herpes zoster?
take long time to recover, even up to 2 years.)
A: As follows:
• Trigeminal nerve (commonly ophthalmic division).
Q: What is Ramsay Hunt syndrome?
A: It is the herpes zoster of geniculate ganglia • Facial nerve.
• Vestibule-cochlear nerve.
characterized by:

Acanthosis Nigricans

Presentation of a Case

• There are brown (or black), velvety plaques


of skin (thick and rugose-like warts) in axilla,
neck and limb flexures (may be found around
the umbilicus, nipple and groins), which are
symmetrically distributed.

My diagnosis is acanthosis nigricans.


(Rarely involves conjunctiva, buccal mucosa and lip
when the lesion is extensive.) Acanthosis nigricans (axilla)
______________________ ,__
, • DERMATOLOGY I!IIIIII
ataxia-telangiectasia and Morfan (Mental
retardation, pre- and post-natal overgrowth,
remarkable face and acanthosis nigricans)
syndrome.
Q: What is the relation between acanthosis nigricans
and malignancy?
A: Acanthosis nigricans may occur before the develop-
ment of malignancy in 18% or. may parallel with
malignancy in 60% and may follow malignancy in
22% cases. Remission occurs with cure of malig-
Acanthosis nigricans (neck)
nancy and worsens again with recurrence of tumour.
Q: What are the types of acanthosis nigricans?
Q: What is acanthosis nigricans? A: As follows:
A: Acanthosis nigricans is a disorder of skin character- • Type I: acanthosis nigricans associated with
ized by dark, thick, velvety skin in body folds and malignancy.
creases. •. Type II: familial acanthosis nigricans.
• Type III: acanthosis nigricans associated with
Q: What are the causes of acanthosis nigricans? other diseases (e.g. obesity, insulin resistant state
A: As follows: and endocrinopathy).
1. In <40 years of age:
• Obesity (commonest cause). Q: How to treat acanthosis nigricans?
• Insulin resistance. A: As follows:
• Endocrinopathy (Cushing syndrome, • Treatment of primary cause.
acromegaly, hypo- and hyperthyroidism and • Reduction of weight, if obese.
polycystic ovary syndrome). • Topical application such as etretinate, tretinoin,
calcipotriol, urea and salicylic acid.
2. Above 40 years of age, it is commonly due to
• CO2, laser ablation and long-pulse alexandrite
malignancy:
laser therapy.
• Carcinoma of stomach (the commonest
cause) and other GIT malignancies. Q: What are the dermatological manifestations of
• Bronchial carcinoma. malignancy?
• Lymphoma. A: As follows:
• In female: carcinoma of uterus, OV31Y, breast. • Acanthosis nigricans.
• Dermatomyositis.
N.B. Remember the following: • Thrombophlebitis migrans (crops of tender nodules
• Acanthosis nigricans can affectotherwise healthy along the blood vessels): commonly associated
people. Sometimes, genetically inherited; it is with carcinoma of pancreas (body and tail).
common in people of African descent. • Ichthyosis.
• Some drugs such as human growth hormone, • Tylosis (thickening of palm and sole and
oral contraceptive pills, steroid, fusidic acid dysphagia).
can cause acanthosis nigricans. • Exfoliative dermatitis.
• Besides, acanthosis nigricans may occur in • Erythema nodosum.
various syndromes such as Bloom syndrome, • Urticaria.

Ichthyosis
My diagnosis is ichthyosis.
Presentation of a Case

• Skin of the affected area (mention the site) is Q: What is ichthyosis?


rough, thick and dry. A: Ichthyosis is the dry and rough skin with persistent
• There are multiple fish-like scales. visible scaling in the body, which may resemble
fish scale.
__ SHORT CASES IN CLINICAL MEDICINE

Ichthyosis is not a single disease, but a group of


diseases in which homeostatic mechanism of epi-
dermal cell kinetics or differentiation is altered,
resulting in the clinical appearance of scale.

Q: What does ichthyosis indicate?


6n A: It may be secondary to underlying malignancy or
-@ other diseases or due to the usage of some drugs.
g Detailed history and physical examination should
:u be done.
Q
Q: What are the types of ichthyosis?
A: As follows:
1. Congenital: Ichthyosis vulgaris (back)
a. Major:
• Autosomal dominant: Ichthyosis vulgaris
and bullous ichthyosiform erythroderma.
• Autosomal recessive: Lamellar ichthyosis
and non-bullous congenital icthyosiform
erythroderma.
• X-linked ichthyosis.
b. Minor: Harlequin foetus and ichthyosis
Jinearis circumflexa.
c. Syndromes: Refsum syndrome, Rud
syndrome and Sjogren-Larsson syndrome.
2. Acquired:
• Hodgkin and non-Hodgkin lymphoma,
Ichthyosis vulgaris (face)
mycosis fungoides, multiple myeloma,
leprosy, AIDS, dermatomyositis and SLE.
• Drugs: Nicotinic acid, triparanol,
butyrophenones and clofazimine.
• Others: Sarcoidosis, hypothyroidism and
nutritional deficiency.

Ichthyosis vulgaris (legs)

Q: How to treat ichthyosis?


A: As follows:
• Treatment of primary cause.
• Symptomatic treatment with lactic acid, 12%
ammonium lactate lotion, 10% urea cream,
Ichthyosis vulgaris (arm) topical calcipotriene and so on.
11 • DERMATOLOGY __

Pemphigus Vulgaris
Usual instructions are:
• Look at here, what is your diagnosis?

Presentation of a Case :1---------,

• There are multiple thin-walled blisters in the


trunk, axilla and face.
• Some burst lesions, ulcer with crust and red
margin are present.
• Mouth shows few ulcers. Pemphigus vulgaris (thigh)

My diagnosis is pemphigus vulgaris.

Q: What are your differential diagnoses?


A: As follows:
• Bullous pemphigoid.
• Stevens-Johnson syndrome.

Q: What is the site of lesion in pemphigus vulgaris?


A: In the epidermis (above the basal layer).

Q: What is pemphigus vulgaris?


A: It is an autoimmune blistering disease character- Pemphigus foliaceous (back)
ized by thin-walled, flaccid, easily ruptured bullae
that appear in apparently normal skin and mucous
membrane or on erythematous base (associated
with mouth uicer).

Q: What are the causes of pemphigus vulgaris?


A: Exact etiology is unknown. Probable factors are:
• Autoimmunity, as suggested by the intercellular
deposition of IgG and C3 complement in
epidermis.
• Genetic predisposition (associated with HLA
DR4 and DR6).
• Drugs (penicillin, penicillamine, captopril,
rifampicin, cephalosporin, pyrazolone etc). Pemphigus foliaceous (face)

• UV light, PUVA and ionising radiation. Q: What are the presentations of pemphigus vulgaris?
• Increased incidence in myasthenia gravis and A: It is common in middle age, 50-60 years, and it is
thymoma. equally present in both sexes. Patients present with:
• Thin-walled flaccid bullae, which easily rupture,
Q: What are the types of pemphigus?
causing erosion, ulcer and crust formation with
A: As follows: oozing and bleeding.
• Pemphigus vulgaris. • Mouth ulcer is common (in 60%), also
• Pemphigus foliaceus. conjunctival and genital ulcer are noted.
• Paraneoplastic pemphigus. • Ulcer heals slowly with hyperpigmented patch
• IgA pemphigus. without scarring.
__ SHORT CASES IN CLINICAL MEDICINE

Q: What is the bedside test in pemphigus vulgaris? Q: How biopsy and DIF materials are collected?
A: As follows: A: A small early intact bulla should be taken. Site of
• Nikolsky sign: Rubbing of unaffected skin results biopsy is frozen with aerosol refrigerant spray so
in separation of epidermis. that the punch may include firm tissue. Tissue for
biopsy is taken in test tube containing formalin.
• Bullae spread phenomenon or Asboe-Hansen
Normal appearing perilesional skin is taken for DIF
sign: Pressure on the intact bullae gently forces
in saline soaked gauze.
the fluid to wander under the skin away from
pressure site. Q: How would you treat the patient?
A: As follows:
Q: What are the investigations done in pemphigus 1. General measures:
vulgaris? • Bed rest.
A: As follows: • Daily bath to remove thick crusts and foul odour.
1. Routine: • Maintenance of fluid and electrolyte balance
and nutrition.
• FBC.
• Antibiotic and blood transfusion (if necessary).
• Blood sugar.
2. Topical:
• Urine RME.
• 1% silver sulfadiazine is applied topically.
• Liver function tests.
• Antiseptic mouth wash and viscous xylocaine
• Renal function tests. are to be applied in the mouth.
2. Diagnostic: • Care of the eye.
• Skin biopsy for histopathology and 3. Systemic:
immunofluorescence test (an early intact • High-dose prednisolone 100-200 mg/day.
bullae <12 hours duration should be taken The dose should be tapered when remission
with perilesional area). occurs with no new blister. Maintenance dose
• Cytological (Tzanck method): Smears are is required for long time (may require life-
taken from the base of a bulla and (using long treatment). If new blister occurs during
Giemsa stain) is used for rapid demonstra- treatment, the dose of prednisolone should
tion of acantholytic cell (which shows no be increased.
intercellular bridge, darkly staining cyto- • Other treatment:
plasm and large nuclei surrounded by lightly o TVmethylprednisolone 1 g/day for 5 days
staining halo). (pulse therapy).
• Direct immunofluorescence shows intercel- a Mycophenolate mofetil 1-1.5 g is given
lular deposition of IgG throughout epidermis twice a day (commonly used as steroid
(both involved and normal skin) or oral sparing drug).
mucosa and C3 deposition in acantholytic a Other drugs: Azathioprine, cyclophos-
area (net-like, honeycomb or mosaic pattern). phamide, methotrexate, cyclosporine and
dapsone.
o In resistant case, IV immunoglobulin may
Histological findings:
be tried.
• Acantholysis (separation of individual a Biologic agents (infliximab, rituximab and
keratinocyte from one another). etanercept] .
• Superficial intraepidermal split. a Extracorporeal photochemotherapy.
• Intraepidermal blister above basal layer (in
pemphigus vulgaris) or subcorneal epidermal Q: What is the prognosis?
split (in pemphigus foliaceous). A: Prognosis is bad, and recurrence is common with
high mortality.
• Acantholytic cells are found lining the bulla as
well as lying free in the bulla cavity. Q: What are the complications of pemphigus vulgaris?
• Eosinophilic spongiosis and occasionally A: Secondary bacterial infection (pneumonia, septi-
neutrophilic spongiosis may be seen in the caemia), hypoproteinaemia, side effects of systemic
spongiotic epidermis in pemphigus vulgaris. prednisolone.
___ 1.;_1;_.DERMATOLOGY _

Bullous Pemphigoid
Usual instructions are:
• Look at here, what is your diagnosis?

Presentation of a Case

• There are multiple tense bullae in the trunk,


axilla and limbs.
• Also some red and urticarial patches are present.
• Mouth with no ulcer.
Pemphigus vulgaris

My diagnosis is bullous pemphigoid.


Q: What is the site of lesion in pemphigoid?
A: The lesion is in the basement membrane between
epidermis and dermis (subepidermal), hence less
tendency to rupture.

Bullous pemphigoid

Q: What are your differential diagnoses?


A: As follows:
• Pemphigus vulgaris.
• Dermatitis herpetiformis.
• Stevens-Johnson syndrome.
• Toxic epidermal necrolysis (TEN).

Q: What investigations do you suggest?


A: As follows:
1. Routine:
Bullous pemphigoid (body and arm)
• Complete blood count.
• Random blood sugar.
• Urine R/M/E.
2. Confirmatory:
• Tzanck test.
• Skin biopsy for histopathology and DIF.

Q: How to confirm the diagnosis of bullous


pemphigoid?
A: Skin biopsy for histopathology and direct immu-
nofluorescence shows deposition of IgG and
complement C3 at the basement membrane (in a
linear pattern). Histopathological findings are:
• Subepidermal bullae.
• Absence of acantholysis.
• Superficial dermal infiltration of many
Bullous pemphigoid (body) eosinophils.
_ SHORT CASES IN CLINICAL MEDICINE

Q: What are the presentations of pemphigoid?


Pemphigus Bullous
A: It is common in the elderly, >60 years. Patient
Points vulgaris pemphigoid
presents with:
• Tense bullae, red urticarial patch, less rupture, Age Middle age (50-60 Elderly (>60 years),
with a tendency to heal, may be urticarial, mostly years). few in infants and
in trunk, limbs and flexures. children.

• Mouth involvement (rare, and in <20%). Distribution Scalp, face, flexures, Trunk, limb,
may be generalized. flexures.
N.B. Pemphigoid may be associated with lymphoma.
Mucosa of mouth Commonly involved Rarely involved
conjunctiva and (60%). (20%), oral mucosa
Q: How to treat bullous pemphigoid?
genitalia is involved usually.
A: As follows:
1. General treatment: Bed rest, maintenance of Lesion Intraepidermal Subepidermal,
electrolyte, adequate nutrition etc. flaccid bullae that large, tense
rupture easily and blisters that do
2. Steroid:
has a less tendency not rupture easily
• Prednisolone 0.5-0.75 mg/kg/day, should be to heal. and has a more
tapered slowly over few weeks after clinical tendency to heal.
improvement (may be required to continue Urticarial plaques,
for 2-3 years). erythematous
patches, papules
• Potent topical steroid can be given alone.
and nodules may
• In severe cases, methylprednisolone 1.5rug/kg be found.
IV daily for 3 doses.
3. Tetracycline 500 mg 6 hourly with nicotinamide Asboe-Hansen & Positive Negative
500 mg 8 hourly. Nikolsky signs

4. Other drugs include dapsone, azathioprine, Target antigen Desmoglein 3, BP230 and BP 180.
methotrexate, cyclophosphamide, cyclosporine sometimes
and mycophenolate mofetil. desmoglein 1
5. IV immunoglobulin.
Antigens (kDa) 130 kDa and 230 kDa and
6. Antihistamine, if needed. 160 kDa. 180 kDa.

N.B. Bullous pemphigoid responds to lower dose Direct immuno- Intercellular deposi- Basement rnern-
of prednisolone. fluorescence tion of IgG and (3 in brane deposition
mosaic pattern (C3 of IgG and C3 in
Q: What are the causes of bullous lesion in skin with only lesional). linear pattern ((3
mouth ulcer? both lesional and
A: As follows: non lesional).

• Pemphigus vulgaris. Treatment Steroid with higher Steroid with lower


• Bullous pemphigoid. dose. dose.

• Stevens-Johnson syndrome.
Prognosis Variable prognosis. Better prognosis.
• Toxic epidermal necrolysis. May be fatal due to Does not affect
• Behcet syndrome. treatment compli- general health.
cations and sepsis. Usually self-limited
• Bullous SLE.
Recurrence is over 5-6 year
common. period, may be
Q: How do you differentiate pemphigus vulgaris from
fatal in very elderly
bullous pemphigoid? person.
A: As follows:
11 • DERMATOLOGY __

Arsenicosis (Chronic Arsenic Toxicity)


Usual instructions are:
• Look at here, what is your diagnosis?

Presentation of
Case No.1

• There are multiple hyperpigmented areas (men-


tion the site), with scattered hypopigmented area
giving rise to raindrop appearance.

My diagnosis is chronic arsenic toxicity (arsenicosis).


Rain drop pigmentation

Presentation of
Case No.2

• As above. Plus
• Multiple hard papular and plaque-like lesions
with hyperpigmentation involving ... (report the
site),
• Hyperkeratosis of palm and sole (rough and
thick).

My diagnosis is chronic arsenic toxicity.

Hyperpigmentation with scaly lesions

Q: What are the differential diagnoses?


A: As follows:
1. For pigmentation:
• Lepromatous leprosy.
• Haernochromatosis.
Hyperkeratosis of palm
• Addison disease.
• Xeroderma pigmentosum.
• Guttate psoriasis.
• Tinea versicolor.
• Idiopathic guttate hypomelanosis.
• Post-inflammatory hypopigmentation and
hyperpigmentation.
• Drug melanosis.
2. For keratosis:
• Epidermodysplasia verruciform is.
• Multiple corn or callosities.
• Verruca vulgaris.
• Hereditary palmoplantar keratoderma.
Hyperkeratosis of palm • Acrokeratosis verruciformis.
I

__ SHORT CASES IN CLINICAL MEDICINE


I

Q: What is the mechanism of arsenic toxicity? Q: How to confirm chronic arsenic poisoning?
A: After absorption, arsenic is widely distributed to all A: By measuring the arsenic concentration in hair,
the tissues of body; it combines with sulphydryl- nail, urine and serum.
containing substances and inhibits the activity of
many enzymes. It interferes with cell enzymes, cell
respiration and mitosis.
Si3 Q: What are the clinical presentations of arsenicosis?
o
B A: As follows:
'"§ • Melanosis: Hyperpigmentation (generalized or
Q)

c localized) with few scattered hypopigmented


areas giving rise to raindrop appearance.
• Hyperkeratosis of mainly palm and sole. May be
multiple, punctate, hard, discrete, papule and
verrucous plaque.
• Nails: Brittle, may show transverse white striae of
finger nails (Mee line). Palmar hyperkeratosis
• Hair: Dry, may fall off.
• Eye: Conjunctivitis.
• Nose: Rhinitis, epistaxis, nasal obstruction and
septal perforation.
• It may involve any system of the body (liver,
kidney, lung and heart).
Q: What are the diagnostic signs of arsenicosis?
A: As follows:
• Hyperkeratosis of palm and sole.
• Hyperpigmentation and hypo pigmentation (on
trunk and extremities) are the hallmarks.
Q: What are the systemic effects of chronic arsenic
Hyperkeratosis in sole
toxicity?
A: As follows:
1. CIT:Anorexia, nausea and vomiting. Normal value of arsenic:
2. Hepatic: Hepatomegaly, cirrhosis of liver and
• Blood: <3 mg/L (or 5-50 ppb/rng/L).
non-cirrhotic portal hypertension.
• Urine: 0.005-0.04 mg/L.
3. Central nervous system (CNS): Peripheral neu-
ropathy, seizure, confusion and encephalopathy. • Hair: 0.08-0.25 mg/kg (pubic hair is preferred I
due to lack of contamination).
4. Haematological: Anaemia, leucopaenia and
• Nail: 0.43-1.08 rug/kg.
th rom bocytopaenia.
• Daily urinary excretion: <0.1 mg/L,
5. Musculoskeletal: Myalgia, arthralgia and atro-
phy of extensor muscles causing wrist drop or
N.B. Remember the following points:
foot drop.
6. Renal: Dysuria, anuria and renal tubular necrosis. • Skin manifestation requires 1 year to develop.
7. Heart: Cardiomyopathy, dysrhythmia and heart • Systemic manifestations require 10 years to
develop.
failure.
8. Vascular: Peripheral vascular insufficiency causing • Early signs of arsenic toxicity: Conjunctivitis,
transient icterus and hyperhydrosis.
black foot disease.
9. Endocrine: Diabetes mellitus may be . Q: What other investigations are done in chronic arse-
precipitated. nic toxicity?
10. Malignancy: A: As follows:
• Skin: Squamous cell carcinoma, basal cell • FBe.
carcinoma and Bowen disease. • Urine for routine examination.
• Carcinoma of lung, kidney, urinary bladder, • Chest x-ray.
liver (angiosarcoma), prostate and colon. • Liver function tests.
-----11 • DERMATOLOGY_

• Renal function tests. 7. For skin lesion: Keratolytic emollients (salicylic


• ECG. acid, urea and retinoic acid), cryotherapy, elec-
• EEG. trocautery and laser therapy.
• Nerve conduction test. 8. Drugs (chelating agent may be used):
• D-penicillamine (250 mgTOS) for 3 months, or
Q: How to treat chronic arsenic toxicity? • Dimercaprol (SAL), dimercaptosuccinic acid
A: As follows: (DMSA) and dimercaptopropane sulphonate
1. Drinking of arsenic contaminated water must be (DMPS). SAL has a tendency to redistribute
stopped. arsenic to brain and testes. DMSA and DMPS
2. High protein diet. are more preferable because of their low
3. Antioxidant (vitamin A 50,000 lU, vitamin toxicity.
C 500 mg and vitamin E 200 mg daily for
Dose:
3 months).
o DMSA 10 rug/kg/day for 7 days followed
4. Vitamins and minerals supplement.
by 10 rug/kg/day three times for 14 days.
5. Vegetables and fresh fruits.
o DMPS: 100 mg TDS to QDS for every
6. Spirulina (an alga colony), which is rich in high
alternate week for three courses.
protein, may help to clear arsenic.
Preventive therapy: Drinking water should be safe.

Scabies
Usual instructions are:
• Look at here, what is your diagnosis?

Presentation of a Case

• There are multiple erythematous, excoriated


papules, vesicles, pustules and scratch marks
involving the interdigital area, fingers, ulnar
edge of the hand and anterior part of the wrist.

My diagnosis is scabies.
Q: What are the other sites of scabies?
A: Anticubital fossa, elbow joint, axilla, areola, around Scabies (in genitalia)
umbilicus, lower abdomen, genitalia, buttock and
dorsum of foot (face and scalp are never involved
except in infants, children and immunocornpro-
mised population).
Q: What is the causative organism?
A: Sarcoptes scabiei.

Scabies (hand) Infected scabies


_ SHORT CASES IN CLINICAL MEDICINE

l. Classical scabies.
2. Others:
• Scabies in a clean person.
• Scabies incognito.
• Nodular scabies (pink, tan, brown or red
nodules can be seen): Range from 2 to
20 mm in diameter. The mite is not present
in the nodular lesion.
• Crusted scabies (Norwegian): Atypical form,
common in immunocompromised and
institutionalized population. Lesion may
be hyperkeratotic and crusted. Scaling is
common, and pruritus may be less.
• Bullous scabies.
Scabies
Q: What are the complications of scabies?
A: As follows:
Q: What are the clinical presentations of scabies?
A: As follows: • Secondary bacterial infection.
• Intense itching, mostly at night. This may be • Eczematisation and lichenification.
associated with secondary infection. • Post-streptococcal glomerulonephritis.
• Papular lesions, excoriations and burrows at the • Others: Urticaria, exfoliative dermatitis,
sites of predilection. acrophobia and vasculitis.
• Presence of the same disease among family Q: How to treat scabies?
members or associates.
A: As follows:
• In women, itching of nipple associated with
1. Drugs:
generalized pruritic papular eruption.
• Permethrin 5% cream: Single application (from
• In men, itchy papules in scrotum and penis.
neck to toe) may be repeated after 1 week.
The patient may present with typical classical features • 1% gamma benzene hexachloride lotion.
or atypical form or features of complication. • Benzyl benzoate 25% lotion (apply for
consecutive 3 nights).
Q: What is the diagnostic sign of scabies? • Monosulphiram 5-8% emulsion (apply for
A: Burrow, which is short, wavy, dirty appearing line, consecutive three nights).
found in edge of fingers, toes or at the sides of hand • 10% precipitated sulphur in white petrolatum
and foot. Burrow contains female mites and eggs (apply for consecutive three nights).
and faeces of the mite. • Crotamiton 10% lotion or cream.
• Ivermectin may help in immunocom-
Q: How to diagnose scabies?
promised, ousted or Norwegian scabies
A: As follows:
(200 mg/kg in single dose).
• Usually clinical (hand lens is used to see burrow
• Scabetic nodules may require intranodular
and mite).
corticosteroid injection.
• Microscopical examination by scraping from
• Ivermectin orally can be used in cases where
lesion to see the mites.
topical therapy is difficult or impractical
Q: What are the differential diagnoses of scabies? (e.g.widespread infestations in nursing homes).
A: As follows: 2. General measures:
• Papular urticaria. • Control of secondary infection.
• Atopic dermatitis. • For itching, antihistamine can be prescribed.
• Dermatitis herpetiformis.
• Scrub bath before the application of topical
• Pityriasis rosea.
medicine.
• Pediculosis corporis.
• Washing of the cloths and bed sheet.
Q: What are the types of scabies? • Simultaneous treatment of the affected family
A: As follows: members.
11 • DERMATOLOGY_

Q: What are the causes of treatment failure in scabies? • Secondary bacterial infection is not controlled,
A: As follows: if the clothes and bed sheets are not properly
sanitized.
• Improper dilution of topical medicine. • If simultaneous treatment is not given to other
• Faulty method of application. affected members of the family.
• Scrub bath not taken. • If personal hygiene is not properly maintained.

Lupus Vulgaris
Usual instructions are:
• Look at here, what is your diagnosis?

Presentation of a Case

• There is reddish-brown plaque (or nodules) with


irregular margin in the right (or left) side of the
face and surface is smooth and glistening with
fine scaling over it.
• Non-tender, soft in consistency with atrophic
scar in one place and spread in other.
Lupus vulgaris (papular form)

My diagnosis is lupus vulgaris. Q: What is lupus vulgaris?


A: It is a cutaneous tuberculosis (TB) occurring in a
Q: What are the differential diagnoses? person with moderate or high degree of immunity,
A: As follows: as a post-primary infection.
• Sarcoidosis.
• Cutaneous leishmaniasis. Q: What is the common site?
• Leprosy. A: It commonly involves the skin of head and neck
• Discoid lupus erythematosus (OLE). in 80% of the cases, particularly seen around the
nose, also in arms and legs (in India, buttock
• Dermatomyositis.
and trunk are commonly involved). Looks like
• Psoriasis.
apple-jelly, when seen by a glass slide pressed
• Rosacea. against the lesion. Lesions heal with scarring and
• Deep mycosis. new lesions slowly spread out to form a chronic
• Wegener granulomatosis. solitary erythematous plaque. Chronic lesions
• Bowen disease. are at high risk of developing squamous cell
• Lymphocytoma. carcinoma.

Lupus vulgaris (plaque form) Lupus vulgaris (ulcerative)


_ SHORT CASES IN CLINICAL MEDICINE

• Mycobacterium marinum (responsible for swim-


ming pool granuloma; it comes from swimming
pool, lagoon or lake).
• Others are M. scrofulaceum, M. gordonae,
M. szuigai, M. avium-intracellulare complex,
M. haemophilum, M. chelonea and M. ulcerans
(causes Buruli ulcer).
Q: What is the relationshipof lupus vulgaris with TB
in other organs?
Lupus vulgaris (ulcerative) A: As follows:
• 46% have tuberculous lymphadenitis.
Q: What are the clinical types of lupus vulgaris?
• 15-20% have pulmonary TB and TB of bones
A: The five types are:
and joints.
• Plaque form.
• Ulcerative and mutilating form. Q: What is the pathogenesis or mode of infection in
• Vegetating form. lupus vulgaris?
• Tumour-like form. A: As follows:
• Papular and nodular form. • It commonly occurs in normal skin, at the site of
inoculation.
Q: What are the types of cutaneous TB?
• Direct extension from underlying infected gland
A: According to the source of infection, there are four
or joints.
categories of cutaneous TB, and they are:
• Lymphatic spread from mucous membrane of
1. Inoculation TB (exogenous source): Primary nose and throat.
inoculation complex, TB chancre, warty Tfs (ver- • Haernatogenous spread from other site.
rucous cutis) and some lupus vulgaris. • At the site of primary inoculation, also BCG
2. Secondary TB (endogenous source): vaccination or in the scar of scrofuloderma.
• Contiguous spread: Scrofuloderma Organisms may remain latent in the skin for many
• Auto-inoculation: TB orofacialis. years. Local trauma, nonspecific inflammatory
3. Haematogenous spread: Some lupus vulgaris, change or immunocompromised disorder may be
acute miliary TB, tuberculous ulcer and gumma responsible for reactivation of cutaneous TB.
or abscess. Q: How to investigate?
4. Eruptive Tls (tuberculids): A: As follows:
• Micropapular: Lichen scrofulosorum. • CBC and erythrocyte sedimentation rate (ESR).
• Papular: Papular or papulonecrotic • Mantoux test.
tuberculid. • Chest x-ray.
• Nodular: Erythema induratum (also called • Skin biopsy for histopathology.
Bazin disease).
Q: How to treat?
Q: What are the atypical mycobacteria causing A: Standard anti-tubercular therapy should be given:
cutaneous TB? four drugs for initial 2 months. In continuation
A: As follows: phase: 4-10 months.

Lichen Planus
Usual instructions are:
• Wickham striae and Koebner phenomenon are
• Look at here, what is your diagnosis?
present (report, if any).
Presentation of a Case • Oral mucosa shows reticulated whitish (or viola-
ceous) plaques consisting of pinhead papules on
• This patient has flat-topped, pruritic and polygonal the inner aspect of cheeks.
violaceous papules on the flexors of wrist, trunk, • Nail shows longitudinal grooving, proximal and
medial aspect of thighs and shins. distal onycholysis, ridging and splitting.
~~ 1_1_. DERMATOLOGY _

My diagnosis is lichen planus. Q: What are the types of lichen planus?


A: As follows:
Q: What are the other causes of flat-topped papules?
A: As follows: 1. Morphological type:
• Lichen amyloidosis. • Annular.
• Lichen nitidus. • Linear.
• Lichen myxoedematosus. • Hypertrophic.
• Atrophic.
Q: What is the cause of Wickham striae?
• Vesicobullous.
A: It is due to focal increase in the thickness of granu-
lar layer and total epidermis. It is better seen with a • Ulcerative and erosive.
hand lens (and by aniline dye). • Actinic.
• Follicular.
Different types of mucosal lesions:
• Lichen planus pigmentosus,
• Ulcerative: 50%.
• Guttate and perforating.
• Reticulate: 30%.
• Atrophic: 20%. 2. Special variants:
• Lichenoid drug reaction.
Q: What are the causes of lichen planus?
• Lichen planus pemphigoides.
A: As follows:
• Lichen planus and LEoverlap.
• Idiopathic
• Keratosis lichenoides chronica.
• Drugs
• Lichen planus and malignant transformation.
• Hepatitis C infection predisposes to lichen
planus. • Lichen planus and hepatitis C association.
• Lichenoid reaction of graft versus host disease.
Q: What are the drugs that may cause lichenoid drug
reaction? Q: What are the differences between lichen planus and
A: P-Blockers, thiazides. spironolactone, frusemide, lichen nitidus?
ACE inhibitors, calcium channel blockers, antima- A: As follows:
larials, heavy metals, arsenic, lithium, iodides.
Points Lichen planus Lichen nitidus

Size Variable, usually 1-2mm


large

Shape Polygonal Round

Colour Erythematous to Shiny and


violaceous discrete

Wickham striae Present Absent

Pruritus Marked Uncommon


Lichen planus (nails) Mucosal change Frequently present Rare

Q: How would you differentiate between lichenoid


drug reaction and lichen planus?
A: As follows:

Lichen drug Lichen planus


Points reaction

Site Sun-exposed areas Flexor areas

Lesions Larger and scaly Smaller

Wickham striae Absent Present

Alopecia More Less

Mucosal Less More


involvement
Lichen planus (leg and foot)
__ SHORT CASES IN CLINICAL MEDICINE

Q: What investigations should be done in lichen


planus?
A: As follows:
• Complete blood count.
• Viral marker for hepatitis Band C.
• Skin biopsy for histopathology and DiF.
• Liver function tests and renal function tests for
therapeutic purpose.

Lichen planus (toes) Q: What are the histopathological and DIF findings
in lichen planus?
A: As follows:
• Histopathology shows hyperkeratosis, beaded
hypergranulosis and saw tooth pattern of
epidermal hyperplasia. There is destruction of
basal layer, which is squamatized. In superficial
dermis, there is dense band-like infiltration of
lymphocytes and melanophages. Civatte bodies
represent necrotic keratinocytes.
• DIF shows clumps of IgM and less frequently
IgA, IgG and C3 subepidermally corresponding to
Lichen planus (mouth) Civatte bodies.
Q: How to treat lichen planus?
Q: What are the differential diagnoses of lichen planus?
A: Treatment depends according to the type and extent
A: Differential diagnoses of lichen planus depends on of the disease, and they are:
the type, and they are:
1. General measures:
l. Hypertrophic:
• Avoid drugs like diuretics, ~-blockers and
• Lichen simplex chronicus. antimalarials.
• Psoriasis. • Protection from trauma.
• Lichen amyloidosis.
2. Cutaneous lesions:
2. Annular:
• Topical steroid, and sometimes intralesional
• Granuloma annulare. steroid.
• Annular syphilis.
• Systemic therapy: In widespread lesions,
• Annular psoriasis.
systemic steroid 1 rug/kg/day for 7 days,
3. Linear: then taper (40 mg for 7 days and 20 mg for
• Lichen striatus. 7 days).
• Linear morphea. • Narrow-band UVBand PUVA.
4. Atrophic: • Isotretinoin and acitretin (0.5-1 rug/kg/day].
• Lichen sderosus atrophicus. • Cyclosporine.
• OLE. • Mycophenolate mofeti!.
5. Guttate: 3. Oral lesions:
• Guttate psoriasis. • Topical steroid in orabase, nystatin with
• Pityriasis rosea. clobetasol, topical tretinoin with steroid and
G. Oral: 0.1 % topical tacrolimus.
• Candidiasis. • PUVA and 308 nm excimer laser.
• Secondary syphilis. • Systemic: Hydroxychloroquine 200-400 mg/
7. Follicular: day for G months. Thalidomide 150 mg/day.
• Lichen spinulosus. Other agents used in cutaneous lesions also improve
• Lupus erythematosus. oral lesions.
__________~l~l
__.~D~E~R~MATOLOGY IIDIIIII
Q: What is the course or prognosis of lichen planus? 3. Haematological: Polycythaemia rubra vera (after
A: Two-thirds of the patients will have lichen planus of warm bath), lymphoma (especially Hodgkin
less than 1 year. Many patients get cured spontane- disease), leukaemia, multiple myeloma, iron
ously within 1 year. Recurrence occurs in half of the deficiency anaemia.
patients. 4. Endrocrine cause: Hypothyroidism, thyrotoxi-
cosis, diabetes mellitus (especially associated
Q: What are the medical causes of itching?
with candidiasis).
A: As follows:
l. Liver disease: Primary biliary cirrhosis, obstruc- 5. Any internal malignancy.
tive jaundice. 6. HIV.
2. Chronic renal failure. 7. Psychogenic.

Exfoliative Dermatitis
Usual instructions are:
• Look at here, what is your diagnosis?

Presentation of a Case

• The patient has generalized exfoliation and


erythema involving different parts of the body
(report the site). There is also oozing.
• Skin is dry and warm.
• Nails are brittle, yellowish with subungual
hyperkeratosis, onycholysis and dystrophy.
Exfoliative dermatitis in psoriasis
My diagnosis is exfoliative dermatitis.

Q: What are the clinical features of exfoliative


dermatitis?
A: As follows:
1. Cutaneous manifestations:
• Extensive exfoliation, scaling covering more
than 90% of the body surface, and pruritus
with widespread erythema.
• Skin thickening and loss of hair (often).
• Nails may be dystrophic.
• Palms and soles are involved, mucous
membranes are usually spared, but mucous
membrane of upper respiratory tract and Erythroderma in psoriasis
conjunctiva may be involved.
Q: What are the causes of erythroderma in adults?
2. Systemic manifestations: A: Primary or idiopathic and secondary to other
• Diarrhoea. disease.
• Anaemia, oedema and tachycardia.
The causes are:
• Lymphadenopathy (due to reactive
hyperplasia). 1. Cutaneous disease:
• Hepatomegaly (in 7-37% cases). • Psoriasis.
• Splenomegaly (in 3-23% cases). • Atopic dermatitis.
• Occasionally, gynaecomastia. • Neurodermatitis.
~ SHORT CASES IN CLINICAL MEDICINE

• Dermatophytosis.
• Contact dermatitis.
• Seborrhoeic dermatitis.
• Stasis dermatitis.
• Pityriasis rubra pilaris.
• Pemphigus foliaceus.
2. Systemic disease:
• Lymphoma.
• Leukaemia.
• Carcinoma (of lung, rectum, other
malignancy) .
• Multiple myeloma.
• HN infection.
• Graft versus host disease. Exfoliative dermatitis (legs)
3. Drugs: Barbiturate, carbamazepine, dapsone,
sulphonamide, allopurinol, lithium, pheno-
Complications of exfoliative dermatitis
thiazines and thiazide.
1. Metabolic:
Q: What are the causes of erythroderma in childhood?
• Loss of permeability barrier causes xerosis,
A: As follows:
water loss and dehydration. Marked scaling
• Atopic dermatitis.
causesprotein loss and hypoalbuminaemia.
• Leiner disease.
• Marked vasopermeability causes edema.
• Bullous ichthyosiform erythroderma and non-
• Marked vasodilatation causes chills, hypo-

d
bullous ichthyosiform erythroderma.
thermia and high output cardiac failure.
• Lamellar ichthyosis.
2. Other complications:
• Pityriasis rubra pilaris.
• Idiopathic. • Secondary infection.
• Electrolyte imbalance.
• Drugs. • Dermatogenic enteropathy (diarrhoea).
• Generalized dermatophytosis.
• Thrombophlebitis .
•. Leukaemia.
• Childhood dermatomyositis.
Q: What are the causes of exfoliative dermatitis with
Q: What is the pathogenesis of exfoliative dermatitis? nail changes?
A: There is increased rate of epidermal turnover. The A: As follows:
number of germinative cells and their absolute • Psoriasis.
mitotic rate are increased. Transit time of the cells • Pityriasis rubra pilaris.
through epidermis is shortened. Consequently, • Lichen planus.
more material is lost from the epidermis. Desqua- • Dermatophytosis.
mated cells show increased amount of nucleic acids • Atopic dermatitis.
and their degenerative products, decreased amount
Q: What are the investigations done in erythroderma?
of free amino acids and increased amount of solu-
A: As follows:
ble protein.
1. CBC: Normochromic normocytic anaemia, leu-
cocytosis, eosinophilia and ESR (high).
2. Urine (proteinuria).
3. Chest x-ray (to see pneumonia, lymphoma, sar-
coidosis and carcinoma).
4. Total protein and albumin-to-globulin ratio
(hypoproteinaemia, altered albumin-to-globulin
ratio).
5. Serum IgE (high in some cases, e.g. atopic
dermatitis) .
6. Serum electrolytes (hypokalaemia and
Exfoliative dermatitis (hand) hyponatraemia) .
11 • DERMATOLOGY __

7. Others (according to suspicion of causes): fourth, seventh and tenth day according to me
• Skin biopsy for histopathology and DIP. condition of the patient).
• Skin scraping for fungus and fungal culture • Other drugs: Methotrexate, cydosporine and
(in dermatophytosis). acitretin can be used in psoriatic erythroderma.
• ECC and echocardiogram in suspected cases lsotretinoin can be used in erythroderma caused
of heart failure. by pityriasis rubra pilaris. Immunosuppressives
• Bone marrow examination (to exclude leu- such as azathioprine and cyclophosphamide may
kaemia and myeloma, secondary deposits). be required occasionally.
• USC of whole abdomen. • PUVA therapy can be used in mycosis fungoides
• Test for HIV. or psoriasis.
• Computed tomography (CI') and magnetic • Treatment of primary causes (lymphoma and
resonance imaging (MRl), if needed. leukaemia).
Q: What are me causes of hypoproteinaemia in exfo-
N.B. If possible, systemic steroid should be avoided
liative dermatitis? due to me dangers of fluid retention, secondary
A: As follows:
infection, diabetes and other complications.
• Increased protein loss via scaling or leaking
They should be avoided in psoriatic erythro-
through the skin.
derma for they may provoke development of
• Protein losing enteropathy. pustular psoriasis. Steroid should be used cau-
• Decreased synthesis and increased catabolism of
tiously in atopic and seborrhoeic dermatitis.
protein.
• Dilution by increased plasma volume. Q: What is the prognosis of exfoliative dermatitis?
Q: How to treat? A: Depends on causes:
A: As follows: • Prognosis is good in drug-induced cases after the
• General measures: Maintenance of fluid and offending drug is withdrawn.
electrolyte balance and nutrition and protein • Prognosis is poor in cases of idiopathic
balance by high-protein diet. Intake and output eryth roderma.
monitoring, maintenance of environmental • For patients with psoriasis, atopic dermatitis
temperature and frequent bathing. or seborrhoeic dermatitis, it may continue for
• Emollients and lubricants: Liquid paraffin, months, respond slowly and tend to relapse.
Vaseline and olive oil or 12% ammonium • For the patients with underlying diseases or
lactate. malignancy, prognosis depends on the outcome
• Symptomatic: Antibiotic to control infection, and the course of me disease process.
antihistamine to control pruritus and diuretics in • The mean duration of the disease is 5 years with
case of oedema and cardiac failure. a median of 10 months,
• In severe persistent cases, systemic steroid • The overall mortality is 20-40%. In 20% of
may be given (triamcinolone acetonide 80 mg the fatalities, the cause of death is unrelated to
intramuscularly (lM) as initial dose, repeated on eryth roderma.

Alopecia
Usual instructions are: My diagnosis is alopecia areata.
• Examine the head. What are your findings? What
is the diagnosis? Presentation of
Case No.2
Presentation of
Case No.1 • There is total loss of hair over the whole scalp.
• The eyebrows and lashes are also absent.
• There is discrete, well-circumscribed patch of
hair loss over the scalp in different parts.
My diagnosis is alopecia totalis.
• Hair follicles are also seen.
• Eyebrows and eyelashes are present. Q: What else would you like to examine?
A: Hair loss in other pal1s of the body.
__ SHORT CASES IN CLINICAL MEDICINE

Q: If there is total loss of body hair, what it is called? bulb is seen as a result of atrophy of that portion;
A: Alopecia universalis. hence called exclamation point hair.
• Skin is smooth and shiny without any signs of
Q: What is alopecia areata? inflammation and scaling.
A: It is the localized loss of hair in the scalp. It may • Pruritus is absent.
be due to autoimmune mechanism. Found in SLE, • In 10% cases, especially in long-standing cases
may be associated with other autoimmune diseases, with extensive involvement, nails develop
such as Hashimoto thyroiditis, Graves disease, per- uniform pits that may form transverse or
nicious anaemia, diabetes mellitus and vitiligo. longitudinal lines.

Q: What are the differential diagnoses of alopecia


areata?
A: As follows:
• Non-inflammatory tinea capitis (characterized by
multiple scaly lesions and stumps of broken hair
and minimal inflammation; scraping from the
scalp reveals hyphae of dermatophyte in micro-
scopical examination in 10% KOH solution).
• Trichotillomania (inflammation and scaling are
absent; circumscribed lesions are very rare).
• Alopecia of secondary syphilis and lupus erythe-
matosus (history, clinical features, serological
test, scalp biopsy and immunofluorescence con-
firm the diagnosis).
Alopecia totalis
Read the following topics carefully.

Q: What are the causes of alopecia?


A: As follows:
1. Non-scarring (may be diffuse hair loss and focal
hair loss):
a. Diffuse hair loss:
• Abnormality of shedding; telogen effluvium
and anagen effluvium.
• Endocrine: Hypopituitarism,
hypothyroidism, hyperthyroidism,
hypoparathyroidism, androgenetic
alopecia and pregnancy.
• DIUgS: Cytotoxic drugs, anticoagulants,
Alopecia areata thyroid antagonists, lithium, oral contra-
ceptives and hypervitaminosis A.
• Others: Severe prolonged illness, malnu-
Q: What are the types of alopecia?
trition, deficiency of protein, iron, zinc
A: The three types are:
and biotin, surgery, acute febrile illness
• Alopecia areata (localized loss of hair in the scalp).
and radiation.
• Alopecia total is (hair loss of entire scalp).
b. Focal hair loss:
• Alopecia universalis (total loss of body hair).
• Trichotillomania, traction alopecia, alope-
Q: What are the diagnostic points of alopecia areata? cia areata, secondary syphilis and SLE.
A: The following points are diagnostic: 2. Scarring alopecia:
• Usually patches are '1-5 cm in diameter. • Infective: Furuncle, carbuncle, folliculitis, lupus
• Rapid and complete loss of hair, which is patchy vulgaris, tertiary syphilis, kerion and favus.
in distribution and round or oval in shape. • Physical factors: Burn and radiation.
• At the periphery of bald patches, there are loose • Chemicals: Acid and alkali.
hairs that may be broken off leaving short stumps. • Autoimmune: DLE, morphea and cicatricial
When they are pulled out, a tapered attenuated pemphigoid.
11 • DERMATOLOGY _
----------------

• Neoplastic: Basal cell carcinoma and squa- 2. Intralesional injection of steroid (triamcinolone
mous cell carcinoma. 2-10 rng/rnl.).
• Others: Lichen planus, sarcoidosis, lichen 3. Photochemotherapy using topical or systemic
sclerosus, follicular mucinosis, pseudopelade, methoxsalen and UVA (PUVA).
folliculitis decalvans and dissecting cellulitis 4. 308 nm xenom chloride excimer laser has been
of scalp. reported to produce regrowth after 11-12 ses-
Q: How to investigate a case of alopecia areata? sions over a period of 9-11 weeks.
A: According to clinical findings: Q: What is the prognosis?
• Skin scraping for fungus (to exclude tinea capi tis). A: Usually, spontaneous recovery occurs in post-
• ANA, anti-double-stranded DNA (SLE). pubertal patients. Predictors of poor prognosis are:
• Serological test for syphilis.
• Presence of atopic dermatitis.
• Others: According to suspicion of causes.
• Childhood onset.
Q: How to treat alopecia areata?
• Widespread involvement.
A: As follows:
1. Topical: • Duration longer than 5 years.
• Topical steroid. • Onychodystrophy.
• 1% anthralin cream. • Ophiasis (1oss may occur confluent along the
• Topical minoxidil (2-5%). temporal and occipital scalp).

Vitiligo
Usual instructions are: Family history may be present in one-third cases;
it equally affects both sexes. Although familial
• Look here. What are your findings? What is the
in 30% cases, it is not inherited as autosomal
diagnosis? dominant, or recessive trait, rather seems to have
multifactorial genetic basis. Individual is otherwise
healthy. Koebner phenomenon may be present
Presentation of a Case
(lesions appear at the site of skin damage).
• There are few areas of depigmentation of
Q: What are the associated diseases in vitiligo?
variable size and shape, surrounded by area of
A: Vitiligo may be associated with autoimmune
hyperpigmentation.
diseases, such as systemic sclerosis, Addison disease,
pernicious anaemia, Graves disease, Hashimoto
My diagnosis is vitiligo. thyroiditis, premature ovarian failure, diabetes mel-
litus and primary biliary cirrhosis.
Q: What else do you want to see?
A: Check for vitiligo in other parts of the body (around
the eyes, mouth, knee, dorsum of foot, hands,
axilla, groin and genitalia). Also, check the sensa-
tion (iflost, suggest tuberculoid leprosy. In advance
stage with widespread vitiligo, loss of sensation
may occur in lepromatous leprosy).

Q: What is vitiligo? What is the mechanism?


A: It is the area of localized depigmentation, prob-
ably due to autoimmune mechanism. Vitiligo is
due to focal loss of melanocyte. It affects 1% of the
population. Generalized vitiligo may occur, usu-
ally symmetrical involving hand, wrist, knee, neck,
around the eyes, mouth, dorsum of feet and so on.
The sites at friction or trauma are often affected. Vitiligo in fingers
_ SHORT CASES IN CLINICAL MEDICINE

• Tuberous sclerosis (ash-leaf spot).


• Chemical leucoderma.
• Bum.
Q: What are the investigations done in Vitiligo?
A: Diagnosis is clinical. Investigations are:
• Woods light examination shows chalky or ivory
white fluorescence.
• Skin scraping for Malassezia furfur (to differentiate
from tinea versicolor).
• Others: Blood sugar, thyroid function tests and
serum cortisol can be done according to the
Vitiligo in legs suspicion of causes.
Q: How do you treat Vitiligo?
Q: What are the types of vitiligo? A: As follows:
A: As follows: 1. General measures:
1. Focal vitiligo: Isolated macules or few scattered • Reassurance.
macules. • Use of sunscreen.
2. Segmental vitiligo: Unilateral macules in a der- • Use of self-tanning cream containing
matomal distribution. It has stable course and is dihydroxyacetone.
unlikely to be associated with thyroid or other
2. Topical:
vitiligo-associated diseases.
3. Generalized vitiligo: Most common, character- • Steroid (betamethasone and clobetasol
ized by few to many macules. propionate).
4. Acrofacial vitiligo: Involves distal digits and • Topical calcipotriene can be added to topical
periorificial areas. steroid.
5. Universal vitiligo: Widespread vitiligo with few • 0.10f0tacrolimus ointment in treating facial
remaining normal pigmentation. vitiligo.
3 Phototherapy:
Q: What are the differential diagnoses of vitiligo (or, • Narrow band UVB.
what are the causes oflocalized hypopigmentation)? • Topical application of 8-methoxypsoralen
A: As follows: followed by UVA.
• Tinea versicolor: Common in back and chest;
4. Surgical:
it has a fine scale. Yeast and hyphal forms are
• Epidermal grafting or autologous minigraft.
present (detected with 10% KOH solution).
• Transplantation of cultured melanocytes
• Pityriasis alba: Associated with fine scale; lesion
can be applied in patients with segmental
is poorly defined.
vitiligo or with stable vitiligo, which does not
• Tuberculoid leprosy.
respond to other therapy.
• Morphea and lichen sclerosis: Hypopigmented
or depigmented area associated with a change in N.B. Spontaneous re-pigrnentation occurs in
skin texture. 15-25% cases.

Neurofibroma
Usual instructions are: • Also, multiple cafe-au-lair spots on the back of
• Look at the patient. What is the diagnosis? the trunk.
• Examine this patient.

Presentation of a Case My diagnosis is neurofibromatosis type I.


Q: What else do you like to see?
• There are multiple nodular lesions of variable
A: As follows:
sizes and shapes on the face, both forearms and
• Axillary freckling.
hands.
• Lisch nodule.
11 • DERMATOLOGY __

Q: What are the features of type 2 neurofibromatosis?


A: As follows:
• Bilateral acoustic neuroma.
• Glioma-cerebral or optic nerve.
• Meningioma.
• Spinal neurofibroma.
• Schwannoma.
• Juvenile posterior subcapsular lenticular opacity.
Q: What are Cafe-au-lair spots?
A: These are round to ovoid, pale yellow or brown
macules, usually present on the trunk. May be 1 em
to more than 15 cm. Up to 5 may be present in a
normal person.
Q: What is Lisch nodule?

Neurofibroma
A: It is a melanocytic hamartoma on the surface
of iris, clear to yellow or brown. [t increases with
age, almost always present in patient older than
• Examination of the ear to exclude acoustic
20 years.
neuroma.
• Eyes for optic glioma. Q: What is plexiform neurofibroma?
• Spine for scoliosis. A: In this type, entire nerve trunk and its branches are
• Blood pressure (may be associated with involved in diffuse neurofibromatosis with over-
phaeochromocytoma) . growth of overhanging tissues, leading to gross
deformities in temporal and frontal scalp. Com-
Q: What is the triad of neurofibromatosis?
monest site of plexiform neurofibroma are temporal
A: As follows:
region in relation to trigeminal nerve, upper eyelid
• Neurofibroma. and back of the neck.
• Cafe-au-lair spots.
• Lisch nodules. Q: What are the associated findings or complications
of neurofibroma?
Q: What is neurofibroma?
A: As follows:
A: It is a benign tumour of peripheral nerves arising
• Kyphoscoliosis.
from neurilemmal sheath.
• Lung cyst (honeycomb lung).
Q: What is neurofibromatosis? • Pseudoarthrosis and other orthopaedic
A: It is an autosomal dominant disease characterized abnormalities.
by multiple neurofibroma and skin lesions like • Glioma, meningioma, medulloblastoma.
cafe-au-lair spots and axillary freckling. • Pheochromocytoma (in MEN la).
• Posterior mediastinal tumour called dumb bell
Q: What are the types of neurofibromatosis?
tumour.
A: Two types:
• Rarely, sarcomatous change (dangerous
• Type 1 or von Recklinghausen disease or complication of neurofibromatosis).
peripheral.
• Type 2 or central. Q: Is biopsy necessary for diagnosis?
A: No, diagnosis is done clinically.
Q: What are the features of type 1 neurofibromatosis?
A: As follows: Q: What is phacomatosis?
• Multiple cutaneous neurofibroma. A: It is a group of diseases in which neurological
• Cafe-au-lair patches (up to 5 may be" found abnormalities are associated with cutaneous dis-
in normal person), 5 mm in prepubertal and ease. These are:
15 mm in postpubertal patients. • Neurofibromatosis type 1.
• Axillary freckling. • Tuberous sclerosis.
• Hamartoma of iris (Lisch nodules). • Von Hippel-Lindau syndrome.
• Optic glioma. • Sturge-Weber syndrome.
__ SHORT CASES IN CLINICAL MEDICINE

Mycosis Fungoides
Usual instructions are: • Psoriasis.
• Look at the patient's back. Or, perform the general • Eczematous dermatitis.
examination. • Contact dermatitis.
• Drug eruption.
• Hansen disease (leprosy).
Presentation of a Case ~I----------,
• Tinea corporis .
• There are multiple, brownish red, indurated Q: What is mycosis fungoides?
plaques of various size and shape over the upper A: This is a slowly progressive T cell lymphoma of the
back of the patient. The largest one is ... cm in skin.
diameter.
• Scratch marks are seen. Q: How does it present?
• Few nodular lesions are noted in the same area. A: Initially, it presents with non-specific scaly erup-
tions, which may be thick and plaque like, confused
with eczema or psoriasis. May involve any area ofthe
My diagnosis is mycosis fungo ides.
skin. Usually progress very slowly over many years
from a plaque stage through to nodules and finally
a systemic stage. Extracutaneous involvement (such
as liver, lungs, spleen) and lymphadenopathy occur
in advance stage only. It is more common in males,
5th to 7th decade.

Q: What are the stages of mycosis fungoides?


A: As follows:
• Stage I Eczema or psoriasis like lesions.
• Stage II Plaque like.
• Stage III Nodules, ulcers, tumours.
• Stage IV Lymphadenopathy with or without
systemic involvement.
Mycosis fungoides on back
Q: What investigation should be done to confirm the
diagnosis?
A: Skin biopsy (hallmark is malignant T cell or Sezary-
Launter cells). There may be Pautrier micro abscess,
atypical cells in epidermis, large hyperchromic cells
with irregular nuclei called mycosis cell.

Q: What is Sezary syndrome?


A: It is a variant of mycosis fungoides associated with
erythroderma. There are large mononuclear cellsin the
skin and blood. Its prognosis is poor. Extracorporeal
photo therapy (with oral methoxsalen, lymphocyte
enriched blood fraction to UVAlight and reinfusion
of the cells into the patients may be helpful).
Mycosis fungoides
Q: How to treat mycosis fungoides?
A: As follows:
Q: What else do you want to see?
1. Initially local therapy:
A: I want to see lymph nodes (may be localized or
• Corticosteroid, nitrogen mustard
generalized) . (mechlorethamine), bexarotene gel.
Q: What are your differential diagnoses?
• PUVA or narrow band UVBphototherapy.
A: As follows: 2. After plaque stage: Electron beam radiation.
11 • DERMATOLOGY __

3. If the disease progresses: PUVA plus retinoid, Q: What is the prognosis'?


PUVA plus interferon, bexarotene, c-interferon A: It progresses slowly, usually over decades. Progno-
with or without retinoid, IL-12, denileukin or sis is better in patient with patch or plaque stage
total skin electron beam may be used. disease and worse in patient with erythroderma,
4. If all fails: Systemic anti-lymphoma chemother- tumour and lymphadenopathy.
apy may be required.

Tuberous Sclerosis
Usual instruction is: Q: What else do you like to see?
• Look at the patient's back Or, perform the general A: As follows:
examination. • Subungual fibroma (nodule arising from the nail
bed).
Presentation of a Case 11--------,
• Shagreen patches (firm, flesh coloured, patches
of leathery thick skin over the lower back).
• There are muJtiple pink or yellowish papules
on the face involving cheeks, nasolabial fold, • Ash leaf patches (hypopigmented areas of skin).
sides of the nose and chin. Also few nodules are • Cafe-au-lair spots present in 30% cases.
present on the forehead.

My diagnosis is tuberous sclerosis.


Q: What are these papules on the face called?
A: These are angiofibroma called adenoma sebaceum.

Ash leaf patch in tuberous sclerosis

Q: What history do you like to take?


A: As follows:
• Family history of similar disease.
• History of convulsion and mental retardation.

Adenoma sebaceum in tuberous sclerosis Q: What is tuberous sclerosis?


A: It is an autosomal dominant disease characterized
by classic triad of mental retardation (or leaming
disability), epilepsy and skin lesions.

Q: What other lesions may be associated?


A: As follows:
• CNS hamartomas: Cortical tubers and subepend-
ymal hamartomas. There may be cerebral glioma
and calcification of basal ganglia.
• Retinal phacoma (glial mass).
• Hyperplastic gum.
• Benign rhabdomyoma of heart.
• Renal angiomyolipoma.
• Cysts in lung, liver, pancreas, kidneys and bones.

Q: What will you see in skull x-ray?


Shagreen patch of the same patient A: Tram-line calcification at the basal ganglia.
~ SHORT CASES IN CLINICAL MEDICINE

Q: How to treat? Q: What is the cause of death?


A: No specific treatment. Symptomatic treatment for A: Usually from seizure, intercurrent illness or associ-
seizure. Genetic counseling should be done. ated neoplasm.

Kaposi Sarcoma
Usual instruction is: Q: What is Kaposi sarcoma?
• Look at the patient's back. Or, perform the general A: It is a tumour of vascular and lymphatic endothe-
examination. lium characterised by multiple purplish nodules and
plaque caused by human herpes virus 8 (HHV8).
It is an AIDS defining illness.
Presentation of a Case
Q: What is the cause?
• There are multiple red or purple or brownish A: Human herpes virus 8 (HHV8), also known as
plaques and nodules of various sizes and shapes Kaposi sarcoma-associated herpes virus (KSHV).
all over the back (70-100% of Kaposi sarcoma have antibody to
HHV8 compared with 1-5% in general population).
My differential diagnoses are:
Q: What are the types?
• Haemangioma. A: There are four types:
• Bacillary angiomatosis.
• Classic or sporadic form: Affects middle aged
• Cutaneous mycobacterial infection.
men, common in Jews and Mediterranean region,
• Drug rash.
indolent course, mainly affects lower limbs,
• Sarcoidosis.
confined to skin and is not fatal.
• Cutaneous lymphoma.
• African endemic KS: Occurs in children and
• Kaposi sarcoma.
younger men, more aggressive and ultimately fatal.
There is violaceous skin plaque, may be associated
with generalized lymphadenopathy in children.
• Transplantation associated KS (or KS in
iatrogenically immunocompromised patientsj=-
especially in patients getting immunosuppressive
therapy. It often regresses when the therapy is
stopped.
• AIDS-related KS: More aggressive, often rapidly
progress to plaques and nodules affecting the
upper trunk, face, and oral mucosa. Affects
one-third patients with AIDS, more common
Kaposi sarcoma in homosexuals. About one-third develop a
second malignancy like lymphoma, leukaemia,
myeloma etc. In AIDS patient, it may occur with
high CD4 count and low viral load.

Q: What is the presentation?


A: As follows:
• In early case: Small, raised, nonpruritic, reddish
purple nodule on the skin or a discoloration
of the oral mucosa or a swollen lymph node.
Commonly affects lower limbs, back, face,
mouth and genitalia. The cutaneous lesions can
be solitary, localized or disseminated.
• With progression, the skin lesions become larger
and numerous. The lesions may be macular,
Kaposi sarcoma patch, plaque, nodular and exophytic.
11 • DERMATOLOGY __

• There may be chronic leg oedema. intra1esional vinblastine, topical immunotherapy


• Visceral disease occurs in 10% at presentation. (imiquimod), interferon-c.
Commonly involved sites are lymph nodes, ora] • AIDS-related KS: Patient should first get HAART
cavity, GIT lungs, heart CNS etc. which improves Kaposi sarcoma.
• More widespread disease, or disease affecting
Q: How to diagnose? internal organs, is treated with systemic therapy
A: Diagnosis is made with biopsy of suspicious lesion. with interferon-a, liposomal anthracyclines or
If needed, internal imaging may be done. paclitaxel. N chemotherapy (e.g. combination
of vincristine and bleomycin or newer liposomal
Q: How to treat? preparation of doxorubicin) and immunotherapy
A: Depends on the subtype, localized or associated may be given.
with systemic disease: • In KS from immunosuppression: Immuno-
• Localized mucocutaneous disease responds to suppressive therapy should be reduced or
cryotherapy, radiotherapy, surgical excision, discontinued.

Pityriasis Versicolor
Usual instruction is: Q: What are the differential diagnoses?
• Look at the patient's chest. Or, perform the general A: As follows:
examination. • Vitiligo.
• Other fungal infections like tinea corporis.
Presentation of a Case Q: What are the sites of lesion of pityriasis versicolor?
A: Trunk, upper arm, neck groin.
• There are multiple small hypopigmented
macules over the front of the chest, both sides of Q: How to confirm?
neck and upper part of the back. A: Skin scrapping for fungus with KOH, which shows
large blunt hyphae and thick-walled budding spore
(spaghetti and meatballs).
My diagnosis is pityriasis (tinea) versicolor.
Q: How to treat?
A: As follows:
1. Topical treatment:
• Selenium sulfide lotion applied from
the neck to waist daily and kept for 5-15
minutes and continued for 7 days. Repeated
weekly for a month and then monthly for
maintenance.
• Ketoconazole shampoo (1% or 2%): Applied
and left for 5 minutes, may be used weekly.
• Imidazole cream, solution or lotion may be
used.
Pityriasis versicolor in neck 2. Oral antifungal:
• Ketoconazole 200 mg daily for 1 week or
Q: What is pityriasis versicolor? What is the cause? 400 mg in a single dose.
A: It is a benign superficial skin condition character- • Fluconazole 300 mg 2 doses 14 days apart.
ized by multiple small white or pink macules. It • Itraconazole may be given.
is caused by a fungus called Malassezia furfor. It is
common in warm temperature. It is more severe in N.B. Recurrence is quite common. Maintenance
imrnunocomprornised. treatment may be necessary.
MISCELLANEOUS
"Experience is never limited, and it is never complete"

- Henry James

Face in Different Diseases

Usual instructions are: • Paget disease.


• Look at the face; or, examine the face. What is your • Facial asymmetry (hemiplegia and hemiatrophy).
diagnosis? What relevant do you like to see? • Lupus pernio in sarcoidosis.

There is likely to be obvious diagnosis by looking at the


face. Further examination depends on the suspicion of
the underlying cause. Possible diagnoses are as follows:

• Myxoedematous face (page 260).


• Thyrotoxic face or Gravesdisease (page 261 and 264).
• Cushingoid face (page 284).
• Acromegalic face (page 280).
• Mongoloid face (Down syndrome; page 507).
• Haemolytic face (frontal and parietal bossing, and
.prominent malar bones, page 247). Lupus pernio in sarcoidosis
• Puffy face (page 503).
• Leonine facies (lepromatous leprosy, page 29). Q: What are the causes of depressed nasal bridge!
• Facial palsy (page 367). A: As follows:
• Parkinsonian face (page 361). • Trauma.
• Marfanoid face (page 109). • Tuberculosis (TB) (lupus vulgaris).
• Mitral face (page 76). • Leprosy.
• Myopathic face (page 359). • Sarcoidosis.
• Nephrotic face. • Congenital syphilis (also tertiary).
• Achondroplasia (skull appears enlarged, page 296). • Wegener granulomatosis.
• Superior vena caval (SVC) obstruction (page 30 • Fungal infection (deep).
and 31). • Idiopathic midline granuloma.
• Sturge- Weber syndrome (page 511). • Cutaneous leishmaniasis.
• Hepatic facies (muddy or pigmented
discolouration, pinched face and sunken eyes).
• Hippocratic facies (in advance peritonitis]: sunken
eyes, collapsed temples, pinched nose with crust
on lips and clammy forehead.
• Psychiatric disorders (depressed or anxious facies).
• Butterfly rash (page 504).
• Systemic sclerosis (page 382).
• Myasthenic face (page 367).
• Turner face (page 505).
• Virile (virilization in female).
• Tabetic face. Depressed nasal bridge in Wegener granulomatosis
12 • MISCELLANEOUS __

Q: What are the causes of puffy face?


A: As follows:
• Nephrotic syndrome.
• Acute glomerulonephritis (AGN).
• Myxoedema.
• SVC obstruction.
• Acromegaly.
• Angioneurotic oedema.
• Cushingoid face.
• Chronic alcoholism.
Plethoric face in polycythaemia
• Severe congestive cardiac failure.
• Hereditary angio-oedema.

Plethoric face in Cushing syndrome


Puffy face in nephrotic syndrome
Q: What are the causes of chloasma or melasma?
Q: Whal are the causes of malar flush? A: As follows:
It is a discrete pigmentation in the face of females,
A: As follows:
due to imbalance between oestrogen and proges-
• Normal person.
terone. Melanocyte-stimulating hormone (MSH) is
• Mitral stenosis.
normal. The causes are:
• Hypothyroidism.
• Pregnancy and sunlight exposure.
• Polycythaemia.
• Drugs (oral contraceptive and phenytoin).
• Others: ovarian tumour (rare).

Malar flush
Chloasma/melasma
Q: What are the causes of plethoric face?
A: As follows: Brief Notes on Wegener Granulomatosis
• Polycythaemia (due to any cause).
• Cushing syndrome. It is a disorder of unknown aetiology characterized
• Alcoholism. by necrotising granulomatous vasculitis of upper and
• SVC obstruction. lower respiratory trad with glomerulonephritis.
__ SHORT CASES IN CLINICAL MEDICINE

• Deafness (due to serous otitis media).


• Respiratory problems: cough, haemoptysis and
breathlessness.
• Eye: proptosis, conjunctivitis, episcleritis and iritis.
• Features of glomerulonephritis or renal failure.

Investigations:

• Chest x-ray: single or multiple nodules (migrating


lung lesion in 50% cases).
• Biopsy: from nasal lesions or nasal crusts and also
from kidney.
• Serum anti-neutrophil cytoplasmic antibody
(c-ANCA) for diagnosis and to check the relapse.

Q: ITow to treat?
A: As follows:
• Cyclophosphamide (2 mg/kg) plus prednisolone
(1 rug/kg), OR
• Intravenous cyclophosphamide (15 mg/kg) plus
IV methylprednisolone (10 mg/kg) every fort-
Wegener granulomatosis night and then every month.
• Once remission occurs (takes 3 to 6 months),
Diagnosis is made by: prednisolone is rapidly reduced, and cyclophos-
phamide is replaced by azathioprine.
History:
• Oral cotrimoxazole (960 mg, three times weekly)
• Nasal discharge, epistaxis, nasal obstruction, nasal is given to prevent pneumocystis pneumonia.
crust, rhinitis and sinusitis. If untreated, destruction • To check relapse, periodic measurement of
.of nasal bone and cartilage causes depressed nose. c-ANCA is performed.

Butterfly Rash
Usual instructions are:
are scaly and reddish with clear margin, more
• Look at the face; or, examine the face. marked on the right (or left) side of face.
• There is presence of nodule or telangiectasia,
Pay careful attention to the following points:
vitiligo and hyper- and hypopigmentation.
• Rash distribution (check whether present in other
parts of face) and character, scaly desquamation
and redness or other colour follicular plugging.
• Any nodular lesion (in face or ear lobule).
• Heliotrope rash (eyelid), telangiectasia, tightening
or thickening of skin and pinched up nose.
• Alopecia.
• Mouth ulcer.

Presentation of a Case

• There are multiple skin rashes on the face along


the butterfly distribution, also involving the
forehead and cheeks (mention, if any). Some
Butterfly rash (SLE)
12 • MISCELLANEOUS _

• Proximal myopathy (dermatomyositis).


• Sensation over skin lesion (leprosy).
• History of PKDL.
• History of drugs.
• Check for any lymphadenopathy, hepatomegaly,
splenomegaly and erythema nodosum
(sarcoidosis) .

Butterfly rash (dermatomyositisl

Q: What are the differential diagnoses for butterfly


rash?
A: As follows:
• Systemic lupus erythematosus (SLE) or discoid
lupus erythematosus (DLE).
• Dermatomyositis.
• Mixed connective tissue disease (McrD). Skin rash in leg and foot (SlE)
• Sarcoidosis.
• Drug rash.
• Acne rosacea (characterized by red patch with
telangiectasia on the face with papules and
pustules, which are absent in SLE).
• Lepromatous leprosy.
• Posr-kala-azar dermal leishmaniasis (PKDL).
Q: What else do you want to examine?
A: As follows:
• Rash in other parts of the body, alopecia and
mouth ulcer SLE. Mouth ulcer in SlE

- Turner Syndrome

Usual instructions are: • Hand: short fourth metacarpal (other metacarpals


• Look at the patient. What is your diagnosis? may be short), lymphoedema of hands (also
feet) and hypoplastic nails.
• Elbow: increased carrying angles (cubitus valgus).

Proceed and Present


as Follows: My diagnosis is Turner syndrome.

(Ask the patient her age. The patient is a female


with obvious short stature for her age.)
• Short and webbed neck, low hairline and
redundant skinfold on the back of neck.
• Face: small lower jaw (micrognathia), smalL and
fish-like mouth, high-arched palate and low set
and deformed ears.
• Chest: broad, wide apart nipples (shield-like
chest).
Webbing of neck
_ SHORT CASES IN CLINICAL MEDICINE

• Ultrasonogram (USG; small uterus, fallopian


tube and streak gonad).
• Hormone assay (low oestrogen, high luteinising
hormone [LH] and follicle-stimulating hormone
[FSH]).

Q: What is mosaicism?
A: The presence of two or more cell lines within the
body, either a 46XX or 46XY karyotype.
Low hairline
Q: How to treat Turner syndrome?
A: As follows:
• Oestrogen therapy at puberty.
• Growth hormone may accelerate height, but it is
not yet established whether there is any effect on
final height.
• Gonadal tumour may occur rarely, especially
in mosaic involving Y chromosome. Hence, it
should be removed.

Q: What are the associations or diseases that occur in


Turner syndrome?
Shield-like chest A: As follows:
1. Heart:
Q: Why does webbing of the neck occur? • Coarctation of aorta (10-20% cases). There
A: It occurs due to the fan-like fold of skin extending may be atrial septal defect (ASD), ventricular
from shoulder to the neck or an abnormal splaying, septal defect (VSD) and aortic stenosis (AS).
out of trapezius muscle. • Hypertension.
Q: What are the causes of webbing of the neck? 2. Kidney:
A: As follows: • Horseshoe kidney.
• Turner syndrome • Hydronephrosis.
• Noonan syndrome 3. Others (incidence is more):
• Watson syndrome • Diabetes mellitus (DM).
• Klippel-Fleil syndrome • Hashimoto thyroiditis (may be frank
• Diamond-Blackfan anaemia hypothyroidism in 20% cases).
• Lymphoedema in infancy.
Q: What is Turner syndrome? How does the patient • Red and green colour blindness.
present? • Strabismus and ptosis.
A: It is a sex chromosomal abnormality, characterized • Premature osteoporosis.
by absence of one of the X chromosomes • Pigmented nevi.
(45, XO). It only affects females. Allor part of one • Mental retardation (rare).
X chromosome is deleted, leading to failure of ovary
development. Externally, patient appears female,
Noonan Syndrome
but does not produce female sex hormones. Hence,
the patient remains sexually immature. It is also called male Turner. Noonan syndrome is
The patient usually presents with short stature. characterized by:
skeletal abnormalities and amenorrhoea. Secondary • Short stature.
sexual characters are underdeveloped. Intelligence is • Mental retardation (common).
usually normal. • Downward slanting and wide-spaced eyes.
• Low set ear.
Q: What investigations do you suggest?
• Webbing of the neck.
A: As follows:
• Low posterior hairline.
• Karyotyping from buccal smear: 45 (XO) is
• Pulmonary stenosis.
classical, occasionally 46 (XX) mosaics.
12 • MISCELLANEOUS _

It may affect both male and female equally. Female


patients have Turner phenotype but with normal 46,
XX.They have normal ovarian function and normal
fertility. In male, there is 46XY.
Cardiac lesion is present more on right side
pulmonary stenosis (PS). In Turner syndrome, left-
sided cardiac lesion is present more.
Short fourth metacarpal

Down Syndrome
Usual instructions are:
• Examine the patient. What are your findings? What
else do you want to examine?

Proceed and Present


as Follows: Down syndrome

1. Face:
• Appears flat.
• Nasal bridge appears flat.
• Low set, small ears.
• Mouth appears small and tends to remain
open with high-arched palate. Tongue
appears protruding with large, horizontal
fissure.
2. Eyes: Horizontal fissure in tongue
• Epicanthic folds and slanting eyes.
• Brush-field spots on iris (yellow speckles).
• Conjunctivitis.
3. Neck: short and wide.
4. Hands: single palmar crease (simian) and
short stubby finger. Hand looks small
and round (short, broad hands) and has
clinodactyly (short inward curving of little
finger).
Single palmar crease in Down syndrome
5. Others: short stature, muscle tone (hypo-
tonia), joint hyperextensibility, heart disease
(VSD is common, also ASD, patent ductus
arteriosus [PDAj, tetralogy of Fallot and mitral
regurgitation [MRj due to endocardial cushion
defect), straight pubic hair, gap between first
and second toes, and low IQ (from mild to
severe).

N.B. The child is fond of music.


lymphoedema hand in Down syndrome
~ SHORT CASES IN CLINICAL MEDICINE

• Endocrine: Autoimmune hypothyroidism,


diabetes mellitus type I.
• Infertility almost in all males, female may be
fertile.
• Lenticular opacity.

Q: What is Down syndrome?


A: It is a chromosomal abnormalit.y: trisomy 21
(47, XX/XY, + 21).
Q: How prenatal screening is done?
Lymphoedema foot in Down syndrome and A: As follows:
gap between first and second toe
1. First trimester:
• Ultrasonography to see nuchal translucency.
Q: What history do you lil{e to consider?
A: Maternal age during pregnancy (incidence is high • Serum human chorionic gonadotropin
with advancing maternal age). (~hCG).
• Pregnancy-associated plasma protein A
Q: What are the complications of Down syndrome? (PAPP-A).
A: As follows: 2. 13-20 weeks:
• Mental retardation (mild to severe). There is • Maternal serum for o-fetoprotein, hCG and
learning disability or cognitive impairment. unconjugated oestriol (uE3)'
• Haematological: In neonates, acute myeloid • Foetal USC.
leukaemia and in older children, acute lymphoid • Amniocentesis.
leukaemia.
• GIT: Duodenal atresia, Hirschsprung disease, N.B. Mother with risk of Down syndrome: Cho-
Meckel diverticulum, imperforate anus. rionic villous sampling before 13 weeks of
• Presenile dementia of Alzheimer type (in third to gestation or amniocentesis after 15 weeks of
fourth decade). pregnancy may be done.

Bilateral Parotid Enlargement


Usual instructions are:

• Look at the face of the patient. What is your


diagnosis?

Presentation of (I easel--1------

• There is bilateral swelling of parotid glands,


which are firm and nontender.

Bilateral parotid enlargement


My diagnosis is bilateral parotid enlargement.
Q: What are the causes of bilateral parotid
enlargement?
A: As follows:
• Sarcoidosis.
• Alcoholic liver disease or chronic alcoholism.
• Bilateral mumps (usually it is painful).
• Sjogren syndrome.
• Malnutrition.
• DM.
Bilaterar parotid enlargement • Lymphoma.
12 • MISCELLANEOUS _

• Leukaemia. Q: What is Mikulicz syndrome?


• Mikulicz syndrome. A: It is the enlargement of salivary and lacrimal glands.
The causes are:
Q: What else do you want to examine in this patient?
A: As follows: • Sarcoidosis.
• Dryness of eye and mouth (Sjogren syndrome). • Lymphoma.
• Lymph node (sarcoidosis, lymphoma and • Leukaemia.
leukaemia). • Tuberculosis.
• Other evidences of sarcoidosis (erythema nodo- • Sjogren syndrome.
sum and lupus pernio). Secondary Sjogren syndrome is called Mikulicz dis-
• Other collagen diseases (in secondary Sjogren ease (where no salivary or lacrimal gland enlarge-
syndrome). ment is present).

Xanthelasma
Usual instructions are:
• Examine the face. What does it present?

Presentation of a Case

• There are yellowish plaque or nodular lesions at


the upper eyelid (may be also in lower eyelid),
near the inner canthus in one or both eyes.

My diagnosis is xanthelasma. Tendon xanthoma (feet)

Xanthelasma

Tuberous xanthoma (elbow)

Corneal arcus

Eruptive xanthoma Tendon xanthoma (knuckle)


_ SHORT CASES IN CLINICAL MEDICINE

Q: What else do you want to examine?


A: Corneal arcus, xanthoma in other parts (patella,
Achilles tendon and dorsum of the hand) and evi-
dence of primary disease.

Q: What is xanthoma? What are the various types of


xanthoma?
A: Xanthomas are deposits of fatty material in the
skin and subcutaneous tissue and tendons due to
primary or secondary hyperlipidaernia. The four
types are:

• Eruptive xanthoma (multiple, yellow or papule


Tendon xanthoma, in Achilles tendon in trunk and buttock).
• Tendon xanthoma (subcutaneous nodules
attached to tendons over dorsum of fingers and
Achilles tendon).
• Tuberous xanthoma (elbow and knee).
• Palmar xanthoma.

Q: What are the diseases associated with hypercholes-


terolaemia and hypertriglyceridaemia?
A: As follows:
• Hypercholesterolaernia: Tendon xanthoma,
Tendon xanthoma (hands) tuberous xanthoma, xanthelasma, corneal arcus,
atherosclerosis and ischaemic heart disease
(IHD).
• Hypertriglyceridaemia: Acute pancreatitis,
lipaemia retinalis and retinal vein thrombosis
(alone it is not associated with atherosclerosis
and IHD).
• Hypertriglyceridaemia associated with eruptive
xanthoma.
• Mixed hyperlipidaernia: Tuberous, tendon, and
palmar xanthoma.

Q: What are the causes of lipaemia retinalis?


A: Hypertriglyceridaemia and OM.

Tendon xanthoma in hands and leg

Q: What is xanthelasma? What are the causes of


xanthelasma?
A: These are yellowish plaque in the subcutaneous or
intracutaneous tissues due to deposition of choles-
terol or lipids in the eyelids. The causes are:
1. Familial.
2. Primary biliary cirrhosis.
3. Hyperlipidaemia (types II and III).
4. Others
• Hypothyroidism.
• Diabetes mellitus.
• Nephrotic syndrome. Lipaemia retinalis

• Alcoholism.
• Drugs (thiazide diuretic, cycJosporine, steroid, Q: How to treat hypercholesterolaemiat
androgen and oral contraceptive pill). A: As follows:
12 • MISCELLANEOUS _

1. General measures: • fibrates (clofibrate, bezafibrate and gemfibro-


• Weight reduction in obesity, exercise, diet zil) are used in treating hypertriglyceridaemia.
(avoid cholesterol-containing diet and • Others: nicotinic acid, probucol and fish oil
animal fat). Avoid smoking and alcohol. (omega-3 triglyceride).
Treatment of primary disease.
2. Lipid lowering drugs may be given. N.B. Tendon xanthoma may be confused with:
Lipid-lowering drugs are: • Rheumatoid nodule.
• HMG Co-A reductase inhibitors (sirnvastatin, • Tophi of gout.
pravastatin and lovastatin) are used in • Neurofibroma.
treating hypercholesterolaemia. • Lipoma.

Sturge-Weber Syndrome
Usual instructions are: Q: What history do you like to take into consideration ~
A: Epilepsy (on the side opposite to the skin lesion).
• Examine the face. What are your findings? What is
the diagnosis?
Q: What is Sturge-Weber syndrome?
A: It is a disease characterized by capillary or cavern-
ous haemangioma (port-wine stain) along the
Presentation of a Case
cutaneous division of trigeminal nerve (commonly
• There is port-wine stain (reddish, slightly first or second division). There is venous haernan-
pigmented area) at the right outer part of face gioma in subjacent leptomeninges, which may
near the outer and upper part of right eye. spread causing atrophy of cortex. It may be sporadic
or inherited as autosomal dominant. Underlying
brain damage is a rare cause of infantile hemiplegia,
My diagnosis is Sturge-Weber syndrome.
mental retardation and epilepsy. Lesion is on the
face or the trunk in a dermatological distribution.
In middle age, it may be dark with formation of
angiomatous nodules. The patient may have men-
tal retardation.

Q: What investigations do you suggest?


A: X-ray of the skull that shows tramline calcification
(in cortical capillaries) and computed tomography
(CT) or magnetic resonance imaging (MRJ)
investigations.

Q: What are the findings in the eye in Sturge-Weber


syndrome?
A: As follows:
• Glaucoma (which may lead to blindness).
• Strabismus.
• Buphthalmos (ox-eye appearance).
• Optic atrophy.
• Angiomata of choroid.

Sturge-Weber syndrome Treatment: Unsatisfactory laser therapy for skin lesion.


_ SHORT CASES IN CLINICAL M~DICINE

Hereditary Haemorrhagic Telangiectasia


Usual instructions are:

• Examine the face. What are your findings? What is


the diagnosis?

Presentation of a Case If----------,

• There is telangiectasia in lip, face, under surface


of tongue, palate, buccal mucosa and nasal
mucosa.

My diagnosis is hereditary haemorrhagic telangiecta- Telangiectasia (lip)


sia (also called Osler-Weber-Rendu syndrome).
Q: What is telangiectasia? What are the sites of
telangiectasia?
A: It is the localized collection of multiple non-
contractile capillaries. If punctured, it shows pro-
longed bleeding time. It is found in lip, face, tongue
(also under surface). buccal mucosa, nasal mucosa,
nail bed, palm, feet, and gastrointestinal tract, also, in
any part of the body (lungs, nervous system, liver etc).

Q: What are the causes of telangiectasia?


A: As follows:
Telangiectasia (tongue - on side) • SLE.
• Dermatomyositis.
• Necrobiosis lipoidica diabeticorum.
• Systemic sclerosis.
• Carcinoid syndrome.
• Topical steroid.
• Ataxia telangiectasia.
• Hereditary haemorrhagic telangiectasia.

Q: What is hereditary haemorrhagic telangiectasia 1


A: It is a disease inherited as autosomal dominant,
characterized by the formation of multiple tel-
angiectasia in the skin and mucous membrane in
different parts of the body. It is a family disorder,
Telangiectasia (tongue - undersurface) caused by mutations in various genes. The disease
is frequently heralded by recurrent epistaxis in early
childhood. In some cases, telangiectasia in GIT may
cause blood loss.
Sometimes, pulmonary arteriovenous (AV) fistula
may develop resulting in haernoptysis, cyanosis and
clubbing. Paradoxical embolism, stroke and cerebral
abscess may occur. AV aneurysm may also occur in
liver. In the eye, bloody tear (conjunctival telangi-
ectasia), retinal haemorrhage, and even detachment
may occur. Neurological telangiectasia may cause
haemorrhage and the formation of bland or mycotic
Telangiectasia (toe and sole) aneurysm.
12 • MISCELLANEOUS _

Q: What are the presentations? 2. If epistaxis is the main symptom:


A: As follows: • Oestrogen therapy (induces squamous
• Epistaxis (common), usually recurrent and metaplasia of nasal mucosa) is used.
sometimes, the only site of bleeding. • Laser ablation.
• GIT bleeding. • Septal dermoplasty.
• Haemoptysis. • Antifibrinolytic agent (aminocaproic acid).
• Bleeding from other sites.
3. Skin lesion:
• Anaemia (due to chronic blood loss, especially
• Cosmetic surgery
from GIT).
• Laser ablation.
Q: How to treat HHT?
4. GIT: Photocoagulation therapy.
A: As follows:
1. General measures: 5. Lung arteriovenous malformation: emboliza-
• Continuous iron therapy. tion or ligation of the artery or surgical resection.
• Sometimes, blood transfusion in severe cases. (Individual lesions should not be cauterized.)

Yellow Nan Syndrome

Usual instructions are:


• Examine the fingers or toes. What are your findings?
What is the diagnosis?
• Perform the general examination.

Presentation of a Case ~f--------'-'

• The nails are thick, curved from side to side and


yellow (or greenish yellow) with onycholysis
(separation of distal part of nail plate from its bed).
• Cuticles and lunulae are lost.
• Finger tips: bulbous and uncovered (stunted nail
growth).
Yellow nail syndrome (lymphoedema)

My diagnosis is yellow nail syndrome. Q: What else do you like to examine?


A: Ankle oedema (non-pitting). Also, I want to exam-
ine the chest to check for bronchiectasis and pleural
effusion (also may be chronic obstructive pulmo-
nary disease [COPD) and malignant neoplasm).

Q: What is yellow nail syndrome?


A: it is an inherited disease associated with hypoplasia
of the lymphatic system, characterized by thick and
yellow nails and lymphoedema of legs.
N.B. Other associations of this syndrome are
D-penicillamine therapy, nephrotic syndrome,
hypothyroidism and acquired immunodefi-
Yellow nail ciency syndrome (AIDS).
__ SHORT CASES IN CLINICAL MEDICINE

Normal Case
It is not uncommon that occasionally examiner may palpable spleen and small abdominal mass). This
ask to examine a normal patient to assess the efficiency time, examine very carefully.
of a candidate's clinical skill or to see whether a candi- • Examine the cardiovascular system (CVS):there may
date can examine systematically. not be any abnormality in the heart, and the candi-
date make a "commission" of having murmur or rub.
• It is frequently asked: "Examine the fundus of the
• Perform the neurological examination; examine
patient". The candidate makes many commissions
the reflexes, sensory test, cranial nerve and so on.
in a normal fundus.
• Show how to see anaemia, jaundice, clubbing,
• Examine the abdomen: There may not be any
oedema, lymph nodes, thyroid, pulse (quite
abnormality and you should present the case
common in under-graduate examination).
sincerely that there are no abnormal findings.
However, remember that if the examiner asks N.B. Remember the following points:
to examine the abdomen again, probably some • Commission is dangerous than omission.
findings have been missed (just palpable liver, just • Omission is safer than commission.
Common
Interpretations
in Medicine
CHAPTER 13

DATA INTERPRETATION
"A good clinician is a good physician. Clinical medicine will always persist as a basic diagnostic tool.
Investigations, even sophisticated ones, will remain as complimentary to the clinician"
-The author

Hepatology

I N.B. High bilirubin with high alkaline phosphatase is


Case 01: I
highly suggestive of obstructive jaundice. Serum
Obstructive Jaundice glutamic pyruvate transaminase (SGPT) may be
slightly high. Other investigations like cr scan of
A 45-year-old male presented with the following
abdomen, ERCP, MRCP may be necessary.
blood report:
• Serum bilirubin: 20 mg/dl, (normal up to
1.2 rng/dl.). Case 02:
• SGPT: 80 U/L (normal <20 U/L).
Acute Viral Hepatitis
• Serum alkaline phosphatase: 1400 UIL (normal
20-100 U/L). A 26-year-old man presents with anorexia, nausea
• Prothrombin time: 24s (control: 12s). and repeated vomiting for 3 days. He also noticed
high-coloured urine and pain in right upper
abdomen for 2 days. Investigation reveals:
Q: What is the likely diagnosis?
A: Obstructive jaundice. • Hb 14 gm/dl, WBCs 3900/cmm, polymorphs
46%, lymphocytes 50%, eosinophil 4%.
Q: Mention three important points in the history of • Serum bilirubin: 7.2 mg/dl. (normal up to 1.2
the patient. mg/dl.).
A: As follows: • SGPT: 800 lUll (normal <20 IU/L).
• Serum alkaline phosphatase: 95 lUll (normal
• Ceneralized itching.
20-100 lUll).
• Stool: Clay, pale or muddy coloured.
• Pain in the epigastrium or upper abdomen.

Q: Mention one investigation helpful for the Q: Mention two physical signs you should look for.
diagnosis. A: As follows:
A: Ultrasonography (USG) of whole abdomen (or
• Jaundice.
hepatobiliary system).
• Liver (enlarged and tender).
Q: Write down four important causes of this condition.
Q: What is the likely diagnosis?
A: As follows:
A: Acute viral hepatitis.
• Choledocholithiasis.
• Carcinoma of the head of pancreas. Q: Mention one investigation that is helpful to see the
• Periampullary carcinoma. prognosis.
• Cholangiocarcinoma. A: Prothrombin time.
_ SHORT CASES IN CLINICAL MEDICINE

Q: Mention two other investigations. Q: Mention two haematological investigations to find


A: As follows: out the cause.
A: As follows:
• Viral screening (HBsAg, anti-HAV, anti-HEV).
• Ultrasonogram of hepatobiliary system. • Peripheral blood film (shows microcytic
hypochromic anaemia in hereditary haemolytic
Q: Mention two other differential diagnoses.
anaemia, macrocytic in autoimmune anaemia).
A: As follows:
• Reticulocyte count (by supravital stain): High.
• Drug-induced hepatitis.
Q: Mention two further investigations to confirm your
• Leptospirosis.
diagnosis.
Q: Mention two complications. A: As follows:
A: As follows:
• Haemoglobin electrophoresis (to see type of
• Acute fulminating hepatic failure. hereditary haemolytic anaemia).
• Chronic liver disease. • Coombs test (to exclude autoimmune haemolytic
anaemia).
Q: Mention one serious complication.
A: Acute fulminating hepatic failure.
N.B. Anaemia with high bilirubin but normal hepatic
enzyme (normal SGPT, alkaline phosphatase) is
N.B. With the above history and high bilirubin, high highly suggestive of haemolytic anaemia. Mostly
SGPT is suggestive of acute viral hepatitis. Alkaline it may be due to hereditary haemolytic anaemia
phosphatase is normal or slightly high. Usually B (e.g. thalassaernia major) or autoimmune haemo-
and C infection may cause chronic liver disease. lytic anaemia.
Hepatitis E may be serious, if associated with
pregnancy.
Case 04:
Gilbert Syndrome
Case 03:
A young student presented with weakness, anorexia,
Haemolytic Jaundice
nausea and high-coloured urine. He was suffering
A 20-year-old man presented with high-coloured from sore throat 10 days back There is history of
urine and yellow colouration of the whole body. previous similar attack On examination: Anaemia -
There is no history of anorexia, nausea or vomiting. mild, jaundice - mild, liver - just palpable, non tender.
Laboratory investigations reveal: Investigations show:

• Hb 8 g/dl, WBCs 6600/cmm, polymorphs 65%, • CBC: Hb 10.2 g/dl, ESR 32 mm in 1st hour,
lymphocytes 20%, platelet count 1,80,000/cmm, WBCs 9900/cmm, polymorphs 61%, lympho-
ESR95 mmHg in the 1st hour. cytes 35%, monocytes 4%.
• Serum bilirubin: 3.2 mgJdL (normal up to • Reticulocyte count: 0.7% (normal 0.2-2%).
1.2 mgJdL). • Total bilirubin: 52 umol/L (normal <17 umol/L),
• SGPT: 45 lUlL (normal <20 lUlL) • Alanine transaminase (ALT) (SGPT): 15 lUlL
• Serum alkaline phosphatase: 250 lUlL (normal (normal <20 lUlL).
20-100 lUlL) • AST (SGOT): 21 lUlL (normal <25 lUfL).
• Alkaline phosphatase: 42 lUlL (normal 20-100
lUlL).
Q: What is the likely diagnosis? • USG of abdomen: Mild hepatomegaly.
A: Haemolytic jaundice.
Q: What is the likely diagnosis?
Q: Mention three important physical findings in this A: Gilbert syndrome.
patient.
A: As follows: Q: Suggest two investigations.
A: As follows:
• Anaemia.
• Jaundice. • 48 h. 400 Kcal restriction test.
• Splenomegaly. • IV 50 mg nicotine (there is rise of bilirubin).
Q: What treatment should be given in this patient? asymptomatic. Jaundice is usually mild and occurs
A: As follows: intermittently during infection or prolonged fast-
ing. Liver enzymes are normal and there are no
• Reassurance. signs of liver disease. It is due to defect in the
• Prolong fasting should be avoided.
uptake of bilirubin by the liver and also there is a
• Therapeutic trial with phenobarbitone 60 mgTDS.
deficiency of UDP-glucuronyl transferase activity,
N.B. Recurrent jaundice but presence of normal liver which conjugates bilirubin with glucuronic acid.
enzymes in a young patient is highly suggestive Liverbiopsy is normal. Other causes ofnonhaerno-
of Cilbert syndrome. It is a type of unconjugated lytic hyperbilirubinaemia are Criggler-Najjar
nonhaernolytic hyperbilirubinaemia, occurs in syndrome (type 1 and 2), Dubin-Johnson syn-
2-7% of normal individual, some cases inherited drome (liver is black due to increased deposition
as autosomal dominant. Most patients remain of lipofuscin and melanin) and Rotor syndrome.

Nephrology

Q: Write down principles of management.


Case 01: A: As follows:
Acute Glomerulonephritis
• Salt and fluid restriction.
A 12-year-old boy presents with scanty micturition • Diuretic such as frusemide.
and puffy face. Urine examination shows: • Antibiotic: Penicillin.

• Colour: Smoky. N.B. History of sore throat followed by scanty,


• Albumin +. smoky urine associated with puffy face and
• Pus cell 0-2/HPF. periorbital oedema is highly suggestive of acute
• RBC 20-30/HPF. glomerulonephritis (usually poststreptococcal
• RBCcast: Present. glomerulonephritis) .

Q: What is the probable diagnosis?


A: Acute glomerulonephritis. Case 02:
Nephrotic Syndrome
Q: Mention one history you should take.
A: History of sore throat 1-3-weeks back. A 7-year-oJd girl presented with swelling of the
whole body with scanty micturition for 10 days.
Q: Write down three clinical signs you should look for. Her urine examination reveals:
A: As follows:
• Colour: Straw.
• Periorbital oedema. • Albumin +++.
• Blood pressure (high). • RBC: Nil.
• Lungs [bilateral basal crepitation: Indicates acute • Pus cell 0-2/high powered field (HPF).
left ventricular failure (LVF)I. • Fatty cast present.
Q: Write four further investigations.
A: As follows:
Q: What is the likely diagnosis?
• Blood urea and serum creatinine.
A: Nephrotic syndrome.
• Serum electrolytes.
• Ultrasonogram of renal system. Q: Mention three investigations.
• X-ray chest (PA view). A: As follows:
Q: Mention two complications.
• Serum total protein and albumin.
A: As follows: • 24 h urinary protein (>3 g).
• Acute LVF. • Serum lipid profile (there is high cholesterol and
• Acute renal failure. triglyceride).
_ SHORT CASES IN CLINICAL MEDICINE

Q: Mention one drug you think that should be given


to this patient.
Case 04:
A: Prednisolone. Acute Pyelonephritis
A30-year-old female presents with high temperature,
N.B. Generalized oedema, massive proteinuria and anorexia, nausea, vomiting, frequency of micturition
hypoalbuminaemia are suggestive of nephrotic and pain in the lumbar region. Urine examination
syndrome. Commonest cause in children is min- shows:
imal-change disease and in adult membranous
• Albumin +.
or proliferative glomerulonephritis. Prognosis is
• RBC few.
better in children, but there may be relapse.
• Pus cell plenty.

Case 03: Sterile Pyuria Q: What is the likely diagnosis?


A: Acute pyelonephritis.
A 40-year-old male presents with low-grade
fever, weakness, loss of appetite and difficulty in
Q: Mention one investigation to confirm the diagnosis.
micturition. Urine examination shows:
A: Urine culture and sensitivity.
• Albumin +.
• RBC S-6/HPF. N.B. Any patient with frequency, burning or difficulty
• Pus cell SO-60/HPF. in micturition associated with high temperature
• Culture: No growth. is suggestive of acute pyelonephritis. Treatment
should be given according to culture and sensitiv-
ity of urine.
Q: What does the urine report shows?
A: Sterile pyuria.
Q: Mention three causes. Case 05:
A: As follows: Acute Interstitial Nephritis
.. Renal tuberculosis. with ARFdue to Cefixime
• Partially treated urinary tract. infection.
A 60-year-old male presented with fever, headache,
• Nongonococcal urethritis.
dry cough, frequency of micturition and weakness for
• Analgesic nephropathy.
10 days. Cefixime was given by a general practitioner.
• Tubulointerstitial nephritis.
After 5 days, the patient complains of scanty urine,
• Prostatitis.
puffiness of face, swelling of legs, polyarthralgia and
• Bilharziasis.
multiple skin rashes. Investigations reveal:
Q: Mention two commonest causes.
• Full blood count: Hb 10.9 gfdL, WBCs 11,200/
A: As follows:
cmm, polymorphs 64%, lymphocytes 28%,
• Renal tuberculosis. eosinophils 8%, ESR 110 mm in 1st hour,
• Partially treated urinary tract infection. platelets l,75,OOO/cmm.
Q: Mention four further investigations. • Urine: Pus cells - plenty, proteinuria (++); total
A: As follows: protein 1 g/24 h.
• Urea: 19 mmol/L (normal 2.5-6.6 mrnol/L).
• Urine foracid fastbacteria (AFB)and mycobacterial • Creatinine: 833 umol/L (normal 60-120 mrnol/L),
culture. • Serum electrolytes: Sodium 130 mmol/L,
• Ultrasonogram of renal system. chloride 90 mmol/L, potassium 6.1 mmol/L,
• Tuberculin test. bicarbonate 17 mrnol/L.
• Chest X-ray.

N.B. Presence of pus cells in the urine but sterile on Q: What is your diagnosis?
culture is called sterile pyuria. Tuberculosis in the A: Acut.e interstitial nephritis with acute renal failure
renal syst.em is the likely cause. (ARF) due to cefixime.
13 • DATA INTERPRETATION _

Q: Suggest two further investigations. • Prednisolone.


A: USC of kidney, renal biopsy. • Dialysis for ARF.

Q: How to confirm the diagnosis? N.B. This patient presents with fever, skin rash, eosi-
A: Renal biopsy. nophilia and renal impairment shortly after
antibiotic therapy. These findings are consist-
Q: How to manage this case?
ent with acute interstitial nephritis. It is an acute
A: As follows:
inflammation of tubular interstitium, probably
• Offending drug should be stopped. due to hypersensitivity reaction.

Neurology

L
Q: What four other investigations will you suggest?
Case 01: A: As follows:
Tubercular Meningitis
• Complete blood count (CBC) with erythrocytic
A 25-year-old male presented with fever for 3 weeks sedimentation rate (ESR).
and disorientation for 3 days. Cerebrospinal fluid • Tuberculin test.
(CSF) study shows: • Chest X-ray (PA view).
• CT scan of head.
• Pressure: High.
• Colour: Clear.
Q: What is the finding on fundoscopy?
• Cytology: Total WBC 350/cmm, neutrophil 5%,
A: Choroid tubercle in retina.
lymphocyte 95%, no RBC.
• Biochemistry: Protein 300 mgJdL (normal up to
40 mgJdL), sugar 50 mg/dl, (low). N.B. History oflow-grade fever,weight loss and anorexia,
• Microbiology: No organisms were found in along with signs of meningitis and typical findings
Cram stains. in CSF (high lymphocytes, high protein and low
sugar) is highly suggestive of tuberculous rneningi-
tis.It should be treated with standard antitubercular
Q: What is likely diagnosis? therapy for at least 9 months plus prednisolone
A: Tubercular meningitis. 60 mg for 3 weeks and then should taper. Com-
plications [e.g. cranial nerve palsy, hydrocephalus,
Q: Mention four differential diagnoses. syndrome of inappropriate antidiuretic hormone
A: As follows: secretion (SIADH), seizure] may occur.
• Viral meningitis.
• Fungal meningitis.
I
• Sarcoidosis. Case 02: I

• Neurosyphilis or meningovascular syphilis. Pyogenic Meningitis

Q: Mention one other investigation in CSF that is A 19-year-old man presented with high-grade fever
helpful for your diagnosis. for 2 days and disorientation for 4 h. Neck rigidity
A: ADA (adenosine deaminase). and Kernig sign are present. CSF examination
shows:
Q: What is the other finding in CSF?
• Colour: Turbid.
A: If it is kept overnight, there is cob web appearance.
• Pressure: Increased.
• Biochemistry: Protein 190 mgJdL, sugar 16 mgJdL,
Q: Write down two important physical signs.
chloride 670 mgJdL.
A: As follows:
• Cytology: Total cells 6200/cmm, polymorphs
• Neck rigidity. 96%, lymphocytes 4%.
• Kernig sign.
_ SHOI'{T CASES IN CLINICAL MEDICINE

Q: What is the likely diagnosis?


A: Pyogenic meningitis. Case 03:
Acute Viral Meningitis
Q: What are the causes of bacterial meningitis? A 24-year-old man presented with fever, headache,
A: As follows: nausea, vomiting for 5 days. Neck rigidity and Kernig
1. Neonate: Gram negative bacilli (Escherichia sign are present. CSF examination shows:
coli, Proteus), Group B streptococci, Listeria • Colour: Clear.
monocytogenes. • Pressure: Increased.
2. Preschool child: Haemophilus inj1uenZlle, Neis- • Biochemistry: Protein 60 mgjdL, sugar 50 mg/dl,
seria meningitidis, Streptococcus pneumoniae, • Cytology: Total cell 100/crnm, polymorphs 2%,
Mvcooaaerium tuberculosis. lymphocytes 98%.
3. Older child and adult: Neisselia meningitidis,
S. tmeumoniae, L. monocytogenes, M. tuberculosis,
Staphylococcus aureus (skull fracture), H. inj1uenzLle. Q: What is the likely diagnosis?
A: Acute viral meningitis.
Q: What is the commonest organism?
Q: What are the common organisms?
A: N. meningitides (also called meningococcus). A: EnterovilUses like Echovirus and coxsackie are the
commonest.
Q: What are the complications?
A: As follows: N.B. Low-grade fever; headache; nausea; vomiting and
• Brain abscess. CSFfindings-clear, slightly high protein with nor-
mal glucose and chloride, with high lymphocyte is
• Hydrocephalus.
• Cranial nerve palsy. suggestive of viral encephalitis. There are usually
no serious sequelae unless encephalitis is present.

Q: What other physical findings will you look in the


body? Case 04: :1-------,
A: Skin rash such as petechial or purpuric, which indi- Subarachnoid Haemorrhage
cates meningococcal septicaemia.
A 45-year-old man presented with sudden, severe
occipital headache followed by unconsciousness.
Q: If the patient develops shock, what is the likely
Neck rigidity and Kernig sign are present. CSF
cause?
shows:
A: Bilateral adrenal haemorrhage (Waterhouse-
Friedriechson syndrome). • Colour: Red (frank blood).
• Pressure: Increased.
Q: What would you look for before doing lumbar
• Biochemistry: Protein 100 mgj dl, sugar 60 mg!dL.
puncture? Why? • Cytology: Plenty of RBCs, polymorphs 60%,
lymphocytes 40%.
A: I will do fundoscopy to see papilloedema [indi-
cates raised intracranial pressure). If it is present,
then lumbar puncture should be avoided because
Q: What is the likely diagnosis?
there may be herniation of cerebellar tonsils
A: Subarachnoid haemorrhage.
through foramen magnum, which compresses the
vital centre in medulla oblongata and causes sud- Q: Mention two causes?
den death. A: As follows:
• Rupture of berry aneurysm.
N.B. In any patient with high fever, headache, nausea, • Rupture of arteriovenous malformation.
vomiting, photophobia with signs of meningism, Q: Mention one fundoscopy finding.
it indicates pyogenic meningitis. A: Subhyaloid haemorrhage.
13 • DATA INTERPRETATION _

Q: Suggest one investigation. • Trauma.


A: CT scan of head. • Bleeding disorders like haemophilia, Christmas
disease.
Q: Mention two other investigations.
A: As follows:
N.B. Sudden, severe thunderclap headache followed
• Magnetic resonance angiography (MRA).
by unconsciousness is highly suggestive of sub-
• Digital substraction angiogram (DSA).
arachnoid haemorrhage. Control of hypertension,
• Cerebral angiography.
administration of nimodipine and surgical oblite-
Q: Mention two causes of haemorrhagic CSF. ration of the aneurysm by dipping is the mainstay
A: As follows: of the treatment.
• Subarachnoid haemorrhage.

Haematology

Case 01: Case 02:


Acute leukaemia Chronic Myeloid Leukaemia

A 35-year-old lady presented with following blood A 45-year-old man presented with weakness, loss
report: of appetite and heaviness in the abdomen for 3
months. Laboratory investigation reveals:
1. Hb 6 g/dL, ESR70 mm in l st hour.
2. WBe: 50 x109/cmm. 1. Hb 9 g/dL, WBCs 1,50,000/cmm, polymorphs
• Neutrophil: 35%. 45%, lymphocytes 8%, myelocytes 22%, met-
• Lymphocyte: 28%. arnyelocytes 14%, myeloblasts 3%, basophils
.' Monocyte: 03%. 8%, platelets 2,10,000/cmm, ESR 90 mm in
• Eosinophil: 05%. 1st hour.
• Atypical cell: 29%. 2. PBF: Normocytic normochromic anaemia.
3. Platelet: 100xl09/cmm.
Q: What are the abnormalities in this blood picture?
A: Anaemia, leucocytosis with myelocyte and meta-
Q: What are the abnormalities in this blood picture? myelocyte, few blasts, high basophil and high ESR.
A: Anaemia, leucocytosis with atypical cell, thrombo- Q: What is your diagnosis?
cytopaenia, high ESR. A: Chronic myeloid leukaemia.

Q: What is the likely diagnosis? Q: What is the cause of abdominal discomfort?


A: Acute leukaemia. A: Splenomegaly, which may be huge.
Q: What investigation do you suggest?
Q: Mention one further investigation. A: As follows:
A: Bone marrow study.
• Bone marrow study.
• Cytogeneticanalysis for Philadelphia chromosome.
N.B. Presence of atypical cell with high WEC count • RNA analysis to see the presence of BCR-ABL
is suggestive of acute leukaemia, which may be gene product.
either myeloid or lymphoblastic. Blast cell may be • Leukocyte alkaline phosphatase (LAP) score
present. (decreases).
Q: What are the therapies that may cure this condition? very-high ESR with marked rouleaux formation
A: Bone marrow transplantation, imatinib and c- in peripheral blood film (PBF). Bone marrow
interferon. study shows presence of atypical plasma cells
(usually >20%). Urine shows Bence-lones
N.B. In a middle-aged patient with huge splenomegaly proteinuria.
and presence of anaemia, leucocytosis with myelo-
cyte and metamyelocyte, few blasts, high basophil,
high ESR is suggestive of chronic myeloid leukae- Case 04: Pancytopaenia}f---------,
mia (CML). Blastic crisis, myelofibrosis may occur.
A 40-year-old male presented with generalized
CML may be associated with low LAP score, high
weakness and gum bleeding for 2 months.
uric acid, high vitamin B12and high LDH.
Investigation reveals:

• Hb: 7 gjdL.
Case 03: • WBC: Is,000/mm3.
Multiple Myeloma • Neutrophil: 16%.
• Lymphocyte: 81 %.
A 65-year-old man presented with generalized body • Platelet: 20,000jmm3.
ache for the last 3 months. Laboratory investigation • ESR:70 mm in Ist hour.
reveals:
• Hb 8 g/dl, WBC normal, platelet 2,00,000jcmm.
• ESR: l l S mm in 1st hour. Q: What is the haematological diagnosis?
• PBF: Normocytic normochromic anaemia with A: Pancytopaenia.
increased rouleaux formation.
Q: Mention three causes.
• Blood Ca": High. A: As follows:
• Aplastic anaemia.
Q: What is the likely diagnosis? • Hypersplenism.
A: Multiple myeloma. • Megaloblastic anaemia.
Q: Write down two important investigations to con- • Aleukaernic leukaemia.
• Paroxysmal nocturnal haemoglobinuria.
firm the diagnosis.
A: As follows: Q: Mention one investigation to confirm the diagnosis?
• Bone marrow study (shows atypical plasma cells). A: Bone marrow study.
• Serum protein electrophoresis or immuno-
electrophoresis. N.B. Anaemia, Jeucopaenia and thrombocytopaenia indi-
• X-ray skull (shows multiple lytic lesions). cates pancytopaenia. Commonest cause is aplastic
anaemia. The causesof aplastic anaemia are-primary
Q: Write down three complications. idiopathic, secondary (bone marrow infiltration,
A: As follows: multiple myeloma, drugs, radiotherapy, etc.).
• Pathological fracture of bone.
• Renal failure. Case 05: Hypochromic fl-------,
• Hyperviscosity syndrome. Microcytic Anaemia
• Bone marrow failure.
• Repeated infection. A 29-year-old female presented with weakness,
palpitation and shortness of breath on exertion.
Q: Mention one drug that may be curative.
Laboratory investigation reveals:
A: Bortezomib.
• Hb 8.5 mgjdL.
N.B. Elderly patient may present with generalized • RBC,WBC and platelet counts are within normal
bodyache, unexplained anaemia, repeated infec- limits.
tion, spontaneous fracture, hyperviscosity • PBF shows hypochromia.
syndrome, bleeding disorder, renal failure in • MCV 65 fL (normal >80 fL).
multiple myeloma. Blood examination shows
13 • DATA INTERPRETATION _

Q: What is haematological diagnosis? • Anaemia.


A: Hypochromic microcytic anaemia. • Jaundice.
• Splenomegaly.
Q: Mention four causes.
A: As follows: Q: What history is important in such condition?
A: Family history of such illness.
• Iron-deficiency anaemia.
• ~-Thalassaemia. Q: What physical sign will you look for?
• Sideroblastic anaemia. A: Anaemia, jaundice, frontal and parietal bossing,
• Anaemia of chronic disorder. mongoloid facies and splenomegaly.
Q: Which one is more likely diagnosis in this patient?
Q: What investigation will you do to confirm the
A: Iron-deficiency anaemia.
diagnosis?
Q: What is the commonest cause of iron-deficiency A: Haemoglobin electrophoresis.
anaemia? Q: How to treat?
A: Bleeding (menorrhagia, haemorrhoid).
A: As follows:
Q: What history do you [ike to take in this female
• Blood transfusion to keep the haemoglobin
patient? above 10 g/dl, usually every 4 months (life span
A: History of menorrhagia. of RBC is 4 months).
Q: What further investigation will you do? • Folic acid daily.
A: As follows: • Iron-containing drugs and diet should be
avoided.
• Iron profile [serum iron, serum ferritin, total iron
• Desferrioxamine to prevent transfusion
binding capacity (TlBC)].
haemosiderosis.
• Haemoglobin electrophoresis.
• Erythropoietin and hydroxyurea may be given.
• Bone marrow study.
• Specific treatment: Allogenic bone marrow
N.B. Microcytic hypochromic blood picture with transplantation.
history of blood loss suggests iron-deficiency • If huge splenomegaly with features of
anaemia. Serum iron and ferritin will be low and hypersplenism: Splenectomy may be done.
TIBC will be high. Treatment includes correction
of anaemia by blood transfusion, iron therapy for N.B. Presence of microcytic hypochromic anae-
3-6 months (to replenish the store) and treat- mia with high reticulocyte count, jaundice
ment of primary cause. and splenomegaly in a young patient indicates
hereditary haemolytic anaemia, most commonly
I ~-thalassaemia major. Other common hereditary
Case 06: r
haemolytic anaemia-HbE disease, thalassaemia
13-Thalassaemia Major E disease (double heterozygous).
A 15-year-old girl presented with weakness, abdom-
inal distension and loss of appetite. Examination of Case 07:
I
r
the abdomen reveals huge splenomegaly and mild Macrocytic Anaemia
hepatomegaly. Laboratory results show:
• Hb 7%, WBC 7200/cmm, neutrophil 62%, A 50-year-old woman presented with weakness
lymphocyte 23%, platelet 2,30,000/cmm. and anorexia for 3 months. Her laboratory
investigations reveal:
• Reticulocyte: 10%.
• Mean cell volume (MCV): 69 fL (normal >80). • Hb 8%, WBC 6700/cmm, neutrophil 65%,
• Serum bilirubin: 3.1 mg/dl, lymphocytes 25%, platelet 2, 10,000/cmm.
• Reticulocyte: 1%.
• MCV: 112 fL (normal up to 96 fL).
Q: What is the likely diagnosis?
• PBF shows macrocytosis and hypersegmented
A: ~-Thalassaemia major.
neutrophils.
Q: What is the triad of signs in this condition? • Serum bilirubin: 1.0 mg/dl,
A: As follows:
_ SHORT CASES IN CLINICAL MEDICINE

Q: What is the haematological diagnosis? Q: What is the likely cause?


A: Macrocytic anaemia. A: Bone marrow infiltration from carcinoma of breast.

Q: What are the causes of macrocytosis? Q: Mention one further investigation to confirm your
A: As follows: diagnosis?
A: Bone marrow study.
• Megaloblastic anaemia (due to B12or folic acid
deficiency) .
• Hypothyroidism. N.B. Leukemoid reaction means that the peripheral
• Chronic liver disease. blood picture resembles leukaemia; but there
• Chronic alcoholism. is no leukaemia. It may be myeloid or lym-
• Haemolysis. phatic. Causes of myeloid leukemoid reaction
• Azathioprine therapy. are-leukoerythroblastic anaemia, infection,
malignancy, acute haemolysis (LIMA). Causes
Q: What next investigation should be done? of lymphatic leukemoid reaction: viral (infec-
A: Bone marrow study. tious mononucleosis, cytomegalovirus infection,
measles, chicken pox), whooping cough. Rarely,
Q: What are [he findings in bone marrow study and
tuberculosis and carcinoma.
how to interpret?
A: As follows:
• If macrocytosis is associated with megaloblast Case 09:
in bone marrow: The diagnosis is megaloblastic Leukoerythroblastic Anaemia
anaemia.
A 65-year-old woman presented with pain in the
• If macrocytosis is associated with normoblastic
epigastrium, loss of appetite, fever, generalized
bone marrow: The causes are hypothyroidism,
bodyache, night sweating, occasional headache,
chronic liver disease, chronic alcoholism,
dizziness and weight loss for 3 months. She is pale
haemolysis, azathioprine therapy, etc.
and emaciated. There is massive splenomegaly.
Q: What other investigationsdo you suggest? Investigations reveal:
A: Serum vitamin B12and folic acid assay. • Hb 9.1 gldL, WBC 18,500/cmm, polymorphs 60%,
lymphocytes 28%, rnyelocytes6%, rnetarnyelocytes
N.B. Anaemia with high MCV indicates macrocytic 2%, myeloblasts 1%, erythroblast 3%, ESR93 mm
anaemia. Most common cause is megaloblastic in 1st hour, platelets 6,65,000/cmm.
anaemia due to vitamin B12or folic acid deficiency. • Liver function tests: Serum bilirubin 28 umol/L,
SGPT 38 IU/L, alkaline phosphatase 200 IU/L.
Case 08: ,1------- • Chest X-ray: Normal.
Leukemoid Reaction

A 60-year-old man presents with fever, cough Q: What is the haematological diagnosis?
and weight loss. She was suffering from breast A: Leukoerythroblastic anaemia.
cancer, which was treated by mastectomy and
chemotherapy. Her blood picture shows: Q: What is the likely diagnosis?
A: Myelofibrosis.
• Hb 11.4 g/dL, WBCs 23, 700/cmm, polymorphs
82%, lymphocytes 6%, metarnyelocytes 3%, Q: What additional findings may be seen in blood
rnyelocytes 3%, prornyelocytes 1%, myeloblast film?
1%, erythroblast 4%, platelets 1,80,000/cmm. A: Tear-drop poikilocytes.
• ESR: 10 mm in 1st hour.
• Chest X-ray: Absent left breast shadow. Q: What are the other causes of such blood picture?
A: As follows:

Q: What is the haematological diagnosis? • Secondary deposits in bone marrow.


A: Leukemoid reactjon. • Lymphoma.
13 • DATA INTERPRETATION _

• Multiple myeloma. Q: What is the diagnosis?


• Active haemolytic anaemia. A: Blastic crisis in CML
Q: Suggest two other tests. Q: What is the cause of abdominal pain?
A: As follows: A: Splenic infarction.

• Bone marrow study: Maybedrytap.Trephinebiopsy Q: Suggest one investigation to confirm your diagnosis.
is needed, which shows increased megakaryocyte, A: Bone marrow study.
ina-eased reticulin and fibrous tissue.
• LAPscore: High. N.B. Appearance of increased number of blast cells in
PBF in a patient with CMLsuggests blastic crisis. It
Q: Mention one dangerous complication. may be myeloid or lymphatic.
A: Transform to acute myeloid leukaemia (AML)
(10-20% cases). I
Case 11: Idiopathic
Q: What treatment should be given? Thrombocytopenic Purpura
A: As follows:
9 days after recovering from a viral fever, a 13-year-
• Blood transfusion.
old girl presents with several small bruises on her
• Folic acid. arms and legs. Investigations reveal:
• Hydroxycarbamide (hydroxyurea) may be given.
• In young patient, bone marrow transplantation. • Full blood count: Hb 11.3 gJdL, WBCs
• Radiotherapy for huge spleen. lO,700/cmm, polymorphs 59%, lymphocytes
• If evidence of hypersplenism and huge spleen 38%, ESR 60 mm in 1st hour. Platelet count
with pressure symptoms, splenectomy may be 80,000/cmm.
necessary. • Blood film: Normocytic and normochromic.
• Chest X-ray: Normal.
N.B. In any elderly patient with huge splenomegaly
and leukoerythroblastic blood picture, the likely
cause is myelofibrosis. PBF shows tear-drop or Q: What is the likely diagnosis?
pear-drop poikilocytes. Myelofibrosis is a disorder A: Idiopathic thrombocytopenic purpura.
of unknown cause, characterized by bone marrow Q: Suggest two diagnostically useful investigations.
fibrosis, extramedullary haemopoiesis and leu- A: Bone marrow study, antiplatelet antibody.
koerythroblastic blood picture due to neoplastic
Q: What disease should be excluded in such finding?
proliferation of primitive stem cells. It is common
A: Systemic lupus erythematosus (SLE).
above 50 years. There may be peptic ulcer, pruritus
after hot bath, gout, etc. Q: What treatment should be given?
A: In child-usually self-limiting. If no improvement:
Case 10: • Prednisolone (2 mg/kg)
Blastic Crisis in Chronic Myeloid • If persistent bleeding, IjV immunoglobulin.
Leukaemia (CML) • Platelet transfusion, if persistent bleeding.
• If thrombocytopaenia persists for more than
A 53-year-old man is diagnosed as a case of chronic 6 months, it is chronic. In that case, splenectomy
granulocytic leukaemia. He was treated with should be considered.
chemotherapy and responded well. Three years later,
the patient presented with the complaints of severe N.B. Purpuric spot in a patient with recent history of
weakness, loss of weight, lethargy, epistaxis and pain viral fever, associated with low platelet count and
in left upper abdomen. Investigations show: increased megakaryocytes in bone marrow is sug-
• Hb 7.5 g/dl, WBC 12,800/cmm, platelets gestive of idiopathic thrombocytopenic purpura
1,00,000/cmm, RBC 2.8 million/cmm, ESR 85 (ITP). In ITP, low platelet, increased megakaryo-
mm in 1st hour. cyte in bone marrow, prolonged bleeding time,
• Differential count: Neutrophils 34%, basophil normal clotting time are common. There may
8%, lymphocyte 6%, myeloblast 40%, myelocyte be antiplatelet antibody and anticardiolipin
10%, metamyelocyte 2%. antibody. Initially, SLEand antiphospholipid syn-
drome may present like ITP.
Case 12: Polycythemia 11- -----, Case 13:
Rubra Vera Disseminated Intravascular
Coagulation (DIC)
A 41-year-old man presented with frequent
headache, dizziness, lack of concentration, pruritus A 29-year-old woman was admitted in the hospital
and heaviness in the left upper abdomen for with the complaints ofPV (blood loss pervaginum)
3 months. Laboratory investigations reveal: bleeding and high grade, continuous fever,
following a self-induced abortion. She looks toxic,
• I-Ib 17.6 g/dl, WBCs 21,000/cmm, polymorphs
anaemia - severe, BP 80/60 mrnl-lg, pulse 124/min,
83%, lymphocytes 15%, eosinophils 2%, RBCs
temperature 39.6 "C. Lower abdomen - very tender.
8.2 million/cmm, platelets 8,50,000/cmm, ESR
Investigations reveal:
1 mm in 1st hour.
• Packed cell volume (PCV): 65% (normal up • Full blood count: Hb 5.0 g/dl, WBCs 28,700/
to 45%). cmrn, polymorphs 87%, lymphocytes 13%,
• Random blood sugar (RES): 7.2 mmol/L. platelets 30,000/cmm, ESR 85 mm in l st
• Chest X-ray: Normal. hour.
• PBF: Microcytic, hypochromic and fragmented
RBC.
• Prothrombin time: 30 s (control 12).
Q: What is your diagnosis?
• AP1T: 60 s (control 32).
A: Polycythemia rubra vera (PRV).
• Fibrinogen: 0.05 gil (1.5-4.0 g).

Q: Suggest one investigation.


A: Measurement of red cell mass (increased).

Q: What is the main diagnosis in this blood disorder?


Q: Suggest one treatment.
A: Disseminated intravascular coagulation (DIC).
A: Venesection.

Q: What drugs can be used? Q: Mention four abnormalities in this blood picture.
A: Radioactive phosphorus, hydroxyurea, interferon. A: DIC with microangiopathic haemolytic anaemia
with iron-deficiency anaemia along with
septicaemia.
Q: Mention fOUT complications.
A: As follows:
Q: Give three possible causes of haematologicaJ
• AML. abnormality.
• Thromboembolism (cerebral, coronary).
A: Septicaemia, retained product of conception or
• Hypertension.
amniotic fluid embolism.
• Gout.
• Peptic ulcer.
• Myelofibrosis.
N.B. DIC is a haemorrhagic disorder in which dif-
N.B. This patient has high haemoglobin and RBC, fuse intravascular clotting causes a haemostatic
also high PCV, which suggests polycythaemia. defect resulting from utilization of coagulation
High WEC and platelet count associated with factor and platelet in the clotting process. There
splenomegaly are all in favour of polycythemia is secondary activation of fibrinolysis, leading to
rubra vera. Red cell volume is high in PRV. production of fibrin degradation products (FDP).
There is increased LAP (leukocyte alkaline phos- 'the consequence of these changes is a mixture of
phatase), vitamin B12 and uric acid (may cause initial thrombosis, followed by bleeding tendency
gout). Bone marrow shows hypercellular with due to consumption of coagulation factors and
increased megakaryocyte. fibrinolytic activity.
13 • DATA INTERPRETATION _

Respiratory
N.B. In restrictive lung disease, both PVC and FEV! are
Case 01: Interstitial proportionately reduced; but ratio of this two is
Lung Disease normal. Total lung capacity is reduced and resid-
ual volume is reduced or normal. CO transfer
A 46-year-old woman is admitted in the emergency
factor is also low.
department with shortness of breath, cough,
swelling in both feet and extreme weakness.
Respiratory function tests reveal: Case 02: Emphysema !
I

• Vital capacity: 2.0 L (predicted 2.4-3.6 L). A lady of 42 years, housewife, has been suffering
• FEV!: 2.1 L (predicted 2.25-3.25 L). from breathlessness, occasional dry cough and
• Transfer factor: 4.1 L (predicted 5.8-8.7 L). weight loss for 2 years. Her lung function test shows:
• Residual volume: 1.1 L (predicted 1.55-2.32 L).
• FVC: 2.00 L (predicted 2.4-3.6 L).
• FEV!: 1.45 L (predicted 2.25-3.25 L).
• RV: 2.89 L (predicted 1.5-2.32 L).
• FRC: 3.56 L (predicted 2.17-3.25 L).
Q: What pulmonary defect is present?
• TLC: 5.89 L (predicted 3.96-5.66 L).
A: Restrictive lung disease.
• TLCO: 4.7 mmol/rnin/kl'a (predicted 5.8-8.7 L).

Q: What is the likely diagnosis with the above findings?


A: Interstitial lung disease. Q: What is the lung function test?
A: Obstructive airway disease.
Q: Suggest five investigations to find out cause. Q: What is the most likely diagnosis?
A: As follows: A: Emphysema.
• Chest X-ray. Q: Suggest two possible causes.
• CT scan of chest. A: <XI-Antitrypsin deficiency, cigarette smoking.
• ~C (arterial blood gas) analysis.
• Bronchoscopy and bronchoalveolar lavage. Q: Name two obstructive airway disease.
• Transbronchial or open lung biopsy. A: Chronic obstructive pulmonary disease (COPD),
bronchial asthma.
Q: What are the causes of restrictive lung disease?
N.B. In obstructive lung disease, forced expiratory vol-
A: As follows:
ume in 1second (FEVJ is markedly reduced; forced
• Sarcoidosis. vital capacity (FVC) is also reduced. The ratio of
• DPLD. these two is also reduced. CO transfer factor is low.
• Ankylosing spondylitis. Residual volume and total lung capacity are high.

Endocrine

Case 01: Primary • Plain X-ray KUB (kidneys, ureters, bladder):


Nephrocalcinosis.
Hyperparathyroidism
• USC of abdomen: Calcification in both kidneys.
A 52-year-old woman presented with frequency • Urine: Protein (+), few pus cells.
of micturition, extreme weakness, loss of appetite, • RES: 6.9 mmol/L,
constipation, abdominal pain and weight loss ~or 7 • Creatinine: 1.4 mgjdL.
months. Investigations reveal: • Serum electrolytes: Sodium 139 mmol/L,
chloride 110 mrnol/L, potassium 3.6 mrnol/L,
• Full blood count: Hb 10.6 gjdL, WBCs 8,900/ bicarbonate 22 mmol/L,
cmm, polymorphs 55%, lymphocytes 43%,
monocytes 2%, ESR 90 mm in 1st hour.
__ SHORT CASES IN CLINICAL MEDICINE

Q: What is the likely diagnosis? Q: Write down two other causes of such investigation
A: Primary hyperparathyroidism. findings.
A: As follows:
Q: Suggest five investigations to confirm your diagnosis.
• Toxic multinodular goitre.
A: As follows:
• Toxic adenoma.
• Serum calcium, phosphate and alkaline
Q: Write down two important finding by examining
phosphatase.
the nervous system of this patient.
• Parathormone assay.
A: As follows:
• Hydrocortisone suppression test.
• X-ray of hand (to see subperiosteal erosion in the • Tremor of the outstretched hand .
medial side of phalanges) and X-ray of skull (to • Jerks are exaggerated
see pepper-pot appearance). Q: Write down four other important investigations.
• USG or Cl' scan of neck (parathyroid adenoma or A: As follows:
hyperplasia) .
• Other test: Thallium/technetium subtraction • Radio-iodine uptake test.
scan of thyroid and parathyroid. • Thyroid scan.
• USG of neck.
Q: What single treatment should be given immediately? • TSH receptor antibody (TRAb) test.
A: Plenty of fluid (infusion of normal saline 4-6 L
Q: Write down the modalities of treatment.
daily) for hypercalcaemia. A: As follows:
• Antithyroid drugs: Carbimazoleor propylthiouracil.
N.B. The above symptoms associated with hyper-
• Radioactive iodine therapy.
calcaemia and nephrocalcinosis indicates
• Thyroid surgery.
hyperparathyroidism. It may be primary (due
to adenoma, hyperplasia or carcinoma of par- N.B. Graves disease is characterized by diffuse goi-
athyroid), secondary (chronic renal failure, tre, exophthalmos and dermopathy. The natural
malabsorption, rickets or osteomalacia) or terti- history of Graves disease is hyperthyroidism, fol-
ary (autonomous from secondary). In mild and lowed by euthyroid and later, hypothyroidism.
asymptomatic case follow-up. Total parathyroid- TSH receptor antibody (TRAb) is high. Antirni-
ectomy with transplantation of parathyroid in crosomal (anti peroxidase) and anti thyroglobulin
the forearm muscles is done in primary hyperpar- antibody may be present in low titre.
athyroidism. Surgery is also required for tertiary
hyperparathyroidism. Treatment of secondary
causes should be done.
Case 03: Primary
Hypothyroidism
A 35-year-old female presented with generalized
Case 02: Thyrotoxicosis 11----------,
weakness and weight gain. Her thyroid function
due to Graves Disease test reveals:
A 32-year-old lady presented with bilateral • Serum Ff3: 2 pmol/L (normal 1.3-3.5 pmoljL).
proptosis, gritty sensation and discomfort in both • Serum Ff4: 7 pmol/L (normal 70-160 pmolfL).
eyes, palpitation and weight loss. Investigation • TSH: 35 mIU/L (normal 0.5-5.1 mlU/L).
reveals:
• Serum Ff3: 9.0 pmol/L (normal 1.3-3.5 pmol/L). Q: What is the most likely clinical diagnosis?
• Serum Ff4: 215 pmoljL(normaI70-160pmol/L). A: Primary hypothyroidism .
• TSH: 0.04 mIUjL (normal 0.5-5.1 mIUjL).
Q: Mention four important clinical signs of this patient.
A: As follows:

Q: What is your diagnosis? • Hoarseness or croaky voice.


A: Thyrotoxicosis due to Graves disease. • Puffiness of face.
13 • DATA INTERPRETATION __

• Bradycardia. N.B. In any extrathyroidal illness [acute myocardial


• Delayed relaxation of ankle jerk infarction, pneumonia, cardiovascular disease
(CVD)], there may be abnormal thyroid func-
Q: What are the ECG changes in hypothyroidism? tion tests; although the patient is euthyroid. This
A: As follows: is called sick euthyroid syndrome. There may be
low total and free T3 and T4 along with low or
• Sinus Bradycardia.
normal thyroid stimulating hormone (TSH). It
• Low-voltage ECG tracing.
is confused with secondary hypothyroidism. The
• T-wave inversion.
tests should be repeated after recovery from the
systemic disease, usually after 6 weeks.
Q: What is the treatment?
A: Lifelong thyroxine replacement, starting with
low dose (25 microgram/day) and then gradually
increasing the dose. Case 05: de Quervain
Thyroiditis
N.B. Weight gain, increased sleepiness, cold intoler- A 32-year-old woman presents with fever, dry
ance and constipation are the usual features of cough, pain in the throat, headache, bodyache,
hypothyroidism. Polyserositis may occur in myxo- palpitation and intolerance to heat for 1 week.
edema. Muscular pain, arthralgia, effusion in any There is fine tremor of the outstretched hands.
serous cavity may occur. SlADH, high creatinine Palms are warm and sweaty. Thyroid is diffusely
phosphokinase (CPK), serum glutamic pyruvic enlarged, soft, nontender, no bruit. Investigations
transaminase (SePT), serum glutamic oxaloace- reveal:
tic transaminase (SGOT), lactic dehydrogenase
• Full blood count: Hb 11.1 g/dl, WBCs 3,500/
(LDH) may also be found.
cmm, polymorphs 48%, lymphocytes 52%, ESR
110 mm in 1st hour, platelets 3,65,OOO/cmm.
• Chest X-ray: Normal.
Case 04: :f------~ • Serum Fr3: 8.7 pmol/L(normal 1.3-3.5 pmol/L).
Sick Euthyroid Syndrome • Serum FT4: 205 pmol/L (normal 70-160
pmol/L).
A 65-year-old, obese woman is hospitalized with
• TSH: 0.4 mIU/L (normal 0.5-5.1 mIU/L).
the complaints of fever, cough with purulent
• Radio-iodine uptake: After 2 h, 7% (normal
sputum for 3 days. Investigations reveal:
5-15%). After 24 h, 5.1% (normal 15-30%).
• Full blood count: Hb 10.5 g/dL, WBCs 18,540/
cmm, polymorphs 82%, lymphocytes 16%,
monocytes 2%, platelets l,80,OOO/cmm.
• Chest X-ray: Consolidation (left upper zone). Q: What is the likely diagnosis?
• ECG: Low-voltage tracing and sinus bradycardia. A: de Quervain thyroiditis (subacute thyroiditis).
• Serum Fr3: 2.2 pmol/L (normal 1.3-3.5 pmol/L).
• Serum Fr4: 104pmol/L(normaI70-160pmoljL). Q: Suggest one investigation.
• TSH: 0.2 mIU/L (normal 0.5-5.1 mIU/L). A: Fine-needle aspiration cytology (FNAC) of thyroid
gland.

Q: What is the likely diagnosis of thyroid disorder? Q: Mention three other causes of such thyroid func-
A: Sick euthyroid syndrome. tions (higb Ty T4 but low radio-iodine uptake).
A: As follows:
Q: What further investigation would you suggest?
A: Repeat fr3, FT4 and TSH after cure of pneumonia. • Postpartum thyroiditis.
• Factitious hyperthyroidism.
Q: What is the cause of ECG abnormality? • Iodine-induced hyperthyroidism.
A: Obesity.
_ SHORT CASES IN CLINICAL MEDICINE

N.B. de Quervain thyroiditis is a virus-induced tran- Any causes of high thyroxine-binding globulin
sient inflammation of thyroid gland, usually (TBG) will also cause high T, and T4, but IT, and
self-limiting. It is caused by Coxsackie B4, also fT4 will be normal. Alternately, any cause of low
may be associated with influenza, infectious mon- TBG will also cause low T3 and T4.
onucleosis, measles, mumps, common cold. It is
Causes of high TBG: Congenital or hereditary, preg-
more in females, 20-40 years. Presents with pain
nancy, drug (oestrogen, clofibrate, phenothiazine),
over thyroid, radiate to jaw, ear, and gets worse
oral contraceptive pills, acute intermittent por-
by coughing, swallowing or movement of neck.
phyria, acute viral hepatitis; also in hypothyroidism.
Systemic features are common. Thyroid gland is
enlarged and tender. FNAC shows presence of Causes of low TBG: Hereditary, malnutrition,
giant cells. ESR is typically high. Inflammation nephrotic syndrome, liver failure, drugs (sulpho-
of thyroid releases thyroid hormones, which are nylurea, salicylate, phenytoin, phenylbutazone,
high in the blood, responsible for the features of anabolic steroid, androgen, corticosteroid), active
thyrotoxicosis. This may persist for 4-6 weeks. acromegaly. Also in hyperthyroidism.
Iodine uptake is low. Hyperthyroidism followed
by transient hypothyroidism may occur. Com- Case 07: r
plete recovery occurs in 4-6 months. Treatment: Cushing Syndrome
Symptomatic (NSAID). Propranolol may be used.
Prednisolone 40 mg/day for 3-4 weeks may be A housewife of 40 years, presented with frequent
needed. Antithyroid drug should not be given. headache, vertigo and insomnia for 2 months. Blood
pressure (BP) is 160/100 mmHg. Investigations
reveal:
Case 06: I

Euthyroid State • Full blood count: Hb ll.5 gfdL, WBCs 8300/


cmrn, polymorphs 64%, lymphocytes 33%,
A young lady with 4 months' pregnancy presented monocytes 3%, ESR 48 mm in 1st hour, platelets
with the complaints of palpitation, excessive 2,70,OOO/cmm.
sweating and intolerance to heat. Thyroid gland is • Urine: Glycosuria (++), proteinuria (+).
diffusely en larged and nontender, soft. Investigation • Serum electrolytes: Sodium 145 mmol/L,
reveals: chloride 101 mmol/L, potassium 3.1 rnmol/I,
bicarbonate 30 mrnol/L.
• FuJI blood count: Hb 10.5 g/dl, WBCs 9,800/
• Creatinine: 1.3 mgfdL.
crnrn, polymorphs 66%, lymphocytes 30%,
• Chest X-ray: Heart slightly enlarged.
eosinophils 4%, platelets 2,OO,OOO/cmm, ESR 85
• Fasting blood sugar (FBS): 9.0 mrnol/L.
mm in 1st hour.
• Serum T4: 290 nmol/L (normal 70-160 nmol/L).
• Serum T,: 9 nmolfL (normal 1.32-3.1 nmol/L). Q: Suggest three differential diagnoses.
• Serum TSH: 4 mIU/L (normal 1.52-5 mIU/L). A: Cushing syndrome, primary aldosteronism, ectopic
adrenocorticotropic hormone (ACTH) syndrome.

Q: What is your inference from the above result? Q: What is the likely diagnosis in this case?
A: Euthyroid state. A: Cushing syndrome.

Q: What further investigations should be done? Q: What five physical signs will you see?
A: FTs and IT4• A: Obesity, striae, plethoric face, skin is thin with bruise,
proximal myopathy.
Q: Mention four causes of such thyroid function
Q: Mention four further investigations.
abnormality.
A: As follows:
A: Pregnancy, oral contraceptive pill, oestrogen therapy,
acute intermittent porphyria. • Serum morning and midnight cortisol.
• Dexamethasone suppression test.
N.B. Patient's symptoms are due to pregnancy. Thyroid • Ultrasonography of abdomen (to see suprarenal
gland may be normally enlarged in pregnancy, so gland).
her total T3 and T4 are high, but TSH is normal. • cr scan or MRI of suprarenal glands.
13 • DATA INTERPRETATION __

N.B. Hypertension with hypokalaemia and diabetes aldosteronism. n'eatment: If adenoma, surgery. If
mellitus is highly suggestive of Cushing syndrome. hyperplasia, spironolactone 100-400 mg daily.
In primary aldosteronism, diabetes mellitus is not For hypertension, amiloride and calcium channel
common and hypokalaemia is more severe. To blocker may be used.
diagnose Cushing syndrome, serum morning and
midnight cortisol, 24 h urinary-free cortisol, short
Case 09: Hypokalaemic f---------,
overnight dexamethasone suppression test should
Metabolic Alkalosis
be done, to see whether Cushing syndrome is pre-
sent or not. Then further test should be done to A 45-year-old female presented with blood pressure
find out causes (e.g. ACTH, X-ray chest usc of of 170/100 mmHg. Investigation reveals:
suprarenal gland, CT or MRJ of suprarenal glands
and pituitary gland). • Na': 156 rnmol/L.
• K+: 2.5 mmol/L.
• CI-: 102 mrnol/L.
I
Case 08: Primary I • HC03-: 30 mmol/L,
Hyperaldosteronism

A 25-year-old female is complaining of excessive Q: What are the abnormalities in the investigations?
thirst, weakness, polyuria, nocturia and insomnia for A: As follows:
6 months. Her BP is 165/115 mmHg. Investigations
• Hypernatraemia.
reveal:
• Hypokalaemia.
• full blood count: Hb 12.7 g/dl, WBCs 6800/ • Metabolic alkalosis.
cmm, polymorphs 60%, lymphocytes 39%,
Q: Name three possible causes of this investigation
monocytes 1%, ESR 20 mm in 1st hour.
finding.
• Urea: 38 mg/dL (normal 9-11 mg/dL).
A: As follows:
• Creatinine: 1.2 mg/dL (normal 0.68-1.36 mg/dL).
• Serum electrolytes: Sodium 148 rnmol/L, • Cushing syndrome.
chloride 107 mmol/L, potassium 2.8 mrnol/L, • Conn syndrome (primary hyperaldosteronism).
bicarbonate 32 mmol/L. • Renal artery stenosis.
• RBS: 9.3 mrnol/L, Q: Name four other investigation to confirm the diag-
• Chest X-ray: Cardiomegaly, left ventricular type.
nosis.
A: As follows:
Q: What is your diagnosis? • Plasma cortisol.
A: Primary hyperaldosteronism. • Urinary free cortisol.
Q: Suggest three investigations to confirm your • Plasma rennin and aldosterone assay (to see the
ratio ).
diagnosis.
A: As follows: • USC of kidney and adrenal gland.
• CT or MRI of suprarenal gland.
• Serum aldosterone and rennin assay.
• 24-h urine potassium.
Case 10: Addison
• cr scan or MRI of adrenal glands.
Disease
N.B. Hypertension with hypokalaernic alkalosis is
A 22-year-old man was admitted with the complaints
highly suggestive of primary hyperaldosteronism;
of weakness and weight loss for several weeks. His
it is also called Conn syndrome. It is an aldoster-
blood pressure is 80/50 mmHg. Investigations
one-secreting tumour responsible for <1% cause
reveal:
of hypertension. Excess aldosterone secretion
leads to sodium retention, hypokalaemia and • Na": 122 mrnol/L.
hypertension. It is due to adrenal adenoma in • K+: 6.3 mmol/L.
60% (Conn syndrome) and 30% bilateral adrenal • CI-: 98 mmol/L,
hyperplasia. Serum renin is low in primary aldos- • HC03-: 20 rnmol/L.
teronism, whereas renin is high in secondary
_ SHORT CASES IN CLINICAL MEDICINE

Q: What are the biochemical abnormalities? Q: What is the likely diagnosis?


A: As follows: A: Diabetic ketoacidosis (DKA).

• Hyponatraernia. Q: What other investigations should be done?


• Hyperkalaemia. A: Urine for ketone bodies.
• Acidosis.
Q: What are the complications?
Q: What is the likely diagnosis? A: As follows:
A: Addison disease.
• Acute respiratory distress syndrome.
Q: Mention one important clinical sign. • Thromboembolism.
A: Pigmentation of skin and mucous membrane. • DIC.
• Acute renal failure.
Q: What are the common causes?
• Cerebral oedema.
A: As follows:

• Autoimmune. N.B. Dry tongue, weight loss, high blood sugar with
• Tuberculosis. severe metabolic acidosis indicates DKA. It is
common in type 1 diabetes mellitus. Three main
Q: Name other four investigations to confirm the
problems in DKA are: (i) hyperglycaemia, (ii)
diagnosis. hyperketonaernia and (iii) metabolic acidosis.
A: As follows:
Marked dehydration is common. Five percent
• Plasma cortisol and ACTH level. cases develop coma in DKA. If bicarbonate is <12
• Synacthen test. mrnol/L, it indicates severe acidosis.
• USG of kidney and suprarenal gland.
• Plain X-ray abdomen (to see suprarenal Treatment: Intravenous (IV) normal saline rapidly,
calcification) . IV soluble insulin preferably with pump, potassium
therapy, control of infection, correction of acidosis by
N.B. In any patient with weakness, hypotension, pig- isotonic 1.26% soda bicarb, if arterial pH is <7 (com-
mentation associated with low sodium and high plete correction of acidosis is avoided because there is
potassium is highly suggestive of Addison disease. correction of extracellular acidosis, but acidosis in brain
. It is the primary adrenocortical insufficiency due is persistent, which aggravates cellular dysfunction in
to destruction of adrenal gland, resulting in defi- brain). Mortality is 5-10%, more in elderly.
ciency in glucocorticoid and mineralocorticoid.
Supine BP may be normal; but marked drop of Case 12: Hyperosmolar
systolic pressure on standing may occur. There Nonketotic Diabetic Coma with
will be low serum cortisol and high ACTH. Short Urinary Tract Infection (UTI)
Synacthen test is helpful for diagnosis. Treatment:
Hydrocortisone should be given, mineralocor- A 70-year-old lady was suffering from fever and
ticoid may be necessary. Steroid card should be frequency of micturition for 10 days. She became
maintained. unconscious for the last 2 h. BP 90/60 mmHg, pulse
lI0/min, marked dehydration. Investigations reveal:
Case 11: Diabetic • Full blood count: Hb ll.2gfdL, WBCs 17,000/cmm,
Ketoacidosis polymorphs 85%, lymphocytes 15%, platelets
2,10,000/cmm, ESR 12 mm in 1st hour.
A 19-year-old woman was suffering from high-grade
• Serum electrolytes: Sodium 162 mrnol/L,
fever, productive cough, shortness of breath, diffuse
chloride 110 mmol/L, potassium 4.6 mmol/L,
abdominal pain and vomiting. She is emaciated
bicarbonate 22 mmol/L,
and dehydrated. Investigations reveal:
• Serum creatinine: 1.4 mg/dL.
• Blood glucose: 28 mrnol/L. • Urine: Plenty of pus cells, albumin (++), glucose
• Serum electrolytes: Na 118 mmolfL, potassium (+++ ).
4.7 mmol/L, chloride 98 mmol/L and • Chest X-ray: Normal.
bicarbonate 8.1 mmolfL. • CT scan of brain: Diffuse age-related cerebral
• Serum creatinine: 1.1 mg/dL atrophy.
13 • DATA INTERPRETATION _

Q: What is the likely diagnosis? Q: What is the glycaemic status?


A: Hyperosrnolar nonketotic diabetic coma with uri- A: Impaired glucose tolerance (lCT).
nary tract infection (Uff}.
Q: What does it indicate?
Q: Suggest two investigations.
A: The patient is at high risk of developing type 2 dia-
A: As follows:
betes mellitus.
• RBS.
• Urine for ketone bodies. N.B. When fasting glucose is <7 mmol, but during oral
Q: What immediate therapeutic measures would you glucose tolerance test (OGTT), blood glucose is
start? 7.8-11.0 mmolfL, 2 h after glucose load, it is called
A: As follows: impaired glucose tolerance (lCT). This patient has
increased risk of developing frank diabetes mel-
• IV half-strength saline (0.45%).
litus (DM) type 2 with time and macrovascular
• Soluble insulin (preferably with insulin pump, (cardiovascular) diseases are more. Lifestyle mod-
2-6 units hourly). ification for type 2DM and annual check-up for
• When osmolality is normal, 0.9% normal saline
glucose are recommended for the patient. Cardio-
should be given.
vascular risk factors should be treated aggressively.
• Other treatment: Nasogastric (NC) tube feeding,
catheter if needed, antibiotic if infection,
correction of electrolytes, low-dose heparin (as Case 14: Renal r
thrombosis is common). Glycosuria

A young patient during routine check-up presents


N.B. Hyperosmolar nonketotic diabetic coma (HNDC)
with the following laboratory investigation:
may be the first presentation in diabetes mellitus.
Common in elderly, Noninsulin-dependent diabe- • CBe: Normal.
tes mellitus (NIDDM), characterized by very-high • Urine R/E:Albumin nil, glucose ++, few epithelial
blood glucose (>50 mmolfL) and high plasma cells, RBCs nil, pus cells nil.
osmolality. No ketosis because insulin deficiency • Blood glucose: 5.5 mrnol/L.
is .partial and low insulin is present which is suf-
ficient to prevent ketone body formation, but
insufficient to control hyperglycaernia. Precipi- Q: What is the diagnosis?
tating factors are large amount of sweet drinks, A: Renal glycosuria.
infection, steroid, thiazide, myocardial infarction,
etc. Plasma sodium is usually high. There may be Q: What is the prognosis?
high blood urea nitrogen (BUN); urea, creatinine A: It is a benign disease.
and serum osmolality may be very high. Mortality
rate may be up to 40%. N.~ Renal glycosuria is due to low renal threshold
for glucose. Blood glucose level is normal. It is
I
the most common cause of glycosuria and often
Case 13: Impaired found in pregnancy and young individuals. It
Glucose Tolerance is usually asymptomatic and rarely may lead to
polyuria and polydipsia.
This is the glucose tolerance test (CTT) report of a
46-year-old woman:
• FBS:5.1 mmol/L,
• After 0.5 h: 7.6 rnmol/L.
• After 1 h: 8.3 mmol/L,
• After 1.5 h: 10.6 mmol/L.
• After 2.0 h: 10.1 mrnol/L.
CI-IAPTER 14

x-RAy, CT AND MRI


«Treat the patient, not the x-ray"

- James M. Hunter

Pleural Effusion
Q: Write down other investigations to confirm the
diagnosis.
A: As follows:
• Complete blood count (CBC) with erythrocyte
sedimentation rate (ESR).
• Tuberculin test.
• Pleural fluid analysis [physical character, cytology,
biochemistry, Gram (Gm) stain, culture, acid
fast bacilli (AFB), malignant cell, adenosine
deaminase (ADA)].
• Pleural biopsy.
• Bronchoscopy and biopsy (if needed in bronchial
carcinoma) .

Fig. A: Left-sided pleural effusion

Q: Write down the radiological findings of the X-ray


shown in Figure A.
A: X-ray chest PA view showing:
• Dense homogenous opacity in left lower zone
with a curvilinear upper border.
• There is obliteration of left costophrenic angle.
• Trachea and heart (mediastinum) are shifted to
the right.

Q: What is your radiological diagnosis?


A: Left-sided pleural effusion.

Q: Write down one percussion and two auscultatory


findings of chest.
A: As follows:
Fig. B: Bilateral pleural effusion
• Percussion note stony dull.
• On auscultation: Diminished or absent breath Q: Write down the radiological findings of the X-ray
sound and diminished or absent vocal resonance.
shown in Figure B.
Q: Write down four common causes. A: X-ray chest PA view showing dense homogenous
A: As follows: opacity with curvilinear upper border in both lower
• Pulmonary tuberculosis (TB). zones obliterating both costophrenic angles.
• Parapneumonic.
• Bronchial carcinoma. Q: What is your radiological diagnosis?
• Pulmonary infarction. A: Bilateral pleural effusion.
14 • X-RAY, CT AND MRI __

Q: What are the causes?


A: As follows:
• Cirrhosis of liver.
• Nephrotic syndrome.
• Congestive cardiac failure.
• Bilateral extensive tuberculosis.
• Collageri disease: Systemic lupus erythematosus
(SLE), rheumatoid arthritis.

Q: Write down the radiological findings of the X-ray


shown in Figure C.
A: X-ray chest right decubitus view showing dense
homogenous opacity in peripheral pan of the chest
with clear upper border.

Q: What is your radiological diagnosis? Fig. C: Right-sided pleural effusion (decubitus view)
A: Right-sided pleural effusion.

Mass Lesion
• Sputum for malignant cells (exfoliative
cytology).
• CT- or ultrasonography-guided fine-needle
aspiration cytology (FNAC).
• Bronchoscopy and biopsy.
• If palpable lymph node, then FNAC or biopsy.

Fig. A: Mass lesion (bronchial carcinoma) in right lung

Q: Write down the radiological finding of the X-ray


shown in Figure A.
A: X-ray chest PA view showing opacity with irregular
margin, occupying the right upper and part of mid
zone.

Q: What is your radiological diagnosis? Fig. B: Bronchial carcinoma with left phrenic nerve palsy
A: Bronchial carcinoma.
Q: Write down the radiological finding of the X-ray
Q: Mention two differential diagnoses. shown in Figure B.
A: As follows: A: X-ray chest PA view showing:
• Tuberculosis. • Opacity with irregular margin in the left upper
• Consolidation. and part of mid zones.
• Left dome of the diaphragm is raised.
Q: Mention three investigations to confirm your
diagnosis? Q: What is your radiological diagnosis?
A: As follows: A: Bronchial carcinoma with left phrenic nerve palsy.
_ SHORT CASES IN CLINICAL MEDICINE

Solitary Pulmonary Nodule


• Bronchial carcinoma.
• Secondary deposit.
• Bronchial adenoma.
• Lung abscess before burst.
• Hydatid cyst.

N.B. Others causes are encysted pleural effusion,


hamartoma, rheumatoid nodule, Wegener
granulomatosis, aspergilloma, neurofibroma,
fungal infection or granuloma (histoplasmo-
sis).

Q: Mention one investigation to confirm.


A: CT-guided FNAC.

Q: What investigations should be done?


A: As follows:
• CXR lateral view.
Left-sided solitary pulmonary nodule • CBC and ESR.
• Sputum for Gm staining, CIS, AFB,malignant cell.
Q: Write down the radiological finding of this Xvray. • Mantoux test (MT).
A: Chest X-ray (CXR) PA view showing a homogene- • CT-guided FNAC.
ous, rounded, nodular shadow with clear margin in • Bronchoscopy and bronchial brushing for AFB,
left midzone. malignant cell.

Q: How to differentiate neurofibroma from dermoid


Q: Mention five causes.
cyst radiologically?
A; As follows:
A: By lateral film:
• Consolidation.
• If neurofibroma: Itis in the posterior mediastinum
• Tuberculoma. • If dermoid cyst: It is in the anterior mediastinum.

Multiple Secondaries in lung


Q: Write down the radiological finding of the X-ray
shown in Figure A.
A: X-ray chest PA view showing multiple, nodular
shadows of variable size and shape, some having
'cannon-ball' appearance in both lung fields.

Q: What is your radiological diagnosis?


A: Multiple secondary deposits in the lung.

Q: Mention five sources of such lesion.


A: As follows:
• Carcinoma stomach.
• Carcinoma of prostate.
• Carcinoma of breast.
• Teratoma.
• Renal cell carcinoma.
Fig. A: Multiple secondaries in both lungs • Carcinoma of thyroid gland.
Q: Write down the radiological finding of the X-ray
shown in Figure B.
A: X-ray of chest PA view showing:
--
• Multiple, nodular shadows of variable size and
shape, some having 'cannon-ball' appearance in
both lung fields.
• Right dome of diaphragm is elevated.

Q: What is your radiological diagnosis?


A: Multiple secondary deposits in the lung with right
phrenic nerve palsy.

Fig. B: Multiple secondaries in both lungs

Consolidation

Right-sided consolidation

Q: Write down the radiological finding of this X-ray. • Percussion note is dull (or woody dull).
A: X-ray chest PA view showing dense, homogeneous • Auscultation: Bronchial breath sound, increased
opacity involving the right upper and part of mid vocal resonance.
zone with air bronchogram within it.
Q: Write down other investigations to confirm the
Q: What are the differential diagnoses from this X-ray? diagnosis.
A: As follows: A: As follows:
• Consolidation. • CSC with ESR.
• Sputum for Gram staining, culture and sensitivity.
• Bronchial carcinoma.
• Sputum for AFBand malignant cell.
• Tuberculosis.
Q: Mention two complications?
Q: What is the likely diagnosis? A: As follows:
A: Consolidation. • Lung abscess.
• Empyema.
Q: Write down the percussion and auscultation
findings of this patient. Q: What is the commonest organism?
A: As follows: A: Pneumococcus.
_ SHORT CASES IN CLINICAL MEDICINE

Lung Abscess
• CBC with ESR.
• cr scan of chest.
• Sputum for Gram staining, CS and AFB.

Q: Write down three important complications.


A: As follows:
• Empyema thoracis.
• Bronchiectasis.
• Cerebral abscess.

Q: What is the commonest cause?


A: Aspiration of infected material due t? any cause.

Fig- A: Right-sided lung abscess

Q: Write down the radiological finding of the X-ray


shown in Figure A.
A: X-ray chest PA view showing a cavity with air-fluid
level in the right mid and lower zone.

Q: What is your diagnosis?


A: Right-sided lung abscess.

Q: Write down the percussion and auscultation find-


ings of this patient.
A: As follows: Fig_ B: Left-sided lung abscess
• Percussion note is dull (may be dull in the lower
part and hyperresonant in the upper part). Q: Write down the radiological finding of the X-ray
• Auscultation: Bronchial breath sound, increased shown in Figure B.
vocal resonance. A: X-ray chest PA view showing a cavity with air-fluid
level in the left upper and part of midzone.
Q: What other investigations will you do to confirm
this diagnosis? Q: What is your diagnosis?
A: As follows: A: Left-sided lung abscess.

Pulmonary Tuberculosis
Q: Write down the radiological finding of the X-ray
shown in Figure A.
A: X-ray dlest PA view showing patchy opacities with
some translucent shadows within it involving right
upper and part of mid-zone.
Q: What is the radiological diagnosis?
A: Right-sided pulmonary tuberculosis.
Q: Write down other investigations to confirm the
diagnosis.
A: As follows:
• CBC with ESR.
• Sputum for AFB staining (three consecutive
samples).
Fig_ A: Right-Sided pulmonary tuberculosis
• Tuberculin test.
• PCR forTB.
14 • X-RAY, (T AND MRI __

Q: How to treat? Q: What is the radiological diagnosis?


A: Standard anti-TS therapy for 6 months in the A: Bilateral extensive tuberculosis.
following regimen:
• Initial phase (2 months): Isoniazid (INH) +
Rifampicin + Pyrazinamide + Ethambutol.
• Continuation phase (4 months): JNH +
Rifampicin.
• Tab. Pyridoxine (20 mg) for 6 months.
Q: Mention one complication of each drug.
A: As follows:
• Rifampicin: Hepatitis.
• INH: Peripheral neuropathy.
• Ethambutol: Optic neuritis.
• Pyrazinamide: Hepatitis (also hyperuricaemia
and gout).

Fig. C: Tubercular cavity in left side

Q: Write down the radiological finding of the X-ray


shown in Figure C.
A: X-ray chest PA view showing patchy opacities with
cavity in the left upper zone.
Fig. B: Bilateral extensive tuberculosis
Q: What is the radiological diagnosis?
Q: Write down the radiological finding of the X-ray A: Pulmonary tuberculosis.
shown in Figure S.
A: X-ray chest PA view showing patchy opacities Q: What is the significance of this finding?
involving both upper and middle zone in both right A: Presence of cavity indicates active and open case of
and left lung. tuberculosis.

Miliary Tuberculosis
Q: Write down the radiological finding of
this X-ray.
A: X-ray chest PA view showing multiple miliary mot-
tling involving all the zones of both lung fields.

Q: What is the diagnosis?


A: Miliary tuberculosis.

Q: Mention four common differential diagnoses.


A: As follows:
• Sarcoidosis.
• Pulmonary eosinophilia.
• Histoplasmosis.
• Pneumoconiosis.
Q: Mention four investigations.
A: As follows:
Miliary tuberculosis • CSC with ESRand circulating eosinophil count,
__ SHORT CASES IN CLINICAL MEDICINE

• Sputum for APB. Q: What are the common presentations of this patient?
• Tuberculin test. A: Low-grade continued fever, mostly evening lise, night
• Bronchoscopy and bronchoalveolar lavage. sweat, weight loss, cough with haemoptysis. etc.

Calcification of the Lung Parenchyma

Calcification of lung parenchyma

Q: Write down the radiological findings in this X-ray. • Tuberculosis (usually healed, commonly in
A: X-ray chest PA view showing multiple calcified upper zone).
shadows of variable size and shape, involving all • Adult chicken pox pneumonia (widely
the zones of both lung fields. distributed, usually small. 1-3 mm).
• Histoplasmosis (surrounded by small halo) and
Q: What is your radiological diagnosis? other fungal infection.
A: Multiple calcifications in lung parenchyma. • Hamartoma (popcorn calcification).
• Hypercalcaemia due to any cause.
Q: Mention five causes. • Alveolar microlithiasis.
A: As follows: • Silicosis.

Emphpema .
Q: Write down the radiological finding of this X-ray.
A: CXR PA view showing:
• Lung fields are hypertranslucent.
• Low and flat diaphragm.
• Heart is elongated and tubular.
• Ribs are widely spaced.
Q: What is your radiological diagnosis?
A: Pulmonary emphysema.
Q: What is the pathognomonic sign in CXR of this
disease?
A: Bullae.
Q: Mention two investigations.
A: As follows:
• Lung function tests (obstructive type: FtVI and
FVC are reduced and ratio of FEVI:FVC is also
reduced).
Emphysema • High resolution computed tomography (HRCT)
of chest.
14 • X-RAY, (T AND MRI _

Bullae
and blood vessel, bounded by a thin-line (hairline)
shadow.

Q: What is your radiological diagnosis?


A: Right-sided giant bullae.

Q: What are the other possibilities?


A: As follows:
• Apical pneumothorax (collapsed lung margin
should be present in such case).
• Big cavity.

Q: Mention three complications.


Bullae A: As follows:
• Rupture causing pneumothorax.
Q: Write down the radiological findings in this X-ray.
A: X-ray chest PA view showing a large translucent
• Secondary infection.
area in the right upper zone with no lung marking • Aspergilloma.

Surgical Emphysema

Surgical emphysema with bilateral chest tube

Q: Write down the radiological findings in this X-ray. Q: Mention three causes.
A: X-ray chest PA view showing: A: As follows:
• Increased translucency with collapsed lung • Traumatic (road traffic accident, penetrating
margin on the right side. injury).
• There are multiple translucent shadows in the soft
• During aspiration of pneumothorax or
tissue outside the thoracic cavity in both sides.
introduction of IT tube.
• Intrathoracic tube (IT tube) is seen on both sides.
• Acute severe asthma (due to rupture of alveol i).
Q: What is your radiological diagnosis?
• Intermittent positive pressure ventilation.
A: Right-sided pneumothorax with subcutaneous
emphysema.
_ SHORT CASES IN CLINICAL MEDICINE

Pneumothorax

Fig. A: Right-sided pneumothorax

Q: Write down the radiological findings of the X-ray • If large: Intercostal chest tube drainage (or water
shown in Figure A. seal drainage).
A: X-ray chest PA view showing hypertranslucent area • Treatment of underlying cause.
without bronchovascular markings with collapsed
lung margin in right side.

Q: What is the radiological diagnosis?


A: Right-sided pneumothorax.

Q: What is the type?


A: Closed.

Q: Write down the findings in percussion and


auscultation.
A: As follows:
• Percussion note is hyperresonant on left side.
• On auscultation: Diminished (or absent) breath
sound and diminished (or absent) vocal resonance
on left side.

Q: Write down three common causes.


A: As follows:
• Rupture of subpleural emphysematous bullae.
Fig. B: Left-sided pneumothorax
• Rupture of subpleural tuberculous focus.
Q: Write down the radiological findings of the X-ray
• Rupture of subpleural bleb in young patient. shown in Figure B.
Q: Write down the principle of management of this A: X-ray chest PA view showing hypertranslucent area
without bronchovascular markings with collapsed
patient.
A: As follows: lung margin in left side.
• If small and asymptomatic: Complete fest and Q: What is the radiological diagnosis?
follow-up. A: Left-sided pneumothorax.
14 • X-RAY, CT AND MRI _

Hydropneumothorax
• Increased translucency with collapse lung margin
on the right side.
• There is a horizontal fluid level with obliteration
of right costophrenic and cardiophrenic angles.

Q: What is your radiological diagnosis?


A: Right-sided hydropneumothorax.

Q: Mention one important bedside physical finding.


A: Succussion splash.

Q: What are the causes?


A: As follows:
• Iatrogenic (during aspiration of pleural fluid)-
common cause.
• Bronchopleural fistula.
• Trauma (penetrating injury, thoracic surgery).
• Rupture of lung abscess.
• Oesophageal rupture.
• Erosion by bronchial carcinoma.
• Pulmonary tuberculosis.

Right-sided hydropneumothorax Q: How to treat?


A: As follows:
Q: Write down the radiological findings of this X-ray. • Insertion of intrathoracic tube.
A: X-ray chest PA view showing: • Treatment of specific cause.

Collapse of Lung
• Trachea and heart shifted to the left.
• Hypertranslucency of the light lung field.
Q: What is your radiological diagnosis?
A: Collapse of the left lung.
Q: Mention two causes.
A: As follows:
• Bronchial carcinoma with complete bronchial
obstruction.
• Foreign body in the left bronchus.
Q: Mention three physical findings.
A: As follows:
• Palpation: Trachea and apex beat shifted to the
left.
• Percussion: Dullness in the whole left hemithorax.
Collapse of left lung • Auscultation: Breath sound and vocal resonance
are diminished in the left side.
Q: What is your radiological finding? Q: Mention two further investigations.
A: X-ray chest PA view showing: A: As follows:
• Homogenous opacity involving the whole left • CT scan of the chest.
lung field. • Bronchoscopy.
__ SHORT CASES IN CliNICAL MEDICINE

Rib Resection
• There is rib resection on the left side.
• Homogeneous opacity with air-fluid level
involving the left lower and part of middle
zone. The rest of the left lung field shows
hypertransl ucency.
• Left costophrenic and cardiophrenic angles are
obscure.
• Trachea and heart are shifted to the left.
• The right lung field shows compensatory
hypertransl u cency.

Q: What is your radiological diagnosis?


A: Pneumonectomy of left lung.

Q: Mention five indications.


A: As follows:
• Bronchial carcinoma (if localized).
• Bronchial adenoma.
• Extensive bronchiectasis.
Rib resection
• Extensive fibrosis with repeated chest infection.
Q: Write down the radiological findings in this X-ray. • In some cases of TB [rnultidrug-resistant TB
A: X-ray chest PA view showing: (MDR-TB) or no response to drug].

Bilateral Hilar Lymphadenopathy


• Sarcoidosis.
• Lymphoma.
• Tuberculosis.

Q: Mention one drug that can cause this type of


finding.
A: Anticonvulsant like phenytoin or diphenylhydan-
toin (called pseudolymphoma).

Q: Mention three physical findings.


A: As follows:
• Lymphadenopathy in other parts of the body.
• Hepatomegaly.
Bilateral hilar lymphadenopathy • Splenomegaly.

Q: Write down the radiological finding of this X-ray. Q: Write two investigations.
A: X-ray chest PA view showing bilateral hilar A: As follows:
lymphadenopathy. • CBC, ESR.

Q: Write three causes. • Tuberculin test.


A: As follows: • FNAC or biopsy from lymphnode.
14 • X-RAY, (T AND MRI __

Sarcoidosis

Bilateral hilar lymphadenopathy with parenchymal lung involvement

Q: Write down the radiological finding of this X-ray. • Erythema nodosum.


A: X-ray chest PA view showing: • Lymphadenopathy in other parts of the body.
• Bilateral hilar lymphadenopathy. • Hepatosplenomegaly.
• Reticulonodular shadow in both lung fields. • Bilateral parotid enlargement.

Q: What is your radiological diagnosis? Q: Write three investigations to confirm.


A: Sarcoidosis. A: As follows:
• Tuberculin test (usually 0).
Q: Mention four physical findings. • FNAC or biopsy from the lymph node.
A: As follows: • Bronchoscopy and biopsy.

Azygos Lobe

Azygos lobe

Q: Write down the radiological finding of this X-ray. Q: What is the significance?
A: X-ray chest PA view showing an inverted-comma- A: It is physiological and may be confused with path-
shaped opacity in the right apical region. ological conditions such as consolidation, TB and
mass lesion.
Q: What is your radiological diagnosis?
A: Azygos lobe.
_ SHORT CASES IN CLINICAL MEDICINE

Cavity Superimposed on Cardiac Shadow


Q: Write down the radiological finding of this X-ray.
A: X-ray chest PA view showing air-fluid level
superimposed on cardiac shadow.
Q: Mention three differential diagnoses.
A: As follows:
• Hiatus hernia.
• Achalasia cardia.
• Lung abscess.
Q: Mention three investigations.
A: As follows:
• Chest X-ray left lateral view.
• Barium swallow of oesophagus in Trendelenburg
position.
Cavity superimposed on cardiac shadow • Endoscopy.

Bronchiectasis

Fig. A: Bilateral bronchiectasis

Q: Write down the radiological findings of the X-ray


shown in Figure A.
A: X-ray chest PA view showing multiple ring shadows
involving the mid and lower zones of both lung
fields, more on the right side.

Q: What is your radiological diagnosis?


A: Bilateral bronchiectasis

Q: What is the likely cause in this disease?


A: Cystic fibrosis.

Q: Write down one investigation to confirm the diag-


nosis.
A: HRCf of chest. Fig. B: Right-sided bronchiectasis
14 • X-RAY, CT AND MRI _

Q: Write down the radiological findings of the X-ray Q: What is the definitive treatment of this case?
shown in Figure B. A: Lobectomy.
A: X-ray chest PA view showing multiple ring shadows
involving the lower zone of right lung field. Q: Mention three other treatment modalities.
Q: What is your radiological diagnosis? A: As follows:
A: Right-sided bronchiectasis. • Postural drainage.
Q: What is the commonest cause? • Chest physiotherapy.
A: Childhood pneumonia or pulmonary infection • Antibiotic, if infection is present.
after whooping cough and measles.

Homogeneous Opacity of One Hemithorax


Q: Write down the radiological findings of this X-ray.
A: X-ray chest PA view showing homogenous opacity
involving the whole right hemithorax.

Q: Mention three differential diagnoses.


A: As follows:
• Massive consolidation.
• Massive pleural effusion.
• Complete collapse of right lung.

Q: Mention three investigations.


A: As follows:
• cr scan of chest.
• Bronchoscopy and biopsy.
Homogenous opacity of right lung • CT-guided FNAC.

Mediastinal Widening
Q: Mention four differential diagnoses.
A: As follows:
• Lymphoma.
• Sarcoidosis.
• Bronchial carcinoma.
• Retrosternal goitre.

Q: Mention one important physical finding of this


patient.
A: Puffiness of the face with engorged, nonpulsatile
neck veins due to superior vena caval obstruction.

Q: Mention four other investigations.


A: As follows:
Mediastinal widening • CBC.
Q: Write down the radiological findings of this X-ray.
• X-ray chest lateral view.
A: X-ray chest PA view showing widening of the supe- • cr scan of chest.
rior mediastinum. • Cf-guided FNAC.
_ SHORT CASES IN CLINICAL MEDICINE

Gas Under the Diaphragm


• Nothing by mouth.
• Nasogastric suction.
• Intravenous (IV) fluid.
• Broad spectrum antibiotic with metronidazole.
• Surgical repair.

Fig. A: Gas under the right dome of the diaphragm

Q: Write down the radiological findings of the X-ray


shown in Figure A.
A: X-ray chest PA view showing gas under the right
dome of diaphragm.
Fig. B: Gasunder both domes of the diaphragm
Q: What is your radiological diagnosis?
Q: Write down the radiological findings of the X-ray
A: Perforation of gas containing hollow viscus.
shown in Figure B.
Q: Write down two common causes. A: X-ray chest PA view showing gas under both domes
A: As follows: of the diaphragm.
• Perforation of chronic duodenal ulcer.
• Perforation ofileum (due to typhoid, tuberculosis, Q: Write down five causes.
Crohn disease). A: As follows:
• Perforation of hollow viscus containing gas.
Q: Mention one most important clinical finding.
• Laparotomy or laparoscopy.
A: Obliteration of liver dullness on percussion.
• Penetrating injury of abdomen.
Q: Write down the modalities of treatment. • Burst appendicitis.
A: As follows: • Tubal insufflation.

Situs Inversus
• Cardiac apex directed towards right side.
• Fundic gas shadow is on the right side.
• Left dome of the diaphragm is raised.

Q: What is your radiological diagnosis?


A: Situs inversus.

Q: What is the prognosis?


A: Normal lifespan.

Q: What is the clinical importance of this disorder?


A: As follows:
• Patient's appendix is on the left side. So, the
Situs Inversus appendicitis may be missed as it is on the left
side.
Q: Write down the radiological finding of this X-ray. • Sometimes, liver biopsy is done mistakenly on
A: X-ray chest PA view showing: the right side; it will be missed.
14 • X-RAY, CT AND MRI _

Dextrocardia
Q: Write down the radiological finding of this X-ray.
A: X-ray chest PAview showing:
• Cardiac apex directed towards right side.
• Fundic gas shadow is on the left side.

Q: What is your radiological diagnosis?


A: Dextrocardia.

Q: What other investigation should be done? Why?


A: X-ray of paranasal sinuses (PNS) to see sinusitis
or frontal sinus agenesis, which may be present in
Kartagener syndrome.

N.B. Kartagener syndrome is characterized by dex-


trocardia, bronchiectasis and frontal sinus
Dextrocardia agenesis or sinusitis.

Mitral Stenosis
• Double contour of the right heart border.
• Upper lobe diversion.
• Kerley B line.

Q: Mention one investigation to confirm your


diagnosis.
A: Colour Doppler echocardiography.

Q: Mention two complications.


A: As follows:
• Congestive cardiac failure (CCF).
• Atrial fibrillation.

Fig. A: Mitral stenosis

Q: Write down the radiological finding of the X-ray


shown in Figure A.
A: X-ray chest PA view showing fullness of the pulmo-
nary conus with straightening of the left border of
the heart.

Q: What is your radiological diagnosis?


A: Mitral stenosis.

Q: Mention four important radiological findings that


may be present in this disease.
A: As follows:
• Widening of the carina with horizontal left main Fig. B: Mitral stenosis with mitral regurgitation
bronchus.
_ SHORT CASES IN CLINICAL MEDICINE

Q: Write down the radiological finding of the X-ray Q: What is the predominant lesion?
shown in Figure B. A: Mitral regurgitation (as heart is enlarged).
A: X-ray chest PA view showing:
• Straightening of the left cardiac border. Q: Mention one investigation to confirm your
• Heart is enlarged in transverse diameter. diagnosis.
A: Colour Doppler echocardiography.
Q: What is your radiological diagnosis?
A: Mitral stenosis with mitral regurgitation.

Ventricular Aneurysm
Q: Write down the radiological finding of this X-ray.
A: X-ray chest PA view showing:
• Heart is enlarged in transverse diameter.
• There is bulging of the left lower border with
calcification of the outer border.

Q: What is your radiological diagnosis?


A: Ventricular aneurysm.

Q: Mention one investigation to confirm your


diagnosis.
A: Echocardiography.

Q: Mention three complications.


A: As follows:
• Acute left ventricular failure (LVF).
• Arrhythmia.
Ventricular aneurysm • Systemic embolism.

Atrial Septal Defect


Q: What is your radiological diagnosis?
A: Atrial septal defect.

Q: Mention one investigation to confirm your


diagnosis.
A: Colour Doppler echocardiography.

Q: Mention three auscultatory findings.


A: As follows:
• Wide, fixed splitting of second heart sound.
• Mid-diastolic murmur.
• Ejection systolic murmur.

Atrial septal defect Q: Mention the BeG finding.


A: As follows:
Q: Write down the radiological finding of this X-ray.
• In ostium primum defect: Right bundle-branch
A: X-ray chest PA view showing:
block (RBSB) with left-axis deviation.
• Heart is enlarged in transverse diameter.
• There is fullness of the pulmonary conus with • In ostium secundum defect: RBBBwith right-axis
bulging of the left border. deviation.
14 • X-RAY, CT AND MRI __

Tetralogy of Fallot

• Right ventricular hypertrophy.


• Ventricular septal defect (VSD) (large subaortic).
Q: What are the physical findings?
A: As follows:
• Clubbing and central cyanosis.
• Palpation: Left parasternal lift, epigastric
pulsation and systolic thrill in pulmonary area.
• Auscultation: First heart sound is normal, P2 is
soft or absent in pulmonary area, A2 is normal.
Harsh ejection systolic murmur in pulmonary
area radiating to suprasternal notch.

Q: What are the complications?


A: As follows:
• Infective endocarditis.
Tetralogy of Fallot • Paradoxical emboli.
• Cerebral abscess (in 10% cases).
Q: Write down the radiological finding of this X-ray. • Polycythaemia secondary to hypoxia (it may
A: CXR PA view showing: cause cerebral infarction).
• Boot-shaped heart (apex is lifted up and there
Q: Mention two investigations.
is concavity or bay in the region of pulmonary
A: As follows:
artery).
• Colour Doppler echocardiogram.
• Oligaemic lung fields.
• Cardiac catheter.
Q: What is your radiological diagnosis?
Q: How to treat?
A: Tetralogy of Fallot (TO F).
A: As follows:
Q: Mention the anatomical defects in this disease. • Surgical correction should be done.
A: As follows: • If total correction is not possible, then temporarily
• Pulmonary stenosis (valvular or infundibular). Blalock-Taussig shunt is performed. Corrective
• Overriding and dextroposition of a011a (aortic surgery is done later on.
origin two-third from the left ventricle and one- • Prophylactic antibiotic to prevent infective
third from the right ventricle). endocarditis.

Ca rd iomega Iy

Q: Write down the radiological finding of this X-ray.


A: X-ray chest PA view showing increased transverse
diameter of cardiac shadow.

Q: What is the radiological diagnosis?


A: Cardiomegaly.

Q: Mention five important common causes of produc-


ing this X-ray finding.
A: As follows:
• Pericardial effusion.
• Multiple valvular heart disease.
• Biventricular failure.
• Myocarditis.
Cardiomegaly • Cardiomyopathy.
__ SHORT CASES IN CLINICAL MEDICINE

• Shunt anomaly (VSD, ASD, PDA). Q: What other investigations should be done?
• Hyperdynamic circulation due to any cause A: As follows:
(anaemia, beriberi, thyrotoxicosis, arteriovenous • ECG.
fistula, etc.). • Echocardiogram.

Pericardial Effusion
Q: Name five common causes of pericardial effusion.
A: As follows:
• Tuberculosis.
• Lymphoma.
• Following acute pericarditis.
• Collagen diseases.
• Myxoedema.

Q: Write down four important signs in favour of your


diagnosis.
A: As follows:
• Raised jugular venous pressure OVP).
• Narrow pulse pressure, may be pulsus paradoxus.
Pericardial effusion • Heart sounds are muffled or absent.
• Enlarged, tender liver.
Q: Write down the radiological findings of this X-ray.
A: X-ray chest PA view showing: Q: Name one investigation to confirm the diagnosis.
• Heart is enlarged in transverse diameter, globular, A: Echocardiogram.
pear shaped with clear margin. Q: Mention one serious complication of this. How to
• Lung fields are oligaemic. manage?
Q: What is the radiological diagnosis? A: Cardiac tamponade. Managed by immediate
A: Pericardial effusion. paracentesis.

Pericardial Calcification

Fig. A: Pericardial calcification


14 • X-RAY, CT AND MRI _

Q: Write down the radiological finding of the X-ray


shown in Figure A.
A: X-ray chest PA view showing:
• Calcified shadow at the left and lower border of
heart,
• Heart is enlarged in transverse diameter.
Q: What is your radiological diagnosis?
A: Pericardial calcification.
Q: What is the clinical diagnosis?
A: Chronic constrictive pericarditis.
Q: Mention three causes.
A: As follows:
• Tuberculosis.
• Hypercalcaemia due to any cause.
• Haemopericardium.
Q: Write down the radiological finding of the X-ray
shown in Figure B.
A: X-raychest left lateral view showing flecks of calcifica-
tion in the pericardium at inferior and anterior border.
Q: What is your radiological diagnosis?
A: Pericardial calcification. Fig. B: Pericardial calcification (lateral view)

Pulmonary Oedema

Pulmonary oedema

Q: Write down the radiological finding of this X-ray. Q: How to manage?


A: CXR PA view showing: A: As follows:
• FluffyorwooJly opacities, spreading from both hilar- • Propped-up position.
region, giving a butterfly- or bat's-wing appearance.
• Oxygen inhalation 4-6 L/min (60-100%).
• Heart is enlarged in transverse diameter.
• Injection frusernide IV.
Q: What is your radiological diagnosis?
A: Pulmonary oedema. • Injection morphine 10-20 mg IV, with cyclizine
or metoclopramide if vomiting.
Q: Mention three causes.
A: As follows: • Angiotensin-converting-enzyme (ACE)
• Acute LVFdue to any cause. inhibitor (captopril. lisinopril), vasodilator
• Mitral stenosis. [isosorbide) and digoxin (in some cases).
• Excessive or rapid transfusion of fluid or blood. • Treatment of primary cause.
Pacemaker
• Infection.
• Displacement.
• Malfunction or pacemaker failure.
• Pacemaker-mediated tachycardia
(by dual-chamber pacing).
• Pacemaker syndrome (occurs in single-chamber
pacing).

Fig. A: Dual-chamber pacemaker

Q: Write down the radiological finding of the X-ray


shown in Figure A.
A: X-ray chest PA view showing:
• Metallic suture wire of sternotomy.
• Dual-chamber pacemaker on the left side.
Q: What is your radiological diagnosis? Fig. B: Single-chamber pacemaker
A: Dual-chamber pacemaker. Q: Write down the radiological finding of the X-ray
Q: Mention two common indications. shown in Figure B.
A: As follows: A: X-ray chest PA view showing:
• Complete heart block (with syncope or Stokes-
• Metallic suture wire of sternotomy.
Adams attack).
• Single-chamber pacemaker on the left side.
• Sick sinus syndrome.
Q: Mention five complications. Q: What is your radiological diagnosis?
A: As follows: A: Single-chamber pacemaker.
"

Metallic Prosthetic Valve


Q: Write down the radiological finding of the X-ray
shown in Figure A.
A: X-ray chest PA view showing:
• Metallic wire of suture in the sternum (indicates
sternotomy).
• A metallic prosthetic heart valve.

Q: What is your radiological diagnosis?


A: Metallic prosthetic mitral valve.

Q: Mention two complications.


A: As follows:
• Infective endocarditis.
• Valve failure.

Fig. A: Metallic prosthetic mitral valve


14 • X-RAY, CT AND MRI __

Q: Write down the radiological finding of the X-ray


shown in Figure B.
A: X-ray chest PA view showing:
• Metallic; wire of suture in the sternum (indicates
sternotomy) .
• Two metallic prosthetic heart valves.
Q: What is your radiological diagnosis?
A: Metallic prosthetic mitral and aortic valves.

Fig. B: Metallic prosthetic mitral and aortic valve

Ankylosing Spondylitis
Q: Mention one bedside test.
A: Schober test.

Q: Write one investigation that is helpful for your


diagnosis.
A: HLA-B27.

Fig. A: Ankylosing spondylitis

Q: What are your findings in the X-rayshown in FigureA?


A: X-ray of lumbosacral spine anteroposterior (AP)
view showing:
• Calcification of anterior longitudinal and
interspinous ligaments.
• There is syndesmophyte formation with bridging,
giving rise to bamboo-spine appearance.

Q: What is your radiological diagnosis?


A: Ankylosing spondylitis. Fig. B: Ankylosing spondylitis
_ SHORT CASES IN CLINICAL MEDICINE

Q: What are your findings in the X-rayshown in Figure B? • Osteopaenia with marginal sclerosis of
A: X-ray of lumbosacral spine lateral view showing: vertebral body.
• Syndesmophyte formation along the comers
of vertebral body with bridging, giving rise to Q: What is your radiological diagnosis?
bamboo-spine appearance. A: Ankylosing spondylitis.

Pott Disease
Q: Write down the radiological finding of this X-ray.
A: X-ray of dorsolumbar spine AP view showing:
• Reduction of joint space between TG, 17 and also
T8, with partial destruction of vertebral body.
• Paravertebral shadow on both sides.
• There is marginal sclerosis.

Q: What is your radiological diagnosis?


A: Pott disease (tuberculosis of spine or tuberculous
spondylitis ).

Q: Mention three investigations.


A: As follows:
• CBC, ESR.
• X-ray chest PA view (to see primary focus).
• Tuberculin test.
• MRI of thoracic spine.

Q: How to confirm?
Pott disease A: CT or ultrasonogram-guided FNAC.

Multiple Myeloma

Q: What is the most likely diagnosis?


A: Multiple myeloma.

Q: Write two important differential diagnoses.


A: As follows:
• Secondary deposit.
• Hyperparathyroidism.

Q: Name other investigations to confirm the


diagnosis.
A: As follows:
• CBC with ESRwith peripheral blood film
(PBF) (shows high ESRwith marked rouleaux
formation).
Multiple myeloma • Bone marrow examination (shows atypical
plasma cells).
Q: Write down the radiological finding of this X-ray.
• Plasma protein electrophoresis (shows M-band).
A: X-ray of skull in lateral view showing multiple
punched-out lytic lesions of variable sizes and • Serum immunoelectrophoresis.
shapes all over the skull. • Urine for Bence-lones protein.
____________ 1_4_- X-RAY, CT AND MRI __

Hereditary Haemolytic Anaemia


Q: Mention one investigation to confirm the diagnosis.
A: Haemoglobin electrophoresis.

Q: Mention one simple investigation helpful for the


diagnosis.
A: CBC with PBF (microcytic hypochromic anaemia).

Q: What investigations should be done in this patient?


A: As follows:
• CBCwith PBF(microcytic hypochromic anaemia).
• Reticulocyte count by supravital stain (usually
high in haemolytic anaemia).
• Serum bilirubin (increased).
Fig. A: Thalassaemia • Haemoglobin electrophoresis (shows absent
or grossly reduced HbA and increased HbF in
Q: Write down the radiological fmding of the X-ray
~-thalassaemia major. In case of ~-thalassaemia
shown in Figure A. minor, HbA2 is increased).
A: X-ray of skull in lateral view showing:
- Widening of diploic space.
• Thinning of outer table.
• Thickening and coarsening of trabeculae, giving
rise to hair-on-end appearance.

Q: What is the most likely diagnosis?


A: Hereditary haemolytic anaemia, most likely
~-thalassaemia major.

Q: Name four examples.


A: As follows:
• ~-Thalassaemia major. Fig. B: X-ray of hands in hereditary haemolytic anaemia

• Haemoglobin E disease (more common).


• Thalassaemia E disease (double heterozygous). Q: Write down the radiological finding of the X-ray
shown in Figure B.
• Hereditary spherocytosis.
A: X-ray of both hands in anteroposterior view
showing:
Q: Write down five important physical findings in this
• .Widening of medullary cavity.
patient.
• Thinning of cortex.
A: As follows:
• Reduction of number of trabeculae and thickening
• Anaemia. of remaining trabeculae.
• Jaundice. • Generalized osteopaenia.
• Splenomegaly.
• Frontal and parietal bossing. Q: What is the most likely diagnosis?
• Mongoloid facies. A: Hereditary haemolytic anaemia.
Acromegaly
Q: What is your radiological diagnosis?
A: Acromegaly (due to pituitary adenoma).

Q: What other investigations should be done.


A: As follows:
• X-ray of hands and feet (to see heel pad).
• Magnetic resonance imaging (MRI) of skull.
• Growth hormone (GH) assay
(radioimmunoassay) .
• Glucose tolerance test with simultaneous
measurement of GH.
• Measurement of insulin-like growth factor-1
(IGF-I) (also called somatornedin C).
• Assessment of other anterior pituitary hormones.
• Perimetry (bitemporal hemianopia).

Acromegaly Q: Mention one specific treatment.


A: Trans-sphenoidal removal of adenoma.
Q: What are your radiological findings?
A: X-ray of skull in lateral view showing: Q: Mention two drugs that can be used.
• Skull is enlarged. A: As follows:
• Frontal sinus is enlarged. • Somatostatin analogue, e.g. octreotide or
• Sella turcica is enlarged. lanreotide.
• There is erosion of anterior and posterior clinoid • Bromocriptine (dopamine agonist).
process. • Pegvisomant (a peptide GH receptor antagonist).

Tophi
• Two bony outgrowths-one large involving the
first MTP joint and one small in the second distal
interphalangeal (DIP) joint.
• Presence of lytic lesion.
• Destruction of the first interphalangeal joint.

Q: What is your radiological diagnosis?


A: Chronic tophaceous gout.

Q: How to confirm your diagnosis?


A: As follows:
• Aspiration of fluid or tophi to see monosodium
urate monohydrate (MSUM) crystals under
Tophi polarized microscope (needle-shaped, negatively
birefringent crystals).
Q: What are your radiological findings?
• Serum uric acid (high).
A: X-ray of left foot showing:
14 • X-RAY, (T AND MRI __

Rheumatoid Hand
Q: Mention five bad prognostic factors of this disease.
A: As follows:
• Higher baseline disability.
• Female gender.
• Involvement of metatarsophalangeal (MTP)
joints.
• Positive rheumatoid factor.
• Disease duration of over 3 months.

Q: Mention five investigations.


A: As follows:
• CBC with ESR (high ESR, anaemia).
• Rheumatoid arthritis (RA) and Rose-Waaler
(RW) tests.
• Anticyclic citrullinated peptide (anti-CCP)
antibody (helpful for early diagnosis, highly
specific for rheumatoid arthritis).
Rheumatoid hand • X-ray chest [to see diffuse parenchymal lung
disease (DPLD), Caplan syndrome].
Q: Write down the radiological findings of this X-ray. • C-reactive protein (CRP) (high).
A: X-ray of both hands in AP view showing: Q: What are the principles of treatment?
• Reduction of all joint spaces. A: As follows:
• Periarticular osteopaenia involving • Relief of symptoms: Rest, nonsteroidal anti-
metacarpophalangeal and wrist joints of both inflammatory drug (NSAID), physiotherapy,
hands. explanation and reassurance.
• Disorganisation of all proximal interphalangeal • Suppression of activity and progression of
(PIP) joints of both hands. disease by disease-modifying antirheumatic
• 'Z' -deformity of thumb. drugs (DMARDs) (methotrexate, sulphasalazine,
• Ulnar deviation of both hands. hydroxychloroquine. leflunomide. etc.).
Q: What is your radiological diagnosis? • Biological agents (rituximab, tocilizumab, etc.).
A: Rheumatoid arthritis • Restoration of function of affected joints.

Resorption of Terminal Phalanges


Q: What are your radiological findings?
A: X-ray of both hands in AP view showing:
• Periarticular osteopaenia.
• Tapering of terminal phalanges.
• Resorption of distal phalanges of fingers .
• There is calcification at the tip of left index and
right thumb.

Q: What is your probable diagnosis?


A: Systemic sclerosis.

Q: What is the likely cause in this case?


A: eRST or CREST syndrome.
Resorption of terminal phalanges
_ SHORT CASES IN CLINICAL MEDICINE

Rickets
• 25-Hydroxycholecalciferol, 50 ug daily or active
vitaminD metabolite( l-c-bydroxycholecalciferol},
1-2 ug daily or 1,25 di-hydroxychoJecalciferol,
0.25 to 1.5 I-lgdaily.
• Plus calcium-SOO-1000 mg daily. Higher
dose may be required in patients with
malabsorption.
• Adequate exposure to sunlight.
• Dietary supplement.

Fig. A: Rickets

Q: Write down the radiological findings of the X-ray


shown in Figure A.
A: X-ray of both hands in AP view showing:
• Widening, splaying, cupping and irregularity of
metaphysic.
• Distance between epiphysis and metaphysis is
increased (zone of provisional calcification is
lost).
Fig. B: Rickets

Q: What is your radiological diagnosis? Q: Write down the radiological findings of the X-ray
A: Rickets. shown in Figure B.
A: X-ray of leg induding knee joint and ankle joint
Q: Mention three other investigations. showing:
A: As follows: • Widening, splaying, cupping and irregularity of
• X-ray of elbows and knees. metaphysis.
• Serum calcium and phosphate (both are low). • Distance between epiphysis and metaphysis is
increased (zone of provisional calcification is
• Serum alkaline phosphatase (high). .
lost).
• Serum 25-hydroxy proline (low or absent).
• Bowing of tibia and fibula.

Q: How to treat? Q: What is your radiological diagnosis?


A: As follows: A: Rickets.
14 • X-RAY, CT AND MRI __

Scurvy
• Epiphysis as ring-shaped, sclerotic and sharply
marginated called Wimberger sign.
• Metaphysis is dense (zone of provisional
calcification), giving a white line called Frankel
line.
• Beneath this line, there is a lucent zone caJled
Trummerfeld zone.
• At the corner, there is a spur called Pelkan spur.

Q: What is your radiological diagnosis?


A: Scurvy.

Q: What are the causes?


A: As follows:
• Dietary deficiency=lack of fruits and fresh
vegetables in children >2 months of age.
• In infants-delayed weaning, lack of fruit juice,
deficiency in the lactating mother.

Q: How to treat?
A: As follows:
• Vitamin C, 250 mg-three to four times daily
orally.
• Dietary supplements especially fresh fruits
Scurvy (orange, mango, pineapple, guava, etc.) and liver
extract.
Q: Write down the radiological findings of this X-ray. • Bottle-fed infants should be given fruit juice.
A: X-r~y of knee including lower end of femur and Nursing mother should take sufficient vitamin C,
upper end of tibia-fibula showing: which is secreted in the breast milk.

Avascular Necrosis of Femoral Head


• Rarefaction and sclerotic small head of left femur.
• Joint space between head and acetabulum is
increased.

Q: What is your radiological diagnosis?


A: Avascular necrosis of head of left femur (called oste-
onecrosis) .
Q: What is the presentation?
A: Pain in the hip and difficulty in walking.
Q: How to confirm your diagnoses?
A: As follows:
• X-ray (may be normal in early stage).
• MItt-more confirmatory (100%).
• Bone scan-increased uptake (helpful in early
Avascular necrosis of the femoral head stage).

Q: Write down the radiological finding of this X-ray. Q: Mention five causes.
A: X-ray of pelvis in AP view showing: A: As follows:
__ SHORT CASES IN CLINICAL MEDICINE

• Prolonged steroid therapy (other drug- • Exposure to high barometric pressure-in


prolonged use of heparin). deep sea divers (Caisson disease) and tunnel
• Cushing syndrome. workers.
• Osteoporosis. • Perthes disease.
• Alcohol abuse. • Radiation therapy.
• Collagen disease (SLE, rheumatoid arthritis, • Post-traumatic.
systemic sclerosis, vascul itis).
N.B. Other causes are:
• Sickle cell anaemia (and other
haemoglobinopathies) .

Endoscopic Retrograde Cholangiopancreatography (ERCP)


Q: What are your findings?
A: This is an endoscopic retrograde cholangiopan-
creatography (ERCP) showing a linear translucent
shadow within the common bile duct.

Q: What is your diagnosis?


A: Round worm in common bile duct.

Q: How does the patient present?


A: As follows:
• Upper abdominal pain, mostly in right
hypochondrium.
• Features of obstructive jaundice and
cholangitis.

Q: How to treat such a case?


ERCPshowing Ascaris lumbricoides in common bile duct A: The worm is removed during ERCP.

Achalasia Cardia
• Dilatation of the oesophagus with smooth taper-
ing of its lower end.
• Absence of fundal gas.

Q: What is your radiological diagnosis?


A: Achalasia cardia.

Q: Mention one investigation to confirm your diagnosis.


A: Oesophageal manometry.

Q: Mention three complications.


A: As follows:
• Oesophageal carcinoma.
• Aspiration pneumonia
• Malnutrition.

Achalasia cardia Q: How to treat?


A: As follows:
Q: Write down the radiological finding of this X-ray. • Endoscopic dilatation by pneumatic bougie.
A: Barium swallow of the oesophagus showing: • Heller cardiomyotomy.
14 • X-RAY, (T AND MRI __

Carcinoma of Oesophagus
Q: Write down the radiological finding of this X-ray.
A: Barium swallow of the oesophagus showing
irregular filling defect at the lower end of
oesophagus.

Q: What is your radiological diagnosis?


A: Carcinoma of oesophagus.

Q: Mention one investigation to confirm your


diagnosis.
A: Endoscopy and biopsy.

Carcinoma of oesophagus

Carcinoma Stomach

Carcinoma stomach

Q: Write down the radiological finding of this X-ray. Q: Mention one investigation to confirm your
A: Barium meal X-ray of stomach showing irregular diagnosis.
filling defect at the distal part of stomach. A: Endoscopy and biopsy.

Q: What is your radiological diagnosis?


A: Carcinoma of stomach.
_ SHORT CASES IN CLINICAL MEDICINE

Gastric Outlet Obstruction


Q: Write down the radiological finding of this X-ray.
A: Barium meal X-ray of stomach showing:
• Stomach is hugely dilated with food residue.
• Dye is not passing beyond the pylorus.

Q: What is your radiological diagnosis?


A: Gastric outlet obstruction.

Q: What is the likely cause?


A: Chronic duodenal ulcer with pyloric stenosis.

Q: What other investigations do you suggest?


A: Endoscopy and biopsy.

Q: What is the treatment?


A: Gastrojejunostomy with vagotomy.

Q: Name one physical sign.


A: Visible peristalsis from left to light in upper
abdomen.

Q: Mention one symptom.


A: Projectile or induced vomiting.

Gastric outlet obstruction

Pancreatic Calcification
Q: What is the underlying diagnosis?
A: Chronic pancreatitis.

Q: Mention three causes.


A: As follows:
• Chronic pancreatitis, commonly alcoholic.
• Fibrocalcific pancreatic disease (FCPD).
• Hereditary pancreatitis.
• Hypercalcaemia due to any cause.
• Haemochromatosis.
• Idiopathic chronic pancreatitis.
• Post-traumatic pancreatitis.
Pancreatic calcification
Q: Mention five further investigations.
A: As follows:
Q: Write down the radiological finding of this X-ray.
A: Plain X-ray of abdomen showing multiple calcified • Ultrasonogram.
shadows superimposed on vertebral column and • cr scan or MRI of abdomen.
also on left side along the anatomical distribution • Magnetic resonance cholangiopancreatography
of pancreas. (MRCP).
• Endoscopic retrograde cholangiopancreatog-
Q: What is your radiological diagnosis? raphy (ERCP).
A: Pancreatic calcification. • Pancreatic function test.
14 • X-RAY, CT AND MRI __

Polycystic Kidney Disease


Q: Write down the radiological finding of this X-ray.
A: Intravenous urograms (IVU) showing:
• Enlargement of both kidneys-left one larger
than the right.
• Distortion, stretching and elongation of
pelvicalyceal system, giving rise to spidery
appearance.
• Left ureter is dilated and shifted medially towards
the vertebral column.

Q: What is your radiological diagnosis?


A: Polycystic kidney disease.

Q: Mention a single investigation.


A: Ultrasonography of both kidneys.

Q: If the patient is unconscious,


what is the likely
diagnosis?
A: Subarachnoid haemorrhage (due to rupture of berry
aneurysm).

Q: Mention one treatment.


Polycystic kidney disease A: Ultrasonic-guided aspiration oflarge cyst.

CTSCAN

cr scan is a radiological procedure that uses X-ray and • Cheaper than MRJ and many other imaging
a computer to produce a series of cross-sectional image modalities.
of the inside of the body. • Quickerthan MRIand manyotherimagingmodalities,
so it is very useful for quick screening in emergency
Uses: conditions like subarachnoid haemorrhage.
• To diagnose internal injuries, bleeding, mass Advantages of multislice cr over helical CT:
lesion, etc. • Increased speed and ability to perform volume
• To diagnose various rheurnatological conditions acquisition.
including bone loss, joint deformity, calcification, • Multislice cr allows faster scanning, thinner
etc. sections or larger scan ranges, which lead to less
• To guide procedures such as surgery, biopsy and motion artifact, especially in critically ill patients,
radiation therapy. children and trauma patients.
• To monitor progression of various diseases like • Helical cr is a continuous volume acquisition that
lung nodule, cancer, etc. ensures that no lesions are lost due to respiration or
• Contrast cr scan is used to assess the vascularity after motion-related artifact. The prime advantage
and the nature of a lesion. of volume acquisition is the improved three-
• cr angiogram is used to evaluate the vascular dimensional capability.
system of an organ. Disadvantages of CT scan:
Advantages: • Exposure to radiation.
• Higher image resolution than X-ray. • Reaction to contrast materials if contrast CT scan
• Noninvasive technique. is performed.
• May be used in situations where magnetic reso- • Image quality is lower than MRI.
nance imaging (MRl) is contraindicated. • Poor visualization of infarction.
~ SHORT CASES IN CLINICAL MEDICINE

MRI SCAN
MRI is a nonionising imaging technique using mag- • It enables the detection of abnormalities that might
netic fields and radiofrequency waves to visualize the be obscured by bone with other imaging methods.
internal anatomical structures. • Direct multiplanar imaging in transverse, coronal
Uses: and sagittal planes or in any other desired plane is
possible.
• To diagnose internal injuries, infarction, mass • Free from ionizing radiation. So, safe for pregnant
lesion, etc. women and children.
• To diagnose defect with myelination (e.g. multiple • Contrast material used in MRI is less likely to
sclerosis). produce an allergic reaction than the iodine-based
• To diagnose diseases of the spine and spinal cord. materials used for conventional X-ray and cr
• Contrast MRI is used to assess the vascularity and scanning.
the nature of a lesion. • Noninvasive technique.
• Magnetic resonance angiography (MRA) is used to
Disadvantages:
evaluate the vascular system of an organ.
• Functional MRI (fMRI) of the brain can be used • Patient with electronic devices (e.g. cardiac pace-
to identify important language and movement maker, implantable heart defibrillator, etc.),
control areas in the brain in people who are being metallic implant (e.g. metallic valve, joint prosthe-
considered for brain surgery. ses, etc.) or metallic artificial life support cannot
be scanned by MRI due to strong magnetic fields.
Advantages:
• Scanning is time consuming.
• High-resolution imaging. • Movement degrades image.
• Images of the brain, other cranial structures and • More expensive.
the spine are more clear and detailed than with • AJlergic reaction to contrast material may occur
other imaging methods. So MRI is velY useful in rarely (especially in patients with kidney problem
early diagnosis and evaluation of intracranial and who are on dialysis).
spinal disorders including tumour. • Difficult to perform in claustrophobic patients.

Few Common CT Scans

cr scan of the brain shows hyperdense lesion in right cr scan of the head shows high attenuation (white
parietal lobe with perilesional oedema. The diagnosis is areas) with obliteration of the ventricles. The diagnosis
intracerebral haemorrhage in right parietal lobe. is subarachnoid haemorrhage.
14 • X-RAY, (T AND MRI _

cr scan of the head shows crescent-shaped hyperdense


extra-cerebral collection within the subdural space. The
diagnosis is subdural haematoma on the left side.
Contrast cr scan of head showing a ring-like cystic
hypodense lesion on the right parietal lobe. The diag-
nosis is cerebral abscess.

cr scan of the head shows high density bi-convex area.


The diagnosis is extradural haematoma on the left side.

cr scan of the head shows hyperdense, irregular mass


in the frontal region. The diagnosis is meningioma.

cr scan of the brain shows hypodense area on the right cr scan of the head shows marked dilatation of both
side with shifting of the ventricle to the left side. The lateral ventricles with thinned brain tissue outside the
diagnosis is right-sided cerebral infarction. hypodense area. The diagnosis is hydrocephalus.
_ SHORT CASES IN CLINICAL MEDICINE

cr scan of the abdomen shows a large hypodense cystic


lesion with septation within the right lobe of liver. The
cr scan of the abdomen shows irregular hypodense diagnosis is hydatid cyst in liver.
mass in the left lobe of the liver. The diagnosis is
hepatoma.

cr scan of the abdomen shows multiple hypodense MRI of the dorsolumbar spine lateral view showing
areas within both right and left lobes of the liver. The destruction of the L2 and L3 vertebral bodies with
diagnosis is multiple secondaries in liver. reduction of joint space. The diagnosis is Pott disease.

I \
~;

~
~ J
••
j
• ~

.,~ J'" .
j
~.
· i
\
cr scan of the abdomen shows multiple hypodense MRI of brain sagittal view shows large irregular hyper-
area with ragged irregular margin within the right lobe dense area in pituitary fossa. The diagnosis is pituitary
of the liver. The diagnosis is multiple liver abscess. macroadenoma.
14 • X-RAY, CT AND MRI _

MRI of brain coronal view shows large irregular hyper-


dense area in pituitary fossa. The diagnosis is pituitary
macroadenorna. MRI of the brain showing mixed hyperintense mass in
the left cerebellopontine angle. The diagnosis is acous-
tic neuroma.

MRI of brain shows large hyperintense mass in the


left cerebellar hemisphere. The diagnosis is cerebellar
tumour.
CHAPTER 15

ECG
"A good start is a good help"
-Author

Basic Concept of ECG

Electrocardiogram (ECG) is the graphical representation Leads


of electrical potentials that are produced when the
electric current passes through the heart. In a normal ECG recording, there are 12leads-3 bipolar
standard leads, 3 unipolar limb leads and 6 chest leads.
Waves in ECG 1. Bipolar standard leads (limb leads) designated as
ECG records the electrical impulse as waves or deflec- LI, LII and LIIl.
tions on ECG paper. One beat is recorded as a group of • LI: Difference of potential between left arm and
waves (P-QRS-T). right arm (IA and RA).

Waves in ECG tracing

1. P: Represents atrial depolarization. • LII: Difference of potential between right arm


2. P-R interval: Represents the time taken for the cardiac and left leg (RA and LL).
impulse to spread from SAnode to the ventricles. • LIIl: Difference of potential between left arm
3. QRS complex: Represents ventricular depolarization. and left leg (LA and LL).
• Q( q): First negative deflection before R-wave.
2. Unipolar limb leads (augmented limb leads)
• R(r): First positive deflection.
designated as augmented vector right (AVr),
• S(s): Negative deflection after R-wav_e.
augmented vector left (AVI) and augmented vector
4. T-wave - Represents ventricular repolarization.
foot (AVf). These leads have very-low voltage that
Other waves: cannot be recorded clearly. So, recordings from
1. J: At the beginning of ST-segment. these leads are increased in amplitude called aug-
2. U: Found afterT-wave, preceding the next P-wave. mented unipolar leads.
1S. ECG_

• AVr: Augmented unipolar right arm (RA) lead. • V5: 5th Intercostal space in left anterior axillary
Records the changes of potential occurring in line.
the part of heart facing towards right shoulder. • V6: 5th Intercostal space in left midaxillary line.
• AVI: Augmented unipolar left arm (LA) lead.
View of the heart in all leads:
Records the changes of potential of heart facing
towards the left shoulder. • LI, AVI, V5 and V6: Reflects lateral (or anterolateral
• AVf: Augmented unipolar left leg (LL) lead. aspect of heart). M
()
Records the changes of potential of heali facing • LIl, LIll and Avi: Reflects inferior aspect of heart. (0
towards the left hip. • vi and V2: Reflects right ventricle.
3. Chest Leads (unipolar) Designated by V. Elec- • V3 and V4: Reflects interventricular septum.
trodes are placed in the following places on the • V5 and V6: Reflects left ventricle.
chest wall: • VI-V6: Reflects anterior aspect of heart.
• Vl.: 4th Intercostal space at right sternal border. • LI, AVI, Vl- V6: Reflects extensive anterior aspect of
• V2: 4th Intercostal space at left sternal border. heart or anterolateral.
• V3: Midway between V2 and V4 lead on left side. • LI and AVI: High lateral.
• V4: 5th Intercostal space in left midclavicular line. • Lit un. AVf, 11, AVI, V5 and V6: Inferolateral,

Interpretation of ECG
Before interpretation one must know details about ECG • Myocardial infarction.
paper, standardization and different waves in ECG. • Arrhythmias.
Look at the following points carefully: • Block [first-degree block, sinoatrial block (SA
1. St.andardization (see in the beginning): This is 10 block), atrioventricular block (AVblock), bundle-
mm (1 mY). branch block].
2. Paper speed: 25 mm/s. • Drug effect (such as digoxin).
3. Rhythm: See R-R interval (R-wave to R-wave • Extracardiac abnormalities: Electrolyte imbalance
interval), regular or irregular (LJI is called rhythm (such as hypokalaemia or hyperkalaemia},
lead). hypo- or hypercalcaemia, low-voltage tracing (in
4. Count the heart rate. myxedema, hypothermia, emphysema).
5. Different waves: • Exercise ECG to see coronary artery disease.
• P: Normal, small or tall, inverted, wide, notched,
bifid, variable configuration. ECG Paper
• P-R interval: Normal or prolonged or short.
ECG paper shows small and large squares. In each small
• Q: Normal or pathological.
square, thin horizontal and vertical lines are present at
• R: Normal ortall or short, notched or M-pattern.
I-rnrn interval. A heavier, thick line is present at every
• QRS complex: Normal or wide, high or low
5-mm (5 small squares) interval. Time is measured
voltage, variable or change of shape.
horizontally and height is measured vertically.
• ST-segment: Elevated or depressed.
• T: Normal or tall or small or inverted. 1. One small square:
• U-wave: Normal or small. • Height = 1 mm.
• QT: Short or prolonged. • Horizontal (in time) = 0.04 s.
6. Axis: Whether normal or right- or left-axis
2. One big square (5 small squares):
deviation.
• Height = 5 mm.
7. Abnormalities: Any arrhythmia, infarction,
• Horizontal (in time) = 0.04 x 5 s = 0.2 s.
hypertrophy.
So, 0.2 s = 5 mm.
Q: What are the diseases diagnosed by looking at an 1s = 5/0.2 = 25 mm.
ECG?
So, recording speed is 25 ttun]«. (i.e. 1500 mrn/min}.
A: As follows:
3. Isoelectric line: It is the base line in ECG paper.
• Tachycardia or bradycardia. Waves are measured either above (positive
• Chamber enlargement. deflection) or below (negative deflection).
__ SHORT CASES IN CLINICAL MEDICINE

Standardization of ECG Remember the following points:


• Depolarization indicates initial spread of stimulus
• Normally, l-mv current: 10-mm height (10 small
through the muscle, causing activation or
squares).
contraction.
• Half strength: 5 mm.
• Recording speed: 25 mm/s (i.e. 1500 rum/min]. • Repolarization indicates return of stimulated
muscle to the resting state (recovery from activation
Before telling low voltage or high voltage, see the
or contraction).
normal standardization (i.e. 10 mm in height).
• Depolarization towards the electrode: Positive
Criteria of low-voltage tracing:
deflection (above the isoelectric line).
• In standard limb leads: QRS complex <5 mm
• Depolarization away from the electrode: Negative
(mainly R-wave).
de.flection (below the isoelectric line).
• In chest leads: QRS complex <10 mm (mainly
R-wave). • Rhythm: Interval betwee.n two successive RR.

Normal ECG
Characteristics of normal ECG: • P-P interval: Distance between two successive
• Normal ECG consists ofP-wave (atrial beat), P-waves. In sinus rhythm, P-P interval is regular.
followed by QRS complex, ST- and T-wave • R-R interval: Distance between two successive
(ventricular beat). R-waves.
• Capital letter P, Q, R, S, T: Indicates large wave • Q- T interval: Distance interval between the
beginning of Q-wave and the end ofT-wave.
(>5 mm).
• Small letter p, q, I, S, t: Indicates small wave «5 mm). • S- T segment: Distance from the end of QRS
complex to the beginning of T-wave. It indicates
Intervals and segment in ECG
the beginning of ventricular repolarization.
• P-R interval: Distance between the beginning of Normally, it is in isoelectric line, but may vary
P and beginning of QRS (Q). from -0.5 to +2 mm in chest leads.

Details of Waves and Intervals

P-Wave Causes of absent P-wave:


• Atrial fibrillat.i.on (P is absent or replaced by
Characters of normal P-wave:
fibrillary f-wave).
• P-wave is better seen in LII (also seen in VI). • Atrial flutter (P is replaced by flutter wave, which
• Normal P is rounded-neither peaked nor notched. shows saw-tooth appearance).
• Width or duration (in time, horizontally): 0.10 s • Sinoatrial (SA) block or sinus arrest.
(2.5 small sq.}. • Nodal rhythm (usually abnormal, small P-wave).
• Height: 2.5 mm (2.5 small squares) height x • Ventricular ectopic and ventricular tachycardia.
duration = 2.5 x 2.5 small squares. • Supraventricular tachycardia (P-wave is hidden
within QRS complex, due to tachycardia).
• P-wave is upright in all leads, mainly LI, LII
and Avf (except AVr. P is inverted in AVr and • Hyperkalaemia.
• Idioventricular rhythm.
occasionally in AVl).
• P-wave in VI may be biphasic (equal upward Causes of tall P-wave:
and downward deflection), notched and wide • Tall P-wave is called P-pulmonale (height> 2.5
(activation of tight atrium produces positive mrn, i.e. > 2.5 small squares).
component and activation of left atrium produces • It is due to right atrial hypertrophy or enlargement.
negative component). Causes of small P-wave:
• Atrial tachycardia.
Abnormalities of P-wave: It may be absent, tall or
• Atrial ectopic.
small, wide, notched, biphasic, inverted, variable,
• Nodal rhythm (high nodal).
multiple.
• Nodal ectopic (high nodal).
_____________________________ l_S__ •__ E_CG I3IIIIIII
Causes of wide P-wave: Variable P-l? interval: Causes are-
• Broad and notched P-wave is called P-mitrale • Wenckebach phenomenon (Mobitz type I): There
(duration> 0.11 s or> 2.5 small squares). is progressive lengthening of P- R interval followed
• It is due to left atrial hypertrophy or enlargement. by a drop beat.
• In VI, P-wave may be biphasicwithasmall positive • Partial heart block (Mobitz type II): P-R interval
wave preceding a deep and broad negative wave is fixed and normal, but sometimes P-wave is not
(indicates left atrial enlargement or hypertrophy). followed by QRS complex.
Causes of inverted P-wave (negative in LI, LII and AVf): • 2:1 AVblock: Alternate P-wave is not followed by
QRS complex.
• Incorrectly placed leads (reversed arm electrodes).
• Complete AV block: No relation between P-wave
• Dextrocardia. and QRS complex.
• Nodal rhythm with retrograde conduction.
• Wandering pacemaker: Variable configuration of
• Low atrial and high nodal ectopic beats. P-wave.
Causes of variable P-waves:
• Presence of variable P-waves indicates wandering Q-Wave
pacemaker.
Characters of normal Q-wave:
Causes of multiple P-waves (consecutive two or more):
• Q-wave is usually absent in most of the leads.
• AVblock (either partial or complete heart block).
However, small Q-wave may be present in I, [I,
• Supraventricular tachycardia (SVf) with AVblock.
AVI,V5 and VG.This is due to septal depolarization.
• Small q may be present in LIII (which disappears
P-R Interval with inspiration).
Characters of normal P-R interval: • Depth: <2 mm (2 small squares).
• Normal P-R interval: 0.12-0.20 s (maximum five • Width: One small square.
small squares). • It is 25% or less in amplitude of the following
o In children, upper limit is 0.16 s. R-wave in the same lead.
o In adolescent, upper limit is 0.18 s. Characters of pathological Q-wave:
o In adult, upper limit is 0.22 s. • Deep >2 mm (2 small squares).
• PR interval is short, if it is < 0.10 s; and long, if it • Wide> 0.04 s or more (>1 mm or 1 small square).
is >0.22 s. • Should be present in more than one lead.
Abnormalities of P-R Interval: It may be prolonged, • Associated with loss of height of R-wave.
short and variable. • Q-wave should be >25% of the following R-wave
of the same lead.
Prolonged P-R interval (>0.2 s): It is due to first-degree
Causes of pathological Q-wave:
heart block Causes are:
• Myocardial infarction (com monest cause).
• lschaemic heart disease (occasionally, inferior
myocardial infarction). • Ventricular hypertrophy (left or right).
• Acute rheumatic carditis. • Cardiomyopathy.
• Myocarditis (due to any cause). • LBBB.
• Atrial dilatation or hypertrophy. • Emphysema (due to axis change or cardiac rotation).
• Q-wave only in LIII is associated with pulmonary
• Hypokalaemia.
embolism (Sl, QIII and TIll pattern).
• Drugs: Digitalis toxicity, quinidine, occasionally
~-blocker, calcium channel blocker (verapamil).
N.B. Remember the following points:
Short P-R interval «0.12 s): Causes are- • Q-wave in VI, V2 and V3 may be seen in left
• Wolff-Parkinson-White (WPW) syndrome: In this ventricular hypertrophy (LVH), confused with
case, there is 8-wave. old myocardial infarction.
• Lown-Canong-Levine (LGL) syndrome: In this • Abnormal Q-wave in LIII may be found in
case, there is no 8-wave. pulmonary embolism.
• Nodal rhythm. • Abnormal Q-wave in Llll and AVf may be
• Nodal ectopic (high nodal). found in Wolff-Parkinson-White syndrome
• Occasionally, if dissociated beat is present and also (WPW syndrome) (confuses with old inferior
in infant, steroid therapy. myocardial infarction).
_ SHORT CASES IN CLINICAL MEDICINE

R-Wave S-Wave
Characters of normal R-wave: Characters of normal S-wave:
• Duration <0.01 s. • Normally, deep in VI and V2.
• R-wave is usually small «1 mm) in VI and V2. It • Progressively diminished from VI to V6.
ina-eases progressively in height in V3-V6 (tall in • In V3, R- and S-waves are almost equal (correspond
V5 and V6). with interventricular septum).

Normal height of R-wave (If R-wave is >25 mrn, it is


QRSComplex
always pathological):
• AVI <13 mm. Characters of normal QRS complex:
• AVf <20 mm. • QRS complex is predominantly positive in Ll , AVI,
• V5 and V6 <25 mm. V5 andV6.
• It is negative in AVr, VI and V2.
Abnormalities of R-wave: It may be tall, small. poor
• In VI, S-wave is greater than R-wave, and in V5
progression.
and V6, R-wave is tall.
Causes of tall R-wave: • Normal duration of QRS complex is 0.08-0.11. s
1. Left ventricular hypertrophy (in V5 orV6 >25 mrn, (<3 small squares) and height < 25 mm.
AVI >13 mm, aVf>20 mm).
Abnormalities of QRS complex: It may be of high
2. In VI, tall R-wave may be due to:
voltage, low voltage, wide, may change in shape and
• Normal variant. be variable.
• Right ventricular hypertrophy (RVH).
Causes of high-voltage QRS complex:
• True posterior myocardial infarction.
• WPW syndrome (type A). • lncorrect calibration.
• Right bundle branch block (REBB). • Thin chest wall.
• Dextrocardia. • Ventricular hypertrophy (right or left or both).
• WPW syndrome.
Causes of small Rs-uiaue: Looks like low-voltage tracing.
• True posterior myocardial infarction (in.Vl and V2).
• Incorrect ECG calibration (standardization).
• Obesity.
Causes of low-voltage QRS complex «5 mm in LJ, LlI, LlII
and <10 mm in chest leads):
• Emphysema.
• Pericardial effusion. • Incorrect, calibration.
• Hypothyroidism. • Thick chest wall or obesity.
• Hypothermia. • Hypothyroidism.
• Pericardial effusion.
R-wave progression: The height of R-wave gradually
• Emphysema.
increases from Vl to V6. This phenomenon is called
• Chronic constrictive pericarditis.
R-wave progression.
• Hypothermia.
Poor progression of R-wave: Normally, amplitude of
R-wave is tall inV5 and V6. In poor R-wave progression, Causes of wide QRS complex (>0.12 s, 3 small squares):
amplitude ofR-wave is progressively reduced in V5 and • Bundle branch block (LBBB or REBB).
V6. Causes are: • Ventricular ectopics.
• Anterior or anreroseptal myocardial infarction. • Ventricular tachycardia.
• Left bundle branch block (LBBB). • Idioventricular rhythm.
• Left ventricular hypertrophy (though R-wave is • Ventricular hypertrophy.
tall in most cases). • Hyperkalaemia.
• Dextrocardia. • WPW syndrome.
• Cardiomyopathy. • Pacemaker (looks like LBBS with spike).
• Chronic obstructive pulmonary disease (COPD). • Drugs (quinidine, procainamide, phenothiazine,
• Left-sided pneumothorax. tricyclic antidepressants).
• Left-sided pleural effusion (massive). Causes of changes in shape of QRS complex:
• Marked clockwise rotation. • Right or left bundle branch block (slurred or
• Chest electrodes placed incorrectly. M-pattern).
• Deformity of the chest wall. • Ventricular tachycardia.
• Normal variation. • Ventricular fibrillation.
• Hyperkalaernia.
• WPW syndrome.
Abnormalities
peaked, tented.
of T-wave: It may be inverted, tall,
--
Causes of variable QRS complex: Causes of T inversion:
• Multifocal ventricular ectopics. • Myocardial ischaemia and infarction.
• Torsades de pointes. • Subendocardial myocardial infarction (non-Q-
• Ventricular fibrillation. wave myocardial infarction).
• Ventricular ectopics.
• Ventricular hypertrophy with strain.
STSegment
Characters
--------------------------------
of normal ST segment:
• Acute pericarditis.
• Cardiomyopathy.
• Normally, it is in isoelectric line (lies at same level • Myxoedema.
of ECG baseline). • Bundle branch block.
• ST elevation is normal up to 1 mm in limb leads • Drugs (digitalis, emetine, phenothiazine).
and 2 mm in chest leads (mainly VI-V3). • Physiological (smoking, anxiety, anorexia, exercise,
• Normally, ST segment may be depressed, -cIrnm. after meal or glucose).

Abnormalities of ST segment: It may be elevated or Causes of tall, peaked T-wave:


depressed.
• Hyperkalaemia (tall, tented or peaked).
Causes of ST elevation (>2 mm): • Hyperacute myocardial infarction (tall T-wave).
• Recent myocardial infarction (ST elevation with • Acute true posterior myocardial infarction (tall
convexity upward). T'-wave in VI-V2).
• Acute pericarditis (Sf elevation with concavity • May be normal in some Africans and Asians.
upward, chair shaped or saddle shaped).
• Prinzmetal angina (ST elevation with tall Causes of small T-wave:
T-wave). • Hypokalaemia.
• Ventricular aneurysm (persistent ST elevation). • Hypothyroidism.
• Early repolarization (high take off). • Pericardial effusion.
• Normal variant in Africans and Asians.
• May be in hyperkalaernia. U-Wave
Causes of ST depression (below the isoelectric line):
Characters of normal U-wave:
• Acute myocardial ischaemia (horizontal or
• It is better seen in chest leads (V2-V4).
down slope ST depression with sharp angle
• Normal amplitude is 1 mm (2 mm in athlete).
Sf - T junction).
• Ventricular hypertrophy with strain (ST depression Abnormalities of U-wave: It may be inverted or
with convexity upward and asymmetricTinversion). prominent.
• Digoxin toxicity (sagging of Sf depression-like
Causes of inverted U-wave:
thumb impression, also called reverse tick).
• Acute true posterior myocardial infarction (in VI • Ischaemic heart disease.
and V2), associated with dominant R- and tall • Left ventricular hypertrophy with strain
upright T-wave. (hypertensive heart disease).

- Causes of prominent U-uiaue:


T-Wave
• May be normally present (usually small).
Characters of normal T-wave: • Hypokalaemia (commonest).
• Upright in all leads, except AVr. • Bradycardia.
• Usually, more than 2 mm in height. • Ventricular hypertrophy.
• May be normally inverted in VI and V2. • Hyperthyroidism.
• Normally, not more than 5 mrn in standard leads • Hypercalcaemia.
and 10 mm in chest leads. • Drugs (phenothiazine, quinidine, digitalis).
• Minimum one-fourth of R-wave of the same lead.
• Tip ofT-wave is smooth (rounded). ~...:!!.. Large If-wave may cause torsades de pointes.
_ SHORT CASES IN CLINICAL MEDICINE

Causes of long Q-T interval:


Q-T Interval
• Hypocalcaemia.
Characters of normal Q- T interval:
• Bradycardia.
• Normal Q-T interval is 0.35-0.43 s. • Acute myocarditis.
• Its duration varies with heart rate, becoming • Acute myocardial infarction.
shorter as the heart rate increases and longer as the • Hypothermia.
heart rate decreases. • Drug (quinidine, procainamide, flecainide, ami-
• It is better seen in AVI (because there is no U-wave). odarone, tricyclic antidepressant, disopyramide,
pentamidine) .
Abnormalities of Q- T interval: It may be short or long. • Cerebral injury (head injury, intracerebral
Causes of short Q-T interval: haemorrhage) .
• Hypertrophic cardiomyopathy.
• Digoxin effect.
• Hypercalcaemia. • During sleep.
• Hyperthermia. • Hereditary: Jervell and Lange-Nielsen syndrome,
Romano-Ward syndrome.
• Tachycardia

Rhythm of Heart
To see the rhythm-see the successive R-R interval. Characters of sinus rhythm: It shows the following five
• If the R-R interval is equal, it is called regular characters:
rhythm. • P-wave is of sinus origin (means characters of
,it,! ', ... ''t~",,·!f the R- R in terval is irregular, then it is called normal P-wave).
",. ... ,," ,I r :.... irregular rhythm. • P-waves and QRS complexes are regular (which
q. .J \., ...~.
means P-P and R-R interval should be constant
Causes of irregular rhythm
and identical).
1. Physiological: Sinus arrhythmia.
• Constant P-wave configuration in a given lead.
2. Pathological:
• P-R interval and QRS interval should be within
• Atrial fibrillation. normal limit.
• Atrial flutter. • Rate should be between 60 and 100 beats/min
• Ectopic beat. (atrial and ventricular rates are identical).
• SA block or sinus arrest.
Q: What is arrhythmia?
• Atrial tachycardia with block
A: It is the abnormality in initiation or propagation of
• Second-degree heart block.
cardiac impulse.
• Ventricular fibrillation.

Calculation of Heart Rate


Methods vary according to the cardiac rhythm, whether • If small square is calculated, heart rate is =
regular or irregular. Heart rate is the number of beats/min. 1500/(sma]] squares between R-R interval or
In the ECG paper: p-p interval).
• 0.04 s = 1 small square. • If large square is calculated, heart rate is = 300/
• 0.2 s = 5 small squares or 1 large square.
(large squares between R-R interval or P-P
• So, 1 s = 25 small squares or 5 large squares.
interval).
• So, 1 min = 25 x 60 = 1500 small squares or
5 x 60 = 300 large squares.
2. When the rhythm is irregular:
Heart rate is determined in the following way:
• Count the number of R-waves 111 30 large
1. When the cardiac rhythm is regular:
squares (it is equivalent to 6 s).
• Calculate the R-R or P-P interval in small
squares or large squares (if the rhythm is sinus, • Then simply multiply this by 10 (it becomes
R-R or P-P interval is same). rate in 1 min).
15 • ECG I3IIIIIII
--------------~----~----------------~----~
Cardiac Axis
Definition: It is the sum of all the depolarization waves • Left ventricular hypertrophy.
as they spread through the ventricles as seen from the • Left anterior hemiblock.
front. • Left bundle branch block.
Axis determination • Inferior myocardial infarction.
• It can be derived easily from the amplitude of QRS • T-wave from apex of left ventricle.
complex in LI, LII and LIlI. • WPW syndrome (some).
• Greatest amplitude of R-wave in LI or LII or LIIl • Pacing from the apex of the right or left ventricle
indicates the proximity of cardiac axis to that lead. (endocardial pacing).
• Axis lies at 90° to the isoelectric complex, i.e. posi- • Emphysema.
tive and negative deflections are equal in any of the
lead LI, LIl, LIll, AVI, AVr and AVf. Right-axis deviation: When the cardiac axis is between
+90° and +180°. Causes are:
Normal axis is between -30° and +90°.
, • Normal variant (common in children and young
Quick and simple way of determination of cardiac axis:
adult).
• Positive QRS complex in both LI and LII means • Right ventricular hypertrophy (due to any cause
axis is normal. such as-chronic cor pulmonale, pulmonary
• Positive QRS complex in LI and negative in LIII embolism, congenital heart diseases, i.e. tetralogy
(tall R-wave in LI and deep S-wa've in LIll)-means of Fallot).
left-axis deviation.
• Anterolateral myocardial infarctio? (high lateral MI).
• Negative QRS complex in LI and positive in LIII
• Left posterior hemiblock.
(tall R in LIll and deep S in LI)-means right-axis
• Dextrocardia . .\'~ ~"'I\'\
. deviation.
Left-axis deviation: When the cardiac axis is between
• WPW syndrome (type A). 1!L J. LA~
-30 and -90°. Causes are:
0
• Right bundle branch block. 1JI. l' ~D
• Epicardial pacing.
• Normal variant (with advanced age).

SOME COMMON ECG TRACINGS

ECG01: Atrial Fibrillation

Atrial fibrillation
SHORT CASES IN CLINICAL MEDICINE

Q: Write down three important abnormal findings in • Chronic rheumatic heart disease, usually with
this EeG. mitral stenosis.
A: As follows: • Acute myocardial infarction.
• Thyrotoxicosis.
• Ps-wave is absent.
• Hypertension.
• Rhythm is irregularly irregular (R-R interval is
• Lone atrial fibrillation (AF).
irregular) .
• There are fibrillary waves. Q: Write down two important complications.
A: As follows:
Q~ What is the heart rate?
• Thromboembolism (systemic and pulmonary).
A: 125/min.
• Heart failure.
Q: What is your diagnosis?
Q: If the patient is asymptomatic, mention simple
A: Atrial fibrillation. management.
Q: Write down five important causes. A: Low-dose aspirin should be given to prevent
A: As follows: thromboembolism.

ECG 02: Atrial Flutter

Atrial flutter

Q: Write down three important abnormal findings in Qt How to treat?


this ECG. A: As follows:
A: As follows:
• To control heart rate: Digoxin, ~-blocker or
• P wave: Saw-toothed appearance (normal P-
verapamil, etc. may be used.
wave is replaced by flutter or F-wave).
• If no response and patient has troublesome
• R-R interval: Irregular.
• Atrial rate: 300 beats/min, ventricular rate is symptoms: DC cardioversion or atrial overdrive
variable (2:1,3:1, etc.). pacing may be done.

Q: What is your diagnosis? • If still no response: Radiofrequency catheter


A: Atrial flutter with variable block. ablation.

- -- -
ECG 03: Ventricular Tachycardia

VentritUlaf tachycardia
15. ECG_

Q: Write down three important abnormal findings in • Supraventricular tachycardia with right bundle
this ECG. branch block.
A: As follows: • Supraventricular tachycardia with WPW
• P-wave: Absent. syndrome.
• QRS complex: Broad >0.14 s (abnormal or Q: Mention five causes.
bizarre pattern). A: As follows:
• Ventricular rate: 130 beats/min.
• Acute myocardial infarction.
Q: What is your diagnosis? • Myocarditis.
A: Ventricular tachycardia. • Cardiomyopathy.
Q: Mention two differential diagnoses.
• Ventricular aneurysm.
A: As follows: • Electrolyte imbalance (mainly hypokalaemia and
hypomagnesaemia) .

---

ECG 04: Acute Myocardial Infarction

Fig. A: Acute inferior myocardial infarction

Q: Write down two important abnormal findings in • ST depression with inverted T-wave in I, AVI
the ECG shown in Figure A. and V2.
A: As follows:
Q: What is your diagnosis?
• ST elevation in Il, III and AVf. A: Acute inferior myocardial infarction.

aVR Vl V4

II aVL V2

Fig. B: Acute atitetoseptal myocardial infarction


_ SHORT CASES IN CLINICAL MEDICINE

Q: Write down two important abnormal findings in 1. Early complications:


the ECG shown in Figure B. • Arrhythmia: Ventricular ectopics (more
A: As follows: common), ventricular fibrillation, ventricular
• ST elevation in VI, V2 and V3. tachycardia, sinus bradycardia [common in
• Pathological Q-waves in VI and V2. inferior myocardial infarction (Ml)], sinus
tachycardia, atrial fibrillation, heart block.
Q: What is your diagnosis? • Cardiogenic shock.
A: Acute anteroseptal myocardial infarction. • Cardiac failure [left ventricular failure (LVF) ,
Q: Mention two investigations. biventricular failure].
A: As follows: • Acute pericarditis (common in 2nd or 3rd day).
• Thromboembolism (systemic and pulmonary).
• Serum troponin 1. • Rupture of the papillary muscle or chordae
• Serum CK-MB. tendineae resulting in mitral regurgitation.
Q: Write down immediate management plan. • Rupture of interventricular septum, causing
A: As follows: VSD.
• Rupture of the ventricular wall leading to
• Complete bed rest. cardiac tamponade.
• Oxygen inhalation.
2. Late complications:
• Tab. Chewable aspirin 300 mg orally.
• To relieve pain: Morphine with antiemetic • Ventricular aneurysm (10%).
(cyclizine or metoclopramide). • Postmyocardial infarction syndrome
(Dressler syndrome).
• Thrombolysis (by streptokinase). If possible,
primary coronary intervention. • Frozen shoulder.
• Postinfarct angina (may occur in up to 50%
Q: What are the complications? of patients).
A: As follows:

ECG 05: Old Myocardial Infarction


.,. c •

"

II
III "t

Fig. A: Old anterior myocardial infarction

Q: Write down two important abnormal findings in Q: What is your diagnosis?


the ECG shown in Figure A. A: Old anterior myocardial infarction.
A: As follows:
• Pathological Q-waves in CI, C2 and C3.
• T inversion in C4, CS and CG.
lS· ECG_

Fig. B: Old inferior myocardial infarction

Q: What is the abnormal finding in the ECG shown in Q: What is your diagnosis?
Figure B? A: Old inferior myocardial infarction.
A: Pathological Q-wave in It III and AVf.

ECG 06: First-Degree AV Block

First-degree AVblock

Q: What is the abnormal finding in this ECG? • Digoxin toxicity.


A: P-Rinterval is prolonged >0.22 s (normal 0.12-0.20 s).
• Acute myocardial infarction (common in inferior
Q: What is your diagnosis? myocardial infarction).
A: First-degree AVblock.
• Acute rheumatic carditis.
Q: Mention four causes?
• Hyperkalaemia.
A: As follows:

ECG 07: Second-Degree AV Block (Mobitz Type I)

Second-degree AVblock (Mobitz type J)


__ SHORT CASES IN CLINICAL MEDICINE

Q: Write down two important abnormal findings in Q: Mention three causes?


this ECC. A: As follows:
A: As follows: • Physiological: Athlete, during rest, sleep (due to
• Progressive lengthening of P-R interval followed increased vagal tone).
by absent QRS complex (one P is not followed by • Drugs: Digoxin toxicity.
a QRS complex). • Acute myocardial infarction (commonly inferior
• R-R interval: Irregular (progressive shortening of myocardial infarction) .
R-R interval until block occurs). Q: Where is the lesion?
A: The block is in the higher area of AV node (proxi-
Q: What is your diagnosis? mal to bundle of His).
A: Second-degree AV block, Mobitz type I (Wenck- Q: What is the prognosis?
ebach phenomenon). A: Cood prognosis.

ECG 08: Second-Degree AV Block (MobitzType II)

Second-degree AVblock (Mobitz type II)

Q: Write down three important findings in this ECC. Q: Mention the most likely cause.
A: As follows: A: Common in acute anterior myocardial infarction .

• P-Rinterval is constant (also P-P interval constant). Q: How to treat?


• QRS complex: Wide. A: As follows:
• Alternate P-wave is conducted. 1. When associated with acute inferior myocardial
Q: What is your diagnosis? infarction:
A: Mobitz type II second-degree AV block with 2:1 • If asymptomatic-close monitoring and
conduction. follow-up.
Q: What are the complications? • If symptomatic-Inj. atropine O.G mg intra-
A: As follows: venous (IV). If no response, temporary
pacemaker. Majority will resolve in 7-10 days.
• Complete heart block.
2. If associated with acute anterior myocardial
• Stokes-Adams syndrome.
infarction-temporary pacing followed by per-
• Heart failure.
manent pacemaker (because complete heart
Q: Where is the lesion? block may develop).
A: Disease of His-Purkinje system.

ECG09: Complete Heart Block

Complete heart block


Q: Write down three important abnormal findings in • Atrial rate is GO/min, P-P interval is constant.
this ECC. • Ventricular rate is 35/min.
A: As follows: • There is no relationship between P-wave and
QRS complex
15· ECG_

Q: What is your diagnosis? • Pulse: Bradycardia «40/min), high volume, does


A: Complete heart block. not increase by exercise.
• Cannon waves (large a-wave) in neck vein.
Q: Write down three findings you expect 111 CVS • Variable intensity offirst heart sound.
examination. Q: What is the management?
A: As follows: A: Permanent pacemaker.

ECG 10: Left Bundle Branch Block

Left bundle branch block


Q: Write down two important abnormal findings in Q: What finding might you get on auscultation?
this ECG. A: Reverse splitting of second heart sound.
A: As follows: Q: Mention four causes.
A: As follows:
• Wide QRS complex in all leads.
• Severe coronary artery disease.
• RSR' pattern in I, AVI, AVf, VS and V6.
• Acute myocardial infarction.
Q: What is your diagnosis? • Cardiomyopathy.
A: Left bundle branch block. • Aortic valve disease (stenosis or regurgitation).

ECG 11: Right Bundle Branch Block

Right bundle branch block

Q: Write down two important abnormal findings in Q: Mention three causes.


this ECG. A: As follows:
A: As follows:
• Normal variant.
• Wide QRS complex in all leads. • Coronary artery disease (acute myocardial
• RSR' pattern in VI, V2 and V3. infarction).
Q: What is your diagnosis? • Atrial septal defect.
A: Right bundle branch block. • Cardiomyopathy.
SHORT CASES IN CLINICAL MEDICINE

ECG 12: Left Ventricular Hypertrophy

o
IU
W

Fig. A: Left ventricular hypertrophy

Q: Write down two abnormal findings in the ECG Q: Mention one investigation to confirm the diagnosis.
shown in Figure A. A: Echocardiography (M-mode).
A: As follows:
Q: Mention five causes.
• S-wave in VI + R-wave in V6= 39 mm (criteria: A: As follows:
S-wave in Vl + R-wave in V6> 35 mm).
• Left-axis deviation. • Systemic hypertension.
Q: What is your diagnosis? • Aortic stenosis.
A: Left ventricular hypertrophy. • Coarctation of aorta.
Q: Mention one finding in examination of precordium. • Hypertrophic cardiomyopathy.
A: Apex beat is heaving in nature. • Ventricular septal defect.

U~~r L,--' 1~~J-l,·~~~


;'------vf""'- t-~--i"V"'--!n...
aVR L----'7'\L- aVF

Fig. B: Left ventricular hypertrophy with strain


15 • ECG ~

Q: Write down two abnormal findings in the ECG Q: What are the differential diagnoses?
shown in Figure B. A: As follows:
A: As follows:
• Hypertrophic cardiomyopathy.
• S-wave in VI + R-wave in V6= 51 mm (criteria: • Subendocardial myocardial infarction.
S-wave in Vl + R-wave in V6> 35 mm).
Q: Mention one investigation to confirm the diagnosis.
• ST-wave depression and T-wave inversion in Ll ,
A: Echocardiography (2D or M-mode).
AVI,V4-V6.
Q: What is your diagnosis?
A: Left ventricular hypertrophy with strain.

ECG 13: Right Ventricular Hypertrophy

Right ventricular hypertrophy with strain

Q: Write down two abnormal findings in this ECG. Q: Mention one investigation to confirm the
A: As follows: diagnosis.
A: Echocardiogram (2D and M-mode).
• R-wave in V1 = 10 mm (criteria: tall R-wave in
Vl > 7 mm).
Q: Mention five causes.
• ST-wave depression and T-wave inversion (in VI
A: As follows:
and V2).
• Chronic cor pulmonale.
Q: What is your diagnosis? • Mitral stenosis with pulmonary hypertension.
A: Right ventricular hypertrophy with strain. • Primary pulmonary hypertension.
• Pulmonary stenosis.
Q: Mention four clinical findings.
• Eisenmenger syndrome.
A: As follows:
• Fallot tetralogy.
• Palpable P2.
• Left parasternal heave.
• Epigastric pulsation.
• Loud P2 on auscultation.
_ SHORT CASES IN CLINICAL MEDICINE

ECG 14: P Mitrale

P mitrale

Q: Write down three abnormal findings in this EeG. Q: What does it indicate?
A: As follows: A: It indicates left atrial hypertrophy or enlargement.
• P-wave is wide (> 0.12 s) in LII and III.
Q: Mention two causes of such EeG finding.
• P-wave is notched (or bifid) in LII (called P
A: As follows:
mitrale).
• P-wave in VI is biphasic with prominent deep • Mitral stenosis (commonest).
negative deflection (c- l-rnm depth) and small • Mitral regurgitation.
initial positive deflection.
Q: What is your diagnosis?
A: P mitrale.
15· ECG_

ECG 15: P Pulmonale

!l
M
o
o

Ppulmonale

Q: What is the abnormal finding in this ECG? Q: Mention five causes of such ECG finding.
A: P-wave is tall (>2.5 mm) in Ll, II, III (P pulmonale). A: As follows:

Q: What is your diagnosis? • COPDwith chronic cor pulmonale (commonest).


A: P pulmonale. • Atrial septal defect (ASD).
• Tricuspid regurgitation or stenosis.
Q: What does it indicate?
• Pulmonary stenosis.
A: It indicates right atrial hypertrophy or enlargement.
• Pulmonary hypertension (due to any cause).

ECG 16: Pulsus Bigeminy

Pulsus bigeminy

Q: What is the abnormal finding in this ECG? • Digoxin toxicity.


A: Every normal beat is followed by a ventricular
• Myocarditis.
ectopic beat.
• Cardiomyopathy.
Q: What is your diagnosis?
A: Ventricular bigeminy. • After acute myocardial infarction.
• Electrolyte imbalance (hypokalaemia).
Q: Mention five causes.
A: As follows: • Hypoxaemia.
Q: How to treat? • Treatment of primary cause or any organic heart
A: As follows: disease.
• Any offending drug should be stopped. • If asymptomatic: No other treatment.
• Correctionofelectrolytes,especiallyhypokalaemia, • If symptomatic: ~-Blocker (antiarrhythmic drugs
hyperkalaernia and hypomagnesaemia. should be avoided, may worsen the prognosis).

ECG 17: Acute Pericarditis

Acute pericarditis

Q: Mention the abnormal finding. • Malignancy (from carcinoma of bronchus, breast,


A: ST is elevated with upward concavity in LII, AVf,V2, lymphoma, leukaemia).
V3, V4, VS and V6. • Collagen disease [systemic lupus erythematosus
(SLE), scleroderma].
Q: What is your diagnosis?
A: Acute pericarditis. Q: How to treat?
A: As follows:
Q: Mention one clinical finding.
• To relieve pain-NSAID (indomethacin or
A: Pericardial rub.
ibuprofen or aspirin).
• In severe or recurrent case-corticosteroid should
Q: Mention five causes. be given.
A: As follows: • If no response to steroid-azathioprine or
• Following acute myocardial infarction (usually colchicine may be given.
in 2nd or 3rd day). • If recurrence with no response to medical
• Infective: Viral (coxsackie B, echovirus) common treatment, pericardiotomy may be done.
cause. Others are bacterial (Staphylococcus aureus, • Treatment of primary cause-antibiotic, if
Haemophilus inj1uenzae), tuberculous pericarditis bacterial infection. Anti-Koch, if tuberculosis is
and fungal (histoplasmosis, coccidioidomycosis). suspected.
• Acute rheumatic fever.
• Acute renal failure.
15. ECG_

ECG 18: Pacemaker

Ventricular pacemaker

Q: Mention two abnormal findings. Q: Mention two absolute indications.


A: As follows: A: As follows:
• There is a spike followed by QRS complex. • Sick sinus syndrome.
• QRS complex is wide (looks like LBBB).
• Stokes-Adams syndrome.
Q: What is your diagnosis?
A: Ventricular pacemaker.

ECG 19: Sinus Tachycardia

Sinus tachycardia

Q: Write down three important findings in this ECC. • Fever.


A: As follows: • Thyrotoxicosis.
• Shock (except vasovagal attack in which
• Heart rate: liO/min.
bradycardia is present).
• Rhythm: Normal.
• Heart failure.
• P-wave, QRS complex and T-wave: Normal.
Q: What is your diagnosis? N.B. Other causes are:
A: Sinus tachycardia. • Chronic constrictive pericarditis.
• Acute anterior myocardial infarction
Q: Write down five important causes. (bradycardia is common in acute inferior
A: As follows: myocardial infarction).
• Physiological: Anxiety, emotion, exercise, pain, • Drugs (salbutamol. atropine, adrenaline,
pregnancy. isoprenaline, ephedrine, propantheline,
• Anaemia. thyroxine).
_ SHORT CASES IN CLINICAL MEDICINE

- - - ------

ECG 20: Sinus Bradycardia

Sinus bradycardia

Q: Wtite down three important findings in this EGG. • Raised intracranial pressure (due to inhibitory
A: As follows: effect on sympathetic outflow).
• Drugs (digoxin, ~-blockers, veraparnil).
• Heart rate: SO/min.
• Rhyth m: Regular. N.B. Other causes are:
• P-wave, QRS complex and T-wave: Normal. • Acute inferior myocardial infarction.
• Obstructive jaundice (due to deposition of
Q: What is your diagnosis?
bilirubin in conducting system).
A: Sinus Bradycardia.
• Electrolyte imbalance (hypokalaemia).
Q: Write down five important causes. • Neurally mediated syndromes due to a reflex
A: As follows: (Bezold-Iarisch), which causes bradycardia
and reflex peripheral vasodilatation.
• Physiological: In athletes, during sleep.
These are-carotid sinus syndrome,
• Hypothyroidism.
neurocardiogenic (vasovagal) syncope
• Hypothermia. (syndrome), which presents as syncope or
presyncope.

--- - --

ECG 21: Supraventricular Tachycardia

Supraventricular tachycardia
1S· ECG_

Q: Write down four important findings in this EGG, 1. Rest and reassurance.
A: As follows: 2. Carotid sinus massage or Valsalva manoeuvre. It
acts by increasing the vagal tone.
• Heart rate: 140/min.
3. If no response:
• Rhythm: Regular.
• P-wave: Absent. • IV adenosine-3 mg over 2 s. If no response in
• QRS complex: Narrow. 1-2 min, then 6 mg TV. If still no response
in 1-2 min, then 12 mg.
Q: What is your diagnosis?
• Or IV veraparnil 10 mg slowly over 5-10 min
A: Supraventricular tachycardia.
(verapamil should be avoided if QRS complex
Q: Write down five important causes. > 0.12 s or history ofWPW syndrome or if the
A: As follows: patient is on ~-blocker).
4. Other drugs: ~-Blocker, disopyramide or digoxin
• Physiological: Anxiety, tea, coffee, alcohol.
may be used.
• Thyrotoxicosis.
5. If the patient is haemodynamicaJly unstable
• Ischaemic heart disease.
(hypotension, pulmonary oedema), then direct
• WPW syndrome.
current (DC) shock should be given.
• Digitalis toxicity.
6. If the attack is frequent or disabling: Prophy-
Q: What is the complication? lactic oral therapy with ~-blocker, verapamil,
A: Heart failure may occur due to reduction of stroke disopyramide or digoxin may be given.
volume (rapid heart rate causes short diastolic fill- 7. In WPW syndrome: Transvenous radiofrequency
ing time with low diastolic filling). catheter ablation is the treatment of choice.
Q: Ilow to treat? 8. If there is no response: Antitachycardial pacing
A: As follows: is done (overdrive atrial pacing).

- - -

ECG 22: Ventricular Premature Beat

l.0t10 II

Ve'rltl'icular premature beat

Q: Write down three important findings in this ECG. • Electrolyte imbalance (especially hypokalaemia).
A: There are two ventricular premature beats with the • Digoxin toxicity.
following characteristics: • Mitral valve prolapse.

• P: Absent. Q: What are the types?


• QRS complex: Wide >0.12 s (3 small squares). A: As follows:
• T: Opposite to major deflection. • Unifocal: Similar configuration of ectopics in aU leads,
Q: What is your diagnosis? originates from a single ectopic ventricular focus.
A: Ventricular premature (ectopic) beats. • Multifocal: Variable configuration of ectopics in
same lead, ectopics originate from different foci
Q: Write down five important causes.
of ventricle.
A: As follows:
• Interpolated ven.tricular ectopics: It means when
• Normally in young adults, also in anxiety, excess ventricular ectopics occur between two normal
caffeine, alcohol. sinus beat without compensatory pause.
• Acute myocardial infarction. Other Types
• Myocarditis. • Couplet: Two ventricular ectopics in a row,
• Cardiomyopathy. multifocal.
~ SHORT CASES IN CLINICAL MEDICINE

• Triplet: Three ventricular ectopics in a row (runs • Grouped ventricular ectopics:Two to five consecutive
of ectopic, three or more ventricular ectopics in ventricular ectopics.
a row, may be taken as ventricular tachycardia). Q: How to treat?
• Ventricular bigeminy: Every one normal beat A: As follows:
followed by ventricular ectopic.
• In the absence of any heart disease and in
• Ventricular trigeminy: Every two normal beats
asymptomatic case: No treatment is necessary.
o followed by ventricular ectopic.
u • Ventricular quadrigeminy: Every three normal beats
~-Blocker may be used.
w • With organic heart disease: Treatment of primary
followed by ventricular ectopic.
cause.

ECG 23: Ventricular Fibrillation

Ventricular fibrillation

Q: What is the finding in this ECG. • Patient: Unconscious.


A: QRS complex: Chaotic, wide, bizarre, irregular. • Pupil: Dilated, less or no reaction to light.
• Heart sounds: Absent.
Q: What is your diagnosis?
A: Ventricular fibrillation. Q: How to treat?
A: As follows:
Q: Write down five important causes.
A: As follows: • Immediate defibrillation: 200 Joules. If no
response, another shock with 200 Joules is given.
• Acute myocardial infarction.
If still no response, another shock with 360
• Electrolyte imbalance (hypokalaemia,
Joules is given.
hypomagnesaemia).
• If three shocks unsuccessful: Adrenaline is given
• Electrocution.
IV, followed by cardiopulmonary resuscitation.
• Drowning.
• If defibrillator is not available: Cardiopulmonary
• Drugoverdose (digitalis, adrenaline, isoprenaline).
resuscitation should be given.
Q: Mention six clinical signs in this patient. • The patient who survives from ventricular
A: As follows: fibrillation (VF) in the absence of identifiable
• Pulse: Absent. cause is at high risk of sudden death. This
• BP: Not recordable. case is treated with implantable cardioverter
• Respiration: Ceases or absent. defibrillator.

ECG 24: Hyperkalaemia

Hyperkalaemia
___________ 1_5_- ECG ~

Q: Write down two important findings in this ECG. 4. Drugs: Potassium sparing diuretics
A: As follows: (spironolactone, amiloride, triarnterine), ACE
inhibitor, NSAlD, cyclosporine.
• T-wave: Tall, peaked and tented.
• P-wave: Wide and small. Q: Write four clinical features.
A: As follows:
N.B. Other probable findings are (not seen in this
ECG): • Muscular weakness; it may be severe causing M
flaccid paralysis, loss of tendon jerk.
n
• P-R interval: Prolonged. 4)
• QRS complex: Wide, slurred and bizarre. • Paralytic ileus (abdomen may be distended).
• Tingling around the lip or finger.
Q: What is your diagnosis? • Sudden death due to cardiac arrest or arrhythmia.
A: Hyperkalaemia. Q: How to treat?
A: As follows:
Q: Write down four important causes.
A: As follows: • Withdrawal of potassium, potassium-containing
food and offending drug.
1. High potassium intake.
• Injection 10% calcium gluconate 10-20 cc IV
2. Renal diseases: slowly over 10 min.
• Acute and chronic renal failure. • Injection 50 ml of 50% glucose IV + Inj. insulin
• Impaired tubular secretion of K« (renal lupus, 10 units.
amyloidosis, transplanted kidney). • Correction of acidosis: By TV sodibicarb (1.26%),
3. Endocrine diseases: 500 m16-8 hourly (until serum HC03 is normal).
• Treatment of primary causes.
• Addison disease.
• In some cases, exchange resins (calcium resoniurn
• Diabetic ketoacidosis.
15-30 gm orally).
• Primary hypoaldosteronism.
• If all fail: Haemodialysis or peritoneal dialysis.
:'CR-APTER
, ~ - _.
. -
16·

PICTURES
"Diagnosis is not the end, but the beginning ofpractice"
- Martin H. Fischer

Picture 01: Infected Scabies

Q: Write down the finding shown in this picture. • Dermatitis herpetiformis.


A: Multiple vesicular and pustular lesions in inter- • Pediculosis corporis.
digital and dorsal surface of both hands.
Q: Write down two important complications of this
Q: What is your diagnosis?
clinical condition.
A: Infected scabies. A: Secondary bacterial infection, poststreptococcal
Q: What is the causative organism? glomerulonephritis.
A: Sarcoptes scabiei.
Q: Write down the treatment.
Q: What are the common sites?
A: As follows:
A: As follows:
l. General measures: Control of infection by anti-
• lnterdigital space.
biotics; for itching-antihistamine, washing of
• Anticubital fossa.
cloths and bed sheet, simultaneous treatment of
• Axilla. other family members.
• Nipple.
• Umbilicus. 2. Specific treatment:
• Scrotum. • Local: 5% permethrin cream. Other drugs
are 1% y-benzene hexachloride, 25% benzyl
Q: What are the differential diagnoses?
benzoate, 10% precipitated sulfur in white
A: As follows: petroleum.
• Popular urticarial. • Systemic: Ivermectin orally when local
• Atopic dermatitis. treatment fails.
16 • PICTURES _

Picture 02: Purpura

Q: What are the findings seen in the photograph? Q: Mention six important investigations for this patient.
A: Multiple purpuric spot of variable size and shape in A: As follows:
both lower limbs.
• Complete blood count (CBC) with peripheral
Q: Mention six differential diagnoses.
blood film (PBF).
A: As follows: • Antidengue antibody.
• Blood for culture and sensitivity (CIS).
• Drug rash. • Coagulation profile: Prothrombin time, activated
• Idiopathic thrombocytopenic purpura (ITP). partial thromboplastin time (APTf).
• Henoch-Schonlein purpura. • Screening for disseminated intravascular
• Dengue haemorrhagic fever. coagulation (DIC): Fibrin degradation products
• Aplastic anaemia. (FOP), D-dimer.
• Meningococcal septicaemia. • Bone marrow study.

Picture 03: Rheumatoid Arthritis


Q: What are the abnormalities in this photograph?
A: As follows:

• Swan-neck deformity of fingers.


• Z-deformity of thumb.
• Ulnar deviation of hands.
• Flexion deformity of metacarpophalangeal joints.

Q: What is the diagnosis?


A: Rheumatiod arthritis.

Q: Mention five important investigations.


A: As follows:

• CBC, erythrocyte sedimentation rate (ESR).


• C-reactive protein (CRP).
__ SHORT CASES IN CLINICAL MEDICINE

• Rheumatoid arthritis (RA) and Rose-Waaler A: As follows:


(RW) tests.
1. To relieve pain: NSAID.
• Anti-cyclic citrullinated peptide (anti-CCP)
2. Suppression of activity and progression of
antibody.
disease: DMRD.
• X-ray of both hands.
3. Physiotherapy.
Q: Mention the modalities of treatment. 4. Orthopaedic measures.

Picture 04: Arthritis Mutilans


Q: What are the findings?
A: Complete disorganization and deformity involving
all the joints of both hands.

Q: What is the likely diagnosis?


A: Arthritis mutilans in rheumatoid arthritis.

Q: What specific measures can be done in such a case?


A: Orthopaedic measures such as reconstructive sur-
gery along with treatment of rheumatoid arthritis.

Picture 05: Graves Disease


Q: What other finding you should look for in this case?
A: Dermopathy.

Q: Mention three important questions you will ask the


patient.
A: As follows:

• Preference to hot or cold.


• Increased appetite and weight loss.
• Excessive sweating.
Q: Write down three important clinical findings.
A: As follows:

• Palm: Warm and sweaty.


• Tremor of outstretched hand.
• Tachycardia.
Q: Wrile down the most important investigation to
diagnose the case.
A: As follows:

• Free triiodothyronine (FT3), free thyroxine (FT4),


Q: Write down four important findings in this picture.
thyroid stimulating hormone (TSH).
A: As follows:
• Radioiodine uptake test.
• Bilateral exophthalmos. • Thyroid scanning.
• Diffuse goitre. • UItrasonogram of neck.
• Anxious look. • TSH receptor antibody.
• Wasting. Q: What is the natural history of this disease?
Q: What is your diagnosis? A: The patient may be hyperthyroid, followed by
A: Graves disease. euthyroid and then followed by hypothyroid.
16 • PICTURES _

Picture A

Q: Write down the important findings in Picture A. Q: What are the differential diagnoses?
A: Multiple, well-circumscribed erythematous plaques A: As follows:
with silvery white scales on both lower limbs.
• Dermatomyositis.
Q: What is your diagnosis? • Lichen planus.
A: Psoriasis. • Seborrhoeic dermatitis.
• Secondary syphilis.

Q: Mention two important physical signs that should


be seen in this patient.
A: Auspitz sign and Koebner phenomenon.

Q: Mention one investigation to confirm your diagnosis.


A: Skin biopsy.

Q: How to treat?
A: As follows:

1. General measures: Explanation and reassurance,


avoid trauma.
PictureS 2. Specific therapy:
• Local therapy: Crude tar 3-5%, dithranol,
Q: Write down the important findings in Picture B.
calcipotriol, ultraviolet radiation (UVR).
A: Multiple, well-circumscribed erythematous plaques
• Systemic therapy: PUVA (psoralen plus UV
with silvery white scales on the back.
ray), retinoid, methotrexate (MIX).
Q: What is your diagnosis? 3. Other therapy: anti-TNF-u (infliximab,
A: Psoriasis. etanercept].
~ SHORT CASES IN CLINICAL MEDICINE

Picture 07: Systemic Lupus ErYthematosus


A: As follows:

• Dermatomyositis.
• Drug rash.
• Post-kala-azar dermal leishmaniasis (PKDL).
• Lepromatous leprosy.

Q: Mention two investigations to confirm your


diagnosis.
A: As follows:

• Antinuclear antibody (ANA).


• Anti-ds-DNA (Anti-double-stranded
deoxyribonucleic acid).

Q: Name five drugs that may cause this disease.


Q: Write down the important findings in this picture. A: As follows:
A: Multiple skin rashes on the face along the butterfly
• Hydralazine.
distribution.
• Procainamide.
• Anticonvulsant: Carbarnazine, phenytoin.
Q: What is your diagnosis?
A: Systemic lupus erythematosus (SLE). • Penicillamine.
• Oral contraceptive pill.
Q: Mention four differential diagnoses • Isoniazid (INH).

Picture 08: Erythema Multiforme

Picture A Picture B
Q: What is the pathognomonic lesion in this disease?
Q: Write down the important findings in these A: Target lesion.
pictures.
Q: What are the differential diagnoses?
A: Multiple erythematous vesicular lesions on both
A: As follows:
forearms and erythematous lesions involving both
palms. • Drug reaction.
• SLE.
Q: What is your diagnosis? • Dermatitis herpetiformis.
A: Erythema multiforme. • Dermatomyositis.
16 • PICTURES _

Q: Mention two causes. Q: What investigations should be done?


A: As follows: A: As follows:
• Infection: Herpes simplex type 1, Mycoplasma • CBC, ESR.
pneumoniae. • Antibody to herpes simplex virus and
• Drugs: Sulfonamide, carbamazepine. Mycoplasma pneumoniae.
thioacetazone. • Chest X-ray.

Picture 09: Leprosy


Q: What is your diagnosis?
A: Lepromatous leprosy.

Q: What is the causative organism?


A: Mycobacterium lepiae.

Q: Mention four differential diagnoses.


A: As follows:
• Post-kala-azar dermal leishmaniasis (PKDL).
• Sarcoidosis.
• Drug rash.
• Dermatomyositis.
Q: What investigations should be done to confirm the
diagnosis?
A: Skin slit smear for acid fast bacilli (AFB) staining.
Q: Write down the important findings in this picture.
A: As follows: Q: What drugs are used to treat the case?
A: As follows:
• Multiple erythematous nodules of variable size
and shape on forehead, ear and face. • Dapsone.
• Multiple pigmented and depigmented areas over • Rifampicin.
the same area. • Clofazimine.

Picture 10: Stevens-Johnson Syndrome


Q: Write down the important findings in Picture A.
A: Multiple erythematous lesions and ulcerations
involving skin and mucous membrane of face, lips
and oral cavity.

Q: What is your diagnosis'?


A: Stevens-Johnson syndrome.

Picture A
_ SHORT CASES IN CLINICAL MEDICINE

A: As follows:

• Desquamation and ulceration of skin of scrotum.


• Multiple, sharply demarcated pigmented lesions
over lower abdomen and upper thighs.

Q: What is your diagnosis?


A: Stevens-Johnson syndrome.

Q: Mention three causes.


A: As follows:

• Drug reaction: Sulfonamide, carbarnazepine,


thiacetazone.
• Infection by herpes simplex virus, Mycoplasma
Picture B
pneumoniae.
Q: Write down the important findings in Picture B. • Idiopathic.

Picture 11: Ring Worm


Q: What is your diagnosis?
A: Ring worm infection (Tinea corporis).

Q: How to confirm?
A: Microscopic examination of skin scrapping in KOH
preparation.

Q: What are the causative organisms?


A: As follows:

• Trichophyton.
• Epidermophyton.
• Mycosporurn.
Q: How to treat?
A: As follows:
Q: Write down the important findings in this picture.
A: Multiple circular lesions on both antecubital fossa, • Maintain hygiene.
showing central healing with peripherally active • Local antifungal like rniconazole.
lesion. •. Systemic antifungal like fluconazole.

Picture 12: Subconjuctival Haemorrhage


Q: Write down the important findings in this picture.
A: Subconjuctival haemorrhage in both eyes.
Q: Mention three causes?
A: As follows:

• Trauma.
• Dengue fever.
• Bleeding disorder.
Q: How to treat?
A: As follows:

• Local: Antibiotic eye drop, analgesics for pain.


• Treatment of the cause.
16 • PICTURES ~

Picture 13: Bell Palsy

Picture B

Q: Write down the important findings in Picture B.


Picture A
A: As follows:

• Bell's phenomenon on closing the right eye.


Q: Write down the important findings • Nasolabial fold less prominent on right side.
in Picture A. Q: What is your diagnosis?
A: As follows: A: Right-sided Bell palsy.

• Difficulty in closing the right eye (mention if Bell Q: What are the causes if it was bilateral?
phenomenon is also seen). A: As follows:

• Mouth is deviated to the left on showing the • Cuillain-Barre syndrome (CBS).


teeth. • Sarcoidosis.
• Lyme disease.
• No wrinkling of the forehead on right side.
Q: flow to treat?
• Facial asymmetry. A: As follows:
• Nasolabial fold less prominent on right side. • Prednisolone 40-60 mg daily plus acyclovir for
7 days.
Q: What is your diagnosis?
• Artificial tear.
A: Right-sided Bell palsy. • Physiotherapy.

Picture 14: Ramsay Hunt Syndrome


Q: What is the finding in this picture?
A: Vesicular rash on external ear with pus in external
auditory meatus.

Q: This patient also has light-sided facial palsy. What is


the diagnosis?
A: Ramsay Hunt syndrome.

Q: What is the cause?


A: Varicella-zoster virus.
_ SHORT CASES IN CLINICAL MEDICINE

Picture 15: Goitrous Myxoedema


Q: Mention two causes.
A: As follows:

• Hashimoto thyroiditis.
• Iodine-deficiency goitre.

Q: Write three clinical features.


A: As follows:

• Intolerance to cold.
• Weight gain.
• Increased sleepiness.

Q: Write one important bedside physical sign.


A: Slow relaxation of ankle jerk.

Q: If the patient complains of tingling and numbness


of fingers, what is the likely diagnosis?
A: Carpal tunnel syndrome.

Q: Mention three investigations to confirm your diag-


nosis.
A: As follows:
Q: What are the abnormal findings in th is picture?
A: As follows: • FT3, FT4, TSH.

• Diffuse goitre. • Antithyroglobulin and antiperoxidase antibody.

• Puffy face. • Ultrasonogram of neck.

Q: What is the likely diagnosis? Q: How to treat?


A: Goitrous myxoedema. A: Lifelong thyroxine therapy.

Picture 16: Gonorrhoea


Q: Write down the important finding in this picture.
A: Purulent urethral discharge.

Q: What is your diagnosis?


A: Gonorrhoea.

Q: What investigations should be done?


A: Urethral swab for Gram stain and CIS.

Q: How [0 treat'?
A: Antibiotic like ceftriaxone, ciprofloxacin. etc.
16 • PICTURES __

Picture 17: Hirsutism

Picture B

Q: Mention three findings in Picture B.


A: As follows:
Picture A
• Increased facial hair in a young female patient.
Q: What is the finding in Picture A?
• Puffy face with flushing.
A: Increased facial hair in an elderly female patient.
• Facial acne.
Q: What is the diagnosis? Q: What is the likely diagnosis in this patient?
A: Hirsutism. A: Cushing syndrome.
Q: Mention three causes? Q: Mention three differential diagnosis:
A: As follows: A: As follows:

• Androgen-secreting ovarian tumour. • PCOS.


• Adrenal carcinoma. • Androgen-secreting ovarian tumour.
• Idiopathic. • Late-onset congenital adrenal hyperplasia.

Picture 18: Osteoarthrosis


Q: What are the findings in this picture?
A: Nodular swelling on the distal interphalangeal joint
of right little and ring fingers.

Q: What is the likely diagnosis?


A: Osteoarthrosis.

Q: What is the lesion called?


A: Heberden node.
_ SHORT CASES IN CLINICAL MEDICINE

Picture 19: Peripheral Vascular Disease


Q: What is the finding in this picture?
A: Blackening and necrosis of index fingers of both
hands and middle and little fingers of left hand.

Q: What is the diagnosis?


A: Peripheral vascular disease, most likely Buerger
disease.

Q: Mention three differential diagnoses.


A: As follows:

• Atherosclerosis.
• Raynaud disease.
• Systemic sclerosis.

Picture 20: Hypopituitarism

Q: What are the findings in this picture of a 45-year-old Q: What is the likely diagnosis?
man? A: Hypopituitarism.
A: As follows:
Q: Mention one investigation.
• Short stature. A: Magnetic resonance imaging (MRI) of the pituitary
• Bilateral gynaecomastia. fossa.
16 • PICTURES _

Picture 21: Herpes Zoster

Picture B
Picture A
Q: What is the finding in Picture B?
Q: What is the finding in Picture A? A: Ulcerated lesions over right-half of forehead, around
A: Multiple vesicular lesions over left shoulder, left the right eye and over the right maxillary region.
upper chest and upper part of left arm along left C4
dermatome.
Q: What is the diagnosis?
A: Herpes zoster ophthalmicus.
Q: What is the diagnosis?
A: Herpes zoster.
Q: What is the causative organism?
A: Varicella-zoster virus.
Q: How to treat?
A: Acyclovir and local skin care (such as antiseptic
Q: How to treat?
cream, regular cleaning)
A: As follows:
Q: Mention one complication. • Acyclovir.
A: Post herpetic neuralgia. • Care of the eye.

. :. - Picture 22: Obstructive Jaundice


Q: Write two findings in this picture?
A: As follows:

• Deep jaundice.
• Cachexia.
Q: What is the likely diagnosis?
A: Obstructive jaundice, most likely due to carcinoma
of head of pancreas.
Q: Mention three investigations.
A: As follows:

• Liverfunction tests: Serum bilirubin, alanine


aminotranferase (AL1'),aspartate aminotransferase
(AST),alkaline phosphatase, prothrombin time.
• Ultrasonogram of hepatobiliary system and
pancreas.
• cr scan of abdomen.
_ SHORT CASES IN CLINICAL MEDICINE

Picture 23: Wegener Granulomatosis


A: Wegener granulomatosis.

Q: Mention three investigations.


A: As follows:

• X-ray of chest PA view.


• Cytoplasmic antineutrophil cytoplasmic
antibodies (c-ANCA).
• Biopsy from nasal crust or affected part.

Q: What are the differential diagnoses?


A: As follows:

• Sarcoidosis.
• Lymphoma.

Q: What are the findings? • Tuberculosis.


A: Swelling on the forehead, left upper eyelid and • Midline granuloma (idiopathic).
nasal bridge. • Fungal: Histoplasmosis.

Q: Ihis patient presented 'with recurrent rhinitis with • Lymphomatoid granulomatosis.


cough. What is the likely diagnosis? • Rhinoscleroma.

Picture 24: Xanthelasma

Q: What is the finding in this picture? Q: Mention three causes.


A: Xanthelasma. A: As follows:

Q: What does it indicate? • Primary biliary cirrhosis.


A: Dyslipidaemia.
• Nephrotic syndrome.
Q: What investigation should be done? • Familial hypercholesterolaemia.
A: Fasting lipid profile.
16 • PICTURES _

Picture 25: Baker Cyst

Q: What is the 1ikely diagnosis?


A: Baker cyst.

Q: Mention three causes.


A: As follows:
• Rheumatoid arthritis.
• Osteoarthrosis.
• Congenital.

Q: Mention one complication.


A: Rupture of Baker cyst.

Q: What is the finding in this picture? Q: Mention one investigation.


A: Swelling on the back of right knee. A: Ultrasonogram of the knee joint.

Picture 26: Pectus Carinatum


Q: What are the findings in this picture?
A: As follows:

• Pectus carinatum (pigeon chest).


• Kyphosis.

Q: What are the causes?


A: As follows:

• Congenital.
• Ricket.
• Marfan syndrome.
• Homocystinuria.
• Repeated respiratory infection in childhood.
• Bronchial asthma since childhood.
• Osteogenesis imperfecta.

: - - .~ --....
~....Picture 27: Corneal Arcus
Q: Write down the abnormal finding in the eyes of this
75-year-old patient.
A: Whitish opaque ring around the corneal margin.

Q: What is it called?
A: Corneal arcus.

Q: Mention two causes.


A: As follows:

• Arcus senilis.
• Dyslipidaemia.
_ SHORT CASES IN CLINICAL MEDICINE

Picture 28: Acromegaly


A: As follows:

• Enlarged skull, body and limbs.


• Hands are large, spade like.
Vl
v • Thick skin with skin tag.
Bu • Bitemporal hemianopia.
~
Q: What investigations should be done to confirm
your diagnosis?
A: As follows:

• X-ray of skull, hands.


• MRl of brain.
• Growth hormone measurement with
simultaneous oral glucose tolerance test (OGTT) .
• Insulin-like growth factor-I (IGF-l).
Q: What is the abnormal finding? What is it called?
A: Protrusion of the lower jaw. It is called Q: What arc the modalities of treatment?
prognathism.
A: As follows:

Q: What is the likely diagnosis? • Medical treatment: Octreotide,


A: Acromegaly. • Surgical: Transphenoidal removal of pituitary
Q: Mention four other clinical findings 111 this tumour.
patient. • Radiotherapy.

Picture 29: Primary Optic Atrophy


Q: What is the diagnosis?
A: Primary optic atrophy.

Q: What is the mechanism?


A: Degeneration of optic nerve fibers.

Q: Mention four causes.


A: As follows:

• Intracranial-space-occupying lesion.
• Secondary to optic neuritis.
• Glaucoma.
• Optic nerve compression by tumour or aneurysm.

Q: What are the findings?


A: As follows:

• The disc is pale with clear margin .


• There is reduction of number of capillaries in the
disc.
16 • PICTURES _

Picture 30: Papilloedema

Q: What is the finding in this picture?


A: There is blurring of the disc margin with fullness of
the optic disc.

Q: What is the diagnosis?


A: Papilloedema.

Q: Mention two causes?


A: As follows:

• Raised intracranial pressure due to space-


occupying lesion (e.g. tumour, abscess,
haematoma).
• Benign intracranial hypertension.

Picture 31: Central Retinal Vein Occlusion


Q: Mention five causes.
A: As follows:

• Hypertension.
• Atherosclerosis.
• Diabetes mellitus.
• Hyperviscosity syndrome.
• Hyperlipidaernia.
Q: Mention three serious complications.
A: As follows:

• Glaucoma (rubeotic).
• Optic atrophy.
• Permanent loss of vision.
Q: What are the findings?
Q: What investigations should be done?
A: As follows:
A: As follows:
• Multiple scattered haemorrhages over the whole • Complete blood count (CBC).
retina giving a stormy sunset appearance.
• Blood sugar.
• Veins are dilated and tortuous.
• Plasma viscosity.
• Few soft exudates and papilloedema are present.
• Plasma protein electrophoresis.
Q: What is the diagnosis? • Lipid profile.
A: Central retinal vein occlusion. • Bone marrow (to exclude multiple myeloma).
CHAPTER -------
17

INSTRUMENTS
"When you no longer know what headache, heartache, or stomachache means without cistern punctures, electro-
cardiograms and six x-ray plates, you are slipping"
- Martin H. Fischer

Instrument 01: Bone Marrow Aspiration Needle

• Macrocytic anaemia due to any cause.


• Multiple myeloma.
• Leukaemia.
• Myelofibrosis.
• Pancytopaenia due to any cause.
• Idiopathic thrombocytopaenic purpura (ITP).
• Kala-azar to see Leishman-Donovan (LD) bodies.

Q: What are the contraindications?


Q: What is this instrument? A: As follows:
A: Bone marrow aspiration needle (also called sternal • Local infection or sepsis.
puncture needle). • Bleeding disorder like haemophilia.
Q: What are the parts? • Platelet count <40,OOO/cmm.
A: Three parts: Q: What are causes of dry or blood tap?
A: As follows:
• Trocar.
• Cannula. • Faulty technique.
• Adjustable guard. • Myelosclerosis or myelofibrosis.
Q: Mention the sites of use. • Marrow hypoplasia.
A: As follows: • Sometimes in marrow hyperplasia or leukaemia,
if the marrow is heavily packed with cells.
• Manubrium sternae or body of sternum. • Tumour infiltration, e.g. lymphoma, secondary
• Posterior superior iliac crest. malignancy.
• Other sites: Upper part of the medial surface of
tibia just below the tibial tuberosity in children. Q: What are the complications?
Rarely, spinous process of lumbar vertebrae. A: As follows:

Q: How can you confirm that the needle has reached • Suction pain.
the cavity? • Vasovagal attack due to fear or pain.
A: When the needle is in the bone marrow cavity, there • Bleeding and localized haematoma.
is sudden loss of resistance and marrow material is • Injury to underlying structure due to over
seen at the tip of the trochar. penetration.
• Infection, e.g. osteomyelitis.
Q: What are the indications of bone marrow study?
Q: What is trephine biopsy?
A: As follows:
A: Trephine biopsy shows histological section that
• Aplastic anaemia. contains bony trabeculae, haemopoietic tissue, fat
• Megaloblastic anaemia. cells and blood vessels.
~ ~1~7 • INSTRUMENTS IIEIIIII
Q: What are the indications of trephine biopsy? • Myelofibrosis or myelosclerosis.
A: As follows: • If bone marrow aspiration fails to establish a
• Dry or blood tap. diagnosis.
• In aplastic anaemia: For better assessment of • Diagnosis and staging of lymphoma.
cellularity. • Secondary deposit.

Instrument 02: Lumbar Puncture Needle


• Raised intracranial pressure (clinically detected if
there is papilloedema).
• Localized infection.
• Bleeding disorder (e.g. haemophilia, Christmas
disease, etc.).
Q: What clinical examination would you do before
lumbar puncture? Why?
Q: What is this instrument?
A: Ophthalmoscopy to see papilloedema, which indi-
A: Lumbar puncture needle.
cates raised intracranial pressure. Iflumbar puncture
Q: What are its parts? is done in such case, there may be herniation of
A: As follows: cerebellar tonsil through foramen magnum, and
may compress the vital centre in medul1a oblongata
• Cannula with cap. and cause sudden death.
• Trocar.
Q: What are the complications?
Q: What are the uses?
A: As follows:
A: As follows:
• Post lumbar puncture headache.
1. Diagnostic:
• Infection (causing meningitis, arachnoiditis).
• Meningitis.
• Bleeding.
• Encephalitis.
• Herniation of cerebellar tonsil.
• Subarachnoid haemorrhage.
• Persistent cerebrospinal fluid (CSF) leaking.
• Guillain-Barre syndrome.
• Injury to local structures like intervertebral disc,
• Multiple sclerosis.
vessels, nerves, etc.
2. Therapeutic:
• Intrathecal methotrexate in acute Q: Why post lumbar puncture headache? How to
lymphoblastic leukaemia (ALL). manage?
• Spinal anaesthesia. A: Headache usually occurs if lumbar puncture eLI')
• Removal of CSF in benign intracranial is done in normal intracranial tension. It is due
hypertension, to low intracranial tension due to withdrawal of
CSF, which causes traction on the meningeal blood
Q: Mention the anatomical sites of using this instru- vessel resulting in headache.
ment. Why this site is chosen?
Treatment is as follows:
A: In the space between L3 and L4 or L4 and L5. This
site is chosen because the spinal cord is absent at • Increased fluid intake.
this space. So, there is no chance of injury. • The patient should lie flat for 8-24 h.
• Foot-end should be raised and pillow should be
N.B. The line between the highest points of both removed.
iliac crests runs through the spinous 'process • Analgesic.:
of the 4th lumbar vertebra.
Q: How much CSF is drawn during LP?
Q: What are the contraindications? A: For diagnostic purpose, 5-8 ml and for therapeutic
A: As follows: purpose, 10-20 ml is drawn.
_ SHORT CASES IN CLINICAL MEDICINE

Q: What are the causes of dry tap? Q: What are the different colours of CSF?
A: As follows: A: As follows:

• Faulty technique. • Clear: Normal, viral encephalitis.


• Spinal subarachnoid block (e.g. due to • Haemorrhagic: Due to blood (see below).
meningioma, neurofibroma, epidural abscess, • Yellow: Xanthochromia (see below).
etc. ). • Straw: Tuberculosis (cobweb is formed when
kept overnight).
Q: What should be seen in CSF?
• Turbid or cloudy: Pyogenic meningitis.
A: As follows:
Q: What are the causes of blood-stained CSF?
• Pressure.
• Physical character: Colour (dear, purulent, A: As follows:
haemorrhagic, straw). If the fluid is kept for • Trauma.
8-12 h, there may be cobweb appearance. Also • Subarachnoid haemorrhage.
xanthochromia may be seen. • Blood leakage from cerebral tumour
• Biochemistry: Protein, sugar, chloride. • Coagulation disorder (haemophilia, Christmas
• Cytology: Cell count, differential count. disease, excess use of anticoagulants, etc.).
• Microbiology: Gram stain, culture and sensitivity,
acid fast bacilli (AFB). Q: How can you differentiate traumatic bleeding from
• Serology: Viral serology, Venereal Disease subarachnoid haemorrhage?
Research Laboratory (VDRL), Cryptococcus, etc. A: Usually three samples are taken.
• Polymerase chain reaction (PCR) done in herpes
simplex virus, MycobacteJium tuberculosis, etc. Points Traumatic Subarachnoid
• Oligoclonal band on protein electrophoresis. bleeding bleeding

Q: What are the characteristics of normal CSF? Colour Initially red, but All the samples are
A: As follows: becomes faint, red uniformly red
or clear in later
• Amount: 100-150 ml. samples
• Pressure: 50-150 mm H20 on lying, 150-250
Clot Present Absent
mm H20 on sitting.
Xanthochromia Absent Present when kept
• Colour: Clear. for some time
• Protein: 20-40 mg/dl,
• Glucose: 50-80 mg/dL (2/3rd of blood glucose
level). Q: What are the causes of xanthochromia (yellow
• Chloride: 720-750 mg/dl, colour)?
• Cytology: 0-5 cell/cmm (all are lymphocytes). A: As follows:

Q: What are the causes of raised intracranial pressure? • Old subarachnoid haemorrhage.
A: As follows: • Froin syndrome.
• Deep jaundice.
• Meningitis.
• Encephalitis. Q: What are the causes of increased protein in CSF?
• Intracranial space-occupying lesion. A: As follows:
• Benign intracranial hypertension.
• Guillain-Barre syndrome.
• Intracranial haemorrhage.
• Spinal block
• Intracranial sinus thrombosis.
• Acoustic neuroma.
• Hydrocephalus.
• Froin syndrome.
• Hypertensive encephalopathy.
• Tubercular meningitis.
Q: What are the causes of decreased intracranial • Pyogenic meningitis.
pressure? • Neurosyphilis (rarely).
A: As follows: • Carcinomatosis.
• Dehydration.
N.B. Protein level is very high in the first four.
• Spinal block.
17 • INSTRUMENTS __

Q: What are the causes of raised 'Y-globulin in CSF? Q: What are the causes of raised sugar in CSF?
A: As follows: A: Hyperglycaemia (diabetes mellitus).
• Multiple sclerosis.
Q: What are the causes of increased lymphocyte count
• Neurofibromatosis. in CSF?
• Connective tissue disorder. A: As follows:

Q: What are the causes of decreased sugar in CSF? • Tubercular meningitis.


A: As follows: • Viral meningitis or encephalitis.
• Neurosyphilis.
• Tubercular meningitis.
• Pyogenic meningitis. Q: What is the cause of increased neutrophil count in
• Hypoglycaemia. CSF?
• Carcinomatous meningitis. A: Pyogenic meningitis.

Instrument 03: Liver Biopsy Needle


• Metabolic disorder:

~~==~ o Haernochrornatosis.
o Wilson disease.
• Infiltrative disease:
~I======== o Sarcoidosis.
o Lymphoma.
• Amyloidosis.
• Storage diseases (e.g. glycogen storage disease).
• Unexplained hepatomegaly.
Q: What is this instrument?
A: Vim.Silverman liver biopsy needle. Q: What are the contraindications of liver biopsy?
A: As follows:
Q: What are its parts?
• Bleeding disorder, e.g. haemophilia, Christmas
A: As follows:
disease.
• Cannula. • Hydatid cyst.
• Trocar. • Passive venous congestion of liver.
• Split needle. • Extrahepatic cholestasis or biliary obstruction.
Q: Name different types of instruments for liver • Severe jaundice.
biopsy. • Hepatic encephalopathy (the patient may
A: As follows: develop coma).

• Vim Silverman needle (commonly used). Q: What prerequisites should be taken before doing
• Menghini needle. liver biopsy?
• Trucut needle. A: As follows:

Q: What are the indications of liver biopsy? • The patient should be cooperative, consent
A: As follows: should be taken.
• To be excluded: Biliary obstruction, marked
• Chronic liver disease (CLD) (cirrhosis .of liver, ascites, severe anaemia and high bilirubin.
chronic hepatitis).
• Prothrombin time should not be >4 of control.
• Space-occupying lesions:
• Platelet count should not be <l,OO,OOO/cmm.
o Hepatic carcinoma.
o Secondary deposits in liver. • Blood grouping and cross matching.
_ SHORT CASES IN CLINICAL MEDICINE

Q: How to be sure that the needle is in the liver? • May precipitate hepatic encephalopathy in pre-
A: After introducing the needle, the patient is asked to existing liver disease.
take deep breath in and out. The needle will move • Intrahepatic arteriovenous (AV)fistula.
with respiration.
Q: What are the methods of liver biopsy?
Q: What are the causes of failure of liver biopsy? A: As follows:
A: As follows:
• Percutaneous (better ultrasonography (USG)
• Faulty technique. guided).
• Severe fibrosis of liver. • Transjugular.
Q: How to do the follow-up after biopsy? • Laparoscopic or laparotomy (if done for other
A: As follows: reason).

• The patient should be in complete bed rest for


Q: What are the indications of transjugular liver
24 h.
biopsy?
• Regular monitoring of pulse, BP.
A: It is done if there is massive ascites, coagulation
• Blood should be kept ready for transfusion.
abnormality or small, shrunken liver.
Q: What are the complications?
A: As follows: '
~.B. Remember the following:
• Bleeding. • If bilirubin is high (>3 mg%), biopsy should
• Shock. not be taken as the liver tissue does not take
• Secondary infection. stain.
• Injury to colon and other viscera. • If prothrombin time is prolonged, it can be
• Pleurisy. corrected by vitamin K, 10 mg daily for 3 days.
• Pneumothorax. • Fine-needle aspiration cytology (FNAC) under
• Biliary peritonitis. USG guidance is more preferred now-a-days.

Instrument 04: Pleural Biopsy Needle


anaesthesia, the needle is introduced after giving
a small incision through the skin at the intercos-
tals space. Stylet is removed and pleural fluid is
aspirated. Then the cutting trochar is introduced

~====::II:).. and parietal pleura is cut. The needle is then


removed, reopened and the specimen is taken out
for histopathology.
Q: What is this instrument?
Q: What are the indications?
A: Abram pleural biopsy needle.
A: As follows:
Q: What are its parts? • Malignant pleural effusion.
A: As follows: • Tuberculous pleural effusion.
• Pleural effusion of unknown aetiology.
• The outer part with a cutting groove into which
tissue is taken. N.B. In tuberculosis, AFBis positive in pleural fluid
• A solid stylet. in 20% cases and pleural biopsy is positive
• A cutting trocar. in 80% cases. In malignancy, pleural biopsy
is positive in 40% cases (may be up to 60%
Q: What is the procedure?
cases).
A: Pleural biopsy should be taken during aspira-
tion of pleural fluid with this needle. Under local
17 • INSTRUMENTS _

Instrument 05: Aspiration Needle with Rubber Tube


Q: What are the complications of pleural fluid aspiration?
A: As follows:
• Iatrogenic pneumothorax (hydropneumothorax).
• Infection may cause empyema.
• Acute pulmonary oedema.
• Injury to neurovascular bundle.
• Vasovagal attack due to fear or severe pain
(pleural shock).
Q: How to avoid injury to neurovascular bundJe?
A: The needle should be inserted near the upper bor-
der of the lower rib.
Q: How to avoid acute pulmonary oedema?
Q: What is this instrument? A: To avoid pulmonary oedema, more than 1 L of fluid
A; Aspiration needle with rubber tube. should not be removed.
N.B. Remember the following:
Q: What are its parts?
A: Wide-bore needle with rubber tube. • If more than 1-1.5 L of fluid is taken out
or fluid is taken out very rapidly, there may
Q: What are the uses? be pulmonary oedema. It is due to rapid
A: Aspiration of pleural fluid, ascitic fluid, pericardial expansion of the compressed lung that causes
fluid, liver abscess, etc. leakage of fluid from the pulmonary vessels.
• If the patient complains of cough or
N.B. If aspiration needle is not available, aspiration
respiratory distress or tightness of the chest,
can be done by wide-bore blood-set needle con-
aspiration should be stopped.
nected to any rubber tube, which is connected
• After aspiration, a check X-ray should be
with an empty saline bag. Fluid comes out easily
done to see the amount of fluid or any
by negative suction.
development of pneumothorax.

Pleural Fluid Aspiration Q: What are the causes of failure of aspiration of pleu-
ral fluid?
Q: What are the sites of aspiration of pleural fluid? A: As follows:
A: It is usually done through the 6th intercostal space • Faulty technique.
in the midaxillary line or 8th intercostal space in the • Encysted effusion (in such case, USC or CT-guided
posterior scapular line. Clinically, it should be done aspiration is more preferable).
at the site of maximum dullness. • Thick fluid like empyema.
Q: What are the indications of pleural fluid aspiration? Q: What should be done after taking the pleural fluid?
A: As follows: A: As follows:
1. Diagnostic: To diagnose the cause of pleural • Physical character: Colour (clear, straw,
effusion (tuberculosis, malignancy). haemorrhagic, purulent, chylous).
2. Therapeutic: • Gram staining, cytology (routine) and exfoliative
• Massive effusion especially with severe cytology (malignant cell).
respiratory distress or cardiorespiratory • Biochemistry: Protein, sugar [simultaneous
embarrassment. blood sugar, protein and lactate dehydrogenase
• Introduction of drugs like talc, .kaoline, (LDH) may be done].
tetracycline, etc. for chemical pieurodesis • Culture and sensitivity (CjS).
(to prevent recurrence of effusion or • AFBand mycobacterial CjS (in some cases).
pneumothorax). • Adenosine deaminase (ADA).
• Introduction of bleomycin in malignant • Other tests according to suspicion of cause
effusion. (amylase, cholesterol, LDH).
_ SAORT CASES IN CLINICAL MEDICINE

Ascitic Fluid Aspiration Q: What are the contraindications of ascitic fluid aspi-
ration?
Q: What is the site of aspiration of ascitic fluid? A: As follows:
A: At the right iliac fossa, just outside the spinournbili-
• Bleeding disorder.
cal line or at the flank. • Hepatic encephalopathy.
Q: How much fluid can be removed?
Q: What are the complications?
A: About 3-5 L of fluid may be drained daily. A: As follows:
Q: What are the indications of ascitic fluid aspiration?
• Hypovolaemia leading to shock.
A: As follows:
• Infection.
• Diagnostic: To find the cause of ascites like • Injury to viscera.
tuberculosis, malignancy, infection, etc. • Hepatic encephalopathy.
• Therapeutic: Tense ascites causing cardiorespiratory
embarrassment, resistant ascites refractory to
medical therapy.

Instrument 06: Intravenous Cannula


Q: Mention its uses.
A: Intravenous administration of any IVfluid, IVinjec-
tion, TPN (total parenteral nutrition) and blood
products.
Q: What are the complications?
A: As follows:
• Thrombophlebitis.
• Sepsis.
Q: What precautions should be taken to prevent
venous thrombosis and embolism?
Q: What is this instrument? A: The cannula should be changed every 3-4 days. Ifkept
A: Intravenous (IV) cannula. for more than 3 days, heparin wash should be given.

Instrument 07: Foley Catheter


• Retention of urine.
• Spastic paraplegia.
• Neurogenic bladder.
• Incontinence of urine.
• Unconscious patient.
• Postoperative patient (major abdominal, pelvic
or perineal surgery).
• Urinary bladder irrigation.
Q: What are its complications?
A: As follows:

• Trauma.
Q: What is this instrument?
• Infection.
A: Bichannel Foley Catheter.
• Blockage of catheter.
• Stone formation, if kept for a long time.
Q: What are the uses?
A: As follows:
17 • INSTRUMENTS _

Instrument 08: Nasogastric Tube (Ryle Tube)


Q: How can you test whether the instrument has
reached the correct site or not?
A: By the following:
• Aspiration of gastric content.
• Listening to the sound with stethoscope over the
epigastrium made by injecting 20-30 ml of air
through the tube.
• Emersion of the tube in water and looking for
any bubble (which appears if the tube is in the
airway).
• Also, if the tube enters the airway, the patient will
cough violently.
Q: Why there is a metallic bead at the tip?
A: It helps in smooth passage of the tube by gravita-
tional force. It also helps to localize the position of
Q: What is this instrument? the tube in the stomach by X-ray.
A: Nasogastric tube.
Q: What are the contraindications of insertion of
Q: What are the uses? nasogastric tube?
A: As follows: A: As follows:
1. Therapeutic: • Tracheoesophageal fistula.
• Nasogastric feeding. • Oesophageal atresia.
• Nasogastric suction (e.g. intestinal
Q: If nasogastric tube cannot be inserted, what else
obstruction, acute abdomen, acute dilatation
should be done?
of stomach, postoperative).
A: Gastrostomy tube should be inserted.
• Nasogastric medication in comatose patient.
• Gastric lavage (noncorrosive poisoning).
Q: What are the complications of nasogastric tube
2. Diagnostic:
insertion?
• Aspiration of gastric juice for gastric juice
A: As follows:
analysis.
• Aspiration of gastric fluid for toxicological • Cough.
screening. • Aspiration pneumonia.
• Fasting gastric lavage for AFB in a child • Haemorrhage.
suspected of pulmonary tuberculosis. • Injury.

Instrument 09: Air Way Tube


Q: What is its use?
A: As follows:
• To maintain a dear airway.
• To prevent tongue fall back in unconscious
patients.
• To prevent tongue bite in epileptic or unconscious
patient.

Q: What are its complications?


Q: What is this instrument?
A: Injury to lip, gum, tongue, palate, pharynx, etc.
A: Airway tube (oropharyngeal tube).
_ SHORT CASES IN CLINICAL MEDICINE

Instrument 10: ESRTube


Q: What are the causes of raised ESR?
A: As follows:
• Physiological: Pregnancy.
• Tuberculosis.
• Multiple myeloma.
• Aplastic anaemia.
• Connective tissue disorder:
o Systemic lupus erythematosus (SLE).
o Acute rheumatic fever.
o Rheumatoid arthritis.
o Polymyalgia rheumatica,
),I~I ~;rfl,.._I;.~r
~,r~~I, • Giant cell arteritis.
'.---W;--'.--' Q: What are the causes of very-high (>100) ESR?

-
~ JWg~~
A: As follows:
• Multiple myeloma.
Q: What is this instrument? • Giant cell arteritis.
A: Westergren erythrocyte sedimentation rate (ESR) • Polymyalgia rheumatica.
tube with ESRstand. • SLE.
• Acute rheumatic fever.
Q: What are the markings in this tube? Q: What are the causes of decreased ESR?
A: It is graduated from 0 to 200 mm. A: As follows:

Q: What are the methods of measuring ESR? • Polycythaemia due to any cause.
A: As follows: • Afibrinogenaemia.

, Westergren method. Q: What is the significance ofESR?


• Wintrobe method. A: As follows:

Q: What is the normal value of ESR? • ESR has no specific diagnostic significance.
A: 0-8 mm in t st hour in male and 0-12 mm in 1st However, it can support a diagnosis.
hour in female. • Itmay indicate response to therapy and prognosis.

Instrument 11: Metered-Dose Inhaler


Q: What is this instrument?
A: Metered-dose inhaler.

Q: What are its parts?


A: As follows:

• Canister.
• Actuator.
• Nozzle.
Q: Name two important conditions where this device
is used.
A: As follows:
• Bronchial asthma.
• Chronic obstructive pulmonary disease (COPD).
______________________ 1_7~. INSTRUMENTS I!IIIIII
Q: Name some important drugs delivered through this Q: Name one complication of the use of a steroid
device. inhaler.
A: As follows: A: Oral candidiasis (also husky voice).
• Salbutamol. Q: How would you prevent it?
• Steroid. A: I will advise the patient to wash oral cavity after
• lpratropium bromide. using inhaler containing steroid preparation.

Instrument 12: Accuhaler


Q: Name some important drugs delivered through this
device.
A: As follows:

• Salmeterol.
• Fluticasone.
• Salmeterol plus fluticasone.
Q: What are its advantages over metered-dose inhalers?
A: As follows:
• This device is easier to use than conventional
metered-dose inhalers, which require careful
coordination.
• A numerical dose counter helps monitor the
Q: What is this instrument?
asthma therapy.
A: Accuhaler. • More environment friendly.
Q: What are the disadvantages?
Q: Name two conditions where this device is used.
A: As follows: A: It may be difficult to use for young children or
adults who are short of breath.
• Bronchial asthma.
• COPD.

Instrument 13: Evohaler


Q: Name two conditions where this device is used.
A: As follows:
• Bronchial asthma.
• COPD.
Q: Name some important drugs delivered through this
device.
A: As follows:
• Beta agonist like salmeterol, salbutarnol.
• Steroid like fluticasone.
• Combination.
Q: What are its advantages over conventional metered-
dose inhalers?
A: Evohaler uses HFA 134a as a propellant instead of
Q: What is this instrument? chlorofluorocarbon (CFC). So this is more environ-
A: Evohaier. ment friendly.
_ SHORT CASES IN CLINICAL MEDICINE

Instrument 14: Peak Flow Meter


Q: What is this instrument?
A: Peak flow meter.

Q: What are its uses?


A: It is used to monitor the progress of the disease and
its treatment in:

• Bronchial asthma.
• COPD.

N.B. Regular measurement of peak expiratory flow


rate (PEFR) on waking from sleep, in afternoon
and before going to bed demonstrates the wide
diurnal variation of airflow limitation in bron-
chial asthma.

Instrument 15: AMBU Bag


1. Inlet:
• Air inlet.
• Oxygen inlet.
2. Bag proper/rubber bag.
3. Safety valve/one-way valve.
4. Outlet.
Q: Name some indications for its use.
Q: What is this instrument? A: As follows:
A: AMBU bag with face mask. • Cardiopulmonary resuscitation.
• Any respiratory distress.
Q: Name its parts. • Temporarily it can be used before intubation.
A: As follows:

Instrument 16: Tongue Depressor


• Depressor part (broad part, used to depress
anterior 2/3rd of the tongue).
• Holding part.
Q: What are the uses?
A: As follows:

• Diagnostic:
Q: What is this instrument?
o To examine the oral cavity: Oral ulcer, cleft
A: Metallic tongue depressor.
palate, Koplik spot, etc.
o To examine the throat: Tonsillitis, pharyngitis,
Q: What are the types?
diphtheria, etc.
A: Metallic, plastic and wooden.
o To collect throat swab.
Q: What are the parts? • Therapeutic: Removal of foreign body from
A: As follows: posterior part of the tongue and throat.
17 • INSTRUMENTS _

Q: What are the causes of white patch in throat? • Oral candidiasis.


A: As follows: • Vincent angina.
• Agranulocytosis.
• Acute follicular tonsillitis.
• Infectious mononucleosis.
• Diphtheria.

Oral Rehydration Salt


Water to be added: 1 L
Q: What are its indications?
A: As follows:

• Acute watery diarrhoea.


• Correction of dehydration.
Q: How long can it be preserved?
A: It can be preserved for up to 12 h. It should be dis-
carded after this time.

Q: What is the function of glucose?


A: It helps in absorption of sodium chloride.

Q: What are the composition of rice ORS?


A: As follows:
Q: What is it?
A: This is oral rehydration salt CORS). • Sodium chloride: 3.5 g.
• Potassium chloride: 1.5 g.
Q: What is its composition? • Sodium bicarbonate: 2.5 g.
A: As follows: • Rice powder: 50 g.
• Water to be added: 1100 ml.
Content Amount
Sodium chloride 3.5 giL
Potassium chloride 1.5 giL
Trisodium citrate 2.9 giL
Glucose 20g/L

Now, this is not the end. It is not even the beginning of


the end. But it is perhaps, the end of the beginning.
- Winston Churchill
ibliography

Burns T, Breathnach S, Cox N, Griffiths C. Rook's Text Kumar ~ Clark M. Kumar & Clark's Clinical Medicine, 8th
Book of Dermatology, 7th edn. Philadelphia: WB edn. London: Saunders Elsevier: 2012.
Saunder, 2011. Longo DL, Fauci AS, Kasper DL, Hauser SL, Jameson TL,
Chugh SN. Bedside Medicine without Tears, 2nd edn. Loscalzo J. Harrison's Principles of Internal Medicine,
laypee Brothers Medical Publishers (P) Ltd, 2011. 18th edn. New York: McGraw Hill, 2011.
Chugh SN. Clinical Methods in Medicine, 1st edn. Iayppee Macleod J, Douglas G, Nicol EF, Robertson CEo Macleod's
Brothers Medical Publishers (P) Ltd, 2008. Clinical Examination, 11 th edn. Churchill Livingstone
Colledge NR, Walker BR, Raiston SH. Davidson's Principles Elsevier, 2009.
and Practice of Medicine, 21 th edn. Edinburgh: Mir MA. Atlas of Clinical Diagnosis, 1st edn. Philadelphia:
Churchill Livingstone, An imprint of Elsevier: 2010. WB Saunders, 1995.
Dooley JS, Lok A, Burroughs A, Heathcote J. Sherlock's Papadakis MA, McPhee SI, Rabow MW. Current Medical
Diseases of the Liver and Biliary System (Sherlock Diagnosis and Treatment, 52nd edn. New York: Mc
Diseases of the Liver), 12th edn. Oxford: Blackwell Graw Hill, 2013.
Scientific Publications, 2011. Pappworth MH. A Primer of Medicine, sth edn. Iaypee
Firkin E Chesterman C, Rush B. de Gruchy's Clinical Brothers Medical Publishers (P) Ltd, 1993.
Haematology in Medical Practice, Sth edn. Oxford: Ryder REI, Mir MA, Freeman EA. An Aid to the MRCP
Blackwell Scientific, 2008. PACES, 3rd edn. Oxford: Blackwell Publishing, 2003.
Forbes CD, Jackson WF. Color Atlas and Text of Clinical Seaton A, Seaton D, Leitch AG. Crofton and Douglas's:
Medicine, 2nd edn. London: Mosby-Wolfe, 1997. Respiratory Disease, 5th edn. Wiley India Pvt Ltd, 2008.
Glynn M, Drake MW. Hutchison's Clinical Methods, 23rd Talley NJ, O'Connor S. Clinical Examination: A Systematic
ed. WB Saunders, 2012. Guide to Physical Examination, 6th edn. Elsevier
HoughtonAR, Gray D. Chamberlain's Symptoms and Signs Australia, Dec 29, 2009.
in Clinical Medicine, 13th edn. Hodder Arnold, 2010. Warrell DA, CoxTM, John D. Oxford Textbook of Medicine,
James -WD, Berger T, Elston D. Andrew's Diseases of the 5th edn. Oxford: Oxford University Press, 2012.
Skin Clinical Dermatology, nth edn. Philadelphia: Zatouroff M. A ColorAtlas ofPh),sicalSign in GeneralMedicine,
WB Saunder, 2011. 1st edn. London: Wolfe Medical Publications, 1976.
I d X

c-Petoprotein 194 ArgyllRobertson pupil (ARpupil) 453 Bitot spot 462


~-Thalassaemia major 525 Arsenicosis 483 Blackwater fever 182
Arthritis mutilans 598 Blalock-Taussig shunt 94
Aschoff nodule 117 Blastic crisis 250
Ascites 215 Blastic crisis in chronic myeloid
Abdominal mass 218 Ascitic fluid aspiration 618 leukaemia 527
Absent radial pulse 102 Ash leaf patch 499 Bleeding abnormality 243
Absent tendon reflex 336 Aspiration needle with rubber Blood pressure 69
Acanthocytes 256 tube 617 Blue nail 15
Acanthosis nigricans 476 Astasia abasia 376 Blue sclera 109, 463
Accuhaler 621 Asterixis 208 BODE index 161
Acetylcholine 421 Asymptomatic hyperuricaemia 425 Bone marrow aspiration needle 612
Achalasia cardia 564 Ataxic breathing (Biot breathing) 125 Bone marrow study 612
Achondroplasia 295 Ataxic nystagmus 458 Boutonniere deformity 389,390
Acoustic neuroma 571 Atrial fibrillation 102, 579 Bradycardia 71
Acromegaly 279,560,610 Atrial flutter 580 Brittle diabetes 298
Acute glomerulonephritis 519 Atrial septal defect 98, 552 Brittle nail (easily broken) 15
Acute interstitial nephritis 520 Atypical pneumonia 151 Bronchial 'breath sound 127
Acute leukaemia 523 Auspitz sign 467 Bronchial carcinoma 166
Acute myocardial infarction 581 Autonomic neuropathy 300 Bronchial sound 127
Acute pericarditis 115,590 Avascular necrosis 429 Bronchiectasis 140, 143, 548
Acute pyelonephritis 520 Avascular necrosis of femoral Bronchoalveolar cell carcinoma 169
Acute viral hepatitis ] 97,517 head 563 Bronchophony 127
Acute viral meningitis 522 Axillary lymphadenopathy 238
Bronchopneumonia 151
Acyanotic Fallot 94 Axonal degeneration 329 Bronchoscopy 168
Addison disease 288,533 Azygos lobe 547
Brown nail 15
Addisonian crisis 292 Brown-Sequard syndrome 320
Adult Still disease 401 u Buerger disease 52
Aegophony 127 Buffalo hunp 285
Baggy eyelids 282
Air way tube 619 Baker cyst 390, 609 Bullae 164,543
Akathisia 345 Bullous lesion 471
Bald tongue 55
Allopurinol 424 Band keratopathy 463 Bullous pemphigoid 481
Alopecia 493 Barlow syndrome 80 Burr cells 256
Alopecia areata 494 Barrel-shaped chest 126 Butterfly rash 504
AMBU bag 622 Becker muscular dystrophy 359
Amoebic liver abscess 199 Bell palsy 366, 603
Amyotrophic lateral sclerosis 335 Bell phenomenon 367
Anaemia 4 Benign essential tremor 348 Calcification of the lung
Anakinra 395 Benign malaria 180 parenchyma 542
Aneurysm of aorta 227 Bilateral claw 355 Campbell de Morgan spots 242
Angiomata of choroid 511 Bilateral effusion 135 Caplan syndrome 393
Anisocoria 454 Bilateral exophthalmos 456 Caput medusae 176
Ankylosing spondylitis 411,41.3,557 Bilateral facial palsy 368 Carcinoma of colon 232
Anti-phospholipid syndrome 432 Bilateral facial weakness 368 Carcinoma of head of
Anti-TNF-alpha 394 Bilateral hilar lymphadenopathy 546 pancreas 224
Antimitochondrial antibody 212 Bilateral parotid enlargement 508 Carcinoma of oesophagus 565
Aortic regurgitation 84 Bilateral ptosis 451 Carcinoma of stomach 222, 565
Aortic stenosis 82 Bilateral renal mass 303 Cardiac tamponade 113
Apex beat 73, 126 Biological agents 394 Cardiomegaly 553
Arachnodactyly 109 Bitemporal hemianopia 461 Carotenaemia 7
INDEX

Caroticocavernous fistula 457 Cyanosis 4, 7 Epigastric mass 219


Carpal tunnel syndrome 360,361 Cystic fibrosis ] 43 ERCP 564
Catamenial pneumothorax ) 38 Eruptive xanthoma 509
Cavernous sinus thrombosis 457 Erysipelas 50
Cavity superimposed on cardiac Erythema ab igne 22
Dark nail 15
shadow 548 Erythema induratum (Bazin
Decompensated cirrhosis 203
Cellulitis 49 disease) 27
Decubitus 4
Central retinal vein occlusion Erythema multiforme 470,600
Deep venous thrombosis (Dvl') 46
442,611 Erythema nodosum 26
Dehydration 5 Erythema nodosum leprosum 29
Cerebellar lesion 338
Demyelination 329
Cerebellar tumour 571 ESR tube 620
De Quervain thyroiditis 262, 53]
Cerebral abscess 569 Euthyroid Graves disease 265
Dermatitis herpetiformis 472
Cervical lymphadenopathy 16 Euthyroid state 532
Dermatomyositis 415
Charcot joint 398,435 Evohaler 621
Dextrocardia 111, 551
Charcot-Marie- Tooth disease 378 Examination of hands 353
Diabetes mellitus 296
Chewing Tongue 56 Examination of pulse 101
Diabetic amyotrophy 41
Cheyne-Stokes breathing 1.25 Examination of the chest 123
Diabetic bullae 42
Chloasma 503 Examination of thyroid gland 257
Diabetic coma (DKA) 30]
Chorea 345, 346 Exfoliative dermatitis 491,492
Diabetic foot 39
Choroidoretinitis 460 Exophthalmos 264, 455
Diabetic ketoacidosis 534
Christmas disease 406 Extensor plantar 316
Diabetic retinopathy 445
Chronic hepatitis ] 96 Extradural haematoma 569
DIDMOAD syndrome 439
Chronic bronchitis 161 Extrapyramidal lesion 315
Dimorphic anaemia 254
Chronic constrictive pericarditis 113 Exudative ascites 217
Dislocation of lens 109
Chronic hepatitis 196 Exudative pleural effusion 133
Disseminated intravascular
Chronic liver disease 202
coagulation 528
Chronic lymphatic leukaemia 239
Disseminated lupus
Chronic myeloid leukaemia 249,523 Facial (Vllth) nerve palsy 366
erythematosus 428
Chronic obstructive pulmonary Fasciculation 335, 336
Dissociated sensory loss 356
disease 157 Fascio-scapulo-humeral
Distal intestinal obstruction
Chylothorax 135 dystrophy 359
syndrome (meconium ileus
Circinate balanitis 407 Felty syndrome 393
equivalent syndrome) 143
Cirrhosis of liver 202 Fetor hepaticus 208
Distended abdomen 175
Claw hand 333, 354 Fibrillation 335
DMARD 393
CLL 240 Fibrosing alveolitis 145
Dorsal column lesion 332
Clubbing 5, 8 Fibrosis of lung 156
Double apex beat 73
Coagulation abnormality 243 First-degree AV block 583
Down syndrome 507
Coarctation of aorta 106 First cranial nerve 363
DPLD 145
Collapse oflung 155, 545 Fitz-Hugh-Curtis syndrome 194
Drug-induced lung disease 129
Combined degeneration 331 Flaccid paraplegia 325
Dry bronchiectasis 141.
Complete heart block 104,584 Flail chest 126
Duchenne muscular dystrophy 360
Congenital heart disease 92 Flapping tremor 208
Dupuytren contracture 59, 208
Consolidation 147,539 Foley catheter 618
Dysarthria 350, 352
Constructional apraxia 210 Follicular carcinoma 272
Dyskinesia or akinesia 342
COPD 157 Foot drop 377
Dysphasia 350
Corneal arcus 462,509, 609
Dysphonia 350 Foster-Kennedy syndrome 441
Corrigan sign 72 Friedreich ataxia 121,317,
Courvoisier's law 220 336,337
Cranial nerves 363 Fulminating hepatic failure 210
Crepitation 128, 130 Eaton-Lambert syndrome 421
Fungal nail 15
Cretinism 276 Ebstein anomaly 88
Crohn disease 229 Eisenmenger syndrome 96
G
Cruvei lhier - Baumgarten Emphysema 163, 529, 542
syndrome 174 Empyema 134 Gallop rhythm 73
Cullen sign 177 End of dose dyskinesia 345 Gastric lymphoma 223
Cushing syndrome 284, 532 Enthesopathy 413 Gastric outlet obstruction 566
CVD with hemiplegia 372 Eosinophilic fasciitis 386 Gas under the diaphragm 550
INDEX

Generalised lymphadenopathy 18 Hess test 242 Intracerebral haemorrhage 568


Generalized oedema S3 High arch palate 109 Intravenous cannula 618
Geographical tongue 55 Hirsutism 32, 605 Involuntary movements 347
Giant 'v' wave 72 Hodgkin disease 234 Iron-deficiency anaemia 252
Gigantism 281 Hodgkin lymphoma 234 IXth and Xth cranial nerves :?65
Gilben syndrome 518 Hoffman syndrome 278
Glaucomatous OA 439 Holmes-Adie pupil 455
Global dysphasia 351 Homocystinuria 111
Goitrous myxoedema 604 Homogeneous opacity of one Jaundice 4
Gonorrhoea 604 hemithorax 549 Jerky nystagmus 458
Gottron sign 417 Horner syndrome 452 Jerky pulse 71
Gout 422 Howell-Jolly bodies 255 Jugular venous pressure 69, 72
Granuloma in liver 212 Huntington chorea 347 Juvenile hypothyroidism 276
Graves disease 456, 598 Hydatid cyst 200,201 Juvenile idiopathic arthritis 399
Grey Turner sign 177 Hydatid cyst in liver 570
Cuillain-Barre syndrome 325 Hydrocephalus 569
Gum hypertrophy S6 Hydropneumothorax 545 Kala-azar 182
Guttate psoriasis 466 Hydroxychloroquine 394 Kala-azar in pregnancy 185
Cynaecornastia 23, 24 Hyperkalaemia 594 Kala-azar treatment Failure 185
Hyperosmolar nonketotic diabetic Kaposi sarcoma 500
H coma 534 Kartagener syndrome 111
Hyperprolactinaemia 283 Kayser-Fleischer ring 463
Haernaturia 311,312 Hypersplenism 189 Keratoderma blennorrhagica 407
Haemochromatosis 213 Hypertensive retinopathy 443 Keratomalacia 463
Haemoglobinuria 312 Hypertonia 315 Klinefelter syndrome 293
Haemolytic anaemia 254 Hypertrichosis 33 Knee joint arthritis 396
Haemolytic jaundice S 18 Hypertrophic cardiomyopathy 119 Koebner phenomenon 467
Haemoperitoneum 177 Hypertrophic osteoarthropathy ]0 Koilonychia 5, 11
Haemophilia 404 Hyperuricaemia 423 Kussmaul sign 72
Haemophilic arthritis 403 Hypochromia 255 Kussmaul breathing 124
Haemorrhagic stroke 375 Hypochromic microcytic
Haemothorax 134 anaemia 524
Hairy leukoplakia 58
l
Hypogastric swelling 175
Half-and-half nail 13 Hypoglycaemic coma 301 Lactulose 210
Halitosis 56, 208 Hypokalaemic metabolic Lateral medullary syndrome 375
Harrison sulcus 125 alkalosis 533 Laurence-Moon-Bardet-Biedl
Hashimoto thyroiditis 275, 279 Hypopituitarism 606 syndrome 52
Heart sounds 73 Hypothyroidism 273 Lef1unomide 394
Heinz bodies 256 Hypotonia 316 Left bundle branch
Heliotrope rash 417 Hysterical rigidity 342 block 585
Hemisphere 339 Left ventricular hypertrophy 586
Henoch-Schonlein Leg swelling 43
purpura 245, 398 Leg ulcer 35'
Hepatic precoma 209 Ichthyosis 477 Lepra reaction 30
Hepatic rub 191 Ichthyosis vulgaris 478 Leprosy 28, 601
Hepatoma 194, 570 Idiopathic thrornbocytopaenic Leptospirosis 197
Hepatomegaly 192, 198,211,213 purpura 243, 527 Leuconychia 5, 12
Hepatopulmonary syndrome 206 IIlrd, IVth and VIth cranial Leukemoid reaction 526
Hepatorenal syndrome 206 nerves 364 Leukoerythroblastic
Hepatosplenomegaly 178, 179, 182 ILD 145 anaemia 526
Hereditary haemolytic Ileocaecal tuberculosis 228 Leukoplakia 58
anaemia 247,559 Impaired fasting glucose 298 Levocardia 111
Hereditary haernorrhagic Impaired glucose tolerance S35 Lhermitte sign 324
telangiectasia 512 Infected scabies 596 Lichen planus 488
Herpes zoster (shingles) 474 Intention tremor 348 Lid lag 456
Herpes zoster 474,607 Internuclear ophthalmoplegia 458 Lid retraction 455
Herpes zoster ophthalmicus 476 Interstitial lung disease 529 Limb-girdle myopathy 358
INDEX

Linear scleroderma 383 Methotrexate 394 limb 313


Linitis plastica 223 Microglossia 56 Neuropathic ulcer 41
Lipodystrophy of thigh 42 Middle lobe syndrome 156 Non-Hodgkin lymphoma 237
Lipomastia 24 Migrating polyarthritis 118 Non-small-cell carcinoma 170
Lisch nodule 497 Mikulicz syndrome 509 Noonan syndrome 506
Liver abscess 198, 570 Miliary tuberculosis 541 Normal vesicular breath sound 127
Liver biopsy needle 615 Millard-Gubler syndrome 375 Normocytic normochromic
LMN 315 Miosis 454 anaemia 254
Localized distension 175 Mitral facies 76 Nosocomial pneumonia 150
Lone atrial fibrillation 103 Mitral regurgitation 79 Nucleated RBC 256
Long-term domiciliary oxygen Mitral stenosis 75,551 Nystagmus 457, 458
therapy 160 Mitral valve prolapse 80
Loss of nail (or dystropby) 14 Mitrale 588
Low hairline 506 Mixed aortic valve disease 87
Lumbar puncture needle 613 Mixed mitral valve disease 81 Obstructive jaundice 517,607
Lung abscess 153, 540 MND 317 Ochronosis 22
Lung function test 129, 165 Mononeuritis multiplex 330 Ocular myopathy 451
Lupus pernio 65 Monoplegia 320, 321 Oculopharyngeal myopathy 452
Lupus vulgaris 487 Moon face 285 Oedema 5
Luternbacher syndrome 76 Morphoea scleroderma 383 Old myocardial infarction 582
Lymph nodes 5 Mosaicism 506 Onycholysis 14
Lymphoedema 44, 45 Motor neuron disease 317, 332 Opening snap 75
Motor neuropathy 328 Optic atrophy 438,610
Multinodular goitre 268 Optokinetic nystagmus 459
Multiple cranial nerve palsy 370 Oral candidiasis 54
Macroadenoma 570 Multiple myeloma 524, 558 Oral rehydration salt 623
Macrocytic anaemia 525 Multiple sclerosis 322 Orofacial dyskinesia 377
Macrocytosis 255 Multiple secondaries in Liver 570 Osler node (in toe) 69
Macroglossia 56 Multiple secondaries in lung 538 Osler-Weber-Rendu syndrome 512
Macrosomia 299 Murmur 74 Osteoarthrosis 434, 605
Maculopathy 446 Mycoplasma pneumoniae 151 Osteophyte 414
Main points in general examination 4 Mycosis fungoides 498
Malar flush 503 Mydriasis 454
Malaria 179 Myelofibrosis 251
Malignant exophthalmos 266 Myopathic facies 357 Pacemaker 556,591
Malignant hypertension 440 Myopathy (muscular dystrophy) 358 Pachydermoperiostosis 11
Malignant malaria 180 Myotonia 357 Paget disease 60
Malignant RA 391 Myotonia congenita 357 Painful gynaecomastia 25
Marcus-Gunn phenomenon Myotonia dystrophica 452 Painful muscle 42]
or pupil 454 Myotonic dystrophy 356 Painless haematuria 312
Marfan syndrome 108, 111 Myxoedema coma 278 Pale nail 13
Mass in centra] abdomen 221 Myxoedema madness 279 Palindromic RA 393
Mass in flank 221,302 Pallor 6
Mass in lower abdomen 221 Palmar erythema (liver palm) 207
Mass lesion 537 Pancreatic calcification 566
Mass lesion in lung 166 Nail changes in different diseases 12 Pancreatic pseudocyst 226
Mass reflex 319 Nail fold infarction 13 Pancytopaenia 524
Mechanism of airflow Nail fold telangiectasia 14 Papillary carcinoma 272
limitation 159 Nail hyperpigmentation 15 Papillitis (optic neuritis) 440
Mediastinal widening 549 Nail pitting 14 Papilloedema 440, 611
Medullary carcinoma of thyroid 273 Nasogastric tube 619 Paraneoplastic syndrome ] 69
Mee line 14 Necrobiosis lipoidica Paraneoplastic syndrome of
Melasma 503 diabeticorum 43 cerebellum 339
Meningioma 569 Nelson syndrome 288 Paraplegia 317,318
Metallic prosthetic valve 556 Nephrotic syndrome 308,393, 519 Parkinsonian plus 343
Metered-dose inhaler 620 Neurofibroma 496, 497 Parkinsonism 340
Methanol poisoning 464 Neurological examination of lower Patent ductus arteriosus 99
INDEX

Peak flow meter 622 Portosystemic anastomosis 206 Ramsay Hunt syndrome 603
Pectus carinatum 125,609 Portosystemic encephalopathy 209 Raynaud phenomenon 386
Pectus excavatum 125 Post-kala-azar dermal Rebound phenomenon 338
Pel-Ebstein fever 235 leishmaniasis 185 Recurrent pleural effusion 135
Pemberton sign 258 Pott disease 558, 570 Recurrent pneumonia 149
Pemphigus foliaceous 479 P Pulmonale 589 Recurrent pneumothorax l38
Pemphigus vulgaris 479 Precordium 69, 72 Red nail 15
Pendred syndrome 278 Pretibial myxoedema 266 Red or dark urine 312
Pendular nystagmus 458 Primary biliary cirrhosis 211 Reed-Sternberg cell 234
Pentalogy of Fallot 94 Primary hyperaldosteronism 533 Reed-Sternberg giant cell 235
Pericardial calcification 554 Primary hyperparathyroidism 529 Refractory ascites 204
Pericardia] effusion 112, 554 Primary hypothyroidism 530 Reiter syndrome 406, 407
Pericardia! rub 74 Primary lateral sclerosis 335 Renal biopsy 246
Pericardiocentesis 113 Primary Sjogren syndrome 395 Renal cell carcinoma 304
Peripheral vascular disease 50, 606 Progressive bulbar palsy 335 Renal glycosuria 535
Periungual or subungual Progressive muscular atrophy 335 Renal vein thrombosis 311
fibroma 15 Prosthetic heart valves 90 Resistant ascites 204
Permanent atrial fibrillation 103 Proximal myopathy 420 Resistant kala-azar 185
Pernicious malaria 182 Pseudo-cushing syndrome 286 Resorption of terminal
Persistent atrial fibrillation 103 Pseudogout 398, 423 phalanges 561
Pes cavus 336,337 Pseudohypoparathyroidism 295 Respiratory rate 124
Peutz-Ieghers syndrome 21 Pseudotumour 133 Retinal detachment 461
Phacornatosis 497 Psoriasis 466, 599 Retinal haemorrhage 444
Pigeon chest (pectus Psoriatic arthropathy 408 Retinitis pigmentosa .449
carinatum) 125 Psoriatic patch 409 Reverse coarctation 107
Pigmentation 5, 20 Ptosis 449 Rheumatic AR 86
Pigmentation in palate 290 Puffy face 285, 503 Rheumatic fever 116, 403
Pingueculae 462 PuLmonary oedema 555 Rheumatoid arthritis 597
Pituitary macroadenoma 571 Pulmonary stenosis 89 Rheumatoid factor 392
Pityriasis versicolor 501 Pulmonary tuberculosis 540 Rheumatoid hand 388,561
PKDL 1.85 Pulsatil e live r 191 Rheumatoid nodule 390
Plaque psoriasis 466 Pulse 69,70 Rhonchi 128
PLatypnoea 129 Pulsus alternans 71 Rib notching 108
Pleural biopsy needle 616 Pulsus bigeminy 589 Rib resection 546
Pleural effusion 130, 536 Pulsus paradoxus 71 Rickets 562
Pleural fluid aspiration 617 Purpura 241, 243, 597 Rickety rosary 126
Pleural rub 128, 130 Pyoderma gangrenosum 37 Right-sided cerebral infarction 569
Pleuropericardial rub 128 Pyogenic liver abscess 198 Right bundle brand) blod< 585
Plexiform neurofibroma 497 Pyogenic meningitis 52] Right iliac fossa 219
Plummer-Vinson syndrome 12 P-R interval 575 Right ventricular hypertrophy 587
P Mitrale 588 P-Wave 574 Rigidity 320, 341
Pneumatosis cystoides Ring sideroblast 256
intestinalis 384 o Ring worm 602
Pneumothorax 136, 544 Rituxirnab 245, 395
QRS complex 576
Polychromatia 256 Rubeosis iridis 298
Q-T interval 578
Polycystic kidney disease 303, 567
Q-Wave 575
Polycystic ovarian syndrome 34
Polycythaemia 305
p
Polycythemia rubra vera 528 S-wave 576
Polydactyly 52 R-Wave 576 SAH 460
Polymyositis 415 Radial deviation 390 Sarcoidosis 66, 547
Polyneuropathy 328 Radial nerve palsy 363 Scabies 485
Polyneuropathy (subacute Radial pulse 101 Scalp psoriasis 466
combined degeneration) 330 Radical cure 180 Scaphoid abdomen 176
Portal hypertension 203 Radioiodine therapy 262 Schistocytes 256
Portopulmonary hypertension 206 Raised intracranial pressure 614 Scleroderma sine sderoderma 383
INDEX

Subclinical hypothyroidism 278 Tremor 341,348


Scleroedema 386
Subconjuctival haemorrhage 602 Tricuspid regurgitation 88
Scrotal tongue 55
Subcutaneous nodule 118 Trilogy ofFallot 94
Scurvy 563
Second-degree AV block (Mobitz Subdural haematoma 569 Triple rhythm 73
Subhyaloid haemorrhage 459 Trombone tongue 56
type I) 583
Subtotal thyroidectomy 263 Tropical splenomegaly
Second-degree AV block (Mobitz
Sulphasalazine 394 syndrome 189
type U) 584
Superior vena caval obstruction 30 Tubercular meningitis 521
Second cranial nerve 364
Supraventricular tachycardia 592 Tuberculous peritonitis 194
Sensory dysphasia 351
Surgical emphysema 543 Tuberous sclerosis 499
Septic arthritis 402
Swan neck deformity 389,390 Tuberous xanthoma 509
Serum-ascites albumin gradient 217
Sydenham chorea 118, 347 Turner syndrome 505
Shagreen patch 499
Synacthen test 291 Typical pneumonia 15]
Shawl sign 417
Shield-like chest 506 Syndesmophyte 414
Short stature 295 Syphilitic AR 86
Sick euthyroid syndrome 278, 531 Syringomyelia 333, 354 U-Wave 577
Sickle cell 256 Systemic lupus erythematosus Uhthoff phenomenon 324
Signs of hypothyroidism 258 425,600 Ulnar deviation 390
Simple diffuse goitre 269 Systemic sclerosis 382 Ulnar nerve palsy 362
Simple multinodular goitre 267 Unilateral papilloedema 441
Simple nodular goitre 270 Upperlimb drift 316
Sinus bradycardia 71,592 Upper motor neuron 315
Sinus tachycardia 70, 591 T-Wave 577 U rinary incontinence 311
Sister Mary Joseph nodule 177 Tabes dorsalis 332
Situs inversus Ill, 550 Takayasu disease lOS
Small-cell carcinoma 168, 170 Tall stature 292
Sol itary nodule 270 Target cells 255 Vasculitis 388, 433
Solitary pulmonary nodule 538 Target lesion 471 Vasculitis in rheumatoid
Spasticity 341 Telangiectasia 512 hand 389
Spastic paraplegia 316,318,320 Tender hepatomegaly 195 Venous hum 75, 100
Speech 349 Tendon xanthoma 509 Venous stars 207
Spherocytes 256 Tension pneumothorax 138 Ventricular aneurysm 552
Spider angioma 207 Terry nail 15 Ventricular fibrillation 594
Spinal cord compression 317 Tetralogy of Fallot 92, 553 Ventricular premature beat 593
Spirometry 165 Thickened pleura 130, 155 Ventricular septal defect 95
Splenomegaly 187; 189 Thoracoplasty 126 Ventricular tachycardia 580
Splinter haemorrhage 13 Thyroid acropachy 10 Vertical nystagmus 459
Spontaneous bacterial Thyroid gland 5 Vestibular nystagmus 459
peritonitis 205 Thyrotoxic crisis 263 Vllth cranial nerve 365
Spurious anaemia 254 Thyrotoxicosis 260, 530 VIlIth cranial nerve 365
Spurious polycythaemia 254 Thyrotoxic periodic paralysis 263 Viral hepatitis 195
Sterile pyuria 312,520 Tocilizumab 395 V1th nerve palsy 369
Stevens-Johnson syndrome 470,601 Tongue-tie 56 Vitiligo 495
Still disease 400 Tongue 54 Vocal fremitus 127
Stokes-Adam attack 105 Tongue depressor 622 Vocal resonance 127
Striae 177 Tophi 560 Vth cranial nerve 364
ST segment 577 Tophus 423
Sturge-Weber syndrome 511 Toxic adenoma 271
Subacute combined Toxic multinodular goitre 268
Toxic nodular goitre 268 Waldeyer ring 238
degeneration 317
Trachea 126 Water hammer pulse 71
Subacute cutaneous lupus
Transplanted kidney 306 Watermelon stomach 384
erythematosus 428
Transudative ascites 217 WATSON 323
Subarachnoid bleeding 614
Transudative pleural effusion 133 Wegener granulomatosis 503, 608
Subarachnoid haemorrhage
Traumatic bleeding 614 Whispering pectoriloquy 127
460,522,568
INDEX

x v
Xanthelasma 509,608 Yellow body 6
Xanthelasma in PBC 211 Yellow nail 14
Xanthochromia 614 Yellow nail syndrome 133,513
Xanthoma in tendo calcaneus 211
XIIth cranial nerve 365
l
XIth cranial nerve 365 Z-deformity of thumb 389, 390
ISBN: 978-81-312-3468-6

A division of Reed Elsevier india Pvt. Ltd.


www.elsevier.co.in
Join us onOfacebook.comlElsevierlndia 9 788131234686

Das könnte Ihnen auch gefallen